Prehospital Emergency Care Test Bank [11 ed.] 9780134704487

Test Bank for the corresponding textbook, Prehospital Emergency Care, 11th Edition.

10,977 731 3MB

English Pages 781 Year 2018

Report DMCA / Copyright

DOWNLOAD FILE

Polecaj historie

Prehospital Emergency Care Test Bank [11 ed.]
 9780134704487

Table of contents :
Chapter 01 - Emergency Care Systems, Research, and Publich Health
Chapter 02 - Workforce Safety and Wellness of the EMT
Chapter 03 - Medical, Legal, and Ethical Issues
Chapter 04 - Documentation
Chapter 05 - Communication
Chapter 06 - Lifting and Moving Patients
Chapter 07 - Anatomy, Physiology, and Medical Terminology
Chapter 08 - Pathophysiology
Chapter 09 - Life Span Development
Chapter 10 - Airway Management, Artificial Ventilation, and Oxygenation
Chapter 11 - Vital Signs, Monitoring Devices, and History Taking
Chapter 12 - Scene Size-Up
Chapter 13 - Patient Assessment
Chapter 14 - General Pharmacology and Medication Administration
Chapter 15 - Shock and Resuscitation
Chapter 16 - Respiratory Emergencies
Chapter 17 - Cardiovascular Emergencies
Chapter 18 - Altered Mental Status, Stroke, and Headache
Chapter 19 - Seizures and Syncope
Chapter 20 - Acute Diabetic Emergencies
Chapter 21 - Allergic and Anaphylactic Reactions
Chapter 22 - Toxicologic Emergencies
Chapter 23 - Abdominal, Hematologic, Gynecologic, Genitourinary, and Renal Emergencies
Chapter 24 - Environmental Emergencies
Chapter 25 - Submersion Incidents: Drowning and Diving Emergencies
Chapter 26 - Psychiatric Emergencies
Chapter 27 - Trauma Overview: The Trauma Patient and the Trauma System
Chapter 28 - Bleeding and Soft Tissue Trauma
Chapter 29 - Burns
Chapter 30 - Musculoskeletal Trauma and Nontraumatic Fractures
Chapter 31 - Head Trauma
Chapter 32 - Spinal Trauma and Spine Motion Restriction
Chapter 33 - Eye, Face, and Neck Trauma
Chapter 34 - Chest Trauma
Chapter 35 - Abdominal and Genitourinary Trauma
Chapter 36 - Multisystem Trauma and Trauma in Special Patient Populations
Chapter 37 - Obstetrics and Care of the Newborn
Chapter 38 - Pediatrics
Chapter 39 - Geriatrics
Chapter 40 - Patients with Special Challenges
Chapter 41 - The Combat Veteran
Chapter 42 - Ambulance Operations and Air Medical Response
Chapter 43 - Gaining Access and Patient Extrication
Chapter 44 - Hazardous Materials
Chapter 45 - Multiple-Casualty Incidents and Incident Management
Chapter 46 - EMS Response to Terrorism Involving Weapons of Mass Destruction

Citation preview

Prehospital Emergency Care, 11e (Mistovich et al.) Chapter 1 Emergency Care Systems, Research, and Public Health 1) What is a primary purpose of the modern-day EMS system? A) Provide a means of transport to and from the hospital B) Ensure that all members of society have equal access to hospitals C) Decrease the incidence of death and disability related to injury and illness D) Provide emergency health care services to medically underserved areas Answer: C Diff: 2 Page Ref: 7 Objective: 1-2 2) Most authorities agree that the modern-day EMS system evolved after the release of which document? A) The Emergency Medical Services Act of 1973 B) The white paper "Accidental Death and Disability: The Neglected Disease of Modern Society" in 1966 C) The American Heart Association's Guidelines for Cardiac Resuscitation D) Emergency Medical Services: Agenda for the Future, in 1996 Answer: B Diff: 1 Page Ref: 3 Objective: 1-2 3) An Emergency Medical Responder comes to you and states that he would like to work for your emergency ambulance service on a full-time basis. Knowing the National EMS Scope of Practice Model, you inform him of which element? A) He will be able to take care of only patients with non-life-threatening complaints B) He will first need approval from your ambulance service's medical director C) He will be able to work only with an Advanced EMT or paramedic D) He will first have to become an Emergency Medical Technician Answer: D Diff: 1 Page Ref: 6-7 Objective: 1-1 4) As defined by the National EMS Scope of Practice Model, the four levels of EMS practitioners include: A) Ambulance drivers B) EMS medical director C) Paramedics D) Emergency medical dispatchers Answer: C Diff: 1 Page Ref: 6-7 Objective: 1-3

1 Copyright © 2018 Pearson Education, Inc.

5) An Emergency Medical Responder (EMR) approaches you and states that he is interested in becoming an EMT. Specifically, he asks what he will be able to do as an EMT that he cannot presently do as an EMR. You respond by saying he will be able to: A) Administer some medications B) Assist in emergency childbirth C) Use an automated external defibrillator D) Obtain vital signs Answer: A Diff: 1 Page Ref: 6-7 Objective: 1-5 6) You have been dispatched for a 61-year-old female in cardiac arrest. Emergency Medical Responders (EMRs) are on scene. In your community, all EMS practitioners are trained according to the National EMS Scope of Practice Model. Given this, which type of care do you expect the EMRs to be providing? A) Emergency medication administration B) Automated external defibrillation C) Intravenous therapy D) Reading an electrocardiogram Answer: B Diff: 2 Page Ref: 6 Objective: 1-5 7) An EMT with your service states that she desires to become a paramedic because she wants to provide advanced care. Under the National EMS Scope of Practice Model, which one of these types of care will she be able to provide as a paramedic that she cannot provide as an EMT? A) Automated blood pressure monitoring B) Assistance with emergency childbirth C) Interfacility transports D) IV therapy Answer: D Diff: 1 Page Ref: 7 Objective: 1-5 8) Which statement made by an EMT requires immediate intervention by a superior or the service supervisor? A) "I always wear my seat belt whenever I am in the patient compartment of the ambulance, unless I have to remove it to care for a patient." B) "I drive as fast as I can to get to the scene of an emergency, especially if a child is sick or injured according to dispatch information." C) "When I am driving with lights and sirens, I instruct my partner that he will need to handle all radio communications." D) "I put exam gloves on for every patient I contact, even if he or she does not have a known or suspected infectious disease." Answer: B Diff: 1 Page Ref: 9 Objective: 1-7 2 Copyright © 2018 Pearson Education, Inc.

9) Which statement shows that the EMT has a good understanding of her role and responsibilities related to being an EMT? A) "The patient's needs come before anyone else's needs." B) "At the scene of an emergency, my responsibility is to get to the patient no matter what." C) "My safety comes first, and then the patient's safety." D) "As an EMT, I recognize that the public's safety comes before the safety of me or my partner." Answer: C Diff: 2 Page Ref: 8-9 Objective: 1-7 10) An ambulance with two EMTs arrives in front of a house for an unknown medical emergency. On scene, bystanders report that a female patient was beaten by her husband in the driveway and then dragged back into the house. They state that the patient appeared unconscious and was bleeding from the head. Which action would the EMTs perform next? A) Do not enter the scene until it has been controlled by law enforcement B) Quietly enter the home and quickly move the patient to the ambulance C) Remove the husband from the house, then provide emergency care to the wife D) Enter the home to protect the patient while waiting for the police to arrive Answer: A Diff: 2 Page Ref: 9 Objective: 1-7 11) Why is EMS in a unique position to contribute significantly to mobile integrated health care? A) EMS providers are familiar with and capable of functioning in the out-of-hospital environment B) Ambulances are easily identified C) The community trusts uniformed health care providers D) EMS providers are familiar with the many routes of medication administration Answer: A Diff: 1 Page Ref: 17 Objective: 1-15

3 Copyright © 2018 Pearson Education, Inc.

12) You have arrived on the scene of a motor vehicle collision. A car has gone off the road and into a utility pole. The pole is broken and wires are hanging just above the car. You do not see any sparking or arcing of the wires. The driver of the car is slumped over the steering wheel and not moving. Several bystanders around the car are yelling for you to help. Which steps represent the correct order of your response? A) Clear the bystanders, allow the fire department to secure the wires, access the patient, extricate the patient, transport B) Access the patient, clear the bystanders, allow the fire department to secure the wires, extricate the patient, transport C) Clear the bystanders, access the patient, extricate the patient, transport, allow the fire department to secure the wires D) Access the patient, allow the fire department to secure the wires, clear the bystanders, extricate the patient, transport Answer: A Diff: 2 Page Ref: 8-9 Objective: 1-7 13) Once a scene is free of safety hazards, the EMT must recognize and focus on the needs of the: A) Rescuers B) Patient C) Family D) Bystanders Answer: B Diff: 1 Page Ref: 9 Objective: 1-7 14) You have been asked to sit on your service's quality improvement (QI) committee. As a knowledgeable EMT, you recognize that in this role you will be involved in projects that are primarily designed to: A) Communicate with insurance companies for increased reimbursement B) Recruit and educate EMTs about the Emergency Medical Services profession C) Develop ways in which the delivery of emergency care can be improved D) Identify and reeducate EMTs who routinely do not provide proper care Answer: C Diff: 2 Page Ref: 13 Objective: 1-10 15) What would be a logical project to be undertaken and developed by a quality improvement (QI) committee? A) Termination of EMTs who are chronically late to work B) A program by which pay raises are based on performance and attitude C) Community fundraiser to raise money for new EMS equipment D) Monthly continuing education programs in which seldom-used skills are practiced Answer: D Diff: 2 Page Ref: 14 Objective: 1-10 4 Copyright © 2018 Pearson Education, Inc.

16) You are interviewing for the position of EMT with a busy urban service. This morning you are to meet with the service's medical director. You have been told that she likes it when applicants ask questions about her role as the medical director. Related to her responsibilities as medical director, which question is most appropriate? A) "How do you determine the wages that the EMTs are paid?" B) "How do you determine the number and location of ambulances within the system?" C) "How often are the emergency care protocols updated?" D) "Do you assign the daily chores that must be completed around the stations?" Answer: C Diff: 2 Page Ref: 12 Objective: 1-9 17) To be a medical director of an ambulance service, which requirement must the interested individual satisfy? A) Licensed physician B) Emergency physician with surgical experience C) Physician who was a paramedic D) Physician with a college degree in EMS Answer: A Diff: 1 Page Ref: 12 Objective: 1-9 18) As an EMT, which requirement must be met so that you can provide routine and emergency medical care to the sick and injured, as an extension of the medical director's authority? A) Prior medical experience B) A service medical director C) A knowledgeable partner D) A safe driving record Answer: B Diff: 1 Page Ref: 12 Objective: 1-9 19) It has been some time since you were involved in the care of a pediatric patient. You decide to review the assessment and treatment basics that your medical director requires. To review this information, which resource should you consult first? A) Emergency Medical Responders B) Reputable websites C) Emergency department nurses D) Service protocols Answer: D Diff: 1 Page Ref: 13 Objective: 1-9

5 Copyright © 2018 Pearson Education, Inc.

20) Before the EMT contacts medical direction for authorization to assist a patient with shortness of breath in using her metered-dose inhaler, protocols state that the EMT must first attempt to provide relief by administering oxygen if the pulse oximeter reading is less than 94%. You recognize that the administration of oxygen is best described as a(n): A) Standing order B) On-line medical direction C) Direct permission D) Quality improvement Answer: A Diff: 2 Page Ref: 13 Objective: 1-9 21) If a patient with chest pain takes a daily aspirin, your medical director wants the EMT to contact an emergency department physician first before administering additional aspirin. The act of contacting the emergency department physician by phone or radio for permission to administer additional aspirin is an example of: A) Quality improvement B) On-line medical direction C) Off-line medical direction D) Indirect medical control Answer: B Diff: 2 Page Ref: 13 Objective: 1-1 22) At the scene of a minor motor vehicle collision, you are attending to a female patient who complains of head and neck pain. A crowd has gathered to watch the action. What will the observing public most likely remember about the role EMS portrayed? A) The number of emergency vehicles that showed up B) The color of the ambulances C) If the EMTs were all wearing the same color uniforms D) EMTs who were yelling and cursing loudly at the fire department to hurry up and get the patient out of the car Answer: D Diff: 2 Page Ref: 11 Objective: 1-8 23) You work in a community that has an "enhanced" 911 system. Given this, which is true about such a system? A) The physical location of the landline used to make the 911 call is displayed to the call taker B) The EMS system uses EMTs, Advanced EMTs, and paramedics C) The fire department responds simultaneously with EMS D) The total response time for EMS will rarely exceed 9 minutes Answer: A Diff: 1 Page Ref: 5 Objective: 1-4

6 Copyright © 2018 Pearson Education, Inc.

24) Which patient would most benefit from calling an enhanced 911 system from the home phone? A) Patient with chest pain B) Patient with sudden onset of confusion C) Patient requiring advanced medical care D) Patient who cannot reach the phone Answer: B Diff: 2 Page Ref: 5 Objective: 1-4 25) A 14-year-old boy has fallen through a plate glass window and is bleeding heavily from a laceration to his arm. His panicked mother has called 911 for help. Since the 911 operators are also emergency medical dispatchers, what sort of assistance will they be able to provide? A) Simultaneous notification of the hospital B) Whether the EMS response is covered by medical insurance C) Instructions on how to control bleeding D) The names of the EMTs who are responding Answer: C Diff: 2 Page Ref: 5 Objective: 1-3 26) Which be an EMS public health initiative? A) Educating the public on the benefit of wearing seat belts B) Volunteering time to help raise money for a sick child and his family C) Providing volunteer emergency ambulance service 24 hours a day, 7 days a week D) Purchasing new equipment to better help those persons who experience a medical emergency Answer: A Diff: 2 Page Ref: 16 Objective: 1-14 27) Your service's medical director would like to explore the idea of EMTs using special transport ventilators when caring for patients who require assistance with their breathing. When using an evidence-based medicine approach, what should you do first? A) Go to a local hospital to see which kind of ventilators it uses B) Contact vendors that sell ventilators and ask for price quotations C) Select specific EMTs to conduct a field trial of particular ventilators D) Search the medical literature for research related to this topic Answer: D Diff: 2 Page Ref: 14 Objective: 1-13

7 Copyright © 2018 Pearson Education, Inc.

28) You are by the side of a confused patient who has diabetes and a low blood sugar level. You know that the patient needs orally administered sugar, so following your service's guidelines, you call the physician on duty at the hospital emergency department and request permission to administer it. This request is an example of: A) Standing orders B) On-line medical direction C) Medical cooperation D) Off-line medical direction Answer: B Diff: 1 Page Ref: 12 Objective: 1-1 29) Which statement about emergency medical services in the United States is true? A) The EMS system is controlled and governed by the National Highway Transport and Safety Administration (NHTSA) B) The federal government oversees all aspects of EMS in the United States C) Each state has governmental control of its own EMS system, independent of the federal government D) The U.S. Department of Health and Human Services establishes curricula for all levels of EMS education Answer: C Diff: 2 Page Ref: 4 Objective: 1-3 30) The county dispatch center where you work employs emergency medical dispatchers (EMDs) as call takers. Using EMDs as opposed to a person who is just a "call taker" enables the dispatch center to: A) Process all emergency calls, including those asking for police, fire, and EMS assistance B) Provide medical instructions to the caller prior to the arrival of EMS personnel C) Use computer technology when dispatching the closest ambulance D) Properly utilize an enhanced 911 call-taking system Answer: B Diff: 2 Page Ref: 5 Objective: 1-3 31) A 40-year-old man has been seriously injured in a car crash. He is unresponsive and has what you believe to be a collapsed lung. Additionally, his left leg is severely deformed with heavy bleeding and his vital signs are unstable. A passenger in the car informs you that the patient has diabetes and significant heart problems. Given this information, what is the best destination for this patient, assuming all of the facilities are the same distance away? A) Cardiac center B) Spine injury center C) Trauma center D) Stroke center Answer: C Diff: 2 Page Ref: 7 Objective: 1-6 8 Copyright © 2018 Pearson Education, Inc.

32) You have arrived at the scene of a medical emergency and find a woman lying on the floor of a grocery store. As you approach her, you know that your first priority as an EMT is: A) Personal safety B) Patient care C) Public safety D) Patient advocacy Answer: A Diff: 1 Page Ref: 8 Objective: 1-11 33) A female jogger has collapsed on a trail at a public park. A crowd of curious onlookers have collected and are watching as the EMT examines the patient for any life-threatening conditions. After determining that there are none and the patient is stable, the EMT acts as a patient advocate by: A) Informing the crowd that the patient appears to be okay B) Transporting the patient to the hospital using lights and sirens C) Continuing care despite the crowd's presence D) Asking curious onlookers to leave the scene Answer: D Diff: 2 Page Ref: 10 Objective: 1-7 34) At a public 911 center meeting, a concerned woman asks whether voice over Internet Protocol (VoIP) can be used to access the community emergency operations center. You would inform her that: A) Internet providers are encouraged to provide 911 access, but are not required by law, so the dispatch center may have to get an address from the caller B) Internet providers are required to provide their own 911 call-taker service to subscribers C) Internet providers must provide a callback number and physical location of the phone from which the 911 call was placed D) Internet providers are required to employ EMDs who can provide medical instructions prior to arrival of EMS Answer: C Diff: 2 Page Ref: 6 Objective: 1-4 35) What would the EMT recognize as an advantage of the 911 universal number? A) It is able to track the locations of all emergency vehicles in an EMS system B) It is a common number through which to request EMS, law enforcement, and fire services C) It is less expensive to operate than a 7- or 10-digit telephone number D) It is the most easily remembered 3-digit combination of numbers Answer: B Diff: 1 Page Ref: 5 Objective: 1-4

9 Copyright © 2018 Pearson Education, Inc.

36) Which statement accurately describes the hospital emergency department? A) It is a health care destination staffed by physicians, nurses, and others designed to evaluate and stabilize patients, preparing them for further care B) It requires patients to make an appointment to be seen by a physician, unless they arrive by ambulance C) It is the only health care destination to which EMS can transport patients with a medical emergency D) It provides lifesaving care instructions to callers accessing the 911 system Answer: A Diff: 1 Page Ref: 7 Objective: 1-6 37) As a supervisor with an emergency ambulance service, which situation regarding a potential employee would you recognize as falling under the Americans with Disabilities Act (ADA), and as case in which you would make reasonable accommodations? A) An EMT who has requested help with a drug and alcohol problem B) Applicant who cannot pass the EMT certification exam because he is dyslexic C) A blind patient who wants to become an EMT, but has not taken a class yet D) A dispatcher with a prosthetic leg who requires parking next to the building entrance Answer: D Diff: 3 Page Ref: 8 Objective: 1-7 38) How would the Americans with Disabilities Act (ADA) be satisfied for a pregnant emergency medical dispatcher (EMD) who suffers from paralysis of both legs and requires the use of a wheelchair? A) Wheelchair accessibility to and from dispatch B) Hourly pay rate 10 percent above that of the other EMDs C) Scheduled shifts during daylight hours only D) Transport by your service to and from work Answer: A Diff: 2 Page Ref: 8 Objective: 1-7 39) A young man wishes to become an EMT and asks you what he will need to do to become certified. You would reply that he must first: A) Volunteer with an ambulance service to gain experience B) Complete a course that meets the requirements of the U.S. Department of Transportation C) Demonstrate that he has prior medical experience D) Attend classes at a local community college Answer: B Diff: 2 Page Ref: 4 Objective: 1-3

10 Copyright © 2018 Pearson Education, Inc.

40) The functions of a state EMS agency include: A) Determination of the number of EMRs, EMTs, AEMTs, and paramedics allowed in the state B) Review of patient care reports (PCRs) to ensure proper care C) Planning and coordination of the statewide EMS system D) Encouraging fair wages for the different levels of EMS providers Answer: C Diff: 2 Page Ref: 4 Objective: 1-3 41) What actions place the patient at greatest risk for a medical mistake? A) Driving to an emergency department at night B) Backing the ambulance into the ambulance bay at the hospital C) Responding to a call in an "unsafe" neighborhood D) Transferring care from one EMT to another Answer: D Diff: 2 Page Ref: 14 Objective: 1-11 42) Which statement best summarizes the underlying benefit of using evidence-based medicine? A) Formulates emergency care guidelines that have been reviewed by the legal community B) Improves patient care while simultaneously reducing the costs associated with EMS C) Provides for legal representation in the quality improvement process D) Allows the use of research and evidence in determining which care is effective Answer: D Diff: 2 Page Ref: 14 Objective: 1-13 43) Which item of protective gear is appropriate to wear at an automobile collision? A) Gown B) A HEPA mask C) Reflective clothing D) SCBA gear Answer: C Diff: 1 Page Ref: 9 Objective: 1-12 44) At a bare minimum, which type of personal protective equipment should be worn by EMS providers at any emergency scene involving a shooting? A) Face mask B) Gloves C) Gown D) Goggles Answer: B Diff: 1 Page Ref: 9 Objective: 1-12

11 Copyright © 2018 Pearson Education, Inc.

45) Which of these tasks has not been identified as one that could be provided by a mobile integrated health care system? A) Manage patients who are at a high risk for hospital readmission B) Assist in the ongoing home management of chronic conditions C) Provide preventive services and check up on patients after discharge D) Conduct health screening and provide minor outpatient surgical services Answer: D Diff: 2 Page Ref: 17 Objective: 1-15 46) In the public health continuum, what is the first responsibility that public health officials must satisfy to improve the health of a community? A) Identify problems that affect the health of the population in question B) Design a plan for remediating the disturbances that cause a population's health to decline C) Integrate their work with EMS services to provide outpatient home care services designed to improve health D) Meet with city administrators to determine which type of public health budget will be provided so cost-effective plans can be established Answer: A Diff: 2 Page Ref: 16 Objective: 1-14 47) Which attribute of a person would not likely work out well in the EMS profession? A) Inability or no desire to "take charge" B) Being able to make good decisions with minimal information C) Being able to adapt to a situation when things become chaotic D) Willingness to hold what is learned about patients, good and bad, in confidence Answer: A Diff: 2 Page Ref: 11 Objective: 1-8

12 Copyright © 2018 Pearson Education, Inc.

Prehospital Emergency Care, 11e (Mistovich et al.) Chapter 2 Workforce Safety and Wellness of the EMT 1) A distraught husband awoke to find his 26-year-old wife not breathing and cold to the touch. Since your assessment indicates that the woman has been dead for some time, you contact medical direction for authorization to withhold emergency care. The medical director agrees and the coroner is notified. At this time, your immediate responsibility is: A) Establishing the time of death B) Preparing for the next call C) Determining the cause of death D) Providing supportive care to the husband Answer: D Diff: 2 Page Ref: 22 Objective: 2-3 2) You are doing CPR on a male patient who is unresponsive, not breathing, and pulseless. Emergency Medical Responders (EMRs) were on scene first and applied the AED, which did not deliver a shock to the patient. The patient's wife is crying and asks how her husband is doing. Your response would be: A) "I have to be honest, he is probably dead, but we are going to take him to the hospital so the doctor can examine him and make sure." B) "Why don't we wait until we get to the hospital to talk about your husband's condition?" C) "Right now he is not breathing and his heart is not beating, but we are doing everything we possibly can for him." D) "I am not a doctor, but his current condition is very grave and he will likely die, based on my previous experiences." Answer: C Diff: 2 Page Ref: 22 Objective: 2-3 3) The EMT practice of protecting oneself from disease transmission through exposure to blood and other body fluids is referred to as: A) Standard Precautions B) Exposure control C) Personal protective equipment D) Body Fluid Precautions Answer: A Diff: 1 Page Ref: 28 Objective: 2-1

1 Copyright © 2018 Pearson Education, Inc.

4) An EMT is correct when he describes a communicable disease as: A) Any of a number of diseases that are deadly once contracted B) A disease that can be spread from one person to another C) Any disease that is caused and spread by a viral agent D) A disease for which no immunization exists Answer: B Diff: 1 Page Ref: 27 Objective: 2-9 5) What is the single best technique that the EMT can use to prevent the spread of infection? A) Disinfecting equipment B) Wearing gloves C) Washing hands D) Getting immunizations Answer: C Diff: 2 Page Ref: 28 Objective: 2-9 6) The EMT is correctly performing routine handwashing when she: A) Uses an alcohol-based hand sanitizer without water B) Rinses with hot water after using a disinfectant soap C) Lathers and scrubs with plain soap for 20 seconds D) Dries hands thoroughly for 30 seconds with a reusable towel Answer: C Diff: 2 Page Ref: 29 Objective: 2-9 7) A patient with human immunodeficiency virus (HIV) infection is complaining of chest pain. In this situation, which personal protective equipment, at a minimum, is required to be donned prior to contact with the patient? A) Gloves and eyewear B) Gloves and gown C) Gloves, gown, and N-95 respirator D) Gloves, gown, goggles, and mask Answer: A Diff: 2 Page Ref: 33-34 Objective: 2-12

2 Copyright © 2018 Pearson Education, Inc.

8) The hospital staff has just returned the long spine board that you used to immobilize a patient from a minor motor vehicle collision. There is no blood on the board, but your service policy states that all equipment is to be disinfected anytime that it is used. To properly disinfect the long spine board, which type of product should you use? A) Soap and water B) Sterilizing solution C) Germicidal solution D) Superheated steam Answer: C Diff: 2 Page Ref: 31-32 Objective: 2-9 9) Which immunization should the EMT receive annually? A) Purified protein derivative B) Measles, mumps, and rubella vaccine C) Hepatitis B vaccine D) Influenza vaccine Answer: D Diff: 2 Page Ref: 32 Objective: 2-13 10) You have been told that you must get your purified protein derivative (PPD) test for tuberculosis (TB). As a knowledgeable EMT, you recognize that the PPD test will: A) Provide continued immunity to tuberculosis B) Need to be repeated annually C) Reveal whether you need a TB immunization D) Determine the effectiveness of treatment for TB Answer: B Diff: 2 Page Ref: 32 Objective: 2-9 11) Which statement made by an EMT demonstrates that he has an accurate understanding of hepatitis C? A) "It is important to get the hepatitis C vaccine to decrease my chance of contracting the disease if exposed to the virus." B) "Wearing gloves and washing hands are the best means by which to prevent the transmission of hepatitis C to health care workers." C) "Since hepatitis C is primarily transmitted in the spring and summer, more precautions are needed during that time." D) "Respiratory droplets are the primary means by which hepatitis C is transmitted, making a mask necessary when caring for patients with this disease." Answer: B Diff: 3 Page Ref: 33 Objective: 2-11

3 Copyright © 2018 Pearson Education, Inc.

12) While you are assessing a patient with abdominal pain, the patient informs you that he is a chronic carrier of hepatitis B. His skin is not jaundiced, nor does he have a fever. As an EMT, you realize: A) The patient has the disease but is not infectious B) The hepatitis B vaccine will not protect you if you are exposed to the virus C) The patient's blood and body fluids are infectious D) A mask must be placed over the patient's mouth Answer: C Diff: 3 Page Ref: 32 Objective: 2-13 13) You are treating a patient with active tuberculosis. What will afford you the best protection from the disease while you care for the patient? A) Surgical mask with a face shield B) Purified protein derivative injection C) Gloves, gown, and goggles D) High-efficiency particulate aspirator mask Answer: D Diff: 2 Page Ref: 33 Objective: 2-12 14) A patient informs you that he was recently diagnosed with the human immunodeficiency virus (HIV) infection, but has yet to develop AIDS. Given this information, which statement is true? A) Until the patient develops AIDS, he is not infectious B) Family members should not share the patient's hand towels or utensils C) The hepatitis B and C vaccines provide the EMT with limited protection from transmission of HIV D) Gloves and eye protection will be enough protection under most conditions unless body fluids are present Answer: D Diff: 2 Page Ref: 33-34 Objective: 2-12 15) Preincident stress education, one-on-one support, and disaster support services are all part of: A) A comprehensive CISM program B) A balanced work environment C) Burnout prevention programs D) Stress reduction techniques Answer: A Diff: 2 Page Ref: 26 Objective: 2-8

4 Copyright © 2018 Pearson Education, Inc.

16) You are dispatched to a motor vehicle collision on a remote country road. It is night, and your scene size-up reveals a heavily damaged car that rolled several times. The driver has been ejected and is lying prone in the middle of the road. In this situation, which type of protective clothing is it most essential that you don prior to making patient contact? A) Rubber-soled boots B) Reflective vest C) Protective helmet D) Leather gloves Answer: B Diff: 2 Page Ref: 37 Objective: 2-14 17) As EMTs leave a house where the patient refused care, a panicked boy approaches them and states that some men just shot his friend in a house two doors up the street. The EMTs' immediate action should be to: A) Instruct the boy to carry the patient from the house to the yard B) Gather basic equipment and go to the home to care for the patient C) Notify dispatch and proceed to the home to care for the patient D) Contact the police and move the ambulance from the immediate area Answer: D Diff: 2 Page Ref: 39 Objective: 2-14 18) You are transporting a 58-year-old female with terminal breast cancer. While in the ambulance, she becomes tearful yet agitated, and repeatedly asks, "Why me?" As a knowledgeable EMT, you recognize that she is in which stage of grieving? A) Anger B) Denial C) Bargaining D) Depression Answer: A Diff: 2 Page Ref: 21 Objective: 2-2 19) A reaction to death or dying where the patient asserts "Not me!" is recognized as which stage of grieving? A) Anger B) Denial C) Bargaining D) Depression Answer: B Diff: 2 Page Ref: 21 Objective: 2-2

5 Copyright © 2018 Pearson Education, Inc.

20) What statement would you expect to hear from a patient who has accepted the fact that he is terminally ill and will die sooner than later? A) "It is just not fair." B) "It's okay. I am not afraid." C) "I have no energy to even think anymore." D) "I cannot go on like this any longer." Answer: B Diff: 2 Page Ref: 21 Objective: 2-3 21) Organisms that cause diseases are called: A) Pathogens B) Toxins C) Antigens D) Antibodies Answer: A Diff: 1 Page Ref: 27 Objective: 2-1 22) A reference tool that should be placed in every emergency vehicle for use when responding to a possible "hazmat" incident is: A) The Emergency Response Guidebook B) The Physician's Desk Reference C) The Merck Manual D) A list of chemicals processed by local industries Answer: A Diff: 1 Page Ref: 36 Objective: 2-14 23) To ensure your own safety and the safety of your crew, you should: A) Request police assistance on all calls B) Search each and every patient for weapons C) Evaluate every scene for danger before entering D) Ensure the availability of a paramedic for most medical calls Answer: C Diff: 1 Page Ref: 26 Objective: 2-14

6 Copyright © 2018 Pearson Education, Inc.

24) You have worked with the same partner for several years and know him well. Over the past three months, the two of you have had four horrific pediatric calls with death as the outcome of each. Which observation would make you the most suspicious that your partner is experiencing a possible stress reaction that might encumber his ability to work as an EMT? A) Excessive and uncharacteristic irritability B) Request to be scheduled for night shifts C) Interest in going back to school to study accounting D) Statement that he intends to drop to part-time status Answer: A Diff: 2 Page Ref: 23 Objective: 2-5 25) Changes in your lifestyle that may help you manage stress include: A) Eating a high-protein and fatty diet B) Concentrating on your job 24 hours a day C) Self-medication with over-the-counter medicines D) Practicing relaxation techniques such as meditation Answer: D Diff: 1 Page Ref: 24 Objective: 2-7 26) You have just been instructed to stop emergency care on an unresponsive and pulseless 61year-old man found by his wife in the bathroom. When you tell his wife that there is nothing more that can be done, she erupts in an episode of rage and starts screaming that there must be something else that can be done and you are not doing your job well enough. In this situation, which action is best? A) Resume resuscitation for another 5 minutes B) Be tolerant of the angry reaction C) Leave the scene immediately D) Contact medical direction for advice Answer: B Diff: 2 Page Ref: 22 Objective: 2-2 27) After a particularly disturbing call the previous day, your partner is unusually angry and having a difficult time concentrating on his work. Which type of stress reaction would you suspect? A) Acute stress reaction B) Posttraumatic stress reaction C) Physical stress reaction D) Cumulative stress reaction Answer: A Diff: 2 Page Ref: 23 Objective: 2-4

7 Copyright © 2018 Pearson Education, Inc.

28) A coworker is suffering from posttraumatic stress. Which statement made by you would be the most appropriate and beneficial to the coworker? A) "It would be best to take a few weeks off from work." B) "Maybe you can get the medical director to write a prescription for some medication." C) "I think that it would be best if you saw a mental health professional." D) "How about taking a Benadryl before bed to promote restful sleep?" Answer: C Diff: 2 Page Ref: 23 Objective: 2-4 29) A prospective student is interested in becoming an EMT, but is unsure how it will affect his family. He says that his wife is very worried about the danger associated with EMS and has never had to deal with him being gone for 24 hours at a time. What is the most appropriate response? A) "The long hours actually make things easier since there is less time to argue." B) "I have found that the family becomes used to it and things are okay after about six months." C) "It may help to bring your wife here so she can see what we do and the safety precautions we take." D) "I would just tell her that everything will be okay and avoid talking about work when you are home." Answer: C Diff: 1 Page Ref: 25 Objective: 2-6 30) You and your partner were among several rescuers on a call involving a house fire in which a 4-year-old girl died. That night, your partner states that he did not sleep well and is extremely bothered by the little girl's death. The next day he calls you and states that he is so upset that he is now having chest pain. What is the most appropriate response? A) "Why don't you request a week off of work to get things sorted out?" B) "Maybe you should get an appointment with a behavioral health professional." C) "Perhaps we need to have a critical incident stress debriefing so we can all talk." D) "You really need to get yourself to the emergency department to get checked out." Answer: D Diff: 2 Page Ref: 26 Objective: 2-5 31) You would be concerned about cumulative stress when an EMS provider informs you that she: A) Is physically tired and would like one week of unpaid leave to recuperate B) Drinks a small amount of alcohol every night to help her sleep C) Is taking prescribed antidepressants to treat mild depression D) Is bothered by a call she had yesterday and does not feel like talking Answer: B Diff: 3 Page Ref: 23-24 Objective: 2-4

8 Copyright © 2018 Pearson Education, Inc.

32) A shooting at a public mall leaves three people dead and nine injured, several critically. As operations wind down, many rescuers are physically and emotionally upset. Which event may be most appropriate at this specific time? A) A critical incident stress debriefing session (CISD) B) A discussion on how to reduce cumulative stress C) A gathering that allows the rescuers to vent their emotions D) Providing information about mental health programs Answer: C Diff: 2 Page Ref: 26 Objective: 2-8 33) A tornado ripped through a small town, killing 5 people and seriously injuring 21, including several young children. At a defusing session, which action is most appropriate? A) Allow the rescuers to vent their emotions B) Evaluate and critique the rescue effort C) Provide information on stress reduction techniques D) Have a mental health professional available to talk with the rescuers Answer: A Diff: 2 Page Ref: 26 Objective: 2-8 34) If you have questions regarding the steps to take when exposed to an infectious disease, your best resource would be: A) The U.S. Department of Health and Human Services B) Your service's written exposure control plan C) Your service's medical director D) Your primary care physician or ED physician Answer: B Diff: 1 Page Ref: 27 Objective: 2-13 35) You have just transferred care of a patient to an emergency department nurse. There is no blood, fluid, or dirt on your gloves. After taking them off, your best action would be to: A) Use an alcohol-based hand sanitizer B) Disinfect the stretcher and equipment C) Wash your hands with soap and water D) Complete the prehospital care report Answer: C Diff: 1 Page Ref: 28-29 Objective: 2-11

9 Copyright © 2018 Pearson Education, Inc.

36) A 38-year-old male is complaining of generalized weakness and fatigue. Assessment findings include abdominal pain, fever, and yellowing of the eyes. Given these signs and symptoms, you would be suspicious of: A) Hepatitis B) Tuberculosis C) AIDS D) West Nile virus Answer: A Diff: 1 Page Ref: 33 Objective: 2-10 37) You are accidentally stuck with a needle used to administer a medication to a patient with a known history of hepatitis B. You have already completed your hepatitis B vaccine series. In this case, which statement is true? A) You have complete protection from hepatitis B and no further action is needed B) You have two weeks from the time of exposure to start medical care for a potential infection C) You will have to get another series of three vaccines to give you protection in the future D) You need to report the needle stick to your supervisor as soon as possible Answer: D Diff: 2 Page Ref: 33 Objective: 2-13 38) A patient with a history of hepatitis C has called 911 because he has a headache. Given the complaint and medical history, which personal protective equipment, at a minimum, is most appropriate? A) Goggles B) Gloves C) Gloves and gown D) No protection is needed. Answer: B Diff: 1 Page Ref: 33 Objective: 2-11 39) The primary personal threat to a person with AIDS is: A) Infection B) Organ damage C) Heart failure D) Weight loss Answer: A Diff: 1 Page Ref: 34 Objective: 2-10

10 Copyright © 2018 Pearson Education, Inc.

40) In the field setting, the best way to avoid becoming infected with human immunodeficiency virus (HIV) is: A) Avoid unnecessarily touching patients with confirmed HIV infection B) Always ask patients if they have HIV infection or AIDS C) Always wear gloves, goggles, and a gown for all patient contacts D) Use the appropriate protective personal equipment Answer: D Diff: 1 Page Ref: 34 Objective: 2-13 41) You are by the side of a patient whom you suspect has severe acute respiratory syndrome (SARS). What is an effective way to prevent transmission of the disease from the patient to you? A) Use eye protection or a face shield B) Use an alcohol-based hand sanitizer frequently during transport C) Place a surgical mask on the patient D) Sterilize all equipment used in the care of the patient Answer: C Diff: 1 Page Ref: 34 Objective: 2-11 42) You have been called to transport a patient from one medical facility to another. The patient is infected with vancomycin-resistant enterococci (VRE), but does not have any open wounds and is medically stable. Given this information, which action is most appropriate? A) Place a surgical mask on the patient B) Place a surgical mask on the patient and yourself C) Use an N-95 respirator or HEPA mask D) Use the appropriate personal protective equipment and handwashing Answer: D Diff: 2 Page Ref: 35 Objective: 2-13 43) Which organism is classified as multi-drug resistant? A) Methicillin/oxacillin-resistant Staphylococcus aureus B) Hepatitis B virus C) Human immunodeficiency virus D) Zika virus Answer: A Diff: 1 Page Ref: 35 Objective: 2-10

11 Copyright © 2018 Pearson Education, Inc.

44) Which rationale best explains the reason for wearing a high-visibility safety vest at the scene of a motor vehicle collision? A) Wearing the vest makes you easily identifiable as an EMT B) The use of the vest can increase your visibility to passing drivers at the scene of an accident C) Federal law requires that you wear the vest or the EMS system can be fined D) The vest increases your visibility to bystanders and protects you from blood and body fluids Answer: B Diff: 2 Page Ref: 37 Objective: 2-14 45) What action taken by an EMT illustrates an effort toward maintaining well-being in the field of EMS? A) Taking a body-building class B) Avoiding all alcohol and fatty foods C) Creating a healthy balance between work and family life D) Establishing a relationship with a mental health professional Answer: C Diff: 1 Page Ref: 41 Objective: 2-15 46) After 20 minutes of performing CPR on a 65-year-old female with end-stage multiple sclerosis, your medical director orders you to stop resuscitation and inform the family that the patient has died. The family members are upset and crying. You notice the patient's daughter holding her hand. Your immediate response would be: A) "It's okay to hold her hand, but you should really use gloves like we do." B) "It's important not to touch her since she may be infectious from her disease." C) "I know that you are upset, but it is best not to touch her since she just passed." D) "It's okay to hold her hand." Answer: D Diff: 3 Page Ref: 22 Objective: 2-3 47) A young girl with end-stage cancer is unresponsive, not breathing, and pulseless. Her mother and father are by her side, both crying as you start resuscitation. Which statement indicates the most appropriate thing you could say to the parents as you begin? A) "I think that it would be best for you both to leave the room so we can care for your daughter." B) "If you feel comfortable, it's okay to stay and watch how we care for your daughter." C) "It is important not to cry since she may still be able to hear you and needs you to be strong for her." D) "I can show you how to do chest compressions so that you can take part in caring for your daughter." Answer: B Diff: 3 Page Ref: 22 Objective: 2-3

12 Copyright © 2018 Pearson Education, Inc.

48) A patient with terminal lung cancer has called EMS for shortness of breath. At the request of your paramedic partner, you are obtaining the patient's blood pressure. While inflating and then deflating the blood pressure cuff, the patient angrily shouts, "That thing is too tight! What are you trying to do–send me to the funeral home early?" Your best reply would be: A) "I have to get your blood pressure. The paramedic needs it." B) "Why are you so mad at me? I am just trying to help you." C) "I know that this is tight and uncomfortable, but I am almost done now." D) "Do not yell at me. There is no need for yelling and I do not appreciate it." Answer: C Diff: 3 Page Ref: 22 Objective: 2-3 49) As an EMT, you would recognize what situation as having the greatest potential for causing emotional stress? A) Lifting and moving an obese patient from a second-floor bedroom B) Being required to show up for a shift 15 minutes prior to its start C) Caring for a child whose parents have appeared to have neglected him D) Responding to many EMS calls during the course of a shift Answer: C Diff: 2 Page Ref: 23 Objective: 2-5 50) Regarding "burnout" and the emergency medical services, what statement is most accurate? A) Employers are required to have a written plan addressing EMT burnout B) Burnout can diminish an EMT's ability to provide competent care C) Research indicates that burnout affects all EMS personnel within a five-year period D) Burnout often tends to resolve on its own over time Answer: B Diff: 2 Page Ref: 23 Objective: 2-4 51) Your supervisor has assigned you the task of designing a program that provides for stress management and health promotion within the department. Which program would accomplish this task in the best way? A) Complementary passes to a local fitness center B) Mandatory monthly meetings for all EMS personnel with a mental health counselor C) Doughnuts and coffee every morning for all EMS personnel D) A weekly night out for beer, wings, and other snacks at a local eatery Answer: A Diff: 3 Page Ref: 24 Objective: 2-7

13 Copyright © 2018 Pearson Education, Inc.

52) An EMT has an understanding of the different types of pathogens when she states: A) "Bacteria tend to be larger than viruses and can be seen without a microscope." B) "Protozoa are considered parasitic worms and can live within the human body." C) "Fungal infections typically attack the immune systems of healthy persons." D) "Viruses are resistant to antibiotics because they live within the body's cells." Answer: D Diff: 1 Page Ref: 27 Objective: 2-10 53) You are at a long-term care facility at the bedside of an 89-year-old female with an infected area on her back. The staff members state that they are fearful that the patient has a MRSA infection, so they have requested transport to the emergency department for further evaluation. To decrease your chance of acquiring MRSA, what should you, as an EMT, do? A) Wash your hands following patient contact B) Place a mask on the patient C) Wash the infected area with an alcohol sanitizer prior to transporting the patient D) Avoid unnecessarily touching the patient Answer: A Diff: 2 Page Ref: 28 Objective: 2-11 54) You are attempting to increase your cardiovascular endurance in an effort to better perform your job as an EMT. Which action would be most appropriate in working toward this goal? A) Increasing sleep to 8 to 10 hours per day B) Weight lifting with attention to the upper body C) Walking on a treadmill 3 to 4 times per week D) Doing stretching exercises every 4 hours while awake Answer: C Diff: 2 Page Ref: 40 Objective: 2-15 55) A coworker is constantly tired and asks you for advice on how to better sleep after a busy night shift. Which response would best address this situation? A) "A small amount of alcohol can be relaxing and effective in helping you to sleep." B) "Try to make your bedroom dark and keep the temperature cool." C) "Research has shown that exercising prior to bed helps you fall asleep faster." D) "Try eating a meal before going to bed, since digestion makes you drowsy." Answer: B Diff: 3 Page Ref: 40-41 Objective: 2-15

14 Copyright © 2018 Pearson Education, Inc.

56) Your friend and coworker got married a little more than a year ago, and then shortly thereafter she became an EMT and started working with you. On your shifts, she complains that her new husband doesn't like the fact she is an EMT, saying that the job is "too risky." Which bit of advice could you provide to your partner to help her calm the nerves of her new husband? A) Explain to her husband that EMS is "just a job" and is not any risker than other occupations B) Let the husband know about all the training EMTs have to complete and which types of safety precautions they take on each call C) Tell your partner that you personally would like to talk to her husband to let him know that you have been in EMS for years without any incident or injury D) Do not tell the husband about the types of calls or situations encountered in EMS so that he starts to believe it's becoming safer Answer: B Diff: 2 Page Ref: 25 Objective: 2-6

15 Copyright © 2018 Pearson Education, Inc.

Prehospital Emergency Care, 11e (Mistovich et al.) Chapter 3 Medical, Legal, and Ethical Issues 1) A patient with diabetes has an elevated blood sugar (545 mg/dL) and, based on your assessment of his condition, will require an injection of insulin. Your EMT partner also has diabetes and administers insulin injections to himself throughout the day as needed. Although the Scope of Practice lists nothing about EMTs giving insulin, your partner administers the insulin injection to the patient. The patient's blood sugar comes down and her condition improves. Given this information, which statement is true? A) It was permissible for your partner to give the insulin since he has diabetes and knows how to give the injections B) Since the patient benefited from the insulin, your partner acted appropriately C) Because your partner used the patient's insulin rather than his own, he cannot be reprimanded D) Your partner violated the Scope of Practice and may have his certification revoked Answer: D Diff: 2 Page Ref: 47 Objective: 3-2 2) You have moved to another state and wish to work as an EMT. In your previous state of employment, EMTs were allowed to administer a specific set of drugs. To determine whether EMTs can administer drugs in your new state of residence, you should review the: A) State Emergency Medical Technicians' Scope of Practice B) National EMS Scope of Practice Model C) National EMS Education Standards D) State Emergency Medical Services Act Answer: A Diff: 2 Page Ref: 47 Objective: 3-2 3) What best illustrates the EMT's failure to meet the standard of care when treating a patient with chest pain? A) Failing to assist the patient in taking her heart palpitation medication B) Failing to notify the patient's primary care physician C) Failing to give the patient oxygen D) Failing to start an IV on the patient Answer: C Diff: 1 Page Ref: 47 Objective: 3-2

1 Copyright © 2018 Pearson Education, Inc.

4) The EMT shows understanding of the standard of care when he states: A) "The standard of care is a state law that describes which actions the EMT can and cannot legally perform." B) "The standard of care is a measure of whether the right assessment and care have been provided to the patient." C) "The standard of care is a document that outlines the curriculum to which all EMT programs in the United States must subscribe." D) "The standard of care is state law that requires the EMS to respond and render care in emergency and nonemergency situations." Answer: B Diff: 3 Page Ref: 47 Objective: 3-2 5) A 77-year-old female is unresponsive, but breathing and with a pulse. On scene, a family member hands you an official document stating that the patient does not want feeding tubes, ventilators, or other long-term life support equipment to keep her alive. There is no mention of whether to administer lifesaving drugs or withhold cardiopulmonary resuscitation. The EMT would recognize this document as a(n): A) Durable power of attorney B) Living will document C) "Do Not Resuscitate" (DNR) order D) "Involuntary Resuscitation" order Answer: B Diff: 1 Page Ref: 50 Objective: 3-1 6) You have been called to a home for an alert and oriented female with terminal lung cancer. At the home, the daughter, who has her mother's durable power of attorney, states that she wants her mother transported to the hospital. The patient, who is alert and oriented, does not want to go. Which statement indicates that the EMT is acting both appropriately and legally in this conflict? A) "Since your daughter has the durable power of attorney, she has the legal right to make all decisions; therefore, we have to take you to the hospital." B) "As the person with your durable power of attorney, your daughter has the legally vested right to make your decisions; however, let us ask your husband what he thinks." C) "I know that you have the durable power of attorney, but your mother is alert and oriented so we cannot legally take her against her will." D) "Since it appears there is a conflict of opinion here, I will need to call the police so that they can determine what to do." Answer: C Diff: 2 Page Ref: 50 Objective: 3-11

2 Copyright © 2018 Pearson Education, Inc.

7) A patient with a terminal disease is unresponsive, not breathing, and without a pulse. The family states that the patient has a "Do Not Resuscitate" (DNR) order signed by her medical doctor, but they cannot find the current one. All they can find is a previous DNR that expired 8 months ago. The EMT should: A) Wait for family to locate the DNR B) Pronounce the patient deceased C) Have the family sign a refusal of care form D) Start cardiopulmonary resuscitation Answer: D Diff: 2 Page Ref: 50 Objective: 3-11 8) You have been dispatched to a residence for an 88-year-old female who is in and out of consciousness. The family provides you with a valid "Do Not Resuscitate" (DNR) order. Assessment of the patient shows her to be very confused with a low blood pressure and low oxygen level. Which action would be appropriate given the situation? A) Administer supplemental oxygen B) Provide transport but no additional care C) Provide care only if the patient's heart stops beating D) Have the patient or family member sign a refusal of care form and clear the scene Answer: A Diff: 2 Page Ref: 50 Objective: 3-11 9) The EMT shows she understands "Do Not Resuscitate" (DNR) orders and living wills when she states: A) "A living will is a form of DNR; only a living will states exactly how the patient wants to be resuscitated." B) "A DNR applies to resuscitation issues only, such as when a patient stops breathing or the heart ceases to beat, whereas a living will pertains to long-term life support care and equipment." C) "If a patient has a valid DNR, the EMT should withhold all care and transport the patient to the hospital. If a living will is present, the EMT should do exactly what it says." D) "If the patient has a living will, it can legally be used as a substitute for a DNR order by the EMT as long as it is properly dated and signed." Answer: B Diff: 3 Page Ref: 50 Objective: 3-11

3 Copyright © 2018 Pearson Education, Inc.

10) What action best indicates that informed consent has been obtained? A) The EMT administers oxygen because the patient is having chest pain and appears short of breath B) The patient agrees to oxygen therapy after being told it may help decrease his chest pain but may dry his nose, causing discomfort C) The EMT tells the patient that he will need oxygen because he is having chest pain, which could be a sign of a heart attack D) The patient is given oxygen because he is having chest pain and was the one who placed the 911 call for help Answer: B Diff: 2 Page Ref: 49 Objective: 3-10 11) A 62-year-old male is short of breath. The EMT asks the patient if it would be okay to assess him, including taking his vital signs. The patient agrees by nodding the head "yes." As such, which is true? A) Informed consent has been obtained by the EMT B) The EMT has obtained expressed consent C) The patient can be transported without further permission D) The EMT has gained implied consent Answer: B Diff: 2 Page Ref: 50 Objective: 3-10 12) The EMT gains the greatest protection from consent-related legal issues by: A) Documenting the patient's consent for treatment on the prehospital care report B) Notifying the emergency physician or medical direction of the patient's consent C) Having his or her partner serve as a witness to the consent D) Asking the patient twice if it is okay to assess and treat him or her Answer: A Diff: 2 Page Ref: 54 Objective: 3-11 13) Which patient may be treated under the guideline of implied consent? A) A 2-year-old boy who fell down a flight of stairs and whose parents want him taken to the hospital B) An unresponsive 33-year-old female who is suspected of taking recreational drugs C) An adult patient with chest pain and shortness of breath who is refusing treatment and transport by EMS D) A 47-year-old female with severe abdominal pain who is refusing transport after placing the call to 911 for help Answer: B Diff: 2 Page Ref: 50 Objective: 3-10

4 Copyright © 2018 Pearson Education, Inc.

14) An alert and oriented patient complaining of a severe headache is refusing all treatment as well as transport by the EMTs. After having signed the refusal, the patient suddenly seizes and becomes unresponsive prior to EMS providers leaving the home. The EMT should: A) Contact medical direction for permission to treat B) Honor the patient's wishes and leave the scene C) Place the patient into the family's car for transport to the hospital D) Provide care under the guideline of implied consent Answer: D Diff: 3 Page Ref: 50 Objective: 3-10 15) You have been dispatched for a 15-year-old female with a headache. On scene, you find the patient holding her 6-month-old son in her arms. This young mother states that she has had a migraine headache for two days and would like to be transported to the hospital for further evaluation. As an EMT, you should: A) Obtain a parent's consent to treat the patient, given her age B) Refuse transport since the patient does not have a true emergency C) Assess and transport the patient as she desires D) Treat the patient under the doctrine of implied consent Answer: C Diff: 2 Page Ref: 50 Objective: 3-10 16) In a community park near his home, a 10-year-old boy has fallen from the top of a slide. His right wrist is deformed and he is in pain. He states that he lives with his mother and she is working. He does not know her work number. The EMT's best action would be to: A) Treat the patient under the guidelines of implied consent B) Do not treat the patient until his mother can be contacted C) Obtain permission from the park director D) Have the police take custody of the boy and give permission to treat Answer: A Diff: 2 Page Ref: 50 Objective: 3-10 17) An adult patient with the capacity to make his own decisions has cardiac-type chest pain and is refusing all treatment. What would be the EMT's most appropriate response? A) "I will need to have you sign this refusal of care form and a family member sign as a witness." B) "Before you refuse, I need to tell you that this could be a heart attack and you could die." C) "Since you are oriented and rational, it is your right to refuse. Please sign this refusal of care form." D) "I am sorry, but since 911 was called, we must transport you to the hospital. You can refuse treatment there." Answer: B Diff: 3 Page Ref: 54 Objective: 3-12 5 Copyright © 2018 Pearson Education, Inc.

18) An intoxicated 72-year-old male with an alcoholic history has fallen at home and has a laceration on the back of his head. He states that he does not want treatment and becomes combative when you try to talk to or assess him. What is your next best course of action? A) Transport him against his will, but do not provide treatment B) Have him sign a refusal of care form C) Contact medical direction D) Transport him by permission of family Answer: C Diff: 2 Page Ref: 54 Objective: 3-12 19) A 46-year-old man who is conscious and has the capacity to make a rational decision is refusing treatment despite crushing chest pain and shortness of breath. He states that the hospital will not do anything for him and he does not want you to treat or transport him. The EMT should recognize that the: A) Patient has a right to refuse treatment and transport B) Police will need to be called so the patient can be transported in custody C) Patient will need to be transported against his will D) Patient's family can give consent to transport the patient Answer: A Diff: 2 Page Ref: 53 Objective: 3-12 20) Your best protection when an alert and oriented patient with the capacity to make rational decisions refuses EMS care or transport is to: A) Inform the patient's family physician of what happened B) Have your partner sign the refusal form as a witness C) Contact the patient at a later time to see how she is doing D) Completely and thoroughly document the incident Answer: D Diff: 2 Page Ref: 54 Objective: 3-12 21) A patient who has the capacity to make rational decisions, although he is dizzy, complains of a headache, and then vomits, declines your care and will not sign the refusal of care form. What is your best action? A) Document that the patient refuses to sign and leave B) Have a family member sign as a witness to the refusal C) Have your partner sign on behalf of the patient D) Inform the patient that by law he must sign the form Answer: B Diff: 2 Page Ref: 55 Objective: 3-12

6 Copyright © 2018 Pearson Education, Inc.

22) For negligence to be proven, what must be present? A) Duty to act, breach of duty, injury, proximate cause B) Duty to act, breach of duty, battery, foreseeability C) Assault, breach of duty, damage, and then abandonment D) Harm to the patient, false imprisonment, duty to act Answer: A Diff: 1 Page Ref: 55 Objective: 3-14 23) Which of these actions constitutes a breach of the EMT's duty, placing him or her at risk for the charge of negligence? A) Transporting an alert and oriented patient who does not want transport to the hospital B) Applying oxygen to a patient who is not complaining of shortness of breath C) Obtaining an informed and signed refusal of care form from an alert and oriented patient with abdominal pain D) Obtaining a refusal of care from a patient without obtaining vital signs Answer: D Diff: 2 Page Ref: 55-56 Objective: 3-14 24) A patient's spouse, who is a doctor, states that an EMT is negligent because he put his wife on low-concentration oxygen when she should have gotten high-concentration oxygen. The patient was having chest pain that was later determined to be caused by anxiety. She was released from the emergency department later in the day. Which statement is true? A) Negligence can be proven because the protocol states high-concentration oxygen should be given B) Negligence cannot be proven since the EMT responded to the call and did render care C) The EMT cannot be proven negligent because that patient suffered no harm or damage D) Negligence is a distinct possibility since the patient was transported and treated in the emergency department Answer: C Diff: 3 Page Ref: 55 Objective: 3-14 25) While transferring a patient from her house to the ambulance on the wheeled cot, one of the EMTs slips on a patch of ice and falls. The stretcher overturns, and the patient suffers a broken wrist as a result. She files a lawsuit in civil court alleging negligence. For the patient, what will be the most difficult component of negligence to prove? A) Proximate cause B) Duty to act C) Damage D) Implied consent Answer: A Diff: 2 Page Ref: 56 Objective: 3-14

7 Copyright © 2018 Pearson Education, Inc.

26) You have arrived at the emergency department with a patient complaining of nausea. The department is extremely busy and you are waiting to give a verbal report to the nurse or doctor. While waiting, dispatch contacts you over the radio and states that you are needed for a critically injured child that was hit by a car. Your best action would be to: A) Take the call and come back to give the report at a later time B) Provide the certified nurse's aide with a report of the patient's condition C) Go to the nurse's station and provide the registered nurse with a quick oral report D) Leave the patient in the emergency department and respond to the call Answer: C Diff: 2 Page Ref: 56 Objective: 3-15 27) You are by the side of a female patient who has overdosed on a drug. She is conscious but very combative and noncompliant with your requests. Which statement made by the EMT best suggests the EMT is committing assault? A) "You win. I cannot get your blood pressure and the doctor is going to yell at me. I am going to blame you." B) "If that is the way you are going to behave, then I am no longer going to try to get your blood pressure." C) "If you throw up in the back of my ambulance, I will use your shirt to clean it up. Just think about that for a minute." D) "If you do not let me get your blood pressure, I will take off the oxygen and you will suffocate." Answer: D Diff: 2 Page Ref: 56 Objective: 3-15 28) Despite a patient's protest that he does not want his blood pressure taken, the EMT places a BP cuff on his arm and takes his blood pressure. The EMT states: "See? It is just as I suspected– your blood pressure is sky high. You really need to be seen in the emergency department." The patient states he did not realize that his blood pressure was that high. In terms of battery, which statement is true? A) Since the BP was elevated and treatment must be provided, battery cannot be charged B) The patient could charge the EMT with battery, regardless of his blood pressure reading C) The patient could charge the EMT with assault, not battery D) Because the patient knows the result of the BP, the EMT is safe from battery Answer: B Diff: 2 Page Ref: 56 Objective: 3-15

8 Copyright © 2018 Pearson Education, Inc.

29) A homeless patient is sick and is refusing transport. The patient is alert and clearly has the capacity to understand her situation and make a rational decision, but the temperature tonight will be near zero. The EMT picks up the patient and puts her on the cot. The patient continues to refuse to allow treatment and transport, despite the EMT's explanation that it will be warm in the hospital and she will be fed. The patient is transported. What could the EMT be charged with? A) Assault B) Abandonment C) Negligence D) False imprisonment Answer: D Diff: 2 Page Ref: 56 Objective: 3-15 30) While off duty, an EMT happens upon the scene of a motor vehicle collision in which a car rolled over the side of an embankment. A young man has been seriously injured; he appears unresponsive, and is bleeding from the head and face. The EMT is alone with his infant baby daughter in the car and cannot leave her to provide help. Therefore, the EMT alerts 911 of the emergency and remains in his car with his daughter until EMS arrives. Later in the week, he finds out that the patient died. Which statement is true regarding this incident? A) The EMT had no duty to act and cannot be held liable for the man's death B) The EMT had a legal obligation to help and can be held liable for the death C) The EMT violated the state standard of care by not providing some form of care D) The EMT must go to court and explain that he did not help because of his daughter Answer: A Diff: 3 Page Ref: 48 Objective: 3-3 31) In which situation would the Good Samaritan law protect the EMT? A) The EMT forgets to provide oxygen to a patient with chest pain and the patient suffers a heart attack as a result B) A patient is paralyzed from the waist down because an off-duty EMT did not take the appropriate measures in caring for him C) A paramedic orders an EMT to administer a medication that the EMT legally may not give, and the patient dies as a result D) An off-duty EMT stops at the scene of an auto collision and provides proper care, but the patient dies anyway Answer: D Diff: 2 Page Ref: 48 Objective: 3-5

9 Copyright © 2018 Pearson Education, Inc.

32) Which action best illustrates an EMT upholding a high ethical standard? A) Administering oxygen to patients who are short of breath B) Providing care that goes beyond the scope of practice C) Treating alcoholics and drug addicts with respect D) Informing other health care providers of patients in the community with AIDS Answer: C Diff: 2 Page Ref: 49 Objective: 3-9 33) A local politician has been involved in a motor vehicle collision. Witnesses state that they saw this man leave a bar, and repeatedly stumbled until he got to his vehicle. Then, just two miles down the road, he struck a utility pole head on. On scene and throughout transport, the patient is confused and combative. When you are leaving the hospital, a reporter asks you what happened. You state that it looks as though the politician may be drunk, but you are not sure. Your statement is printed in the newspaper. Later, it is determined that the man was not drunk, but rather has diabetes and had low blood sugar at the time of the accident. Which charge could the politician levy against you? A) Slander B) Assault C) Battery D) Litigation Answer: A Diff: 2 Page Ref: 56 Objective: 3-15 34) There has been a very serious motor vehicle collision involving a high-profile judge seeking reelection. The judge's car was traveling at a high rate of speed and crossed the center line, striking and killing a motorcyclist. Newspaper reporters are on location, asking you to describe what happened. Which statement is most appropriate? A) "The judge was involved, but I cannot tell you anything that happened." B) "The judge was in the car that hit and killed the motorcyclist." C) "There has been a serious accident in which a person was killed." D) "The accident was not bad, so please leave the scene." Answer: C Diff: 2 Page Ref: 57 Objective: 3-16 35) A woman calls the station and asks if you transported a patient named Patricia Gardner earlier in the day. She says that she is her friend and wants to know if she is okay. An appropriate response would be: A) "I can tell you that she was transported to the hospital, but I cannot tell you why." B) "Although I do not doubt that you are her friend, I cannot release that information." C) "I cannot tell you that, but her family is at home and they can give you more information." D) "She was treated for chest pain, but I cannot release her diagnosis for admission." Answer: B Diff: 2 Page Ref: 57 Objective: 3-16 10 Copyright © 2018 Pearson Education, Inc.

36) The EMT recognizes that the Health Insurance Portability and Accountability Act (HIPAA) limits the EMT's ability to: A) Share patient-specific medical information with others B) Bill for services provided by the Emergency Medical Services system C) Obtain refusals of care from patients who do not have medical insurance D) Transport patients to the hospitals of their choice Answer: A Diff: 1 Page Ref: 57 Objective: 3-16 37) You are pulling into the ambulance bay at a local hospital. On board, you have a patient with behavioral problems who continually seeks treatment at this particular hospital. Before the patient can be unloaded, the physician comes out and states that the patient must be taken to another hospital. As an EMT, you should: A) Take the patient to the facility specified by the doctor B) Call the supervisor to report a HIPAA violation C) Recognize a violation of the EMTALA statute D) Ask the patient which hospital he wants to be taken to Answer: C Diff: 2 Page Ref: 58 Objective: 3-17 38) A young male patient has suffered a gunshot wound to the head. Realizing that there is little chance for survival, the EMT notes that the patient is a potential organ donor. The patient is unresponsive and breathing 4 times per minute on scene. The EMT initiates positive-pressure ventilation. En route to the hospital, the patient stops breathing and loses a pulse. Since the patient is a potential organ donor, the EMT should: A) Stop any and all resuscitation measures since this is a trauma arrest B) Provide the same resuscitation as for a person who is not an organ donor C) Ventilate the patient but not provide chest compressions D) Perform CPR but not use the automated external defibrillator Answer: B Diff: 1 Page Ref: 58 Objective: 3-18 39) A 20-year-old male has been stabbed in the abdomen. Assessment reveals him to be unresponsive with shallow breathing and a low blood pressure. Which instructions will you provide to your new EMT partner given that the patient has been involved in a crime? A) "I am not worried about potential evidence. I am just worried about the patient." B) "Do not worry about documenting the crime scene in your report. The police report will reflect that information." C) "We are not to move the patient to the stretcher until the police are done photographing the scene." D) "When cutting his shirt off, make sure not to cut through the area where the knife penetrated." Answer: D Diff: 2 Page Ref: 60 Objective: 3-20 11 Copyright © 2018 Pearson Education, Inc.

40) The highest priority for the EMT working on a patient when approaching the scene of a crime is: A) Preserving potential evidence B) Ensuring personal safety C) Administering quality patient care D) Providing medical information to law enforcement Answer: B Diff: 1 Page Ref: 60 Objective: 3-20 41) EMS providers must report which of these suspected events to law enforcement? A) Alcohol abuse and child abuse B) Drug addiction and drug-related crimes C) Physical and mental abuse D) Refusal of EMS services Answer: C Diff: 1 Page Ref: 60 Objective: 3-20 42) During orientation, an EMT is told that owing to the nature of her employment, she is covered by sovereign immunity. Based on this statement, the EMT must: A) Be immune from claims of negligence B) Work for a public EMS agency C) Work for a nonprofit, privately owned hospital D) Provide services for a volunteer EMS agency Answer: B Diff: 1 Page Ref: 48-49 Objective: 3-6 43) In discussing an EMS response that involved potential negligence, your medical director asks you to determine whether the statute of limitations is active. As a knowledgeable EMT, you understand that you will need to determine which of these? A) The exact action committed that is the basis for a negligence charge B) If there is a proximate cause that can be linked to the bad outcome C) If the allowed time for the patient to file a lawsuit has passed D) The amount of money that can be awarded to the patient and family Answer: C Diff: 1 Page Ref: 49 Objective: 3-6

12 Copyright © 2018 Pearson Education, Inc.

44) A 22-year-old female has been sexually assaulted and taken to the emergency department. With regard to the release of confidential medical information, which individual would most likely be allowed to receive information related to the patient's injuries without her consent? A) State police B) Patient's mother C) Patient's boyfriend D) Crisis counselor Answer: A Diff: 2 Page Ref: 57 Objective: 3-16 45) A male EMT was discussing a patient's medical condition with a female EMT from a different service. The woman then repeated this information to others, which eventually made its way back to the patient. Regarding issues of confidentiality, which of the statements would be is true? A) The male EMT could be changed with slander B) The female EMT committed an EMTALA violation C) Both EMTs violated EMTALA regulations D) The male EMT violated the HIPAA law Answer: D Diff: 2 Page Ref: 57 Objective: 3-16 46) A patient who was transported by EMS for a laceration on her arm is suing the service and the hospital because her laceration became infected, resulting in loss of the extremity. However, it becomes known that the patient was appropriately treated by EMS and the emergency department, but failed to seek follow-up medical care at a health clinic and did not take the antibiotics that were prescribed to her. Which element would most likely be used by the medical community as a defense in this case? A) Res ipsa loquitor B) Proximate cause C) Contributory negligence D) Intentional tort Answer: C Diff: 2 Page Ref: 49 Objective: 3-6 47) You have been called to transfer a trauma patient from a rural hospital emergency department to the emergency department of a large urban medical center. Which action will best decrease your chance of becoming involved in an EMTALA violation? A) Obtaining a copy of the patient's insurance and billing information B) Taking vital signs every 5 minutes during transport C) Determining the exact location to which the patient is to be transported D) Assessing the patient for stability prior to transfer Answer: D Diff: 2 Page Ref: 58 Objective: 3-17 13 Copyright © 2018 Pearson Education, Inc.

48) You are approached by a college student who states that his college is working with a physician on a project that examines the types of violent trauma occurring within your jurisdiction. He then asks you if he may see copies of all patient care reports that involved patients who were either stabbed or shot. Who would be the best person to contact regarding this request? A) Your service's medical director B) Your service's privacy officer C) The physician overseeing the research project D) The state department of EMS Answer: B Diff: 2 Page Ref: 57 Objective: 3-16 49) You have been called to a local emergency department to transfer a 67-year-old male to another facility. At the hospital, the emergency physician informs you that the patient and his family have requested the transfer because his doctor is affiliated with the other hospital. At the patient's bedside, you see that the patient is receiving an antibiotic through an IV. Although your state does allow EMTs to perform nonemergent transports of patients receiving normal saline through an IV, it does not allow transport of patients receiving IV medications. When informed of this restriction, the physician states that the antibiotic will be done in 10 minutes and that he will give written permission to start the transport while the medication is infusing. Which EMT action would be most appropriate at this time? A) Transfer the patient once all the medication is infused B) Stop the IV infusion and transport the patient C) Follow the doctor's order as long as he writes and signs it D) Recognize a HIPAA violation and refuse the transport Answer: A Diff: 2 Page Ref: 47 Objective: 3-4 50) A paramedic was late in reregistering and now, according to the state EMS agency, must function as an EMT until all of the reregistration requirements are satisfied. As an EMT, she is on the scene of a motor vehicle accident and taking care of a critically injured teenager. While waiting for a paramedic to arrive, she starts an IV, but does not hang the IV fluid that will be used to help stabilize the patient. This is immediately done by a paramedic once he arrives. As such, which of these is true? A) The paramedic/EMT violated the scope of practice but cannot be held accountable since the patient was stabilized by her actions B) The paramedic/EMT did not violate the scope of practice since she is experienced with IV therapy C) The paramedic/EMT violated the scope of practice and can be held accountable even though the patient survived D) The paramedic/EMT did not violated the scope of practice since the patient was critically injured and required lifesaving care Answer: C Diff: 3 Page Ref: 47 Objective: 3-9 14 Copyright © 2018 Pearson Education, Inc.

51) A pregnant woman is choking inside a local restaurant. Who has the most compelling legal duty to act? A) An off-duty EMT who just finished eating and is headed to the bathroom B) EMTs from another county eating lunch after dropping off a patient at a local hospital C) An EMS supervisor who just came into the restaurant and is on a week's vacation D) A retired paramedic who just finished eating and is paying his bill Answer: B Diff: 3 Page Ref: 48 Objective: 3-3 52) You are an off-duty EMT and have come across a motor vehicle collision. A car has crashed head-on into a utility pole. There is moderate damage to the vehicle and the driver is walking about the scene. You stop and assess the patient. He has a large bruise on his arm and is complaining of abdominal pain where the seat belt contacted his body. Emergency Medical Responders (EMRs) are on scene, and a responding ambulance has a 2-minute ETA. At this point, you realize that: A) You cannot leave until another EMT or paramedic arrives to assume care B) You can leave the scene since the patient is stable and EMS is 2 minutes away C) You can only transfer care to a paramedic once he or she arrives on scene D) You can transfer care to EMRs since EMS has been notified and an ambulance is 2 minutes from the scene Answer: A Diff: 3 Page Ref: 48 Objective: 3-3 53) You have been asked to review a legal case in which a patient intentionally overdosed on heroin and then died after EMS responders arrived. Those responders did not have a bag-valve mask to ventilate the nonbreathing patient, so the patient was not ventilated until a second EMS unit arrived. The evidence indicates that the EMT and paramedics both signed their morning equipment check indicating that they had inspected the ambulance and had all the necessary equipment to do their job. In this case, you would realize: A) The EMT and paramedics can seek protection under the Good Samaritan law B) The EMT and paramedics will most likely be charged with contributory negligence C) The EMT and paramedics are not responsible because the patient intentionally overdosed D) The EMT and paramedics were negligent in caring for the patient Answer: D Diff: 3 Page Ref: 55 Objective: 3-14

15 Copyright © 2018 Pearson Education, Inc.

54) Involuntary consent would most likely apply to: A) A 23-year-old prisoner with a laceration to the right cheek and refusing care B) An alert and oriented 34-year-old male who is coughing up blood and refusing care C) A 3-year-old boy found wandering in the street with abrasions and cuts on his bare feet D) A confused elderly woman who is in the presence of the person with her acting power of attorney Answer: A Diff: 2 Page Ref: 62 Objective: 3-10 55) A plaintiff is suing an EMT in civil court for alleged improper care and resultant injury. In this situation, which is true? A) The government is the lead agency that is prosecuting the EMT B) The EMT is facing a very serious criminal charge C) If found guilty, the EMT will face jail time or probation D) If the suit is successful, the plaintiff will most likely be awarded money Answer: D Diff: 3 Page Ref: 55 Objective: 3-13 56) Regarding an EMT's duty to act, the relationship between an EMT and a patient starts: A) Once the EMT is dispatched to the call B) Once the EMT makes physical contact with the patient C) Once the patient consents to be treated D) Once the EMT begins to provide physical care Answer: B Diff: 3 Page Ref: 48 Objective: 3-3 57) Your legal right to function as an EMT is contingent upon: A) Following standing orders and protocols, as approved by medical direction B) Avoiding civil liability C) Completing an approved EMT program D) Gaining paid employment by a public EMS system Answer: A Diff: 2 Page Ref: 49 Objective: 3-7

16 Copyright © 2018 Pearson Education, Inc.

58) You have been called for a 96-year-old female with altered mental status. At the scene, you find that the patient is unresponsive, not breathing, and pulseless. The family provides you with a valid DNR order. What could you document as a presumptive sign of her death? A) Closed eyes B) Poor or weak reflexes C) Dilated pupils D) Absent blood pressure Answer: D Diff: 1 Page Ref: 59 Objective: 3-19 59) Your EMS station has been designated a Baby Safe-Haven. As such, you know that: A) A mother or father can drop a baby off and leave with no questions asked B) You are legally required to provide basic supplies (e.g., bottles and diapers) to the mother or father C) A mother or father can bring a baby to the station for basic care issues D) You are legally required to provide the mother or father with resources to assist in the care of the baby Answer: A Diff: 2 Page Ref: 61 Objective: 3-8 60) The EMT correctly understands a "Physician Orders for Life-Sustaining Treatment" document when he states: A) "A POLST must be signed by the court system and the patient and is valid for a period of no more than 5 years." B) "A POLST legally empowers another person to make health care decisions for the patient in the event he or she cannot do so." C) "A POLST is designed to allow a patient to choose the desired type of care prior to the need for resuscitation." D) "A POLST is the same as a 'Do Not Resuscitate' order, with the difference being that the physician does not have to sign it." Answer: C Diff: 2 Page Ref: 50-51 Objective: 3-1

17 Copyright © 2018 Pearson Education, Inc.

61) What is the primary difference between the EMT's scope of practice and the EMT's standard of care? A) The scope is what the EMT should do; the standard is what the EMT is legally bound to do B) The scope includes which actions the EMT can legally take, while the standard of care is what a EMT should do in an EMS system with similar training and protocols C) The scope of practice is decided by the medical director, while the standard of care is defined by state regulations D) There is no clinically relevant difference between the scope of practice and the standard of care, as both define what the EMT should do in the exact same situation Answer: B Diff: 3 Page Ref: 47 Objective: 3-1 62) A state's Good Samaritan law would NOT protect the health care provider(s) in which of these situations? A) When an EMT is working for a volunteer EMS system and performs an act of gross negligence B) When an EMT is off duty, but stops to assist at a motor vehicle collision and functions in good faith C) When the EMT stops to help a collapsed person at a park while off duty by providing CPR, but the patient still dies D) When a nurse or physician who is off duty stops to assist at an EMS call where paid EMTs are present, but the patient still dies Answer: A Diff: 2 Page Ref: 48 Objective: 3-5 63) In any situation in which a question arises regarding the scope or direction of care the EMT should provide to a patient, the EMT should: A) Notify the on-duty EMS supervisor B) Provide no treatment so as to "not cause harm," and transport the patient C) Contact medical direction at the receiving facility to which the patient is being transported D) Summon an ALS intercept and allow the paramedics to make the care decisions Answer: C Diff: 2 Page Ref: 49 Objective: 3-7 64) What is the primary difference between ethics and morals? A) Ethics are what the EMT should do, whereas morals are what the EMT will do B) Ethics is a branch of philosophy that studies morality, whereas morals are concepts of "right and wrong" C) Morals are what the EMT should do, whereas ethics are what the EMT will do D) Morals define the expectations for professional occupations, whereas ethics are regulations written into state law defining what those expectations are Answer: B Diff: 2 Page Ref: 49 Objective: 3-8 18 Copyright © 2018 Pearson Education, Inc.

65) If an individual is bringing a lawsuit against your EMS system for perceived improper care, which type of liability action will it probably be? A) Civil B) State C) Criminal D) Federal Answer: A Diff: 2 Page Ref: 55 Objective: 3-13 66) You have extricated a male patient who was entrapped in a rollover car wreck. During your immediate on-scene management, you removed the patient's clothing and started CPR. During this time, a police officer retrieved the driver's license from the patient's pants and advises you that he is an organ donor. Given this information, which statement is true? A) The patient should be treated in the same way en route to the hospital as a nondonor B) The patient cannot be an organ donor now since he arrested prior to reaching the hospital C) The EMTs should initiate cooling measures to increase the longevity of the patient's organs D) The patient should be hyperventilated and hyperoxygenated along with receiving CPR to increase the longevity of his organs Answer: A Diff: 1 Page Ref: 58 Objective: 3-18 67) You have responded to a "man down" call one winter morning. Upon arrival, you find an elderly male patient lying parallel to the sidewalk, in a bank of shoveled snow. It appears as if the man was walking and either tripped or fell into the snowbank. The patient is in cardiac arrest. In this scenario, which finding would NOT be considered a presumptive sign of death? A) Closed eyes B) No pupillary response C) Low body temperature D) Absent reflexes Answer: C Diff: 1 Page Ref: 59 Objective: 3-19

19 Copyright © 2018 Pearson Education, Inc.

Prehospital Emergency Care, 11e (Mistovich et al.) Chapter 4 Documentation 1) What would be considered an objective patient assessment finding? A) Complaint of nausea B) Blood pressure 114/68 mmHg C) Patient rates pain as 10 out of 10 D) Complaint of headache for past 12 hours Answer: B Diff: 2 Page Ref: 70 Objective: 4-5 2) While moving a 67-year-old male who complains of dizziness from his residence on the stretcher, you stumble backward and knock a vase from a table, causing it to break. The patient and family are very upset with the damage. In the process, you hurt your ankle and are having a hard time walking. When completing the prehospital care report (PCR), what should be included? A) The fact that property damage occurred B) A factual account of your ankle injury C) The patient's insurance or billing information D) An objective statement about the family's displeasure Answer: C Diff: 2 Page Ref: 69-70 Objective: 4-4 3) Which item is included in the U.S. Department of Transportation minimum data set? A) Insurance and billing information B) Vital signs and skin condition C) Address and type of the call D) Patient's occupation Answer: B Diff: 1 Page Ref: 70 Objective: 4-4 4) A nauseated patient with fever and abdominal pain states that he has not vomited. Which description best represents how that fact should be documented? A) As a treatment finding B) As a subjective finding C) As a pertinent negative D) This fact would not be documented. Answer: C Diff: 2 Page Ref: 71 Objective: 4-1

1 Copyright © 2018 Pearson Education, Inc.

5) A patient tells you that he has been feeling "very weak" for the past three days. Using the SOAP charting format, this information would be included under which heading? A) S: Subjective B) O: Objective C) A: Assessment D) P: Plan Answer: A Diff: 2 Page Ref: 78 Objective: 4-10 6) After oxygen therapy, the patient's SpO2 improves from 90% to 99%. Using the CHEATED method of documentation, this information would be placed in which category? A) C: Chief complaint B) H: History C) E: Evaluation D) D: Disposition Answer: C Diff: 2 Page Ref: 79 Objective: 4-10 7) Which item would the EMT place in the administrative information section of the prehospital care report? A) BLS Unit 51-20; Incident # 67-8971-90 B) Patient: Henry, Steven M C) "Patient found supine on the porch." D) Blue State Insurance #425-22892; GRP# 456298 Answer: A Diff: 2 Page Ref: 70 Objective: 4-6 8) You have placed a 67-year-old female patient on low-concentration oxygen with a nasal cannula. Under which section of the prehospital care report (PCR) would you document this information? A) Administrative B) Patient data C) Patient narrative D) Treatment Answer: D Diff: 2 Page Ref: 71 Objective: 4-5

2 Copyright © 2018 Pearson Education, Inc.

9) When asked, an alert and oriented 44-year-old male tells you that he called 911 because "my chest is hurting." The man is also sweating and feels as if he is going to vomit. He has a history of high blood pressure and states that this pain "feels just like my heart attack two years ago." His pulse is 88 beats per minute, respirations are 18 breaths per minute, and blood pressure is 156/92 mmHg. On room air, he has an oxygen saturation level of 95 percent. Based on this information, how would you document his chief complaint on the patient care report? A) "My chest is hurting." B) Possible heart attack C) Myocardial infarction D) Chest pain with an elevated BP Answer: A Diff: 2 Page Ref: 70 Objective: 4-6 10) An intoxicated patient will not leave the oxygen mask on. What would be the most appropriate way to document this behavior on the prehospital care report? A) The patient is intoxicated and will not cooperate with oxygen therapy B) The patient continually removes the oxygen mask despite continued reapplication C) The patient will not cooperate with care provided by EMS D) The patient appears to be in a drunk-like state and will not cooperate with care provided by EMS Answer: B Diff: 3 Page Ref: 71 Objective: 4-9 11) A patient states that he has had a headache located in his forehead for three days. The EMT should recognize and document this piece of information as a(n): A) Subjective finding B) Pertinent negative C) Objective symptom D) Provoking factor Answer: A Diff: 2 Page Ref: 71 Objective: 4-6 12) When completing a paper-based prehospital care report, you accidentally write that a laceration was on the left side of a patient's face when it was actually on the right side of the face. Correcting this mistake would include which step? A) Color over the term "left" with black ink and write the word "right" next to it B) Start the entire prehospital care report over from the beginning C) Carefully use White-Out to cover the term "left" but nothing else in the narrative D) Draw a single line through the term "left" and write the word "right" next to it Answer: D Diff: 2 Page Ref: 77 Objective: 4-3

3 Copyright © 2018 Pearson Education, Inc.

13) Two days after a call, you realize that you forgot to document that you checked a patient's blood glucose prior to him refusing transport and signing the refusal form. At that time, you did contact medical direction and provide this information to the doctor, prior to him authorizing the patient to refuse. What is now your best course of action? A) Complete a new refusal form and return to the patient's residence to have him sign this form B) Add an addendum to the report with the correct information, the current date, and the EMT's initials C) Report the error to the State Department of Emergency Medical Services D) Disregard the mistake since the patient was not transported to the hospital and medical direction was aware of the blood glucose reading Answer: B Diff: 2 Page Ref: 77 Objective: 4-9 14) Which patient who refuses treatment should still be transported to the hospital? A) 16-year-old female who lives in her parents' house and has a child B) An alert and oriented 57-year-old male who is having chest pain and is diaphoretic C) A homeless man in his 50s who is dirty and disheveled and has a body-wide rash D) A 29-year-old female who attempted to cut her wrists earlier during a failed suicide attempt, but now changes her story and states it was an accident Answer: D Diff: 3 Page Ref: 75-76 Objective: 4-9 15) An alert and oriented male patient with chest discomfort refuses treatment and transport to the hospital. He is angry with his family for calling 911 and refuses to sign the refusal of care form despite several requests. Which of these is your next best action? A) Contact the police to place the patient into protective custody B) Have a family member sign as a witness to the father's refusal C) Call dispatch on a recorded telephone line and notify the dispatcher of the situation prior to departing the scene D) Carefully restrain the patient and place him on the stretcher for transport Answer: B Diff: 2 Page Ref: 75-76 Objective: 4-9 16) After a patient, who is short of breath, signs a refusal of service form, which statement would be appropriate prior to leaving the residence? A) "We will leave this oxygen for you; call us when you feel better." B) "Try taking an aspirin and get a good night's rest; you will probably feel better." C) "Call our dispatch in the morning to let us know how you made out." D) "If you change your mind at any time and want to be transported to the hospital, call 911 again." Answer: D Diff: 2 Page Ref: 75-76 Objective: 4-9 4 Copyright © 2018 Pearson Education, Inc.

17) When getting a refusal from a patient who does not want the services of EMS providers, it is critical for the EMT to complete which step? A) Make sure that the patient is younger than 65 years B) Have the patient sign and date the refusal form C) Make sure that the hospital is notified D) Make sure the patient has another way to get to the hospital Answer: B Diff: 2 Page Ref: 76 Objective: 4-9 18) Which statement shows an accurate understanding of the legal aspects of the prehospital care report (PCR)? A) "The PCR is considered a legal document only when it describes a crime or act of violence." B) "A PCR can be used in a lawsuit only if that lawsuit is filed within six months." C) "The PCR may be subpoenaed even if the lawsuit centers on alleged negligence that occurred in the emergency department." D) "A copy of the PCR should be forwarded to the police anytime law enforcement is involved in the call." Answer: C Diff: 3 Page Ref: 75 Objective: 4-8 19) You have transferred care of a 21-year-old woman who overdosed on an unknown drug. For the patient, which best describes who is permitted access to the prehospital care report (PCR)? A) The emergency physician but not the nurse B) The patient's mother but not brothers or sisters C) An on-duty police officer who is a friend of the patient D) The health care provider assuming care of the patient Answer: D Diff: 2 Page Ref: 75 Objective: 4-8

5 Copyright © 2018 Pearson Education, Inc.

20) You have transported a confused 46-year-old male who overdosed on an unknown drug to a busy emergency department. After giving an oral report to the ED nurse, your partner informs you that he wants to hurry back to the station so that he can watch the end of the football game; therefore, he is going to complete the patient care report (PCR) at a later time. How should you respond? A) "Make sure that you give the nurse the telephone number to the station so she can call if there are any questions about the patient or our care." B) "If you do not complete and leave the PCR, the ED staff may not otherwise have access to information contained in the PCR." C) "You really need to complete it before we leave. The hospital will not be able to get the patient's insurance information without the PCR." D) "If you do not leave the PCR, the emergency department staff will not know how to treat the patient." Answer: B Diff: 2 Page Ref: 66 Objective: 4-8 21) Which statement regarding a PCR is accurate? A) "The PCR is a public record that becomes part of the patient's permanent medical record." B) "Aside from providing a record of the care given, the PCR also may be used for education and research." C) "National standards allow the EMT to complete the PCR up to three days after the call as long as a verbal report was given to the physician." D) "The EMT should document only the direct care he or she provided, but not the care by EMRs since their report will reflect this care." Answer: B Diff: 2 Page Ref: 66 Objective: 4-2 22) The medical director for your service has put you in charge of designing a new prehospital care report form. He states that it must contain the "minimum data set" as set forth by the U.S. Department of Transportation. In completing this task, you realize that incorporating these data into your report form will: A) Increase EMS funding received from the federal and state governments B) Allow easier comparison of specific EMS data between various types of emergency systems C) Work toward creating a single report form for all EMS systems that handle emergency calls D) Permit easier tracking of motor-vehicle collisions so that hazardous roads and intersections can be identified Answer: B Diff: 1 Page Ref: 69-70 Objective: 4-4

6 Copyright © 2018 Pearson Education, Inc.

23) Which statement describes an advantage of computerized documentation? A) A computerized reporting system is less expensive initially when compared to a system using handwritten PCRs B) A computer system does not need special maintenance; a system using handwritten run reports does C) Large amounts of data can be stored and retrieved much more easily than with handwritten PCRs D) There is much less documentation involved with electronic systems than with handwritten reports Answer: C Diff: 1 Page Ref: 69 Objective: 4-3 24) Your partner states that he is the "world's worst speller" and has great difficulty using medical terms. How would you respond to this statement? A) "Ask the emergency physician or nurse how to spell the words of which you are unsure." B) "Use everyday language if you are unsure of how to apply or spell a medical term." C) "Consider abbreviating medical terms that you are unsure how to spell." D) "Do not document information that requires medical terms you are unsure of." Answer: B Diff: 2 Page Ref: 75 Objective: 4-7 25) You are reviewing a prehospital care report (PCR) and note the following entry: "Pt. took PCN TID PO for 5 days." You would interpret this as the patient took: A) Penicillin injections for five days in a row, three injections a day B) Penicillin was taken orally three times a day for five days straight C) Three penicillin pills were taken every other day for five days D) Antibiotic injections three times a day for five days Answer: B Diff: 2 Page Ref: Table 4-1 Abbreviations Objective: 4-7 26) What would be the best reason for a policy that requires all crew members to synchronize their watches with dispatch at the beginning of each shift? A) Ensures that all EMS staff are on time and ready for the shift B) Allows the supervisor to make sure that all EMTs have a watch C) Provides the best opportunity for the EMT to end the shift on time D) Promotes accurate system-wide time record keeping throughout the shift Answer: D Diff: 2 Page Ref: 70 Objective: 4-4

7 Copyright © 2018 Pearson Education, Inc.

27) A prehospital care report reads: "GSW to LLQ." Based on this, you should recognize that the patient sustained a(n): A) Puncture to the left chest B) Injury to the left torso C) Bullet injury to the left chest D) Gunshot to the lower left abdomen Answer: D Diff: 1 Page Ref: Table 4-2 Abbreviations Objective: 4-7 28) A patient care report reads: "c/o fall with (R) hip pain; FROM to (R) low. ext." Based on this, you should recognize that the patient's: A) Right hip appears broken B) Right hip is dislocated, but not fractured C) Right leg can be moved normally D) Right leg is not broken, but cannot be moved freely Answer: C Diff: 1 Page Ref: Table 4-2 Abbreviations Objective: 4-7 29) A patient care report reads: "PMH includes ESRF and (+) DNR; (+) ASA pta of EMS." Regarding this description, which interpretation is most accurate? A) The patient is alert and oriented B) The patient has kidney disease C) The patient desires resuscitation D) EMS administered aspirin to the patient Answer: B Diff: 1 Page Ref: Table 4-2 Abbreviations Objective: 4-7 30) A patient care report reads: "c/o chest pain with associated DOE; PMH of IDDM." Regarding this description, which interpretation is most accurate? A) The patient has diabetes B) The patient is breathing easily C) The patient takes no medications D) The patient took insulin Answer: A Diff: 1 Page Ref: Table 4-2 Abbreviations Objective: 4-7

8 Copyright © 2018 Pearson Education, Inc.

31) Your protocol reads: "O2 via NC prn if symptomatic for CP / SOB and SpO2 < 94%." To follow this order, you should do perform which action? A) Call medical direction to administer oxygen if the pulse oximeter reading is greater than 94% B) Consider oxygen for a patient complaining of chest pain if the SpO2 is 92% C) Administer oxygen via face mask if the patient denies a complaint and has a pulse oximeter reading of 98% D) Administer oxygen through a nasal cannula at 3 liters per minute if the pulse oximeter reads 94% Answer: B Diff: 2 Page Ref: Table 4-2 Abbreviations Objective: 4-7 32) Immediately after giving a prehospital care report to the nurse in the emergency department, dispatch informs you that there are no more ambulances available and you must immediately leave the hospital to cover another portion of the county. Since your service uses a computerized documentation system and there is no time to complete your report, your best course of action should be to: A) Give a verbal report to the ED physician B) Repeat your assessment findings and treatment to the nurse C) Complete an abbreviated transfer-of-care report D) Leave and return to complete the report as soon as possible Answer: C Diff: 2 Page Ref: 77 Objective: 4-2 33) You have been called for a 2-year-old female with an arm injury. The child presents with deformity and bruising to her left forearm. When asked, the mother states that the child fell from the bed, but the father states that his daughter fell down the stairs. Given the inconsistent stories, you are suspicious of child abuse. Which narrative would be most appropriate when documenting this situation? A) "Patient fell off of bed injuring left arm–per mother; patient fell down stairs injuring left arm– per father." B) "Mother and father cannot agree on how their daughter got the bruise and deformity to the arm." C) "Patient has injuries consistent with child abuse; the mother and father cannot determine how the patient was injured." D) "Patient appears to be abused; the police will be called and the physician in the ED will be notified." Answer: A Diff: 2 Page Ref: 78 Objective: 4-9

9 Copyright © 2018 Pearson Education, Inc.

34) You have been called to the house of a patient with altered mental status. You encounter a 41-year-old male who exhibits slurred speech, an unsteady gait, and an odor resembling that of alcohol on his breath. His wife states that he is an alcoholic and needs help to get better. When documenting this information, which statement would be best? A) Alcoholic history disclosed to EMS B) Patient with history of alcoholism C) Family member states patient ETOH D) Per wife–patient has alcoholic history Answer: D Diff: 2 Page Ref: 78 Objective: 4-9 35) Just before the end of their shift, EMTs transported a young male who bystanders state "passed out" and then had a seizure. Now, an hour later, the patient has been stabilized and the physician asks the nurse if she knows what the seizure looked like. To easily answer this question, she would: A) Contact and question the bystanders B) Call the EMTs at home C) Question the patient D) Check the prehospital care report Answer: D Diff: 2 Page Ref: 66 Objective: 4-5 36) You are involved in a lawsuit over a stabbing that occurred six months ago. Since you work in a busy EMS system and some time has passed, your recollection of the incident in spotty. In this situation, your best means of remembering what occurred would be to: A) Review the prehospital care report B) Meet and discuss the incident with other witnesses C) Confer with your partner at the time D) Obtain and review the police report Answer: A Diff: 2 Page Ref: 66 Objective: 4-2 37) A patient states that he has suffered from chronic neck pain ever since an accident. Further, he states that he is filing a lawsuit against the EMTs since they failed to put a cervical collar on him at the time; in his eyes, this is the reason he has daily neck and back pain. The EMT can remember the incident well and remembers putting a collar on the patient. As such, his best defense would be: A) Presentation of the emergency department chart in court B) Testimony from the supervisor stating that the EMTs always place a cervical collar C) Written documentation of collar placement on the PCR D) Presentation of the standard of care showing that a cervical collar is indicated Answer: C Diff: 2 Page Ref: 66 Objective: 4-2 10 Copyright © 2018 Pearson Education, Inc.

38) The medical director states that it seems as if the number of patients suffering from shortness of breath with a history of congestive heart failure (CHF) has increased. She adds that she is thinking about modifying the protocols, but first must know the number of patients seen with this condition over the past year. To best determine the number of patients treated for this condition, you would: A) Question the emergency department physicians and nurses B) Review the previous prehospital care reports C) Have employees complete a form anytime they treat a patient with CHF D) Have all employees complete a survey regarding the increase Answer: B Diff: 2 Page Ref: 66 Objective: 4-2 39) You document the following on the prehospital care report: "c/o H/A with associated n/v; pt. denies existing CNS problems or history of the same; states positive history of AAA and ETOH abuse." Regarding this narrative, which interpretation would be most accurate? A) The patient has a headache B) The patient has a history of illegal drug use C) The patient is nauseated but not vomiting D) The patient has a history of migraines Answer: A Diff: 2 Page Ref: Table 4-2 Abbreviations Objective: 4-7 40) Consider the following narrative from a patient care report: "pt. transported 3 days ago for suspected STEMI; pt. currently denies CP and SOB; PMH of HTN and CAD; pt. currently in NAD." Which of these interpretations is most accurate? A) The patient is short of breath B) The patient suffers from hypotension C) The patient has heart problems D) The patient is in moderate distress Answer: C Diff: 3 Page Ref: Table 4-2 Abbreviations Objective: 4-7 41) Consider the following narrative from a patient care report: "pt. restrained passenger involved in 2 car MVC; c/o left lower leg pain rated 2/10; LOC A/O to person, place, time, and event; BBS clear; abd. Soft with tenderness LLQ; hx of NIDDM with am glucose level of 133 mg/dL." Which of these interpretations is most accurate? A) The patient's abdomen appears uninjured B) The patient is being tested for diabetes C) The patient is confused following the accident D) The patient's lungs sounds are clear and equal bilaterally Answer: D Diff: 3 Page Ref: Table 4-2 Abbreviations Objective: 4-7 11 Copyright © 2018 Pearson Education, Inc.

42) Last week, on a computer-generated report, you accidentally documented that a patient suffered from hypertension when, in fact, he did not. Unfortunately, the report has been locked by the computer and cannot be changed; however, it can be printed. Your first action would be to: A) Retype the entire report and include the change B) Print the report and draw a line through the error C) Contact the hospital and have the staff there change the information in the patient's medical record D) Notify the medical director so that he or she can fix the error Answer: B Diff: 3 Page Ref: 77 Objective: 4-9 43) A patient with asthma is using his inhaler TID and prn. You would recognize that: A) The patient is prescribed his inhaler for use only three times a day B) The patient is suffering asthmatic attacks three times a week C) The patient uses an inhaler at least three times a day D) The patient uses his inhaler only when the symptoms are bad Answer: C Diff: 3 Page Ref: Table 4-1 & 4-2 Abbreviations Objective: 4-7 44) A pertinent negative would be illustrated by: A) "Nausea without vomiting." B) "Hypertension and taking medication." C) "Dizziness for three days without notifying the doctor." D) "Short of breath with history of lung problems." Answer: A Diff: 2 Page Ref: 71 Objective: 4-1

12 Copyright © 2018 Pearson Education, Inc.

Prehospital Emergency Care, 11e (Mistovich et al.) Chapter 5 Communication 1) When using your portable radio, you must push the "press to talk" button and wait one second before speaking. This is essential to effective communication because your EMS system uses: A) Digital equipment B) Repeaters C) Portable transmitter/receiver D) Cellular telephone links Answer: B Diff: 1 Page Ref: 84 Objective: 5-2 2) You are returning from a call when you pass an apartment building with flames coming from several windows on the top story. You see several occupants trapped and hanging out of the windows, screaming for help. Which radio transmission is most appropriate when advising dispatch of the situation? A) "Central communications from Unit 118, how do you copy?" B) "Central communications be advised of a working structure fire on Third Street and Raven with occupants trapped." C) "Central communication, be advised there is a structure fire with people trapped." D) "Central communications I need the fire department and additional ambulances for a working structure fire on the north side of town." Answer: A Diff: 3 Page Ref: 88 Objective: 5-5 3) Which statement reflects the most appropriate way to alert a receiving hospital that you are en route with a patient? A) "Inbound with a 76-year-old male complaining of dizziness." B) "St. Margaret's, can you take a report on a person with chest pain?" C) "Dr. Jackson, this is EMT Mike Bennet, can you hear me?" D) "St. Elizabeth Hospital, this is BLS ambulance 104. How do you copy?" Answer: D Diff: 2 Page Ref: 88 Objective: 5-5 4) Why should the EMT provide the physician or nurse with an oral report when transferring care to the hospital emergency department (ED)? A) To summarize patient assessment and treatment information B) To provide an opportunity for the ED staff to critique the care provided C) To inform them of whether they need to see the patient immediately D) To provide input on additional medical care to be given in the ED Answer: A Diff: 2 Page Ref: 91 Objective: 5-7 1 Copyright © 2018 Pearson Education, Inc.

5) Which statement is most appropriate to start with when transferring a patient to the emergency department and giving the oral report? A) "This is Mr. Hedger. He has the state insurance plan." B) "Mr. Hedger has high blood pressure and appears to be having a heart attack." C) "Mr. Hedger called us today because he was having chest pain." D) "Please refer to the recording of my radio report for specific information about Mr. Benton and his chest pain." Answer: C Diff: 2 Page Ref: 91 Objective: 5-9 6) Which important piece of information has been omitted from the following report: "Salem Community Hospital, we are en route with a 61-year-old male complaining of generalized weakness. He states that he was cutting the grass when he suddenly became hot and weak, forcing him to sit down. He has a history of diabetes and prostate cancer, but takes no medications. Right now he is alert and oriented and has the following vital signs: pulse 96, respirations 18, and blood pressure 156/82 mmHg, and room-air pulse ox was 92%. We have placed him on 2 liters of oxygen by nasal cannula and are transporting in semi-Fowler position. He states that the weakness is starting to resolve. Do you have questions or orders?" A) Patient's family physician B) Location of the incident C) ETA to the hospital D) Medications the patient is taking Answer: C Diff: 2 Page Ref: 91 Objective: 5-7 7) Which statement is most appropriate to relay after establishing radio contact with the destination emergency department to which you are transporting a patient? A) "Inbound with a 52-year-old suffering from a myocardial infarction." B) "En route to your facility with a 73-year-old male complaining of dizziness." C) "Five minutes out with Mr. John Panzer, who called us today for high blood pressure." D) "En route with a drug addict who has overdosed on an unspecified amount of heroin." Answer: B Diff: 2 Page Ref: 91 Objective: 5-7 8) What is an effective way to improve communication with patients from most cultures? A) Have the patient look up at you B) Speak clearly and loudly C) Maintain eye contact D) Use medical terminology Answer: C Diff: 2 Page Ref: 96 Objective: 5-12

2 Copyright © 2018 Pearson Education, Inc.

9) You are treating an 82-year-old female who is having trouble hearing some of your questions. Which technique would be most helpful in obtaining her medical history? A) Enlist the help of a neighbor B) Speak with a loud and higher-pitched voice C) Defer further questions until en route to the hospital D) Speak clearly and slowly Answer: D Diff: 2 Page Ref: 94 Objective: 5-12 10) You have been called for a 23-year-old male who is completely deaf. To best communicate with this patient, you should: A) Stand in front of the patient and speak loudly B) Write questions on a note pad C) Use gestures when asking questions D) Have the emergency department get the needed information Answer: B Diff: 2 Page Ref: 99 Objective: 5-13 11) You are treating a 2-year-old girl who fell off her bike. She is scared, continues to cry, and will not answer any of your questions. What would be a good strategy to help find out where the girl hurts? A) Have the girl's mother ask your questions for you B) Tell the patient she will be okay if she tells you what hurts C) Promise the girl a reward like ice cream if she answers the questions D) Use an adult-like stern voice and tell the girl she needs to answer the questions Answer: A Diff: 2 Page Ref: 99 Objective: 5-16 12) What role does the Federal Communications Commission (FCC) play in EMS? A) The FCC regulates only the radio communication systems that are used by police, fire, and EMS B) The FCC establishes and enforces regulations related to the use of radio operations C) The FCC determines the essential information that EMTs must relay to a receiving hospital about patient care D) The FCC approves maintenance and repair contracts for communication equipment used by EMS systems Answer: B Diff: 1 Page Ref: 87 Objective: 5-3

3 Copyright © 2018 Pearson Education, Inc.

13) Your partner accidentally uses a profane word when talking to dispatch. Which agency has the power to fine him and the ambulance service? A) County Board of Commissioners B) Department of Homeland Security C) Local police department's division of communications D) Federal Communications Commission Answer: D Diff: 1 Page Ref: 87 Objective: 5-3 14) A 42-year-old female complains of shortness of breath. Which question would best assist you in getting additional information about the patient's chief complaint? A) "You appear to be short of breath. Are you also having other problems?" B) "Why did you wait 2 hours to call for an ambulance?" C) "Did you get diaphoretic when the shortness of breath started?" D) "Have you ever had a myocardial infarction before?" Answer: A Diff: 3 Page Ref: 97 Objective: 5-15 15) How should the EMT respond when first making contact with an alert and oriented patient who complains of chest pain? A) "Did you call for an ambulance? I am here to help." B) "I am an EMT. Do you want to go to the hospital?" C) "I am here to help you. Do you have chest pain?" D) "My name is Charles Smith, and I am an EMT. Can I help you?" Answer: D Diff: 1 Page Ref: 94 Objective: 5-12 16) Medical direction has advised you to place a patient who is obese and extremely short of breath in a supine position. Knowing that doing so may cause the patient to become even more short of breath, your best action would be to: A) Explain your concern to the physician and ask for clarification B) Place the patient in the supine position and expedite transport C) Contact a different hospital and transport the patient there D) Ask your partner what she would do Answer: A Diff: 2 Page Ref: 90 Objective: 5-8

4 Copyright © 2018 Pearson Education, Inc.

17) You pull up to the scene of a one-car motor vehicle collision and find an off-duty emergency department physician from a local hospital treating the patient. What is the best way to initially communicate with the doctor? A) "Thank you, doctor, you can go now; we will take over." B) "Hello, doctor. We are EMTs. How can we help?" C) "We will be in the ambulance if you need us." D) "The law requires you to let us handle the situation." Answer: B Diff: 2 Page Ref: 97 Objective: 5-15 18) While you are returning from lunch, a frantic woman flags you down and states that she just found a young child on the roadside who appears to have been hit by a car. She is not sure if the child is breathing. You should immediately: A) Grab equipment and get to the child's location B) Advise dispatch that you have been flagged down for a possible emergency, and identify your location C) Call for paramedic assistance and await that unit's arrival D) Inform the woman that she will need to call 911 first Answer: B Diff: 2 Page Ref: 89 Objective: 5-6 19) An effective and reliable communication system is essential to the EMT and the EMS system because: A) It has been shown to decrease the number of lawsuits against EMS providers B) EMTs are better able to assess patients C) It allows hospitals to better prepare for the arrival of a patient D) The safety of an EMT is guaranteed at a dangerous scene Answer: C Diff: 2 Page Ref: 91 Objective: 5-7 20) Why is using a standard format for relaying medical information to the emergency department important? A) A standard format reduces the opportunities for forgetting important medical information B) A standard format assists the EMT in maintaining his or her assessment skills C) Dispatch is better able to monitor and constructively critique the radio traffic to hospitals D) The EMS agency can increase its reimbursement from health care insurance companies Answer: A Diff: 2 Page Ref: 91 Objective: 5-7

5 Copyright © 2018 Pearson Education, Inc.

21) Which of following is a disadvantage in using a cell (or mobile) phones as an emergency backup communication system? A) Cell (mobile) phones do not offer high-quality sound B) There is less privacy when using cell phones, as compared to radios C) Cell (mobile) phone use is not monitored by the FCC D) Cell (mobile) phones may not be effective in disaster situations Answer: D Diff: 2 Page Ref: 86 Objective: 5-4 22) When providing a lengthy radio report, the most important reason to pause every 30 seconds is so that: A) Other people can use the radio for emergency transmissions that may arise B) The repeater can process and transmit all that you are saying C) The person receiving the message has time to think about what you are saying D) The person receiving the message can write down what you are saying Answer: A Diff: 1 Page Ref: 88 Objective: 5-5 23) What is the primary reason why emergency services communications (phone or radio) are recorded? A) The state EMS agency will use the recording to determine if care was appropriate B) The federal government requires that all communications be recorded C) Communications can become part of a legal record if need be D) The Federal Communications Commission reviews all recordings for communication appropriateness Answer: C Diff: 3 Page Ref: 89 Objective: 5-5 24) A patient who was complaining of a headache is being transported to the hospital. After you provide a report to the emergency department, the patient begins to unexpectedly seize. The seizure lasts for 1-2 minutes and then stops, but now you notice the patient's eyes have become unequal in size. You are still 5 minutes from the hospital. After providing care for the seizure, your next action should be to: A) Continue care and transport to the hospital B) Call the hospital and inform its staff of the seizure C) Contact family via cell phone and determine whether the patient has a history of seizures D) Document the seizure on the prehospital care report Answer: B Diff: 1 Page Ref: 91 Objective: 5-7

6 Copyright © 2018 Pearson Education, Inc.

25) You have arrived at the scene of a medical emergency. As you enter the room where the patient is located, you note that he is sitting up and talking without difficulty. Emergency Medical Responders (EMRs) are on scene and providing basic care for the patient. What should you do first? A) Place the patient on your stretcher and transfer him to the ambulance B) Ask the patient's family if he has any past medical problems C) Get the patient's pulse and respiratory rate, along with his blood pressure D) Get a report from the EMRs Answer: D Diff: 1 Page Ref: 91 Objective: 5-9 26) Haptics is the study of: A) Maintaining eye contact B) Nonverbal response C) Touching D) Asking questions Answer: C Diff: 1 Page Ref: 96 Objective: 5-14 27) Your ambulance was dispatched for a patient complaining of shortness of breath at 1512. You arrived on scene at 1523 and left at 1538. You arrived at the hospital with the patient at 1557 and were back in service at 1622. Given this information, which is true? A) Your scene time was 15 minutes B) You arrived on the scene at 5:23 P.M. C) The call took place in the morning D) The time spent at the hospital was 16 minutes and 22 seconds Answer: A Diff: 1 Page Ref: 92 Objective: 5-11 28) Throughout a radio transmission to the dispatch center, the EMT periodically uses the term "break." You should recognize this to mean she is: A) Completing her transmission B) Purposely pausing her transmission C) Awaiting a response from the dispatcher D) Taking time to get her thoughts together Answer: B Diff: 1 Page Ref: 92 Objective: 5-7

7 Copyright © 2018 Pearson Education, Inc.

29) A patient denies difficulty breathing, but displays signs of respiratory distress. You state, "I know that you said you are breathing fine, but you cannot speak more than two to three words at a time without gasping for air." Your response is an example of: A) Confrontation B) Empathy C) Contradiction D) Explanation Answer: A Diff: 2 Page Ref: 94 Objective: 5-13 30) When interviewing a patient with a medical emergency, the EMT is using the technique of summary when she says: A) "With one finger, can you point to where it hurts?" B) "Did you also have diarrhea with the nausea and vomiting?" C) "So the nausea and vomiting started two days ago, correct?" D) "I know how you feel. I had the same thing last week." Answer: C Diff: 2 Page Ref: 93 Objective: 5-13 31) You have arrived by the side of an 82-year-old male complaining of generalized weakness. After you introduce yourself, the patient states, "My name is Edward Burns, but everyone calls me Eddy." What would be your most appropriate response back to him? A) "Okay, Eddy, let's see what is going on with you." B) "Okay, Edward. We need to know which hospital you want to go to." C) "Okay, Mr. Burns. Is it okay if we get your blood pressure?" D) "Do you want us to call you Eddy as well?" Answer: D Diff: 2 Page Ref: 94 Objective: 5-13 32) An advantage to asking the patient close-ended questions is that this practice offers a(n): A) Opportunity to get a detailed response from the patient B) Ability to get medical information very quickly C) Chance to get a significant amount of information D) Opportunity for the patient to elaborate about the medical complaint Answer: B Diff: 1 Page Ref: 97 Objective: 5-1

8 Copyright © 2018 Pearson Education, Inc.

33) Which of these is an open-ended question? A) "Can you describe the pain to me?" B) "What time did the pain begin?" C) "Did you get sweaty when the pain started?" D) "Who is your primary care physician?" Answer: A Diff: 1 Page Ref: 97 Objective: 5-1 34) A 31-year-old female with a history of depression took some pills in a suicide attempt. Her husband is visibly upset and her three young children are crying as you place her into the ambulance. The patient is conscious and stable as you transport her to the hospital. During the transport, she tearfully states she did not mean to take the pills and asks if she is going to die. In regard to therapeutic communication, what would be your best response? A) "You are going to be fine, don't worry." B) "You are stable right now and we are transporting you for continued care so you can get better." C) "If you do not want to die, why did you take the pills?" D) "I think that you have a loving family and you need to be more considerate of them." Answer: B Diff: 2 Page Ref: 97 Objective: 5-13 35) Over the next three days, your ambulance service will be getting new and improved mobile radios. As an EMT with knowledge of communication equipment, you realize that: A) The new radios will allow combined radio and cell phone use B) New portable radios will be issued to the field crews C) The new radios will rebroadcast transmissions at a higher power D) New radios will be installed in all of the ambulances Answer: D Diff: 3 Page Ref: 84 Objective: 5-2 36) A patient does not speak English and no family members are available to translate. Which other options are available to you to facilitate communication with the patient? A) Write questions on a pad B) Check with dispatch or use a toll-free interpreter line C) Proceed with your primary exam D) Treat the patient as if he is unconscious Answer: B Diff: 3 Page Ref: 99 Objective: 5-17

9 Copyright © 2018 Pearson Education, Inc.

37) As you drive the ambulance with lights and sirens activated to the hospital, the paramedic asks you to notify the receiving hospital since he is busy caring for the patient, who is critical. The paramedic tells you to specifically let the emergency department know that the patient's blood sugar is 15 mg/dL (very low). There is no history of diabetes. How should you state this information to the hospital? A) "Be advised that the blood sugar is 15–that is, one-five mg/dL." B) "The blood sugar is 15. Be advised that the paramedic, not I, checked the blood sugar." C) "The patient is not a diabetic but has a low blood sugar." D) "We have a blood sugar that is 15. I think that the patient is very critical." Answer: A Diff: 3 Page Ref: 91 Objective: 5-7 38) Over the radio, medical direction has given the following order for an unstable patient you suspect is having a heart attack: "Have the patient chew four baby aspirin, each one at 81 mg, and transport as soon as possible." You would show reception of this order by stating: A) "Copy. We will administer the aspirin as ordered and begin immediate transport." B) "Copy. We will start transport and administer 324 mg of aspirin en route." C) "Copy. We will have the patient chew four baby aspirin, each one at 81 mg, and transport ASAP." D) "Copy, Dr. Nash. We will follow the orders as received and transport to your facility." Answer: C Diff: 3 Page Ref: 90 Objective: 5-8 39) When relying on a cell (mobile) phone for medical communications, the EMT realizes that: A) He can relay his diagnosis of the patient's problem to the emergency department B) He should use the same format of communication as used for radio transmissions C) He is free to relay personal and confidential information not allowed over the radio D) He can talk more casually and not in as structured a manner as when using the radio Answer: B Diff: 3 Page Ref: 89 Objective: 5-5 40) The wife of a 57-year-old male called 911 because her husband had chest pain. Which piece of information would be included in your SBAR report to the hospital during transport? A) "The call originated at the address to which we responded." B) "The police were called after a neighbor complained about where the ambulance was parked." C) "I am thinking that the patient would benefit from 324 mg of aspirin." D) "His wife also has a history of high blood pressure, so it appears to run in the family." Answer: C Diff: 3 Page Ref: 90 Objective: 5-8

10 Copyright © 2018 Pearson Education, Inc.

41) Currently, there is a debate within your organization regarding the use of codes in your medical communications and radio traffic. As you are in favor of using plain English and doing away with codes, a strong argument to justify this position would be: A) Codes increase the amount of air time needed to relay information B) The code system can be made public, allowing others to hear confidential information C) The use of codes does not allow the EMT to practice his or her medical terminology D) Some medical information can be too complex for a simple code system Answer: D Diff: 3 Page Ref: 92 Objective: 5-10 42) Which phrase best illustrates the EMT using the summary technique of therapeutic communication with a patient experiencing depression? A) "From your story, it seems that you have been depressed for several weeks now." B) "Since you are depressed, we need to transport you to the hospital." C) "I know how you feel; my brother suffers from depression." D) "Since you are not in a normal state of mind, we have to transport you to the ED." Answer: A Diff: 2 Page Ref: 93 Objective: 5-13 43) Your portable radio was accidently dropped at the scene of a car wreck and has become soiled with dirt and mud. How should you clean the portable prior to putting it back into service? A) Wipe it down with a damp cloth B) Run it under warm water to rinse off the mud C) Clean it with multiple alcohol preps from the ambulance D) Return it to the station so it can soak in disinfectant solution, and obtain a replacement radio Answer: A Diff: 1 Page Ref: 87 Objective: 5-4 44) What should be an expected reason why a portable radio suddenly fails to operate normally while on the scene of an emergency? A) Cracked antennae B) Dead battery C) Clogged mic D) Repeater failure Answer: B Diff: 1 Page Ref: 87 Objective: 5-4

11 Copyright © 2018 Pearson Education, Inc.

45) During which situation is it typically NOT necessary for the EMT to contact dispatch first with a request or question? A) When the EMT needs to get medical approval to administer a medication B) If the patient is entrapped and a structural collapse rescue team is needed C) If the scene suddenly turns unsafe and police backup is required D) The patient is acutely deteriorating and ALS intercept is necessary Answer: A Diff: 2 Page Ref: 89 Objective: 5-6 46) While going through your probationary period as a new EMT with the community-based EMS system, you are learning all the numeric codes the department uses during radio traffic. What is NOT a benefit of using and relying on radio codes? A) Use of radio codes shortens the air time during verbal transmissions B) Use of radio codes allows transmission of sensitive information in a format likely not understood by the patient or public C) The complexity involved in remembering all the radio codes helps assure that everyone using the radio is a knowledgeable EMT D) The radio codes can provide clear and concise information Answer: C Diff: 1 Page Ref: 92 Objective: 5-10 47) While reviewing patient care reports as part of a quality assurance program, which trip time documented by an EMT would indicate that the EMT needs refresher education regarding proper military time utilization? A) "Call dispatched at 2233 hours" B) "Medication administered at 0814 hours" C) "Departed scene with patient at 1000 hours" D) "Intercepted with ALS at 0032 hours" Answer: D Diff: 2 Page Ref: 92 Objective: 5-11 48) As your ninth emergency call during a 12-hour shift, you are summoned to a dance club at 0215 hours for a "drunk person." While on scene, your partner stands with arms crossed, looking down at the floor, and shaking her head negatively as you try to gather the SAMPLE history. Her nonverbal behavior may be interpreted by the patient or bystanders as: A) Fear B) Anger C) Acceptance D) Sincerity Answer: B Diff: 2 Page Ref: 92 Objective: 5-14

12 Copyright © 2018 Pearson Education, Inc.

49) You are conducting a verbal interview with a patient who is not very forthcoming with information. If you fail to continue to try to motivate the patient to talk, what would be the most undesirable outcome? A) The patient will not pay the ambulance bill when it arrives B) The hospital emergency department will likely be mad at the EMS providers for failing to gather necessary information C) The family will assume you are not truly interested in helping the patient D) You may not gain valuable information that could impact your patient care decisions Answer: D Diff: 2 Page Ref: 98 Objective: 5-16 50) In which situation should the EMT honor the wishes of the patient's family rather than the patient's wishes regarding treatment and transport decisions? A) When the family, rather than the patient, is responsible for paying the ambulance and hospital bill B) When the patient is a child or is an adult who does not have the capacity to make his or her own decisions C) When the family's decision about treatment and transport coincides with the EMT's decision, even if the patient disagrees D) If the patient is an adopted family member from another culture who has not yet become accustomed to normal practices in the United States Answer: D Diff: 2 Page Ref: 98 Objective: 5-17

13 Copyright © 2018 Pearson Education, Inc.

Prehospital Emergency Care, 11e (Mistovich et al.) Chapter 6 Lifting and Moving Patients 1) Which statement accurately describes the concept of body mechanics? A) Body mechanics refers to an exercise program designed to increase strength and prevent injuries when lifting or moving patients B) Body mechanics describes the personal maintenance of a proper weight and posture so that on-the-job injuries are minimized C) Body mechanics refers to methods that promote using your body in the safest and most efficient way when moving objects or patients D) Body mechanics describes a process by which the EMT can gauge or estimate the type of injuries sustained in a traumatic incident Answer: C Diff: 2 Page Ref: 102 Objective: 6-1 2) When transferring a patient from a bed to a wheeled stretcher, an EMT uses proper body mechanics when he or she: A) Primarily uses the muscles of his back to move the patient B) Aligns his shoulder, hips, and feet when moving the patient C) Transfers the patient to the stretcher all at once, not in stages D) Keeps his arms locked and the patient away from his body during the transfer Answer: B Diff: 2 Page Ref: 102 Objective: 6-3 3) Kyphosis is best described as: A) An exaggerated forward curvature of the thoracic spine B) A pronounced lateral curvature of the thoracic spine C) The exaggerated curvature of the lumbar spine D) A distinct lateral curvature of the cervical spine Answer: A Diff: 1 Page Ref: 103 Objective: 6-1 4) You have been asked to present a continuing education session on body mechanics and posture. In your presentation, you would advise the participants that proper posture is achieved when your: A) Ears, shoulders, and feet are vertically aligned when sitting B) Abdomen and buttocks are positioned slightly to the posterior when standing C) Body weight is evenly distributed over the thoracic spine when sitting D) Ears, shoulders, and hips are vertically aligned when standing Answer: D Diff: 2 Page Ref: 103 Objective: 6-4

1 Copyright © 2018 Pearson Education, Inc.

5) To improve your physical fitness and decrease the risk of injury when lifting and moving patients, you should combine proper body mechanics with: A) Vitamin supplements B) Proper nutrition C) Protein drinks D) Caffeine substitutes Answer: B Diff: 1 Page Ref: 103 Objective: 6-4 6) An obese patient cannot ambulate and must be carried down several flights of stairs. Carrying the patient alone exceeds your physical capabilities; however, your partner is very strong and physically fit. Which statement, made by you, indicates the best means to safely lift and move the patient as well as minimize the likelihood of either of you suffering an injury? A) "When we move him, let's go slowly down the stairs." B) "As long as I lean backward when moving him, we will be okay." C) "When we go down the steps, you take the 'head' end and I'll carry the legs and go down the steps first." D) "We must call for assistance to get the patient from this location." Answer: D Diff: 2 Page Ref: 104 Objective: 6-5 7) In preparing the schedule, you have been instructed to pair EMTs in a way that decreases the chance of injury when lifting and moving patients. Given this directive, who would make the best partner for a tall and very strong EMT? A) A tall and very strong EMT B) A short and very strong EMT C) A short and weaker EMT D) A tall and weaker EMT Answer: A Diff: 3 Page Ref: 103 Objective: 6-5 8) A large patient has been placed on the stretcher, and you and your partner are preparing to raise the stretcher using the power lift. Which observation of your partner would cause you to pause the lift to correct his technique? A) Both feet are turned slightly outward B) His hands are approximately 10 inches apart C) Both feet are flat on the ground D) Both feet are positioned closely together Answer: D Diff: 2 Page Ref: 104-105 Objective: 6-8

2 Copyright © 2018 Pearson Education, Inc.

9) When lifting or moving any patient, the EMT must remember to primarily use the muscles of his or her: A) Legs B) Abdomen C) Back D) Arms Answer: A Diff: 1 Page Ref: 102 Objective: 6-7 10) When carrying equipment in the right hand, the EMT should: A) Lean to the left side for compensation B) Bend backward for counterbalance C) Tilt his body to the right side D) Keep his or her back straight as best possible Answer: D Diff: 1 Page Ref: 105 Objective: 6-7 11) You have been charged with writing a policy regarding reaching for equipment. In that policy, you indicate that the maximum distance an EMT should reach is no more than: A) One arm's length in front of her body B) 10 to 12 inches in front of her body C) 15 to 20 inches in front of her body D) 30 inches in front of her body Answer: C Diff: 2 Page Ref: 106 Objective: 6-7 12) When lifting the stretcher with a patient secured to it, the EMT must avoid which action? A) Maintaining a straight back B) Bending over at the waist C) Contracting the abdominal muscles D) Lifting with the thigh muscles Answer: B Diff: 1 Page Ref: 105 Objective: 6-6 13) Which set of instructions would you give to a new EMT who is preparing to use a squat lift to raise a patient secured to a stretcher? A) "Make sure to place your weaker leg slightly back." B) "Try not to use a power grip when performing the squat lift." C) "Support yourself with your stronger leg and push up with your weaker one." D) "Make sure that you push yourself up with your stronger leg." Answer: D Diff: 2 Page Ref: 105 Objective: 6-6 3 Copyright © 2018 Pearson Education, Inc.

14) When an EMT places his palm and all fingers in contact with the object being lifted, he is using the: A) Power lift B) Lock grip C) Power grip D) Squat grip Answer: C Diff: 1 Page Ref: 105 Objective: 6-7 15) A patient has fallen in his upstairs bathroom and is lying on the floor. Assessment indicates no life threats to the airway, breathing, or circulation; however, the patient does have pain, deformity, and bruising to his right hip. In this situation, the EMT should recognize the need for which type of move? A) Emergency B) Non-urgent C) Urgent D) Passive Answer: B Diff: 2 Page Ref: 109 Objective: 6-9 16) A small car and a tanker truck loaded with gasoline have collided. The car is lodged under the tanker, the car engine is smoking, and the threat of fire is significant. After being waved in by the fire department, assessment reveals the patient to be unresponsive with a patent airway, but breathing with difficulty at 8 times per minute. In caring for this patient, your immediate action should be to: A) Promptly open the airway and suction prior to moving the patient B) Rapidly extricate the patient and provide care away from the wreckage C) Quickly obtain vital signs and then extricate the patient from the car D) Immediately start positive pressure ventilation and place a vest immobilization device for extrication Answer: B Diff: 3 Page Ref: 108 Objective: 6-9 17) A patient must be removed immediately from a structure that is on the verge of collapse following an explosion. He is lying on the ground and dressed in shorts and a T-shirt. You are by yourself and have no other supplies or equipment. Which type of move would be most appropriate in this situation? A) The armpit-forearm drag B) The extremity lift C) The pants drag D) The blanket drag Answer: A Diff: 2 Page Ref: 107 Objective: 6-8 4 Copyright © 2018 Pearson Education, Inc.

18) A patient involved in a motor vehicle collision has critical injuries and must be removed from the car for care and transport to the hospital. Prior to removing the patient from the vehicle, what must be done? A) Application of a vest-type immobilization device B) Completion of the primary and secondary exams C) Application of a cervical collar D) Completion of the primary assessment and vital signs Answer: C Diff: 2 Page Ref: 108 Objective: 6-10 19) An EMT would perform an urgent move when: A) The patient exhibits no problems with the airway, breathing, or circulation B) The patient has a life-threatening injury C) The scene contains an immediate threat to the patient and EMT D) The patient is stable but has pain Answer: B Diff: 2 Page Ref: 108 Objective: 6-9 20) An elderly patient is being cared for at home by family members and lies in a hospital-type bed in the living room. After providing necessary care, you prepare to move her from the bed to the stretcher. There is no mechanism that suggests a head, neck, or spinal injury. Which method would be most appropriate in this situation? A) Draw sheet method B) Direct lift C) Extremity lift D) Direct carry method Answer: A Diff: 1 Page Ref: 110 Objective: 6-11 21) One advantage of using the wheeled stretcher is: A) It rolls smoothly over rough terrain B) It can be safely moved by one EMT with a patient on it C) It cannot become unbalanced on a slope or rough terrain D) It can be adjusted to different positions or heights Answer: D Diff: 1 Page Ref: Table 6-2 Objective: 6-11

5 Copyright © 2018 Pearson Education, Inc.

22) You and two other EMTs are preparing to move a patient down several flights of stairs using a stair chair. After assigning one rescuer to hold the chair at the head and another to hold the chair at the foot facing the patient, you should: A) Assist the EMT at the patient's head B) Direct movement from behind the EMT at the head C) Assist the EMT at the patient's feet D) "Spot" by standing behind the EMT at the feet Answer: D Diff: 3 Page Ref: 113 Objective: 6-11 23) Which observation demonstrates that the EMTs and EMRs are correctly using the stair chair? A) The wheels of the stair chair touch each and every step B) The stair chair slides down the stairs while tilted forward C) The patient is carried down the stairs feet first D) The stair chair is carried down the stairs with the chair tilted forward Answer: C Diff: 2 Page Ref: 113 Objective: 6-11 24) Your coworkers would like to purchase a scoop (orthopedic) stretcher with some grant money. Before the purchase can be authorized, you have been asked to inform the purchasing committee of how this piece of equipment will benefit the service as well as enhance patient care. Your response would be: A) "The scoop stretcher is good for moving patients from confined areas." B) "The scoop stretcher does not require straps to secure the patient." C) "The scoop stretcher is considered the best device for patients with a possible spinal injury." D) "The scoop stretcher is much more comfortable than a wheeled stretcher." Answer: A Diff: 2 Page Ref: 115 Objective: 6-11 25) You are caring for a young male patient who has overdosed on an unknown medication and has a decreased level of consciousness. When positioning the patient on the stretcher, which instructions will you give to the Emergency Medical Responders assisting you? A) "When you put him on the stretcher, elevate his head and chest." B) "He will need to be positioned flat on his back when on the stretcher." C) "Place him in a supine position and then carefully elevate his feet." D) "When you move him to the stretcher, put him on his left side." Answer: D Diff: 2 Page Ref: 116 Objective: 6-12

6 Copyright © 2018 Pearson Education, Inc.

26) Which position would be most appropriate for an alert and oriented patient complaining of shortness of breath? A) Prone B) Supine with the head elevated C) Supine with the feet elevated D) Left lateral recumbent position Answer: B Diff: 3 Page Ref: 116 Objective: 6-12 27) When you must push an object, it is important to: A) Push using muscles between your waist and your shoulders B) Push from an overhead position C) Keep your elbows straight and locked D) Push with your hands spread as far apart as you can Answer: A Diff: 2 Page Ref: 109 Objective: 6-3 28) When reaching forward to perform a logroll, the EMT should: A) Keep the back flexed B) Lean from the waist C) Use the shoulder muscles D) Reach no more than 30 inches Answer: C Diff: 2 Page Ref: 106 Objective: 6-3 29) After providing spinal motion restriction to an elderly female patient who fell in her bathroom, you prepare to carry her down a flight of stairs on a backboard. Which verbal instructions would be most appropriate to give to your partner? A) "Make sure to hold the board 6 inches from your body so that you get maximum stabilization." B) "It is best if we do not talk so that we can concentrate on safely moving her down the stairs." C) "I can take the head if you want the foot end, and then we will carry her down the stairs feet first." D) "Let's have the patient hold onto the stair railing to prevent the board from getting off balance." Answer: C Diff: 2 Page Ref: 117 Objective: 6-5

7 Copyright © 2018 Pearson Education, Inc.

30) A patient has developed chest pain while working in his basement wood shop. After EMTs have assessed and provided care to the patient, the chest pain remains, although his vital signs are normal. To get him to the ambulance, the EMTs must first go up a short flight of stairs from the basement to the first floor, where the front door is located. Which action would be the safest for both the patient and crew? A) The two EMTs grasp the sides of the portable stretcher and move the patient up the stairs B) The stronger of the EMTs takes the head of the portable stretcher, and the EMTs carry the patient up the stairs head first C) The patient carefully walks up the stairs under the watchful assistance of the EMTs D) The patient is carried up the stairs on the stair chair feet first, with the stronger EMT carrying the back of the device Answer: B Diff: 2 Page Ref: 118 Objective: 6-10 31) You and your partner are preparing to move a trauma patient down a flight of stairs using a portable stretcher. As you begin the move, a third EMT shows up and asks to help. Which instructions would be most appropriate to ensure a safe move for both the patient and crew? A) "Please stand behind the EMT at the foot end and count steps for her." B) "You can help by grasping the right side of the stretcher to keep it balanced." C) "Come up and help me with the head end, since most of the weight is here." D) "Stand behind us at the top of the stairs and let us know if you see any obstacles." Answer: A Diff: 2 Page Ref: 118 Objective: 6-5 32) You have been called to assist a pediatric physician, a pediatric nurse, and a respiratory therapist in the transfer of a 2-week-old baby who is very sick. The baby is to be transferred in an isolette to a large metropolitan teaching hospital in a nearby city, where specialized care can be provided. The isolette is secured to the stretcher, and the stretcher is loaded into the ambulance. The physician, nurse, and respiratory therapist will provide all patient care. Prior to departing the hospital, it is essential that you: A) Advise the family of the baby's critical condition B) Make sure the stretcher is secured in the ambulance C) Mix and prepare the medications needed for transport D) Disinfect the outside of the isolette Answer: B Diff: 1 Page Ref: 118 Objective: 6-14

8 Copyright © 2018 Pearson Education, Inc.

33) You are correctly using the blanket drag to move a patient when you: A) Place the patient in a blanket and drag him by grasping his forearms B) Secure the patient in a blanket and drag him head first toward you C) Place the patient in the blanket in a prone position and drag him toward you D) Secure the patient in a supine position and drag him feet first along the ground Answer: B Diff: 1 Page Ref: 107-108 Objective: 6-11 34) Which patient is appropriately positioned for transport? A) An alert 67-year-old woman with nausea in a sitting position B) A critically injured 18-month-old child in a car seat C) A hypotensive 48-year-old man in Fowler's position D) A 23-year-old pregnant woman in a supine position Answer: A Diff: 2 Page Ref: 116 Objective: 6-12 35) A large female patient has fallen in a parking lot between two cars. She must be moved on the backboard to the waiting stretcher, which is located approximately 200 feet from your location. Four rescuers are present. How would they best be used to promote the safest movement of the patient? A) One at the head, two at the feet, and the fourth to act as a spotter during the move B) The strongest person at the head, and the other three at the foot end C) Two at the head, two at the feet, with all rescuers using their right hand to carry the backboard D) One at the head, one at the feet, and the other two on opposite sides on the backboard Answer: D Diff: 2 Page Ref: 118 Objective: 6-10 36) You are trying to convince the equipment committee to purchase a hydraulic power cot. When discussing advantages of this device over a more traditional stretcher, which point would you emphasize? A) It is considered much safer for transporting a child car seat B) It is much lighter, more durable, and easier to maneuver C) It will be easier to lift and lower when it is loaded with a patient D) It enables one EMT to safely move the stretcher, freeing another for patient care Answer: C Diff: 2 Page Ref: 112 Objective: 6-11

9 Copyright © 2018 Pearson Education, Inc.

37) You are moving a critically injured patient to a helicopter for rapid transport to a trauma center. The rotors are spinning so that the aircraft can immediately take off. As team leader, which instruction would you provide? A) "Let's get many people around the backboard so we can safely load the patient into the aircraft." B) "I need someone to hold the IV bag in the air while we load so the patient continues to get IV fluids." C) "Will someone please lay the O2 tank on the patient's legs so he continues to get oxygen as we load?" D) "Will someone please securely cover the patient's eyes so that they are not struck with flying dirt?" Answer: D Diff: 3 Page Ref: 117 Objective: 6-13 38) The safest and most efficient way of using your body to gain mechanical advantage is: A) The power lift B) To call for backup C) To reduce the distance an object must be used D) Body mechanics Answer: D Diff: 1 Page Ref: 102 Objective: 6-2 39) While teaching a continuing education class on the lifting and moving of patients, which points would you constantly emphasize? A) Keeping the legs straight while lifting any heavy object to the cot or into the ambulance B) Maintaining a normal inward curve in the lower back while moving patients or equipment C) Lifting a patient or equipment should always occur with the wrists hyperextended D) Do not rely on equipment to help you lift, as the equipment often fails and results in patient injury Answer: B Diff: 1 Page Ref: 102 Objective: 6-2

10 Copyright © 2018 Pearson Education, Inc.

40) You are at the scene of a motorcycle accident on a rural road. The EMS captain has set up a landing zone and the local helicopter EMS agency has just landed; the aircraft is still running with both rotors turning. Which observation would cause you to take immediate action? A) You see a new EMT start walking toward the aircraft as if he is going to open a door B) The patient was removed from the ambulance on the wheeled cot in anticipation of the flight crew arrival C) The EMTs who will be helping to transfer the patient to the aircraft are removing their hats and sunglasses D) The flight crew has exited the aircraft and are walking toward you while the main and tail rotor blades of the aircraft are still rotating Answer: A Diff: 1 Page Ref: 117 Objective: 6-13 41) You and your partner have just been dispatched to the local children's hospital to pick up the neonatal transport team and the neonatal isolette so that you can transport them to a small community hospital 2 hours away and retrieve a neonate who is in distress. While driving to the children's hospital, which statement from your partner demonstrates he understands how to transport a patient with in an isolette? A) "Once we get to the children's hospital, I will remove the wheeled cot so we can load the neonatal isolette." B) "I realize that if I'm driving to the community hospital, I will not use the lights and sirens." C) "I hate taking these types of transports because it is so boring having to sit in the front of the ambulance." D) "If you are the one in the back with the transport team on our way back to the children's hospital with the patient, make sure you turn the heat up to the highest level so the neonate stays warm." Answer: A Diff: 1 Page Ref: 118 Objective: 6-14

11 Copyright © 2018 Pearson Education, Inc.

Prehospital Emergency Care, 11e (Mistovich et al.) Chapter 7 Anatomy, Physiology, and Medical Terminology 1) A 23-year-old male patient has been shot with a rifle by an assailant standing in front of him. The bullet entered the victim's chest just above the left nipple and exited his body laterally left of his fourth lumbar vertebrae. Proper documentation of this injury would include: A) Entry wound to the anterior thorax B) Exit wound to the posterior thorax C) Entry wound to the abdomen D) Exit wound to the posterior torso Answer: A Diff: 1 Page Ref: 131 Objective: 7-5 2) An 89-year-old female patient has fallen in her living room and has pain in her left hip. As you approach, you note that she is on her back with her left leg turned inward. When relaying this information to the receiving hospital, how would you describe her position? A) Supine with lateral rotation of the left lower extremity B) Lateral recumbent with dorsal rotation of the left leg C) Supine with medial rotation of the left lower extremity D) Supine with left rotation of the right leg Answer: C Diff: 1 Page Ref: 129 Objective: 7-4 3) A female patient has suffered a stroke to the right side of her brain and has paralysis to the contralateral portion of the body. Which description of this condition is correct? A) She is paralyzed on the right side of her body B) Her left and right legs are paralyzed C) She cannot move her arms or legs D) She cannot move her left arm and leg Answer: D Diff: 2 Page Ref: 131 Objective: 7-27 4) A paramedic tells you that a patient with a probable spine injury still retains his plantar reflexes. As a knowledgeable EMT, you should understand that the patient has reflex activity in which part of the body? A) Hands B) Feet C) Arms D) Fingers Answer: B Diff: 1 Page Ref: 131 Objective: 7-5

1 Copyright © 2018 Pearson Education, Inc.

5) Another EMT informs you that a patient has been stabbed once with a knife in the right midaxillary region. Which description best identifies where is this injury located? A) Right chest B) Right shoulder C) Right lateral chest D) Right side of the abdomen Answer: C Diff: 1 Page Ref: 131 Objective: 7-5 6) After you apply and use the automated external defibrillator, a patient regains his pulse and has adequate respirations. The paramedic asks you to place the patient in a lateral recumbent position. How should you position this patient? A) On his back B) On his left side C) On his stomach D) Supine with his head turned left Answer: B Diff: 1 Page Ref: 129 Objective: 7-4 7) Which plane would show the liver, stomach, and spleen? A) Transverse plane B) Midline plane C) Abdominal plane D) Sagittal plane Answer: A Diff: 2 Page Ref: 131 Objective: 7-1 8) Which statement about the human anatomy is true? A) The hand is proximal to the elbow B) The shoulder is distal to the hand C) The hip is distal to the knee D) The knee is proximal to the ankle Answer: D Diff: 2 Page Ref: 131 Objective: 7-1 9) Which statement best describes the anatomic position? A) Supine position with arms at the side and palms facing downward B) Standing with the arms down at the side and palms facing forward C) Prone position with arms upward and palms facing upward D) Upright and facing backward with the arms raised and legs straight Answer: B Diff: 1 Page Ref: 129 Objective: 7-1 2 Copyright © 2018 Pearson Education, Inc.

10) Which line divides the body into right and left halves when viewing it from the front? A) Midaxillary line B) Anterior axillary line C) Midline D) Midclavicular line Answer: C Diff: 1 Page Ref: 131 Objective: 7-1 11) A patient informs you that he injured a tendon in his knee three years ago and has had a difficult time walking since then. You recognize that this injury involves a structure that: A) Connects muscles to the knee B) Connects the muscles together surrounding the knee C) Holds and secures the bones of the knee D) Attaches a ligament to the knee Answer: A Diff: 2 Page Ref: 132 Objective: 7-6 12) You have been called to transport a patient who has an illness that has impaired the involuntary muscles' ability to contract. What condition would you expect with this patient? A) Decreased peristaltic movement of the large intestine B) Decreased heart rate and increased blood pressure C) Difficulty in walking and grasping objects with the hands D) Difficulty in maintaining an upright posture Answer: A Diff: 3 Page Ref: 141 Objective: 7-9 13) Involuntary or smooth muscles are found in the walls of which types of structures? A) Muscles of the arms and legs B) Blood vessels and intestines C) Spinal cord D) Heart and brain Answer: B Diff: 2 Page Ref: 141 Objective: 7-9

3 Copyright © 2018 Pearson Education, Inc.

14) A female patient has a muscular disease and cannot walk, so she requires a wheelchair. Because the patient cannot ambulate by her own will, you would recognize that the muscles involved are: A) Voluntary B) Involuntary C) Cardiac D) Smooth Answer: A Diff: 2 Page Ref: 141 Objective: 7-9 15) A patient has fractured bones in his left leg. Which bones could be involved in this injury? A) Acetabulum, calcaneus, or carpals B) Femur, tibia, or fibula C) Orbit, lumbar, or shin D) Radius, ulna, humerus, or carpals Answer: B Diff: 2 Page Ref: 136 Objective: 7-7 16) The segment of the spinal column that forms the posterior pelvis is the: A) Thoracic spine B) Cervical vertebrae C) Lumbar segment D) Sacral vertebrae Answer: D Diff: 2 Page Ref: 136 Objective: 7-7 17) A patient complains of pain to the upper jaw. This would be documented as pain in the: A) Mandible B) Zygoma C) Maxilla D) Mastoid Answer: C Diff: 2 Page Ref: 134 Objective: 7-7 18) The bones of the upper extremities include the: A) Humerus and radius B) Humerus and calcaneus C) Phalanges and tibia D) Radius, ulna, and tarsals Answer: A Diff: 2 Page Ref: 138 Objective: 7-7 4 Copyright © 2018 Pearson Education, Inc.

19) Occlusion to which pulmonary structure would cause total cessation of airflow into and out of the lungs? A) Right bronchi B) Nasopharynx C) Trachea D) Esophagus Answer: C Diff: 2 Page Ref: 146 Objective: 7-10 20) What is the basic function of the respiratory system? A) Transport oxygen throughout the body B) Provide nutrients to the cells absorbed into the bloodstream C) Release oxygen from the body via hemoglobin sites D) Exchange oxygen and carbon dioxide with the environment Answer: D Diff: 1 Page Ref: 143 Objective: 7-10 21) The alveoli of a patient's lungs are filled with fluid and pus secondary to pneumonia. How does this disturbance place the patient at risk? A) Decreased movement of air into the lungs B) Decreased movement of the diaphragm C) Decreased absorption of oxygen into the body D) Collapse of the trachea and bronchi Answer: C Diff: 2 Page Ref: 147 Objective: 7-13 22) The structure containing a person's "voice box" is termed the: A) Pharynx B) Larynx C) Trachea D) Sternum Answer: B Diff: 1 Page Ref: 146 Objective: 7-10

5 Copyright © 2018 Pearson Education, Inc.

23) A patient has severe swelling of the leaf-shaped flap that helps to prevent food from entering the lower respiratory system while swallowing. Knowing that the suffix "itis" refers to inflammation, what is this patient's condition? A) Epiglottitis B) Tracheitis C) Pharyngitis D) Bronchitis Answer: A Diff: 3 Page Ref: 146 Objective: 7-10 24) What represents the correct sequence for the passage of air into the lungs, when starting from outside the body? A) Nose, bronchi, larynx, trachea, pharynx, air sacs B) Larynx, esophagus, trachea, bronchi, alveoli C) Epiglottis, trachea, cricoid, bronchi, air sacs D) Mouth, pharynx, trachea, bronchi, alveoli Answer: D Diff: 2 Page Ref: 143 Objective: 7-10 25) As you arrive on scene, an Emergency Medical Responder informs you that the patient has agonal respirations. Based on this description, you would expect the patient's breathing to be: A) Irregular and deep B) Occasional and gasping C) Rapid and shallow D) Deep and fast Answer: B Diff: 1 Page Ref: 150 Objective: 7-12 26) When the diaphragm and intercostal muscles relax, which phase of the respiratory cycle happens next? A) Inhalation B) Retraction C) Expansion D) Exhalation Answer: D Diff: 2 Page Ref: 148 Objective: 7-12

6 Copyright © 2018 Pearson Education, Inc.

27) Which action produces inhalation and contributes to airflow into the lungs? A) The intercostal muscles relax B) The chest cavity decreases in size C) The diaphragm contracts D) The pressure in the chest increases Answer: C Diff: 3 Page Ref: 148 Objective: 7-12 28) The infant airway differs from the adult airway in that: A) The infant's tongue takes up proportionately more space in the mouth B) The infant's trachea is highly rigid and less flexible to movement C) All of the infant's respiratory structures are proportionally smaller and more easily obstructed D) The infant's epiglottis is smaller and located behind the esophagus Answer: A Diff: 2 Page Ref: 147 Objective: 7-11 29) The EMT shows correct understanding of the circulatory system when he identifies the primary function of the heart as: A) Transferring oxygen to the cells B) Pumping blood throughout the body C) Oxygenating blood in the lungs D) Maintaining the volume of blood Answer: B Diff: 1 Page Ref: 150 Objective: 7-14 30) Which portion of the heart is in failure when it can no longer effectively pump blood to the lungs? A) Left atrium B) Left ventricle C) Right ventricle D) Pulmonary vein Answer: C Diff: 2 Page Ref: 150 Objective: 7-15 31) The left side of a patient's heart is in severe failure, causing blood to back up behind it. Which sign or symptom are you most likely to observe first? A) Swelling in the feet B) Decreased respirations C) Bone pain in the legs D) Crackles in the lungs Answer: D Diff: 3 Page Ref: 150 Objective: 7-15 7 Copyright © 2018 Pearson Education, Inc.

32) What is the primary function of the left atrium? A) It receives blood from the vena cava B) It receives blood from the pulmonary veins C) It pumps blood to the lungs D) It pumps blood to the body Answer: B Diff: 1 Page Ref: 150 Objective: 7-14 33) A patient has a total occlusion of the left femoral artery. With this condition, which assessment finding would the EMT expect? A) Pale skin to the left foot B) Swelling in the legs C) Difficulty moving the left arm D) Decreased radial pulse Answer: A Diff: 3 Page Ref: 151 Objective: 7-15 34) When palpating a pulse on top of the foot, the EMT is assessing blood flow in which vessel? A) Posterior tibial vein B) Femoral vein C) Coronary artery D) Dorsalis pedis artery Answer: D Diff: 1 Page Ref: 151 Objective: 7-15 35) A patient complains of extreme dizziness, headache, and other "stroke-like" symptoms. Which condition is most likely to be responsible for these symptoms? A) Occlusion of the carotid arteries B) Clot in the abdominal aorta C) Rupture of the radial vein D) Occlusion of the popliteal vein Answer: A Diff: 3 Page Ref: 151 Objective: 7-15 36) What is the major artery of the thigh called? A) Carotid artery B) Femoral artery C) Radial artery D) Brachial artery Answer: B Diff: 1 Page Ref: 151 Objective: 7-15 8 Copyright © 2018 Pearson Education, Inc.

37) Which structure or vessel carries oxygen-depleted blood to the right atrium? A) Posterior tibial B) Internal jugular C) Vena cava D) Aorta Answer: C Diff: 1 Page Ref: 151 Objective: 7-15 38) A patient is experiencing the backup of blood and fluid in her lung tissue. Failure of which of the heart's valves is likely responsible for this condition? A) Tricuspid B) Right C) Bicuspid D) Pulmonary Answer: C Diff: 3 Page Ref: 150 Objective: 7-15 39) Following radiation therapy for breast cancer, a female patient has a low white blood cell count. Which condition does this factor place her at risk to acquire? A) Hypoxia B) Infection C) Bleeding D) Low blood pressure Answer: B Diff: 2 Page Ref: 154 Objective: 7-15 40) You are assessing a patient who informs you that her body abnormally forms blood clots, even when they are not needed to stop bleeding. Which condition could you expect the patient to have that contributes to the blood clotting? A) Low red blood cell count B) High white blood cell count C) Decreased clotting factors D) Elevated number of platelets Answer: D Diff: 2 Page Ref: 154 Objective: 7-15

9 Copyright © 2018 Pearson Education, Inc.

41) What is the name of the fluid that carries blood cells and nutrients? A) Platelet fluid B) Hemoglobin C) Total body water D) Plasma Answer: D Diff: 1 Page Ref: 154 Objective: 7-15 42) The pressure exerted on the walls of the arteries as the left ventricle contracts is called: A) Systolic B) Arterial C) Diastolic D) Osmotic Answer: A Diff: 2 Page Ref: 155 Objective: 7-15 43) The nervous system is structurally subdivided into the: A) Brain and vertebral column B) Involuntary and voluntary nervous systems C) Skull and vertebral column D) Central and peripheral nervous systems Answer: D Diff: 1 Page Ref: 157 Objective: 7-17 44) A patient's blood glucose (sugar) level begins to drop. Which organ would be affected first due to its heavy reliance on circulating glucose? A) Spleen B) Brain C) Muscles D) Heart Answer: B Diff: 2 Page Ref: 157 Objective: 7-16 45) The terms "occipital," "parietal," and "frontal" can refer collectively to which part of the body? A) Chest B) Cranium C) Pelvis D) Spine Answer: B Diff: 1 Page Ref: 134 Objective: 7-7 10 Copyright © 2018 Pearson Education, Inc.

46) Assessment of an elderly male patients reveals that he is confused. His family states that this status is normal for him because he has a disease of his brain that has affected his ability to remember things. Based on this information, in which area of the brain is this disease located? A) Pons B) Brainstem C) Cerebrum D) Cerebellum Answer: C Diff: 2 Page Ref: 157 Objective: 7-16 47) A patient has experienced a stroke that has permanently injured his brainstem. Which sign or symptom might the EMT expect to find as a result of this condition? A) Confusion B) Abnormal respirations C) Blindness D) Muscle spasms Answer: B Diff: 3 Page Ref: 157 Objective: 7-17 48) Which body system protects internal organs of the chest and provides for movement of the extremities? A) Endocrine B) Nervous C) Musculoskeletal D) Immune Answer: C Diff: 1 Page Ref: 132 Objective: 7-6 49) The spinal column is composed of which sections, described in order from most superior to most inferior? A) Cervical, lumbar, thoracic, sacral, and coccyx B) Coccyx, lumbar, thoracic, cervical, and sacral C) Thoracic, lumbar, cervical, coccyx, and sacral D) Cervical, thoracic, lumbar, sacral, and coccyx Answer: D Diff: 1 Page Ref: 135-136 Objective: 7-17

11 Copyright © 2018 Pearson Education, Inc.

50) In the healthy heart, the electrical impulses that cause normal contraction of the cardiac muscle are initiated in the: A) Sinoatrial node B) Left atrium C) Bundle branches D) Ventricles Answer: A Diff: 2 Page Ref: 151 Objective: 7-14 51) A patient has taken a medication that increases the activity of his sympathetic nervous system. Given this treatment, which sign or symptom would the EMT expect to find? A) Decreased heart rate B) Dilated pupils C) Low blood pressure D) Dry and flushed skin Answer: B Diff: 3 Page Ref: 160, Figure 7-35 Objective: 7-18 52) As a patient provides you with her past medical history, which condition should you recognize as directly related to the endocrine system? A) Blockage of the gallbladder B) Failure of the kidneys C) Removal of the thyroid D) Enlargement of the appendix Answer: C Diff: 2 Page Ref: 162 Objective: 7-20 53) In which body system are the thyroid glands, adrenal glands, pituitary glands, and gonads located? A) Nervous B) Endocrine C) Hormonal D) Respiratory Answer: B Diff: 1 Page Ref: 162 Objective: 7-20

12 Copyright © 2018 Pearson Education, Inc.

54) A patient has experienced a burn to the skin. Based on the function of the skin, he is now more susceptible to: A) Infection B) Fluid overload C) Heart problems D) Endocrine disorders Answer: A Diff: 1 Page Ref: 162 Objective: 7-22 55) You are assessing a patient with an obvious left midshaft femur fracture. The most immediate threat to this patient's survival is: A) Infection B) Blood loss C) Pain D) Nerve damage Answer: B Diff: 2 Page Ref: 140 Objective: 7-7 56) What is a function of the skeletal system? A) Producing blood cells B) Protecting all of the abdominal organs C) Providing structure and points of attachment to blood vessels D) Forming the vital organs of the body Answer: A Diff: 1 Page Ref: 132 Objective: 7-6 57) Emergency Medical Responders inform you that a 73-year-old female patient has fallen and has a hematoma to the occipital area of her head. Based on this description, you should expect to find the injury in which area of the head? A) Top B) Side C) Back D) Front Answer: C Diff: 1 Page Ref: 133, Figure 7-4 Objective: 7-5

13 Copyright © 2018 Pearson Education, Inc.

58) A patient with an orbital fracture is most likely to complain of: A) Pain around the ear B) Difficulty swallowing C) Pain when moving his lower jaw D) Pain around the eye Answer: D Diff: 1 Page Ref: 134 Objective: 7-5 59) A patient experiencing pain in the thoracic region of his spinal column has pain in his: A) Neck B) Lower back C) Upper back D) Tailbone Answer: C Diff: 1 Page Ref: 135 Objective: 7-5 60) Degenerative disk disease to the vertebrae is characterized by damage to which structure(s) between the vertebrae? A) Intervertebral disks B) Spinal joints C) Spinal cord D) Vertebral nerve endings Answer: A Diff: 1 Page Ref: 135 Objective: 7-8 61) Assessment of a patient who was involved in a motor vehicle collision reveals tenderness over ribs 9-12 on the lateral right side. Based on this injury, the EMT should be alert for which additional concern? A) Damage to the heart B) Injury to the liver C) Pain from possible rib fractures D) Damage to the false ribs Answer: B Diff: 2 Page Ref: 163 Objective: 7-23 62) A patellar fracture affects which part of the body? A) Tibia B) Hip C) Elbow D) Knee Answer: D Diff: 1 Page Ref: 136 Objective: 7-6 14 Copyright © 2018 Pearson Education, Inc.

63) A patient suffering a carpal fracture would most likely complain of pain in her: A) Ankle B) Wrist C) Knee D) Elbow Answer: B Diff: 1 Page Ref: 138 Objective: 7-7 64) Knowing that muscular dystrophy affects the voluntary muscles, which additional problem should you expect to be a direct effect of the disease? A) Constipation B) Slow heart rate C) Inability to walk D) Vasodilation Answer: C Diff: 2 Page Ref: 141 Objective: 7-9 65) Which statement best describes the difference between cardiac and smooth muscle? A) Cardiac muscle can generate its own electrical impulse; smooth muscle cannot B) Cardiac muscle is able to extend; smooth muscle can only contract C) Cardiac muscle relies on an electrical impulse to be stimulated; smooth muscle does not D) Cardiac muscle does not need a blood supply; smooth muscle does Answer: A Diff: 3 Page Ref: 141 Objective: 7-9 66) Which statement regarding the endocrine system is most correct? A) It includes the thyroid, parathyroid, adrenal, and salivary glands B) It is a system of glands that releases hormones via ducts into the bloodstream C) It is a system of glands that regulates consciousness D) It is made up of several glands whose secretions cause specific actions in specific organs Answer: D Diff: 3 Page Ref: 160 Objective: 7-20 67) A patient who has had both her thyroid and parathyroid glands removed would most likely have difficulty with: A) Producing the hormones that govern sex characteristics B) Maintaining strong and healthy bones C) Regulating blood sugar D) Maintaining an adequate blood pressure Answer: B Diff: 2 Page Ref: 162 Objective: 7-20 15 Copyright © 2018 Pearson Education, Inc.

68) Which finding is a beta-1 sympathetic (adrenergic) effect? A) Bronchodilation B) Increased heart rate C) Vasodilation D) Vasoconstriction Answer: B Diff: 2 Page Ref: 162 Objective: 7-21 69) A patient with asthma is short of breath because her small airways are constricted. For this patient, the best medication is one that has which sympathetic (adrenergic) properties? A) Alpha-1 B) Beta-1 C) Beta-2 D) Alpha-2 Answer: C Diff: 2 Page Ref: 162 Objective: 7-21 70) An abrasion affects which part of the skin? A) Epidermis B) Dermal layer C) Dermal and subcutaneous layers D) Epidermal and dermal layers Answer: D Diff: 2 Page Ref: 162 Objective: 7-22 71) Which statement regarding the integumentary system is true? A) The skin protects the body against bacteria and other organisms B) The skin contains four layers: epidermal, dermal, subcutaneous, and nerve layer C) The epidermis is the thickest and most important layer of the skin D) The dermal layer is the outermost layer of the skin and contains sensory nerves Answer: A Diff: 1 Page Ref: 162 Objective: 7-22 72) Which assessment finding should the EMT expect after administering epinephrine to a patient? A) Slowing of the heart rate B) Increased blood pressure C) Labored breathing D) Generalized weakness Answer: B Diff: 2 Page Ref: 162 Objective: 7-21 16 Copyright © 2018 Pearson Education, Inc.

73) A patient informs you that he has had his gallbladder surgically removed. Based on the physiology of the gallbladder, which statement is most true regarding this patient? A) He may have difficulty digesting large amounts of fats B) He cannot store glycogen produced by the liver C) He will be more prone to gastric ulcers D) He must avoid meals high in complex carbohydrates Answer: A Diff: 2 Page Ref: 163 Objective: 7-23 74) A patient informs you that he must sprinkle artificial digestive enzymes on all his foods before eating them. Based on this information, which endocrine gland should you assume is dysfunctional? A) Thyroid B) Adrenal C) Spleen D) Pancreas Answer: D Diff: 2 Page Ref: 163 Objective: 7-23 75) A patient with kidney failure has lost his ability to: A) Produce all hormones B) Absorb nutrients C) Filter and excrete wastes D) Digest fatty foods Answer: C Diff: 2 Page Ref: 165 Objective: 7-24 76) You are using cryotherapy to treat a deformed and painful ankle. This treatment involves the use of: A) Compression B) Ice C) Massage D) Heat Answer: B Diff: 1 Page Ref: 170, Table 7-2 Objective: 7-27

17 Copyright © 2018 Pearson Education, Inc.

77) Pain that is described as "caustic" by the patient is: A) Burning B) Sharp C) Stabbing D) Achy Answer: A Diff: 1 Page Ref: 170, Table 7-2 Objective: 7-27 78) A patient with bronchitis is experiencing: A) Fluid in the trachea and bronchioles B) Infection of the alveoli C) Inflammation of the bronchi D) Accumulation of mucus in the trachea Answer: C Diff: 2 Page Ref: 170, Table 7-2 Objective: 7-27 79) A patient with a calcaneal fracture has a broken: A) Foot B) Thigh C) Knee D) Heel Answer: D Diff: 1 Page Ref: 138 Objective: 7-5 80) A patient with dentalgia is suffering from a(n): A) Painful tooth B) Throat infection C) Inflammation of the tongue D) Sinus infection Answer: A Diff: 1 Page Ref: 170, Table 7-2 Objective: 7-27 81) A patient described as dysphagic presents as: A) Having difficulty breathing B) Having difficulty swallowing C) Being unable to speak D) Exhibiting a painful cough Answer: B Diff: 2 Page Ref: 171, Table 7-2 Objective: 7-27

18 Copyright © 2018 Pearson Education, Inc.

82) Which term is most closely related to the elbow? A) Acetabular B) Tibial C) Calcaneal D) Cubital Answer: D Diff: 1 Page Ref: 133, Figure 7-4 Objective: 7-5 83) A tachypneic patient is breathing: A) Slowly B) Deeply C) Rapidly D) Shallowly Answer: C Diff: 1 Page Ref: 172, Table 7-3 Objective: 7-21 84) An enlarged liver is best documented as: A) Hepatitis B) Hepatomegaly C) Ascites D) Gastric distention Answer: B Diff: 1 Page Ref: 171, Table 7-2 Objective: 7-27 85) Nephritis indicates which kind of infection? A) Urinary tract B) Bladder C) Kidney D) Digestive Answer: C Diff: 2 Page Ref: 171, Table 7-2 Objective: 7-27 86) Acrocyanosis is best described as a bluish discoloration of the patient's: A) Head B) Extremities C) Chest D) Abdomen Answer: B Diff: 1 Page Ref: 170, Table 7-2 Objective: 7-27

19 Copyright © 2018 Pearson Education, Inc.

87) What is the term that refers to the structure of the body? A) Physiology B) Metabolism C) Combining forms D) Anatomy Answer: D Diff: 1 Page Ref: 129 Objective: 7-3 88) Accurately communicating patient injury information to other health care professionals is most reliant upon: A) A strong radio signal B) Properly placed cellular towers C) A good knowledge of proper anatomic terminology D) Good handwriting Answer: C Diff: 1 Page Ref: 129 Objective: 7-2 89) Which components of the nervous system must be functional for a patient to be conscious? A) Both cerebral hemispheres B) One cerebral hemisphere and the pineal gland C) One cerebral hemisphere and the reticular activating system D) The RAS and pineal gland Answer: C Diff: 1 Page Ref: 159 Objective: 7-19 90) Which organ(s) is (are) responsible for production and secretion of the primary female hormone progesterone? A) Ovaries B) External genitals C) Testes D) Uterus Answer: A Diff: 1 Page Ref: 165 Objective: 7-25 91) Which term refers to how the parts of the body function and interact? A) Physiology B) Anatomy C) Aerobic metabolism D) Biome Answer: A Diff: 1 Page Ref: 129 Objective: 7-2 20 Copyright © 2018 Pearson Education, Inc.

92) Understanding physiology will allow the EMT to understand that bronchoconstriction will have which effect on breathing? A) There will be a decrease in the work of breathing B) There will be an increase in the work of breathing C) There will be an increase in the flow of air through the bronchioles D) There will be a decrease in the patient's respiratory rate Answer: B Diff: 2 Page Ref: 146 Objective: 7-2 93) For the lungs to oxygenate blood, there must be blood flow provided by the right ventricle to the alveoli. This is an example of which principle? A) Anatomy B) Pathophysiology C) Physiology D) Anatomic actions Answer: C Diff: 2 Page Ref: 149 Objective: 7-3 94) A psychologically disturbed patient decided to kill himself by shooting himself in the head. The bullet entered the right temporal lobe and exited the skull on the opposite side. If this patient is unresponsive, what is the most likely cause? A) Both cerebral hemispheres are damaged B) The cerebellum is damaged C) The patient has likely lost too much blood D) The endocrine system (hypothalamus) has failed Answer: A Diff: 3 Page Ref: 159 Objective: 7-19 95) What is the name of the male reproductive structure that stores sperm? A) Testes B) Vas deferens C) Seminal vesicles D) Epididymis Answer: D Diff: 2 Page Ref: 167 Objective: 7-25

21 Copyright © 2018 Pearson Education, Inc.

96) You are reading a patient care report written by your partner. Which sentence would cause you to tell your partner that you have found an error? A) "The patient was placed in a face-up prone position." B) "The patient was in a lateral recumbent position so as to allow airway monitoring while en route to the hospital." C) "The patient was placed in Fowler's position with the upper body elevated 45 degrees." D) "The patient was found in a supine position with his hips and legs flexed toward the core." Answer: A Diff: 2 Page Ref: 129-130 Objective: 7-2 97) Which type of joint provides the greatest amount of free motion? A) Hinged joint B) Ball-and-socket joint C) Gliding joint D) Saddle joint Answer: B Diff: 1 Page Ref: 140 Objective: 7-8 98) An example of a pivot joint of the body would be: A) Where the femur joints the pelvis B) Where the radius and ulna meet the humerus C) Where the first and second vertebrae meet D) Where the ankle meets the tibia and ulna Answer: B Diff: 2 Page Ref: 140 Objective: 7-8 99) Based on the anatomic and physiological differences between an infant and an adult, which finding would suggest respiratory distress in an infant, but would be less likely to be observed in an adult? A) Nasal flaring during exhalation B) Excessive abdominal wall motion C) Increased respiratory rate D) Breathing with the mouth open Answer: B Diff: 2 Page Ref: 148 Objective: 7-11

22 Copyright © 2018 Pearson Education, Inc.

100) Which individual finding in a patient is the most reliable indicator that the patient is breathing inadequately? A) Respiratory rate of 22 breaths/min in an elderly patient B) Chest wall motion outward of 1 inch during inhalation in an adult patient C) Respiratory rate of 16 breaths/min in a child D) Shallow breathing at a rate of 30 breaths/min in an infant Answer: D Diff: 2 Page Ref: 150 Objective: 7-13 101) Which division of the peripheral nervous system is responsible for increasing gastric juice secretions after a person eats a meal? A) Autonomic B) Voluntary C) Sympathetic D) Somatic Answer: A Diff: 3 Page Ref: 157-158 Fig 7-35 Objective: 7-18 102) What is the name of the structure in males and females that transports the urine from the kidneys to the urinary bladder? A) Urethra B) Vulva C) Ureters D) Renal pelvis Answer: C Diff: 1 Page Ref: 165 Objective: 7-24

23 Copyright © 2018 Pearson Education, Inc.

Prehospital Emergency Care, 11e (Mistovich et al.) Chapter 8 Pathophysiology 1) Normal cellular metabolism can be defined as: A) Oxygenated B) Aerobic C) Glycolytic D) Anaerobic Answer: B Diff: 1 Page Ref: 178 Objective: 8-3 2) When cells undergo normal metabolism in the body, which byproduct(s) is (are) produced? A) Heat, carbon dioxide, and water B) Lactic acid C) Protein, carbon dioxide, and water D) Adenosine monophosphate Answer: A Diff: 2 Page Ref: 179 Objective: 8-3 3) Under normal circumstances, carbon dioxide is excreted from the body through the: A) Kidneys B) Urinary tract C) Lungs D) Sweat ducts Answer: C Diff: 2 Page Ref: 179 Objective: 8-3 4) The patient in anaerobic metabolism is deficient in/of: A) Glucose (sugar) B) Protein C) Lactic acid D) Oxygen Answer: D Diff: 1 Page Ref: 179 Objective: 8-3 5) A patient's brain cells are undergoing anaerobic metabolism. As a result, those cells: A) Are becoming dysfunctional in an environment that is increasingly acidic B) Have too much adenosine triphosphate to function properly C) Are producing elevated quantities of carbon dioxide (CO2) as a result of aerobic metabolism D) Cannot make any energy in the form of ATP due to a lack of glucose Answer: A Diff: 3 Page Ref: 179 Objective: 8-3 1 Copyright © 2018 Pearson Education, Inc.

6) A patient's lungs are full of fluid and cannot transfer oxygen to the blood. As a result, she is undergoing anaerobic metabolism. She has a decreased level of consciousness with a patent upper airway and inadequate respirations. Her pulse is rapid and weak. Which prehospital treatment would best address this situation? A) High-concentration oxygen through a nonrebreather face mask B) Administration of one tube of oral glucose by the EMT C) Positive pressure ventilation with supplemental oxygen D) Insertion of a nasal airway and delivery of oxygen through a nasal cannula Answer: C Diff: 2 Page Ref: 187 Objective: 8-9 7) What happens after a cell in anaerobic metabolism breaks down glucose and creates energy in the form of adenosine triphosphate (ATP)? A) Oxygen and water are created B) Lactic acid is formed C) Carbon dioxide and water are formed D) Electrolytes in the cell become balanced Answer: B Diff: 3 Page Ref: 179 Objective: 8-3 8) For cells to undergo the process of aerobic metabolism, the cells require: A) Water and oxygen B) Sugar (glucose) and protein C) Adenosine triphosphate (ATP) and lactic acid D) Oxygen and glucose Answer: D Diff: 2 Page Ref: 179 Objective: 8-3 9) Failure of the sodium-potassium pump can result in: A) Accumulation of acidic wastes B) Excess amounts of sodium outside the cell C) Large amounts of potassium inside the cell D) Cellular damage, swelling, and rupture Answer: D Diff: 3 Page Ref: 180 Objective: 8-3

2 Copyright © 2018 Pearson Education, Inc.

10) Perfusion is best described as: A) Availability of oxygen in the lungs for movement into the blood B) An adequate number of white blood cells to carry oxygen to the cells C) Delivery of essential products and nutrients to the cell for its use D) Exchange of oxygen and carbon dioxide between the lungs and blood Answer: C Diff: 2 Page Ref: 180 Objective: 8-4 11) A patient is hemorrhaging internally within his colon, which in turn is impairing perfusion to the cells of his body. Aside from the bleeding, the patient has no other problems. In this scenario, the problem impairing adequate perfusion would be: A) Decreased blood volume B) Inability to get oxygen to the alveoli C) Poor myocardial pump function D) Inadequate ventilation to eliminate carbon dioxide Answer: A Diff: 2 Page Ref: 194 Objective: 8-20 12) What is required for normal perfusion to occur? A) Airway patency, proper amounts of carbon dioxide, and adequate glucose levels B) Adequate breathing, sufficient numbers of red blood cells, and sufficient heart function C) Intact breathing, an ample number of red blood cells, and body temperature of 98.6°F D) Patent airway, sufficient breathing, high amounts of glucose, and good heart function Answer: B Diff: 3 Page Ref: 180 Objective: 8-4 13) A patient who has lost one liter of blood is conscious with a patent airway, but her extremities are pale and cold to the touch. Vital signs are as follows: pulse is 130, respirations are 22 breaths/min and adequate, blood pressure is 74/56 mmHg, and SpO2 is 95% on room air. What is most needed by this patient to increase perfusion to the cells? A) Increased blood pressure as a result of paramedics administering IV fluids B) Administration of oxygen via nonrebreather mask C) Increased number of red blood cells and plasma volume D) Keeping the patient warm by covering her with a blanket Answer: C Diff: 3 Page Ref: 192 Objective: 8-4

3 Copyright © 2018 Pearson Education, Inc.

14) The EMT is administering oxygen at 95% through a nonrebreather face mask. He would document this as a FiO2 of: A) 0.21 B) 0.50 C) 0.95 D) 1.00 Answer: C Diff: 1 Page Ref: 181 Objective: 8-4 15) For a patient breathing in room air, the EMT would estimate the FiO2 to be: A) 0.15 B) 0.21 C) 0.55 D) 1.00 Answer: B Diff: 1 Page Ref: 181 Objective: 8-5 16) The EMT would document an FDO2 level when the patient is: A) Assisting breathing with a bag-valve-mask device B) Inserting a nasopharyngeal airway C) Providing oxygen at greater than 10 liters per minute D) Using a nonrebreather face mask that delivers high-concentration O2 Answer: A Diff: 1 Page Ref: 181 Objective: 8-5 17) A patient with a low blood sugar is unresponsive with snoring respirations. His breathing is labored and his pulse is rapid and weak. Examination of the skin reveals it to be cool and diaphoretic. Your partner informs you of the following vital signs: pulse, 120; respirations, 18 breaths/min; blood pressure, 132/60 mmHg; and SpO2 at 84% on room air. Which action would you perform first? A) Administer oral glucose B) Perform a head-tilt, chin-lift maneuver C) Provide high-concentration oxygen D) Start positive pressure ventilation Answer: B Diff: 2 Page Ref: 182 Objective: 8-2

4 Copyright © 2018 Pearson Education, Inc.

18) A young patient is experiencing epiglottitis. He is working hard to breathe, has stridorous respirations, and is extremely hypoxic. His skin is cyanotic and his pulse is rapid but strong. What is the most likely cause (not result) of the poor delivery of oxygen to the cells? A) Partial occlusion of the airway B) Impaired transfer of O2 between the lungs and blood C) Poor function of the heart as a pump D) Elevation of the heart rate Answer: A Diff: 2 Page Ref: 183 Objective: 8-2 19) A 20-year-old female patient has called 911 for chest pain. On scene, you find that she has shallow breathing with an SpO2 reading of 91% on room air. She states she was in a car crash yesterday and diagnosed in the hospital with broken ribs. Her pain is right where the ribs are broken, and she rates the pain as a 10/10. Breath sounds are present bilaterally. In this situation, the EMT should attribute the hypoxia to which cause? A) The fractured ribs have pierced the lung, causing it to collapse B) Swelling from the rib fractures has compressed the lung C) The patient is bleeding into the lungs, compressing the alveoli D) Pain from rib injury prevents full expansion of the chest cage Answer: D Diff: 2 Page Ref: 185 Objective: 8-9 20) A patient who is hypoxic has a pulmonary disease that involves low lung compliance. With this condition, you realize that: A) The lung tissue is easily inflated and can be damaged B) Oxygen and carbon dioxide cannot be exchanged across the alveolar wall C) The patient has tremendous swelling of the small airways D) Ventilation with a bag-valve mask will likely require more effort Answer: D Diff: 2 Page Ref: 185 Objective: 8-7 21) Assessment of a hypoxic patient who is showing signs of fatigue and has a history of lung disease reveals him to be using well-developed accessory muscles to exhale. As an EMT, you should recognize that the patient: A) Is inadequately exhaling oxygen from his lungs, which is causing hypoxia B) Is using energy to exhale and is in danger of respiratory failure if hypoxia continues C) Has a problem with the red blood cells' ability to transport oxygen, which is causing hypoxia D) Is adequately compensating for the problem with his breathing so he does not become hypoxic Answer: B Diff: 3 Page Ref: 185 Objective: 8-7

5 Copyright © 2018 Pearson Education, Inc.

22) A patient with a high fever has an accelerated metabolism and is producing abnormally large amounts of carbon dioxide at the cellular level. To compensate for this effect, what will occur? A) Decreased levels of acid in the blood B) Decreased respiratory rate C) Decreased depth of breathing D) Increased respiratory rate Answer: D Diff: 3 Page Ref: 187 Objective: 8-10 23) What situation could impair a patient's respiratory status by directly damaging the central chemoreceptors of the body? A) Brainstem stroke or injury B) Lung cancer C) Aerobic metabolism D) Collapsed lung Answer: A Diff: 2 Page Ref: 187 Objective: 8-10 24) Which condition is most likely to cause acidosis? A) Irregular heart rhythm B) Hypothyroidism C) Depressed respirations D) Low blood glucose level Answer: C Diff: 2 Page Ref: 187 Objective: 8-2 25) Which statement about chemoreceptors in the human body is true? A) The central chemoreceptors are located in the lungs and provide the primary stimulus to breathe B) The peripheral chemoreceptors are extremely sensitive to oxygen and are located in the brain C) The central chemoreceptors constantly monitor the oxygen levels in the body and increase the rate when more O2 is needed D) The peripheral chemoreceptors are more sensitive to oxygen than carbon dioxide Answer: D Diff: 2 Page Ref: 187-188 Objective: 8-10

6 Copyright © 2018 Pearson Education, Inc.

26) Which statement is true of a patient who relies on the hypoxic drive to breathe? A) The primary gas affecting the respiratory rate is carbon dioxide B) The peripheral chemoreceptors have become less sensitive to oxygen C) The respiratory rate is set according to the level of O2 in the body D) The central chemoreceptors are the primary influence on the respiratory rate Answer: C Diff: 3 Page Ref: 188 Objective: 8-10 27) For a patient who relies on the hypoxic drive to breathe, the respiratory rate will increase when: A) CO2 levels increase B) O2 levels decrease C) CO2 levels decrease and O2 levels increase D) O2 and CO2 levels both increase Answer: B Diff: 2 Page Ref: 188 Objective: 8-10 28) What is the primary stimulus to breathe in human beings without pulmonary diseases? A) Level of CO2 in the body B) Amount of oxygen required by the body C) Level of oxygen in the body D) Amount of nitrogen in air Answer: A Diff: 1 Page Ref: 187 Objective: 8-10 29) What is the best description of minute ventilation? A) Number of breaths a patient breathes in 1 minute B) Amount of air moved into and out of the lungs in 1 minute C) Total milliliters of air moving into and out of the lungs in one breath D) Amount of oxygen needed by the body in 1 minute Answer: B Diff: 1 Page Ref: 185 Objective: 8-9 30) A 100-kilogram patient with a 500-milliliter tidal volume, breathing 16 times each minute, would have a minute ventilation of how many milliliters? A) 8000 mL B) 1200 mL C) 1600 mL D) 2400 mL Answer: A Diff: 1 Page Ref: 185 Objective: 8-9 7 Copyright © 2018 Pearson Education, Inc.

31) An unresponsive patient with a pulse is breathing with very shallow respirations at a rate of 6 breaths per minute. Which intervention would be most beneficial for this patient? A) Positive pressure ventilation with supplemental O2 at a rate of 30 breaths/min and a tidal volume of 200 mL B) Oxygen at 15 liters per minute delivered through a nonrebreather face mask and then reevaluate in 60 seconds C) Positive pressure ventilation with a rate of 12 breaths/min and a tidal volume of 500 mL D) Oral airway and positive pressure ventilation at 40 breaths/min, supplemental O2, and a tidal volume of 100 mL Answer: C Diff: 2 Page Ref: 186 Objective: 8-9 32) What is the best description of the ventilation/perfusion (V/Q) ratio? A) The ability of the red blood cells to offload oxygen to the cells of the body B) The amount of blood and the pressure at which it carries oxygen throughout the body C) The amount of air that is moved into and out of the lungs in 1 minute D) The ability of the body to exchange gases across the alveolar capillary membrane Answer: D Diff: 2 Page Ref: 189-190 Objective: 8-1 33) Which statement about the ventilation/perfusion (V/Q) ratio in a healthy person is true? A) Blood flow and amount of ventilation are equal throughout the lungs B) The upper portion of the lungs has wasted ventilation C) The amount of blood perfusion throughout the lungs is greater than the amount of oxygen in the lungs D) The lower portion of the lungs has more oxygen than perfusion Answer: B Diff: 2 Page Ref: 189-190 Objective: 8-11 34) Which condition is recognized as one that will affect the ventilation component of the ventilation/perfusion ratio? A) Blood clot in the pulmonary blood vessels B) Loss of blood plasma from dehydration C) Infection and pus in the distal airways and alveoli D) Decreased amount of hemoglobin in the blood Answer: C Diff: 2 Page Ref: 190-191 Objective: 8-11

8 Copyright © 2018 Pearson Education, Inc.

35) A patient's SpO2 increased from 89% to 95% after he received a bronchodilating drug for his asthma. Where did the patient's problem most likely originate? A) Perfusion portion of the V/Q ratio B) Minute volume of the V/Q ratio C) Cellular perfusion portion of the V/Q ratio D) Ventilation portion of the V/Q ratio Answer: D Diff: 2 Page Ref: 189-190 Objective: 8-11 36) What is the primary way in which oxygen is transported to the cells? A) Red blood cells B) White blood cells C) Plasma D) Platelets Answer: A Diff: 1 Page Ref: 192 Objective: 8-12 37) What is the actual site of attachment of oxygen in the red blood cell? A) Oxygen dissolves into the liquid portion of the red blood cell for transport B) On oxygen receptor sites on the surface of the red blood cell C) Within the nucleus of the red blood cell D) Iron sites on hemoglobin within the red blood cell Answer: D Diff: 2 Page Ref: 192 Objective: 8-12 38) Which condition would most likely account for an elevated CO2 level in a patient's body? A) Renal failure B) Lung disease C) Liver injury D) Endocrine stem dysfunction Answer: B Diff: 1 Page Ref: 192-193 Objective: 8-13 39) Which condition must be met for carbon dioxide to move from the cell to the bloodstream? A) The perfusing blood must be low in carbon dioxide B) The sodium-potassium pump must be functional C) There must be a large amount of oxygen in the blood D) There must be a large number of red blood cells in the blood Answer: A Diff: 3 Page Ref: 193 Objective: 8-13 9 Copyright © 2018 Pearson Education, Inc.

40) A patient who is weak informs you that she has a history of her "iron being too low." This should concern the EMT because iron is needed to: A) Manufacture the white blood cells required for fighting infection B) Help the blood to clot C) Carry oxygen throughout the body D) Maintain an adequate blood pressure in the body Answer: C Diff: 1 Page Ref: 192 Objective: 8-12 41) A patient has failure of the left side of his heart. Consequently, his blood is backing up into the pulmonary artery and seeping into the lung tissue, causing the patient to be short of breath and moderately hypoxic. The EMT should recognize this condition as a product of: A) Increased hydrostatic pressure B) Decreased circulating blood volume C) Increased osmotic pressure D) Decreased pulmonary perfusion Answer: A Diff: 2 Page Ref: 194 Objective: 8-15 42) On scene, a 43-year-old male states that he has a history of liver failure and does not produce enough of the protein albumin. Given this condition, which finding(s) would you expect? A) Elevated temperature and signs of infection B) Low SpO2 reading from significant edema C) Swelling of the extremities and abdomen D) Decreased heart rate and signs of hypoxia Answer: C Diff: 2 Page Ref: 194 Objective: 8-15 43) While completing some clinical time in the hospital for his EMT class, an EMT student observed a patient being administered an IV fluid with a high oncotic pressure. Once this fluid is in the patient's body, the EMT student would expect which action to occur? A) Fluid from the tissues will move into the bloodstream B) Fluid will move evenly from the bloodstream into the cells C) Fluid from the IV will move into the cells and rehydrate them D) The patient's blood pressure will decrease dramatically Answer: A Diff: 3 Page Ref: 194 Objective: 8-15

10 Copyright © 2018 Pearson Education, Inc.

44) A patient with high blood pressure takes a medication to slow his heart rate, thereby lowering his blood pressure. The EMT would recognize this action as impacting: A) Preload B) Afterload C) Systemic vascular resistance D) Cardiac output Answer: D Diff: 2 Page Ref: 194 Objective: 8-16 45) Paramedics have administered a medication to a patient with heart failure. The medication specifically causes the veins (only) to dilate, thereby decreasing the amount of blood that returns to the heart for pumping. The EMT would recognize this action as decreasing: A) Afterload B) Pulse pressure C) Preload D) Systemic vascular resistance Answer: C Diff: 2 Page Ref: 195 Objective: 8-17 46) What would be a direct result of increasing a patient's preload? A) Increased cardiac output B) Increased oxygen delivery into the lungs C) Decreased blood pressure D) Decreased myocardial contraction Answer: A Diff: 3 Page Ref: 195 Objective: 8-17 47) A patient has a blood pressure of 140/98 mmHg. What can the EMT ascertain from this reading? A) The pulse pressure is narrowed B) The afterload is increased C) The preload is diminished D) The SpO2 level must be abnormal Answer: B Diff: 2 Page Ref: 195-196 Objective: 8-16

11 Copyright © 2018 Pearson Education, Inc.

48) A chronically elevated afterload increases the patient's chance for developing which pathological condition? A) Liver disease B) Hypotension C) Diabetes D) Heart failure Answer: D Diff: 1 Page Ref: 195 Objective: 8-16 49) What definition best describes the concept of afterload? A) Pressure that the left ventricle must pump blood against to open the aortic valve B) Ability of the heart to adequately contract and pump blood out of the pulmonic valve C) Amount of blood that a person has in the arteries at any given moment D) Amount of blood returned to the lungs for oxygenation by the right ventricle Answer: A Diff: 2 Page Ref: 195 Objective: 8-1 50) What is the best way to decrease a patient's afterload? A) Slow the heart rate B) Reduce the blood pressure C) Administer intravenous fluids D) Provide supplemental oxygen Answer: B Diff: 2 Page Ref: 196 Objective: 8-16 51) A slightly confused but conscious 61-year-old female patient with generalized weakness has a heart rate of 190 beats/min. She has a history of high blood pressure and diabetes. Her blood pressure is 78/56 mmHg, and she has a pulse oximetry reading of 93%. Which disturbance would best explain this patient's presentation? A) Unrecognized hemorrhage B) Hypoglycemia C) Hypoxia D) Excessive tachycardia Answer: D Diff: 3 Page Ref: 195 Objective: 8-16

12 Copyright © 2018 Pearson Education, Inc.

52) A patient with a temperature of 105.4°F is experiencing systemic vasodilation. Which finding would be consistent with this condition? A) Slow heart rate B) Elevated afterload C) Decreased blood pressure D) Pale and cool skin Answer: C Diff: 2 Page Ref: 197 Objective: 8-17 53) If a patient's blood pressure cannot be increased by adding volume, then which other action would most likely be successful in increasing it? A) Blocking beta actions of the sympathetic nervous system B) Decreasing the patient's preload C) Vasoconstriction of the body's arteries D) Decreasing the patient's afterload Answer: C Diff: 2 Page Ref: 196-197 Objective: 8-17 54) Which situation would most likely cause a patient to have a simple episode of syncope (fainting)? A) Increase in systemic vascular resistance B) Parasympathetic stimulation C) Activation of the sympathetic nervous system D) Sudden release of norepinephrine Answer: B Diff: 2 Page Ref: 197 Objective: 8-17 55) The EMT would most likely see a narrowed pulse pressure in a patient who: A) Is bleeding internally B) Has a fever C) Has diabetes D) Has an irregular heartbeat Answer: A Diff: 2 Page Ref: 198 Objective: 8-17 56) Which blood pressure reading best indicates a narrowed pulse pressure? A) 120/76 mmHg B) 108/88 mmHg C) 210/138 mmHg D) 88/58 mmHg Answer: B Diff: 1 Page Ref: 198 Objective: 8-17 13 Copyright © 2018 Pearson Education, Inc.

57) When a patient has a narrowed pulse pressure, what is occurring? A) The heart rate is decreasing B) The veins and arteries are dilating C) The blood vessels are constricting D) The blood is being pumped more quickly Answer: C Diff: 2 Page Ref: 198 Objective: 8-17 58) What is the protective mechanism underlying a narrowed pulse pressure? A) Increase the amount of oxygen entering the lungs B) Stop the loss of blood internally or externally C) Increase the oxygen-carrying capacity of the red blood cells D) Maintain an adequate blood pressure for perfusion Answer: D Diff: 2 Page Ref: 198 Objective: 8-17 59) The body will initially compensate for a decrease in cardiac output by: A) Increasing systemic vascular resistance B) Increasing the respiratory rate C) Dilating to arteries, veins, and capillaries D) Decreasing the heart rate Answer: A Diff: 1 Page Ref: 198 Objective: 8-17 60) When a healthy person has a sudden increase in blood pressure, what will occur next? A) Chemoreceptors will increase the rate and depth of breathing B) Baroreceptors will directly stimulate the parasympathetic nervous system C) The heart rate will increase and blood vessels will constrict D) Baroreceptors will signal the brain to decrease the heart rate Answer: D Diff: 2 Page Ref: 199-200 Objective: 8-19 61) What is baroreceptors' role in the body? A) Monitor the heart rate B) Ensure adequate oxygenation of the red blood cells C) Monitor the blood pressure D) Stimulate the production of red blood cells Answer: C Diff: 1 Page Ref: 199 Objective: 8-19

14 Copyright © 2018 Pearson Education, Inc.

62) A drop in blood pressure below a critical threshold is a threat to the body because it directly impairs: A) Anaerobic metabolism B) Cellular perfusion C) Airway patency D) The sympathetic nervous system Answer: B Diff: 2 Page Ref: 199 Objective: 8-19 63) A patient with cancer is receiving chemotherapy. As a side effect of the treatment, her white blood cell count is critically low. Given this situation, which sign or symptom should the EMT find particularly concerning? A) History of diabetes B) Oral temperature of 102.7°F C) Complaint of weakness D) Blood pressure of 102/64 mmHg Answer: B Diff: 3 Page Ref: 193 Objective: 8-14 64) A patient with liver disease has a low platelet count. Which additional finding would the EMT directly correlate to this condition? A) Pale skin color B) SpO2 of 90% C) Excessive bruising to arms D) Heart rate of 92 beats/min Answer: C Diff: 3 Page Ref: 193 Objective: 8-14 65) You are caring for a 66-year-old male patient who is severely dehydrated. How does severe dehydration affect the cardiovascular system? A) It reduces the hemoglobin available to carry oxygen B) It impairs the ability of oxygen to diffuse from the lungs to the blood C) It slows the production of white blood cells to fight the infection D) It decreases the blood volume available for circulation Answer: D Diff: 3 Page Ref: 193 Objective: 8-15

15 Copyright © 2018 Pearson Education, Inc.

66) Which condition would directly compromise the average patient's cardiac output? A) Heart rate of 190 beats/min B) Circulatory volume of 5 liters C) Increased number of circulating WBCs D) Blood pressure of 118/52 mmHg Answer: A Diff: 3 Page Ref: 196 Objective: 8-16 67) You and the critical care transport team are taking a critically ill patient to another hospital. Among many IV infusions and monitors, the patient is also on a ventilator. You note that the FDO2 is set at 0.50. What does this mean? A) Fifty percent of the patient's respirations are provided by the ventilator B) Ventilations are provided when the patient breathes fewer than 50 times per minute C) Oxygen is not provided by the ventilator until the patient's pulse oximetry drops below 50% D) The provided oxygen concentration is set at 50 percent Answer: D Diff: 2 Page Ref: 181 Objective: 8-4 68) A patient with asthma is extremely short of breath and hypoxic. Related to the ventilation/perfusion ratio (V/Q), the EMT would recognize the problem as related to: A) Inadequate oxygen in the alveoli B) Decreased cardiac output to circulate oxygen C) Inadequate oxygen in the ambient air D) Decreased blood volume to carry oxygen Answer: A Diff: 3 Page Ref: 191 Objective: 8-11 69) A patient has been shot three times in the abdomen and has massive internal hemorrhage and blood loss. Which treatment will give this patient the best chance at survival? A) High-concentration oxygen B) Stopping the bleeding C) Positive pressure ventilation D) Paramedic intervention for pain management Answer: B Diff: 3 Page Ref: 191 Objective: 8-4

16 Copyright © 2018 Pearson Education, Inc.

70) A patient with a severe infection is very sick. Circulating toxins from bacteria in his blood have decreased his systemic vascular resistance (SVR). Which blood pressure change would the EMT correlate to this condition? A) Blood pressure of 174/126 mmHg B) Diastolic blood pressure greater than 200 mmHg C) Blood pressure of 64/26 mmHg D) Systolic blood pressure of 160 mmHg Answer: C Diff: 3 Page Ref: 197 Objective: 8-17 71) What blood pressure change as most likely to cause left ventricular failure? A) Chronically elevated systolic blood pressure between 140 and 160 mmHg B) Chronically decreased diastolic blood pressure between 30 and 50 mmHg C) Chronically decreased systolic blood pressure between 60 and 80 mmHg D) Chronically elevated diastolic pressure between 120 and 140 mmHg Answer: D Diff: 3 Page Ref: 195 Objective: 8-17 72) For a patient who is acutely bleeding, what is the immediate response of the human body? A) Vasoconstriction caused by the parasympathetic nervous system B) Increased production of red blood cells and hemoglobin C) Vasoconstriction caused by the sympathetic nervous system D) Increased pulse pressure to promote better circulation of oxygen Answer: C Diff: 3 Page Ref: 200 Objective: 8-18 73) A 35-year-old male patient is lethargic and dehydrated after working at a construction site for 12 hours on a very hot day. You obtain the following vital signs: pulse, 136; respirations, 22 breaths/min; blood pressure, 102/88 mmHg; and SpO2, 100% with supplemental oxygen. As a knowledgeable EMT, you would recognize: A) Widened pulse pressure to circulate more blood B) Elevated heart rate to increase cardiac output C) Rapid respirations to decrease CO2 loss D) Low blood pressure to conserve the body's energy Answer: B Diff: 3 Page Ref: 200 Objective: 8-16

17 Copyright © 2018 Pearson Education, Inc.

74) A confused and lethargic 24-year-old male patient has intentionally overdosed on a narcotic medication. His vital signs are as follows: pulse, 36; respirations, 10 breaths/min; blood pressure, 50/20 mmHg; and SpO2, 88% with 15 liters per minute of oxygen applied by Emergency Medical Responders. What is the greatest danger to this patient's well-being? A) Decreased cellular perfusion B) Altered mental status C) Increased sympathetic nervous system stimulation D) Left ventricular heart failure Answer: A Diff: 3 Page Ref: 181 Objective: 8-4 75) What is the result of an opening developing that allows air into the space between the visceral and parietal pleura of the thorax? A) Progressive collapse of the lung B) Hyperventilation C) Hypocarbia D) Left ventricular heart failure Answer: A Diff: 3 Page Ref: 185 Objective: 8-8 76) Which process of ventilation is correctly adhering to Boyle's law as it relates to ventilation? A) When the diaphragm contracts, it creates a negative intrathoracic pressure causing air to enter the lungs B) When the diaphragm relaxes, it creates a negative intrathoracic pressure causing air to enter the lungs C) When the diaphragm contracts, it creates a positive intrathoracic pressure causing air to enter the lungs D) When the diaphragm relaxes, it creates a positive intrathoracic pressure causing air to enter the airways Answer: A Diff: 2 Page Ref: 183-184 Objective: 8-6 77) Which gas law is disrupted when a patient sustains a spinal cord injury resulting in paralysis to the diaphragm and intercostal muscles? A) Charles' law B) Boyle's law C) Henry's law D) Dalton's law Answer: A Diff: 2 Page Ref: 183-184 Objective: 8-6

18 Copyright © 2018 Pearson Education, Inc.

78) A patient has sustained a gunshot injury to his right thorax. When you listen to his breath sounds, they are diminished over the right thorax. What is the most logical explanation for this finding? A) The gunshot wound has damaged the brainstem, causing the patient to stop breathing B) The pressure between the visceral pleura and the lung tissue has become higher than the atmospheric pressure C) The negative pressure between the pleural linings has been lost, causing the lung tissue to collapse D) The pressure between the parietal pleura and the rib cage has become lower than the atmospheric pressure Answer: C Diff: 3 Page Ref: 185 Objective: 8-8 79) Why would a drop in the oxygen content of the blood stream cause an increase in the patient's blood pressure? A) Parasympathetic stimulation of the pulmonary system improves oxygen on-loading to the red blood cells passing through the alveoli B) Sympathetic stimulation of the cardiovascular system improves oxygen delivery to the brain and tissues C) Vasoconstriction caused by the parasympathetic nervous system causes oxygenated blood to be shunted from the peripheral capillary beds and to the brain D) Vasodilation will allow more blood flow to reach the lungs, thereby increasing the oxygen content of the blood and providing for better perfusion to the brain and body Answer: B Diff: 3 Page Ref: 201 Objective: 8-18 80) A patient has a pulmonary condition known as asthma that results in significant bronchoconstriction. If this patient is found to be hypoxic during an asthma attack, which type of disturbance is most likely causing the hypoxia? A) Neurological disturbance B) Perfusion disturbance C) Myocardial disturbance D) Ventilation disturbance Answer: D Diff: 2 Page Ref: 190-191 Objective: 8-20

19 Copyright © 2018 Pearson Education, Inc.

Prehospital Emergency Care, 11e (Mistovich et al.) Chapter 9 Life Span Development 1) A mother has called 911 for medical assistance. She states that her 3-month-old son has a rectal temperature of 99.7°F and is not feeding. Which statement would be appropriate when talking with the mother about the infant's temperature? A) "You are right, he does have a fever. Have you given him any Tylenol?" B) "Fever in babies is always a concern. Has he ever had a febrile seizure?" C) "Why don't we get him undressed so we can cool him down?" D) "Given his age, a temperature of 99.7°F is normal." Answer: D Diff: 2 Page Ref: 205 Objective: 9-2 2) A young new mother has called 911 because her baby will not stop crying. On scene, you find a 6-month-old baby who alternates between crying and sleep. Which statement to the mother would be most appropriate given the infant's crying? A) "Is there a possibility of child abuse?" B) "Does this cry seem different to you?" C) "Has the baby had all his immunizations?" D) "It is normal for babies to cry and he seems to be sleeping well." Answer: B Diff: 2 Page Ref: 207 Objective: 9-3 3) When assessing a sick 3-week-old baby, which finding would be most concerning? A) Fontanelle level with the skull B) Soft spot to the top of the head C) Mucus occluding both nares D) Rectal temperature of 99.4°F Answer: C Diff: 2 Page Ref: 205 Objective: 9-2 4) The mother of a 9-month-old baby informs you that her child has yet to make any sounds that resemble words or speech. She asks you if this is normal. What is your best response? A) "Babies typically begin making words at 6 months. It may be a good idea to call your pediatrician for additional information." B) "It usually takes 10 to 12 months for babies to begin talking. Give it time. This is normal." C) "To be honest, all babies develop at their own pace; this is likely fine." D) "Why don't you take him to your pediatrician to find out what is wrong." Answer: A Diff: 3 Page Ref: 207 Objective: 9-3

1 Copyright © 2018 Pearson Education, Inc.

5) When assessing a 5-month-old baby girl, you note that she cries when her mother holds her, but is quiet and at peace when her father takes her. Assessment reveals no outward problems. Based on your knowledge of psychosocial development, what is a likely explanation? A) The father is most likely the baby's primary caregiver B) Female babies tend to bond more closely with their father C) The baby was most likely breast-fed by the mother D) There is a problem with the baby's normal development Answer: A Diff: 3 Page Ref: 207 Objective: 9-3 6) You must assess a crying 7-month-old baby who is vomiting. To best assess this patient, you should: A) Wait for the baby to stop crying B) Allow the mother to hold the baby C) Secure the baby to the cot and then assess her D) Have the mother leave the room Answer: B Diff: 1 Page Ref: 207 Objective: 9-3 7) You are assessing a 4-year-old child with a fever and cough. The child is very calm and receptive to your presence. Given this, which action would be most appropriate during your ongoing assessment? A) Ask the mother to leave until the assessment is complete B) Take the child to the ambulance for assessment C) Have the mother remain with the child for the assessment D) Refrain from touching the patient until in the ambulance Answer: C Diff: 1 Page Ref: 209 Objective: 9-4 8) A 7-year-old child has fallen and exhibits deformity of his left wrist. He will not cooperate with you as you attempt a closer examination of the wrist. Which statement would be appropriate given the patient's age and behavior? A) "This will not hurt you at all. Please let me see your wrist." B) "I know that you are scared, but I need to see your hand." C) "The doctor is going to get upset with me if you do not let me see what is wrong." D) "You can trust me. If I can see your wrist, I will get you kid's stickers at the hospital." Answer: B Diff: 2 Page Ref: 210 Objective: 9-5

2 Copyright © 2018 Pearson Education, Inc.

9) You are en route to a home for an 8-year-old child with "noisy" breathing and a cough. As you respond, which piece of advice would you give your partner, a new EMT? A) "We will need to be careful. Children of this age group tend to throw tantrums, and we could get hurt if we are not careful." B) "The scene probably will be chaotic, so let's just get the patient out to the ambulance and assess him there." C) "Although most children of this age are trusting of authority, let's approach slowly and make the scene work for us." D) "Most children in this age group have a short attention span, so we need to assess him as quickly as possible before he gets angry or upset." Answer: C Diff: 2 Page Ref: 210 Objective: 9-5 10) When assessing and treating a 15-year-old female patient for abdominal pain, the EMT must remember that: A) The patient probably will be scared and most likely to answer questions when her mother or father is present B) There is no need to ask about sexual activity since the complaint is abdominal pain C) If the patient has experienced menstruation, the EMT should leave the assessment to the emergency physician at the hospital D) A parent's or guardian's consent should be obtained prior to assessment and treatment Answer: D Diff: 2 Page Ref: 210 Objective: 9-6 11) You have received a grant to promote healthy living habits among the 20- to 40-year-old age group. Given the most common cause of death within this age group, which topic should be the focus of your program? A) Safe driving B) Cancer screening C) Safe sex practices D) Healthy heart living Answer: A Diff: 1 Page Ref: 211 Objective: 9-7 12) A common concern about the well-being of a female patient in the 41- to 60-year-old age group is: A) Cancer B) Drug overdose C) Accidental death D) Low cholesterol levels Answer: A Diff: 1 Page Ref: 212 Objective: 9-8 3 Copyright © 2018 Pearson Education, Inc.

13) You have been asked to speak to a local EMT class about caring for patients older than age 60 years. When presenting your material, which point would you emphasize? A) All persons older than age 60 have chronic health problems B) Older adults often do not want to give up their independent lifestyle even if medical conditions necessitate it C) Most persons older than age 70 are in assisted living facilities or nursing homes D) Most persons older than age 60 now reside with their grown children Answer: B Diff: 2 Page Ref: 213 Objective: 9-9 14) After you transfer care of a 77-year-old female patient to the ED nurse, the patient informs the nurse that she has a history of heart problems and is having chest pain. On scene and throughout transport, the patient denied any complaint, despite your repeated inquiry. This has greatly upset your partner since he feels the patient made him look foolish in front of the emergency department staff. Based on your knowledge of the patient's age group, which explanation would you give your partner? A) "It is very common for patients in this age group not to tell you everything because most have some form of early dementia." B) "In the older population, most do not trust young people with personal information, such as their medical history." C) "This happens very frequently with people in this age group, so you must accept it and not get upset." D) "Many times persons in this age group will not be forthcoming with all that is going on because they fear losing their independence." Answer: D Diff: 2 Page Ref: 213 Objective: 9-10 15) You have been called for a 2-week-old male patient whose mother feels as though he is "sick." When assessing this patient and obtaining a history from the mother, which finding would be most concerning? A) He has lost weight since he was born B) The fontanelles are lower than the level of the skull C) The father states that the baby feeds every 2 to 3 hours D) You observe that the baby seems to breathe only through his nose Answer: B Diff: 2 Page Ref: 207 Objective: 9-2

4 Copyright © 2018 Pearson Education, Inc.

16) An EMT would recognize and categorize a 15-year-old female patient as a(n): A) Adolescent B) School-age child C) Young adult D) Pubescent Answer: A Diff: 1 Page Ref: 210 Objective: 9-1 17) When assessing a neonate, which finding would deserve closer investigation? A) Intermittent crying B) Temperature of 99.6°F C) Heart rate of 82 beats per minute D) Respirations of 40 breaths/min Answer: C Diff: 2 Page Ref: 206 Objective: 9-2 18) Which statement made by a 49-year-old female patient describes a situation common to those in this life stage? A) "I have never felt better than I do now. I am in the best shape of my life!" B) "I do not have time to see the doctor about my blood pressure. I am too busy taking care of my parents." C) "Life is very enjoyable at this stage since I have managed to accomplish all the goals I set for myself." D) "I am very careful when I drive since it is well known that car accidents pose the greatest risk to people my age." Answer: B Diff: 3 Page Ref: 212 Objective: 9-8 19) The average years of life remaining based on an individual's date of birth is called: A) Life span development B) Maximum life span C) Life expectancy D) Physiological expectancy Answer: C Diff: 1 Page Ref: 212 Objective: 9-1

5 Copyright © 2018 Pearson Education, Inc.

20) Five minutes have passed since you assisted in the delivery of a full-term baby boy. Based on your knowledge of physiological changes in the neonate, which finding would be most concerning? A) Heart rate of 88 beats/min B) Respiratory drop from 52 to 38 breaths/min C) Rectal temperature of 99.8°F D) Systolic blood pressure of 70 mmHg Answer: A Diff: 3 Page Ref: 206 Objective: 9-3 21) You are transporting a 3-week-old male patient who has a fever and cough to the hospital. When making your radio report, you would advise the emergency department that you are bringing in a(n): A) Infant B) Baby C) Toddler D) Neonate Answer: D Diff: 1 Page Ref: 205 Objective: 9-1 22) You are caring for an 18-day-old boy with inadequate respirations. Which instruction would you provide to your new partner as she is ventilating him with the bag-valve mask? A) "Squeeze the bag just enough so that you get a nice rise and fall of his chest." B) "Do not be afraid to give a little extra pressure since his lungs are not fully developed." C) "Since he is so small and does not need a lot of oxygen, just give 4 breaths per minute." D) "Let's breathe for him until his heart rate gets above 60; then we can stop." Answer: A Diff: 2 Page Ref: 205-206 Objective: 9-10 23) Which of these patients would be categorized as a toddler? A) 6-month-old female B) 48-month-old male C) 4½-year-old female D) 28-month-old male Answer: D Diff: 1 Page Ref: 208 Objective: 9-1

6 Copyright © 2018 Pearson Education, Inc.

24) You are caring for a 6-month-old male patient who is fussy and crying, and who has a fever. Which action would be most appropriate? A) Talk to the parents rather than the baby, since a stranger's voice is upsetting B) Wait for the baby to stop crying so that you can properly assess him C) Keep the parents calm even though you are actually caring for the baby D) Distract the baby with a small toy or ball so that you can assess him Answer: C Diff: 2 Page Ref: 207-208 Objective: 9-10 25) The systolic blood pressure of a 4-year-old female patient is 110 mmHg. Her mother is concerned because last year at the pediatrician's office the systolic blood pressure was 92 mmHg and now it is higher. How would you respond? A) "Her blood pressure is most likely increased because she is sick." B) "This is normal; not only will the blood pressure increase with age, but so will her heart rate." C) "Her blood pressure will increase naturally. This is normal and healthy." D) "I agree; that is a little bit of a jump in blood pressure. It very well could be the reason she is not feeling well." Answer: C Diff: 3 Page Ref: 208 Objective: 9-4 26) A 3-year-old child is very fearful when you listen to his breathing and lungs with the stethoscope. Your best course of action would be to: A) Forego listening to the boy's breath sounds B) Listen to the breath sounds regardless of the boy's reaction C) Have the boy watch as you listen to his father's lungs D) Tell the patient that you must listen to his lungs and then do it Answer: C Diff: 2 Page Ref: 209 Objective: 9-10 27) Puberty is best described as: A) Maturation of the sexual organs B) A milestone developmental stage in school-age children C) A two- to three-year growth spurt in adolescents D) A developmental stage that occurs earlier in males than in females Answer: A Diff: 1 Page Ref: 210 Objective: 9-6

7 Copyright © 2018 Pearson Education, Inc.

28) A 16-year-old female patient has a history of bulimia. As a knowledgeable EMT, you would realize that bulimia is: A) A normal stage of adolescence and often is self-limiting B) Probably rooted in a concern related to her body image C) Most often related to a developing taste and distaste for certain foods D) Driven by a drive for maximum health and fitness Answer: B Diff: 2 Page Ref: 211 Objective: 9-10 29) A 16-year-old male was killed after driving at excessive speeds and then losing control of his car. He was not wearing a seat belt. In an attempt to understand why such a tragedy would occur, you would recognize that: A) During this stage of life, the "right versus wrong" part of the brain is not fully developed B) Adolescent boys have a drive to compete with their peers C) Within the adolescent period, there exists a strong desire to show independence and selfsufficiency D) Common to this age group is a perceived sense of strength and invulnerability Answer: D Diff: 3 Page Ref: 211 Objective: 9-6 30) During which stage of life do most chronic illnesses typically become evident? A) Early adulthood B) Middle adulthood C) Late adulthood D) Adolescence Answer: B Diff: 1 Page Ref: 212 Objective: 9-10 31) A female patient informs you that she has already gone through menopause. You would realize that: A) Her menstrual periods are irregular and she cannot have a baby B) She has stopped outright menstrual periods but can still become pregnant C) She will still have menstrual periods but is incapable of pregnancy D) She has stopped menstrual periods and can no longer become pregnant Answer: D Diff: 1 Page Ref: 212 Objective: 9-8

8 Copyright © 2018 Pearson Education, Inc.

32) You have been dispatched to a residence for an 82-year-old female with unknown medical problems. En route to the scene, your partner states, "At 82 years old, she probably has a ton of medical problems." Your response would be: A) "You are probably right since more than half of persons older than 60 years have heart disease." B) "I am guessing she does not, given her age and the fact she is not in a nursing home." C) "As we were taught in school, it is middle adulthood where people have the most medical problems." D) "Maybe not, since in late adulthood there is such a wide variety of health status." Answer: D Diff: 3 Page Ref: 213 Objective: 9-9 33) Which statement is true regarding the maximum life span? A) Almost all persons die before reaching the maximum life span B) The maximum life span is different for every person and takes into account health status and occupation C) The maximum life span is approximately 100 years of age D) The maximum life span is the average years a person is expected to live based on health status Answer: A Diff: 1 Page Ref: 212 Objective: 9-1 34) You are at the residence of an alert and oriented 87-year-old female patient who is very short of breath but is refusing to go to the hospital. Instead, she wants you to treat her at home and then leave when she feels better. As an EMT knowledgeable of the characteristics and behaviors of this age group, you would become suspicious that her request is based on the fact that she: A) May have Alzheimer's dementia and has yet to be diagnosed B) Is concerned that she does not have the money to pay for the ambulance transport C) Is concerned that she may be giving up the independence of living in her home D) Does not trust those who are younger in age to take care of her Answer: C Diff: 3 Page Ref: 213 Objective: 9-9 35) Due to the body systems operating at optimal level and because typically individuals in this age bracket are in excellent physical condition, risky behavior and accidents are the leading causes of death in which stage of development? A) Preschool-age children B) Adolescence C) Early adulthood D) Late adulthood Answer: C Diff: 1 Page Ref: 211 Objective: 9-7 9 Copyright © 2018 Pearson Education, Inc.

Prehospital Emergency Care, 11e (Mistovich et al.) Chapter 10 Airway Management, Artificial Ventilation, and Oxygenation 1) Which of these sequences best describes the path that oxygen must travel to get into the cells of the body? A) Nasopharynx, oropharynx, carina, trachea, alveoli B) Oropharynx, larynx, trachea, carina, bronchioles C) Nose, pharynx, esophagus, bronchi, alveoli D) Pharynx, larynx, trachea, alveoli, bronchioles Answer: B Diff: 1 Page Ref: 219-220 Objective: 10-3 2) You are assessing an unresponsive patient who overdosed on a narcotic pain medication. Given the situation, what is the greatest concern regarding potential airway occlusion? A) Spasm of the epiglottis B) Swelling of the carina C) Collapse of the bronchi D) Relaxation of the tongue Answer: D Diff: 1 Page Ref: 228 Objective: 10-6 3) Which of these patients should the EMT recognize as having a potential occlusion of the upper airway? A) A 3-year-old male with a fever and swelling of the larynx B) A 45-year-old female with spasm of the bronchioles C) A 61-year-old male with a piece of fruit stuck in his trachea D) A 78-year-old female with a large mucus plug in her bronchi Answer: A Diff: 1 Page Ref: 220 Objective: 10-5 4) Which statement indicates that the EMT has an accurate understanding of the bronchioles? A) "They are large structures that pass oxygen from the lungs directly into the bloodstream." B) "They are small air sacs located just before the alveoli." C) "They are small passages located in the lower airway that have smooth muscle surrounding them." D) "They are large airways located after the carina and are supported by cartilage." Answer: C Diff: 2 Page Ref: 224 Objective: 10-1

1 Copyright © 2018 Pearson Education, Inc.

5) Assessment findings on an elderly patient with congestive heart failure indicate that her failing heart is causing fluid to collect and fill her alveoli. Based on this pathophysiology, for which pulmonary complication should the EMT treat the patient? A) Decreased blood pressure B) Collapse of the bronchi C) Swelling of the tongue D) Decreased gas exchange Answer: D Diff: 3 Page Ref: 226 Objective: 10-4 6) If an injured patient has lost all use of his diaphragm, the EMT should recognize that the patient: A) Has lost a major portion of his ability to breathe B) Is breathing adequately but most likely will complain of chest pain C) Requires immediate cardiopulmonary resuscitation and frequent suctioning D) Requires supplemental oxygen with a nonrebreather face mask Answer: A Diff: 2 Page Ref: 221 Objective: 10-3 7) The EMT exhibits an accurate understanding of breathing when he states: A) "When a person inhales, carbon dioxide is drawn into the lungs, and when he exhales, oxygen leaves the body." B) "Respiration is best described as the process of moving air into and out of the body." C) "Inhalation requires energy, while exhalation for most patients is a passive process." D) "In between breaths, the epiglottis protectively closes over the trachea." Answer: C Diff: 2 Page Ref: 221-222 Objective: 10-3 8) When a person's diaphragm contracts and the intercostal muscles pull the ribs upward, what will occur relative to the exchange of air and/or gases? A) Oxygen will leave the body B) Air will flow into the lungs C) Pressure inside the chest will increase D) Carbon dioxide will be exhaled Answer: B Diff: 2 Page Ref: 221 Objective: 10-3

2 Copyright © 2018 Pearson Education, Inc.

9) Which of these patients is most likely to lose neurologic control and regulation of the respiratory system? A) A 31-year-old patient with a severe head injury B) A 44-year-old patient who smokes and is nauseated C) A 56-year-old patient with heart problems and chest pain D) A 61-year-old patient diagnosed with diabetes Answer: A Diff: 2 Page Ref: 222 Objective: 10-5 10) When completing a primary assessment, what would be a good indication that the patient has an open airway? A) The patient is conscious B) The breath sounds are clear C) The respiratory rate is normal D) The patient speaks with ease Answer: D Diff: 2 Page Ref: 228, Table 10-1 Objective: 10-4 11) You have been called to a long-term nursing care facility for a 77-year-old female patient with Parkinson's disease. According to the nursing staff, she is more confused than normal and is to be transported to the emergency department for evaluation. Since the patient neither understands nor can answer any of your questions, which sign provides the best evidence that her breathing is adequate? A) Respiratory rate of 18 breaths/min B) Pulse oximeter reading at 98% C) Clear bronchiole breath sounds D) Strong radial pulse Answer: B Diff: 2 Page Ref: 269 Objective: 10-4 12) Your medical director is holding an in-service program focusing on the prehospital management of adult patients with respiratory complaints. He asks the group to explain how they can determine if the patient is breathing adequately. Which response is most appropriate? A) "A patient is breathing adequately if the depth of breathing is shallow, but the rate is increased above normal." B) "If the pulse oximeter reads 94% or greater, then the patient is breathing adequately." C) "For an adult patient to be breathing adequately, the rate should be within 8 to 24 breaths/min with a full chest rise with each breath." D) "The EMT can assume breathing is adequate if the patient has an open airway and a normal respiratory rate." Answer: C Diff: 3 Page Ref: 242 Objective: 10-7 3 Copyright © 2018 Pearson Education, Inc.

13) You have been called for a patient who has overdosed on drugs and alcohol. The patient is responsive to painful stimuli, has shallow respirations and is breathing at a rate of 8 times per minute, but exhibits no cyanosis. What is the priority treatment? A) Adequate breathing; administer oxygen at 15 lpm through a nonrebreather face mask B) Inadequate respirations; start positive pressure ventilation with supplemental oxygen C) Adequate breathing; initiate positive pressure ventilation with supplemental oxygen D) Inadequate breathing; administer oxygen at 15 lpm through a nonrebreather face mask Answer: B Diff: 2 Page Ref: 242 Objective: 10-5 14) Which of these patients should the EMT suspect as being mildly hypoxic? A) A 24-year-old female breathing 8 times per minute with cyanosis of the lips B) A 36-year-old female with asthma, breathing 22 times per minute, with cyanosis of the fingertips C) A 44-year-old male breathing 28 times per minute who is confused and has an SpO2 of 97% D) An 18-year-old male who is restless, has shallow respirations and is breathing at a rate of 16 times per minute, with no cyanosis Answer: D Diff: 3 Page Ref: 223 Objective: 10-5 15) The EMT believes that a patient is mildly hypoxic based on findings from the primary assessment. Which other sign could the EMT use to confirm his suspicion of mild hypoxia? A) Cyanosis B) Restlessness C) Responsive to verbal stimuli D) Sleepy appearance Answer: B Diff: 2 Page Ref: 223 Objective: 10-4 16) Which of these patients would benefit from the head-tilt, chin-lift manual airway maneuver as opposed to any other initial airway management technique? A) A 39-year-old male who just had a seizure and has snoring respirations B) A 45-year-old female who is alert and vomiting blood C) A 67-year-old female who fell down a flight of stairs and is unresponsive D) An 85-year-old female who hit her head on a tree limb and is now responsive to painful stimuli Answer: A Diff: 2 Page Ref: 229 Objective: 10-7

4 Copyright © 2018 Pearson Education, Inc.

17) A 2-year-old boy who just seized must have his airway opened. When performing the headtilt, chin-lift airway maneuver, the EMT must remember: A) The head must be flexed forward B) The head must be hyperextended C) A towel roll should be placed under the head D) If necessary, the head should be only slightly extended Answer: D Diff: 2 Page Ref: 230-231 Objective: 10-7 18) You have arrived on the scene of an assault in which several people were injured. You are directed to assist another crew with a patient who has been shot once. As you approach, you are told that the patient has slow and irregular breathing that is inadequate. You observe the patient being ventilated with a bag-valve mask while another EMT maintains her airway with the jawthrust maneuver. Based on your observation, where might you expect that this patient was shot? A) Head B) Lower abdomen C) Chest D) Pelvis Answer: A Diff: 3 Page Ref: 231 Objective: 10-7 19) Why is the jaw-thrust maneuver indicated for a patient with a possible spinal injury? A) It is a permanent airway intervention that does not require a mechanical airway if performed correctly B) It manipulates the head and cervical spine less than the head-tilt, chin-lift maneuver C) It is less painful for the patient since he is already in pain from the injury D) It is the EMT's preference as to whether to use the head-tilt, chin-lift maneuver or the jawthrust maneuver Answer: B Diff: 2 Page Ref: 231 Objective: 10-7 20) Which statement regarding airway management in the trauma patient is correct? A) The jaw-thrust maneuver is useful in the trauma patient, but only if he or she is complaining of head or neck pain B) Research has shown that the jaw-thrust maneuver is easier to perform and, therefore, is indicated for the more critical trauma patient C) Performing the head-tilt, chin-lift maneuver on a patient with a possible spinal injury could further injure or paralyze the patient D) Trauma patients should always have their airway opened with the jaw-thrust maneuver, whereas medical patients should receive the head-tilt, chin-lift only Answer: C Diff: 2 Page Ref: 231 Objective: 10-7 5 Copyright © 2018 Pearson Education, Inc.

21) The EMT should use the jaw-thrust maneuver to open the airway for the patient with which type of presentation? A) Overdose with snoring respirations B) Stroke with gurgling respirations C) Cardiac arrest in bed D) Unresponsive after falling from a porch Answer: D Diff: 1 Page Ref: 231 Objective: 10-7 22) The EMT is correctly performing the jaw-thrust airway maneuver when she: A) Places her hands on the side of the patient's head and lifts the jaw upward B) Maintains the patient's head in a neutral position and slightly tilts the head backward C) Opens the patient's airway by slightly extending the head and thrusting the jaw upward D) Places one hand on the patient's forehead and lifts the jaw upward with her other hand Answer: A Diff: 2 Page Ref: 231 Objective: 10-1 23) You arrive at a residence for a male patient who is unresponsive. As you enter the room, you observe an obese man lying on a recliner. His eyes are closed, and he has gurgling respirations. While your partner is quickly performing a primary assessment, you should immediately prepare to: A) Auscultate breath sounds B) Suction the airway C) Administer high-concentration oxygen D) Insert an oropharyngeal airway Answer: B Diff: 2 Page Ref: 233 Objective: 10-6 24) While you are transporting a patient with altered mental status, he suddenly vomits partially digested food along with large blood clots. The vomitus is too big for the rigid suction catheter. The EMT should immediately place the patient on his side and: A) Use a soft suction catheter B) Administer abdominal thrusts C) Perform a finger sweep D) Encourage the patient to cough Answer: C Diff: 2 Page Ref: 233 Objective: 10-8

6 Copyright © 2018 Pearson Education, Inc.

25) You must suction the oropharynx of a male patient who has a decreased ability to swallow secondary to a previous stroke. He also has a history of smoking and lung cancer. What would be the most appropriate suction catheter for use with this patient? A) Soft B) Texas C) Yankauer D) French Answer: C Diff: 2 Page Ref: 234 Objective: 10-9 26) A young child with multiple physical and developmental abnormalities has a severe nosebleed. You must suction his nasopharynx to decrease the amount of blood he is swallowing. Which action should be part of this care? A) Insert a soft catheter into the nose the same length as the distance from the patient's nose to the tip of his ear B) Place the patient on his side and gently insert a rigid-tip catheter 1 inch into each nostril, starting with the right nostril C) Insert the soft catheter through the nose past the base of the tongue D) Irrigate the nostrils with water to thin and loosen the blood prior to suctioning with a rigid catheter Answer: A Diff: 2 Page Ref: 235 Objective: 10-9 27) You are caring for a 46-year-old male patient who is unresponsive. Which statement made by your partner indicates the need to immediately stop suctioning the patient's airway and resume positive pressure ventilation? A) "He is starting to have gurgling respirations." B) "His heart rate is 48 beats per minute." C) "You have been suctioning now for 10 seconds." D) "His pulse oximeter has dropped 2 percentage points." Answer: B Diff: 2 Page Ref: 235 Objective: 10-9 28) Which statement made by an EMT shows that she understands how to suction patients in a field setting? A) "When caring for a young child, I suction for a minimum of 5 seconds." B) "To thoroughly suction a patient, I insert the rigid-tip catheter past the base of the tongue and into the pharynx." C) "It is helpful to pour sterile water into the patient's mouth to further liquefy vomit and make it easier to suction." D) "I activate the suction device only when I am withdrawing the suction catheter." Answer: D Diff: 2 Page Ref: 235 Objective: 10-9 7 Copyright © 2018 Pearson Education, Inc.

29) You must suction frothy secretions from the mouth and pharynx of a 31-year-old male patient whose respiration rate is 4 breaths/min. Which process to do so is most appropriate given the patient's condition? A) Suction for no more than 15 seconds at a time, followed by 2 minutes of positive pressure ventilation B) Forego suctioning and continue to provide positive pressure ventilation, as frothy secretions rarely complicate the airway C) Place the patient into a side-lying position and alternate 30 seconds of suctioning and ventilation D) Use a soft-tip catheter and suction for no more than 30 seconds before providing additional oxygen with a nonrebreather face mask Answer: A Diff: 2 Page Ref: 235 Objective: 10-9 30) Which statement made by an EMT with whom you are working for the first time shows he understands the use of the pocket mask? A) "Although the pocket mask is very effective at ventilating a patient, the EMT is directly exposed to secretions from the patient." B) "In comparison to the bag-valve mask, the pocket mask can deliver just as good, if not better, tidal volume." C) "When using the pocket mask to ventilate a patient, the EMT must make sure that the reservoir bag is in place." D) "The EMT must remove the oral airway and replace it with a nasal airway when ventilating with a pocket mask." Answer: B Diff: 2 Page Ref: 251 Objective: 10-14 31) You have opened an apneic patient's airway using the head-tilt, chin-lift maneuver and are ready to ventilate with a pocket mask. The first ventilation you attempt is unsuccessful. You should immediately: A) Reposition the patient's head B) Use a jaw-thrust maneuver C) Ventilate over 3 seconds D) Provide five abdominal thrusts Answer: A Diff: 2 Page Ref: 252 Objective: 10-14

8 Copyright © 2018 Pearson Education, Inc.

32) When using a pocket mask to ventilate an unresponsive adult patient, the EMT must always: A) Deliver 1 breath every 3 seconds B) Maintain the head-tilt, chin-lift maneuver or jaw-thrust maneuver C) Ventilate at twice the patient's normal rate of breathing D) Provide each breath over 3 seconds Answer: B Diff: 2 Page Ref: 252 Objective: 10-14 33) You are assessing a young male patient who was ejected from an automobile at a high rate of speed. He responds to painful stimuli by posturing and displays agonal breathing. His pulse is weak and thready, and his skin is cool and diaphoretic. When addressing this patient's airway and breathing, the EMT must immediately: A) Administer oxygen at 15 lpm with a nonrebreather face mask B) Perform the jaw-thrust maneuver and begin artificial ventilations with the bag-valve mask C) Immobilize the patient on a long spine board, and then start positive pressure ventilation D) Open the patient's airway using the head-tilt, chin-lift maneuver and begin ventilations with the bag-valve mask attached to supplemental oxygen Answer: B Diff: 2 Page Ref: 256 Objective: 10-14 34) Which configuration on the bag-valve mask (BVM) will enable the EMT to deliver the highest concentration of oxygen to the patient? A) BVM with room-air oxygen being used by two EMTs B) BVM attached to oxygen at 15 lpm C) BVM with oxygen reservoir attached, receiving oxygen at 15 lpm D) BVM with oxygen reservoir attached Answer: C Diff: 2 Page Ref: 253 Objective: 10-14 35) Which statement about the bag-valve mask (BVM) is true? A) Oxygen must be attached so that the squeezable bag will reinflate B) A 15/22 mm adapter allows the BVM to be attached to an advanced airway such as the endotracheal tube C) The nonrebreather valve prevents the EMT from being exposed to possible pathogens in the patient's exhaled air D) The oxygen reservoir should deflate by one-third every time a breath is given Answer: B Diff: 2 Page Ref: 254 Objective: 10-14

9 Copyright © 2018 Pearson Education, Inc.

36) You have arrived next to a patient who reportedly had a seizure. Assessment reveals him to be apneic with vomitus in the airway. He has a radial pulse, and his skin is cool and diaphoretic. What should you do immediately? A) Start artificial ventilation B) Suction the airway C) Check breath sounds D) Apply a nonrebreather mask Answer: B Diff: 2 Page Ref: 233 Objective: 10-9 37) Which description demonstrates the EMT knows how to correctly position the BVM on the patient's face? A) "I place the narrow part of the mask over the bridge of the nose, and the bottom part in the cleft above the chin." B) "If two rescuers are using the BVM, the mask needs to be placed only over the mouth and the nose pinched closed." C) "The wide portion of the mask is placed at the top of the nose and the narrow part below the lower lip." D) "The mask is properly positioned when the top portion lies over the bridge of the nose and the lower portion below the chin." Answer: A Diff: 2 Page Ref: 254-255 Objective: 10-14 38) You are teaching a group of firefighters who are taking classes to become Emergency Medical Responders. The topic is artificial ventilation with the bag-valve mask (BVM). Which point related to use of the BVM is it important to emphasize? A) It is better to ventilate too quickly than too slowly B) Maintaining a good mask-to-face seal is difficult C) The BVM is useless unless it is hooked to supplemental oxygen D) The bag must be fully squeezed to give the proper amount of volume with each breath Answer: B Diff: 1 Page Ref: 253 Objective: 10-14

10 Copyright © 2018 Pearson Education, Inc.

39) Which tip should you give a new EMT about artificially ventilating a patient with the bagvalve mask? A) "If the patient has dentures, they should be removed so a better mask-to-face seal can be achieved." B) "It is important to maintain the head-tilt, chin-lift maneuver or the jaw-thrust maneuver while ventilating the patient." C) "To direct air into the lungs and away from the stomach, flex the patient's head forward when ventilating with the BVM." D) "Always remove the oropharyngeal airway. If it is in place when you are ventilating with a BVM, it may be pushed deep into the airway." Answer: B Diff: 2 Page Ref: 253 Objective: 10-14 40) A 52-year-old male patient with a brain tumor has gone into respiratory arrest, but still has a pulse and obtainable blood pressure. You assign the task of artificially ventilating the patient with the bag-valve mask to a new Emergency Medical Responder. How will you instruct him to ventilate the patient? A) Deliver 10 breaths per minute B) Give the patient 20 breaths per minute C) Administer 8 breaths per minute D) Give the patient 24 breaths per minute Answer: A Diff: 2 Page Ref: 247 Objective: 10-15 41) You are artificially ventilating an adult male patient who is in respiratory arrest with a bagvalve mask. Which finding would demonstrate that you are ventilating this patient properly? A) Adequate chest rise and fall B) Ability to fully deflate the self-inflating bag C) Clear and equal breath sounds D) Ventilation rate of 20 breaths per minute Answer: A Diff: 2 Page Ref: 255 Objective: 10-14 42) While a 61-year-old female patient in cardiac arrest receives emergency care, you note that her abdomen grows larger with each ventilation provided from a bag-valve mask. Which instructions should you provide to your partner? A) "Try delivering each ventilation over 3 seconds, and let's slow down the rate." B) "The ventilation rate and force of ventilation need to be increased so air goes into the lungs." C) "Let's slow the ventilation rate to 10 per minute, providing each breath over 1 second." D) "I need another rescuer to apply firm pressure over the stomach, while we ventilate this patient." Answer: C Diff: 3 Page Ref: 256 Objective: 10-16 11 Copyright © 2018 Pearson Education, Inc.

43) You are having a difficult time getting air from the bag-valve mask to pass into the lungs of a patient with suspected stroke. What should your first action be? A) Perform a finger sweep of the airway B) Administer abdominal thrusts C) Turn the patient on his side and try ventilations again D) Perform the head-tilt, chin-lift maneuver again Answer: D Diff: 2 Page Ref: 255 Objective: 10-14 44) You arrive at a residence to assist another crew with a patient in cardiac arrest. As you enter the room, a brand-new EMT informs you that he is having difficulty maintaining the mask seal to the patient's face while ventilating with the BVM. Which response shows that you understand and can help with the problem? A) "I will check breath sounds to see if air is reaching the lungs." B) "Why don't you try using a larger adult mask and flex the head forward?" C) "I will administer some cricoid pressure. That should help." D) "Let me maintain the face-to-mask seal while you squeeze the bag." Answer: D Diff: 2 Page Ref: 254-255 Objective: 10-14 45) Which statement about the flow-restricted, oxygen-powered ventilation device is true? A) "It can be used to ventilate spontaneously breathing patients whose breathing is too shallow." B) "It is an effective tool by which children can be ventilated when they are apneic." C) "It has a safety valve that prevents lung damage from overinflation." D) "It is very easy to use and has almost no complications associated with its use." Answer: A Diff: 2 Page Ref: 257 Objective: 10-17 46) The EMT shows that he is correctly using the flow-restricted, oxygen-powered ventilation device when he: A) Stops ventilating as soon as the patient's chest begins to rise B) Administers each ventilation over a 2- to 3-second period C) Ventilates the patient at a rate of 20 breaths per minute D) Increases pressure if the air enters the stomach instead of the lungs Answer: A Diff: 2 Page Ref: 258 Objective: 10-14

12 Copyright © 2018 Pearson Education, Inc.

47) Which of these is a correct statement about the ventilation of a nonbreathing patient through a stoma? A) The EMT should ventilate through the stoma with a child-sized face mask attached to a bagvalve mask B) A head-tilt, chin-lift maneuver or jaw-thrust airway maneuver is still needed to open the airway C) Mouth-to-stoma ventilation is an easy and safe procedure to perform on a nonbreathing patient D) A nasal airway should be inserted into the stoma prior to any attempt at ventilating the patient Answer: A Diff: 2 Page Ref: 265 Objective: 10-14 48) The EMT has just placed an oropharyngeal airway in a patient. By doing so, what has the EMT accomplished? A) Protected the airway from vomit or other secretions B) Displaced the tongue away from the back of the airway C) Obtained a patent airway by preventing closure of the mouth D) Minimized the risk of vomiting by closing off the esophagus Answer: B Diff: 2 Page Ref: 236 Objective: 10-10 49) The EMT shows that she understands how to use an oropharyngeal airway when she states: A) "Once this airway has been placed, the head-tilt, chin-lift maneuver is no longer needed." B) "If this airway is too small, it could push the epiglottis over the opening of the trachea." C) "This airway protects the patient from aspirating vomit or other secretions." D) "I must remain alert for vomiting or spasm of the vocal cords even with proper insertion." Answer: D Diff: 2 Page Ref: 236 Objective: 10-10 50) For which of these patient is placement of an oropharyngeal airway indicated? A) A responsive but confused patient with stridorous respirations B) A patient who responds to painful stimuli by moaning and has vomited C) An unresponsive patient who has no gag or cough reflex D) A patient with snoring respirations who coughs as the oral airway is placed into his mouth Answer: C Diff: 1 Page Ref: 236 Objective: 10-10

13 Copyright © 2018 Pearson Education, Inc.

51) You are observing an EMT insert an oropharyngeal airway into the airway of a 36-year-old male patient who has overdosed on a street drug. Which observation indicates a correct technique? A) The airway is inserted into the mouth upside-down, and then turned 180 degrees once it contacts the soft palate B) The EMT uses a tongue depressor to press the back of the tongue downward, and then inserts the oral airway upside-down C) The airway is inserted in its normal anatomic position until the flange of the airway is 1 cm above the lips D) The oral airway is introduced sideways into the mouth, and then rotated 180 degrees once it has reached the base of the tongue Answer: A Diff: 1 Page Ref: 237 Objective: 10-10 52) While assisting with the review of airway skills with new EMT hires, which of these statements from a new EMT demonstrates he knows how to properly size an oral airway prior to placement? A) "It is better to have an oral airway that is too large, rather than one that is too small." B) "To select an appropriately sized oral airway, you must estimate the patient's height and weight." C) "The airway should approximate the distance from the front teeth to the angle of the jaw." D) "An appropriately sized oral airway can be selected by looking in the patient's mouth and estimating the length of the tongue." Answer: C Diff: 1 Page Ref: 236 Objective: 10-10 53) The EMT should recognize that the oropharyngeal airway has been appropriately inserted when: A) It cannot be dislodged by the EMT B) The respiratory rate returns to normal C) Vomiting is no longer occurring D) Its flange rests on the teeth Answer: D Diff: 1 Page Ref: 237 Objective: 10-10 54) While the EMT is inserting an oropharyngeal airway, the patient gags. The EMT should: A) Remove the airway and maintain the head-tilt, chin-lift maneuver B) Lubricate the oropharyngeal airway and reattempt insertion C) Reattempt insertion using a smaller oropharyngeal airway D) Use a tongue depressor to better place the oral airway Answer: A Diff: 2 Page Ref: 237 Objective: 10-10 14 Copyright © 2018 Pearson Education, Inc.

55) You are transferring an unresponsive patient from her house to the ambulance on the stretcher. An oral airway has been placed and ventilations with the bag-valve mask are being administered. Suddenly, the patient regains consciousness and starts to gag. Your immediate action should be to: A) Leave the airway in but stop ventilations B) Remove the oral airway C) Suction the airway D) Reassure and calm the patient Answer: B Diff: 2 Page Ref: 237 Objective: 10-10 56) When inserting a nasopharyngeal airway, the EMT should remember that: A) The patient cannot be responsive or have a gag reflex B) An oil-based lubricant is needed for smooth insertion C) The head-tilt, chin-lift maneuver or jaw-thrust maneuver is not needed after insertion D) The nasal mucosa may bleed even with proper insertion Answer: D Diff: 1 Page Ref: 238 Objective: 10-10 57) The EMT is appropriately sizing the nasal airway when she measures the: A) Distance from the mouth to the angle of the jaw B) Diameter of the larger nostril C) Distance from the tip of the nose to earlobe D) Diameter of the patient's little finger Answer: C Diff: 1 Page Ref: 237-238 Objective: 10-10 58) Which of these statements best describes the EMT's understanding of oxygen? A) It is a gas classified as a drug B) It is a highly flammable substance C) It is an odorless cloudy gas when pressurized D) It is a moist gas that supports cellular activity Answer: A Diff: 1 Page Ref: 270 Objective: 10-21 59) Which of these statements about different sizes of oxygen cylinders is true? A) H tanks are the smallest oxygen cylinders B) D cylinders hold the most oxygen C) G and E tanks have the same psi when full D) M tanks are the easiest to carry on the stretcher Answer: C Diff: 1 Page Ref: 266 Objective: 10-21 15 Copyright © 2018 Pearson Education, Inc.

60) You must transport a patient with carbon monoxide poisoning to a specialized facility for treatment. Transport time is estimated at 1.5 hours. The patient is on a nonrebreather face mask with a continuous oxygen flow of 15 lpm. In the ambulance, you have a G tank with 1100 psi inside. Knowing that the cylinder constant is 2.41, which action should the EMT take in relation to the amount of available oxygen and transport time? A) Return to the EMS station so you can replace the G cylinder with a full one B) Decrease the oxygen flow to 10 lpm for the transport C) Apply a nasal cannula at 8 lpm for the transport D) Complete the transport, and replace the G cylinder when back at the station Answer: D Diff: 3 Page Ref: 267 Objective: 10-21 61) You are watching an EMT prepare the ambulance for the upcoming shift. In regard to oxygen cylinders and equipment, which action requires immediate intervention? A) The EMT lays tanks on the floor when they are removed from the ambulance B) The EMT cleans a dirty oxygen tank by spraying it with a petroleum-based agent cleanser C) The EMT ensures that the temperature in the oxygen storage room is less than 100°F D) The EMT makes certain that all valves on the empty oxygen tanks are closed Answer: B Diff: 2 Page Ref: 267 Objective: 10-22 62) You arrive at work and note that the portable oxygen tank in the ambulance is now yellow instead of green, as during the previous shift. As a knowledgeable EMT, you should: A) Remove the tank and retrieve a green cylinder B) Recognize the addition of more effective synthetic oxygen C) Determine that your service has switched to the new lightweight oxygen tank D) Realize that a yellow color indicates a new-style tank that holds more oxygen Answer: A Diff: 2 Page Ref: 267 Objective: 10-22 63) A paramedic has asked you to place the patient on a nonrebreather mask at 15 liters per minute. To administer oxygen at this flow rate, you should: A) Attach a high-pressure regulator to the oxygen cylinder B) Adjust the tank pressure to 15 psi C) Slowly decrease the tank pressure valve until 15 psi is achieved D) Select "15" on the oxygen therapy regulator Answer: D Diff: 2 Page Ref: 267 Objective: 10-23

16 Copyright © 2018 Pearson Education, Inc.

64) You are checking the D oxygen tank in the ambulance and note that the reading on the pressure regulator is 1000 psi. You should recognize that the tank is: A) Leaking B) Overfilled C) Half full D) Almost empty Answer: C Diff: 2 Page Ref: 268 Objective: 10-23 65) When providing oxygen via nonrebreather face mask, which piece of equipment is essential for the EMT to have? A) Oxygen humidifier B) Therapy regulator C) D or E oxygen cylinder D) High-pressure regulator Answer: B Diff: 1 Page Ref: 267 Objective: 10-23 66) When using a therapy regulator, what can be readily observed by the EMT? A) Temperature of oxygen in the cylinder B) Liters of oxygen remaining in the oxygen cylinder C) Concentration of oxygen received by the patient D) Liter flow (flow rate) of oxygen leaving the cylinder Answer: D Diff: 2 Page Ref: 267 Objective: 10-23 67) Which of these statements indicates that the EMT has a correct understanding of the use of oxygen in the prehospital setting? A) "Oxygen should be withheld from a patient with chronic obstructive pulmonary disease." B) "If the patient has a poor tidal volume, oxygen should be administered at 15 lpm through a nonrebreather mask." C) "Oxygen should never, ever be withheld from a patient whom you think may need it." D) "If providing positive pressure ventilation, lower concentrations of oxygen may be used." Answer: C Diff: 2 Page Ref: 223 Objective: 10-12

17 Copyright © 2018 Pearson Education, Inc.

68) Which action should you perform when attaching a therapy regulator to a new oxygen tank? A) Open the cylinder valve for 1 second prior to attaching the regulator B) Tighten the regulator to the oxygen cylinder with an oxygen wrench C) Stand the tank upright on the floor for 2 minutes prior to attaching the regulator D) Lubricate the stem of the cylinder with oil prior to attaching the regulator Answer: A Diff: 1 Page Ref: 280, 10-7B Objective: 10-22 69) Which of these actions indicates that the EMT is correctly using a nonrebreather face mask on an adult patient? A) He ensures that the reservoir fully collapses with each breath B) He instructs the patient to take deeper breaths when the mask is on C) He sets the oxygen flow rate to between 6 and 16 liters per minute D) He fully inflates the reservoir prior to placing the mask on the patient Answer: D Diff: 2 Page Ref: 271 Objective: 10-22 70) When properly using a nonrebreather face mask, which of these statements is true? A) The air exhaled by the patient will be recycled with oxygen B) The mask is comfortable and well tolerated by all patients when properly applied C) The concentration of inhaled oxygen will be approximately 90% D) Oxygen is delivered into the lungs even when the patient is not inhaling Answer: C Diff: 1 Page Ref: 271 Objective: 10-24 71) You are assessing a patient with altered mental status. Which of these assessment findings contraindicates the use of the nonrebreather face mask? A) Confusion B) Respiratory rate of 18 breaths/min, with poor tidal volume C) Clear and equal breath sounds D) Respiratory rate of 24 breaths/min Answer: B Diff: 2 Page Ref: 240 Objective: 10-11

18 Copyright © 2018 Pearson Education, Inc.

72) Assessment findings for an alert and oriented patient complaining of shortness of breath reveal an open airway and strong radial pulse, accompanied by skin that is cool and diaphoretic. The depth of respirations is adequate, and breath sounds include wheezing throughout the lungs. Vital signs are a pulse of 124, respiratory rate of 24 breaths/min, blood pressure of 146/82 mmHg, and SpO2 of 93%. The patient has a history of asthma, for which he takes medications. How should you initiate oxygen therapy on this patient? A) Nonrebreather mask B) Venturi mask C) Nasal cannula D) Bag-valve mask Answer: C Diff: 2 Page Ref: 272 Objective: 10-12 73) Despite coaching and explaining the benefits of a nonrebreather face mask, a hypoxic patient with moderate shortness of breath in conjunction with lung cancer states that she cannot tolerate the mask over her face. The more she panics, the worse the shortness of breath becomes. What is your best course of action? A) Remove the nonrebreather mask and apply a simple face mask B) Decrease the oxygen flow rate entering the nonrebreather face mask C) Disconnect the nonrebreather mask and replace it with a nasal cannula D) Discontinue oxygen therapy and continually monitor the patient's breath sounds during transport Answer: C Diff: 2 Page Ref: 271 Objective: 10-24 74) What is the preferred method of providing high-concentration oxygen to a breathing patient in the prehospital setting? A) Nasal cannula B) Nonrebreather face mask C) Simple face mask D) Venturi mask Answer: B Diff: 1 Page Ref: 271 Objective: 10-24 75) Which oxygen administration systems will provide the patient with the greatest amount of oxygen? A) Nasal cannula at 15 liters per minute of oxygen B) Simple face mask at 12 liters per minute of oxygen C) Nonrebreather face mask at 15 liters per minute of oxygen D) Venturi face mask set to deliver 40% oxygen Answer: C Diff: 2 Page Ref: 271 Objective: 10-24 19 Copyright © 2018 Pearson Education, Inc.

76) When monitoring a patient receiving oxygen through a nasal cannula, which of these observations warrants immediate intervention? A) Oxygen flow rate of 15 liters per minute B) Prongs in the nostrils curving posteriorly into the nose C) Tubing positioned over the ears and under the chin D) Breathing normally with the nasal cannula in place Answer: A Diff: 1 Page Ref: 272 Objective: 10-24 77) An elderly male patient who is short of breath is on home oxygen. He is wearing a nasal cannula connected to an oxygen tank with the flow regulator set to 3 liters per minute. As a knowledgeable EMT, you should recognize that the patient is getting approximately what percentage of oxygen with each breath? A) 20% B) 24% C) 32% D) 40% Answer: C Diff: 2 Page Ref: 272 Objective: 10-24 78) Which of these statements regarding the use of a nasal cannula is true? A) "I can adjust the oxygen flow rate anywhere from 1 to 8 lpm, depending on how much oxygen the patient needs." B) "When the nasal cannula is set to the highest appropriate oxygen flow rate, a large portion of the air inspired by the patient is still ambient air." C) "The nasal cannula is preferred over the nonrebreather mask because it gives the oxygen through the nose instead of the mouth." D) "It is best to see how the patient tolerates the nasal cannula before attempting to place a nonrebreather mask on the patient." Answer: B Diff: 2 Page Ref: 272 Objective: 10-24 79) Which of these statements about an automatic transport ventilator (ATV) is true? A) It can help minimize gastric distention associated with positive pressure ventilation B) When it is in use, there is no need for a face-to-mask seal C) It eliminates the need for a head-tilt, chin-lift maneuver or jaw-thrust maneuver D) The EMT can more readily identify an increase or decrease in the patient's lung compliance Answer: A Diff: 2 Page Ref: 258 Objective: 10-24

20 Copyright © 2018 Pearson Education, Inc.

80) You are using an automatic transport ventilator (ATV) for a patient who has an insufficient respiratory effort. What is the best indicator of adequate artificial ventilation by the ATV? A) The preset tidal and minute volumes on the ATV are being used B) There is no air leak between the mask and the patient's face C) The inspiratory to expiratory ratio is 2 seconds to 1 second D) The patient's chest rises and falls with each mechanical ventilation Answer: D Diff: 2 Page Ref: 259 Objective: 10-24 81) What is a benefit of using a humidification system when administering oxygen? A) Humidification reduces the chances of respiratory infection B) Oxygen therapy is less drying to the upper respiratory tract C) Increased diffusion of oxygen occurs across the respiratory membrane D) A higher flow rate can be used with a nasal cannula when humidification is added Answer: B Diff: 2 Page Ref: 268 Objective: 10-24 82) A patient was eating dinner when a piece of food became caught in his throat. Although coughing has failed to remove the food, he remains able to move an adequate amount of air into and out of his lungs. A nasal cannula has been applied and vital signs are stable. During transport, the patient suddenly begins to exhibit stridorous respirations. Your immediate action should be to: A) Continue care and transport B) Start positive pressure ventilation C) Perform abdominal thrusts D) Place the patient supine and suction his airway Answer: C Diff: 2 Page Ref: 266 Objective: 10-6 83) When providing an in-service program on ventilating the geriatric patient, you should emphasize which point in regard to a patient who has dentures? A) It is always best to remove dentures, regardless of whether they are loose B) It is best to leave the top appliance in place, but remove the bottom one C) Removal of dentures enables the EMT to maintain a better face to mask seal D) Loose appliances should be removed if they interfere with the airway or the seal between the mask and face Answer: D Diff: 1 Page Ref: 266 Objective: 10-5

21 Copyright © 2018 Pearson Education, Inc.

84) Which of these statements best describes the purpose of a Venturi mask? A) The device is used to deliver positive pressure ventilation B) It allows a 100% concentration of oxygen to be delivered without a leak between the patient's face and mask C) Precise concentrations of oxygen can be delivered with this device D) It provides a seal over the larynx, preventing aspiration during positive pressure ventilation Answer: C Diff: 1 Page Ref: 273 Objective: 10-24 85) You are starting to ventilate a patient with a laryngectomy and stoma. As you provide the first few ventilations, you note that the air passes easily into the patient, but does not cause the chest to rise. Your next action should be to: A) Seal the patient's mouth and nose B) Suction the tracheostomy tube C) Perform a head-tilt, chin-lift maneuver D) Ventilate through the mouth and nose Answer: A Diff: 2 Page Ref: 264-265 Objective: 10-14 86) A patient with a stoma and tracheostomy tube is responsive to painful stimuli. He has shallow respirations and is breathing at a rate of 6 times per minute. He has cyanosis of the fingertips and a weak, rapid radial pulse. Which strategy should be used in providing respiratory care to this patient? A) Remove the tracheostomy tube, place a child-size mask on the bag-valve device, and ventilate the patient through the stoma B) Attach the bag-valve device directly to the tracheostomy tube C) Use a continuous positive airway pressure device over the stoma D) Place a tracheostomy mask over the tube and administer high-concentration oxygen Answer: B Diff: 2 Page Ref: 264 Objective: 10-11 87) A 21-year-old female patient has overdosed on an unknown drug. She has snoring respirations and an intact gag reflex. She has weak respiratory effort and is slightly cyanotic. Given these assessment findings, your care should include: A) placement of an oropharyngeal airway B) oxygen via nonrebreather face mask C) insertion of a nasal airway D) transport of the patient in the lateral recumbent (recovery) position Answer: C Diff: 2 Page Ref: 237 Objective: 10-6

22 Copyright © 2018 Pearson Education, Inc.

88) What is obstructing the airway when snoring is heard? A) Fluid in the upper airway B) Swelling of the larynx C) Bronchospasm D) The tongue Answer: D Diff: 1 Page Ref: 228-229 Objective: 10-6 89) A patient who inhaled superheated steam in an industrial fire is most likely to exhibit which abnormal upper airway noise? A) Stridor B) Wheezing C) Gurgling D) Snoring Answer: A Diff: 2 Page Ref: 229 Objective: 10-6 90) Which of these statements, when made by the EMT, reflects correct knowledge regarding the use of oxygen in a patient with a suspected acute coronary syndrome who denies dyspnea, has a pulse oximeter reading of 96%, and has acceptable vital signs? A) "Aside from patients with chest pain, stroke, and trauma, oxygen is no longer used in emergency care." B) "Oxygen administration is no longer considered standard therapy for this patient." C) "Oxygen administration should be used in such a way that a patient's SpO2 reaches 97% to 100% if he or she is having chest pain." D) "Over-oxygenating a patient is not an issue for EMS, given the short amount of time we have patient contact." Answer: B Diff: 2 Page Ref: 268 Objective: 10-12 91) A patient with chest discomfort has the following vital signs: pulse, 88; respirations, 14 breaths/min; blood pressure, 154/84 mmHg; and SpO2, 97% on room air. In this situation, the EMT would address oxygen therapy by: A) Using a nasal cannula with flow of 4 lpm B) Administering blow-by oxygen at 5 lpm C) Withholding the use of oxygen D) Providing oxygen via a nonrebreather at 15 lpm Answer: C Diff: 2 Page Ref: 268 Objective: 10-11

23 Copyright © 2018 Pearson Education, Inc.

92) A patient complaining of shortness of breath and a history of asthma has the following vital signs: pulse, 96; respirations, 20 breaths/min; blood pressure, 132/86 mmHg; and SpO2, 92% on room air. Mild wheezing is noted bilaterally, and the patient states that she has run out of her rescue inhaler (albuterol). Regarding the patient's respiratory status, how should you initially administer oxygen? A) 2 lpm O2 through a nasal cannula B) 4 lpm O2 through a nasal cannula C) 12 lpm O2 through a nonrebreather mask D) 15 lpm O2 through a nonrebreather mask Answer: A Diff: 2 Page Ref: 268 Objective: 10-12 93) You are administering oxygen at 2 lpm through a nasal cannula to a confused patient. During transport, you note the SpO2 to be 91%, up from 90% on scene. The patient remains confused. What would your next action be? A) Keep the oxygen flow at 2 lpm and continue to monitor B) Administer oxygen at 15 lpm through a nonrebreather mask C) Start positive pressure ventilation with supplemental oxygen D) Increase the oxygen flow to 4 lpm Answer: D Diff: 3 Page Ref: 268 Objective: 10-12 94) You are by the side of a patient who fell 5 feet from a ladder, landing on a concrete porch and hitting his lower back and buttocks. Assessment reveals an intact airway, adequate respirations, rapid radial pulse, and skin that is pale, cool, and diaphoretic. The patient complains of lower back and left leg pain. No deformity to the extremity is noted; however, the lower back is tender on palpation. The following vital signs are obtained: pulse, 132; respirations, 20 breaths/min; blood pressure, 102/88 mmHg; and SpO2, 97% on room air. Regarding oxygen therapy, you would: A) Provide 2 lpm O2 through a nasal cannula B) Start positive pressure ventilation to support breathing C) Administer oxygen at 15 lpm through a nonrebreather mask D) Recognize a normal SpO2 and forego oxygen therapy Answer: C Diff: 3 Page Ref: 269 Objective: 10-12

24 Copyright © 2018 Pearson Education, Inc.

95) A listless and lethargic 84-year-old male patient responds to physical stimuli with garbled speech. His respirations show poor effort and are rapid and labored, with a room-air SpO2 of 84%. Additionally, you cannot appreciate breath sounds in his right lung. The best form of oxygen therapy for this patient would be: A) Positive pressure ventilation B) High-concentration oxygen via a nonrebreather mask C) Oxygen at 2 lpm via nasal cannula initially and increase the liter flow as needed D) Continuous positive airway pressure (CPAP) Answer: A Diff: 3 Page Ref: 224 Objective: 10-11 96) Your paramedic partner has applied continuous positive airway pressure (CPAP) to a patient in respiratory distress from congestive heart failure (CHF). As an EMT, you realize that this treatment should benefit the patient by: A) Collapsing portions of the diaphragm B) Causing the diaphragm to relax during inspiration C) Removing air from the lungs during exhalation D) Displacing fluid in the alveoli back into the bloodstream Answer: D Diff: 3 Page Ref: 260 Objective: 10-18 97) The EMT would recognize that administering continuous positive airway pressure (CPAP) at a pressure that is too high can cause: A) Hypertension B) Lung collapse C) Pulmonary edema D) Oral cavity pressure trauma Answer: B Diff: 3 Page Ref: 263 Objective: 10-19 98) To receive treatment with continuous positive airway pressure (CPAP), the patient must exhibit: A) Minor to moderate respiratory distress B) Ability to breathe on his own adequately C) SpO2 between 90% and 100% D) Inability to maintain his airway Answer: B Diff: 2 Page Ref: 261 Objective: 10-17

25 Copyright © 2018 Pearson Education, Inc.

99) For which of these disease processes would continuous positive airway pressure (CPAP) be most beneficial? A) Asthma with mild shortness of breath B) Stroke with agonal respirations C) CHF with moderate to severe dyspnea D) Altered mental status with history of COPD Answer: C Diff: 2 Page Ref: 261 Objective: 10-18 100) Which of these statements made by another crew member would cause you to disregard the use of continuous positive airway pressure (CPAP) for a patient? A) "His respirations are agonal and slow." B) "He had bleeding stomach ulcers two years ago." C) "His SpO2 has fallen from 92% to 88%." D) "His blood pressure is 186/108 mmHg." Answer: A Diff: 3 Page Ref: 261 Objective: 10-18 101) The EMT is properly applying continuous positive airway pressure (CPAP) when he: A) Checks and documents the pulse oximetry reading every 15 minutes B) Places a confused patient receiving CPAP in a lateral recovery position C) Adjusts the ventilation rate to between 16 and 18 breaths/min D) Places the mask over the patient's mouth and nose and tightens it to the face Answer: D Diff: 3 Page Ref: 262 Objective: 10-17 102) After 2 minutes of continuous positive airway pressure (CPAP) at 4 cmH2O, you see no improvement in the patient's respirations or SpO2 level. What is your next step? A) Stop the CPAP and begin to provide positive pressure ventilation B) Place an oral airway and continue the continuous positive airway pressure C) Continue the CPAP as is and monitor the patient for another 5 minutes D) Increase the continuous positive airway pressure by 2 cmH2O and reassess the patient Answer: D Diff: 3 Page Ref: 262 Objective: 10-18

26 Copyright © 2018 Pearson Education, Inc.

103) Typically, the highest continuous positive airway pressure (CPAP) pressure used by the EMT without special permission from medical direction is: A) 2 cmH2O B) 5 cmH2O C) 10 cmH2O D) 20 cmH2O Answer: C Diff: 2 Page Ref: 262 Objective: 10-18 104) When administering continuous positive airway pressure (CPAP) to a patient, you would recognize that the patient is benefiting from the treatment when the: A) Patient states he is breathing easier B) Heart rate increases from baseline by 10 percent C) Systolic blood pressure drops below 100 mmHg D) Patient can resume control of his airway Answer: A Diff: 1 Page Ref: 263 Objective: 10-18 105) When administering continuous positive airway pressure (CPAP) to a patient in respiratory distress, which of these statements made by the patient would be most concerning? A) "The mask is too tight." B) "I feel nauseated." C) "My mouth is getting dry." D) "My nose hurts from the mask." Answer: B Diff: 2 Page Ref: 261 Objective: 10-19 106) Which of these statements is most correct regarding the difference between respiration and ventilation? A) Ventilation describes the absorption of O2 into the cells, while ventilation is the excretion of CO2 from the cells B) Ventilation describes the exchange of gases at the cellular level, while respiration describes the same process between the lungs and the external environment C) Respiration describes the exchange of O2 and CO2, while ventilation describes the movement of air that carries them into and out of the body D) Respiration and ventilation are similar terms that both describe normal and healthy breathing in the living organism Answer: C Diff: 3 Page Ref: 218 Objective: 10-2

27 Copyright © 2018 Pearson Education, Inc.

107) The structure that lies between the trachea and bronchi is the: A) Larynx B) Carina C) Bronchioles D) Cricoid cartilage Answer: B Diff: 2 Page Ref: 220 Objective: 10-1 108) Which of these statements is true regarding the form or function of the alveoli? A) The alveoli permit gas exchange between the lungs and bloodstream B) The alveoli are thin walled and covered by smooth muscle C) The alveoli are air passages that permit ventilation of the lung D) The alveoli are covered with small arteries that allow gas exchange Answer: A Diff: 2 Page Ref: 220 Objective: 10-1 109) A young male patient has been stabbed in the right lateral chest. Regarding the respiratory system, which finding presents the greatest danger to his well-being? A) Loss of serous fluid from membranes in the chest B) Fracture of the rib and predisposition to a pulmonary infection C) Loss of oxygen from the lungs through the stab wound D) Collection of air between the pleural and visceral membranes Answer: D Diff: 3 Page Ref: 221 Objective: 10-13 110) In a healthy patient, for air to be exhaled from the lungs, what must occur? A) The diaphragm must relax B) The CO2 levels must be low C) The pressure is the lungs must be low D) The intercostals muscles must contract Answer: A Diff: 1 Page Ref: 222 Objective: 10-13 111) In a healthy person, the primary stimulus to breathe is: A) Lung pressures B) Oxygen levels in the arteries C) Diaphragm position D) Carbon dioxide levels in the bloodstream Answer: D Diff: 2 Page Ref: 222 Objective: 10-2

28 Copyright © 2018 Pearson Education, Inc.

112) Which of these conditions would the EMT associate with a hypoxic respiratory drive? A) Congestive heart failure B) Pneumonia C) Chronic obstructive pulmonary disease D) Diabetes Answer: C Diff: 2 Page Ref: 270 Objective: 10-2 113) A patient with a heavy smoking history and chronic obstructive pulmonary disease (emphysema) presents in marked respiratory distress with cyanosis noted to her fingertips. When administering oxygen to this patient, the EMT would remember that: A) Oxygen should be provided at the same level as the patient receives at home B) High-concentration oxygen must be provided if the patient's condition warrants such therapy C) Oxygen must be administered only through a nasal cannula D) High-concentration oxygen will be toxic to the patient and must be avoided Answer: B Diff: 2 Page Ref: 270 Objective: 10-22 114) When does hypoxia occur? A) When carbon dioxide accumulates within the body B) If the respiratory rate decreases to less than 12 breaths per minute C) When carbon dioxide is excreted too rapidly from the lungs D) If inadequate amounts of O2 are available to the cells Answer: D Diff: 1 Page Ref: 223 Objective: 10-4 115) Which body system or structure is responsible for the control and regulation of a person's breathing? A) Lungs B) Brain C) Spinal cord D) Alveoli Answer: B Diff: 2 Page Ref: 222 Objective: 10-3

29 Copyright © 2018 Pearson Education, Inc.

116) When administering oxygen to a hypoxic patient with Alzheimer's disease, which of these findings would seemingly indicate that the patient's hypoxic status is improving? A) The patient has an SpO2 reading of 94% B) The patient becomes more restful and wants to sleep C) The blood pressure increases from 136/78 to 150/86 mmHg D) The respirations slow from 24 to 22 breaths/min Answer: A Diff: 3 Page Ref: 269 Objective: 10-20 117) The EMT recognizes a late sign of hypoxia in a patient in a coma when he observes: A) Restlessness and agitation B) Elevation in blood pressure C) Cyanosis in the fingers D) Complaint of a headache Answer: C Diff: 3 Page Ref: 223 Objective: 10-20 118) Which of these conditions would result in the greatest decrease in oxygen delivery to the cells? A) Heart rate greater than 120 beats/ min B) Diastolic blood pressure greater than 100 mmHg C) Decreased amount of circulating hemoglobin D) Temperature greater than 103°F Answer: C Diff: 2 Page Ref: 224-225 Objective: 10-2 119) You must immobilize a 6-year-old male patient who was involved in a motor vehicle accident. Regarding the airway, why should you consider placing padding under his shoulders and back? A) The child's cricoid cartilage is prone to collapse B) The child's diaphragm is lower and more difficult to move C) The child's tongue is larger and weaker than an adult's D) The child's occipital skull is larger in proportion to his body Answer: D Diff: 2 Page Ref: 227 Objective: 10-3

30 Copyright © 2018 Pearson Education, Inc.

120) On arrival at a lethargic patient's side, you note her to have gurgling respirations. What is the best treatment for this condition? A) Suction B) Head-tilt, chin-lift maneuver C) Oropharyngeal airway D) Jaw-thrust maneuver Answer: A Diff: 1 Page Ref: 229 Objective: 10-6 121) Which type of portable suction unit has the lowest potential for problems during its use? A) Hand powered B) Battery powered C) Oxygen powered D) Air powered Answer: A Diff: 2 Page Ref: 234 Objective: 10-8 122) Proper positioning of the airway with the head-tilt, chin-lift maneuver during ventilation can help reduce gastric inflation by: A) Reducing airway resistance B) Allowing slower ventilation C) Reducing the volume of exhaled air D) Obstructing the esophagus Answer: A Diff: 2 Page Ref: 247 Objective: 10-16 123) A 6-year-old boy has gotten into his mother's prescribed medications and ingested a large number of her opioid tablets. The patient is now found unresponsive and apneic, but he still has a weak palpable carotid pulse. You would instruct your partner to ventilate this patient in which manner? A) 6 times a minute B) 10 times a minute C) One ventilation every 2 seconds D) One ventilation every 6 seconds Answer: B Diff: 2 Page Ref: 247 Objective: 10-15

31 Copyright © 2018 Pearson Education, Inc.

Prehospital Emergency Care, 11e (Mistovich et al.) Chapter 11 Vital Signs, Monitoring Devices, and History Taking 1) Which of these patient descriptions contains only vital signs? A) Chief complaint of dizziness; skin, cool and clammy; respiratory rate, 16 breaths/min B) Chief complaint of dizziness; BP, 110/76 mmHg; breath sounds, clear and equal C) Skin, warm and dry; heart rate, 74 beats/min; pupils equal and reactive D) Heart rate, 88 beats/min; respiratory rate, 14 breaths/min; blood glucose level, 98 mg/dL Answer: C Diff: 1 Page Ref: 288 Objective: 11-1 2) You are transporting a depressed patient who states that she is thinking of killing herself by taking a "whole bunch of pills." After providing this information, the patient has refused to speak and has remained silent. Your physical assessment of the patient reveals she is stable. How often should vital signs be reassessed? A) Once at the scene and once en route to the ED B) Only baseline vital signs are required C) Every 5 minutes D) Every 15 minutes Answer: D Diff: 2 Page Ref: 299 Objective: 11-14 3) The EMT is correctly calculating a patient's respiratory rate when he or she: A) Counts the number of breaths in 1 minute and divides by 4 B) Obtains a pulse oximetry reading and divides it by 3 C) Assesses the patient for any sign of respiratory difficulty D) Counts the number of breaths for 30 seconds and multiplies by 2 Answer: D Diff: 1 Page Ref: 289 Objective: 11-2 4) Emergency Medical Responders inform you that a 27-year-old male patient with altered mental status has an open airway and is breathing 9 times every 30 seconds. His pulse rate is 40 beats/min, and he has bruises to his chest. Based on this vital sign and presentation information, the EMT should recognize the: A) Respiratory rate as normal B) Heart as beating adequately C) Blood pressure as normal D) Heart rate as irregular Answer: A Diff: 2 Page Ref: 288 Objective: 11-4

1 Copyright © 2018 Pearson Education, Inc.

5) Which of these findings related to breathing would be most concerning to the EMT in an adult patient? A) Respiratory rate of 18, complaint of weakness B) Respiratory rate of 22, normal chest wall expansion C) Respiratory rate of 10, chest expansion of about 1 inch D) Respiratory rate of 20, use of accessory muscles Answer: D Diff: 2 Page Ref: 289 Objective: 11-4 6) Which of these statements made by your EMT partner indicates that he understands assessment of a patient's breathing? A) "As long as the patient is breathing more than 20 times per minute, he is getting enough oxygen in his body." B) "If the respiratory rate is normal, the patient is breathing adequately and getting enough oxygen." C) "A rate less than 8 times per minute may allow adequate breathing, but requires further evaluation and assessment." D) "To determine if a patient is adequately breathing, the EMT must get a full set of vital signs." Answer: C Diff: 2 Page Ref: 288 Objective: 11-3 7) A family has called 911 for a 41-year-old male family member whom they cannot wake up. They state he has been threatening to kill himself and believe that he may have intentionally overdosed on his pain medications. As you enter the patient's bedroom, you observe him supine on the floor with his eyes closed. As he breathes, you hear snoring respirations. As a knowledgeable EMT, you recognize that this condition is likely caused by: A) The nasopharynx being blocked by mucus or heavy secretions B) The airway swelling shut C) The tongue partially blocking the airway D) The respiratory rate being less than 10 breaths/min Answer: C Diff: 3 Page Ref: 290, Table 11-2 Objective: 11-3

2 Copyright © 2018 Pearson Education, Inc.

8) A young female patient has been stung by a bee and complains that her "throat is closing up." She states that she is allergic to bee stings; the last time this happened, she had to have a "tube put into my windpipe." She is struggling to breathe and can speak only a few words at a time. Knowing that allergic reactions can cause swelling in the pharynx and the larynx, what airway sound would indicate that the patient indeed has swelling in this area? A) Gurgling B) Stridor C) Snoring D) Wheezing Answer: B Diff: 2 Page Ref: 289 Objective: 11-3 9) An EMT is correctly assessing a patient's radial pulse when he: A) Uses the palm of his hand to feel the pulse on the upper arm B) Simultaneously checks for a heart rate on both sides of the neck C) Uses his thumb to feel for the pulse on the patient's lower arm, near the base of the thumb D) Uses his fingertips to feel for a pulse on the anterior and lateral aspects of the patient's wrist Answer: D Diff: 1 Page Ref: 292 Objective: 11-2 10) When assessing a patient's pulse, you note that it is irregular. To get an accurate heart rate, you would: A) Listen to the pulse with a stethoscope B) Count the number of beats that occur in 1 minute C) Double the number of beats counted in 30 seconds D) Feel the carotid pulse for 30 seconds and multiply by 2 Answer: B Diff: 2 Page Ref: 292 Objective: 11-7 11) What is the name of the pulse that is palpated in the groin area? A) Carotid B) Inguinal C) Pedal D) Femoral Answer: D Diff: 1 Page Ref: 290 Objective: 11-6

3 Copyright © 2018 Pearson Education, Inc.

12) You have been called to assess a conscious and alert 5-year-old child whose chief complaint is nausea and vomiting over the last 2 hours. When assessing his pulse, you should first check which pulse? A) Brachial B) Carotid C) Pedal D) Radial Answer: D Diff: 1 Page Ref: 290 Objective: 11-6 13) You are assessing a 61-year-old male patient who is confused. During the primary assessment, you cannot locate a radial pulse. Your immediate action should be to: A) Apply the AED B) Call for ALS assistance C) Check for a carotid pulse D) Start CPR Answer: C Diff: 2 Page Ref: 290-291 Objective: 11-6 14) During the primary assessment of a geriatric patient complaining of shortness of breath and fever, you quickly locate the radial pulse. What should you do next? A) Assess the patient's breathing B) Obtain a blood pressure C) Determine the rate and quality of the pulse D) Establish the patient's level of consciousness Answer: C Diff: 1 Page Ref: 292 Objective: 11-2 15) Which of these statements concerning assessment of the pulse is correct? A) "The heart rate can be determined by doubling the number of beats counted in 30 seconds." B) "To get the most accurate rate, the EMT should place a stethoscope over the pulse site and count the number of beats in 1 minute." C) "A brachial pulse should be felt first in any patient younger than 6 years of age." D) "After much experience, an EMT can calculate the pulse rate by just feeling it for a few seconds." Answer: A Diff: 2 Page Ref: 292 Objective: 11-6

4 Copyright © 2018 Pearson Education, Inc.

16) You determine a patient's heart rate to be 48 beats/min. That heart rate would be categorized as: A) Tachypneic B) Normal C) Bradycardic D) Tachycardic Answer: C Diff: 1 Page Ref: 291-292 Objective: 11-7 17) The EMT recognizes which heart rate would be considered normal for a 24-year-old male patient who is resting comfortably? A) 54 beats/min B) 62 beats/min C) 110 beats/min D) 124 beats/min Answer: B Diff: 1 Page Ref: 292 Objective: 11-7 18) When assessing a patient's pulse, you can locate the right radial pulse, but not the left. What is the most likely explanation for this finding? A) The patient is in the early stage of cardiac arrest B) The left radial artery is extremely large C) There is a problem with the patient's veins D) The left radial artery may be occluded Answer: D Diff: 2 Page Ref: 292 Objective: 11-7 19) Which of these characteristics would be considered a pulse "quality" factor? A) Strength B) Rate C) Regularity D) Tone Answer: A Diff: 1 Page Ref: 292 Objective: 11-6

5 Copyright © 2018 Pearson Education, Inc.

20) You determine a patient's radial pulse is readily palpable, but has uneven intervals between beats. This finding would best be documented on the prehospital care report as: A) Bounding and regular B) Prominent and bounding C) Strong and irregular D) Thready and regular Answer: C Diff: 2 Page Ref: 292 Objective: 11-7 21) When evaluating a patient's skin color, the EMT should: A) Observe the nail beds and/or mucous membranes inside the eyelids B) Examine the change in skin color when applying, then releasing pressure to the nail beds C) Look at the skin on the upper chest and feel it for warmth D) Examine the skin color of the face and compare it with the color of the arms Answer: A Diff: 2 Page Ref: 293 Objective: 11-2 22) You are assessing a confused dark-skinned patient who complains of a headache. When checking his skin color, you should: A) Examine the skin around his ears and nose B) Evaluate the palms of his hands or conjunctiva C) Observe the skin on his forehead D) Look at and feel the skin on his neck and chest Answer: B Diff: 2 Page Ref: 293 Objective: 11-2 23) An EMT has accurately checked the skin color of a dark-skinned patient. How would this finding be best documented on a patient care report? A) "Warm skin noted on the upper arm." B) "Forehead and face show pink skin color." C) "Pale color noted to the conjunctiva." D) "Thoracic and abdominal skin normal in color." Answer: C Diff: 2 Page Ref: 293 Objective: 11-8 24) Which of these methods most accurately assesses a patient's skin temperature? A) Placing the tips of two fingers on the patient's wrist B) Using the palm to feel the patient's forehead C) Placing the back of the hand on the patient's abdomen D) Placing the inside of the EMT's arm against the patient's arm Answer: C Diff: 2 Page Ref: 293-294 Objective: 11-2 6 Copyright © 2018 Pearson Education, Inc.

25) Which of these statements made by an EMT indicates an accurate understanding of checking a patient's skin temperature? A) "Checking the skin temperature is not precise, but provides a good estimate of abnormally high or low body temperatures." B) "To properly evaluate a patient's skin temperature during the primary assessment, the EMT needs a thermometer." C) "Flushed skin always indicates a high body temperature." D) "With experience, the EMT can precisely determine a patient's body temperature by feeling the skin." Answer: A Diff: 2 Page Ref: 294 Objective: 11-8 26) To test a pediatric patient's capillary refill, the EMT will: A) Apply gentle pressure to the radial pulse and watch color changes to the hand B) Firmly compress and then release pressure on the nail bed C) Examine the inside lining of one or both eyelids D) Apply pressure to an arm and then release that pressure Answer: B Diff: 2 Page Ref: 294 Objective: 11-2 27) The EMT should recognize normal skin color in a healthy patient as: A) White B) Warm C) Red D) Pink Answer: D Diff: 1 Page Ref: 293 Objective: 11-8 28) Assessment of a patient's skin color reveals a bluish discoloration. As a knowledgeable EMT, you should identify: A) Pallor caused by a decrease in blood flow to the tissues B) Cyanosis caused by inadequate oxygenation of the tissues C) Erythema caused by a decrease in body temperature D) Jaundice caused by an increase in blood flow to the extremities Answer: B Diff: 2 Page Ref: 293 Objective: 11-9

7 Copyright © 2018 Pearson Education, Inc.

29) A patient presents with extremely pale skin. His family states that this is not his normal skin color. His vital signs are pulse, 118; respirations, 18 breaths/min; blood pressure, 102/86 mmHg; and SpO2, 96%. Which complaint would make the most sense given this clinical presentation? A) Persistent cough B) Fever C) Rectal bleeding D) Headache Answer: C Diff: 3 Page Ref: 293 Objective: 11-9 30) A patient has advanced liver disease from years of exposure to toxic chemicals in his place of employment. Which skin color would the EMT expect given this status? A) Bluish-gray B) Pale and cyanotic C) Pinkish and flushed D) Yellow-orange Answer: D Diff: 3 Page Ref: 293 Objective: 11-9 31) A patient with liver cancer exhibits a yellow discoloration to his entire body. The EMT would correctly document this condition as: A) Jaundice B) Cyanotic C) Pallor D) Flushed Answer: A Diff: 2 Page Ref: 293 Objective: 11-9 32) Which of these patients should the EMT recognize as having a normal skin temperature? A) A 25-year-old female who is dizzy with cool skin B) A 36-year-old male complaining of nausea with warm skin C) A 47-year-old female with chest pain and warm-to-hot skin D) An 88-year-old male who is weak with cool and dry skin Answer: B Diff: 2 Page Ref: 294 Objective: 11-9

8 Copyright © 2018 Pearson Education, Inc.

33) You are called to a long-term care facility to assess and transport a patient with a high fever. You do not have a thermometer on your unit. Which alternative sign should you use to confirm the patient's increased body temperature? A) Cyanotic skin around the mouth B) Decreased heart rate and flushed skin C) Fingertips that are cool and blue D) Skin on the abdomen that is hot and flushed Answer: D Diff: 2 Page Ref: 293-294 Objective: 11-8 34) A patient presents with skin that is cool to the touch. The EMT shows he understands this condition when he states: A) "This may indicate a problem with the patient's blood circulation." B) "The patient must have an infection somewhere in his body." C) "We should check his breath sounds; the patient probably has a lung infection, such as pneumonia." D) "The patient most likely has a past medical history of hypertension." Answer: A Diff: 3 Page Ref: 294 Objective: 11-9 35) You are assessing a female patient with a complaint of shortness of breath. When assessing her skin, which finding should be recognized as abnormal? A) Dry B) Pink C) Moist D) Warm Answer: C Diff: 1 Page Ref: 294 Objective: 11-9 36) For which of these patients would a capillary refill time of 3 seconds be most indicative of poor perfusion? A) A 4-year-old male B) A 24-year-old female C) A 67-year-old female D) An 81-year-old male Answer: A Diff: 2 Page Ref: 294 Objective: 11-8

9 Copyright © 2018 Pearson Education, Inc.

37) You are called for a 2-year-old female child who is sick. When testing her capillary refill, what would be the greatest cause for concern? A) Immediate return of color B) 1-second capillary refill time C) 2-second capillary refill time D) 3-second capillary refill time Answer: D Diff: 2 Page Ref: 294 Objective: 11-8 38) A patient who was shoveling snow developed chest pain, immediately sat down in a nearby bus shelter, and called 911. Prior to moving him into the ambulance, your partner performs a primary assessment and reports that the patient's capillary refill is 5 seconds. Which of these would be an appropriate statement for you to make? A) "That is a significant finding. We really need to get him on oxygen and get moving to the hospital." B) "The cold temperature may cause the capillary refill to be 5 seconds; we should assess further." C) "That finding likely indicates he has lost blood and the skin is not being adequately perfused." D) "If his skin is also warm and moist, then the patient is most likely in shock." Answer: B Diff: 3 Page Ref: 294 Objective: 11-9 39) Which of these statements made by an EMT student indicates that he correctly understands the use of capillary refill when assessing a patient? A) "Capillary refill is a very reliable sign of shock in any patient between the ages of 18 and 65 years." B) "Capillary refill is a more reliable sign of perfusion quality in children younger than 6 years than in adolescents or adults." C) "The possibility of chronic circulatory diseases makes the capillary refill test an excellent sign of circulation in the adult patient." D) "If a delayed capillary refill is observed in a patient of any age, there is a high probability he or she is in shock." Answer: B Diff: 3 Page Ref: 294 Objective: 11-8 40) When assessing a patient's pupils, the EMT should: A) Shine a light in one eye and watch for the pupil to dilate B) Cover one eye and shine a light in the other eye, watching for changes to the size of the pupil C) Determine the size of the pupil and then look for a change in pupillary size while shining a light in the eye D) Instruct the patient to blink several times and observe the size of the pupils Answer: C Diff: 2 Page Ref: 295 Objective: 11-2 10 Copyright © 2018 Pearson Education, Inc.

41) You are assessing a patient who is in bright sunlight after collapsing in the hot sun at a picnic. If you cannot immediately move the patient out of the sun, assessment of the pupils will be made more accurate by: A) Shading the patient's eyes B) Using a blue-tinged light C) Retracting the eyelid D) Hydrating the eyes with saline drops first Answer: A Diff: 2 Page Ref: 295 Objective: 11-10 42) Friends called 911 for a 37-year-old female who intentionally overdosed on a narcotic drug at a college party. Which eye-related findings would reinforce that the patient did indeed take a narcotic? A) Pupils that constrict to light B) Bloodshot eyes with large pupils C) Nonreactive pupils that are unequal D) Pupils that are constricted Answer: D Diff: 3 Page Ref: 295, Table 11-6 Objective: 11-10 43) Assessment of a patient who is unresponsive reveals pupils that are both large at 6 millimeters and do not change size in response to light. The EMT would best document this finding as: A) Bilateral dilation of the pupils, unresponsive to light B) Constricted pupils bilaterally, nonreactive to light C) Bilateral midsize pupils that are reactive to light D) Nonreactive pupils of decreased size bilaterally Answer: A Diff: 2 Page Ref: 295 Objective: 11-10 44) Which of these would indicate a normal pupillary exam has occurred? A) One pupil dilates and the other constricts in response to light B) Both pupils constrict when light is directed into one eye C) The patient reflexively closes the eyes to light D) The pupils are equal and do not change shape in response to light Answer: B Diff: 2 Page Ref: 295 Objective: 11-10

11 Copyright © 2018 Pearson Education, Inc.

45) The EMT is correct when he makes which of these statements about assessment of the pupils? A) "Dilated pupils are less of a concern than pupils that are constricted." B) "If a patient's pupils are dilated but react to light, the pupillary exam is considered normal." C) "Some people naturally have unequal pupils, but both should react to light." D) "Constricted pupils are less of a concern than are dilated pupils." Answer: C Diff: 3 Page Ref: 295 Objective: 11-2 46) A 41-year-old male patient reacts to painful stimuli by moaning. You shine a light at his right pupil only, while looking at both pupils. What would be a normal finding? A) Dilation of the right pupil only B) Constriction of the right and left pupils C) Dilation of the left pupil only D) Constriction of the right pupil only Answer: B Diff: 3 Page Ref: 295 Objective: 11-10 47) As you enter the scene of a medical emergency, the Emergency Medical Responder informs you that the patient is not breathing and has pupils that are fixed and dilated. Based on this description, what do you expect to see when you reach the patient's side? A) Both pupils are large and change shape in response to light B) The patient must wear corrective lenses C) The patient has a past medical history of blindness D) Neither pupil will react when light is directed into the eyes Answer: D Diff: 2 Page Ref: 295 Objective: 11-10 48) To assess a patient's blood pressure, the EMT will need a: A) Pulse oximeter B) Sphygmomanometer C) Pulse oximeter and stethoscope D) Pair of gloves and stethoscope Answer: B Diff: 1 Page Ref: 288 Objective: 11-1

12 Copyright © 2018 Pearson Education, Inc.

49) Your partner is having a difficult time determining normal and abnormal blood pressures for a child. Which of these statements will help him most? A) "Since pediatric emergencies are less common than adult emergencies, it is best to call medical direction and ask what is normal." B) "Any systolic blood pressure of less than 100 mmHg in a child is an emergency." C) "A minimally acceptable systolic pressure can be estimated by doubling the child's age and adding it to 70." D) "It is helpful to ask the parent or caregiver what is normal for the child and compare your reading to that." Answer: C Diff: 3 Page Ref: 297 Objective: 11-12 50) You have placed the blood pressure cuff on the arm of a patient who is short of breath. What is your next step? A) Inflate to a reading of 70 mmHg, check for a radial pulse, and continue inflation until the pulse is no longer felt B) Inflate the cuff to 300 mmHg, and then leave the cuff inflated for 30 seconds prior to deflation C) Inflate the cuff to a reading of 300 mmHg, and then deflate the cuff slowly until you hear a pulse D) Inflate the cuff to three times the patient's age and listen for a heartbeat Answer: A Diff: 2 Page Ref: 298 Objective: 11-12 51) You are observing a new EMT just hired by your EMS agency. You know he has correctly positioned the blood pressure cuff on the patient's arm when: A) The cuff is placed over the antecubital space and elbow B) The cuff is centered over the top of the radial artery C) The cuff is on the same level as the patient's heart D) The cuff covers one-third of the patient's upper arm Answer: C Diff: 2 Page Ref: 298 Objective: 11-12 52) When obtaining a blood pressure, the EMT listens for a pulse over which blood vessel? A) Radial artery B) Antecubital vein C) Carotid vessel D) Brachial artery Answer: D Diff: 1 Page Ref: 298 Objective: 11-12

13 Copyright © 2018 Pearson Education, Inc.

53) An EMT student asks you if it matters whether the blood pressure cuff seems too small for a patient, if a reading can still be obtained. You should respond: A) "As long as the reading is obtained, it should be accurate." B) "Cuffs that are too small provide inaccurately high readings." C) "Using a cuff that is too small can cause damage to the blood vessels." D) "As long as the radial pulse remains intact, the cuff size is irrelevant." Answer: B Diff: 3 Page Ref: 298 Objective: 11-11 54) You observe your partner correctly obtaining a patient's blood pressure when he: A) Inflates the cuff maximally prior to slowly deflating and listening for a pulse B) Deflates the cuff 2 mmHg per second while listening with a stethoscope C) Stops inflation and starts deflation as soon as the carotid pulse disappears D) Places the stethoscope under the cuff and deflates it Answer: B Diff: 2 Page Ref: 298 Objective: 11-12 55) A blood pressure is reported as 116/68 mmHg. Given this finding, which of these statements is true? A) The systolic blood pressure is 116 mmHg B) The top number reflects the diastolic blood pressure C) The systolic pressure is determined by subtracting 68 from 116 D) The diastolic blood pressure is 116 mmHg Answer: A Diff: 1 Page Ref: 296 Objective: 11-1 56) An EMT has an accurate understanding of systolic blood pressure when he tells you that systolic blood pressure is: A) Represented by the bottom number B) Produced when the heart contracts C) The pressure in the veins D) Caused by constriction of the arteries Answer: B Diff: 3 Page Ref: 296 Objective: 11-1

14 Copyright © 2018 Pearson Education, Inc.

57) When obtaining a blood pressure for a patient, the radial pulse disappears when the gauge reads 130 mmHg. When deflating the cuff, the EMT hears a pulse at 118 mmHg. The pulse disappears at 76 mmHg. Given this, which of these statements is true? A) The systolic blood pressure is 118 mmHg B) The diastolic blood pressure is 130 mmHg C) The systolic blood pressure is 76 mmHg D) The diastolic blood pressure is 118 mmHg Answer: A Diff: 2 Page Ref: 298 Objective: 11-11 58) Your partner reports that a patient's blood pressure is 156/78 mmHg. From this reading, you realize: A) The pulse pressure is 224 mmHg B) The systolic blood pressure is 78 mmHg C) The constant pressure in the veins is 156 mmHg D) The diastolic blood pressure is 78 mmHg Answer: D Diff: 1 Page Ref: 296 Objective: 11-11 59) An EMT has an accurate understanding of diastolic blood pressure when she tells you that diastolic pressure: A) Is the pressure in the arteries when the heart is not contracting B) Can be estimated as one-third of the systolic blood pressure C) Can be easily obtained by palpating the blood pressure D) Should always be rounded to the nearest 10 Answer: A Diff: 3 Page Ref: 296 Objective: 11-1 60) Which pulse site is commonly palpated during the process of obtaining a blood pressure by the palpation technique? A) Brachial B) Radial C) Carotid D) Pedal Answer: B Diff: 1 Page Ref: 298 Objective: 11-12

15 Copyright © 2018 Pearson Education, Inc.

61) Which of these would be an indication to palpate the blood pressure? A) Noisy environment B) Normal pulse rate C) Tachycardic pulse rate D) Non-life-threatening condition Answer: A Diff: 2 Page Ref: 298 Objective: 11-12 62) While reviewing a patient care report for a quality assurance process, the EMT should recognize that a blood pressure has been palpated when she sees: A) Palp: 178/118 mmHg B) P / 118 C) 118/178 (palpated) D) 178 / P Answer: D Diff: 2 Page Ref: 298 Objective: 11-11 63) Which of these statements indicates an appropriate understanding of palpating a blood pressure? A) "A palpated blood pressure is recorded when the brachial pulse fades out as the BP cuff is deflated." B) "Palpated blood pressure is the technique of choice when the patient has a cardiac complaint." C) "A palpated blood pressure is typically a little lower than a blood pressure obtained by auscultation." D) "It is best to determine a palpated blood pressure using a stethoscope and pulse oximeter." Answer: C Diff: 3 Page Ref: 298 Objective: 11-11 64) Your partner states that he wants to palpate the blood pressure. Which equipment would you hand him? A) Blood pressure cuff B) Stethoscope C) Blood pressure cuff and stethoscope D) Stethoscope and pulse oximeter Answer: A Diff: 1 Page Ref: 298 Objective: 11-12

16 Copyright © 2018 Pearson Education, Inc.

65) Which of these questions would be asked when obtaining a medical history, using the SAMPLE mnemonic, for a female patient who is crying and complaining of dizziness? A) "What were you doing when the dizziness started?" B) "Who is your doctor and when was your last office visit?" C) "Why exactly did you call the ambulance today?" D) "You seem upset. Do you want to talk about it?" Answer: A Diff: 2 Page Ref: 307 Objective: 11-20 66) When obtaining a medical history using the SAMPLE mnemonic, which of these statements relates to the "P" component? A) "The physician is Dr. Coleman." B) "The pulse is 116 beats per minute." C) "The pain is rated at 8/10." D) "There is a history of pancreatitis." Answer: D Diff: 1 Page Ref: 307 Objective: 11-20 67) Which of these descriptions would be obtained when asking about the "A" component of the SAMPLE history? A) Abdomen soft B) History of asthma C) Allergy to penicillin D) Airway is open Answer: C Diff: 2 Page Ref: 306 Objective: 11-20 68) A 69-year-old man complains of weakness to the left arm and leg. He also states that he is nauseated and has a headache. The patient's past medical history includes stroke and diabetes. In relation to this information, which of these statements is true? A) The headache is a symptom related to the chief complaint B) The history of diabetes is a sign related to the chief complaint C) The dizziness is a sign indicating the severity of the chief complaint D) The history of diabetes is a contributing symptom of the chief complaint Answer: A Diff: 2 Page Ref: 305 Objective: 11-18

17 Copyright © 2018 Pearson Education, Inc.

69) A patient states that he is short of breath, is light-headed, and has chest pain that worsens when he takes a deep breath. Assessment reveals an open airway, adequate breathing, and a rapid pulse of 120 beats/min. The patient's skin is hot to the touch, and he has a blood pressure of 116/84 mmHg. The patient informs you that he was diagnosed with pneumonia 3 days ago and has not been taking the prescribed antibiotics. Which of these assessment findings would the EMT best recognize as a sign related to the patient's chief complaint? A) Chest pain that worsens with inspiration B) Recent diagnosis of pneumonia C) Complaint of light-headedness D) Blood pressure of 116/84 mmHg Answer: D Diff: 2 Page Ref: 306 Objective: 11-18 70) The EMT is correct when he identifies baseline vital signs as: A) A set of vital signs most close to normal B) The most important set of vital signs C) The first set of vital signs obtained D) Any change in two consecutive sets of vital signs Answer: C Diff: 1 Page Ref: 288 Objective: 11-1 71) You have assisted the patient in taking one of his nitroglycerin tablets due to cardiac-type chest pain. Five minutes later, you note the patient's blood pressure is 108/74 mmHg. To determine if the nitroglycerin has affected the blood pressure, the EMT should: A) Take another blood pressure measurement in five minutes B) Compare this finding to the baseline blood pressure C) Ask the patient if he feels his blood pressure has changed D) Retake the blood pressure and compare it to a normal BP of 120/80 mmHg Answer: B Diff: 1 Page Ref: 288 Objective: 11-12 72) Which of these statements is true regarding vital signs obtained from a patient with nausea and vomiting? A) If the vital signs are within normal limits, it is safe to let the patient refuse transport B) The vital signs will help the EMT determine the relative stability or instability of the patient C) The patient's complaint is considered significant only if accompanied by abnormal vital signs D) The vital signs will allow the EMT to diagnose the cause of the nausea and vomiting Answer: B Diff: 3 Page Ref: 288 Objective: 11-2

18 Copyright © 2018 Pearson Education, Inc.

73) An obese female patient has called you with a complaint of abdominal pain. The regular adult cuff will not fit around her arm, so you retrieve the large adult cuff from your supplies. Embarrassed, the woman says, "My arm is so fat. Do you really need to roll up my sleeve to get my blood pressure?" Your best response would be: A) "If you do not want me to take your blood pressure, I do not have to." B) "I have seen plenty of big arms, and yours is nothing compared to some of them." C) "I am getting your blood pressure, not measuring your arm, but you really need to lose some weight." D) "I am not concerned with the size of your arm. I am interested in your blood pressure." Answer: D Diff: 2 Page Ref: 303-304 Objective: 11-17 74) A female patient called 911 when she noticed blood in her stool, and became scared and anxious. Her pulse rate is 124 and her blood pressure is 88/60 mmHg. After you get her vital signs, she nervously asks you how they are. Given her anxiety, you should: A) Change the subject B) Inform her of the vital signs C) Tell her they are normal D) Reassure her that she is fine Answer: B Diff: 2 Page Ref: 304 Objective: 11-17 75) You notice that your partner routinely takes only one set of vital signs when treating and transporting patients. When asked, he states that he gets just baseline vital signs because they are most important, as they provide information related to the patient's clinical status at the time of the EMS call. How would you respond? A) "Examining several sets of vital signs can also indicate the effectiveness of your treatment." B) "You must at least get a pulse rate every 15 minutes, since this is the most important vital sign." C) "Documenting several sets of vital signs is the only way to get full reimbursement from insurance companies for the EMS call." D) "You must get at least two sets of vital signs because that is what the national standard calls for." Answer: A Diff: 2 Page Ref: 288 Objective: 11-2

19 Copyright © 2018 Pearson Education, Inc.

76) A female patient with a history of asthma called 911 with a complaint of shortness of breath. On scene, you assisted her with the administration of her metered-dose inhaler containing albuterol. What can the EMT do to best determine if this patient's condition is improving? A) Ask the patient if she finds it easier to breathe B) Inquire if the patient feels the need for another dose of albuterol C) Obtain a pulse oximetry reading and breath sounds every 5 minutes D) Re-obtain a SAMPLE history and look for new information Answer: C Diff: 2 Page Ref: 300 Objective: 11-5 77) The EMT understands the primary reason to obtain a medical history using the SAMPLE mnemonic when he states: A) "The SAMPLE mnemonic helps EMTs diagnose a patient's medical problem." B) "It is important to use the SAMPLE mnemonic because the emergency department will need the information." C) "After performing the primary assessment, the SAMPLE mnemonic helps guide the EMT in further assessment and care." D) "The SAMPLE mnemonic is important because it will give the EMT the patient's exact chief complaint." Answer: C Diff: 2 Page Ref: 306 Objective: 11-20 78) You are assessing a 49-year-old male patient who complains of lower back pain. Which finding would be pertinent medical information related to the back pain that the EMT will obtain using the SAMPLE mnemonic? A) Heart rate of 48 and cool, clammy skin B) Childhood case of measles C) Last tetanus shot 5 years ago D) Arthritis in the spine and pelvis Answer: D Diff: 2 Page Ref: 307 Objective: 11-18 79) After you apply the pulse oximeter to a patient who complains of chest discomfort, she asks you what you are assessing. You would respond by telling her that the pulse oximeter: A) Provides the most accurate evaluation of the true heart rate B) Is a tool used to see if the airway needs to be opened C) Provides information about the amount of oxygen circulating in the blood D) Can tell how much carbon dioxide she is exhaling with each breath Answer: C Diff: 3 Page Ref: 299 Objective: 11-15

20 Copyright © 2018 Pearson Education, Inc.

80) When using a pulse oximeter on an adult patient, the EMT knows that the sensor is most often attached to the patient's: A) Chest B) Forehead C) Ear D) Finger Answer: D Diff: 1 Page Ref: 299 Objective: 11-15 81) A pulse oximetry reading obtained on room air is considered normal when it is: A) 100 percent B) Greater than 97 percent C) Greater than 90 percent D) Greater than 75 percent Answer: B Diff: 1 Page Ref: 300 Objective: 11-15 82) How should the EMT document a pulse oximetry reading on the prehospital care report? A) 97% SpO2 B) 97% O2 C) 97% PO D) 97% PO2 Answer: A Diff: 1 Page Ref: 300 Objective: 11-15 83) You have arrived by the side of a cyanotic patient who is responsive only to painful stimuli and exhibiting stridorous respirations. What is the most appropriate action by the EMT? A) Prepare and apply the pulse oximeter before administering oxygen B) Provide positive pressure ventilation with supplemental oxygen C) Obtain a full set of vital signs, excluding the patient's temperature D) Attempt to determine what happened to the patient or any existing medical problems Answer: B Diff: 2 Page Ref: 289 Objective: 11-5 84) When should the EMT use the pulse oximeter? A) Only on patients who complain of shortness of breath B) On any and all patients 1 year of age and older C) Only if the patient has a history of lung disease D) Routinely on all patients with a medical or trauma complaint Answer: D Diff: 1 Page Ref: 300 Objective: 11-15 21 Copyright © 2018 Pearson Education, Inc.

85) In patients with which of these conditions would the EMT most likely get an inaccurate pulse oximeter reading despite proper application of the sensor? A) Low blood pressure B) Elevated body temperature C) Unresponsiveness D) Increased heart rate Answer: A Diff: 3 Page Ref: 300 Objective: 11-15 86) You are trying to get a pulse oximeter reading on an 18-month-old child with difficulty breathing. Every time you place the sensor on the child's finger, he becomes upset and removes it. What is your best course of action? A) Place the sensor on the child's toe B) Restrain the child until a reading is obtained C) Place the sensor on the other hand D) Attach the sensor to his neck Answer: A Diff: 2 Page Ref: 299 Objective: 11-15 87) You are caring for a 5-year-old male patient with a complaint of difficulty breathing. Your assessment indicates that the patient is tachypneic with labored breathing. While gaining more information about the patient's respiratory function, which finding would you recognize as most important? A) Nasal congestion and runny nose B) Skin retractions between the ribs during inhalation C) Skin that is hot and dry D) Systolic blood pressure of 92 mmHg Answer: B Diff: 2 Page Ref: 289 Objective: 11-3 88) Where would the EMT palpate for a popliteal pulse? A) Posterior ankle B) Upper arm C) Behind the knee D) Top of foot Answer: C Diff: 1 Page Ref: 290 Objective: 11-6

22 Copyright © 2018 Pearson Education, Inc.

89) Which of these patients should the EMT recognize as tachycardic for his or her age bracket? A) An 86-year-old male with chest pain and a heart rate of 96 B) A 2-year-old female with vomiting and a heart rate of 116 C) A 24-year-old female with diabetic complications and a heart rate of 54 D) A 37-year-old male with a complaint of fatigue and a heart rate of 104 Answer: D Diff: 1 Page Ref: 291, Table 11-3 Objective: 11-7 90) When obtaining vital signs for a 67-year-old male patient with dizziness and nausea, your partner states that the pulse seems to disappear periodically when the patient takes a deep breath. As a knowledgeable EMT, your best response would be: A) "Given the patient's age, that is a typical finding and not an immediate concern." B) "That is an important finding. We will have to let the doctor know about it." C) "Let's move the patient to the stretcher for transport. That is an early sign of cardiac arrest." D) "That finding is important because it suggests the radial artery is partially blocked." Answer: B Diff: 2 Page Ref: 292 Objective: 11-7 91) Which of these blood pressure readings is normal for a 7-year-old male? A) 114/62 mmHg B) 120/80 mmHg C) 76/50 mmHg D) 94/30 mmHg Answer: A Diff: 3 Page Ref: 297 Objective: 11-12 92) Which of these blood pressures represents diastolic hypertension in an adult patient? A) 240/88 mmHg B) 158/44 mmHg C) 136/92 mmHg D) 246/Palpation Answer: C Diff: 2 Page Ref: 297 Objective: 11-11

23 Copyright © 2018 Pearson Education, Inc.

93) An 86-year-old female patient has called 911 for rectal bleeding. There is a large amount of dark red blood and clots in the toilet and on the patient's clothing. The Emergency Medical Responder reports a blood pressure of 84/68 mmHg with a heart rate of 124 beats/min. Given this information, the EMT should expect findings of: A) A narrowed pulse pressure and possible shock B) Normal blood pressure since the patient is alert and oriented C) Diastolic hypertension and risk for stroke D) A normal heart rate given the patient's age Answer: A Diff: 3 Page Ref: 297 Objective: 11-11 94) A mother has called 911 because her 2-year-old daughter is fussy and not eating. When assessing the toddler, what would be the best indication of her perfusion status? A) Heart rate B) Skin color and temperature C) Palpated blood pressure D) Auscultation of breath sounds Answer: B Diff: 1 Page Ref: 293-294 Objective: 11-8 95) When you are obtaining the blood pressure of a patient in a standing position, the patient states that he suddenly feels weak and is going to pass out. Your immediate action should be to: A) Place the patient back into bed B) Determine the blood pressure by palpation C) Ask the patient if he is having chest pain D) Hold the patient upright until the blood pressure is obtained Answer: A Diff: 1 Page Ref: 299 Objective: 11-13 96) You are unable to auscultate a blood pressure in a patient's right arm. What should you do next to obtain the most accurate baseline assessment of this patient's vital signs? A) Move the blood pressure cuff down to the forearm B) Palpate the blood pressure in the right arm C) Use an automatic blood pressure monitor D) Auscultate the blood pressure in the left arm Answer: D Diff: 2 Page Ref: 298 Objective: 11-12

24 Copyright © 2018 Pearson Education, Inc.

97) Medical direction has asked that you obtain orthostatic vital signs for a 54-year-old female patient who complains of dizziness and weakness. Her baseline vital signs obtained while she was sitting in a chair were pulse, 84; respirations, 26 breaths/min; blood pressure, 118/62 mmHg; and SpO2, 95%. Which one set of vital signs upon reassessment would indicate that this patient has a positive orthostatic test? A) Supine blood pressure of 100/64 mmHg and respirations of 22 breaths/min B) Standing blood pressure of 92/54 mmHg and heart rate of 106 beats/min C) Standing pulse of 88 with the additional complaint of nausea D) Standing SpO2 of 88% and heart rate of 96 beats/min Answer: B Diff: 2 Page Ref: 299 Objective: 11-13 98) Correctly assessing orthostatic vital signs involves: A) Taking the blood pressure and heart rate three times, 2 minutes apart B) Determining whether the patient's pulse disappears during deep inspiration C) Taking the blood pressure in the supine, seated, and standing positions D) Determining whether the patient's respirations and heart rate are within normal limits Answer: C Diff: 1 Page Ref: 299 Objective: 11-13 99) When you are applying the pulse oximeter sensor to a patient's finger, he asks you what this device is for. Your response would be: A) "It provides us with the most accurate measure of your heart rate." B) "It measures the amount of oxygen being carried by your blood cells." C) "It tells us how well your heart is pumping blood." D) "It allows us to monitor your blood pressure and respiratory status." Answer: B Diff: 1 Page Ref: 299-300 Objective: 11-15 100) You have applied the pulse oximeter to a 73-year-old male patient who complains of weakness and heart palpitations. Your partner reports that the patient's breath sounds are clear and equal, with a respiratory rate of 16 breaths/min and skin that is warm but pale in color. He then states the following vital signs: pulse, 92; blood pressure, 168/70 mmHg; and SpO2, 92% on room air. Given this information, which of these instructions would be most appropriate? A) "Let's recheck the SpO2 in 2 minutes for any change." B) "The reading must be wrong since he displays no cyanosis." C) "The SpO2 is probably normal for him since he is very pale." D) "Let's initiate oxygen at 2 liters per minute via a nasal cannula." Answer: D Diff: 2 Page Ref: 300 Objective: 11-15

25 Copyright © 2018 Pearson Education, Inc.

101) You cannot obtain a reading on the pulse oximeter after applying the sensor to the patient's finger. What would be the most common reason for this difficulty? A) The patient's pulse is faster than 100 beats per minute B) The patient's hands are cold C) The patient is hypertensive D) The patient's oxygen saturation is less than 90% Answer: B Diff: 2 Page Ref: 300 Objective: 11-15 102) Which step should be taken by the EMT to obtain an accurate pulse oximetry measurement? A) Elevate the patient's hand above his heart B) Secure the probe to the patient's finger with tape C) Remove nail polish from the patient's fingernail if present D) Administer oxygen before assessing the oxygen saturation Answer: C Diff: 1 Page Ref: 300 Objective: 11-15 103) The EMT is correctly using a noninvasive blood pressure monitor when she: A) Auscultates a blood pressure before applying and activating the device B) Alternates arms for each blood pressure reading that is obtained with the device C) Confirms each reading on the device by palpating a pressure in the opposite arm D) Places the cuff of the noninvasive BP monitor on the patient's forearm Answer: A Diff: 2 Page Ref: 301-302 Objective: 11-12 104) The fire department has requested your assistance at a residence where the family's carbon monoxide (CO) detector has been going off. There are three patients. You first assess a 28-yearold female patient who states she has a sinus headache but otherwise feels fine. Her vital signs are pulse, 90; respirations, 16 breaths/min; blood pressure, 118/60 mmHg; and SpO2, 100%. Given these assessment findings, you should immediately: A) Inform her that the CO detector has malfunctioned B) Administer high-concentration oxygen C) Call for advanced life support assistance D) Recheck the oxygen saturation on the patient's opposite hand Answer: B Diff: 2 Page Ref: 300 Objective: 11-15

26 Copyright © 2018 Pearson Education, Inc.

105) You have been dispatched to a residence for a 14-year-old female patient complaining of abdominal pain. As you enter the living room, you see the conscious patient lying on a couch with her father and an Emergency Medical Responder at her side. Which action should the EMT perform first? A) Determine the chief complaint B) Ask her father to leave the room C) Introduce yourself and your partner D) Get permission to treat the patient Answer: C Diff: 2 Page Ref: 303 Objective: 11-17 106) A 43-year-old female has accidentally taken too much of her antidepressant medication and is now confused. Emergency Medical Responders are on scene and providing the patient with high-concentration oxygen. As you arrive at the patient's side, your first action should be to: A) Obtain vital signs B) Perform a primary assessment C) Get a pulse oximeter reading D) Determine the type of medication taken Answer: B Diff: 2 Page Ref: 303 Objective: 11-17 107) You are attempting to get a history from a 56-year-old female patient who is short of breath. When you ask questions, the daughter continually answers for the patient. To best handle this situation, you should: A) Have the Emergency Medical Responders remove the daughter B) Move the patient to the ambulance for the rest of the assessment C) Contact medical direction for advisement D) Explain to the daughter it is important for the patient to answer whenever possible Answer: D Diff: 1 Page Ref: 305 Objective: 11-18 108) You have been called to a bar for a patient who was involved in an altercation and is complaining of abdominal pain. Law enforcement is present. The patient has been handcuffed and is sitting on the sidewalk. He has an odor of alcohol on his breath, his speech is slurred, and he is angrily yelling at the police officers. Which action would be most appropriate for the EMT to take when talking to the patient? A) Maintain eye contact to help establish a rapport B) Place your hand on the patient's shoulder to keep him subdued C) Stand and communicate with the patient from 3 to 4 feet away D) Stand above the patient to establish authority Answer: A Diff: 2 Page Ref: 304 Objective: 11-17 27 Copyright © 2018 Pearson Education, Inc.

109) You have been dispatched to a residence for a female patient with unknown injuries. At the patient's side, you quickly become aware that the 43-year-old female was assaulted by her husband. The husband comes into the room and states, "Don't you touch her, or I will take care of you, too!" What should be the EMT's immediate action? A) Control any active bleeding, and then leave the house B) Stay with the patient and contact law enforcement C) Leave the house immediately, taking the patient with you, if possible D) Restrain the husband so that you can care for the patient Answer: C Diff: 2 Page Ref: 304-305 Objective: 11-17 110) An alert and oriented 87-year-old female patient has fallen and suffered a 3-cm laceration to the back of her head. According to the patient, her walker became caught in the carpet, causing her to trip. When obtaining a medical history, who should the EMT address first? A) Family members B) The patient C) EMRs on scene D) The patient's primary care doctor Answer: B Diff: 1 Page Ref: 304 Objective: 11-17 111) Which of these is an example of an open-ended question? A) "Why did you call 911?" B) "Does your chest hurt?" C) "Are you dizzy?" D) "Did you take your medications today?" Answer: A Diff: 1 Page Ref: 305 Objective: 11-19 112) A patient with chest pain informs you that the pain is greater when he takes a deep breath. The EMT is using the active listening technique of "facilitation" and "clarification" when he responds by saying: A) "I see. Can you describe what the pain feels like?" B) "I understand it hurts. Please go on." C) "Hmmm. That really seems to hurt a lot." D) "So what you are saying is that it hurts more when you breathe?" Answer: A Diff: 1 Page Ref: 306 Objective: 11-19

28 Copyright © 2018 Pearson Education, Inc.

113) Which of these statements made by a patient would the EMT regard as a symptom? A) "I have a history of high blood pressure that I take medicine for." B) "There is a bruise to my left elbow." C) "I slipped and fell down the stairs." D) "My pain is sharp, the worst I've ever felt." Answer: D Diff: 2 Page Ref: 306 Objective: 11-18 114) When using the SAMPLE mnemonic, which of these statements made by the patient would be categorized under the letter "M"? A) "I was diagnosed and treated for breast cancer." B) "I take Zocor, but I am not sure why." C) "I have never had a surgical procedure." D) "My doctor's name is Dr. Hansen." Answer: B Diff: 2 Page Ref: 307 Objective: 11-20 115) Which of these pieces of information would be obtained by the EMT when getting a history using the SAMPLE mnemonic? A) Pulse, 144; respiration, 16 breaths/min; BP, 132/88 mmHg B) No life threats to the airway, breathing, or circulation C) Reason for calling for an ambulance D) Sensation of dizziness prior to falling Answer: D Diff: 1 Page Ref: 306-307 Objective: 11-20 116) The EMT is appropriately using the SAMPLE mnemonic when he asks which question? A) "Who is your doctor?" B) "When did you eat last?" C) "Have you thought about stopping smoking?" D) "Why did you call for the ambulance?" Answer: B Diff: 1 Page Ref: 307 Objective: 11-20 117) The purpose of the OPQRST mnemonic is to guide the EMT in: A) Getting a past medical history B) Determining the stability of vital signs C) Exploring the patient's chief complaint D) Establishing any allergies to medications Answer: C Diff: 1 Page Ref: 307-308 Objective: 11-18 29 Copyright © 2018 Pearson Education, Inc.

118) When using the OPQRST mnemonic, the EMT is getting information as it applies to the "P" category when she asks: A) "Do you feel more short of breath when you are lying down?" B) "Does the chest pain spread to your back?" C) "What time did the symptoms start?" D) "Can you rate the pain on a scale of 1 to 10?" Answer: A Diff: 2 Page Ref: 307 Objective: 11-20 119) Which of these questions would the EMT ask when using the OPQRST mnemonic? A) "Do you have any medical conditions?" B) "Have you ever had any surgeries?" C) "Did you take any medications today?" D) "Does the pain feel dull or sharp?" Answer: D Diff: 2 Page Ref: 307 Objective: 11-20 120) When assessing a patient who does not speak the same language as you or your partner, the best approach in the immediate care of the patient is to: A) Contact medical direction to get physician input B) Transport the patient to a hospital with interpreters for her language C) Try to find a neighbor who speaks the patient's language D) Request that a family member act as an interpreter Answer: D Diff: 1 Page Ref: 309 Objective: 11-21 121) A patient who appears to be intoxicated has fallen and is complaining of pain to his right arm. He is loud and using profane language. What should be your initial approach to the patient? A) "If you do not quiet down, I will call the police and they will arrest you." B) "Let me put an ice pack on your wrist. It will help it feel better." C) "Stop yelling or I will have to strap you to the stretcher." D) "I cannot help you if you keep yelling like that!" Answer: B Diff: 2 Page Ref: 309 Objective: 11-21

30 Copyright © 2018 Pearson Education, Inc.

122) You are transporting a depressed patient who states that she is thinking of killing herself by taking a "whole bunch of pills." After providing you this information, the patient has refused to speak and has remained silent. Your physical assessment of the patient reveals she is stable. Which of these statements would be most appropriate for you to make at this time? A) "I don't understand why you won't talk. You called 911 for help, didn't you?" B) "Has the cat got your tongue? If you talk to me, I can help you." C) "I understand if you do not want to talk. I will be sitting here if you need anything." D) "I need you to talk to me. I cannot help you if you refuse to talk to me." Answer: C Diff: 1 Page Ref: 309 Objective: 11-21 123) The EMT is correctly palpating a central pulse when he palpates the: A) Radial pulse B) Femoral pulse C) Brachial pulse D) Posterior tibial pulse Answer: B Diff: 1 Page Ref: 290 Objective: 11-6 124) The EMT is reviewing a patient care report and finds the following documentation "ETCO2 is 43 mmHg." What does this mean? A) The patient is intubated B) The exhaled carbon dioxide level is normal C) The hemoglobin saturation with carbon dioxide is normal D) The patient is hypoxic and in need of oxygen Answer: B Diff: 1 Page Ref: 302 Objective: 11-16 125) Your EMT student has a good understanding of end-tidal carbon dioxide monitoring when she states: A) "The normal value should be 45 to 55 mmHg." B) "End-tidal carbon dioxide monitoring should be used only when the patient is being artificially ventilated." C) "Any disturbance to the breathing, circulation, or metabolic state of the patient can alter the reading." D) "Only paramedics and AEMTs are allowed to utilize end-tidal capnography." Answer: C Diff: 2 Page Ref: 302 Objective: 11-16

31 Copyright © 2018 Pearson Education, Inc.

126) You believe a patient for whom you are caring is starting to experience respiratory failure. Which of these findings is supportive of this suspicion? A) ETCO2 of 56 mmHg B) SpO2 of 94% C) Heart rate of 118 beats/min D) Bilateral wheezing on exhalation only Answer: A Diff: 2 Page Ref: 302-303 Objective: 11-16 127) You and a paramedic partner are caring for a patient who overdosed on a drug that has caused her to become apneic. Your EMT student is providing BVM ventilations with supplemental oxygen via an endotracheal tube placed by the paramedic. The patient still has a good pulse and acceptable blood pressure. The pulse oximeter reads 95% and the ETCO2 monitor reads 28 mmHg. Given these findings, the EMT should: A) Administer a tube of oral glucose since the patient's airway is protected B) Increase the amount of oxygen being delivered to the BVM reservoir C) Ask the EMT student ventilating the patient to slow the ventilatory rate D) Place the patient in semi-Fowler's position to help reduce intracranial pressure Answer: C Diff: 3 Page Ref: 302-303 Objective: 11-16 128) You and a paramedic partner are caring for a patient who overdosed on a drug that has caused her to become apneic. Your EMT student is providing BVM ventilations with supplemental oxygen via an endotracheal tube placed by the paramedic. How often should vital signs be reassessed on this patient? A) Once at the scene and once en route to the ED B) Only baseline vital signs are required C) Every 5 minutes D) Every 15 minutes Answer: C Diff: 3 Page Ref: 299 Objective: 11-14

32 Copyright © 2018 Pearson Education, Inc.

Prehospital Emergency Care, 11e (Mistovich et al.) Chapter 12 Scene Size-Up 1) You are entering the scene where a 32-year-old intoxicated male has fallen down 13 stairs leading to the basement. He is seated on the floor of the basement, holding a bloody towel over his face. During the initial scene size-up, what is it essential that the EMT quickly identify? A) The patient's level of consciousness B) Possible need for additional personnel C) Status of the patient's airway and breathing D) Amount of alcohol consumed by the patient Answer: B Diff: 1 Page Ref: 315 Objective: 12-5 2) It has been reported that two people have been shot in an alleyway. As you pull up to the scene, you see a police officer standing over a person with blood on his shirt and pants. In this situation, what is your first priority? A) Assessing the patient's airway for patency B) Determining the number of patients on scene C) Considering the need for additional resources D) Determining the overall safety of the scene Answer: D Diff: 2 Page Ref: 315 Objective: 12-2 3) When should the EMT evaluate the scene of an emergency for hazards that may threaten or harm him? A) Prior to leaving the ambulance B) After completing the primary assessment C) Throughout the entire call D) Before responding to the call Answer: C Diff: 1 Page Ref: 315 Objective: 12-2 4) Which personal protective equipment must the EMT take on every call? A) Gloves B) Gloves and face shield C) Gloves, gown, and goggles D) Goggles and gown Answer: A Diff: 1 Page Ref: 316 Objective: 12-1

1 Copyright © 2018 Pearson Education, Inc.

5) You have been dispatched for an 89-year-old female patient who fell in her kitchen and was found by family members several hours later. Should something go awry on scene, which of these serves to give you the most safety and security? A) Notifying dispatch that you are at the residence B) Ensuring that lights are on throughout the residence C) Backing the ambulance into the driveway for rapid egress D) Carrying a portable radio with you into the house Answer: D Diff: 2 Page Ref: 318 Objective: 12-6 6) What is the most common potential hazard an EMT faces on scene? A) Blood and body fluids B) Aggressive patients C) Upset family members D) Unfamiliar settings Answer: A Diff: 1 Page Ref: 316 Objective: 12-2 7) Dispatch has sent you to a residence for a female patient with a possible broken arm. On scene, you find the patient seated in a chair holding her left arm. Her arm appears deformed and she has blood and bruises on her face. She tells you that an intruder, whom she believes lives in her neighborhood, broke into her home and robbed her. Prior to fleeing, he beat her, leaving her as you find her. With this information, you would immediately: A) Leave the patient and scene and wait in the ambulance until the police arrive B) Determine the identity of the intruder and contact and direct the police to his location C) Contact law enforcement, assess the patient, and initiate care D) Recognize a crime scene and not touch the patient to avoid altering any evidence Answer: C Diff: 2 Page Ref: 315 Objective: 12-4 8) A 67-year-old male patient who lives in an extremely "bad" neighborhood is experiencing chest pain. There are vacant homes throughout the area, including the homes to either side of his residence. Additionally, parked/abandoned cars have forced you to park three houses away, making it necessary to walk and carry supplies to the residence. Which action is most appropriate as you approach and/or gain access to the patient's house? A) Stand in front of the door while knocking to make your presence known B) Walk single file with your partner behind you carrying the medical bag C) Walk on the sidewalk instead of the grass to ensure steadier footing D) Hold a lit flashlight in front of you to illuminate shrubbery and trash cans Answer: B Diff: 2 Page Ref: 322 Objective: 12-3

2 Copyright © 2018 Pearson Education, Inc.

9) On scene, frantic family members direct you into the basement where their 67-year-old mother has shot herself in the chest with a small handgun. She is unresponsive and breathing 6 times per minute. The gun remains in her hand by her side. Which action should you take immediately? A) Pick up the gun by the edge of the grip and carefully remove it, and advise the police of your action B) Do not touch the patient or gun until the police arrive on scene C) Leave the gun in the patient's hand and start providing emergency care D) Determine whether the gun contains ammunition and remove it if it is loaded Answer: A Diff: 2 Page Ref: 323 Objective: 12-6 10) It is nighttime and you have been dispatched for a vehicle on the shoulder of the interstate with a driver slumped over the steering wheel. When pulling up onto the scene, how should you best position the ambulance? A) In front of the car with the white strobe lights activated B) Behind the car with your high-beams on C) Next to the car to block one lane of traffic D) Behind the car with the ambulance wheels turned toward the guard rail Answer: B Diff: 2 Page Ref: 324 Objective: 12-4 11) You are providing care to a patient with altered mental status amid a large crowd gathered for a concert. Which characteristic of the crowd should concern you the most in relation to personal safety? A) An angry mood developing within the crowd B) A growing number of people making up the crowd C) The arrival of news vans to "cover" the story D) The presence of alcohol and possibly drugs in the crowd Answer: A Diff: 2 Page Ref: 325-326 Objective: 12-6 12) An intoxicated patient is complaining of a severe headache and a large bruise to the left side of his face after accidentally being struck with a baseball bat during a family picnic softball game. In this situation, being struck with the baseball bat is called the: A) Nature of illness B) Chief complaint C) Mechanism of injury D) Differential diagnosis Answer: C Diff: 2 Page Ref: 326-327 Objective: 12-7

3 Copyright © 2018 Pearson Education, Inc.

13) Dispatch has sent you to the scene of a fight at a local sporting event. Since the EMS system is busy and there are very few ambulances available, dispatch has instructed you to call for additional units after arrival on scene. On scene, you find that the police are present and what appear to be 10 to 12 people are seated or lying on the ground, many with blood on their faces and clothing. When would you call for additional ambulances? A) Once all patients have been triaged B) After assessing and treating life-threatening injuries C) Once the exact number of patients has been determined D) Before getting out of the ambulance Answer: D Diff: 2 Page Ref: 329 Objective: 12-5 14) When would an EMT enter the scene of a crime to provide care to the patient? A) When an additional crew arrives on scene B) When all hazards have been addressed C) When the police return their guns to their holsters D) When the dispatcher informs the crew to enter Answer: B Diff: 2 Page Ref: 322 Objective: 12-4 15) If a scene becomes unsafe after you have entered, you should first: A) Attempt to neutralize the threat B) Call for assistance on the radio C) Remove yourself from the scene D) Notify the EMS supervisor Answer: C Diff: 2 Page Ref: 318 Objective: 12-3 16) A 67-year-old male patient, who has a history of diabetes and previous heart attack, fell down three stairs. His chief complaint is pain and deformity to his right wrist. The EMT would initially classify this patient as a: A) Medical patient B) Trauma patient C) Medical and trauma patient D) Special needs patient Answer: B Diff: 1 Page Ref: 326 Objective: 12-7

4 Copyright © 2018 Pearson Education, Inc.

17) You and your partner are first on the scene of a bleacher collapse at a local raceway. As you exit the ambulance, you note several patients seated throughout the debris from the collapsed structure. In this situation, which action would you perform first? A) Determine the severity of injuries B) Wait for additional units to arrive C) Determine the number of patients D) Notify the local hospitals of a mass-casualty incident Answer: C Diff: 2 Page Ref: 315, 329 Objective: 12-8 18) Which of these emergency responders have articles of personal protective equipment in place? A) An EMT with gloves, gown, and eye protection B) An Emergency Medical Responder with leather gloves and steel-toed boots C) A firefighter with a self-contained breathing apparatus unit on D) All the responders have personal protective equipment in place. Answer: D Diff: 1 Page Ref: 316 Objective: 12-1 19) At the scene of a motor vehicle collision involving two vehicles on a narrow two-lane road, which actions would be most appropriate to ensure rescuer and patient safety? A) Place flares in each direction from the collision scene B) Remove reflective clothing to avoid distracting passing vehicles C) Turn your reflective back to oncoming traffic so those drivers are aware of your presence D) Treat the patients between the vehicles so all rescuers can see each other Answer: A Diff: 1 Page Ref: 319 Objective: 12-6 20) A young male patient was playing hockey on an ice-covered pond and became injured; he remains on the ice. As you arrive on scene, a park ranger informs you that the ice is 12 inches thick and poses no danger of collapse. In this situation, which action would be most appropriate to help keep you safe while accessing and extricating the patient? A) Have a teammate assist the injured player to your location B) Apply salt to the ice to provide traction C) Place tarps on the ice to walk over to the injured player D) Take small slow steps while walking on the ice Answer: C Diff: 1 Page Ref: 320 Objective: 12-6

5 Copyright © 2018 Pearson Education, Inc.

21) You have been called to a residence of a woman who fell down the stairs. On scene, you find the patient on the couch with a large laceration to her forehead. As you begin assessing and treating her, she states that she did not fall down the stairs, but rather was assaulted by her exboyfriend after inviting him into her home. He has since left the residence prior to your arrival. In this situation, which statement would be most appropriate? A) "What is the name of your ex-boyfriend?" B) "Aside from your head, do you hurt anywhere else?" C) "I am not going to bandage your head so we can preserve evidence." D) "Why did you invite your ex-boyfriend over to your house?" Answer: B Diff: 1 Page Ref: 326 Objective: 12-7 22) You have been called to a bar full of intoxicated patrons for a 46-year-old man who fell off his stool, injuring his left arm. As you assess his arm, the patient screams in pain. Several of the patrons get angry and yell for you to give him something for pain. Someone in the back throws a bottle in your direction. What should you do next? A) Leave the scene and return only when police have made it safe to do so B) Ignore the yelling and continue to care for the patient as you would anyone else with a similar injury C) Inform the patrons that you are a medical professional and will care for the patient as you have been educated to do so D) Contact medical command for permission to give aspirin as a pain medication Answer: A Diff: 2 Page Ref: 324 Objective: 12-6 23) When deciding whether to transport a patient to the hospital using lights and sirens to the hospital, which information carries the most weight in your decision? A) Mechanism of injury B) Index of suspicion C) Past medical history D) Assessment findings Answer: D Diff: 1 Page Ref: 327 Objective: 12-7 24) To best determine the type and extent of injuries sustained by a patient who has fallen, the EMT should consider which of these pieces of information? A) The reason that the patient fell B) The distance fallen and the type of surface landed on C) The patient's past medical history D) Vital signs obtained during the assessment Answer: B Diff: 2 Page Ref: 327 Objective: 12-7 6 Copyright © 2018 Pearson Education, Inc.

25) You have been directed to a car that struck another vehicle at a moderate rate of speed. You note deformity and starring of the windshield on the driver's side. What should you do to determine whether the deformity to the windshield was caused by the driver hitting his head or by the air bags? A) Determine if a seat belt was used B) Ask the front-seat passenger C) Determine if the air bags deployed D) Assess the patient's head Answer: D Diff: 1 Page Ref: 327-328 Objective: 12-7 26) You arrive on the scene at an extended care facility where you see the facility employees removing the residents via wheelchairs and hospital beds. As you exit the ambulance, an employee walks up to you and identifies herself as the night-shift charge nurse. She states that the facility had a furnace malfunction and the carbon monoxide alarms are sounding. There are probably 30 residents already in the parking lot a safe distance away, and there are reportedly 15 more inside who are unresponsive. Given the number of patient overall, the EMT should: A) Alert dispatch of the need for additional resources B) Put on a TB mask and gain entry into the facility to remove the unresponsive patients C) Send his partner into the facility with another employee to start rendering care where the patients are lying D) Continue to supervise the actions of the facility employees until all the residents are removed from the facility Answer: A Diff: 2 Page Ref: 329 Objective: 12-8

7 Copyright © 2018 Pearson Education, Inc.

Prehospital Emergency Care, 11e (Mistovich et al.) Chapter 13 Patient Assessment 1) When assessing a sick or injured patient, which of these indicates the appropriate order in which this assessment should occur? A) Primary assessment; reassessment, secondary assessment B) Scene size-up, secondary assessment, primary assessment C) Scene size-up, primary assessment, secondary assessment D) Scene size-up, history, secondary assessment, primary assessment Answer: C Diff: 1 Page Ref: 335 Objective: 13-1 2) You have been dispatched to a residence for an 89-year-old female patient with a nonspecific complaint. When assessing this patient, what should you do first to develop an initial impression regarding emergency? A) Obtain the patient's vital signs and current medications B) Gather a medical history including allergy information C) Contact medical direction for advice D) Determine whether the complaint is medical or trauma related Answer: D Diff: 1 Page Ref: 335 Objective: 13-2 3) Which of these statements made by the EMT indicates that he has appropriately initiated the first phase of patient assessment? A) "I have placed an oral airway in the patient." B) "The scene appears to be free of hazards." C) "Blood pressure is 124/80 mmHg." D) "Can you tell me why you called the ambulance?" Answer: B Diff: 2 Page Ref: 335 Objective: 13-3 4) You arrive at the scene of a fall, where a 42-year-old woman fell backward off a stepladder while cleaning her kitchen windows. She is seated on the floor and complains of pain to her ankle. She tells you, "If I had just gone to work today instead of using vacation time, this would have never happened!" Based on information thus far, what can you conclude? A) The patient is alert with an open airway B) Her pulse rate is within normal limits C) She does not require rapid transport D) She does not have any other injuries Answer: A Diff: 2 Page Ref: 336 Objective: 13-9

1 Copyright © 2018 Pearson Education, Inc.

5) You have been dispatched to an apartment for an elderly male complaining of shortness of breath. When do you start the process of forming a general impression about this patient? A) As you and your partner approach him B) After completing the primary assessment C) As soon as you obtain his chief complaint D) Following the completion of vital signs Answer: A Diff: 1 Page Ref: 336 Objective: 13-4 6) The EMT shows she understands the importance and reason for forming a general impression when she states: A) "The general impression permits a rapid diagnosis of the patient's problem." B) "Forming a general impression allows me to fully assess the emergency situation." C) "The general impression enables me to rule out any life-threatening conditions." D) "Forming a general impression provides me with a rough idea of the patient's overall status." Answer: D Diff: 2 Page Ref: 336 Objective: 13-3 7) You have been called for an adult patient who suffered a seizure. He is sitting on the floor, and looks at you as you enter the room. When questioned, he knows his name but cannot remember the date, what happened, or where he is. This patient's mental status can best be described as: A) Alert but confused B) Lethargic and agitated C) Obtunded and altered D) Agitated and verbally disoriented Answer: A Diff: 2 Page Ref: 342 Objective: 13-8 8) An Emergency Medical Responder informs you that a 59-year-old female patient is responsive to verbal stimuli. What would be the most accurate representation of this status? A) She is able to answer all questions appropriately when asked B) She cannot be awakened when her name is called C) Her eyes are closed, but she will open them if asked D) She is awake, but having difficulty telling you her name and where she is Answer: C Diff: 2 Page Ref: 342 Objective: 13-8

2 Copyright © 2018 Pearson Education, Inc.

9) A 36-year-old patient who has overdosed on an unknown drug presents in a seemingly unresponsive state as you walk into the room. Given this, what should you do first? A) Check the patient's vital signs B) Assess the patient's mental status C) Determine whether a radial pulse is present D) Ask bystanders which drug was taken Answer: B Diff: 2 Page Ref: 342 Objective: 13-8 10) Which of these patients would be classified as unresponsive? A) A 39-year-old-female who is confused B) A 52-year-old male who cannot answer your questions C) A 70-year-old female who has experienced a stroke D) A 46-year-old-male who does not respond when his fingers are pinched Answer: D Diff: 2 Page Ref: 344 Objective: 13-8 11) You are assessing a 4-year-old patient whose panicked mother states that she cannot wake him. The child is breathing but his eyes are closed. To best determine the child's mental status, you should first: A) Apply pressure to the child's nail beds B) Say loudly, "Open your eyes!" C) Ask the mother how long the child has been asleep D) Identify the child as unresponsive Answer: B Diff: 2 Page Ref: 342 Objective: 13-8 12) An EMT is assessing a 9-month-old infant who is "not acting right" per his mother. Assessment reveals the child to be extremely irritable, crying weakly, and resistant to being touched by the EMT. Which description best describes this infant's mental status? A) Alert B) Lethargic C) Verbal D) Obtunded Answer: A Diff: 2 Page Ref: 342 Objective: 13-8

3 Copyright © 2018 Pearson Education, Inc.

13) You are by the side of a patient who complains of shortness of breath and has a history of heart problems, including congestive heart failure (CHF). When assessing the airway, which of these observations best indicates that it is still patent? A) The patient easily tells you that she is having chest pain B) The patient is sitting in an upright position because she cannot lie flat C) The patient can speak only in extremely short sentences D) The patient is lethargic and confused to person, place, and time Answer: A Diff: 1 Page Ref: 345 Objective: 3-9 14) As you arrive by the side of a 64-year-old unresponsive female patient, you hear snoring respirations. Your immediate action would be to: A) Suction the mouth and airway B) Determine the chief complaint C) Administer supplemental oxygen D) Perform a manual jaw-thrust maneuver Answer: D Diff: 2 Page Ref: 345 Objective: 13-9 15) On scene, you find a 91-year-old female patient who fell down two steps after suddenly complaining of a severe headache. She is unresponsive with gurgling respirations. Which instruction should you immediately give your partner after manual spinal motion restriction is applied? A) "Please perform the head-tilt, chin-lift maneuver to open the airway." B) "Check her breathing and let me know the rate." C) "Let's move her to the stretcher and get moving to the hospital." D) "Get the suction out and clear out her airway." Answer: D Diff: 1 Page Ref: 345 Objective: 13-9 16) A 26-year-old male patient has been ejected from a car after it rolled several times in the median of the interstate. As you perform the primary assessment, you find him prone and unresponsive. Your immediate action is to: A) Assess his breathing B) Apply a cervical collar C) Logroll him into supine position D) Place an oral airway Answer: C Diff: 3 Page Ref: 342 Objective: 13-6

4 Copyright © 2018 Pearson Education, Inc.

17) A conscious and alert 64-year-old female patient complains of chest pain and mild shortness of breath. She appears to have no difficulty speaking and states that the pain suddenly started about 2 hours ago. Her daughter on scene advises you there is a history of blood clots in the lungs of her family, and she is scared and would like her mother to be evaluated. Which of these steps should you take first? A) Consider administering oxygen to the patient B) Determine the adequacy of the patient's breathing C) Place the patient on the stretcher for transport D) Perform a secondary assessment on the patient Answer: B Diff: 2 Page Ref: 340 Objective: 13-5 18) You are an assistant instructor for an Emergency Medical Responder course. One of the students asks you how the head-tilt, chin-lift airway maneuver works to open the airway. Your response is: A) "It opens the airway by lifting the tongue from the back of the throat." B) "It creates an open airway by separating the lips, allowing air to enter." C) "It stimulates the patient to begin taking deeper breaths, thereby moving air into the lungs." D) "It opens the airway by relaxing and expanding the size of the throat." Answer: A Diff: 3 Page Ref: 345 Objective: 13-9 19) To stabilize the cervical spine of a trauma patient while simultaneously opening the airway, how should you position her head? A) Tilted slightly forward B) Tilted slightly back C) Neutral and in-line D) In the position found Answer: C Diff: 1 Page Ref: 341 Objective: 13-6 20) During the primary assessment, how should you best determine the adequacy of a patient's breathing? A) Determine the patient's mental status B) Compare the respirations and pulse rate C) Calculate the number of breaths per minute D) Examine the rise and fall of the chest Answer: D Diff: 2 Page Ref: 349 Objective: 13-10

5 Copyright © 2018 Pearson Education, Inc.

21) Which of these patients would the EMT identify as having the most adequate breathing, free from any respiratory distress? A) A 61-year-old confused female breathing at a rate of 8 breaths/min B) A 34-year-old male with abdominal pain who is alert and oriented with good chest rise and fall C) A 19-year-old male breathing 16 times per minute with slight wheezing in the lungs D) A 67-year-old female breathing 26 times per minute with good chest rise and fall Answer: B Diff: 2 Page Ref: 348-349 Objective: 13-9 22) The EMT would recognize which of these patients as having a normal breathing rate? A) An 8-year-old male breathing 24 times per minute B) A 44-year-old male breathing 6 times per minute C) A 6-month-old infant breathing 58 times per minute D) A 22-year-old female breathing 26 times per minute Answer: A Diff: 1 Page Ref: 347 Objective: 13-9 23) Which finding individually would most likely indicate that a patient is breathing inadequately? A) An SpO2 of less than 94% B) A respiratory rate of 20 breaths/min in an adult C) Numbness and tingling of the fingers and toes D) Shortness of breath during exercise Answer: A Diff: 2 Page Ref: 348 Objective: 13-11 24) You have been called to the side of a 37-year-old female patient whose chief complaint is confusion, anxiety, and chest tightness. Assessment shows her airway to be patent, respirations are 46 breaths/min, heart rate is 134 beat/min, blood pressure is 128/54 mmHg, and SpO2 is 93%. In regard to the respiratory rate, you would recognize: A) It will result in too much air being placed in the lungs B) It is not a concern because the patient is still getting oxygen C) It does not allow the lungs enough time to adequately fill D) It will cause damage to the lungs if not corrected Answer: C Diff: 3 Page Ref: 347 Objective: 13-10

6 Copyright © 2018 Pearson Education, Inc.

25) To best evaluate the adequacy of breathing in an unresponsive adult, the EMT should: A) Assess the skin color B) Check the pupils C) Observe the chest rise D) Look for cyanosis Answer: C Diff: 1 Page Ref: 349 Objective: 13-10 26) The patient's family states that their 16-year-old daughter has a history of asthma and has been complaining of shortness of breath for the past two days. She has been taking her metereddose inhaler with some relief, but this morning, they found her lethargic and struggling to breathe. Your assessment reveals the patient to be responsive to verbal stimuli with an open airway and shallow respirations of 44 breaths/min. You hear minimal wheezing in both lungs. What is your immediate action in caring for this patient? A) Insert an oral airway B) Apply a nonrebreather mask C) Position her on her side D) Start positive pressure ventilation Answer: D Diff: 3 Page Ref: 348-349 Objective: 13-10 27) You are by the side of a 64-year-old male patient who collapsed at home. After assessing his level of consciousness, airway, and breathing, you determine that he is unresponsive with an open airway, but not breathing. The patient still has a weak carotid pulse. Which instruction should you immediately give to your partner? A) "Let's attempt placement of an OPA or NPA and start ventilations." B) "Please check the patient's blood pressure and apply the pulse oximeter." C) "Go ahead and apply the automated external defibrillator." D) "We will need to start cardiopulmonary resuscitation." Answer: A Diff: 2 Page Ref: 349 Objective: 13-9

7 Copyright © 2018 Pearson Education, Inc.

28) You arrive on the scene of a motor vehicle collision and are presented with a patient who complains of chest pain after striking his chest on the steering wheel. On-scene Emergency Medical Responders are maintaining cervical spine motion restriction manually. The patient is alert and oriented and has a patent airway. His respirations are 24 breaths/min with good chest rise and fall, and his skin is warm and dry. He describes some shortness of breath, his heart rate is 96 beats/min, and his SpO2 is 92% on room air. At this time you should: A) Apply a cervical collar B) Provide oxygen via nasal cannula at 2-4 liters per minute C) Obtain a blood pressure and medical history D) Start positive pressure ventilation Answer: B Diff: 2 Page Ref: 349 Objective: 13-11 29) Which characteristic regarding breathing rates must the EMT remember when assessing the respirations of a pediatric patient? A) Respirations in a pediatric patient are similar to those of an adult B) Respirations in a pediatric patient are typically slower than those of an adult C) Respirations in a pediatric patient are not significant in the assessment of breathing adequacy D) Respirations in a pediatric patient are usually faster than those of an adult Answer: D Diff: 1 Page Ref: 347 Objective: 13-10 30) A 28-year-old female patient has fallen 20 feet from the roof a house. She is lying supine and does not appear to be conscious or breathing. On arrival at her side, what should the EMT do first? A) Open her airway using a head-tilt, chin-lift maneuver B) Apply manual in-line spinal motion restriction C) Start positive pressure ventilation after placing an OPA or NPA D) Open her airway and assess her circulation Answer: B Diff: 2 Page Ref: 338-339 Objective: 13-1 31) You have just determined that a 92-year-old female patient found in bed is unresponsive, but still has a carotid pulse. Your next action would be to: A) Open the airway manually B) Check for a radial pulse C) Determine the past medical history D) Immediate transport Answer: A Diff: 1 Page Ref: 336 Objective: 13-7

8 Copyright © 2018 Pearson Education, Inc.

32) Near the end of a workday, a construction worker fell 10 feet from scaffolding, landing on his head and left shoulder. He is found to be spontaneously alert, but confused. His airway is open and his breathing is adequate. Coworkers state that he was unresponsive for a period of 2 minutes following the fall and is on medications for depression and heart problems. Based on this information, the EMT should initially classify him as which kind of patient? A) Trauma B) Trauma and behavioral C) Medical and trauma D) Medical, trauma, and behavioral Answer: A Diff: 1 Page Ref: 336 Objective: 13-3 33) Forming a general impression is essential in the assessment of any patient because it: A) Indicates whether a primary assessment is required B) Helps to determine the clinical status of a patient C) Determines the stability or instability of a patient D) Reveals the injuries the patient has suffered Answer: B Diff: 2 Page Ref: 336 Objective: 13-4 34) Which of these patients should receive oxygen via an oxygenation adjunct such as an NRB or nasal cannula? A) A 39-year-old male with a sprained ankle who is breathing 16 times per minute B) An 18-year-old female who overdosed on a drug and is unresponsive and breathing 4 times per minute C) A 42-year-old female who is confused, weak, and dizzy, and adequately breathing 14 times per minute D) A 61-year-old male who is unresponsive, with a foreign body airway obstruction that cannot be removed with abdominal thrusts Answer: C Diff: 2 Page Ref: 349 Objective: 13-10 35) When assessing an adult trauma patient's circulation status during the primary assessment, the EMT's assessment should include: A) Carotid pulse, capillary refill, blood pressure, and skin color B) Radial and/or carotid pulse, skin characteristics, and presence of external bleeding C) Skin color, pulse rate, blood pressure, and skin temperature D) Pulse rate, blood pressure, capillary refill, and skin condition Answer: B Diff: 1 Page Ref: 350; Table 13-6 Objective: 13-12

9 Copyright © 2018 Pearson Education, Inc.

36) You cannot locate a radial pulse on a 54-year-old male patient who is alert, confused, and breathing adequately. What should you do immediately? A) Start cardiopulmonary resuscitation B) Attach the automated external defibrillator (AED) C) Start positive pressure ventilation D) Check for the carotid pulse Answer: D Diff: 2 Page Ref: 350 Objective: 13-12 37) What is the purpose of assessing the pulse on an unresponsive patient during the primary assessment? A) To calculate the precise heart rate B) To decide whether the breathing is adequate C) To determine the presence of a heartbeat D) To establish whether the patient requires oxygen Answer: C Diff: 1 Page Ref: 350 Objective: 13-2 38) When performing the primary assessment of a conscious adult, you typically palpate which pulse first? A) Carotid B) Femoral C) Radial D) Brachial Answer: C Diff: 1 Page Ref: 350 Objective: 13-12 39) You are assessing an 8-month-old child whose mother states that he has been vomiting for two days and not eating or drinking normally. When assessing the pulse, you should check which site? A) Temporal B) Radial C) Femoral D) Brachial Answer: D Diff: 1 Page Ref: 350 Objective: 13-12

10 Copyright © 2018 Pearson Education, Inc.

40) You have been called for a 6-month-old male patient with a high fever who has seized. Currently, the patient is not seizing and appears to be sleeping. When performing the primary assessment on this patient, which of these actions would be most appropriate? A) Evaluate the pulse before examining the breathing B) Check for the presence of a pulse in the brachial artery C) Move the patient to the ambulance before performing the primary assessment D) Obtain a blood pressure when checking the patient's circulation status Answer: B Diff: 2 Page Ref: 350 Objective: 13-12 41) You must check the circulation of an 8-year-old child with a high fever who has been vomiting for the past 2 hours. The child is very frightened and crying. Given the situation, you should: A) Palpate the radial artery B) Obtain a carotid pulse C) Feel for a brachial pulse D) Listen to the heart with a stethoscope Answer: A Diff: 2 Page Ref: 350 Objective: 13-12 42) As you approach a patient who was involved in a motor vehicle crash, you note that he has dark, oozing blood coming from an open fracture to his lower left tibia. The patient was removed from the car by bystanders and is being held by a family member. What should you do immediately? A) Assess and open the airway B) Take manual in-line spine motion restriction C) Apply direct pressure to the bleeding D) Provide supplemental oxygen Answer: B Diff: 2 Page Ref: 338-339 Objective: 13-6 43) An intoxicated 19-year-old female patient has fallen through a large plate glass window. As you approach, you note her to be sitting up and holding her hand over a large laceration on her opposite arm that is still spurting bright red blood. In this situation, you would immediately: A) Provide high-concentration oxygen and treat for shock B) Determine the presence and rate of the carotid pulse C) Apply direct pressure to the laceration D) Assess the airway for patency and breathing for adequacy Answer: C Diff: 2 Page Ref: 351 Objective: 13-13

11 Copyright © 2018 Pearson Education, Inc.

44) A patient has called 911 for abdominal pain and generalized weakness. On arrival, you note that the patient's skin is a yellow-orange color. Which disease revealed to you during the history would best correlate to this patient's skin? A) Lung cancer B) Stroke C) Liver disease D) Diabetes Answer: C Diff: 2 Page Ref: 352 Objective: 13-5 45) At a nursing home, you find an elderly patient extremely short of breath. To best determine whether this patient is cyanotic, you should: A) Press and release the skin on the palm B) Examine the lining of the lips and mouth C) Look at and feel the skin on the back of the hand D) Inspect the skin on the fingers and nail beds Answer: B Diff: 2 Page Ref: 360 Objective: 13-12 46) During your primary assessment, you note the patient's skin to be warm, pink, and dry. This finding would seem to indicate: A) Possible shock B) Possible fever C) Poor oxygenation D) Normal circulation Answer: D Diff: 1 Page Ref: 351-352 Objective: 13-13 47) When performing the primary assessment of a 45-year-old patient, which of these skin findings should concern the EMT? A) Cool and moist skin B) 1-second capillary refill C) Pink color to the nail beds D) Skin that is warm to the touch Answer: A Diff: 2 Page Ref: 353 Objective: 13-12

12 Copyright © 2018 Pearson Education, Inc.

48) A 62-year-old female patient who has been working in her garden has hot, dry skin. You should recognize this finding as a possible sign of: A) Heart problems B) Heat exposure C) Hypoperfusion D) Stroke Answer: B Diff: 2 Page Ref: 352-353 Objective: 13-12 49) Which of these statements made by an EMT student shows he correctly understands capillary refill? A) "Capillary refill is a reliable sign of perfusion in adults, but not as reliable in infants and children." B) "Red flushed skin in the adult is a normal finding when assessing capillary refill." C) "A capillary refill of 4 or more seconds in a patient of any age indicates that he or she is well oxygenated." D) "For a capillary refill test to be normal for a child, it must be assessed at room temperature and be less than 2 seconds." Answer: D Diff: 3 Page Ref: 353 Objective: 13-12 50) A 5-year-old boy has been vomiting profusely for two days. Your assessment indicates that his airway is open, respirations adequate, and radial pulse fast and weak. His skin is cool and moist, and the capillary refill time is 5 seconds. This patient is most likely: A) Hypoxic B) Malnourished C) Poorly perfused D) Feverous Answer: C Diff: 3 Page Ref: 353 Objective: 13-13 51) Which capillary refill time is considered the upper limit of normal for an adult female? A) 3 seconds B) 2 seconds C) 6 seconds D) 4 seconds Answer: A Diff: 1 Page Ref: 353 Objective: 13-12

13 Copyright © 2018 Pearson Education, Inc.

52) Which capillary refill time is considered normal for a 5-year-old child? A) 3 seconds B) 2 seconds C) 5 seconds D) 4 seconds Answer: B Diff: 1 Page Ref: 353 Objective: 13-12 53) Your patient is 82 years old and has diabetes. Which assessment parameter is the least reliable sign indicating this patient's peripheral circulatory status? A) Capillary refill B) Skin color C) Skin temperature D) Pulse strength Answer: A Diff: 2 Page Ref: 353 Objective: 13-12 54) Which of these patients is considered the lowest priority for a rapid secondary exam and immediate transport? A) A 32-year-old woman who is unresponsive B) A 72-year-old man complaining of leg pain for a week C) A 7-year-old boy who has had severe abdominal pain for 4 hours D) A 55-year-old woman with difficulty breathing Answer: B Diff: 2 Page Ref: 353 Objective: 13-18 55) As you perform the primary assessment on an unresponsive patient, you discover vomitus in her airway. What should you do next? A) Check the breathing B) Start positive pressure ventilation C) Apply supplemental oxygen D) Suction the airway Answer: D Diff: 1 Page Ref: 345 Objective: 13-7

14 Copyright © 2018 Pearson Education, Inc.

56) You are interviewing a prospective candidate for an EMT position with your service. During the interview, you ask the applicant to describe the purpose of the primary assessment. Which of these statements best describes that purpose? A) "Establish a chief complaint and gather the past medical history." B) "Determine whether the patient's vital signs are stable." C) "Identify and treat life-threatening conditions." D) "Perform an exam focused on the patient's chief complaint." Answer: C Diff: 2 Page Ref: 336 Objective: 13-4 57) A 43-year-old male patient has called 911 with the complaint of difficulty breathing. Once on scene, you note he is breathing at a rate of 18 breaths/min. He also exhibits intercostal retractions and nasal flaring. The pulse oximeter reads 93% on room air. The EMT should document this patient's breathing as: A) Dyspneic B) Apneic C) Tachypneic D) Bradypneic Answer: A Diff: 1 Page Ref: 348 Objective: 13-11 58) Forming the general impression starts during which phase of patient assessment? A) Reassessment B) Secondary assessment C) Primary assessment D) Care transfer in the emergency department Answer: C Diff: 1 Page Ref: 336 Objective: 13-4 59) You have completed the primary assessment on a patient who was thrown from a motorcycle and landed on his head and back on the nearby sidewalk. After initiating spine motion restriction and finding no life threats during the primary assessment, which action should you take first when starting a secondary assessment? A) Determine the patient's SAMPLE history B) Obtain baseline vital signs C) Conduct a physical exam D) Load the patient into the ambulance Answer: C Diff: 2 Page Ref: 355 Objective: 13-14

15 Copyright © 2018 Pearson Education, Inc.

60) A patient has been shot once in the left hand. After performing the primary assessment, the EMT quickly focuses his attention on the patient's left hand and begins a specific assessment of the injury. Which type of exam is the EMT performing? A) Extremity B) Focused C) Specific D) Rapid Answer: B Diff: 1 Page Ref: 356 Objective: 13-18 61) A 44-year-old male patient was cutting limbs from high in a tree when he fell. He is found to be unresponsive with snoring respirations. His breathing is inadequate at a rate of 8 breaths/min and his carotid pulse is weak and difficult to locate. Following the primary assessment and initial management of life threats, which of these actions will the EMT complete while performing the secondary assessment? A) Obtain vital signs and get a past medical history from the family B) Start positive pressure ventilation with a bag-valve mask C) Place an oral airway and cervical collar on the patient D) Perform a trauma jaw-thrust maneuver to open the airway Answer: A Diff: 3 Page Ref: 382 Objective: 13-16 62) A 62-year-old female patient was struck by a car traveling approximately 45 miles per hour. After being struck, she was thrown 15 feet onto the sidewalk, impacting the pavement with her head and chest. The primary assessment has been completed, spine motion restriction precautions have been provided, and life threats addressed. The patient's husband is by her side. When performing the secondary exam on this patient, what should the EMT do first? A) Examine the head and chest for other life-threatening injuries B) Obtain a full set of vital signs including a pulse oximeter reading C) Ask the family member if the patient has any past medical history D) Perform a rapid head-to-toe physical assessment Answer: D Diff: 3 Page Ref: 368 Objective: 13-14

16 Copyright © 2018 Pearson Education, Inc.

63) When teaching a group of Emergency Medical Responders (EMRs) about the mechanism of injury, which point should you stress? A) The more significant the mechanism of injury, the greater the chance the patient has been injured B) The mechanism of injury is no longer considered in the assessment because research has shown this information causes many patients to be over-treated C) The mechanism of injury is the sole criterion on which an EMT decides whether immediate transport is required D) The mechanism of injury can be used to dictate the order in which the primary assessment is conducted Answer: A Diff: 2 Page Ref: 368-369 Objective: 13-19 64) While treating a patient who was assaulted by a mob of angry people in a bar, the EMT finds the patient to be confused, tachycardic, with cool skin and a poor capillary refill. Based on this information, how would you categorize this patient? A) Stable B) Potentially unstable C) Unstable D) Potential high priority Answer: C Diff: 2 Page Ref: 371 Objective: 13-18 65) For which of these patients should the EMT perform a rapid head-to-toe secondary assessment, as opposed to a focused secondary assessment? A) A 22-year-old male who is alert and oriented with a deformed ankle that occurred when he tripped playing basketball and landed hard B) A 36-year-old construction worker who was shot in the hand with a nail gun; the nail remains impaled in his hand and bleeding is minor C) A 24-year-old male with stable vitals who is alert and oriented and is complaining of chest pain secondary to a motor vehicle collision in which the passenger was killed D) A 46-year-old roofer with a history of coronary heart disease, who complains of a burn to his left lower leg after being splashed with hot tar Answer: C Diff: 2 Page Ref: 370 Objective: 13-14

17 Copyright © 2018 Pearson Education, Inc.

66) When assessing a critically injured patient, the EMT should detect which of these injuries or conditions when performing the secondary assessment? A) Decreased level of consciousness B) Weak carotid pulse and clammy skin C) Inadequate respiratory effort D) Bruising and tenderness to the abdomen Answer: D Diff: 2 Page Ref: 373 Objective: 13-22 67) When performing the secondary assessment, the EMT would likely find which of these injuries first? A) Deformity to the left thigh B) Bruising to the abdomen C) Unstable pelvis D) Decreased pulse in the left foot Answer: B Diff: 2 Page Ref: 373 Objective: 13-22 68) When you assess the lower back of an elderly patient who fell, he grimaces and tells you that it hurts when you palpate the injury site. The perception by the patient of increased pain upon palpation of an injured site is known as: A) Pain B) Tenderness C) Instability or crepitation D) Ecchymosis Answer: B Diff: 2 Page Ref: 373 Objective: 13-22 69) You are performing a secondary assessment on an unrestrained adult passenger who was involved in a motor vehicle collision in which the car rolled end over end several times at a high rate of speed. The patient is responsive to painful stimuli and in a state of shock. When assessing the patient's head, which of these would be appropriate for you to do? A) Clean a laceration of the scalp B) Check the pupils with a penlight C) Perform a blind finger sweep to check the airway D) Apply pressure to a depressed area of the skull Answer: B Diff: 2 Page Ref: 374 Objective: 13-22

18 Copyright © 2018 Pearson Education, Inc.

70) When assessing the chest of a patient who was shot multiple times, you find a gunshot wound to the third intercostal space on the left lateral chest. What should you do immediately? A) Check for equality of breath sounds B) Initiate positive pressure ventilation with oxygen C) Roll the patient and look for an exit wound D) Cover the opening with a gloved hand Answer: D Diff: 2 Page Ref: 341 Objective: 13-7 71) You are performing a secondary assessment and are assessing the patient's chest. Which of these findings would you associate most with fracture of the ribs? A) Ecchymosis to the chest wall B) Decreased breath sounds C) Paradoxical chest wall motion D) Jugular venous distention Answer: C Diff: 3 Page Ref: 361 Objective: 13-22 72) When assessing breath sounds during the secondary assessment, the EMT shows he is employing a correct technique when he listens: A) At the left second intercostal space, followed by the right second intercostal space B) First to the apex and base of the right lung, and then to the apex and base of the left lung C) To the base of the lung by placing the stethoscope at the third intercostal space laterally D) Over the left and right lateral wall while instructing the patient to take fast and shallow breaths Answer: A Diff: 3 Page Ref: 361 Objective: 13-14 73) Assessment of a patient's neck reveals that the trachea is deviated to the left. Based on this assessment finding, the EMT should suspect an injury located to the patient's: A) Chest B) Abdomen C) Cervical spine D) Head Answer: A Diff: 2 Page Ref: 360 Objective: 13-14

19 Copyright © 2018 Pearson Education, Inc.

74) A 41-year-old male patient has fallen from a second-story roof. He complains of right leg pain and pain to his pelvic region. When performing the secondary assessment on this patient, which of these should the EMT intentionally not perform? A) Palpation of the patient's abdomen B) Securing the patient to a long spine board C) Palpation of the pelvic region D) Transporting the patient supine Answer: C Diff: 3 Page Ref: 363 Objective: 13-14 75) Which of these assessment findings indicates that the patient has intact motor function in his lower extremities? A) The skin of his toes is pink and warm B) He does not complain of pain to his lower extremities C) He can wiggle his toes when instructed D) He states that he can feel you touching his toe Answer: C Diff: 1 Page Ref: 364 Objective: 13-14 76) When assessing the breath sounds (during a rapid secondary exam) of a critical trauma patient, it is best to auscultate each lung in at least how many places? A) 2 B) 1 C) 4 D) 3 Answer: A Diff: 1 Page Ref: 361 Objective: 13-14 77) Which of these physical findings is least consistent with an injured extremity? A) Deformities B) Vomiting C) Contusions D) Crepitation Answer: B Diff: 1 Page Ref: 364 Objective: 13-14

20 Copyright © 2018 Pearson Education, Inc.

78) What does the "S" in SAMPLE stand for? A) Significance B) Severity C) Sign D) Seriousness Answer: C Diff: 1 Page Ref: 367 Objective: 13-17 79) A 66-year-old female patient was involved in a vehicle rollover. While evaluating her abdomen, which of these abdominal assessment parameters should you be least concerned about? A) Rigidity B) Distention C) Guarding D) Bowel sounds Answer: D Diff: 1 Page Ref: 362 Objective: 13-14 80) A 40-year-old male patient has crashed his motorcycle. He complains of pain to his right leg. When palpating his leg, you are assessing for: A) Sensation B) Crepitation C) Bruising D) Rigidity Answer: B Diff: 1 Page Ref: 364 Objective: 13-14 81) While you are performing a secondary assessment on a male patient who was ejected from his motorcycle, the patient suddenly states that it has become very hard to breathe. You would: A) Make a mental note and continue with your exam B) Stop your exam and evaluate the patient's new complaint C) Reassess the patient, beginning with the head D) Reconsider the mechanism of injury and look for a chest injury Answer: B Diff: 2 Page Ref: 401 Objective: 13-13

21 Copyright © 2018 Pearson Education, Inc.

82) An 18-year-old female patient with a history of diabetes twisted her right ankle while playing volleyball. She is alert and oriented and states that the pain is excruciating. The primary assessment is complete and yields no life-threatening conditions. What should you do next? A) Perform a head-to-toe exam B) Apply ice to the ankle C) Check her blood sugar D) Assess the right ankle and foot Answer: D Diff: 2 Page Ref: 356 Objective: 13-24 83) The EMT recognizes that for a trauma patient who is alert and oriented, he should obtain the patient's medical history during which component of patient assessment? A) Primary assessment B) Scene size-up C) Secondary assessment D) Reassessment Answer: C Diff: 1 Page Ref: 368 Objective: 13-18 84) At a continuing education seminar being taught by your medical director, the instructor asks the group for one reason why the secondary assessment is beneficial to perform on the trauma patient. Which of these responses is most appropriate? A) "It is useful in finding additional injuries and guiding additional care." B) "It is a good tool that allows the EMT to determine whether critical interventions such as positive pressure ventilation or CPR are needed." C) "It is an effective means to determine the exact mechanism of injury." D) "It is the best assessment for detecting problems with the airway, breathing, or circulation." Answer: A Diff: 2 Page Ref: 355 Objective: 13-1 85) When performing a secondary assessment on an alert and oriented female patient with a deformed left leg, the EMT should: A) Describe to the patient your findings and the injuries as you encounter them B) Reassure the patient that she will be okay and the injury is nothing to worry about C) Inform the patient that if you assess her, she must submit to all treatment by EMS providers D) Explain to the patient what you are doing before and during the assessment Answer: D Diff: 1 Page Ref: 364 Objective: 13-18

22 Copyright © 2018 Pearson Education, Inc.

86) When assessing a stable patient with a medical complaint, which piece of information will typically be obtained first? A) Vital signs B) Chief complaint C) Level of consciousness D) Past medical history Answer: D Diff: 1 Page Ref: 398 Objective: 3-17 87) A 31-year-old male is complaining of left-side flank pain that radiates into his groin. He is alert and oriented, and the primary assessment reveals no life-threatening conditions. Given the patient's status, your next step should be to: A) Gather medical information from the family B) Perform a rapid head-to-toe exam C) Obtain a medical history from the patient D) Repeat the primary assessment Answer: C Diff: 2 Page Ref: 398 Objective: 13-25 88) A 69-year-old male patient called 911 because of a sudden onset of pain and then numbness in his left leg. He states that he has a history of blood clots that form in the leg and the pain feels exactly as it did the last time that this occurred. He denies any trauma to the extremity. You perform the primary assessment and find the patient to be alert and oriented with no lifethreatening deficits to the airway, breathing, or circulation. When initiating the secondary assessment on this patient, what would be appropriate to do? A) Perform a rapid head-to-toe exam B) Initiate immediate transport C) Place a nasal airway as a precautionary measure D) Perform a physical exam of the leg Answer: D Diff: 2 Page Ref: 399 Objective: 13-25 89) An unresponsive 54-year-old male patient was found by family in his bed. They state that he has not been feeling well and complained of chest pain before going to bed approximately 1 hour ago. According to family, he has no prior medical history and does not take any medications. The primary assessment reveals no life-threatening conditions. When performing the secondary assessment on this patient, the EMT should: A) Start by examining the patient's head B) Contact the patient's family doctor C) Determine whether the chest pain still exists D) Focus the examination on the chest Answer: A Diff: 2 Page Ref: 392 Objective: 13-26 23 Copyright © 2018 Pearson Education, Inc.

90) The EMT recognizes that the best source of information for a responsive medical patient with a specific chief complaint and no known medical history is: A) The patient's vital signs B) The patient C) Medical direction D) The family Answer: B Diff: 1 Page Ref: 398 Objective: 13-26 91) The EMT is using the OPQRST mnemonic to gather information from a patient with abdominal pain. Which of these patient statements is a response to the "P" component of this memory aid? A) "It hurts worse when I take a deep breath." B) "My pulse feels as though it is racing." C) "I have a past history of asthma." D) "The pain started about 2 hours ago." Answer: A Diff: 3 Page Ref: 399 Objective: 13-17 92) During assessment of a responsive medical patient, you gather a medical history using the memory aid SAMPLE. To obtain information related to the "M" component, which question should you ask? A) "Do you have any past medical problems?" B) "Are you allergic to any medications?" C) "Are you currently taking any medications?" D) "Can you point to where it hurts the most?" Answer: C Diff: 2 Page Ref: 398 Objective: 13-17 93) The EMT determines the medications a patient takes during which phase of the patient assessment? A) Primary assessment B) OPQRST exam C) Chief complaint D) SAMPLE history Answer: D Diff: 1 Page Ref: 398 Objective: 13-17

24 Copyright © 2018 Pearson Education, Inc.

94) An alert and oriented 69-year-old female patient complains of low, right-sided abdominal pain. She describes the pain as "knife-like" and states that it started suddenly 1 hour ago. Her only medical history is hypertension, for which she takes the medication Lisinopril. Which of these assessment findings shows that the EMT is appropriately assessing this patient given the chief complaint? A) No trauma noted to the head B) No pain on palpation to the chest C) No deformity to the upper extremities D) No abdominal guarding noted Answer: D Diff: 2 Page Ref: 399 Objective: 13-26 95) A 20-year-old female patient is found unresponsive. The first step of the secondary assessment for this patient is to: A) Obtain vital signs B) Conduct a rapid medical assessment C) Gather a SAMPLE history from the family D) Contact medical direction for advisement Answer: B Diff: 2 Page Ref: 392 Objective: 13-25 96) You have been called for an 83-year-old female patient who is having difficulty breathing. She informs you that she has congestive heart failure and feels as though she is filling up with fluid. The primary assessment shows her to be alert and oriented with inadequate breathing. Her pulse is rapid and weak, and her skin is cool and dry. When performing a secondary assessment, which sign related to heart failure would you specifically look for when examining this patient's neck? A) Bruising to the neck B) Deviated trachea C) Jugular vein distention D) Posterior tenderness on palpation Answer: C Diff: 2 Page Ref: 393 Objective: 13-18

25 Copyright © 2018 Pearson Education, Inc.

97) You have been summoned to a retail store, where you are directed to a middle-aged woman who was found in the bathroom on the toilet. Your assessment shows her to be responsive to painful stimuli with an open airway and labored breathing. Her pulse is fast and regular, and her skin is warm to the touch. The primary assessment has been completed and the appropriate care given. As the stretcher is prepared, you perform the secondary assessment. When assessing her chest, you note the presence of crackles to both lungs. What should you do next? A) Place the patient on the stretcher B) Assess the abdomen C) Obtain vital signs D) Evaluate the head and neck Answer: B Diff: 2 Page Ref: 399 Objective: 13-26 98) When you are caring for a patient with either a medical complaint or a traumatic injury, you should typically perform the reassessment: A) Before the secondary assessment B) When a life-threatening condition is found C) Immediately following the primary assessment D) While transporting the patient to the hospital Answer: D Diff: 1 Page Ref: 400 Objective: 13-28 99) You are transporting a 31-year-old male patient who was severely injured in a motor vehicle collision. He is unresponsive and being ventilated by your partner with a bag-valve mask. How will you conduct the rapid secondary assessment on this patient? A) Perform a head-to-toe exam B) Focus on the chest and abdomen C) Palpate the body parts but do not auscultate them D) Focus on the head and torso Answer: A Diff: 2 Page Ref: 368 Objective: 13-22 100) You are transporting an 18-year-old male patient who was injured in a motor vehicle collision. He has a decreased level of consciousness and has injuries to his head, chest, and lower right leg. While you are conducting the reassessment of the patient's head and neck, which of these actions is most appropriate? A) Maintaining the patient's airway with the head-tilt, chin-lift maneuver B) Placing a sterile dressing in the patient's right ear canal, which is oozing blood C) Taking time to carefully re-palpate the patient's face and scalp D) Carefully removing the cervical collar to reassess the back of the patient's neck Answer: C Diff: 2 Page Ref: 370-371 Objective: 13-22 26 Copyright © 2018 Pearson Education, Inc.

101) A responsive patient complains of a migraine headache; she has a known medical history of this condition. Which assessment parameter would be most appropriate to address while conducting her secondary exam? A) Auscultate breath sounds B) Assess for pedal edema C) Look for jugular venous distention D) Assess her visual acuity Answer: D Diff: 2 Page Ref: 399 Objective: 13-26 102) A male patient was found in cardiac arrest by coworkers. On arrival at the scene, you determined him to be in cardiac arrest and started CPR. The AED was quickly placed and you shocked the patient once, restoring a heartbeat. During transport, the patient remains unresponsive, but is breathing and has a pulse. When conducting the reassessment of this patient, which action should you complete? A) Perform a trauma assessment B) Recheck vital signs C) Remove the oropharyngeal airway D) Obtain a blood pressure every 20 minutes Answer: B Diff: 2 Page Ref: 403 Objective: 13-28 103) En route to the hospital, you start the reassessment of a 22-year-old trauma patient. You should begin this assessment by: A) Rechecking vital signs B) Repeating the primary assessment C) Checking all interventions D) Assessing for additional injuries Answer: B Diff: 1 Page Ref: 402 Objective: 13-27 104) Trends in a patient's condition are best noted during which phase of patient assessment? A) Secondary assessment B) Primary assessment C) General impression D) Reassessment Answer: D Diff: 1 Page Ref: 403 Objective: 13-27

27 Copyright © 2018 Pearson Education, Inc.

105) Information concerning the extent of injury can be related to findings of the complete headto-toe exam by: A) The ED B) An accurate primary assessment C) Forming a general impression D) Utilizing the body system approach Answer: D Diff: 2 Page Ref: 366 Objective: 13-15 106) Which numeric evaluation tool is scored and added to the Trauma Scale to determine the Revised Trauma Score? A) Glasgow Coma Scale B) AVPU C) Pulse pressure D) Primary Assessment Score Answer: A Diff: 1 Page Ref: 388 Objective: 13-23 107) A patient has a GCS of 14, is oriented, and converses and opens his eyes when asked. When asked, he will squeeze your fingers and wiggle his toes. How would you describe this patient's motor response? A) Has a positive Babinski sign B) Localizes pain C) Obeys verbal commands D) Withdraws from pain Answer: C Diff: 2 Page Ref: 389, Table 13-12 Objective: 13-21 108) After manual spine motion restriction is established, it should never be released until: A) Self-restriction is initiated B) The patient is secured on a stretcher C) All interventions are checked D) The patient has been assessed for additional injuries Answer: B Diff: 2 Page Ref: 370 Objective: 13-20

28 Copyright © 2018 Pearson Education, Inc.

109) You are completing the secondary assessment of a conscious patient with chest discomfort and a medical history of coronary artery disease. While assessing the cardiovascular system specifically, which one of these assessments should be performed? A) Pulse pressure value B) Blood glucose level C) Analysis by the AED of the patient's heart rhythm D) Assessment of the expiration date of the patient's prescribed nitroglycerin Answer: A Diff: 2 Page Ref: 366 Objective: 13-15 110) While completing the secondary assessment of an unresponsive patient with a history of diabetes and hypotension, how often should you repeat the patient's vital signs? A) Every 5 minutes B) Every 10 minutes C) Every 15 minutes D) Every 20 minutes Answer: A Diff: 3 Page Ref: 384 Objective: 13-16 111) You are treating a 17-year-old patient who fell out of a tree from a height of approximately 6-8 feet and landed against a wooden picket fence that was near the tree trunk. The patient landed with his right lateral chest striking the fence. The patient is alert and oriented, complains of respiratory distress, and has absent breath sounds over his right thorax. The vital signs reveal a blood pressure of 112/72 mmHg, a heart rate of 102 beats/min and regular, respirations of 20 breaths/min, pulse oximetry of 90% on ambient air, and intact peripheral perfusion. Your partner asks you if he should call for ALS intercept. You respond "yes" for which reason? A) The patient's fall height was "critical" B) The patient has a narrow pulse pressure C) The patient's heart rate is more than 100 beats/min D) You suspect the patient may have a pneumothorax Answer: D Diff: 2 Page Ref: 370 Objective: 13-20 112) You are assessing a medical patient whom you suspect has experienced a stroke. Currently he can open his eyes to verbal commands, his verbal responses are incomprehensible sounds, and he is able to localize painful stimuli when applied. What is this patient's Glasgow Coma Scale score? A) 6 B) 8 C) 10 D) 12 Answer: C Diff: 3 Page Ref: 372, Table 13-9 Objective: 13-21 29 Copyright © 2018 Pearson Education, Inc.

113) You are assessing a trauma patient who was ejected from a car when it overturned. Currently he opens his eyes to painful stimuli, demonstrates no verbal response to questioning, and exhibits decerebrate rigidity in response to pain. What is this patient's Glasgow Coma Scale score? A) 3 B) 5 C) 7 D) 9 Answer: B Diff: 3 Page Ref: 372, Table 13-9 Objective: 13-21 114) You are a field supervisor for your EMS system and are working with a new EMT during his probationary period. Both of you are currently on the scene of a 10-month-old pediatric patient for what dispatch describes as "an unknown medical emergency." Upon arrival, the parents state the infant is "just not acting right." Currently the infant opens his eyes when you shout his name suddenly, he can localize painful stimuli when applied, and he is grunting with each respiration. He was born 3 weeks premature. His capillary refill is 2 seconds. You ask your new EMT to calculate the pediatric GCS score. The EMT should respond with which numeric value? A) 6 B) 8 C) 10 D) 12 Answer: C Diff: 3 Page Ref: 372, Table 13-10 Objective: 13-21 115) A trauma patient currently has a blood pressure of 82/60 mmHg, a heart rate of 112 beats/min, respirations of 24 breaths/min and unlabored, and a pulse oximeter reading of 94% on 4 liters of oxygen via nasal cannula. His computed GCS score is 12 points. When computing his revised trauma score, which value would you assign? A) 7 B) 9 C) 11 D) 13 Answer: C Diff: 1 Page Ref: 389, Table 13-2 Objective: 13-23

30 Copyright © 2018 Pearson Education, Inc.

Prehospital Emergency Care, 11e (Mistovich et al.) Chapter 14 General Pharmacology and Medication Administration 1) Which of these medications is included in the EMT scope of practice? A) Tylenol B) Oral glucose C) Motrin D) Benadryl Answer: B Diff: 1 Page Ref: 422 Objective: 14-3 2) Which of these medications is included in the EMT scope of practice? A) Pepcid B) Oxygen C) 50% dextrose (D50) D) Benadryl Answer: B Diff: 1 Page Ref: 421 Objective: 14-3 3) A patient with difficulty breathing has a metered-dose inhaler (MDI). Which of these would be a likely type of medication contained in the MDI? A) Albuterol B) Steroids C) Oxygen D) Epinephrine Answer: A Diff: 1 Page Ref: 423 Objective: 14-1 4) Which of these is a generic drug name? A) Albuterol B) Advil C) Aspirin D) Acetylsalicylic acid Answer: C Diff: 2 Page Ref: 426 Objective: 14-5

1 Copyright © 2018 Pearson Education, Inc.

5) You are by the side of an alert and oriented 56-year-old male patient who complains of generalized weakness and a pounding headache. His pulse is 104, respirations are 18 breaths/min, and blood pressure is 218/110 mmHg. He has a history of high blood pressure, for which he takes lisinopril. He states that he has not taken the Lisinopril for 2 weeks and hands you a container with the medication inside. You would: A) Assist the patient in taking the lisinopril B) Ensure that it is the patient's medication and then administer it C) Contact medical direction to request an order for lisinopril administration by you D) Continue treatment and transport the patient to the hospital Answer: D Diff: 2 Page Ref: 420 Objective: 14-4 6) As an EMT, you may assist patients in taking which kind of prescribed medication? A) Inhalers used for difficulty breathing B) Medications for high blood pressure C) Drugs for heart-rhythm disorders D) Seizure-control medications Answer: A Diff: 2 Page Ref: 423 Objective: 14-4 7) A patient with chest pain tells you that he feels the same now as when he had a heart attack 2 years ago. He is diaphoretic, nauseated, and vomiting; he has a pulse of 84, respirations of 16 breaths/min, and a blood pressure of 78/50 mmHg. The patient is allergic to aspirin. He asks you to assist him in taking one of his nitroglycerin pills because his arthritis makes it too hard for him to open the medication bottle. Which of the assessment findings represents a contraindication to the nitroglycerin? A) Blood pressure of 78/50 mmHg B) Heart rate less than 100 C) Nausea and vomiting D) Allergy to aspirin Answer: A Diff: 2 Page Ref: 429 Objective: 14-1 8) After you assist a patient with use of her EpiPen, she tells you that it is much easier for her to breathe. However, her heart rate has increased from 92 to 118 beats/min. Regarding the epinephrine, the EMT recognizes the increased heart rate as a(n): A) Therapeutic effect B) Side effect C) Contraindication D) Allergy Answer: B Diff: 2 Page Ref: 429 Objective: 14-12 2 Copyright © 2018 Pearson Education, Inc.

9) A patient with a history of diabetes is confused and irritable. According to family members, he accidentally took too much insulin this morning and did not eat breakfast. Since he is conscious with an intact gag reflex, medical direction orders you to administer oral glucose. Which component of this situation best represents the indication for the medication? A) Medical command authorization B) Confusion and irritability C) History of diabetes D) High blood sugar Answer: B Diff: 2 Page Ref: 422 Objective: 14-8 10) After administering oral glucose to a patient with altered mental status, which of these findings best indicates a positive therapeutic effect or response to the medication? A) Blood pressure decrease from 156/90 to 120/80 mmHg B) No signs or symptoms of an allergic reaction C) Pulse oximeter reading of 99% without supplemental oxygen D) Mental status that is now alert and oriented Answer: D Diff: 2 Page Ref: 422 Objective: 14-12 11) You are assisting a patient with her metered-dose inhaler (MDI). In an effort to determine the exact name of the medication, it would be best for the EMT to: A) Ask the patient the name of the medication B) Contact and describe the MDI to medical direction C) Look up the medication using the Physician's Desk Reference (PDR) D) Find the medication name on the label of the MDI Answer: D Diff: 1 Page Ref: 423 Objective: 14-2 12) A 59-year-old male patient has summoned EMS for crushing chest pain. He states that he needs his nitroglycerin, but it is in his car in a nearby parking lot. A friend of the patient explains that he also takes nitroglycerin and hands you his bottle of nitroglycerin. The patient appears very ill, and he says that this chest pain is the worst he's ever experienced. Given the patient's condition, your best action would be to: A) Administer the friend's nitroglycerin B) Have the patient retrieve his nitroglycerin from the car C) Ask the friend to get the patient's nitroglycerin from his car quickly D) Administer his friend's nitroglycerin now, but retrieve patient's nitroglycerin bottle from his car so you have proof and verification that this medication was appropriate to administer Answer: C Diff: 3 Page Ref: 430 Objective: 14-2

3 Copyright © 2018 Pearson Education, Inc.

13) Medical direction has ordered you to administer 162 mg of aspirin to a patient with chest pain. Your best response to this order would be: A) "Confirmed. I will recontact you with any changes in the patient's condition." B) "Copy. I will administer the medication as per your instruction." C) "Received. I will administer 162 mg of aspirin orally." D) "Affirmative. The patient has no allergies so I will administer the aspirin orally." Answer: C Diff: 2 Page Ref: 430 Objective: 14-9 14) Which of these descriptions on a patient care report best illustrates how the EMT should document the administration of a medication? A) 325 mg of aspirin administered orally at 1325 hours B) 0.4 of nitroglycerin administered sublingually at 0800 hours C) Medical direction (Dr. Coyle) ordered albuterol at 1245 hours. Given as ordered D) Assisted patient with administration of medication via metered-dose inhaler Answer: A Diff: 2 Page Ref: 431 Objective: 14-11 15) A patient who is short of breath and has a history of right arm paralysis from a stroke has asked you to help him use his metered-dose inhaler containing albuterol. Since your medical director requires on-line authorization for albuterol, you would: A) Contact medical direction for authorization B) Withhold the medication and transport the patient C) Advise the hospital that the patient will need albuterol D) Administer the medication and document this treatment accordingly Answer: A Diff: 2 Page Ref: 430 Objective: 14-8 16) Which of these medications can be administered by the EMT only if the patient has a prescription for it? A) Aspirin B) Nitroglycerin C) Oxygen D) Oral glucose Answer: B Diff: 1 Page Ref: 430 Objective: 14-9

4 Copyright © 2018 Pearson Education, Inc.

17) The label on a patient's prescribed metered-dose inhaler reads as follows: "Administer three puffs as needed for shortness of breath." Your protocols state that you can assist with only two puffs without medical direction consultation. The patient is extremely short of breath. What should you do? A) Administer three puffs B) Do not administer the medication C) Contact medical direction for orders to administer three puffs D) Administer two puffs Answer: C Diff: 2 Page Ref: 430 Objective: 14-9 18) Which of these is one of the five rights of medication administration? A) Right dose B) Right EMT C) Right allergies D) Right physician Answer: A Diff: 1 Page Ref: 431 Objective: 14-10 19) What is the best description of the purpose of the five rights of drug administration? A) Makes medication administration easier B) Decreases the risk of a medication error C) Ensures therapeutic benefit to the patient D) Prevents clinical deterioration of the patient Answer: B Diff: 2 Page Ref: 431 Objective: 14-10 20) A patient with severe chest pain hands you a bottle of nitroglycerin. After reading the label, you determine that the prescribed dose is two pills when chest pain is present, but the medication expired almost a month ago. You should: A) Administer two of the nitroglycerin tablets B) Withhold the medication and transport the patient C) Administer the nitroglycerin orally D) Place one tablet under the patient's tongue Answer: B Diff: 2 Page Ref: 431 Objective: 14-12

5 Copyright © 2018 Pearson Education, Inc.

21) What is the most reliable source of information about a medication? A) Nurses who routinely administer medications B) Paramedics and Advanced EMTs C) Physician's Desk Reference (PDR) D) EMS-related magazines and websites Answer: C Diff: 1 Page Ref: 432 Objective: 14-13 22) When reviewing a prehospital care report, you note that a patient received nitroglycerin sublingually. As an EMT, you should recognize that this medication was a: A) Mist inhaled into the lungs B) Tablet placed under the tongue C) Medication injected into a muscle D) Tablet chewed or swallowed whole Answer: B Diff: 1 Page Ref: 427 Objective: 14-6 23) After you assist a patient with use of his metered-dose inhaler, the patient's son asks why his father cannot take the medication in the form of a pill. Which of these responses would be most appropriate? A) "It would be best if you asked the doctor because I cannot legally answer that question." B) "By giving the medication into the lungs, the medicine goes directly to where it's needed." C) "Your father could take this medication by spraying it under his tongue. Check with his doctor?" D) "There are only a few medications that come in pill form, and this is not one of them." Answer: B Diff: 2 Page Ref: 427-428 Objective: 14-6 24) In which form is activated charcoal administered? A) Suspension B) Syrup C) Aerosolized powder D) Tablet Answer: A Diff: 1 Page Ref: 422 Objective: 14-7

6 Copyright © 2018 Pearson Education, Inc.

25) What is the best reason why the EMT administers or assists with the administration of medications? A) To help stabilize a patient's medical condition B) To prevent illness and injury C) To justify a lights-and-siren transport D) To expand the scope of the EMT's practice Answer: A Diff: 2 Page Ref: 420 Objective: 14-2 26) Which of these statements, made by the EMT, shows an understanding of medication administration and drug dosages? A) "It is best to administer a little less than the required dose to decrease the chance of an allergic reaction." B) "If the patient is very sick, the dose of a medication can be increased to help the patient recover faster." C) "The proper dose of a medicine should be administered unless medical direction orders otherwise." D) "If a patient has chest pain with shortness of breath, the dose of a medication must be increased." Answer: C Diff: 2 Page Ref: 4319 Objective: 14-10 27) You have administered Xopenex to a patient, using a small-volume nebulizer. You then notify medical direction that you have administered the drug. This is an example of which type of medical control? A) On-line B) Direct C) Off-line D) Retrospective Answer: C Diff: 1 Page Ref: 430 Objective: 14-8 28) What is the most likely indication for the administration of oral glucose to a patient with diabetes? A) Chest pain B) Low blood sugar C) Diabetic history D) Headache Answer: B Diff: 1 Page Ref: 422 Objective: 14-10

7 Copyright © 2018 Pearson Education, Inc.

29) The EMT should consider aspirin for a patient with: A) Chest discomfort B) Headache C) Shortness of breath D) Fever Answer: A Diff: 1 Page Ref: 431 Objective: 14-10 30) Atrovent, if prescribed to the patient, is indicated for: A) Headache B) Chest discomfort C) Nausea or vomiting D) Difficulty breathing Answer: D Diff: 1 Page Ref: 423 Objective: 14-4 31) The EMT shows she understands the difference between a metered-dose inhaler (MDI) and a small-volume nebulizer (SVN) when she states: A) "The MDI uses oxygen to administer the medication; the SVN does not." B) "The MDI provides medication during one or two of the patient's breaths; the SVN provides a continuous flow of medication until gone." C) "The MDI uses a mask to administer the medication; the SVN merely requires the patient to hold the device in his mouth and inhale." D) "The MDI requires the EMT to set up and prepare the medication for administration; the SVN does not." Answer: B Diff: 2 Page Ref: 423 Objective: 14-7 32) A beta-2 medication benefits a patient by: A) Diminishing the need for oxygen B) Slowing the respiratory and heart rate C) Eliminating chest pain D) Dilating the small airways Answer: D Diff: 3 Page Ref: 423 Objective: 14-12

8 Copyright © 2018 Pearson Education, Inc.

33) A 66-year-old male patient is complaining of chest pain. The EMT is considering assisting with the patient's prescribed nitroglycerin. Which of these statements made by the patient is most relevant to the EMT at this time? A) "I forgot to take my high blood pressure medication this morning." B) "I accidentally took five, not four, baby aspirins when the chest pain first started." C) "I took a nitroglycerin pill yesterday for the pain and it didn't do anything." D) "My urologist just started treating me for problems I am having with sex." Answer: D Diff: 3 Page Ref: 424 Objective: 14-2 34) Which medical emergency would most likely be treated with the use of epinephrine by the EMT? A) Cardiac arrest B) Asthma C) Bee sting D) Hypoglycemia Answer: C Diff: 1 Page Ref: 424-425 Objective: 14-2 35) Which of these medications would the EMT administer PO? A) Aspirin B) Nitroglycerin C) Albuterol D) Epinephrine Answer: A Diff: 1 Page Ref: 423 Objective: 14-6 36) The route by which the EMT would administer epinephrine is: A) Oral B) Intramuscular C) Sublingual D) Topical Answer: B Diff: 1 Page Ref: 425 Objective: 14-6

9 Copyright © 2018 Pearson Education, Inc.

37) Which of these actions would be most important when administering aspirin to a patient with chest pain? A) Instructing the patient to chew the medication prior to swallowing B) Giving the patient water with the aspirin C) Determining that the patient has a prescription for the aspirin D) Ensuring that the patient can follow directions and has an intact gag reflex Answer: D Diff: 2 Page Ref: 427 Objective: 14-6 38) A patient with a cardiac history complains of chest pain. He already took four baby aspirins, and medical direction has given you permission to assist the patient with his nitroglycerin dose. After you administer nitroglycerin to the patient, he complains of a headache. What would be your most appropriate next step? A) Inform the patient that he is allergic to nitroglycerin B) Administer additional aspirin for the headache C) Recognize that the patient has a contraindication to the administration of nitroglycerin D) Realize that the headache is an undesirable side effect of the nitroglycerin and continue care Answer: D Diff: 2 Page Ref: 429 Objective: 14-2 39) Which of these medications may be administered by a mucosal atomizer device (MAD) by the EMT if the EMT has authorization from medical direction to do so? A) Naloxone hydrochloride B) Epinephrine hydrochloride C) Glucose D) Activated charcoal Answer: A Diff: 1 Page Ref: 426 Objective: 14-6 40) You are treating a patient who has a history of gout, hypertension, diabetes, emphysema, and high cholesterol. As you review his medications, you notice that one of his pill bottles has the name "levalbuterol" on its label. You would recognize this name as: A) The trade name for glutose B) The generic name for Xopenex C) The generic name for Narcan Nasal Spray D) The chemical name for acetylsalicylic acid Answer: B Diff: 2 Page Ref: 427, Table 14-1 Objective: 14-5

10 Copyright © 2018 Pearson Education, Inc.

41) Which piece of information regarding drug administration by the EMT according to protocol is not necessary to document on the patient care report? A) The dose of the drug given B) The patient's response to the drug given C) The family's reaction to the need for the medication to be administered D) The side effects, both desired and undesired, that occurred following administration Answer: C Diff: 1 Page Ref: 431 Objective: 14-11 42) During a lecture you are attending that covers the pharmacology information often used by EMS providers, the lecturer references a drug source you know to be incorrectly stated. Which of these is a fictitious or incorrect name for an information source? A) American Hospital Formulary Services B) CDC Guide to Pharmacology C) Physician's Desk Reference D) AMA Drug Evaluation Answer: B Diff: 2 Page Ref: 432 Objective: 14-13

11 Copyright © 2018 Pearson Education, Inc.

Prehospital Emergency Care, 11e (Mistovich et al.) Chapter 15 Shock and Resuscitation 1) Immediately following the onset of cardiac arrest, brain cells begin to die after: A) 1 minute B) 5 minutes C) 10 minutes D) 15 minutes Answer: B Diff: 1 Page Ref: 452 Objective: 15-2 2) Resuscitation, when started during which phase of cardiac arrest, provides the patient with the best chance of survival? A) Circulatory B) Metabolic C) Electrical D) Bradycardic Answer: C Diff: 3 Page Ref: 452 Objective: 15-10 3) A patient arrests at 1313 hours. E-911 is activated and dispatches an ambulance at 1315 hours. The ambulance arrives on scene at 1319 hours, and the EMTs reach the patient's side and start care at 1321 hours. After transporting the patient, the patient is transferred to the ED staff at 1346 hours. Based on that information, which of these statements would be considered correct? A) Down time is 8 minutes B) Total down time is 31 minutes C) Down time is 25 minutes D) Total down time is 6 minutes Answer: A Diff: 2 Page Ref: 453 Objective: 15-1 4) You have been asked to describe the American Heart Association's Chain of Survival to a group of Emergency Medical Responder students. Which of these is the best description of this concept? A) A sequence of care that provides instructions on how to use an AED B) A sequence of events that, if enacted quickly, gives the patient the best chance of surviving cardiac arrest C) A treatment plan that helps to prevent and treat cardiac arrest in the general population D) A treatment plan, that if followed, almost guarantees the survival of a patient with cardiac arrest Answer: B Diff: 3 Page Ref: 453 Objective: 15-1 1 Copyright © 2018 Pearson Education, Inc.

5) When presenting information on cardiac arrest and automated external defibrillation to a community group, a man asks why people should perform CPR prior to the arrival of EMS if the EMS responders will provide a shock after they arrive. Your response would be: A) "Immediate CPR can prolong the period in which the heart can be successfully shocked into a perfusing rhythm following cardiac arrest." B) "In many cases, CPR can reverse the cardiac arrest, making a shock by the AED unnecessary since the shock may further damage the heart." C) "In the American Heart Association's Chain of Survival, CPR is the most important link in surviving cardiac arrest that occurs outside the hospital." D) "CPR is needed to keep blood flowing through the body so the EMTs can give the patient IV medications when they arrive." Answer: A Diff: 3 Page Ref: 452 Objective: 15-14 6) Which cardiac arrest rhythm is the AED designed to shock? A) Asystole B) Pulseless electrical activity C) Ventricular fibrillation D) Bradycardia Answer: C Diff: 1 Page Ref: 454 Objective: 15-15 7) The EMT should request advanced life support (ALS) backup for a patient in cardiac arrest because: A) Paramedics must be present for the EMT to use the AED B) Patients with cardiac arrest must be transported by ALS personnel C) ALS care is superior to basic life support care, even when the AED is available D) ALS treatment decreases the possibility of the patient going back into cardiac arrest once successful defibrillation has occurred Answer: D Diff: 1 Page Ref: 454 Objective: 15-19 8) The AED should never be applied to a patient who is not in cardiac arrest because some patients in: A) Ventricular tachycardia may still have a pulse B) Ventricular fibrillation may still be conscious and alert C) Ventricular fibrillation may still have a pulse D) Asystole may still have a pulse Answer: A Diff: 3 Page Ref: 457 Objective: 15-15

2 Copyright © 2018 Pearson Education, Inc.

9) Which of these is considered a benefit of the automated external defibrillator? A) The ease and speed with which it can be used B) Elimination of the need for a primary assessment C) Its ability to identify and confirm cardiac arrest D) The need for little-to-no CPR training to use the device Answer: A Diff: 1 Page Ref: 455 Objective: 15-15 10) Which link of the American Heart Association's Chain of Survival must occur first, if a patient is to survive cardiac arrest? A) Early CPR B) Early defibrillation C) Early activation D) Early advanced life support Answer: C Diff: 1 Page Ref: 453 Objective: 15-12 11) A patient goes into cardiac arrest at 11:40 A.M. Which of these treatments gives the patient the best chance for recovery? A) CPR at 11:42 A.M. and advanced cardiac drugs at 11:48 A.M. B) Defibrillation at 11:46 A.M., followed by ALS (ACLS) at 11:51 A.M. C) CPR at 11:41 A.M. and defibrillation at 11:43 A.M. D) Defibrillation at 11:44 A.M., followed by CPR at 11:49 A.M. Answer: C Diff: 2 Page Ref: 453 Objective: 15-13 12) Why is defibrillation in the first few minutes of cardiac arrest so critical? A) If cardiac arrest is not treated within the first few minutes, ventricular fibrillation will convert to asystole, a nonshockable heart rhythm B) Research has shown that a heart in cardiac arrest will continue to pump blood for a few minutes before cardiac output drops to zero C) In the first few minutes of cardiac arrest, the blood pressure is still normal, but will drop quickly, making successful defibrillation less likely D) In the first few minutes of cardiac arrest, the heart is still warm, but it begins to cool rapidly, which makes it less receptive to a defibrillatory shock Answer: A Diff: 3 Page Ref: 454 Objective: 15-13

3 Copyright © 2018 Pearson Education, Inc.

13) Which of these statements about the heart rhythm of ventricular fibrillation is true? A) Ventricular fibrillation occurs when the heart rate is so slow and weak that a pulse cannot be felt B) When a patient's heart is in ventricular fibrillation, it is unable to pump blood throughout the body C) The most effective treatment for converting ventricular fibrillation to a normal heart rhythm is CPR D) The AED is designed to identify ventricular tachycardia and give a "no shock advised" message when it is present Answer: B Diff: 3 Page Ref: 452 Objective: 15-1 14) A patient has just gone into cardiac arrest. His heart is most likely in which rhythm? A) Ventricular fibrillation B) Ventricular asystole C) Ventricular tachycardia D) Ventricular PEA Answer: A Diff: 1 Page Ref: 452 Objective: 15-16 15) Within 2 minutes of going into cardiac arrest, an AED is applied, the patient is shocked, and a pulse is restored. The EMT should recognize that the patient's heart was in: A) Ventricular fibrillation and now is in cardiac arrest B) Asystole and now is in an organized rhythm C) Ventricular fibrillation and now is in an organized rhythm D) Ventricular fibrillation and now is in asystole Answer: C Diff: 3 Page Ref: 454 Objective: 15-17 16) A 47-year-old patient has been in cardiac arrest for 6 minutes. While you set up the AED, you would direct your partner to: A) Perform CPR at a ratio of 30 ventilations to 2 compressions B) Assist you in making sure that the AED is ready for application C) Perform a primary and secondary assessment D) Perform CPR until the AED is ready Answer: D Diff: 1 Page Ref: 454 Objective: 15-17

4 Copyright © 2018 Pearson Education, Inc.

17) Because of a shortage of paramedics at your ambulance service, it has been announced that there will be AEDs placed on every ambulance for use by EMT crews. Which of these statements indicates an understanding of how this policy will affect the EMTs? A) "Cardiopulmonary resuscitation will no longer be needed." B) "The AED will let us know whether to shock the patient." C) "EMTs will now be able to shock all patients in cardiac arrest." D) "EMTs will have to learn how to interpret ECG tracings." Answer: B Diff: 2 Page Ref: 455 Objective: 15-15 18) Which of these statements about the AED and its use in the treatment of cardiac arrest is true? A) AEDs have simplified the treatment of cardiac arrest to the point where the EMT does not have to be worried about inappropriately shocking a patient B) Research has shown that the first shock delivered by an AED is often faster than the first shock delivered by a manual defibrillator C) To use an AED, the EMT must be able to identify some basic cardiac arrest heart rhythms so that he or she can tell the AED to shock or not shock the patient D) The AED is advantageous in that it will determine if a patient is in cardiac arrest and whether or not to shock the patient Answer: B Diff: 2 Page Ref: 455 Objective: 15-15 19) Your service has a new AED. During the in-service program on the new device, the instructor informs you that it is a semi-automated AED and uses a biphasic wave form, as opposed to the monophasic form used by the previous AED. As a knowledgeable EMT, you should recognize that: A) The new AED will defibrillate the patient with smaller amounts of electrical energy B) The EMT will not need to press a "shock" button to shock the patient C) The new AED will defibrillate the patient with larger amounts of electrical energy D) Less energy but more shocks will be needed to treat cardiac arrest Answer: A Diff: 3 Page Ref: 456 Objective: 15-15

5 Copyright © 2018 Pearson Education, Inc.

20) As you arrive at a metal scrap yard for an unknown medical emergency, you observe a male supine on the ground with AED electrodes on his chest. The AED operator has just ordered coworkers to clear the patient because the AED is going to shock. Within seconds, the AED delivers a shock without the operator pressing a "shock" button. The EMT should recognize that which type of AED is being used? A) Semi-automated B) Manual C) Fully automated D) Biphasic Answer: C Diff: 2 Page Ref: 456 Objective: 15-21 21) It is critical that the EMT never apply the AED to a person who is not in cardiac arrest because an accidental shock could: A) Cause the patient extreme pain B) Cause the beating heart to go into cardiac arrest C) Cause the patient to lose his eyesight D) Produce full-thickness burns and lead to a deadly infection Answer: B Diff: 2 Page Ref: 457 Objective: 15-15 22) If a cardiac arrest patient were in asystole, which message would the AED provide? A) "Shock advised" B) "Check electrodes" C) "No shock advised" D) "Press analyze" Answer: C Diff: 3 Page Ref: 457 Objective: 15-16 23) When a 52-year-old male patient collapsed on his front porch, his family dialed 911. When you arrive, the daughter informs you that the patient has been down for approximately 8 minutes. Assessment reveals him to be in cardiac arrest. What should you do immediately? A) Contact medical direction for permission to use the AED given the down time B) Perform five cycles of CPR at a ratio of 15 compressions to 2 ventilations C) Withhold CPR so that the AED can be applied D) Start CPR while the AED is applied and readied for use Answer: D Diff: 2 Page Ref: 460 Objective: 15-17

6 Copyright © 2018 Pearson Education, Inc.

24) A 7-year-old boy choked on a grape at school. By the time the obstruction was removed, assessment revealed him to be in cardiac arrest. When you arrive, teachers are performing CPR. The school nurse informs you that CPR has been in progress for 6 minutes. You have an AED, but do not have a pediatric conversion device that reduces the energy of defibrillation from that of an adult to that of a child. What should you do immediately? A) Perform five abdominal thrusts and then 1 minute of CPR before using the AED B) Continue CPR and transfer the boy to the stretcher for immediate transport C) Place the adult AED electrodes on the boy's chest and follow the AED's prompts D) Instruct the teachers to continue CPR for 2 additional minutes before applying the AED Answer: C Diff: 2 Page Ref: 458 Objective: 15-17 25) You have arrived at a residence where a 4-month-old baby was found in his crib in cardiac arrest. Emergency Medical Responders have been on scene for 5 minutes prior to arrival. They began CPR immediately on reaching the patient's side. You have an AED with adult pads with you, but not pediatric pads. After rechecking and confirming that the patient is in cardiac arrest, you would: A) Continue CPR and transfer the patient to the stretcher for transport B) Place the AED on the patient and follow all prompts C) Continue CPR for 2 more minutes before placing the AED on the patient D) Discuss with the parents whether they want to proceed with use of the AED Answer: B Diff: 2 Page Ref: 458 Objective: 15-17 26) An unrestrained 37-year-old female patient is in cardiac arrest after her vehicle struck a tree head-on at a high rate of speed. Assessment reveals that the patient suffered massive blunt trauma to the chest and abdomen. The patient is quickly extricated from the vehicle and placed in a supine position on the ground. It is estimated that the patient has been in cardiac arrest for approximately 10 minutes, during which time CPR was not performed. At this time, the EMT would: A) Withhold CPR and perform a focused trauma assessment B) Perform CPR and contact medical command for permission to use the AED C) Administer CPR for 2 minutes prior to applying the AED D) Apply electrodes to the patient's chest and follow the AED's instructions Answer: B Diff: 3 Page Ref: 458 Objective: 15-17

7 Copyright © 2018 Pearson Education, Inc.

27) You have been dispatched to a residence for a male patient with a cardiac history who is complaining of chest pain. On scene, you find a 52-year-old man sitting in a chair. He is alert and oriented. He states that his chest pain feels like the last time he had a heart attack. He reports that in the hospital his heart stopped and the health care personnel had to shock him twice before it restarted. He is breathing adequately and has a strong radial pulse. Which of these actions would be appropriate in the assessment and/or management of this patient? A) Apply the AED to the patient but do not turn it on B) Provide positive pressure ventilation with high-concentration oxygen C) Open the patient's airway using the head-tilt, chin-lift maneuver D) Obtain the patient's heart rate, respiratory rate, and blood pressure Answer: D Diff: 1 Page Ref: 449-450 Objective: 15-9 28) You have arrived at the residence of a 66-year-old female patient who is in cardiac arrest. In the living room, you find Emergency Medical Responders performing CPR on the patient. They quickly report that they found the patient in cardiac arrest and have been doing CPR for 5 minutes. Which of these statements should you make at this time? A) "Let's stop CPR so I can check the airway, breathing, and circulation." B) "Let's stop CPR so I can put the electrodes of the AED on her chest." C) "Let's continue CPR for another 2 minutes, and then I will put the AED on." D) "Let's continue CPR while I talk with the family to see if they want us to continue." Answer: A Diff: 2 Page Ref: 459 Objective: 15-17 29) You have been called for an elderly male patient who suddenly collapsed. On scene, you find an 82-year-old man lying on the garage floor. Assessment reveals him to be unresponsive and not breathing. What should you do immediately? A) Determine the "down time" B) Apply the AED C) Start cardiopulmonary resuscitation D) Check for a carotid pulse Answer: D Diff: 2 Page Ref: 459 Objective: 15-17 30) Which of these unresponsive patients would the EMT recognize as in cardiac arrest? A) A female who has an occasional gasp for a breath but no palpable carotid pulse B) A male with a heart rate of 16 beats/min and agonal respirations C) A female who is not breathing and has a heart rate of 24 beats/min D) A male with an absent radial pulse but breathing 20 times per minute Answer: A Diff: 2 Page Ref: 459 Objective: 15-17

8 Copyright © 2018 Pearson Education, Inc.

31) When in shock, the body has a slightly delayed compensation mechanism that utilizes: A) Hyperventilation B) Vasodilation C) Nerve stimulation D) The release of hormones Answer: D Diff: 2 Page Ref: 446 Objective: 15-5 32) You have just applied the AED to a female patient in cardiac arrest and the machine is ready to analyze the heart rhythm. Which of these instructions is appropriate to give at this time? A) "Stop CPR and clear the patient." B) "Stop chest compressions but continue ventilation." C) "Continue CPR until we see if a shock is advised." D) "Stop CPR but continue to check for a pulse." Answer: A Diff: 2 Page Ref: 461 Objective: 15-15 33) After analyzing the heart rhythm of a patient in cardiac arrest, the AED provides a "deliver shock" message. After clearing the patient, the EMT's next step should be to: A) Press the shock button, and then check for the return of a pulse B) Press the shock button, and then allow the AED to analyze the heart rhythm C) Press the shock button, and then perform CPR for 2 minutes D) Check for a pulse, and then press the shock button if a pulse is not present Answer: C Diff: 2 Page Ref: 461 Objective: 15-17 34) The AED has just been applied to a female patient in cardiac arrest. After analyzing the heart rhythm, it provides a "no shock advised" message. The EMT should immediately: A) Reanalyze the patient's heart rhythm B) Resume cardiopulmonary resuscitation C) Check the electrodes for proper placement D) Check the patient for a pulse and blood pressure Answer: B Diff: 2 Page Ref: 461 Objective: 15-17

9 Copyright © 2018 Pearson Education, Inc.

35) After you applied the AED to a patient in cardiac arrest, it delivered a shock. Immediately after the shock, what should you do next? A) Check the patient for a pulse and reanalyze the rhythm if no pulse is present B) Provide 2 minutes of CPR, and then check the patient for a pulse C) Check the patient for a pulse and start CPR if no pulse can be located D) Provide five cycles of single-rescuer CPR with a ratio of 15 compressions to 2 ventilations Answer: B Diff: 2 Page Ref: 461 Objective: 15-17 36) Which of these statements made by your EMT partner regarding care for the adult patient in cardiac arrest is accurate? A) "It is important to always check for a pulse immediately after the AED has delivered a shock." B) "When using an AED, the machine should be turned on before applying the electrodes." C) "When checking for a patient's pulse, the EMT should take no more than 5 seconds." D) "If two EMTs are performing CPR, a ratio of 15 compressions to 2 ventilations can be used." Answer: B Diff: 3 Page Ref: 460 Objective: 15-17 37) While using the AED to treat a patient in cardiac arrest, your partner informs you that he can feel a carotid pulse. What should you do immediately? A) Obtain a heart rate and blood pressure B) Reanalyze the patient's heart rhythm C) Insert an oropharyngeal airway and begin positive pressure ventilation D) Assess the patient's airway and breathing adequacy Answer: D Diff: 2 Page Ref: 461 Objective: 15-18 38) When should the EMT transport the patient with cardiac arrest? A) After three "no shock advised" messages are received B) Immediately upon determining that the patient is in cardiac arrest C) After one shock has been delivered and the patient remains in cardiac arrest D) Before delivering the first shock when a "shock advised" message is received Answer: A Diff: 3 Page Ref: 462 Objective: 15-17

10 Copyright © 2018 Pearson Education, Inc.

39) A "no shock advised" message is provided by the AED. The EMT understands that this could mean: A) The patient is in ventricular fibrillation B) The electrodes may be loose C) The patient has regained a pulse D) CPR is no longer needed Answer: C Diff: 2 Page Ref: 461 Objective: 15-17 40) After the AED gives a "no shock advised" message, it is determined that the patient has a weak pulse and slow and shallow respirations. You would immediately: A) Transfer the patient to the stretcher for transport B) Provide 2 minutes of CPR C) Start positive pressure ventilation D) Obtain a blood pressure Answer: C Diff: 2 Page Ref: 462 Objective: 15-18 41) You have been assigned to a football game to provide standby coverage. While taking a break, you decide to walk to a nearby concession stand for a soft drink. On the way, you come across a group of people standing around a male patient who collapsed and is on the ground. Your assessment reveals him to be unresponsive, not breathing, and pulseless. A bystander states that the patient just collapsed moments ago. An AED is located less than 1 minute from your location. The nearest EMT is 5 minutes from your location. What should you do immediately? A) Start CPR and wait for the AED B) Retrieve the nearby AED C) Call for assistance and start CPR, continuing it until the other EMT arrives D) Perform manual in-line spinal stabilization Answer: B Diff: 2 Page Ref: 453-454 Objective: 15-17 42) After two cardiac arrests occurred on its premises over the last 6 months, a large home improvement center has purchased a fully automated AED. You have been asked to provide education on its use. During an instructional session, a student asks you what will happen when the AED indicates that a shock is indicated. Your response should be: A) "The AED will prompt you to recheck the pulse." B) "You will need to reanalyze the heart rhythm." C) "The AED will automatically charge and shock the patient." D) "You will need to press the shock button after the system charges." Answer: C Diff: 2 Page Ref: 461 Objective: 15-15

11 Copyright © 2018 Pearson Education, Inc.

43) A patient in cardiac arrest has regained a pulse after two shocks, but is still apneic. Which action would be appropriate next? A) Transport the patient supine and secured to a long spine board as treatment is continued B) Remove the AED once it is determined that a pulse has returned and initiate transport C) Reanalyze the patient's heart rhythm with the AED every 5 minutes D) Cancel the ALS assistance request and proceed directly to the hospital Answer: A Diff: 2 Page Ref: 462 Objective: 15-18 44) You are transporting an unresponsive 31-year-old female patient who suffered cardiac arrest. On scene, Emergency Medical Responders (EMRs) shocked the patient once with the AED and she regained a pulse. She remains unresponsive and has shallow respirations. Ventilation is being provided with a bag-valve mask. An EMR has agreed to ride to the hospital with you to provide assistance in caring for the patient. Which set of instructions given to the EMR next is most appropriate? A) "Place the patient in semi-Fowler's position so her airway is clear if she vomits." B) "Leave the AED on the patient, even though she is breathing and has a pulse." C) "Please check the patient's breathing and pulse every 5 minutes." D) "Place the patient on a nonrebreather mask with 15 liters per minute of oxygen." Answer: B Diff: 2 Page Ref: 462 Objective: 15-18 45) You are transporting a 57-year-old male patient who went into cardiac arrest at home. After two shocks and CPR, he regained a pulse, but he remains unresponsive and in respiratory arrest. During transport, your reassessment reveals the absence of a carotid pulse. What should you do first? A) Reapply the AED and analyze the heart rhythm B) Provide five cycles of CPR prior to using the AED C) Start CPR and continue emergency transport D) Stop the ambulance and analyze the heart rhythm with the AED Answer: D Diff: 2 Page Ref: 462 Objective: 15-18

12 Copyright © 2018 Pearson Education, Inc.

46) A 51-year-old male patient who experienced cardiac arrest has regained a pulse after one shock from the AED. The dispatcher informs you that the paramedic unit you have requested for assistance is coming from the eastern end of the county and has a 20-minute ETA. The hospital is 15 minutes west of your location. Which action would be most appropriate given this situation? A) Load the patient and meet the paramedic unit at a halfway point B) Wait on scene for the paramedic unit to arrive C) Load the patient in the ambulance and wait on scene for the paramedic unit D) Start transport to the hospital and inform the ALS dispatcher of your intent Answer: D Diff: 2 Page Ref: 463 Objective: 15-19 47) An EMT has just received a "shock advised" message from the AED. Just before delivering the shock, the EMT must: A) Press the analyze button once more B) Hold the patient's head to avoid injury when shocking C) Ensure that all rescuers are clear of the patient D) Remove the oral airway to prevent possible choking Answer: C Diff: 1 Page Ref: 457 Objective: 15-20 48) You have been called to a public pool for an unresponsive patient. On arrival, you find lifeguards performing CPR with a pocket mask and oxygen on a 67-year-old male swimmer. They report that the patient was in the water and was seen clutching his chest seconds before going unresponsive. He was immediately pulled from the water and CPR was initiated. The lifeguards estimate that CPR has been performed for 5 minutes. Assessment shows the man to be unresponsive, apneic, and pulseless. What should be your first response? A) "Stop CPR and let's apply the AED." B) "We need to quickly dry him from head to toe before applying the AED." C) "Let's start ventilation with a bag-valve mask and oxygen." D) "Resume CPR and let's take a towel and dry off his chest so we can apply the AED electrodes." Answer: D Diff: 2 Page Ref: 460 Objective: 15-17 49) You are transporting a patient in cardiac arrest. The AED is being used and a shock has been advised. Prior to administering the shock, what should you do? A) Ensure that no rescuer is touching the stretcher B) Move the patient from the metal stretcher to a nonmetal surface C) Withhold all shocks because metal will conduct the shock into the ambulance D) Stop the ambulance and have all rescuers exit prior to shocking the patient with the AED Answer: A Diff: 2 Page Ref: 457 Objective: 15-20 13 Copyright © 2018 Pearson Education, Inc.

50) Which of these actions performed by the EMT indicates appropriate care with the AED when treating a patient in cardiac arrest? A) The EMT places the AED pads 2 inches away from a transdermal medication patch on the patient's chest B) The EMT intentionally withholds a shock on a patient in cardiac arrest who has an implantable defibrillator C) The EMT places, then quickly removes, a set of electrodes in an attempt to remove excessive hair from a patient's chest D) The EMT applies one electrode over the top of the power source for a pacemaker located on the patient's chest Answer: C Diff: 2 Page Ref: 464 Objective: 15-15 51) You have applied the AED's electrodes to an obese male patient in cardiac arrest. When you press the analyze button, the AED gives you a "check electrode" message. In looking at the patient, which of these factors would most likely be responsible for this message? A) Obese chest and abdomen B) History of asthma C) Hairy chest D) AED pads placed too far to the left Answer: C Diff: 2 Page Ref: 463 Objective: 15-15 52) You are instructing a first-aid class at a local chemical plant. The course includes instruction regarding an AED that is to be placed in the plant. Which point would you emphasize to the employees to help them avoid the most common cause of AED failure? A) "The batteries must be checked regularly." B) "An extra set of electrodes should be kept in the AED." C) "The AED should be kept clean and undamaged." D) "Always check the electrode wires for cracks." Answer: A Diff: 2 Page Ref: 466 Objective: 15-21 53) The EMT realizes that the best means of preventing failure of the AED is to: A) Make sure that the AED is kept clean and free of damage B) Check the AED battery and its supplies at the beginning of each shift C) Check the electrodes monthly and replace them when expired D) Obtain a second set of batteries when the original set is no longer functional Answer: B Diff: 2 Page Ref: 466 Objective: 15-21

14 Copyright © 2018 Pearson Education, Inc.

54) Which activity related to the use of an AED would a medical director carry out? A) Overseeing repairs to AEDs that are in need of service or repair B) Responding to calls in which the EMT is using the AED C) Obtaining funding for supplies such as batteries, razors, and electrodes D) Reviewing cases in which an AED was used but no shock was advised Answer: D Diff: 2 Page Ref: 466 Objective: 15-21 55) Your ambulance service director has given you permission to replace an old fully automated AED with a new semi-automated AED produced by a different manufacturer. Before making the purchase, you must receive authorization to do so from the: A) Shift supervisor B) Old manufacturer C) American Heart Association D) EMS system's medical director Answer: D Diff: 1 Page Ref: 466 Objective: 15-21 56) The owner of a day care center for adults with Alzheimer's disease calls you to ask about an AED at her facility. Specifically, she asks if she will need a physician to oversee the AED program. Your reply should be: A) "If you intend to use the AED on anyone younger than 50 years, a medical director will be needed." B) "A medical director is needed only if you are going to bill insurance companies for the provision of care; if you are not, a medical director is not needed." C) "You will need to have a physician medical director since the AED can be used only with his or her permission under his or her license." D) "If you get a semi-automated AED, you will need a physician medical director. Since fully automated AEDs are easier to use, a medical director is not needed." Answer: C Diff: 2 Page Ref: 466 Objective: 15-21 57) A patient in early shock informs the EMT that he has had severe diarrhea and vomiting over the past four days. Given this history, the EMT should recognize the pathophysiology of the shock as: A) Loss of red blood cells B) Decreased formed elements in the blood C) Leakage of the capillaries D) Loss of plasma volume Answer: D Diff: 3 Page Ref: 438 Objective: 15-4

15 Copyright © 2018 Pearson Education, Inc.

58) A patient with a severe gastrointestinal bleeding is in shock. Which ALS or hospital interventions would best correct the patient's underlying problem? A) Intravenous fluids B) Blood replacement C) High-concentration oxygen via endotracheal tube D) Replacement of lost electrolytes by intravenous infusion Answer: B Diff: 3 Page Ref: 438 Objective: 15-4 59) Which of these condition(s) is (are) as the most probable cause of cardiogenic shock? A) Myocardial infarction B) Severe vomiting and diarrhea C) Gastrointestinal bleed D) Systemic infection Answer: A Diff: 2 Page Ref: 438 Objective: 15-4 60) When paramedics administer IV fluids as treatment for hemorrhagic shock, the fluids will: A) Stabilize the shock by restoring needed fluid and electrolytes to the intravascular volume B) Enable the lungs to better oxygenate the blood by increasing the hemoglobin level in the bloodstream C) Not be able to increase the oxygen-carrying capability of the blood D) Reverse the shock by increasing the blood pressure and inhibit anaerobic metabolism Answer: C Diff: 3 Page Ref: 438 Objective: 15-3 61) Which of these are some of the major categories of shock that the EMT will likely encounter? A) Hemorrhagic, distributive, anoxic, and obstructive B) Burn, hypovolemic, distributive, and hypoxic C) Hypoglycemic, obstructive, distributive, and hypovolemic D) Hypovolemic, cardiogenic, obstructive, and distributive Answer: D Diff: 1 Page Ref: 440-442 Objective: 15-4

16 Copyright © 2018 Pearson Education, Inc.

62) Which statement made by a patient's family member would cause the EMT to suspect that a patient is experiencing hypovolemic shock? A) "He has had a rash for the past three days." B) "He cannot stop throwing up." C) "He has been taking an antibiotic for a chest cold." D) "He got up this morning and was having a hard time breathing." Answer: B Diff: 2 Page Ref: 441 Objective: 15-3 63) The underlying pathophysiology of distributive shock is: A) Poor fluid intake B) Loss of blood volume C) Damaged heart with poor contractility D) Dilation of the blood vessels Answer: D Diff: 2 Page Ref: 441 Objective: 15-2 64) Which of these conditions could be responsible for causing obstructive shock? A) Infection throughout the body B) Loss of blood in the urine C) Blood clots in the lungs D) Poor transfer of oxygen at the capillary level Answer: C Diff: 2 Page Ref: 441-442 Objective: 15-2 65) A patient in shock with abdominal pain indicates he noticed a lot of blood in the toilet after having a bowel movement this morning. The EMT should suspect which type of shock? A) Septic hypovolemic B) Hemorrhagic hypovolemic C) Hypoxic hypovolemic D) Nonhemorrhagic hypovolemic Answer: B Diff: 2 Page Ref: 443 Objective: 15-4

17 Copyright © 2018 Pearson Education, Inc.

66) You are treating a patient with severe crushing chest pain. She has had two heart attacks in the past, and you believe that she is currently in cardiogenic shock. Blood pressure is 86/66 mmHg, respirations are 20 breaths/min, heart rate is 102 beats/min, and room-air pulse oximetry reading is 91%. Which intervention is the most appropriate next action in this case? A) Supplemental oxygen B) Application of the AED C) Administration of nitroglycerin D) Rapid transport with the patient supine Answer: A Diff: 2 Page Ref: 445 Objective: 15-9 67) You are assessing an elderly patient with a decreased level of consciousness. Your assessment reveals the patient to have a patent airway, labored respirations, and weak, rapid pulses. The skin is pale, cool, and cyanotic in the extremities. You also observe diaphoresis and a delayed capillary refill. Vital signs for this patient are heart rate, 136 beats/min; blood pressure, 66/40 mmHg; and respirations, 40 and shallow. Auscultation of the lungs reveals profound rales located throughout each lung. The patient's temperature is 99°F and there is obvious jugular vein distention and pedal edema. Additionally, family states that the patient has an extensive cardiac and diabetic history. Based on this information, you should suspect which kind of shock? A) Cardiogenic B) Hypovolemic C) Hypotensive D) Obstructive Answer: A Diff: 3 Page Ref: 441 Objective: 15-3 68) As you approach a seemingly unresponsive patient, you observe a 1- to 2-inch circle of dark blood on his shirt. He also appears pale and diaphoretic. What should you do first? A) Apply oxygen B) Cut his shirt C) Assess his airway D) Treat for shock Answer: C Diff: 2 Page Ref: 451 Objective: 15-9

18 Copyright © 2018 Pearson Education, Inc.

69) You believe that a young male patient, who has been shot in the lower abdomen, is bleeding internally and is in the early stage of shock. Which of these descriptions includes appropriate prehospital care of this patient? A) Semi-Fowler's position and direct pressure over the injury site B) "Shock" position and administration of water by mouth C) Oxygen therapy and rapid transport to the hospital D) Oxygen therapy and warm packs to the abdomen Answer: C Diff: 2 Page Ref: 451 Objective: 15-9 70) When a person is in shock, what is occurring in the body? A) The cells are getting glucose and other nutrients, but not oxygen B) The amount of oxygen to the cells is adequate, but CO2 is not being eliminated C) The blood carries an adequate amount of oxygen, but not enough nutrients for cell survival D) The cells are not getting enough oxygen, and waste products are accumulating Answer: D Diff: 3 Page Ref: 436-437 Objective: 15-2 71) When performing the primary assessment, which sign or symptom best indicates that the patient is in hypovolemic shock? A) Cool and diaphoretic skin B) Radial pulse of 72 beats/min C) Crackles heard in both lungs D) Constricted pupils Answer: A Diff: 2 Page Ref: 450 Objective: 15-7 72) A patient fell 20 feet from a ladder, and is now confused and anxious. Which of these signs/symptoms suggests that the patient is in shock? A) Heart rate of 110 beats/min B) Deformity to the left arm C) Contusion to his head D) Constricted pupils Answer: A Diff: 2 Page Ref: 449 Objective: 15-6

19 Copyright © 2018 Pearson Education, Inc.

73) A patient has fallen down a flight of stairs, and is now restless and confused. His airway is open, and he is adequately breathing, 22 times per minute, with a pulse oximetry reading of 90%. He has a radial pulse of 92 beats/min, which is moderate in strength. Emergency Medical Responders are maintaining manual spine motion restriction. What should your next action be? A) Check the blood pressure and assess for injuries causing blood loss B) Administer supplemental oxygen C) Insert an oral airway and start positive pressure ventilation D) Complete spinal motion restriction interventions by placing the patient on a long spine board with a cervical collar applied, and then applying straps Answer: B Diff: 2 Page Ref: 451 Objective: 15-9 74) You suspect that a patient involved in a motor vehicle collision is in shock. Which sign or symptom of shock would you expect to see last? A) Tachycardia B) Increased respirations C) Decreased blood pressure D) Pale and diaphoretic skin Answer: C Diff: 2 Page Ref: 450, Table 15-5 Objective: 15-6 75) Which of these statements made by your EMT partner best indicates an understanding of the prehospital role in caring for the patient in shock? A) "The job of the EMT is to recognize that a person is in shock and get him or her to the hospital so treatment can be started." B) "Because shock is a life-threatening condition, it is important that the EMT identify the exact cause so the proper care can be given." C) "If shock is in the compensatory or early stage, it is not yet life-threatening and the EMT can take his time in assessing and treating the patient." D) "Since shock is best treated in the hospital, the EMT should provide care to maintain perfusion to the vital organs and transport the patient." Answer: D Diff: 3 Page Ref: 451-452 Objective: 15-9

20 Copyright © 2018 Pearson Education, Inc.

76) A driver was ejected from his vehicle in a rollover-type collision. Assessment findings reveal the patient to be unresponsive, with bruising to the abdominal and pelvic areas as well as an open femur fracture. The patient has an open airway and is breathing 32 times per minute. When you listen to breath sounds, you find them to be absent over the alveolar areas of the lungs. The patient's skin is cool and clammy and the radial pulses weak. Manual spine motion restriction is being maintained. What should your next intervention be? A) Obtain a blood pressure B) Assist respirations C) Examine the fracture site D) Apply a cervical collar Answer: B Diff: 2 Page Ref: 451 Objective: 15-9 77) A 28-year-old male patient was cutting limbs from a tree when he lost his footing and fell approximately 20 feet. He is unresponsive and has shallow breathing, with a rate of 28 breaths/min. His radial pulse is weak and thready, and his skin is cool to the touch. Emergency Medical Responders (EMRs) have placed him on a nonrebreather face mask and are holding manual spine motion restriction. His respirations are sonorous. Based on these assessment findings, which of these instructions would you next provide to the EMRs? A) "The patient has snoring respirations, so let's go ahead and open the airway with the head-tilt, chin-lift maneuver." B) "Do not cover the patient with a blanket, because that will cause his blood vessels to dilate and drop his BP." C) "Let's elevate the patient's legs 8 to 12 inches so more blood gets to his vital organs." D) "Let's take off the oxygen mask and try manually opening the airway." Answer: D Diff: 2 Page Ref: 451 Objective: 15-9 78) While cleaning a gun, a 44-year-old patient accidentally shot himself in the abdomen. On arrival, you observe the patient on the floor lying on his side, with his legs drawn to his chest. His blood pressure and pulse oximetry reading are low, and his heart rate and respirations are high. Blood is evident on his shirt and pants. Which of these indicates the correct sequence of events when caring for this patient? A) Perform primary and secondary assessments, move the patient to the stretcher for immediate transport, start positive pressure ventilation en route to the hospital B) Perform the primary assessment, administer supplemental oxygen, perform a rapid secondary assessment, transfer the patient to the stretcher, provide rapid transport C) Transfer the patient to the ambulance, provide rapid transport, perform the primary assessment and oxygen therapy en route to the hospital D) Transfer the patient to the ambulance, perform the primary assessment and rapid secondary assessment, provide oxygen therapy, provide rapid transport Answer: B Diff: 2 Page Ref: 451 Objective: 15-9 21 Copyright © 2018 Pearson Education, Inc.

79) A 66-year-old female patient has been struck by a car. Your assessment reveals gurgling respirations, rapid breathing, and cool, diaphoretic skin. You also note bruising to her chest and abdomen. What should you do immediately? A) Determine the blood pressure B) Administer supplemental oxygen C) Evaluate for shock D) Suction the airway Answer: D Diff: 2 Page Ref: 451 Objective: 15-9 80) Which of these remarks made by the patient best reinforces your suspicion that he is in the early stages of hypovolemic shock? A) "I have had diarrhea for the past four days." B) "I fell last night and think I hurt my belly; see the bruise?" C) "I must have fallen and hit my head. I am very confused and restless." D) "I have been coughing up green mucus and feel weak." Answer: A Diff: 2 Page Ref: 438 Objective: 15-6 81) A patient has been involved in a very serious motor vehicle collision and is in shock. Assessment findings indicate that he sustained blunt trauma to the abdominal and pelvic areas. Which type of shock is the patient most likely suffering? A) Obstructive B) Cardiogenic C) Distributive D) Hypovolemic Answer: D Diff: 2 Page Ref: 440-441 Objective: 15-3 82) You have been called to transport a patient in septic shock from the emergency department of a local hospital to the critical care unit of another hospital. Two nurses will be accompanying you. As a knowledgeable EMT, you recognize that this state of shock has been caused by: A) Failing heart B) Blood loss C) Infection D) Fluid volume loss Answer: C Diff: 2 Page Ref: 444 Objective: 15-4

22 Copyright © 2018 Pearson Education, Inc.

83) A 44-year-old male patient has been shot in the abdomen. Which assessment findings would lead you to believe that the patient is in compensated shock? A) Alert and anxious, pulse 102, BP 114/88 mmHg, pale and cool skin B) Slightly confused, pulse 116, BP 102/56 mmHg, warm skin that is flushed C) Confused and anxious, pulse 144, BP 82/palpation, cool skin that is mottled D) Confused, pulse 44, BP 110/68 mmHg, cool and cyanotic skin Answer: A Diff: 3 Page Ref: 448 Objective: 15-6 84) You are reassessing a young female who sustained blunt trauma to the chest in a motor vehicle collision. Which of these assessment findings best indicates that she is deteriorating and in the decompensatory phase of shock? A) Heart rate of 100 beats/min B) Blood pressure of 88/50 mmHg C) Blood continuing to ooze from an abdominal laceration D) Restless and confused mental status Answer: B Diff: 3 Page Ref: 448 Objective: 15-7 85) At a continuing education seminar on shock, the presenter asks if there is a point where someone cannot recover from shock. Which statement made by an EMT is correct? A) "Even with treatment, if shock has reached the final stage when multiple organs start to fail, death will result." B) "If the pupils are dilated and pulse is rapid, the patient cannot recover." C) "The patient probably cannot recover if the radial pulse is weak, even if the carotid pulse remains strong." D) "To survive severe shock, the patient needs a large amount of IV fluids immediately or a blood transfusion at the hospital." Answer: A Diff: 3 Page Ref: 448-449 Objective: 15-7 86) When comparing the pediatric Chain of Survival to the adult Chain of Survival, the EMT would note that the pediatric version differs in which way? A) Emphasis on ventilations over compressions B) Less attention to post-resuscitation care C) Prevention of cardiac arrest D) Emphasis on ALS over BLS Answer: C Diff: 2 Page Ref: 454 Objective: 15-8

23 Copyright © 2018 Pearson Education, Inc.

87) Which of these is most appropriate when two EMTs are performing CPR on an adult patient? A) At least 100 compressions per minute B) Airway reassessment after every 30 compressions C) Compression to ventilation rate of 15:2 D) Delivery of ventilations while compressions are being provided Answer: A Diff: 2 Page Ref: 454 Objective: 15-14 88) Proper care of cardiac arrest for a 6-year-old pediatric patient when there are no available pediatric AED pads would include: A) Two EMTs providing a 15:1 ratio of compressions to ventilations B) CPR with no AED placement due to lack of pediatric pads C) Compressing the chest to a maximum depth of 1½ inches D) Placement and use of the adult pads with adult energy levels Answer: D Diff: 2 Page Ref: 459 Objective: 15-17 89) You are by the side of an unresponsive 6-month-old child with a history of congenital heart disease. The patient's airway is patent, but he is not breathing. A heart rate of 24 beats/min is noted. At this time, it is essential that you: A) Apply the AED with pediatric pads B) Start cardiopulmonary resuscitation with compressions and ventilations C) Start positive pressure ventilation with high-concentration oxygen D) Place the patient on the stretcher for immediate and emergent transport Answer: B Diff: 3 Page Ref: 459 Objective: 15-17 90) You are dispatched to a residence for an 82-year-old female who is not breathing. On arrival, the husband directs you to a second-floor bedroom. As you make patient contact, you note that she is rigored. What should your next step be? A) Start CPR B) Withhold CPR and contact medical command C) Start positive pressure ventilation with high-concentration oxygen D) Place the patient on the stretcher for immediate and emergent transport Answer: B Diff: 3 Page Ref: 453 Objective: 15-11

24 Copyright © 2018 Pearson Education, Inc.

91) You are preparing to attach an AED when you notice that the patient has an implanted pacemaker. What should you do? A) Apply the AED, but avoid placing the electrodes over the pacemaker B) Call for ALS backup with a manual defibrillator C) Forego the AED and transport the patient immediately while continuing CPR D) Call medical command and request permission to perform a precordial thump Answer: A Diff: 3 Page Ref: 466 Objective: 15-22 92) Mechanical external compression devices are recommended: A) By AHA 2015 guidelines as a preferred substitute for manual CPR B) Because patient ventilation is not required C) When high-quality manual compressions are not possible D) For short-term use only Answer: C Diff: 2 Page Ref: 467 Objective: 15-23 93) When the body is in a shock state, which hormone released from the adrenal glands stimulates primarily alpha-1 and beta-1 receptors, but has no effects on beta-2 receptor sites? A) Biepinephrine B) Pseudoepinephrine C) Coepinephrine D) Norepinephrine Answer: D Diff: 2 Page Ref: 446 Objective: 15-5 94) You arrive on the scene of an "unknown medical emergency." The local fire department EMRs arrive simultaneously, so you now have five providers at the scene to help. The on-scene police officer tells you the patient is an elderly man who has attempted suicide by shooting himself in the head. As you enter the patient's room, you see the male patient with an extensive skull injury. A large portion of his right temporal and parietal skull is gone, and there are blood splatter and brain tissue on the wall behind him. Given this information, what should you do? A) Withhold resuscitation B) Start CPR, since you have ample help to treat and transport the patient C) Do nothing until you get advice from online medical direction D) Start BLS care, but withhold ALS care Answer: A Diff: 2 Page Ref: 453 Objective: 15-11

25 Copyright © 2018 Pearson Education, Inc.

95) You are treating a 4-year-old child who sustained trauma after being ejected from a motor vehicle rollover. Your assessment has revealed a blood pressure of 72/52 mmHg, a heart rate of 118 beats/min, and respirations of 28 breaths/min and unlabored. The pulse oximetry reading on ambient air is 92%. How would you describe these findings? A) Acceptable vital signs for this age bracket B) Elevation of all vital signs C) Shock findings for this age D) Normal vital signs with an abnormally low pulse oximeter reading Answer: C Diff: 3 Page Ref: 451 Objective: 15-8 96) What is the primary difference between the 2015 American Heart Association's "Chain of Survival" for the adult patient versus the pediatric patient? A) The adult chain allows the use of the AED, whereas use of this device is discouraged in the pediatric chain B) The pediatric chain emphasizes strategies for arrest prevention C) The pediatric chain emphasizes ALS treatment as the first link D) The adult chain includes a 30:2 compression/ventilation ratio, whereas the pediatric chain uses a 15:1 compression/ventilation ratio Answer: B Diff: 3 Page Ref: 453-454 Objective: 15-12 97) You are treating a pediatric patient in cardiac arrest. The patient has a long congenital cardiac history and had a pacemaker implanted a year ago. How will this information change the EMT's use of the AED during arrest management? A) In a pediatric patient, the use of an AED is contraindicated if the patient has an implanted pacemaker B) The AED can be used, with the adhesive pads being placed over, or beside, the implanted pacemaker C) The AED can be used, but only half of the normal pediatric energy level should be used during defibrillatory shocks D) The AED can be used, but the adhesive pad should not be placed directly over the pacemaker Answer: D Diff: 3 Page Ref: 466 Objective: 15-22 98) One of the benefits of utilizing an automated chest compression device is that: A) It frees up EMS providers to tend to other patient care tasks B) It provides a similar cardiac output as a spontaneously beating heart does C) It avoids the need to simultaneously ventilate the patient D) The survival rate when using automated chest compression devices is almost 90% Answer: A Diff: 2 Page Ref: 467 Objective: 15-23 26 Copyright © 2018 Pearson Education, Inc.

Prehospital Emergency Care, 11e (Mistovich et al.) Chapter 16 Respiratory Emergencies 1) When arrive on scene, a family member directs you to a bedroom where a 62-year-old male patient is in respiratory distress. During the scene size-up, which finding would most strongly suggest that the patient suffers from a chronic respiratory disease? A) Furnace turned on high with the humidifier running B) Bottles of aspirin and nitroglycerin on the nightstand C) Patient in a chair in tripod position and has a barrel-shaped chest D) Smell of cigarettes in house and the patient is obese Answer: C Diff: 2 Page Ref: 508 Objective: 16-18 2) What is the most common cause of cardiac arrest in infants and children? A) Heart birth defect B) Accidental overdose C) Respiratory compromise D) Abuse and neglect Answer: C Diff: 1 Page Ref: 497 Objective: 16-17 3) When assessing a 3-year-old male patient in respiratory distress, the EMT should recognize "see-saw" breathing when he observes: A) Alternate breathing through the nose and mouth B) Movement of the chest and abdomen in opposite directions C) Retraction of the muscles between the ribs D) Up-and-down motion of the head as the patient breathes Answer: B Diff: 2 Page Ref: 498 Objective: 16-17 4) When a child starts to become hypoxic, the pulse will initially: A) Decrease B) Not change C) Become irregular D) Increase Answer: D Diff: 2 Page Ref: 497 Objective: 16-17

1 Copyright © 2018 Pearson Education, Inc.

5) The mother of a 5-year-old female patient states that her daughter has been "sick" with a sore throat and fever for several days. The mother became concerned tonight and called 911 because the child was "making funny noises" and could not breathe well. Your assessment reveals a lethargic, well-developed child sitting upright and with high-pitched tracheal noises when she breathes in. What should be the priority action for the EMT? A) Placing an oropharyngeal airway B) Administering supplemental oxygen if indicated C) Inspecting the airway with a tongue depressor D) Performing abdominal thrusts Answer: B Diff: 2 Page Ref: 498 Objective: 16-17 6) An alert and oriented 45-year-old female patient complains of shortness of breath. Although her respiratory volume and rate are satisfactory, you note cyanosis in her fingertips and you cannot get the pulse oximeter to show a reading. What should be your next action? A) Administer supplemental oxygen B) Assist respirations with the bag-valve mask C) Place a nasopharyngeal airway and assist respirations D) Move the patient to the stretcher for immediate and rapid transport Answer: A Diff: 2 Page Ref: 509 Objective: 16-18 7) A 67-year-old male patient complains of shortness of breath. The patient cannot remember how many times he used his inhaler prior to your arrival, but he knows it has been multiple times. He is tachypneic and tachycardic. After administering oxygen, what should be your next action? A) Call medical direction for advice B) Assist the patient in the self-administration of his inhaler C) Transport the patient to the hospital D) Auscultate the patient's lung sounds and administer the inhaler if you hear wheezing Answer: A Diff: 2 Page Ref: 503 Objective: 16-15 8) A confused, lethargic, and nonverbal 50-year-old female patient has altered mental status. You are told she has a history of chronic obstructive pulmonary disease (COPD). The primary assessment reveals a respiration rate of 6 breaths/min and cool, clammy skin with cyanosis around the lips. What should be your priority management? A) Administer oxygen by nasal cannula at 15 lpm B) Assist respirations with positive pressure ventilation C) Administer oxygen by nonrebreather mask at 15 lpm D) Sit the patient up, assess lung sounds, and obtain a blood pressure Answer: B Diff: 2 Page Ref: 480-481 Objective: 16-18 2 Copyright © 2018 Pearson Education, Inc.

9) What is the typical resting respiratory rate for an elderly patient? A) 10 breaths/min B) 15 breaths/min C) 20 breaths/min D) 30 breaths/min Answer: C Diff: 1 Page Ref: 502 Objective: 16-17 10) As compared to an adult, how would you describe the respiratory rate of an infant? A) Faster than an adult's rate B) Slower than an adult's rate C) Same as an adult's rate D) Normally slower than an adult's rate, unless dyspnea is present Answer: A Diff: 1 Page Ref: 502 Objective: 16-17 11) Which of these is considered an early sign of inadequate breathing in children? A) Blue-gray skin B) Barrel chest C) Nasal flaring D) Crying Answer: C Diff: 2 Page Ref: 497 Objective: 16-17 12) Within minutes of administering Proventil (albuterol) to a patient through her metered-dose inhaler (MDI), the patient is breathing easier and states that she feels much better. An Emergency Medical Responder on scene asks you how the medication in the MDI helped the patient. Which of these responses would be most appropriate? A) "The medication travels to the brain and forces the respiratory rate to increase." B) "The medication in the MDI relaxes the throat, allowing more oxygen to enter." C) "The medication is an anti-inflammatory and decreases swelling in the lung tissue." D) "The medication in the inhaler relaxes and opens up the small airways in the lungs." Answer: D Diff: 2 Page Ref: 493 Objective: 16-15

3 Copyright © 2018 Pearson Education, Inc.

13) While you are obtaining a medical history for a patient with generalized weakness and fatigue, she hands you a list of all the medications she is currently taking. Which of these medications should the EMT recognize as a bronchodilator for which the EMT could assist in its administration? A) Metaproterenol B) Mucomyst C) Prednisone D) Metformin Answer: A Diff: 1 Page Ref: 494 Objective: 16-16 14) The medication contained within a metered-dose inhaler is in which form? A) Aerosolized B) Compressed tablet C) Pressurized liquid D) Viscous gel Answer: A Diff: 1 Page Ref: 494 Objective: 16-14 15) Medical direction has ordered you to assist a patient in taking his metered-dose inhaler (MDI). After placing the patient's lips on the mouthpiece, which instruction would be most appropriate? A) "Please hold your breath and I will administer the medication." B) "Please inhale, then hold your breath, and I will give you the medication." C) "Please inhale slowly and I will administer the medication, then hold your breath." D) "Please exhale, hold your breath, and I will administer the medication." Answer: C Diff: 2 Page Ref: 493 Objective: 16-15 16) You are assisting a patient in taking her metered-dose inhaler for the first time. Which of these statements about the side effects of the medication would be most appropriate? A) "You will probably not even remember taking the medication." B) "You may feel anxious and your heart rate may increase." C) "You may experience a runny nose and increase in saliva." D) "The medication may cause you to become very tired and fatigued." Answer: B Diff: 2 Page Ref: 493, Table 16-6 Objective: 16-15

4 Copyright © 2018 Pearson Education, Inc.

17) After you administer a medication through a metered-dose inhaler (MDI) and the patient has inhaled the medication, it is important that the patient: A) Exhale through pursed lips B) Pant for 10 seconds C) Exhale as forcefully as possible D) Hold her breath for several seconds Answer: D Diff: 2 Page Ref: 493 Objective: 16-15 18) A patient requires several doses of medication from her metered-dose inhaler. At a minimum, how long should the EMT wait between administrations? A) 30 seconds B) 2 minutes C) 5 minutes D) 10 minutes Answer: B Diff: 2 Page Ref: 495 Objective: 16-15 19) At a health fair, a patient informs you that she has just been prescribed a metered-dose inhaler (MDI) for her asthma. She is confused about how to take the medication when needed. Which of these statements about the MDI would be most appropriate for you to make? A) "Take it every 4 hours around the clock, even if you feel fine." B) "Make sure to store the metered-dose inhaler in your refrigerator." C) "You should take the medication only after calling your doctor." D) "It is important to shake the inhaler vigorously before taking the medication." Answer: D Diff: 2 Page Ref: 494 Objective: 16-15 20) You are administering a medication through a metered-dose inhaler (MDI) using a spacer. Which of these actions would be most appropriate? A) Administer the medication into the spacer and then have the patient inhale B) Instruct the patient to inhale slowly just prior to the medication being placed in the spacer C) Administer the medication into the spacer and then have the patient exhale D) Instruct the patient to exhale slowly as you place the medication into the spacer Answer: A Diff: 2 Page Ref: 495 Objective: 16-15

5 Copyright © 2018 Pearson Education, Inc.

21) Your medical director's written protocol instructs you to obtain on-line medical direction prior to assisting a patient with a metered-dose inhaler (MDI). Given this protocol, you must contact: A) The patient's family doctor before assisting with the MDI B) The medical director after assisting with the MDI C) An emergency physician before assisting with the MDI D) No one; physician direction is not required when assisting with the MDI Answer: C Diff: 1 Page Ref: 494 Objective: 16-15 22) You have been called for a patient who complains of shortness of breath. On scene, you find the 41-year-old woman sitting upright in bed with nasal flaring and wheezing. A metered-dose inhaler (MDI) is lying on the nightstand next to her. To assist her with the MDI, which criteria must be met? A) The MDI must be prescribed for the patient B) The pulse oximeter reading must be less than 90% C) High-concentration oxygen must be administered first for 5 minutes D) The patient must display altered mental status Answer: A Diff: 1 Page Ref: 494 Objective: 16-15 23) You are called to a motor vehicle collision. A 26-year-old female states that she was unrestrained and struck the steering column with her chest after the car she was driving hit another car from behind. She is somewhat confused, and complains of chest pain and difficult and painful breathing when she inhales. Her airway is patent and her breathing is adequate. Assessment of the radial pulse reveals it to be rapid and very irregular and weak. The patient has cyanosis in her nose and fingertips. Her breath sounds are clear and equal, and her skin is moist and pale. The patient informs you that she has asthma and has a metered-dose inhaler (MDI). What is your best initial action in treating this patient? A) Assist the patient with her MDI B) Apply supplemental oxygen if indicated C) Apply CPAP and transport the patient expediently D) Transport the patient in a supine position on a long spine board Answer: B Diff: 2 Page Ref: 477 Objective: 16-18

6 Copyright © 2018 Pearson Education, Inc.

24) Auscultation of breath sounds in a patient complaining of shortness of breath reveals wheezing. Which condition is responsible for this finding? A) Bronchiole constriction B) Significant hypoxia C) Swelling in the throat D) Mucus in the lungs Answer: A Diff: 2 Page Ref: 507 Objective: 16-1 25) A 4-year-old female patient is having great difficulty breathing. She is responsive to verbal stimuli and has an open airway. Her respiratory rate is 40 breaths/min, and she has an SpO2 reading of 88%, which has dropped from 90% despite high-concentration oxygen through a pediatric nonrebreather mask. Her pulse is rapid, and her skin cool to the touch. As a knowledgeable EMT, you would recognize these findings as associated with which condition? A) Respiratory distress B) Cardiopulmonary arrest C) Respiratory arrest D) Respiratory failure Answer: D Diff: 3 Page Ref: 497-498 Objective: 16-6 26) You have been called for a 2-year-old male patient who is sick and having a difficult time breathing. On scene, you find the patient in his mother's arms. He appears to have labored breathing and skin that is warm to the touch. Which of these additional assessment findings would you recognize as the most serious? A) Continual crying accompanied by a fever of 99.4°F B) Respiratory rate of 28 breaths/min C) Grunting noise heard as the child exhales D) Slight abdominal movement with breathing Answer: C Diff: 2 Page Ref: 497-498 Objective: 16-6 27) A 6-year-old female patient is in acute respiratory distress and is still breathing adequately but will not keep the nonrebreather face mask on her face. In this situation, the EMT should: A) Secure the mask with tape to the patient's face B) Have the mother hold the mask near her daughter's face C) Start positive pressure ventilation D) Immediately transport the patient without oxygen therapy Answer: B Diff: 2 Page Ref: 498 Objective: 16-17

7 Copyright © 2018 Pearson Education, Inc.

28) On scene, you find a 2-year-old male patient in significant respiratory distress. He is responsive to painful stimuli and exhibits noisy respirations through an open airway. His breathing is shallow at a rate of 44 breaths/min. His pulse is rapid. Assessment of the skin shows it to be cool and diaphoretic, with cyanosis to the extremities and around the mouth. What is the immediate priority for this patient? A) Start positive pressure ventilation B) Insert an oropharyngeal airway C) Apply oxygen through a pediatric nonrebreather mask D) Move the patient to the ambulance for immediate transport Answer: A Diff: 2 Page Ref: 498 Objective: 16-6 29) You have been dispatched for a young child with difficulty breathing. As you enter the apartment, which scene size-up clue would best indicate that the patient is experiencing a problem in her upper airway? A) Grunting with exhalation B) Stridorous noises with inspiration C) Low-pitched sobbing and crying D) Audible wheezing on expiration Answer: B Diff: 3 Page Ref: 498 Objective: 16-4 30) A 4-year-old boy in respiratory distress is sitting upright in bed and drooling. His mother states that he complained of a sore throat throughout the day, and tonight became severely short of breath and started drooling. He is alert with adequate breathing. His skin is very warm to the touch. Vital signs are pulse, 132; respirations, 28 breaths/min; and SpO2, 90% on room air. Which is your first action in caring for this child? A) Insert an oral airway and start ventilations B) Suction the airway of secretions C) Apply supplemental oxygen D) Use a tongue depressor to inspect for a foreign body obstruction Answer: C Diff: 2 Page Ref: 488 Objective: 16-14

8 Copyright © 2018 Pearson Education, Inc.

31) Following a call in which you treated a pediatric patient with croup, a new EMT who is in orientation asks you how you can differentiate croup from epiglottitis. Which of these responses would be most appropriate? A) "The swelling associated with epiglottitis typically decreases with exposure to cool air; the swelling with croup does not." B) "Epiglottitis is accompanied by a cough; croup is not associated with a cough." C) "Croup is typically associated with wheezing; epiglottitis is typically associated with crackles in the lungs." D) "The child with epiglottitis is typically drooling; the child with croup rarely, if ever, drools." Answer: D Diff: 3 Page Ref: 489 Objective: 16-14 32) A 911 call was placed for a 3-year-old child with difficulty breathing. On scene, you note that the child is upright with slight stridor upon inspiration. He is alert and breathing at a rate of 30 breaths/min. His skin is warm and dry, and his radial pulse strong and regular. His breath sounds are clear and equal. His mother states that he was fine all day until 15 minutes ago, when he was playing with friends in the playroom and suddenly started having difficulty breathing. As an EMT, you should suspect which condition? A) Foreign body airway obstruction B) Epiglottitis C) Croup D) Bronchial asthma Answer: A Diff: 2 Page Ref: 498 Objective: 16-4 33) A 48-year-old male patient is short of breath and confused. His airway is open, and his breathing is fast. A radial pulse is easily palpated. His skin is warm to cool and dry. Your partner reports the following vital signs: pulse, 124; respirations, 24 breaths/min; blood pressure, 158/86 mmHg; and SpO2, 89% on room air. The patient's lung sounds indicate slight wheezing. Based on this information, which of these conditions poses the most immediate threat to this patient's well-being? A) Hypoxia B) Elevated blood pressure C) Possible infection D) Increased heart rate Answer: A Diff: 2 Page Ref: 475 Objective: 16-1

9 Copyright © 2018 Pearson Education, Inc.

34) A 36-year-old patient has overdosed on an unknown drug and is unresponsive. His breathing is shallow, with a rate of 6 breaths/min. He has cyanosis around his lips and to his fingertips. His skin is cool to the touch, and his radial pulse is rapid. His breath sounds are diminished but clear. The EMT should recognize which respiratory condition from this assessment? A) Hypoxia B) Tachypnea C) Bronchospasm D) Apnea Answer: A Diff: 1 Page Ref: 475-476 Objective: 16-18 35) A son has called 911 for his father, who is having difficulty breathing. On scene, the 81-yearold patient is lying on a couch in the living room, lethargic, and in obvious respiratory distress. He is confused, but has an open airway. His respirations are 38 breaths/min and shallow, and his skin is extremely diaphoretic with cyanosis in the fingertips. At this time, the EMT should: A) Suction the airway of secretions B) Start positive pressure ventilation C) Assess the patient's breath sounds D) Determine the patient's past medical history Answer: B Diff: 2 Page Ref: 477 Objective: 16-18 36) When assessing a patient who complains of shortness of breath but has an open airway, which action should the EMT perform next? A) Ascertain whether the patient has used an MDI B) Assess the adequacy of the patient's breathing C) Get the patient's SpO2 to 100% D) Determine the cause of the shortness of breath Answer: B Diff: 1 Page Ref: 476-477 Objective: 16-6 37) You are treating a patient who states that he has been short of breath for the past month. Tonight, his breathing just "got too bad" and he decided to call 911. The 67-year-old male patient is alert and oriented and has a patent airway. He appears to have an adequate tidal volume. His pulse is strong and regular, and his skin is warm. Vital signs are pulse, 92; respirations, 22 breaths/min; blood pressure, 128/66 mmHg; and SpO2, 93% while on home oxygen at 2 lpm. Which instruction would you give to your partner regarding this patient's care? A) "Let's try 4 lpm through a nasal cannula." B) "The patient seems pretty stable; we can refer him to his family doctor." C) "Why don't we see how the patient tolerates a nasal airway?" D) "Let's go ahead and put the patient on 15 lpm of oxygen." Answer: A Diff: 2 Page Ref: 477 Objective: 16-6 10 Copyright © 2018 Pearson Education, Inc.

38) A confused and cyanotic patient is breathing 8 times per minute with poor chest rise and fall. Which instruction would you give your partner about the respiratory care of this patient? A) "Deliver a ventilation with the bag-valve mask only when he breathes." B) "Ventilate him at 10 times per minute." C) "Ventilate him with the bag-valve mask in between each of his breaths." D) "Since he is breathing, let's go ahead and put him on a nonrebreather mask with 15 lpm of oxygen." Answer: B Diff: 2 Page Ref: 511, Figure 16-17B Objective: 16-6 39) As you enter the living room of an adult patient who called 911 for shortness of breath, you observe the patient sitting upright in a chair with a panicked look on her face, struggling to breathe, with obvious suprasternal retractions. Her breathing rate appears to be fast and her tidal volume poor. You can hear rhonchi from her lungs without the aid of a stethoscope. Once at her side, your initial care should be to: A) Apply oxygen immediately B) Auscultate lung sounds C) Establish her rate of breathing D) Assess her airway Answer: D Diff: 2 Page Ref: 502 Objective: 16-18 40) When assessing an adult patient with difficulty breathing, which of these signs or symptoms would be of most immediate concern? A) Temperature of 101.3°F B) Blue coloration to the lips C) Rate between 20 and 24 breaths/min D) Cyanosis to the fingernails Answer: B Diff: 2 Page Ref: 503 Objective: 16-18 41) You are having a hard time getting the past medical history of a patient who is short of breath. Which piece of information would lead the EMT to believe that the patient has a history of chronic lung disease, such as emphysema or chronic bronchitis? A) She takes a daily aspirin B) Her chest is barrel shaped C) Her respirations are tachypneic D) She continually coughs Answer: B Diff: 2 Page Ref: 504, Table 16-8 Objective: 16-8

11 Copyright © 2018 Pearson Education, Inc.

42) You have been called to a residence for a patient with altered mental status and shortness of breath. On scene, an Emergency Medical Responder meets you and states that he believes the patient had a stroke and is unresponsive. As you enter the room, you see the patient lying in bed with snoring respirations. Oxygen at 15 liters per minute has been applied. Your immediate action when you reach the patient's side would be to: A) Place a nasal airway and suction B) Start positive pressure ventilation C) Perform the head-tilt, chin-lift maneuver D) Determine whether the patient has a pulse Answer: C Diff: 2 Page Ref: 512, Figure 16-18 Objective: 16-7 43) You realize the EMT is properly ventilating an unresponsive adult male patient when: A) He ventilates smoothly at a rate of 24 breaths/min B) He flexes the patient's head as ventilations are administered C) He allows 2 seconds between each ventilation D) You observe the pulse oximeter trending upward Answer: D Diff: 1 Page Ref: 507 Objective: 16-18 44) A 71-year-old female patient complains of the sudden onset of shortness of breath and rapid breathing. She also states that she is experiencing some chest pain that seems to worsen when she takes a deep breath. The primary assessment shows her to be alert and oriented with a patent airway and rapid breathing that is adequate. Her pulse rate is 92 and her blood pressure is 116/68 mmHg. When obtaining a history, which of these statements made by the patient should raise your suspicion that the patient may be experiencing a pulmonary embolism? A) "I think that I may also have a fever." B) "I had leg surgery five days ago." C) "I just started a new blood pressure medication today." D) "I had a headache earlier and took an aspirin." Answer: B Diff: 3 Page Ref: 484-485 Objective: 16-10 45) A patient who is nonverbal from a previous stroke is in severe respiratory distress. A family member states that she has multiple medical problems, including high blood pressure, diabetes, and heart failure. When assessing this patient, which sign or symptom would raise your suspicion that the patient has heart failure? A) Pulmonary crackles B) Diaphoresis C) Edema to left foot D) Pursed lip breathing Answer: A Diff: 2 Page Ref: 504, Table 16-8 Objective: 16-11 12 Copyright © 2018 Pearson Education, Inc.

46) You have been called to a home for an 18-year-old male patient, who informs you that he experienced a sudden onset of shortness of breath and back pain while watching television. He has a history of spontaneous pneumothorax, and the current symptoms he is experiencing are identical to those he felt with a previous pneumothorax. Assessment reveals the patient to be slightly dyspneic with breath sounds clear and intact bilaterally. During transport, what is most critical to continually monitor on this patient? A) Blood pressure B) Mental status C) Spasms to the hands D) Breath sounds Answer: D Diff: 2 Page Ref: 487 Objective: 16-12 47) You have arrived on scene to find a 32-year-old male patient who complains of rapid breathing, light-headedness, tachycardia, and numbness and tingling to his lips and hands. Family on scene report that he had just received bad advice from his attorney about a pending divorce when the symptoms started. The patient denies chest pain as well as any other past medical history. His pulse is 124, respirations are 52 breaths/min, and his blood pressure is 158/68 mmHg. What is your first course of action in treating this patient? A) Apply a nonrebreather face mask with low-flow oxygen, allowing the patient to rebreathe his exhaled carbon dioxide B) Monitor for spasms of the hand muscles and administer nitroglycerin C) Instruct the patient to close his mouth and breathe through his nose D) Have the patient breathe into a paper bag for approximately 5 minutes Answer: C Diff: 2 Page Ref: 487 Objective: 16-13 48) When auscultating breath sounds, the EMT should recognize rhonchi when he hears which type of sound? A) Rattling-like noises that change when the patient coughs B) Crowing-like sounds that are accompanied by a high-pitched cough C) High-pitched whistling-like noises heard more on expiration D) Crackling-like noises heard in the upper portion of the lung Answer: A Diff: 1 Page Ref: 475 Objective: 16-4

13 Copyright © 2018 Pearson Education, Inc.

49) You are by the side of a 77-year-old male patient with a complaint of shortness of breath. Auscultation of his lungs indicates crackles in the bases. In addition, both of his feet are swollen. His SpO2 on room air is 82%. The remainder of his vital signs are pulse, 144; respirations, 26 breaths/min; and blood pressure 168/90, mmHg. The patient reports lower back pain with a history of a "pinched nerve" in that area. He states that he always has this pain. He has difficulty finishing his sentences without gasping for air. What is the most appropriate care for this patient after initiating oxygen? A) Apply the AED in case he arrests en route to the hospital B) Initiate a bronchodilator through a small-volume nebulizer C) Transport him in a sitting position D) Provide spinal motion precautions and secure him to a long spine board for back pain Answer: C Diff: 2 Page Ref: 486 Objective: 16-11 50) The EMT is properly using a small-volume nebulizer when she: A) Places a spacer on the nebulizer B) Attaches the nebulizer to oxygen at 8 to 10 lpm C) Shakes the medication canister for 30 seconds prior to administration D) Instructs the patient to breathe deeply through his nose Answer: B Diff: 2 Page Ref: 495, Figure 16-11 Objective: 16-15 51) After administering a bronchodilator to a patient through a small-volume nebulizer, which of these statements made by the patient should be of most concern to an EMT? A) "I feel shaky." B) "My mouth is really dry." C) "I suddenly feel nervous." D) "My chest feels heavy." Answer: D Diff: 2 Page Ref: 496, Figure 16-11 Objective: 16-15 52) When instructing a patient on how to use a small-volume nebulizer, which of these statements would be correct? A) "Breathe about 20-30 times per minute so the medication gets into your lungs." B) "As I depress the trigger on the canister, take in a deep breath." C) "Take nice slow and deep breaths, and try not to cough the medicine out." D) "It is really important to occasionally cough during the treatment." Answer: D Diff: 2 Page Ref: 495, Figure 16-11 Objective: 16-15

14 Copyright © 2018 Pearson Education, Inc.

53) You have been called for a 3-year-old male patient who is "not breathing right." Aside from a notable expiratory wheeze, which of these findings would be a cause of great concern? A) Respiratory rate of 24 breaths/min B) Clinging to his mother and crying C) Prolonged and forced expiration D) Abdominal wall movement with inspiration Answer: C Diff: 2 Page Ref: 497 Objective: 16-17 54) A 4-year-old female patient with a decreased level of consciousness is in respiratory distress. Her airway is patent and respirations are 12 per minute. Breath sounds are diminished bilaterally, and there is noted accessory muscle use with inspiration and expiration. Her pulse is 84 and SpO2 88%. What should be your immediate action? A) Recheck the SpO2 on another finger B) Start positive pressure ventilation C) Apply CPAP D) Initiate immediate transport to the hospital Answer: B Diff: 2 Page Ref: 497-498 Objective: 16-6 55) The EMT is correctly assessing a patient for subcutaneous emphysema when he: A) Palpates the anterior and posterior chest B) Listens posteriorly for breath sounds C) Observes the chest for abnormalities D) Examines the neck veins for distention Answer: A Diff: 1 Page Ref: 506 Objective: 16-18 56) Which of these statements regarding ventilation in a healthy patient is true? A) Inspiration requires little energy; expiration requires much energy B) Both inspiration and expiration require the same amount of energy C) Inspiration is an active process that requires energy; expiration requires little to no energy D) Inspiration and expiration require no energy in a healthy person Answer: C Diff: 1 Page Ref: 506 Objective: 16-2

15 Copyright © 2018 Pearson Education, Inc.

57) You are administering positive pressure ventilation to an elderly male patient with a history of chronic obstructive pulmonary disease (COPD). Which point must you keep in mind as you care for this patient? A) Use of a CPAP may rupture the lung tissue and cause a pneumothorax B) High concentrations of oxygen will damage the lungs and must be avoided C) Ventilations must be given with greater force to overcome natural stiffening of the rib cage D) High-concentration oxygen is contraindicated for this patient Answer: A Diff: 2 Page Ref: 483 Objective: 16-8 58) When auscultating the lungs of an elderly patient, you note rhonchi-like noises in the upper regions of both lungs. As a knowledgeable EMT, you would recognize that this finding is caused by: A) Fluid accumulation in the alveoli B) Accumulation of mucus in the bronchi C) Narrowing of the larger airways D) Edema to the bronchioles Answer: B Diff: 1 Page Ref: 475 Objective: 16-4 59) When administering albuterol to a patient with shortness of breath, the EMT realizes that the therapeutic effect of this medication is achieved by: A) Decreasing mucus production B) Opening the large airways C) Decreasing inflammation D) Dilating the small airways Answer: D Diff: 2 Page Ref: 494-495, Figure 16-11 Objective: 16-16 60) As you enter a room for an unknown medical emergency, you find an unresponsive male with snoring respirations lying supine in bed. Which action should you take immediately? A) Take manual spinal motion restriction precautions B) Perform a head-tilt, chin-lift maneuver C) Start positive pressure ventilation D) Suction the airway of fluids Answer: B Diff: 2 Page Ref: 512, Figure 16-18 Objective: 16-4

16 Copyright © 2018 Pearson Education, Inc.

61) The EMT indicates that he is properly assessing the patient's breath sounds when he gives which of these instructions to the patient? A) "Please take a deep breath and gently cough as I listen to your lungs." B) "Please sit upright and take a deep breath through your nose as I listen to your lungs." C) "Please breathe deeply through your mouth as I listen to your lungs." D) "I need to listen to your right lung first and then compare it to your left." Answer: C Diff: 2 Page Ref: 506 Objective: 16-4 62) You arrive on scene for a 9-year-old female patient with a severe cough. Her mother states that she has spoken to her pediatrician, who believes that the child may have whooping cough and should be taken to the hospital for evaluation. Her mother is unsure if her daughter has had all her immunizations. Which of these statements would be most appropriate in regard to the care of this patient? A) "She may be contagious, so let's put a surgical mask on her." B) "Since this condition is caused by a virus, she is not contagious and we just need to transport her." C) "She is probably contagious, so we need to each put on a gown and goggles to protect ourselves." D) "Let's get a full set of vital signs, since this disease can cause heart problems, especially in the young." Answer: A Diff: 2 Page Ref: 489 Objective: 16-14 63) You suspect a patient of being infected with pertussis. While performing the primary assessment, you note a deep cough with clear breath sounds. Her vital signs are pulse, 108; respirations, 16 breaths/min; and SpO2, 96% on room air. Given this scenario, which action by the EMT would be most appropriate? A) Assist with use of a metered-dose inhaler B) Administer oxygen via nasal cannula C) Encourage the patient not to cough D) Monitor the patient and transport Answer: D Diff: 2 Page Ref: 489-490 Objective: 16-14 64) Which of these pieces of information would heighten the EMT's suspicion that a child with a persistent cough may have whooping cough? A) Chest pain while coughing B) Cough for the past six months C) Never had childhood immunizations D) Cough worse during the day Answer: C Diff: 2 Page Ref: 489 Objective: 16-14 17 Copyright © 2018 Pearson Education, Inc.

65) You have been dispatched to a home for a 16-year-old female patient with shortness of breath. On scene, you find the patient in bed with her mother at her side. Her mother states that her daughter has a fever with a past medical history of cystic fibrosis; she would like to have her daughter transported to the hospital for evaluation of possible pneumonia. As a knowledgeable EMT, you know that the underlying pathophysiology of cystic fibrosis is a: A) Contagious disease in which the lungs are damaged by bacteria and excessive coughing B) Condition in which excessive and thick mucus often plugs the airways C) Disease in which lifelong antibiotic medications are needed for survival D) Hereditary disease in which the lungs are malformed and smaller than normal Answer: B Diff: 3 Page Ref: 490 Objective: 16-14 66) You have been called to a residence for an 18-year-old female patient with shortness of breath and a history of cystic fibrosis. On scene, you find the patient to be very thin and sick looking. Her airway is patent and breathing adequate, although slightly labored. Her radial pulse is strong, and she has a constant cough, which occasionally produces green- and yellow-colored mucus. Auscultation of the lungs reveals rhonchi to the upper portion of both lungs. Vital signs are pulse, 108; respirations, 20 breaths/min; blood pressure, 98/56 mmHg; and SpO2, 92%. Based on these findings, appropriate care should include: A) Suctioning of the lower airway to remove mucus B) Gentle abdominal thrusts to help the patient expectorate the mucus C) A bronchodilator administered via small-volume nebulizer D) Humidified oxygen via nasal cannula starting at 2 liters per minute Answer: D Diff: 2 Page Ref: 490 Objective: 16-14 67) A 26-year-old male patient has summoned EMS after lifting a heavy box and suddenly experiencing shortness of breath and sharp pain to the left side of his chest that worsens with deep inspiration. Given this description, you immediately become suspicious of a pneumothorax. Which action should you perform next to help confirm this suspicion? A) Inquire about past lung problems B) Obtain an SpO2 reading C) Palpate the chest for subcutaneous emphysema D) Auscultate breath sounds Answer: D Diff: 2 Page Ref: 504, Table 16-8 Objective: 16-18

18 Copyright © 2018 Pearson Education, Inc.

68) You are delivering positive pressure ventilations to a patient who is breathing poorly and has absent breath sounds in the right lung. Which of these instructions would you provide to those responders who are ventilating the patient? A) "Make sure to really get those ventilations into his good lung. He needs the oxygen." B) "Use the minimal tidal volume possible, just enough to make the chest rise and fall." C) "If his SpO2 is 85% or greater, let's stop the ventilations and put him on high-concentration oxygen with a face mask." D) "Let's use just room air for the ventilations. We don't want to cause a tension pneumothorax by giving pure oxygen." Answer: B Diff: 3 Page Ref: 487 Objective: 16-12 69) Assessment of a 67-year-old male patient reveals absent lung sounds to the right lung, accompanied by sharp chest pain to the same side. When obtaining a history, which of these statements made by the patient's wife would the EMT recognize as most important? A) "He did not take his medications today." B) "He had a fever last night." C) "He has emphysema." D) "He has been in bed for 12 hours." Answer: C Diff: 2 Page Ref: 486-487 Objective: 16-4 70) On scene, you suspect that a young female patient is suffering from hyperventilation syndrome. Which additional finding would best assist you in confirming this suspicion? A) Heart rate of 64 beats/min B) Urinary incontinence C) Breathing through the mouth D) Spasm of the hands Answer: D Diff: 2 Page Ref: 487-488 Objective: 16-13 71) You are transporting a 4-year-old female patient whom you found sitting in a chair with labored respirations, inspiratory stridor, and drooling. Which sign or symptom would warrant immediate reassessment? A) The stridor ceases B) The drooling continues. C) The SpO2 drops from 96% to 95% D) The patient remains dyspneic Answer: A Diff: 2 Page Ref: 489 Objective: 16-14

19 Copyright © 2018 Pearson Education, Inc.

72) You have been called to an industrial warehouse for a young adult male patient who was exposed to an unknown chemical gas. The patient was immediately removed from the environment and presents to you with a patent airway, burning to the throat with inspiration, and complaint of shortness of breath. His vital signs are pulse, 120; respirations, 28 breaths/min; blood pressure, 164/60 mmHg; and SpO2, 90%. In addition, you note bilateral wheezing to the lungs. What should be your first action in caring for this patient? A) Initiate manual spinal motion restriction B) Administer supplemental oxygen C) Identify the chemical to which the patient was exposed D) Administer a saline mist through a small-volume nebulizer Answer: B Diff: 2 Page Ref: 491-492 Objective: 16-14 73) You have been called for a 6-year-old male patient with shortness of breath. On scene, you find the patient with a runny nose and mucus coming from the right nare. Breath sounds are clear and his SpO2 is 99% on room air. When asked, the patient states that his throat is very sore. His vital signs are pulse, 124; respirations, 20 breaths/min; and temperature, 98.9°F. There is no medical history, according to the mother. Which statement or instruction would be most appropriate for this situation? A) "Why don't we give him an aspirin for his fever and then you can follow up at your pediatrician's office." B) "I am very concerned he may have epiglottitis, so we are going to take him to the hospital with lights and siren." C) "Let's give him some oxygen since the heart rate is most likely elevated because his oxygen is low." D) "He is stable, but we will take him to the hospital. The danger lies in the infection spreading to the lungs." Answer: D Diff: 3 Page Ref: 497 Objective: 16-3 74) The EMT understands adequate breathing when he states: A) "If the respiratory rate is between 10 and 20 breaths/min, the breathing is most likely adequate." B) "If the patient is breathing faster than 22 breaths/min, it is inadequate." C) "Normal and adequate breathing is specific to the patient and is determined by assessment." D) "If the respiratory rate is between 10 and 20 breaths/min, with an intact airway and clear breath sounds, it is adequate." Answer: C Diff: 2 Page Ref: 474 Objective: 16-3

20 Copyright © 2018 Pearson Education, Inc.

75) The EMT is correctly accessing for accessory muscle use when he examines which part of the body? A) Nares B) Chest C) Back D) Inner mouth Answer: B Diff: 1 Page Ref: 500 Objective: 16-18 76) You are starting to assess a patient's breath sounds and have just listened to the upper right back. You would next listen to the: A) Left upper back B) Left anterior chest C) Right lower back D) Left lateral chest Answer: A Diff: 1 Page Ref: 474 Objective: 16-4 77) Which audible respiratory sounds would make you suspicious of an upper airway problem? A) Wheezing B) Crackles C) Rhonchi D) Stridor Answer: D Diff: 2 Page Ref: 498 Objective: 16-4 78) A patient with a history of COPD is breathing 32 times a minute. Her vital signs are pulse, 140; blood pressure, 168/102 mmHg; and SpO2, 98% on room air. You would classify this patient's condition as: A) Apnea B) Dyspnea C) Bradypnea D) Hypoxia Answer: B Diff: 1 Page Ref: 508 Objective: 16-5

21 Copyright © 2018 Pearson Education, Inc.

79) The best way to gauge the effectiveness of oxygen therapy is to monitor the: A) Pulse oximetry B) Respiratory rate C) Heart rate D) Mental status Answer: A Diff: 2 Page Ref: 507 Objective: 16-19 80) A patient in respiratory distress is exhibiting pursed lip breathing. You realize that he is doing this to: A) Exhale increased amounts of CO2 B) Inhale additional amounts of oxygen C) Keep the small airways open D) Prevent the loss of vapor with exhalation Answer: C Diff: 3 Page Ref: 501 Objective: 16-18 81) Pursed lip breathing is most commonly observed when a patient has: A) Pneumonia B) Croup C) Upper airway obstruction D) COPD Answer: D Diff: 2 Page Ref: 479 Objective: 16-8 82) An alert and distressed 72-year-old male patient with a history of pulmonary edema from heart failure is in severe respiratory distress, breathing 28 to 32 times per minute. His airway is patent, respirations are labored, crackles are heard in the bases of the lungs, and skin is found to be diaphoretic with a rapid radial pulse. Other vital signs are pulse, 136; blood pressure, 202/110 mmHg; and SpO2, 83%. Cyanosis is evident in the fingertips. The best treatment for this patient's breathing difficulty would be: A) CPAP B) Albuterol MDI C) Supplemental oxygen via nasal cannula D) Nasal airway with high-concentration O2 Answer: A Diff: 2 Page Ref: 486 Objective: 16-11

22 Copyright © 2018 Pearson Education, Inc.

83) A patient with an unknown medical problem is responsive to painful stimuli and breathing poorly at a rate of 8 breaths/min. The airway is patent and the skin warm and moist. The patient's radial pulse is strong and irregular. Breath sounds are absent over the bases of the lungs. Vital signs are pulse, 84; blood pressure, 256/120 mmHg; and SpO2, 81%. The best treatment for this patient's breathing difficulty would be: A) High-concentration oxygen through a nonrebreather mask B) Continuous positive airway pressure (CPAP) C) Positive pressure ventilation D) 2 to 4 liters of oxygen through a nasal cannula Answer: C Diff: 2 Page Ref: 485-486 Objective: 16-5 84) An acute infectious disease caused by bacterium or a virus that affects the lower respiratory tract and causes lung inflammation resulting in dyspnea is: A) Chronic bronchitis B) Pneumonia C) Emphysema D) Asthma Answer: B Diff: 1 Page Ref: 483-484 Objective: 16-9 85) Why would a patient with COPD have a higher percentage of red blood cells than a healthy person without COPD? A) To better fight lung infections B) To make the blood "thinner" C) To help prevent spontaneous bleeding in the lungs D) To carry additional oxygen to the cells Answer: D Diff: 3 Page Ref: 480 Objective: 16-8 86) A patient has a medical problem characterized by an increase of interstitial fluid in and around his alveoli. This condition would disrupt normal oxygen and carbon dioxide exchange by creating which type of disturbance? A) A gas exchange disturbance B) A ventilation disturbance C) A perfusion disturbance D) A neurological disturbance Answer: A Diff: 2 Page Ref: 476 Objective: 16-2

23 Copyright © 2018 Pearson Education, Inc.

87) During an asthma attack, three main pathological changes lead to hypoxia in the patient. Which of these changes is not one of these three? A) Bronchospasm B) Decreased alveolar perfusion C) Increased edema to bronchiole walls D) Heavy mucus secretion and airway plugging Answer: B Diff: 2 Page Ref: 481 Objective: 16-7 88) While talking to your partner one day in between emergency calls, he tells you he has recently read a CE article on pulmonary embolisms. Which of these statements indicates that your partner understands the underlying pathophysiology of how a pulmonary emboli can cause hypoxia? A) "During an embolic process, the alveoli become inflamed and the walls of the alveoli thicken and prevent oxygen transfer, resulting in hypoxia." B) "A pulmonary emboli occurs when a small bronchiole of a lung becomes plugged by mucus or a clot in the bloodstream, blocking the movement of air into the alveoli, which results in hypoxia." C) "The primary cause of a pulmonary emboli is a blood clot from the legs that lodges in the pulmonary veins and prevents blood exiting the alveolar capillary beds from reaching the heart." D) "When the pulmonary vasculature becomes blocked by an emboli, then blood cannot reach the alveoli for gas exchange, and hypoxia can result." Answer: D Diff: 3 Page Ref: 484 Objective: 16-7 89) You are at an extended care facility for the elderly to assess and transport a patient suspected of having pneumonia to the hospital for evaluation. During your pulmonary assessment, which breath sound would you expect to hear that would be most specific to pneumonia versus other pulmonary conditions? A) Bilateral rhonchi B) Inspiratory stridor C) Localized wheezing to one side of the chest D) Inspiratory and expiratory crackles to the tops of both lungs Answer: C Diff: 1 Page Ref: 484 Objective: 16-9

24 Copyright © 2018 Pearson Education, Inc.

90) A new employee of your EMS system is about to take a protocol test in front of the EMS supervisor. As he is reviewing the protocol from memory, he gets confused about which amount of oxygen to administer to a patient with a suspected pulmonary embolism. How would you respond if he asks you for help? A) "Administer 2 liters per minute by nasal cannula to all patients with suspected pulmonary embolism." B) "Administer supplemental oxygen to the patient of sufficient amount to maintain the pulse oximeter reading at or greater than 94%." C) "Since a pulmonary embolism is a vascular problem rather than lung problem, administer oxygen to maintain a pulse oximeter reading of greater than 95%." D) "A patient with a pulmonary embolism has a perfusion deficit, not a ventilation deficit, so oxygen will not change things. Simply start ventilations and transport the patient." Answer: B Diff: 3 Page Ref: 485 Objective: 16-10 91) During your reassessment of the respiratory system and oxygenation status of a patient with a chronic pulmonary condition, which of these would most likely not be part of this reassessment process? A) Breath sounds B) Blood glucose level C) Mental status D) Vital signs Answer: B Diff: 2 Page Ref: 509 Objective: 16-19

25 Copyright © 2018 Pearson Education, Inc.

Prehospital Emergency Care, 11e (Mistovich et al.) Chapter 17 Cardiovascular Emergencies 1) The primary function of the heart is to: A) Store blood B) Oxygenate blood C) Circulate blood D) Produce blood Answer: C Diff: 1 Page Ref: 522 Objective: 17-2 2) The tricuspid valve prevents blood from: A) Regurgitating between the right and left ventricles B) Flowing from the right ventricle into the right atrium C) Bypassing the pulmonary vessels and entering the left heart D) Flowing backward through the left atrium Answer: B Diff: 3 Page Ref: 522 Objective: 17-2 3) A patient has pulmonary hypertension, which has increased the resistance in the pulmonary blood vessels. In response, what will most likely occur over a long period of time? A) The right side of the heart will weaken B) The heart muscle will become stronger C) The left side of the heart will weaken and fail D) The aorta will weaken and potentially rupture Answer: A Diff: 3 Page Ref: 522 Objective: 17-6 4) Which portion of the heart, when weakened by a heart attack, is responsible for causing fluid to back up into the alveoli? A) Right atrium B) Left ventricle C) Right ventricle D) Pulmonary vein Answer: B Diff: 2 Page Ref: 523 Objective: 17-9

1 Copyright © 2018 Pearson Education, Inc.

5) If a clot occludes the blood flow through the coronary artery, what will be the detrimental effect? A) The flow of oxygen-rich blood through the peripheral arteries will cease B) Blood will not be able to flow between the left atrium and the left ventricle C) The flow of oxygen-rich blood to the muscle of the ventricle will be decreased D) Blood and fluid will back up into the lung tissue, causing pulmonary edema Answer: C Diff: 2 Page Ref: 527 Objective: 17-11 6) In the blood, which component is largely responsible for the formation of blood clots? A) Red blood cells B) White blood cells C) Plasma D) Platelets Answer: D Diff: 1 Page Ref: 527 Objective: 17-1 7) A patient has a low platelet count. Based on this finding, which complication is he most at risk for? A) Clot formation B) Bleeding C) Hypotension D) Angina Answer: B Diff: 2 Page Ref: 527 Objective: 17-2 8) A patient has been diagnosed with high blood pressure. A medication that can effectively serve to lower this patient's blood pressure is one that: A) Increases the heart rate B) Dilates the coronary arteries C) Dilates the arteries D) Constricts the veins Answer: C Diff: 2 Page Ref: 529 Objective: 17-2

2 Copyright © 2018 Pearson Education, Inc.

9) When treating a patient with chest discomfort, the EMT's primary goal is to: A) Determine whether chest discomfort is originating from the heart B) Recognize the possibility of cardiac compromise and provide proper emergency care C) Provide immediate care to eliminate the chest discomfort, and then transport the patient to the hospital D) Administer fibrinolytic drugs with the permission of medical direction Answer: B Diff: 1 Page Ref: 529 Objective: 17-9 10) A TV reporter has been assigned to your ambulance for a ride-along. He is doing a segment on the local EMS system for EMS Week. In your down time, the reporter tells you that his wife was diagnosed with atherosclerosis. He is unsure what this condition is and asks you to explain it to him. Your response would be: A) "Atherosclerosis affects the arteries of the heart and is caused by a heart attack." B) "The walls of the coronary arteries lose elasticity, which can lead to a heart attack." C) "The coronary arteries become damaged and narrowed by fat deposits." D) "Atherosclerosis affects the veins in the body and is caused by untreated high blood pressure." Answer: C Diff: 2 Page Ref: 529-530 Objective: 17-7 11) You are interviewing a patient whom you suspect has an acute coronary syndrome. Which question is most appropriate initially? A) "Are you having any pain in your chest?" B) "Are you having any chest discomfort?" C) "Does it hurt when I press on your chest?" D) "Does your chest hurt when you breathe?" Answer: B Diff: 2 Page Ref: 530 Objective: 17-10 12) You have arrived at a residence to help another crew with a 62-year-old woman who is complaining of fatigue, chest discomfort, and diaphoresis. The EMTs on scene report that she has a history of hypertension from atherosclerosis, for which she takes a blood pressure pill, and is also prescribed nitroglycerin. She reports an allergy to aspirin. Vital signs are pulse, 64; respirations, 16 breaths/min; blood pressure, 128/86 mmHg; and SpO2, 97% on room air. Which of these statements made by your EMT partner would require immediate correction on your part? A) "I am not going to attach the AED, because the patient is conscious and has a pulse." B) "I am going to put the patient on high-concentration oxygen because she has chest discomfort." C) "I will contact medical direction for permission to administer the patient's nitroglycerin." D) "I am not going to give the patient any aspirin, because she says that she is allergic to it." Answer: B Diff: 2 Page Ref: 532 Objective: 17-14 3 Copyright © 2018 Pearson Education, Inc.

13) You have been called to an office building for a 47-year-old male patient complaining of chest pain. He is alert, oriented, and complaining of mild pain in his chest that came on suddenly about 15 minutes ago. Airway, breathing, and circulation are intact, and his skin is diaphoretic and cool. What should you do first? A) Apply supplemental oxygen through a nasal cannula B) Obtain a heart rate, respiratory rate, blood pressure, and SpO2 reading C) Determine if there is a family history of cardiac illness D) Administer the first of up to three nitroglycerin tablets Answer: B Diff: 2 Page Ref: 544 Objective: 17-10 14) After being dispatched for a male patient experiencing dyspnea, you arrive to find the confused 67-year-old man sitting upright at his kitchen table in severe respiratory distress. You also note remarkable swelling to his lower legs and feet. He has an open airway, but cannot speak more than two to three words without attempting to breathe. His breathing is extremely labored, with inspiratory crackles heard upon auscultation. His skin is cool, diaphoretic, and dusky in color. A family member at the scene reports a history of hypertension, congestive heart failure, and angina, for which the patient takes nitroglycerin. The baseline vital signs are acceptable and the room-air pulse oximeter reading is 45%. What is your immediate action in caring for this patient? A) Assist the patient with his nitroglycerin B) Apply but do not turn on the AED C) Apply continuous positive airway pressure (CPAP) D) Obtain vital signs and give aspirin Answer: C Diff: 2 Page Ref: 537 Objective: 17-11 15) You are caring for a 62-year-old female patient with chest discomfort and shortness of breath. She is alert and oriented with an open airway. Her breathing is adequate and radial pulse strong. Her skin is warm and dry. What should you do next? A) Perform a secondary assessment B) Obtain the patient's heart rate and blood pressure C) Assist the patient to the ambulance for immediate transport D) Assist the patient in taking a nitroglycerin tablet Answer: B Diff: 1 Page Ref: 544 Objective: 13-10

4 Copyright © 2018 Pearson Education, Inc.

16) Which of these statements made by another EMT, regarding chest pain and acute coronary syndrome, is correct? A) "If the chest pain started during activity, it is most likely cardiac. If the pain started while the patient was at rest, it is most likely not cardiac." B) "If the patient's chest pain does not go away with nitroglycerin treatment, the EMT should assume that the medication is expired and ineffective." C) "If the patient experiences chest pain that radiates into the left arm, it is most likely a heart attack. If the pain radiates anywhere else, it is most likely not cardiac in origin." D) "Some patients experiencing acute coronary syndrome do not have actual chest pain; rather, they experience a sensation of discomfort or pressure." Answer: D Diff: 3 Page Ref: 530 Objective: 17-8 17) You are assisting a paramedic in the transport of a patient from a community hospital emergency department to a large medical center. The patient was diagnosed with an acute myocardial infarction and received fibrinolytic therapy in the ED. Currently, you are transporting the patient with the fibrinolytic medication infusing through an IV. In regard to the fibrinolytic therapy, you know that during this transport, you will need to closely monitor the patient for which complication? A) Respiratory arrest B) Internal bleeding C) Hypoglycemia D) Hypertension Answer: B Diff: 2 Page Ref: 543 Objective: 17-19 18) Which of these statements from a patient should the EMT recognize as an absolute contraindication to the use of fibrinolytic medications in the emergency department? A) "I have high blood pressure for which I take medication." B) "Two months ago, I had a stroke on the left side of my brain." C) "My doctor told me to take a baby aspirin every day." D) "I had my appendix removed six months ago." Answer: B Diff: 3 Page Ref: 543 Objective: 17-19

5 Copyright © 2018 Pearson Education, Inc.

19) A patient with chest pain and shortness of breath informs you that the last time he had a heart attack, he went into heart failure. When performing your secondary assessment, which sign or symptom is most indicative that the patient is in heart failure again? A) Flat neck veins B) Crackles in the lungs C) Hypertension D) Shortness of breath Answer: B Diff: 3 Page Ref: 536 Objective: 17-11 20) You are caring for an alert and oriented patient with chest pain. Emergency First Responders have initiated supplemental oxygen, and your EMT partner assisted in the administration of aspirin and two nitroglycerin tablets. Currently, the patient's vital signs are pulse, 76; respirations, 16 breaths/min; blood pressure, 110/56 mmHg; and SpO2, 95%. When transporting this patient on the stretcher, which position is best? A) Semi-Fowler's position B) Supine with head elevated C) Lateral recumbent D) Position of comfort Answer: D Diff: 1 Page Ref: 533 Objective: 17-11 21) For the EMT to administer aspirin in the prehospital setting, which of these criteria must be met? A) Approval from medical direction B) Patient has a prescription for aspirin C) Age younger than 65 years D) Systolic blood pressure greater than 100 mmHg Answer: A Diff: 1 Page Ref: 547, Figure 17-19 Objective: 17-9 22) When administering aspirin to a patient with chest discomfort that is suggestive of a heart attack, the EMT recognizes that this medication will benefit the patient by: A) Breaking up the blood clot that is occluding the coronary artery B) Treating the headache that may occur when nitroglycerin is administered C) Decreasing the ability of the platelets in the blood to form clots D) Decreasing the chest pain caused by the possible heart attack Answer: C Diff: 3 Page Ref: 547, Figure 17-19 Objective: 17-9

6 Copyright © 2018 Pearson Education, Inc.

23) The EMT shows she is correctly administering aspirin to a patient with chest pain when she provides: A) 325 mg of enteric-coated aspirin and instructs the patient to swallow it B) 325 mg of baby aspirin and instructs the patient to chew it C) 325 mg of baby aspirin and instructs the patient to let it dissolve under the tongue D) 160 mg of baby aspirin and instructs the patient to swallow it with a glass of water Answer: B Diff: 2 Page Ref: 547, Figure 17-19 Objective: 17-9 24) When assessing a patient in the prehospital setting, which of these does the EMT recognize as an indication for aspirin? A) Complaint of a headache after taking nitroglycerin B) An aspirin taken daily for cardiovascular disease C) Systolic blood pressure greater than 100 mmHg D) Chest discomfort that is suggestive of a heart attack Answer: D Diff: 2 Page Ref: 547, Figure 17-19 Objective: 17-9 25) You are by the side of a patient who complains of severe chest pain that radiates into his right arm and neck. He is alert and oriented, with an open airway and adequate breathing. His radial pulse is strong, and his skin is cool and diaphoretic. Your partner reports his pulse rate is 84, respirations are 18 breaths/min, blood pressure is 86/62 mmHg, and SpO2 is 98% on room air. You have obtained a medical history and performed the secondary assessment. The patient states that he has had two heart attacks in the past and is allergic to aspirin and sulfa medications. What should you do next? A) Assist the patient in taking a nitroglycerin tablet B) Administer baby aspirin to the patient C) Administer oxygen and nitroglycerin, and then transport the patient D) Transfer the patient to the stretcher and begin transport Answer: D Diff: 2 Page Ref: 533 Objective: 17-11 26) You are transporting a patient who had an episode of chest pain. On scene, you assisted him in taking his nitroglycerin medication, which completely alleviated the chest pain. En route to the hospital, the patient asks you why the nitroglycerin took the chest pain away. Beyond telling the patient that nitroglycerin can increase coronary artery blood flow, you would add: A) "It causes the heart rate to decrease, which relaxes it and decreases pain." B) "It has a sedative effect on the brain, making you less aware of the pain." C) "It dilates the blood vessels, which decreases the workload of the heart." D) "It causes the body to relax, decreasing the need for oxygen." Answer: C Diff: 2 Page Ref: 538 Objective: 17-16 7 Copyright © 2018 Pearson Education, Inc.

27) You are completing the prehospital care report and are documenting the fact that you assisted a patient in taking his nitroglycerin pill. How will you document the route of administration? A) Oral B) Topical C) Sublingual D) Buccal Answer: C Diff: 1 Page Ref: 538 Objective: 17-16 28) Which instruction given by an EMT indicates the proper administration of nitroglycerin? A) "Breathe in when I spray the nitroglycerin into your mouth." B) "This pill will go between your cheek and gum." C) "Please chew and swallow this pill." D) "Let this pill dissolve under your tongue." Answer: D Diff: 1 Page Ref: 538 Objective: 17-16 29) Prior to administering nitroglycerin to a patient with chest pain, the EMT must ensure that: A) The nitroglycerin is prescribed for the patient B) The patient rates his chest pain as 10 out of 10 C) The patient's systolic blood pressure is greater than 110 mmHg D) The patient is standing in an upright position Answer: A Diff: 1 Page Ref: 540 Objective: 17-16 30) Which of these statements made by the EMT indicates a safe understanding of the administration of nitroglycerin in the prehospital setting? A) "I always check for an allergy to sulfa, since most people with an allergy to sulfa also have an allergy to nitroglycerin." B) "Since nitroglycerin constricts the blood vessels, you must always check the blood pressure before and after giving it." C) "A total of three nitroglycerin tablets may be administered in the prehospital setting, and this includes what the patient recently took prior to EMS arrival." D) "If the patient with chest pain also has a headache, the EMT should contact medical command prior to administering nitroglycerin." Answer: C Diff: 3 Page Ref: 540 Objective: 17-16

8 Copyright © 2018 Pearson Education, Inc.

31) A 44-year-old male patient is experiencing chest pain. He has a history of angina and is prescribed nitroglycerin, which he has not taken yet. He is also taking another medication. Which of these medications is a contraindication to the administration of nitroglycerin? A) Coumadin B) Levitra C) Methadone D) Albuterol Answer: B Diff: 2 Page Ref: 540 Objective: 17-16 32) A 67-year-old male patient has chest pain. After you assist him with taking two of his nitroglycerin tablets, his chest pain remains 7 out of 10 and he is still diaphoretic. His vital signs are as follows: pulse, 72; respirations, 18 breaths/min and adequate; blood pressure, 82/60 mmHg; and SpO2, 97% on 2 lpm of oxygen. You should: A) Administer a third nitroglycerin tablet B) Increase the oxygen to 4 lpm C) Administer half of a nitroglycerin tablet D) Transfer the patient to the stretcher and begin transport Answer: D Diff: 2 Page Ref: 540 Objective: 17-16 33) You have been called for a 56-year-old female patient who complains of chest pain. She informs you that she takes nitroglycerin and asks you to get it from the top of her refrigerator. You find it is a clear glass jar hand-labeled "nitroglycerin." What of these statements, based on this observation, would be the most appropriate one to make to the patient? A) "You should keep your nitroglycerin in the refrigerator since it will keep longer in the cold." B) "It is best to keep the nitroglycerin in a dark container since light will inactivate it." C) "The heat from the refrigerator will cause the nitroglycerin to lose its effectiveness." D) "Keeping the nitroglycerin on top of the refrigerator may make it difficult for you to reach it in an emergency." Answer: B Diff: 2 Page Ref: 539 Objective: 17-16 34) Which action is it essential that the EMT take prior to assisting the patient in taking his nitroglycerin tablet? A) Determine whether the patient had a stroke within the last 6 months B) Evaluate the patient's systolic blood pressure and heart rate C) Inquire about whether the patient is allergic to sulfa drugs D) Determine the presence or absence of a headache Answer: B Diff: 2 Page Ref: 538 Objective: 17-16

9 Copyright © 2018 Pearson Education, Inc.

35) A patient has chest pain, and you have just assisted him in taking his nitroglycerin tablet. Which of these statements made by the patient is the cause for greatest concern? A) "It is easier to breathe now." B) "The chest pain does not feel any different." C) "I seem to be getting a headache." D) "My tongue is stinging and burning." Answer: B Diff: 2 Page Ref: 539 Objective: 17-16 36) When obtaining a medical history, which of these statements made by the patient seemingly indicates that he is suffering from classic angina? A) "I took five nitroglycerin tablets over 20 minutes to get rid of the pain." B) "When I stopped cutting the grass, the pain went away." C) "The pain started when I was sitting down reading the paper." D) "The pain lasted about 45 minutes before gradually going away." Answer: B Diff: 3 Page Ref: 531 Objective: 17-10 37) You have been called for a patient with angina. On scene, the 67-year-old man reports that he has a history of angina and the pain started when he was moving firewood from the garage into his house. Unfortunately, the pain has yet to go away, despite three nitroglycerin tablets and 20 minutes of rest. Your next action should be to: A) Check the patient's vital signs B) Administer a nitroglycerin tablet and aspirin C) Attach the AED but do not turn it on D) Provide supplemental oxygen via nonrebreather Answer: A Diff: 2 Page Ref: 532 Objective: 17-10 38) Which of these statements would the EMT expect from a patient with unstable angina? A) "When I stop what I am doing, the pain goes away." B) "If I take a nitroglycerin tablet, the pain stops." C) "The chest pain awakens me from my nighttime sleep." D) "The antacid my doctor prescribed seems to take the pain away." Answer: C Diff: 3 Page Ref: 530 Objective: 17-8

10 Copyright © 2018 Pearson Education, Inc.

39) The EMT shows that she understands the difference between classic angina and an acute myocardial infarction (MI) when she states: A) "Classic angina occurs when the heart rate becomes too fast; an acute MI is the result of coronary artery disease." B) "Classic angina is not accompanied by other symptoms like shortness of breath; the chest pain with an acute MI is." C) "The pain in an acute MI is typically described as stabbing; the pain with classic angina is more pressure-like." D) "An acute myocardial infarction results in the death of cardiac tissue; classic angina does not." Answer: D Diff: 3 Page Ref: 532 Objective: 17-11 40) A patient with a past medical history of angina is now complaining of chest discomfort. When assessing this patient, which sign or symptom should alert the EMT that the patient is probably experiencing an acute myocardial infarction and not an anginal attack? A) Pain radiates into the left arm B) The patient complains of mild shortness of breath C) Chest discomfort is not relieved by nitroglycerin D) The patient's skin is not diaphoretic Answer: C Diff: 3 Page Ref: 532 Objective: 17-11 41) You are assessing a patient in heart failure. Which sign or symptom seemingly indicates that the left ventricle is the cause of the heart failure? A) Crackles in the bases of both lungs B) SpO2 of 95% with no supplemental oxygen C) Difficulty breathing when in an upright position D) Heart rate of 62 beats/min Answer: A Diff: 3 Page Ref: 536 Objective: 17-9

11 Copyright © 2018 Pearson Education, Inc.

42) You are caring for a 69-year-old male patient who is short of breath and in acute congestive heart failure. He is alert, anxious, and panicked, with an open airway and rapid shallow respirations. His heart rate is tachycardic, and his skin is cool and diaphoretic, with cyanosis noted to the lips and fingers. There are crackles noted in both lungs, as well as pedal edema in both feet. The patient denies chest pain or discomfort, but is having a difficult time speaking due to the shortness of breath. When caring for this patient, you should: A) Administer four baby aspirins B) Place the patient in the shock position C) Insert a nasal airway D) Apply continuous positive airway pressure (CPAP) Answer: D Diff: 2 Page Ref: 537 Objective: 17-17 43) You suspect that a patient with a history of chronic obstructive pulmonary disease (COPD) is has right ventricular failure. In your assessment, which finding would reinforce this suspicion? A) Elevated blood pressure B) Edema to the feet C) Crackles in both lungs D) Bounding radial pulse Answer: B Diff: 2 Page Ref: 536 Objective: 17-15 44) The pathophysiology of heart failure that causes the common signs and symptoms of cardiogenic shock is: A) Inadequate ejection of blood from the left ventricle B) Poor gas exchange in the lungs C) Fluid accumulation within the lung tissue D) Backup of fluid in the lower extremities Answer: A Diff: 2 Page Ref: 535-536 Objective: 17-15 45) An alert and oriented patient presents with shortness of breath, crackles in both lungs, jugular venous distention, and edema to the feet and ankles. Her pulse is 132, respirations 26 breaths/min, blood pressure 160/86 mmHg, and SpO2 at 88%. Based on these findings, the EMT should suspect: A) Left ventricular failure B) Hypertensive emergency C) Right ventricular failure D) Congestive heart failure Answer: D Diff: 2 Page Ref: 536 Objective: 17-8

12 Copyright © 2018 Pearson Education, Inc.

46) You are assessing a 73-year-old male patient who complains of altered mental status and shortness of breath. The patient is conscious, but confused. His pulse is rapid and weak, and his skin cool and diaphoretic. An Emergency Medical Responder provides you with the following vital signs: pulse, 136; respirations, 20 breaths/min; blood pressure, 168/88 mmHg; and SpO2, 89%. A family member states that the patient has diabetes and takes insulin. He also has a history of hypertension, colon cancer, irregular heartbeat, and transient ischemic attacks (TIA). Which action should the EMT take next? A) Check the patient's blood sugar B) Perform the secondary exam C) Apply supplemental oxygen D) Administer oral glucose Answer: C Diff: 2 Page Ref: 542 Objective: 17-18 47) A 71-year-old female patient is lying in bed and complaining of respiratory distress. She exhibits moderate dyspnea, crackles in both lungs, and edema to the abdomen and lower extremities. Supplemental oxygen is being provided to the patient by Emergency Medical Responders. Her vital signs are as follows: pulse, 132; respirations, 18 breaths/min; blood pressure, 138/86 mmHg; and SpO2, 95%. Which of these actions would be of the most immediate benefit to the patient? A) Administer four baby aspirins B) Elevate both legs to decrease swelling C) Obtain a medical history and test the blood glucose D) Position the patient in a sitting upright position Answer: D Diff: 2 Page Ref: 545 Objective: 17-18 48) Which of these statements made by the patient should the EMT associate with the condition of congestive heart failure? A) "I noticed that I am having a hard time breathing and have a fever." B) "I have a hard time breathing when I am lying down." C) "When I walk, I feel very weak and get quite dizzy." D) "I can feel my heart pounding in my head and I have a throbbing headache." Answer: B Diff: 2 Page Ref: 538 Objective: 17-18 49) The EMT should most suspect a patient has angina when he takes which medication? A) Lasix B) Aspirin C) Nitroglycerin D) Glucophage Answer: C Diff: 1 Page Ref: 538 Objective: 17-16 13 Copyright © 2018 Pearson Education, Inc.

50) You have been called for a 58-year-old male patient with an unknown medical emergency. On scene, you find the alert and oriented patient complains of headache as well as weakness and "tingling" to his right arm and leg. He has diabetes and high blood pressure, for which he takes the appropriate medications. When asked, he reports that he may have passed out earlier in the day, immediately after taking his morning dose of insulin. His pulse is 96, respirations 16 breaths/min, and blood pressure 180/106 mmHg. Based on this presentation, you should treat the patient for: A) High blood sugar B) Stroke C) Hypertensive emergency D) Syncope Answer: B Diff: 3 Page Ref: 538 Objective: 17-6 51) You are treating a patient for mild chest pain but notice that his blood pressure is elevated. Which of these statements made by the patient would make you suspect he has acute hypertension? A) "The doctor said that my blood pressure is high because my kidneys are bad." B) "I have to take two blood pressure medications at the same time to get my pressure down." C) "The doctor checks my blood pressure regularly and it's not normally that high." D) "My high blood pressure caused my heart to go bad." Answer: C Diff: 2 Page Ref: 538 Objective: 17-6 52) Immediately after assisting a patient with diabetes to take her nitroglycerin, she states that she feels weak, dizzy, and nauseated. What should the EMT do next? A) Position her in an upright position B) Inform her that this feeling is normal C) Check her blood glucose level D) Check her blood pressure Answer: D Diff: 2 Page Ref: 540 Objective: 17-16 53) You have just arrived by the side of a 68-year-old male patient who complains of chest pain. After determining his airway is patent and breathing adequate, it is imperative that you perform which action next? A) Auscultate his breath sounds B) Determine whether he takes nitroglycerin C) Check his radial pulse D) Obtain a full set of vital signs Answer: C Diff: 1 Page Ref: 542 Objective: 17-18 14 Copyright © 2018 Pearson Education, Inc.

54) You have just arrived by the side of a male patient who is in cardiac arrest. Emergency Medical Responders have been on scene for 6 minutes performing CPR. Advanced life support (ALS) has been contacted and is 1 minute out. What should you do immediately? A) Continue CPR and wait for ALS care B) Apply and use the automated external defibrillator C) Check the heart rate, blood pressure, and SpO2 D) Place the patient on the stretcher so ALS can provide care en route Answer: B Diff: 2 Page Ref: 542 Objective: 17-18 55) A patient complains of chest pain that radiates into his neck and arms. The primary assessment reveals a patent airway, adequate breathing, and a strong, regular radial pulse. The patient's skin is warm and dry, and reveals no signs of inadequate perfusion. What should you do next? A) Start the secondary assessment B) Attach the AED C) Provide oxygen D) Check vital signs Answer: D Diff: 1 Page Ref: 542 Objective: 17-18 56) Which of these statements regarding cardiac compromise or acute coronary syndrome should the EMT recognize as true? A) Cardiac compromise should be suspected for any patient with chest discomfort B) Patients suffering from acute coronary syndrome will have pain, not discomfort C) If the pain does not radiate to the arms, then the cause is not typically the heart D) The onset of acute coronary syndrome typically occurs with physical activity Answer: A Diff: 2 Page Ref: 530 Objective: 17-8 57) You are transporting a patient with chest pain to the hospital emergency department. In regard to possible fibrinolytic therapy, what should you relay during your hospital report as a possible relative contraindication to this therapy? A) Blood glucose level of 80 mg/dL B) Use of anticoagulant medication C) History of previous heart attack D) Allergy to aspirin and penicillin Answer: B Diff: 2 Page Ref: 543-544 Objective: 17-19

15 Copyright © 2018 Pearson Education, Inc.

58) Which of these statements made by the patient should cause the EMT to suspect an abdominal aortic aneurysm? A) "I have belly pain that I also feel in my back." B) "My heart is beating so hard, you can feel it on my chest." C) "The pain is sharp and spreads to both arms." D) "I seem to be vomiting up some blood." Answer: A Diff: 2 Page Ref: 534 Objective: 17-12 59) A patient has called 911 for chest pain that he describes as "tearing" and radiating between his shoulders. His airway is patent, breathing accelerated but adequate, and radial pulse intact. Your partner reports the following vital signs: pulse, 120; respirations, 22 breaths/min; blood pressure, 88/64 mmHg; and SpO2, 96% on room air. At this time, you should: A) Assist the patient with his nitroglycerin B) Provide the patient with 325 mg of aspirin C) Encourage the patient to take an antacid such as Maalox D) Recheck the blood pressure in the other arm Answer: D Diff: 2 Page Ref: 534 Objective: 17-12 60) A patient complaining of shortness of breath and dizziness has called 911. Her history includes chronic obstructive pulmonary disease (COPD). When assessing the patient, which of these findings would cause you to immediately prepare the patient for transport? A) SpO2 of 94% despite supplemental oxygen B) Wheezing noted in the bases of both lungs C) Heart rate of 84 beats/min D) Pulsating mass in the abdomen Answer: D Diff: 2 Page Ref: 534 Objective: 17-12 61) A patient informs you that he has a history of an abdominal aortic aneurysm. To best assess the status of this condition, you should: A) Auscultate the lower abdominal quadrants B) Assess the abdomen C) Examine the back for bruising D) Palpate the flanks and back Answer: B Diff: 2 Page Ref: 533-534 Objective: 17-12

16 Copyright © 2018 Pearson Education, Inc.

62) For a patient with a history of an unstable abdominal aortic aneurysm, the EMT should recognize and prepare for: A) Vomiting and airway occlusion B) Hypoxia caused by lung compression C) Massive internal bleeding D) Decreased perfusion to the upper extremities Answer: C Diff: 2 Page Ref: 533 Objective: 17-12 63) You are treating a patient who complains of altered mental status, shortness of breath, and chest pain. A paramedic has just administered a medication that causes the sympathetic nervous system to increase its influence in the body. Which finding should you anticipate after this treatment? A) Increased heart rate B) Decreased blood pressure C) Decreased heart rate D) Increased heart rate and decreased blood pressure Answer: A Diff: 3 Page Ref: 522 Objective: 17-2 64) After blood enters the right ventricle, it next moves into the: A) Left ventricle B) Pulmonary artery C) Left atrium D) Aorta Answer: B Diff: 1 Page Ref: 522 Objective: 17-2 65) All arteries: A) Carry blood away from the heart B) Have lower pressure than veins C) Carry oxygen-rich blood D) Are located in the torso of the body Answer: A Diff: 1 Page Ref: 524 Objective: 17-2

17 Copyright © 2018 Pearson Education, Inc.

66) Which sign or symptom should the EMT expect in a patient who has a chronically low white blood cell count? A) Bruising to the extremities B) SpO2 of 91% on room air C) Blood pressure of 156/100 mmHg D) Temperature of 103.4°F Answer: D Diff: 2 Page Ref: 527 Objective: 17-2 67) The three primary components of the cardiovascular system are best described as: A) Arteries, veins, and capillaries B) Heart, lungs, and blood vessels C) Heart, blood vessels, and blood D) Heart, blood vessels, and brainstem Answer: C Diff: 1 Page Ref: 521 Objective: 17-2 68) The primary pacemaker site of the heart is the: A) Sinoatrial node B) Brainstem C) Right atrium D) Ventricular node Answer: A Diff: 1 Page Ref: 521 Objective: 17-3 69) The primary control or automaticity of the heart rate originates in the: A) Brainstem B) Heart C) Nervous system D) Endocrine system Answer: B Diff: 1 Page Ref: 521 Objective: 17-3 70) Dysfunction of the heart's electrical conduction cells from ischemia or infarction may cause: A) Blood loss B) Lung collapse C) Hyperventilation D) Irregular heartbeats Answer: D Diff: 2 Page Ref: 528 Objective: 17-5

18 Copyright © 2018 Pearson Education, Inc.

71) A paramedic administers a medication to decrease the effect of the parasympathetic nervous system on the heart. In response to this treatment, the EMT would expect which vital sign change? A) Increase in respirations B) Decrease in heart rate C) Decreased blood pressure D) Increase in heart rate Answer: D Diff: 3 Page Ref: 522 Objective: 17-2 72) Sympathetic nervous system stimulation will cause the heart to: A) Lower the systolic blood pressure B) Increase the heart rate C) Decrease automaticity D) Decrease in rate Answer: B Diff: 3 Page Ref: 522 Objective: 17-2 73) Which of these identifies the major elements of the pathway that blood takes through the heart in the correct sequence? A) Aorta, left atrium, left ventricle, pulmonary capillary B) Pulmonary vein, pulmonary capillary, pulmonary artery, right ventricle C) Vena cava, right ventricle, pulmonary artery, left ventricle, aorta D) Right atrium, left atrium, left ventricle, right ventricle Answer: C Diff: 2 Page Ref: 522-523 Objective: 17-2 74) An obstruction of the pulmonary artery will cause: A) Backup of blood in the right side of the heart B) Increased blood flow through the aorta C) Backup of blood in the left side of the heart D) Increased blood flow through the vena cava Answer: A Diff: 3 Page Ref: 522 Objective: 17-2 75) Chronic and untreated hypertension will affect which part of the heart first? A) Left ventricle B) Aorta C) Right atrium D) Vena cava Answer: A Diff: 2 Page Ref: 536 Objective: 17-6 19 Copyright © 2018 Pearson Education, Inc.

76) A patient who is chronically hypoxic because of anemia most likely has a problem with which component of the cardiovascular system? A) Platelets B) Red blood cells C) Pulmonary arteries D) White blood cells Answer: B Diff: 2 Page Ref: 527 Objective: 17-2 77) Which of these statements regarding contraction of the heart is true? A) The two ventricles contract simultaneously, moving blood into the atria B) The right atria and ventricle contract together, moving blood to the left side C) Both atria contract together just before both ventricles contract together D) When the left ventricle contracts, it sends blood into the lungs for oxygenation Answer: C Diff: 3 Page Ref: 527 Objective: 17-2 78) The EMT understands the ECG when he makes which of these statements about the waveform? A) "The ECG shows how well the heart is oxygenating blood." B) "An ECG indicates the amount of blood in the heart." C) "The ECG can be used to determine how well the heart is pumping blood." D) "An ECG represents the electrical activity of the heart." Answer: D Diff: 3 Page Ref: 528 Objective: 17-4 79) On an ECG, the QRS represents: A) Contraction of the atria B) Contraction of the ventricles C) Contraction of both the atria and the ventricles D) The heart at rest following contraction Answer: B Diff: 3 Page Ref: 528 Objective: 17-4 80) The EMT understands diastolic blood pressure when he defines it as: A) "The pressure in the arteries between cardiac contractions." B) "The pressure in the arterial system when the heart contracts." C) "The pressure needed to move blood through the cardiovascular system." D) "The pressure in the venous system when the heart is at rest." Answer: A Diff: 3 Page Ref: 529 Objective: 17-2 20 Copyright © 2018 Pearson Education, Inc.

81) A patient is very dizzy and has an altered mental status. Vital signs are pulse, 84; respirations, 16 breaths/min; blood pressure, 74/32 mmHg; and SpO2, 93%. Medical direction has ordered the paramedic to start an IV infusion of a medication that causes blood vessel constriction. You would recognize that the medication is benefiting the patient when you observe: A) The heart rate increase to 92 beats/min B) The respirations increase to 24 breaths/min C) The blood pressure increase to 110/64 mmHg D) The SpO2 increase to 94% Answer: C Diff: 3 Page Ref: 529 Objective: 17-6 82) A 67-year-old male patient with a history of coronary artery disease and COPD called 911 for chest discomfort and mild dyspnea. When you are at the patient's side, he states that the pain started when he was cleaning out his garage, but after a period of rest, the pain has resolved. His airway is patent, breathing adequate, radial pulse irregular, and skin warm and dry. He has a past medical history of atrial fibrillation (irregular heartbeat), coronary artery disease, and diabetes, for which he takes insulin. Vital signs are pulse, 68; respirations, 16 breaths/min; blood pressure, 148/88 mmHg; and SpO2, 97% on room air. The patient took 81 mg of aspirin as prescribed this morning on awakening. What should you do first? A) Administer another aspirin B) Assess breath sounds C) Assist the patient with taking nitroglycerin D) Provide supplemental oxygen Answer: B Diff: 2 Page Ref: 544 Objective: 17-18 83) To save additional heart muscle from dying when a patient suffers an acute myocardial infarction (heart attack), which treatment must occur? A) Increasing the heart rate B) Lowering the blood pressure C) Relieving the obstruction in the coronary arteries D) Opening blood flow between the atria and ventricles Answer: C Diff: 2 Page Ref: 543 Objective: 17-19 84) Which of these patients is most likely to suffer a silent MI? A) A 55-year-old female with diabetes B) A 72-year-old male with a syncopal episode C) A 60-year-old female with colon cancer D) A 48-year-old male with undiagnosed hypertension Answer: A Diff: 3 Page Ref: 532 Objective: 17-13 21

Copyright © 2018 Pearson Education, Inc.

85) An elderly patient with diabetes who experiences a silent MI will most likely: A) Be on continuous home oxygen B) Experience chest pain but deny that anything is wrong C) Be taking pain medications that mask the heart pain D) Not be awakened by a heart attack as he sleeps Answer: D Diff: 3 Page Ref: 544 Objective: 17-17 86) When assessing an elderly patient with upper back pain, which of these assessment findings would make you very suspicious of a thoracic aortic dissection? A) Lower back pain described as a constant ache B) Dizziness when going from a supine to standing position suddenly C) Different blood pressure readings in each arm D) History of hypertension and lower back injury Answer: C Diff: 3 Page Ref: 534 Objective: 17-17 87) A 71-year-old male patient with a cardiac history and lower abdominal and back pain has called 911 for help. On scene, you detect a large pulsatile mass in his abdomen. With the exception of skin that is diaphoretic, the primary assessment indicates no life threats to the airway, breathing, or circulation. The patient has a history of hypertension, high cholesterol, and diabetes, and is allergic to sulfa-based medications. Vital signs are pulse, 92/min; respirations, 20 breaths/min; blood pressure, 130/66 mmHg; and SpO2, 95% on room air. Which of these statements made by the patient would make you intervene immediately? A) "Since I am not having chest pain, I am not going to take my nitroglycerin." B) "I am going to check my blood sugar with my own glucometer." C) "I am going to leave my home oxygen on 2 liters since it seems to have helped my breathing." D) "I am going to take my daily aspirin since I did not take it this morning." Answer: D Diff: 3 Page Ref: 534 Objective: 17-12 88) Regarding women and cardiac compromise, with which of these statements would you agree? A) "The death rate for women suffering a heart attack is higher than that for men." B) "Although the risk of MI decreases with age, women should take daily aspirin as they get older." C) "Most women, unless they are diabetic, will suffer from chest discomfort as opposed to pain when having an MI." D) "Most women have right-sided chest pain without dyspnea or palpitations when having an MI." Answer: A Diff: 2 Page Ref: 534 Objective: 17-13 22 Copyright © 2018 Pearson Education, Inc.

89) A 56-year-old female patient with a smoking history and COPD complains of the sudden onset of shortness of breath and diaphoresis. Her history indicates no cardiac diagnosis but previous drug abuse 8 years ago. Vital signs are pulse, 96; respirations, 18 breaths/min and mildly labored; blood pressure, 168/102 mmHg; and SpO2, 96% on home oxygen at 4 lpm, which she wears for COPD. The patient's lung sounds are clear but diminished bilaterally. What is your best action? A) Administer aspirin, monitor the patient, and transport B) Turn down the oxygen to 2 lpm, monitor, and transport C) Administer the patient's MDI and transport her to the hospital D) Assist the patient is trying her husband's nitroglycerin Answer: B Diff: 3 Page Ref: 535 Objective: 17-14 90) You are approached by an EMR who informs you that last week, he was on a call in which a patient with chest pain and a known history of coronary artery disease was not given oxygen by the EMTs. You would reply: A) "It has been shown that oxygen given to a patient with an acceptable pulse oximetry reading can be harmful." B) "That may have been an oversight, given that chest pain is caused by a lack of oxygen-rich blood delivery through the coronary arteries." C) "Since extra oxygen slows the respiratory rate, it also causes the body to retain CO2, which is not beneficial for heart patients." D) "If the patient also had a history of COPD, oxygen was most likely withheld since it would be contraindicated." Answer: A Diff: 2 Page Ref: 535 Objective: 17-14 91) On scene, you are treating a 62-year-old female patient with chest pain, diaphoresis, and nausea. She has a history of heart problems as well as diabetes, for which she takes medications. She is allergic to penicillin and iodine. She is alert, oriented, and calm, and there are no threats to the airway, breathing, or circulation. The patient took 81 mg of aspirin this morning, as she always does. Her vital signs are pulse, 100; respirations, 16 breaths/min; blood pressure, 118/90 mmHg; and SpO2, 97% on room air. In overseeing the care of this patient, which of these actions would demand immediate and corrective intervention? A) Palpating the chest wall for tenderness B) Applying oxygen through a nasal cannula C) Determining the patient's blood glucose D) Administering three baby aspirins to the patient Answer: B Diff: 2 Page Ref: 535 Objective: 17-14

23 Copyright © 2018 Pearson Education, Inc.

92) When administering oxygen to a patient with chest pain, which of these should be used a guide to its use and maintenance? A) Patient rating of pain B) The respiratory rate C) Pulse oximetry reading D) The heart rate Answer: C Diff: 2 Page Ref: 535 Objective: 17-14 93) On room air, which of these patients requires supplemental oxygen? A) 66-year-old female; generalized weakness; history of diabetes; SpO2, 97% B) 52-year-old male; chest pain; history of MI; SpO2, 95% C) 39-year-old male; complaint of tingling in hands; headache; respirations, 30 breaths/min; SpO2, 98% D) 47-year-old female; temperature 100.3°F; denies dyspnea; SpO2, 91% Answer: D Diff: 2 Page Ref: 535 Objective: 17-14 94) You are treating a 67-year-old male patient with chest pain and a history of prior heart attacks as well as diabetes and hypertension. The patient has taken three nitroglycerin tablets, which have decreased his pain from 7/10 to 4/10; however, the pain persists. Initially, his SpO2 was 91% on room air. Oxygen was applied by nasal cannula and now, on 3 lpm O2, his SpO2 is 97%. How would you address the oxygen administration, given that the patient still has chest pain? A) Increase the oxygen flow to 4 lpm B) Decrease the oxygen flow to 2 lpm C) Leave the oxygen flow unchanged D) Discontinue oxygen therapy Answer: B Diff: 2 Page Ref: 535 Objective: 17-14 95) A patient with chest pain has an SpO2 of 90% on room air. How would you start providing oxygen to this patient? A) Nasal cannula 2 lpm B) Nasal cannula 4 lpm C) Nasal cannula 6 lpm D) Nonrebreather 15 lpm Answer: A Diff: 2 Page Ref: 535 Objective: 17-14

24 Copyright © 2018 Pearson Education, Inc.

96) Which of these therapies is the best option for definitively treating a known myocardial infarction (AMI)? A) Oxygen B) Fibrinolytics C) Oxygen, aspirin, and nitroglycerin D) Angioplasty Answer: D Diff: 2 Page Ref: 543 Objective: 17-19 97) Which of these scene size-up observations would make you suspicious that a patient who complains of shortness of breath has congestive heart failure? A) Dramatic edema to the right arm B) Home oxygen tank in the bedroom C) Multiple pillows stacked at the head of the patient's bed D) Patient in dirty pajamas and appears unkempt Answer: C Diff: 3 Page Ref: 549 Objective: 17-9 98) A patient informs you that he has a hard time breathing when he lies in bed. You would best document this report as: A) Dyspnea B) Tachypnea C) Nocturnal dyspnea D) Orthopnea Answer: D Diff: 1 Page Ref: 536 Objective: 17-17 99) What are the protein strands that strengthen a clot called? A) Thrombin B) Platelets C) Fibrin D) Plaque Answer: C Diff: 1 Page Ref: 527 Objective: 17-1

25 Copyright © 2018 Pearson Education, Inc.

100) A 64-year-old male patient presents with a blood pressure of 210/132 mmHg. Which of these statements made by this patient would you recognize as most concerning and most likely associated with the elevated blood pressure? A) "I have a history of left-sided heart failure." B) "I feel very thirsty right now." C) "My hair has become very brittle." D) "I have this bruising on my arms and legs." Answer: A Diff: 3 Page Ref: 523 Objective: 17-15 101) You have been called for a 4-year-old female patient who told her mother that her chest hurts. On scene, the child is crying and points to her chest when asked if she has pain. Her airway is patent, respirations slightly labored, pulse rapid and regular, and skin warm and pale. Vital signs are pulse, 132; respirations, 28 breaths/min; blood pressure, 108/74 mmHg; and SpO2, 89%. The mother denies any medical history, medications, or allergies for her daughter. Your first action would be to: A) Contact medical direction B) Apply oxygen C) Give the patient one-half nitroglycerin tablet D) Administer one baby aspirin Answer: B Diff: 3 Page Ref: 539 Objective: 17-17 102) A mother has called 911 because her 6-year-old son told her he had chest pain. On scene, the mother informs you that her son suffers from a congenital heart defect called aortic coarctation. The boy admits to chest pain and appears distressed. His airway is patent, breathing adequate, and radial pulse present and regular. Vital signs are pulse, 108; respirations, 24 breaths/min; blood pressure, 148/92 mmHg; and SpO2, 98%. Your next action would be to: A) Give one baby aspirin B) Try one nitroglycerin for pain relief C) Contact medical command for advice D) Administer high-concentration oxygen Answer: C Diff: 3 Page Ref: 539 Objective: 17-17 103) Failure to promptly treat ventricular tachycardia can result in myocardial cell hypoxia and ischemia, which then contributes to the rhythm deteriorating to: A) Ventricular fibrillation B) Pulseless electrical activity C) Sinus rhythm D) Asystole Answer: A Diff: 2 Page Ref: 529 Objective: 17-5 26 Copyright © 2018 Pearson Education, Inc.

104) You have been called to a residence for an obese 52-year-old male patient who complains of chest pain. When obtaining a history from the patient, which of these statements made by the patient would you find most concerning? A) "I started taking an antianxiety medication last week." B) "The pain came on suddenly when I was reading the paper." C) "I have had chest pain on and off for years now." D) "I occasionally feel that my heart seems to skip a beat." Answer: B Diff: 3 Page Ref: 530-531 Objective: 17-7 105) Which sign or symptom would the EMT typically associate with congestive heart failure? A) Edema to the left arm B) Feeling of palpitations C) Rhonchi to both lungs that is cleared by a cough D) Crackles in both lungs Answer: D Diff: 2 Page Ref: 536 Objective: 17-9 106) Which structure of the neck would the EMT examine on a patient she believes to be in congestive heart failure? A) Trachea B) Neck veins C) Lateral muscles D) Clavicles Answer: B Diff: 1 Page Ref: 536 Objective: 17-9

27 Copyright © 2018 Pearson Education, Inc.

Prehospital Emergency Care, 11e (Mistovich et al.) Chapter 18 Altered Mental Status, Stroke, and Headache 1) Which of these conditions would the EMT recognize as most likely signaling a neurologic deficit? A) Hypoglycemia B) Paralysis C) Constricted pupils D) Headache Answer: B Diff: 2 Page Ref: 559-560 Objective: 18-1 2) Which of these conditions places the patient at the greatest risk for an acute neurologic deficit? A) Acute myocardial infarction (AMI) B) Transient ischemic attack (TIA) C) Chronic obstructive pulmonary disease (COPD) D) New onset of type 2 diabetes (NIDDM) Answer: B Diff: 1 Page Ref: 567 Objective: 18-9 3) When assessing a patient with a possible stroke, what is the priority? A) Identifying risk factors for stroke B) Determining a family history of stroke C) Recognizing the signs of a stroke D) Determining the type of stroke Answer: C Diff: 2 Page Ref: 560 Objective: 18-7 4) While you are teaching a public awareness class about stroke, a participant asks you if all patients with stroke are paralyzed on one side of their bodies. Your reply would be: A) "Yes. Both the arm and the leg must be paralyzed for the health professional to recognize a stroke." B) "No. Some patients suffering stroke will show no signs or symptoms, but all still must be evaluated." C) "Yes. To be diagnosed with a stroke, the arm and/or the leg must be very weak and paralyzed." D) "No. Some stroke patients may have only slight weakness to one side of their bodies." Answer: D Diff: 3 Page Ref: 560 Objective: 18-7

1 Copyright © 2018 Pearson Education, Inc.

5) When assessing a patient with a suspected stroke, the order of the exam would be: A) Primary assessment, then secondary assessment including a stroke scale B) Level of consciousness, facial droop, arm drift, airway, and then secondary assessment C) Abnormal speech, facial droop, arm drift, primary assessment, and then secondary assessment D) Secondary assessment, primary assessment, stroke exam, and then SAMPLE history Answer: A Diff: 1 Page Ref: 568-570 Objective: 18-3 6) When using a stroke scale to assess a patient for abnormal speech, the EMT should: A) Observe verbal ability during assessment B) Ask the patient to repeat a specific but common phrase C) Question family members about the patient's ability to talk D) Write questions on note cards for the patient to read Answer: B Diff: 1 Page Ref: 571, Figure 18-7 Objective: 18-11 7) When assessing a patient for a facial droop, the EMT should: A) Palpate the face one side at a time B) Gently lift and then let go of the skin over the cheeks C) Ask the patient to show you his teeth or smile D) Observe the patient's ability to turn his head to the right and then to the left Answer: C Diff: 1 Page Ref: 571, Figure 18-7 Objective: 18-11 8) Which one of these instructions to a patient with possible stroke indicates proper assessment for an arm drift? A) "Hold out your arms and count to 10 while keeping your eyes open." B) "Grab my fingers and squeeze with both hands as hard as you can for 5 seconds, and I mean squeeze hard." C) "Raise your hands above your head and close your eyes while counting backward from 20." D) "Close your eyes and hold your arms straight out for 10 seconds." Answer: D Diff: 2 Page Ref: 571, Figure 18-7 Objective: 18-11 9) Which question would it be critical that the EMT ask the family of a patient who cannot speak and is suspected of having a stroke? A) "Does your mother have a history of heart problems?" B) "What time did the weakness and dizziness start?" C) "Which medications is your mother taking?" D) "Does your mother have any pain in her arms or legs?" Answer: B Diff: 2 Page Ref: 574 Objective: 18-12 2 Copyright © 2018 Pearson Education, Inc.

10) You have determined that an alert elderly male patient has garbled speech and weakness to the right arm and leg. His vital signs and SpO2 level are within normal limits. The family informs you that he has a history of heart problems and diabetes. Given this scenario, what should you do immediately? A) Administer oral glucose or sugar water B) Attach the automated external defibrillator (AED) C) Check the patient's blood glucose level (if local protocol allows) D) Administer oxygen via a nasal cannula at 2 lpm Answer: C Diff: 2 Page Ref: 577 Objective: 18-12 11) With the Cincinnati Prehospital Stroke Scale, which of these patient parameters is assessed? A) Blood glucose level B) Facial droop C) Time of symptom onset D) Age of the patient Answer: B Diff: 1 Page Ref: 570 Objective: 18-11 12) When attempting to distinguish a spinal injury from a stroke, the EMT should remember that: A) Spinal injuries often result in weakness to one side of the body B) Stroke typically results in weakness or paralysis to one side of the body C) Stroke does not cause paralysis, while spinal injuries typically do D) Stroke usually causes weakness to all extremities; spinal injuries do not Answer: B Diff: 3 Page Ref: 569 Objective: 18-3 13) Assessment of a 63-year-old female patient reveals a patent airway, adequate breathing, and strong radial pulse. She has no difficulty speaking and no facial droop, but exhibits a slight left arm drift and has a headache. The EMT should interpret these specific findings as: A) Not suggestive of a stroke B) Minimally suggestive that a stroke may be occurring C) Moderately suggestive that a stroke may be occurring D) Highly suggestive that a stroke may be occurring Answer: D Diff: 2 Page Ref: 570 Objective: 18-11

3 Copyright © 2018 Pearson Education, Inc.

14) You are called for a 69-year-old male patient with altered mental status. Assessment reveals that he is confused and has left-sided weakness. During transport, you note that the patient is now completely oriented and the weakness has resolved. Which condition would you suspect this patient has? A) Stroke B) Low blood sugar C) Transient ischemic attack D) Syncope Answer: C Diff: 2 Page Ref: 567 Objective: 18-10 15) A 49-year-old male patient is unable to speak. You would document this finding as: A) Apnea B) Dysarthria C) Dysplegia D) Aphasia Answer: D Diff: 1 Page Ref: 566 Objective: 18-1 16) Which of these statements made by your new EMT partner demonstrates an understanding of assessment findings related to stroke? A) "For a stroke to be suspected, both arms must be equally weak." B) "If one arm is weak, a stroke should be suspected." C) "A stroke should be suspected only when one arm is paralyzed, not weak." D) "If one arm is weak, stroke should be suspected only if the blood pressure is also elevated." Answer: B Diff: 3 Page Ref: 570 Objective: 18-7 17) When a patient smiles, the left side of the face shows significantly less movement than the right side of the face. The EMT would describe this finding as: A) Left-side facial droop B) Right-side facial droop C) Left-side facial paralysis D) Right-side facial paralysis Answer: A Diff: 1 Page Ref: 569, Figure 18-5 Objective: 18-12

4 Copyright © 2018 Pearson Education, Inc.

18) Which of these patients would be most eligible to receive a fibrinolytic medication for stroke if no other contraindications are found? A) A 37-year-old female diagnosed in the emergency department with a hemorrhagic stroke 2 hours after its onset B) A 48-year-old male whose symptoms started while watching the 6 P.M. news, at which time he dialed 911 C) An 81-year-old male who awoke with stroke symptoms after an 8-hour sleep D) A 62-year-old female whose stroke symptoms started at 8 A.M. and disappeared half an hour later Answer: B Diff: 3 Page Ref: 561 Objective: 18-12 19) For the patient with stroke to be eligible for a fibrinolytic agent, the therapy must be given within: A) 3 hours of EMS arrival at the scene B) 4 hours after the diagnosis of stroke is officially made C) 3 hours of the onset of stroke symptoms D) 4 hours of the first signs and/or symptoms started Answer: C Diff: 2 Page Ref: 561 Objective: 18-12 20) For the patient with stroke to be eligible for a fibrinolytic agent, the patient must be brought to the hospital emergency department within how many hours of the onset of stroke symptoms? A) 2 B) 3 C) 6 D) 24 Answer: A Diff: 2 Page Ref: 561 Objective: 18-12

5 Copyright © 2018 Pearson Education, Inc.

21) You have been called for a 63-year-old woman with slurred speech and right-arm weakness. On arrival, the patient informs you that the slurred speech and weakness have resolved. Since your assessment reveals no deficits, the patient insists on signing a refusal of treatment form. Which of these statements should you make prior to having her sign the form? A) "It would be best to start taking one baby aspirin every day; this will decrease the chance of this happening again." B) "Sometimes seizures present this way; call your doctor in the morning to schedule an appointment." C) "If this happens again, wait 10 minutes to see if you are okay. If the symptoms persist, call 911." D) "You really need to be evaluated in the hospital. You are at significant risk for a future stroke, which may be debilitating or even fatal." Answer: D Diff: 3 Page Ref: 568 Objective: 18-12 22) Assessment reveals a drooling 48-year-old male patient to have gurgling speech and left-arm paralysis. The EMT's immediate priority treatment is to: A) Suction the airway B) Protect the left arm C) Determine risk factors for stroke D) Determine the time of onset Answer: A Diff: 2 Page Ref: 568 Objective: 18-12 23) A patient who is responsive only to painful stimuli has a patent airway, inadequate respirations at 28 breaths/min and irregular, and a strong radial pulse with skin that is warm and dry. The patient is not moving his right arm or leg and is incontinent of urine. What is the priority for the EMT at this time? A) Insert an oropharyngeal airway B) Administer oxygen by nonrebreather mask C) Administer positive pressure ventilation D) Check the blood glucose level Answer: C Diff: 3 Page Ref: 568-569 Objective: 18-12

6 Copyright © 2018 Pearson Education, Inc.

24) A 73-year-old female patient is responsive to painful stimuli and cannot move her arm or leg on one side of her body. You suspect she is having a stroke. What is the best transport position for this patient? A) Supine with head elevated 30 degrees B) Left lateral recumbent position C) Supine with left arm and leg flexed D) Prone with left arm and leg flexed Answer: B Diff: 2 Page Ref: 576 Objective: 18-12 25) An alert and oriented 69-year-old male patient has slight right-arm weakness but exhibits no deficits to the right leg. The EMT should consider this patient to be a: A) Non-priority transport since no paralysis exists B) Priority only if the patient also has a low blood glucose level C) Non-priority since he is alert and oriented D) Priority for transport to a hospital with a stroke center Answer: D Diff: 2 Page Ref: 577 Objective: 18-12 26) When transporting an alert but nonverbal patient with stroke and a blood pressure of 210/110 mmHg, the EMT should: A) Dim the lights and remain out of the patient's sight to avoid unnecessary stimulation B) Touch the patient only when absolutely necessary, such as when obtaining vital signs C) Make an effort to explain everything that is happening or being done while trying to help the patient feel at ease D) Remain in the patient's sight but avoid talking or stimulating the patient Answer: C Diff: 2 Page Ref: 577 Objective: 18-12 27) Patients with an altered mental status due to a stroke may not be able to control their own airways because of: A) Enlargement and swelling of the epiglottis B) Paralysis or weakness of the throat muscles C) Hypersensitivity of the gag reflex D) Increased oral secretions Answer: B Diff: 2 Page Ref: 568 Objective: 18-12

7 Copyright © 2018 Pearson Education, Inc.

28) Which of these is a good description of an ischemic stroke? A) Rupture of an artery within the brain B) Blood accumulation within the brain tissue C) Simultaneous occurrence of stroke and heart attack D) Obstructive blood clot inside a blood vessel within the brain Answer: D Diff: 3 Page Ref: 563 Objective: 18-8 29) The EMT demonstrates she understands hemorrhagic stroke when she states: A) "It occurs when clots form in blood vessels within the brain." B) "It occurs when a blood vessel ruptures and bleeds within the brain." C) "Its signs and symptoms generally resolve within 24 hours." D) "It can sometimes be treated by clot-destroying medications." Answer: B Diff: 3 Page Ref: 564 Objective: 18-8 30) A patient with stroke cannot move his left arm or leg. Based on this presentation, the EMT should strongly suspect that the stroke has occurred in which part of the central nervous system? A) Left side of the brain B) Left side of the spinal cord C) Right side of the brain D) Right and left sides of the brain Answer: C Diff: 2 Page Ref: 566 Objective: 18-7 31) A patient has an altered mental status accompanied by loss of motor function on one side of the body. The EMT must suspect which condition as the cause of this presentation? A) Seizure B) Hypoglycemia C) Stroke D) Syncope Answer: C Diff: 1 Page Ref: 566 Objective: 18-7 32) When assessing a patient with stroke-like signs and symptoms, it is critical that the EMT also suspect and rule out which condition? A) Narcotic overdose B) Syncope C) Hypoglycemia D) Shock Answer: C Diff: 2 Page Ref: 573 Objective: 18-12 8 Copyright © 2018 Pearson Education, Inc.

33) Which one of these describes the typical signs and symptoms of a transient ischemic attack (TIA)? A) Temporary, with many TIAs lasting less than 15 minutes B) Persistent, with most TIAs lasting more than 1 hour but less than 24 hours C) Mild, but permanent D) Severe and debilitating Answer: A Diff: 1 Page Ref: 567 Objective: 18-10 34) You have gathered and written down all of the medications, including narcotic pain medications, that a confused 46-year-old male patient currently takes. The patient is placed on the stretcher for transport. At this time, how would you handle the medications? A) Return them to their original places B) Give them to a family member C) Turn them over to the police D) Transport them with the patient Answer: D Diff: 1 Page Ref: 557 Objective: 18-3 35) You have been called to the house of a young adult male patient who is unresponsive. The man lives by himself, so no family is present. In trying to determine the origin of the patient's altered mental status, which of these would most likely provide the best information, assuming it is available to you? A) Contact with medical direction B) Examination of patient medications (prescribed and recreational) C) Interpretation of physical assessment findings D) Observation of the living conditions in the house Answer: B Diff: 2 Page Ref: 557 Objective: 18-3 36) Which of these statements shows that the EMT has an accurate understanding of altered mental status? A) "It describes any change away from a normal state of mental function." B) "It applies only when the patient is not responsive to verbal or painful stimuli." C) "It should be suspected only when the patient is on medications that can cause confusion." D) "It is any mental state experienced by a person with diabetes or other serious chronic disease." Answer: A Diff: 1 Page Ref: 556 Objective: 18-1

9 Copyright © 2018 Pearson Education, Inc.

37) When performing a secondary assessment on an elderly male patient who is confused and combative, which of these assessment findings should be of most concern to the EMT? A) Respirations of 20 breaths/min B) Pulse of 164 C) Bruising to the back of the hand D) Pulse oximeter reading of 94% on room air Answer: B Diff: 2 Page Ref: 558 Objective: 18-3 38) What does the first "I" in the AEIOU-TIPPSS mnemonic stand for? A) Influenza B) Infection C) Insulin D) Ischemia Answer: C Diff: 2 Page Ref: 559, Table 18-1 Objective: 18-5 39) Assessment of a 56-year-old male patient reveals him to be responsive to verbal stimuli with garbled speech. Which of these statements made by the patient's wife suggests that the patient's problem may be toxic-metabolic? A) "He has bad kidneys." B) "He had a stroke two years ago." C) "He fell and hit his head yesterday." D) "He was diagnosed last year with Alzheimer's disease." Answer: A Diff: 3 Page Ref: 556 Objective: 18-2 40) You have been called for a patient who suddenly became confused. Which of these assessment findings should concern you most? A) Blood pressure of 110/72 mmHg B) Pupils fully dilated and sluggishly reactive to light C) Heart rate of 180 beats/min but regular D) Blood glucose level of 72 mg/dL Answer: C Diff: 3 Page Ref: 558 Objective: 18-3

10 Copyright © 2018 Pearson Education, Inc.

41) A middle-aged male patient collapsed while shopping and now responds to verbal stimuli with confused speech. The primary assessment indicates no immediate life-threatening conditions. His pulse is 96, respirations 16 breaths/min, blood pressure 158/88 mmHg, SpO2 98% on room air, and blood glucose level 79 mg/dL. The patient is carrying a vial of nitroglycerin in his jacket pocket. What should you do next after determining the patient priority status? A) Administer oral glucose B) Provide oxygen via nasal cannula at 2 lpm C) Administer sublingual nitroglycerin D) Perform a secondary assessment Answer: D Diff: 3 Page Ref: 557-558 Objective: 18-3 42) You are obtaining the past medical history from the family of an elderly patient with an altered mental status who suffered sudden onset of right-side weakness. Which of these statements made by the patient's family member should you recognize as increasing the patient's risk of stroke? A) "She takes an aspirin every morning." B) "Her blood pressure runs low." C) "She has a history of migraines." D) "She has an irregular heartbeat." Answer: D Diff: 3 Page Ref: 564 Objective: 18-4 43) An 82-year-old male patient presents with right-hand weakness and garbled speech. Which question should the EMT ask first to the patient or a reliable family member who is present? A) "Is he allergic to anything?" B) "Does he take any blood thinners?" C) "Does he have a history of high blood pressure?" D) "What time did these signs start?" Answer: D Diff: 2 Page Ref: 574 Objective: 18-12 44) You have just arrived by the side of a 46-year-old male patient who receives dialysis for endstage renal failure. He is positioned in his living room recliner, unresponsive with snoring respirations, and appears to have ineffective ventilations. What is your priority intervention at this time? A) Provide mechanical ventilations B) Manually open the airway C) Find out when he last received dialysis D) Inquire when the symptoms started Answer: B Diff: 2 Page Ref: 557 Objective: 18-3 11 Copyright © 2018 Pearson Education, Inc.

45) A 78-year-old male patient was found lying on his bathroom floor. Emergency Medical Responders report that the patient is unresponsive, with unequal pupils, and vital signs are as follows: pulse, 78; respirations, 20 breaths/min and snoring; blood pressure, 210/106 mmHg; SpO2, 99%. The EMRs have applied oxygen by nonrebreather mask, and also report that the patient has a hematoma on the back of his head. What should you do first? A) Replace the nonrebreather mask with a nasal cannula B) Quickly apply a cervical collar C) Ensure an open airway with a jaw-thrust maneuver D) Lift the patient onto the stretcher for transport Answer: C Diff: 3 Page Ref: 557 Objective: 18-3 46) Which of these questions would the EMT ask when performing the Los Angeles Prehospital Stroke Screen? A) "Do you take blood thinners?" B) "Are your hand grips weak?" C) "Do you smoke cigarettes?" D) "Have you ever had a seizure?" Answer: D Diff: 2 Page Ref: 570 Objective: 18-11 47) You have been called to a long-term care facility for a 77-year-old female patient who was found unresponsive in her wheelchair by staff. The nurse reports that the patient complained of a headache earlier in the day and was given 600 mg of Motrin. Your assessment shows the patient to be unresponsive with a patent airway, slow and irregular respirations, and a weak radial pulse. Her lips are cyanotic, and your partner reports the following vital signs: pulse, 44; respirations, 6 breaths/min; and blood pressure, 228/116 mmHg. Which action should be your next priority? A) Start positive pressure ventilation B) Check the patient's blood glucose level C) Attach the pulse oximeter to get a SpO2 reading D) Perform a prehospital stroke screen Answer: A Diff: 3 Page Ref: 557 Objective: 18-3 48) You are treating an unresponsive patient. Which piece of information given to you by the patient's family is most important in relation to a possible stroke? A) "He forgot to take his blood thinner last night." B) "We think that he had too much to drink last night." C) "We noticed him acting oddly about 7 P.M." D) "He took some ibuprofen last night for a fever and cough." Answer: C Diff: 2 Page Ref: 574 Objective: 18-12 12 Copyright © 2018 Pearson Education, Inc.

49) A 56-year-old patient is in bed and unable to speak. Family members state that he had a stroke in the past and has been bedridden since then, but has always been able to talk, until a few hours ago when his speech became garbled. They also state that the last stroke left him paralyzed on his left side. Your assessment reveals that the patient is unable to speak, but has an open airway, adequate respirations, and a strong radial pulse. Additionally, you note that he is unable to move his right arm. Vital signs are within normal limits. What would be an appropriate step in the care on this patient? A) Rapid transport to a stroke center B) Suction the mouth and then place an NPA C) Administer positive pressure ventilation D) Insert an oropharyngeal airway Answer: A Diff: 2 Page Ref: 577 Objective: 18-12 50) A male patient suddenly found himself unable to talk and with profound weakness to his right arm and leg. Currently he is responsive to verbal stimuli and has the following vital signs: pulse, 88; respirations 14 breaths/min and adequate; and blood pressure, 210/80 mmHg. When transporting this patient to the hospital, in which position should you place him? A) Immobilized to a long board B) Left lateral recumbent C) Prone with head elevated D) Supine with feet elevated Answer: B Diff: 2 Page Ref: 576 Objective: 18-12 51) The EMT shows that he is performing the Los Angeles Prehospital Stroke Screen (LAPSS) correctly when he asks which of these questions? A) "May I assess your pupils?" B) "How old are you?" C) "Did you lose control of your bladder?" D) "Do you know where you are right now?" Answer: B Diff: 1 Page Ref: 570 Objective: 18-11 52) The EMT shows that he recognizes the benefit of using the Cincinnati Prehospital Stroke Scale or Los Angeles Prehospital Stroke Screen when he makes which of these statements? A) "The screening allows the EMT to identify the patient at risk for stroke." B) "The screening allows the EMT to differentiate the type of stroke a patient is having." C) "The screen is very predictive in determining whether the patient will die from stroke." D) "The screening helps to identify stroke, even when it is not the patient's chief complaint." Answer: D Diff: 2 Page Ref: 570 Objective: 18-11 13 Copyright © 2018 Pearson Education, Inc.

53) Which of these statements made by the EMT indicates a correct understanding of caring for someone with a headache? A) "Unless the patient has some sort of past medical history, a headache is not a true emergency." B) "If a patient calls 911 for a headache, it is most likely a life-threatening condition." C) "Assessment is essential in determining if a patient's headache represents a life-threatening condition." D) "In general, a headache is considered serious only if the pain is felt throughout the entire head." Answer: C Diff: 1 Page Ref: 579 Objective: 18-14 54) A 41-year-old female patient has called 911 for a headache. Which of these assessment findings should make the EMT suspicious that the patient has a migraine headache? A) Bright light increases the patient's discomfort B) The pain was preceded by an aura C) Her blood pressure is 192/124 mmHg D) She rates the pain as an 8/10 Answer: B Diff: 3 Page Ref: 577 Objective: 18-13 55) Which of these statements made by a patient suggests that he is suffering from a tension headache? A) "The pain starts at the top of my head and spreads down my back." B) "I get so nauseated with the pain." C) "The light really seems to bother my eyes when I get these headaches." D) "The pain seems to get worse as the day goes on." Answer: D Diff: 2 Page Ref: 578 Objective: 18-13 56) A patient with a headache informs you that his physician has diagnosed him with cluster headaches. He states that his current pain feels just like the last cluster headache that he suffered. The primary assessment reveals no life threats. His vital signs are pulse, 104; respirations, 16 breaths/min; blood pressure, 124/68 mmHg; and SpO2, 98%. You note no neurologic deficits. Based on this information, what is indicated in the treatment of this patient? A) Massage the patient's neck and obtain a refusal for transport if the pain is alleviated B) Administer supplemental oxygen at 15 liters per minute C) Place the patient on the cot and allow him to assume a position of comfort D) Immediately transport the patient to the hospital for possible stroke Answer: C Diff: 2 Page Ref: 581 Objective: 18-14

14 Copyright © 2018 Pearson Education, Inc.

57) A 30-year-old patient with a history of migraine headaches has called 911 for a suspected migraine headache. She says the headache follows her usual pattern of headaches, the pain is a 10/10, and she is nauseated. Her vital signs are pulse, 108; respirations, 18 breaths/min; and blood pressure, 154/90 mmHg. Which of these treatments is indicated in the care of this patient? A) 81 to 162 mg aspirin by mouth B) Providing a darkened environment during transport C) Emergent transport with lights and sirens D) Requesting orders for sublingual nitroglycerin Answer: B Diff: 2 Page Ref: 581 Objective: 18-14 58) For altered mental status to occur, which body system must be affected? A) Cardiovascular B) Respiratory C) Central nervous D) Endocrine Answer: C Diff: 1 Page Ref: 556 Objective: 18-3 59) A male patient has altered mental status secondary to a brain tumor. The EMT would classify the brain tumor as which type of cause? A) Central B) Structural C) Metabolic D) Internal Answer: B Diff: 1 Page Ref: 556 Objective: 18-2 60) A male patient is confused and combative for no apparent reason. There are no life threats to his airway, breathing, or circulation, and his vital signs are pulse, 88; respirations, 16 breaths/min and adequate; blood pressure, 144/68 mmHg; and SpO2, 93% on room air. When addressing the patient's oxygenation status, which of these treatments would be appropriate? A) Oxygen at 6 lpm via nasal cannula B) Oxygen at 15 lpm via a nonrebreather mask C) Positive pressure ventilation with room air D) Oxygen at 2 lpm via nasal cannula Answer: D Diff: 2 Page Ref: 557 Objective: 18-12

15 Copyright © 2018 Pearson Education, Inc.

61) An elderly male patient with a history of COPD, hypertension, and diabetes presents as lethargic and confused. His airway is patent, and breathing is adequate. His skin is hot and his radial pulse is strong, regular, and bounding. Vital signs are pulse, 112; respirations, 20 breaths/min; blood pressure, 108/64 mmHg; SpO2, 94%; and temperature, 102.8°F. Also noted is swelling to his right lower leg. Given these assessment findings, the EMT should suspect which condition is responsible for the patient's altered presentation? A) Infection B) Peripheral edema C) Hypoxia D) Tachycardia Answer: A Diff: 3 Page Ref: 556 Objective: 18-2 62) On scene, a confused 68-year-old female patient with possible stroke has a SpO2 of 91% on room air. Regarding oxygen therapy and the adjunct by which to provide it, which of these represents the most desirable target SpO2 reading for the amount of oxygen delivered? A) SpO2 98% with 12 lpm O2 via nonrebreather mask B) SpO2 95% with 2 lpm O2 via nasal cannula C) SpO2 100% with 15 lpm O2 via nonrebreather mask D) SpO2 100% with 6 lpm O2 via nasal cannula Answer: B Diff: 2 Page Ref: 576 Objective: 18-12 63) Which of these statements indicates that the EMT understands the appropriate use of supplemental oxygen when caring for a patient with stroke? A) "In the short time that we provide care to a patient with stroke, the benefits of highconcentration oxygen outweigh the risks." B) "If positive pressure ventilation is required, it should be given with room air and not supplemental oxygen." C) "Since stroke decreases oxygen delivery to the brain, high-concentration oxygen should always be geared toward achieving an SpO2 as near to 100% as possible." D) "Oxygen should be administered at a minimal amount, just enough to get the SpO2 at or greater than 94%." Answer: D Diff: 3 Page Ref: 576 Objective: 18-12

16 Copyright © 2018 Pearson Education, Inc.

64) The MEND exam for a stroke does not include which assessment parameter? A) Mental status B) BGL assessment C) Cranial nerve function D) Motor and sensory function of the limbs Answer: B Diff: 2 Page Ref: 570 Objective: 18-11 65) A patient is alert and oriented, with a blood pressure of 228/110 mmHg, a heart rate of 82 beats/min, respirations of 20 breaths/min and adequate, and a pulse oximeter reading of 96% on room air. The patient complains of a bad headache that started suddenly. The patient has equal grips and good lower leg function. The blood glucose is 68 mg/dL, and pupils are reactive, although the patient has trouble maintaining a horizontal gaze when asked to do so. He has a history of seizures, hypertension, and atrial fibrillation. Given these assessment findings, which one meets a criterion for the MEND stroke screening tool that suggests the patient may be experiencing a stroke? A) Sudden onset of a bad headache B) Hypertensive blood pressure C) History of seizures and atrial fibrillation D) Horizontal gaze abnormality Answer: D Diff: 3 Page Ref: 570 Objective: 18-11 66) During a continuing education program, the lecturer is talking about the RACE scale as a newer tool the EMT can use to help determine whether a neurologic problem is present. Which neurologic problem does the RACE scale address? A) Structural versus metabolic cause of unresponsiveness B) Large-vessel occlusion stroke C) Seizures that are due to an organic cause versus metabolic cause D) Stroke stemming from a smaller blood vessel above the circle of Willis Answer: B Diff: 1 Page Ref: 572-573 Objective: 18-11 67) When the EMT evaluates a patient with the RACE stroke scale, the score is 3 points. This score would infer: A) The patient has likely experienced a stroke B) The patient has not likely experienced a stroke C) The stroke scale was incorrectly performed, as the minimum score is at least 5 points D) A stroke has likely not occurred if the corresponding GCS score is 12 points or higher Answer: A Diff: 2 Page Ref: 573 Objective: 18-11

17 Copyright © 2018 Pearson Education, Inc.

68) Rapid recognition of stroke signs and symptoms by the public is part of which assessment system? A) AHA's Stroke Chain of Survival B) S.T.R.O.K.E. assessment C) RACE exam D) AEIOU-TIPPSS Answer: A Diff: 2 Page Ref: 560 Objective: 18-6 69) Which past medical condition identified during the SAMPLE history could be a metabolic cause of an altered mental status in an elderly male patient? A) Brain abscess B) Brain tumor C) Hemorrhagic stroke D) Kidney failure Answer: D Diff: 3 Page Ref: 556 Objective: 18-4 70) Your partner is using the AEIOU-TIPPSS mnemonic to assess for common causes of altered mental status in a geriatric patient. During this process, your partner forgets what the "T" stands for. You would remind him that it means: A) Trauma B) Tumor C) Tachycardia D) Toxicology Answer: A Diff: 2 Page Ref: 559 Objective: 18-5 71) To better fulfill the AHA's Stroke Chain of Survival goals, which assessment mnemonic was developed to better prepare the public and EMS to recognize stroke? A) F.A.S.T B) RACE C) S.T.R.O.K.E. D) AEIOU-TIPPSS Answer: A Diff: 2 Page Ref: 560 Objective: 18-6

18 Copyright © 2018 Pearson Education, Inc.

72) What is the pathophysiology underlying a TIA and its typically rapid resolution? A) Vasodilation of the cerebral blood vessel that was constricted tightly enough to stop blood flow B) Agents in the bloodstream bust apart the cerebral clot that is causing the signs and symptoms C) Blood flow from nearby vascular beds start to perfuse those regions cut off by the TIA ministroke D) Neurons from the areas surrounding the TIA's origin start to assume the functions of the brain region affected by the TIA Answer: B Diff: 1 Page Ref: 565-566 Objective: 18-9

19 Copyright © 2018 Pearson Education, Inc.

Prehospital Emergency Care, 11e (Mistovich et al.) Chapter 19 Seizures and Syncope 1) You are summoned to a grocery store for a female patient found having a "fit" by store employees. Upon arrival, you are escorted to the side of an alert but mildly confused 41-year-old woman who is diaphoretic and appears exhausted. After 2-3 more minutes, she becomes more alert and informs you that she has a history of seizures and just had a generalized seizure. Despite your urging, she refuses transport to the hospital. Your safest action would be to: A) Transport the patient despite her refusal to consent B) Advise medical direction of the situation C) Contact law enforcement so your transport of the patient is legal D) Recognize a seizure history and have patient sign a refusal form Answer: B Diff: 2 Page Ref: 589 Objective: 19-7 2) You have been called for a seizure emergency. On scene, you find an adult female patient actively seizing with bystanders attempting to restrain her. You would: A) Place a padded tongue blade into the patient's mouth and then instruct the bystanders to release her B) Loosen any restrictive clothing the patient is wearing, and then summon ALS backup C) Instruct the bystanders to release the patient while you protect her head with your hands D) Have bystanders continue restraining the patient while you complete the primary assessment Answer: C Diff: 2 Page Ref: 592 Objective: 19-8 3) While you are providing free blood pressure readings at a community health fair, you are taking the blood pressure of a mother of two young twin boys. One of the boys tells you that his twin brother suffers from seizures and asks what he can do if he sees his brother convulsing. Your response would be: A) "Move any movable objects and furniture away from him." B) "Hold him securely to the floor until he stops seizing or the EMTs arrive." C) "Insert a spoon into his mouth to keep him from swallowing his tongue." D) "Tell your parents immediately, and then try to hold him as still as possible until the ambulance arrives." Answer: A Diff: 2 Page Ref: 592 Objective: 19-8

1 Copyright © 2018 Pearson Education, Inc.

4) As a general rule, a postictal patient should be placed on the stretcher and transported in which position? A) Supine B) Semi-Fowler's C) Lateral recumbent D) Prone Answer: C Diff: 1 Page Ref: 595 Objective: 19-7 5) After a lengthy response time in a rural community, you arrive at the home of a 62-year-old female patient who is still seizing. Your immediate action would be to: A) Assess the patient's airway and breathing B) Determine if the patient has a seizure history C) Determine the total duration of the seizure D) Move the patient to the ambulance for assessment and initiate transport Answer: A Diff: 2 Page Ref: 592-593 Objective: 19-7 6) You find a patient, who seized for approximately 2 minutes, supine on the kitchen floor. He responds to painful stimuli and has snoring respirations. Emergency Medical Responders are holding manual in-line spinal stabilization, and they report the patient's heart rate is 124 beats/min and his pulse oximetry is 89% on room air. The patient has cool and clammy skin. What should you do first? A) Suction the airway B) Perform a jaw-thrust maneuver C) Provide supplemental oxygen D) Initiate positive pressure ventilation Answer: B Diff: 2 Page Ref: 593 Objective: 19-7 7) A postictal patient is awake and can communicate with you, but has trouble answering your questions correctly. Given this presentation, the EMT would recognize that: A) The airway is open B) Another seizure is coming C) The patient has a seizure history D) High-concentration oxygen is indicated Answer: A Diff: 2 Page Ref: 592-593 Objective: 19-7

2 Copyright © 2018 Pearson Education, Inc.

8) You are preparing to transport a 46-year-old male patient who has had multiple seizures throughout the morning, according to the family. Currently, he is on your stretcher and is postictal. He has a history of seizures for which he takes anticonvulsant medications. He also has diabetes and kidney failure. Of the equipment listed here, which is the most essential to have ready during transport of this patient? A) Immobilization equipment B) Automated external defibrillator C) Oral glucose D) Suction device Answer: D Diff: 2 Page Ref: 593 Objective: 19-7 9) When obtaining a medical history from the family of a patient experiencing seizure, which of these questions is most important for the EMT to ask first? A) "How long has he had his seizure condition?" B) "Has he ever been hospitalized for seizures?" C) "Does he take his seizure medications as prescribed?" D) "Was he grunting and breathing hard during the seizure?" Answer: C Diff: 2 Page Ref: 595 Objective: 19-7 10) While you are placing a 52-year-old female patient, whose complaint is a headache, on the stretcher, she begins to seize. Your immediate action would be to: A) Quickly move her to the ambulance for rapid transport B) Perform a jaw-thrust maneuver and insert an oropharyngeal airway C) Raise the side rails and do not apply the straps very tightly D) Remove the patient from the stretcher and place her on the ground Answer: C Diff: 2 Page Ref: 592 Objective: 19-7 11) How would you transport a patient experiencing a seizure who is strongly suspected of having a cervical spine cord injury? A) Supine on the stretcher with straps loosely applied B) In lateral recumbent position with a cervical collar in place C) With spine motion restriction precautions applied and a cervical collar in place D) In semi-sitting position with a cervical collar in place Answer: C Diff: 1 Page Ref: 595 Objective: 19-7

3 Copyright © 2018 Pearson Education, Inc.

12) You are called to a residence to assess a child. The panicked parents state that their 3-yearold son was playing with his brother when he suddenly "blanked out" and would not respond to them for several seconds. When asked, they deny any convulsing-type movement as well as a history of medical problems. Based on this description, you would be suspicious that the patient experienced which type of seizure? A) Febrile B) Simple partial C) Grand mal D) Absence Answer: D Diff: 2 Page Ref: 589-590 Objective: 19-4 13) When performing the secondary assessment on a confused patient who was reportedly unresponsive just prior to your arrival, which of these findings would be most suggestive of a seizure? A) Bruises to the arms B) Bleeding tongue C) Pinpoint pupils D) Warm skin Answer: B Diff: 2 Page Ref: 592 Objective: 19-7 14) When an EMT performs the secondary assessment on a postictal patient with a known history of seizures, which of these assessment findings would be of most concern? A) Confused mental status B) Heart rate of 116 beats/min C) Loss of bladder control D) New contusion noted to the forehead Answer: D Diff: 2 Page Ref: 592 Objective: 19-7 15) You are caring for a postictal male patient with a known history of seizures. He is confused and cannot remember his 8-year-old son's name. His son tearfully asks you if his father will ever remember him. Your response would be: A) "I do not know. We will have to let the doctor help him first." B) "I am not sure, but at least your mother remembers your name." C) "Although he is confused now, he should remember your name in a little bit." D) "I do not feel comfortable in telling you yes, since there is a chance he may not." Answer: C Diff: 2 Page Ref: 589 Objective: 19-2

4 Copyright © 2018 Pearson Education, Inc.

16) You have been called to a public bus station for a behavioral emergency. On scene, you find a disheveled male in his forties sitting up against a wall. He is confused and incontinent. Bystanders state that he was just sitting on a bench watching people walk by, when suddenly he got up and began to stumble around, shouting obscenities, and then fell to the ground shaking. From this description, the EMT should be suspicious of which condition? A) Generalized seizure B) Alcohol intoxication C) Absence seizure D) Syncopal episode Answer: A Diff: 2 Page Ref: 588 Objective: 19-4 17) As you approach a female patient, she appears unresponsive, with her arms and legs jerking violently. On the prehospital care report, you would document that the patient was found in which phase of a generalized seizure? A) Tonic B) Absence C) Syncopal D) Clonic Answer: D Diff: 3 Page Ref: 589 Objective: 19-4 18) A female patient with a history of seizures has experienced a seizure in a public area and is now refusing further assessment and transport. As you leave, a witness to the entire event tells you that he thinks the patient is "crazy in the head" because immediately before she seized, she looked up toward the ceiling and kept repeating, "Do you see the birds?" As a knowledgeable EMT, you should recognize that the bystander is describing which condition? A) Postictal confusion B) Side effects of seizure medications C) An aura D) Syncopal episode Answer: C Diff: 3 Page Ref: 588 Objective: 19-1 19) Friends of a male patient who experienced a generalized seizure while at a picnic are worried because he cannot remember the seizure. You should inform them that this is: A) A concern, because most people can remember the seizure B) A normal finding that is common among those who suffer this type of seizure C) An important piece of information that warrants immediate transport of the patient to the hospital D) Unrelated to the seizure and may indicate a more serious medical problem Answer: B Diff: 2 Page Ref: 591 Objective: 19-8 5 Copyright © 2018 Pearson Education, Inc.

20) The initial phase of a generalized seizure experienced by some patients, in which they may experience an odd smell or something auditory, is called the: A) Syncopal stage B) Aura C) Clonic phase D) Postictal state Answer: B Diff: 2 Page Ref: 588 Objective: 19-1 21) Status epilepticus is best differentiated from a generalized seizure by the: A) Duration of the seizure B) Length of the postictal period C) Presence of an aura prior to seizing D) Preexisting seizure history Answer: A Diff: 2 Page Ref: 587 Objective: 19-6 22) The EMT shows that she understands the danger posed by status epilepticus when she states: A) "Status epilepticus is an extremely dangerous condition because the patient can go into shock from blood loss." B) "If the patient does not have a history of seizures, he is at greater risk for status epilepticus." C) "The longer the seizure continues, the greater the opportunity for permanent brain damage." D) "Status epilepticus indicates that the medications a patient is taking have reached toxic levels in the body." Answer: C Diff: 3 Page Ref: 587 Objective: 19-6 23) Which of these patients is most at risk for problems to the airway and breathing? A) A 32-year-old male who has stopped taking his seizure medications and experienced a 2minute seizure B) A 23-year-old female with a history of seizures who has been seizing for 24 minutes C) An 11-year-old postictal boy with no history of seizures but with developmental delays D) A 76-year-old female who takes anticonvulsant medications and just began seizing Answer: B Diff: 2 Page Ref: 587 Objective: 19-6

6 Copyright © 2018 Pearson Education, Inc.

24) Which of these patients would be classified as having status epilepticus? A) A 16-year-old male who experienced a partial motor seizure for 7 minutes B) An 89-year-old male who seized immediately upon getting out of bed C) A 19-year-old female who briefly seized but has been postictal for 20 minutes D) A 57-year-old female at a group home who seized during the entire evening movie that was shown in the common living room, according to other group home residents Answer: D Diff: 1 Page Ref: 587 Objective: 19-6 25) You arrive on scene and find Emergency Medical Responders with a seizing patient. Which of these questions should you ask first? A) "How long has the patient been seizing?" B) "What are the patient's vital signs?" C) "Did the patient lose control of his bladder?" D) "Do you know which medications the patient is taking?" Answer: A Diff: 2 Page Ref: 597 Objective: 19-7 26) A patient has been actively seizing for 17 minutes. He is cyanotic with shallow and ineffective respirations. Which immediate care should you provide to this patient? A) Determine if he has a history of seizures B) Place him in the lateral recumbent position and apply oxygen C) Apply the automated external defibrillator D) Attempt to open the airway and begin positive pressure ventilation Answer: D Diff: 2 Page Ref: 596 Objective: 19-7 27) While you are transporting a 41-year-old woman who is not feeling well, she begins to seize. She has no history of seizures, and the seizure lasts approximately 90 seconds. After 30 seconds of being postictal and totally unresponsive, the woman suddenly seizes again for the remainder of the 12-minute transport. The EMT should recognize that the patient is experiencing which emergency condition? A) Undiagnosed epilepsy B) Hypoglycemic-induced seizures C) Status epilepticus D) Partial-motor seizure Answer: C Diff: 2 Page Ref: 587 Objective: 19-6

7 Copyright © 2018 Pearson Education, Inc.

28) While taking an EMT class, you come across a test question that asks you to define the pathophysiology of a seizure. Which answer would you select? A) "A seizure occurs when the heart beats irregularly, causing a decreased amount of oxygenrich blood to reach the brain." B) "All seizures are caused by epilepsy, which describes a problem somewhere in the body." C) "A seizure condition describes a muscle problem that causes the arms and legs to jerk." D) "A seizure occurs when there is a massive and uncoordinated electrical discharge in the brain." Answer: D Diff: 3 Page Ref: 586 Objective: 19-2 29) Which of these patients should be categorized as the highest priority for transport? A) A child with a history of seizures whose mother calls EMS due to a fever of 102°F B) A pregnant female with no history of seizures who is seizing C) A postictal patient who has stopped taking his seizure medication D) An adolescent patient who states he recalls seizing earlier in the day Answer: B Diff: 2 Page Ref: 593 Objective: 19-8 30) Which of these statements about seizures is true? A) Patients who have seizures are at increased risk for stroke B) All seizures are caused by some form of brain injury C) Seizures may be caused by a variety of medical conditions D) If the cause of a seizure is unknown, it is classified as status epilepticus Answer: C Diff: 2 Page Ref: 585-586 Objective: 19-2 31) Which type of seizure occurs in children and is caused by a high core temperature? A) Idiopathic B) Hypoglycemic C) Febrile D) Eclamptic Answer: C Diff: 1 Page Ref: 590 Objective: 19-3

8 Copyright © 2018 Pearson Education, Inc.

32) Which of these statements made by a patient who experienced a syncopal episode should concern the EMT most? A) "I seemed to get very warm just before passing out." B) "My chest felt really funny right before I passed out." C) "I remember feeling faint right before I passed out." D) "The doctor told me to get out of bed slowly, but I didn't." Answer: B Diff: 2 Page Ref: 600 Objective: 19-10 33) A patient who experienced an apparent syncopal episode refuses transport to the hospital. Prior to the patient signing a refusal, which of these statements should you make to the patient? A) "Syncope may indicate a serious underlying condition, so please follow up with a doctor." B) "The most common cause of syncope is diabetes, so follow up with your doctor." C) "People who experience a syncopal episode are not allowed to drive unless they go on medication." D) "Syncope is not dangerous and affects almost everyone, but it is still important to follow up with your doctor." Answer: A Diff: 2 Page Ref: 600 Objective: 19-12 34) You have been called for a 57-year-old female patient who has "passed out." As you enter the patient's apartment, you find her supine on the living room floor with a cool washcloth on her forehead. She states that she was standing and talking on the telephone when everything went dark. When she came to, she was on the floor. What should you do next? A) Immediately sit the patient upright while gathering the SAMPLE history B) Insert a nasopharyngeal airway and apply supplemental oxygen C) Move the patient to the ambulance for immediate transport, and call for an ALS intercept D) Complete the primary assessment, and then obtain vital signs while the patient is supine Answer: D Diff: 2 Page Ref: 600 Objective: 19-12 35) Which of these statements made by a patient would reinforce your suspicion of syncope? A) "I think that I may have had a seizure." B) "Once I hit the floor, I was out for only a few seconds." C) "I have had a low-grade fever for the past three days." D) "I was lying on the bed when I passed out." Answer: B Diff: 2 Page Ref: 598 Objective: 19-11

9 Copyright © 2018 Pearson Education, Inc.

36) A man comes into your station and tells you that he was just diagnosed by his physician as having multiple syncopal episodes the previous week. More specifically, he asks you to explain why he keeps fainting. Which of these statements would be your best response? A) "For some reason, there was a temporary decrease in the flow of blood to your brain." B) "A small clot formed in a blood vessel in your brain, but was quickly broken down by the body." C) "Your blood sugar most likely dropped very quickly, causing you to faint." D) "A syncopal episode is a small seizure that runs its course very quickly." Answer: A Diff: 3 Page Ref: 598 Objective: 19-10 37) You are dispatched to a local mall, where you are met by bystanders who state that the patient was talking to a customer representative when she "passed out." You find an alert and oriented 55-year-old female patient lying supine on the tile floor. The patient tells you that the back of her head is hurting, her neck now hurts, and her arms and legs feel very weak. Which action should you perform first? A) Open her airway with a jaw-thrust maneuver B) Obtain a full set of vital signs C) Take manual cervical spine motion restriction D) Assess the back of the patient's head for injury Answer: C Diff: 2 Page Ref: 600 Objective: 19-12 38) Which of these is the best explanation of why a patient who experiences a syncopal episode often regains consciousness after falling? A) The act of falling causes the heart rate to decrease B) The supine position allows better perfusion to the brain C) The trauma of falling causes the blood glucose level to increase D) The seizure stops once the patient is supine Answer: B Diff: 3 Page Ref: 598 Objective: 19-10

10 Copyright © 2018 Pearson Education, Inc.

39) You are an EMT who finds an adult male patient on his living room floor after the police department had to force the door open. The scene size-up reveals a very cluttered home that smells of urine. On an end table, you find a container of phenytoin (Dilantin) with the patient's name on it. When you pinch his shoulder, the patient responds by moaning and trying to remove your hand from his neck. Based on this information, which of these conditions would you suspect? A) Syncope B) Allergic reaction C) Behavioral emergency D) Seizure Answer: D Diff: 2 Page Ref: 600, Table 19-4 Objective: 19-11 40) You are called to a residence for seizure activity. On scene, the patient's daughter tells you that her father, who has diabetes, stated that he suddenly felt dizzy and then fell to the floor. Within a few seconds, he began asking what had happened and why he was on the floor. Based on this description, the EMT should recognize which condition? A) Generalized seizure B) Diabetic reaction C) Aura of a pending seizure D) Syncopal episode Answer: D Diff: 2 Page Ref: 600 Objective: 19-11 41) Which sign or symptom would be most helpful in determining that a patient had a syncopal episode rather than a seizure? A) The extremities twitched, but only for a few moments B) There was no period of confusion after the event C) The patient never had a seizure before D) The patient complained of headache before the event Answer: B Diff: 2 Page Ref: 600 Objective: 19-11 42) You have been called to an alcoholic rehabilitation center for a 56-year-old male patient whose left arm suddenly began to shake uncontrollably. He is alert and oriented, and he is terrified that he cannot stop his arm from moving. Which type of seizure should you suspect? A) Complex partial B) Generalized C) Simple partial D) Psychomotor Answer: C Diff: 1 Page Ref: 590 Objective: 19-5 11 Copyright © 2018 Pearson Education, Inc.

43) You are transporting a patient who has a history of simple partial seizures. During transport, she experiences a simple partial seizure involving her right arm. Her pulse is 92, respirations are 14 breaths/min and adequate, blood pressure is 168/88 mmHg, and SpO2 is 98%. Your primary concern would be: A) Administering supplemental oxygen via nasal cannula B) Loss of bowel or bladder control C) Placing the patient's arm tightly in a sling to prevent injury D) Progression to a generalized seizure Answer: D Diff: 2 Page Ref: 590-591 Objective: 19-5 44) Which of these statements about seizures is true? A) Simple partial seizures do not cause an alteration in mental status B) The cause of most seizures is easily identified C) Generalized seizures always last longer than partial seizures D) Complex partial seizures involve both cerebral hemispheres Answer: A Diff: 3 Page Ref: 590 Objective: 19-5 45) An 86-year-old male patient has experienced a sudden change in mental status and is repeatedly striking a fist against his leg. When you question him, he does not respond. This presentation is most consistent with which condition? A) Seizure B) Dementia C) Diabetes D) Delirium Answer: A Diff: 1 Page Ref: 591 Objective: 19-5 46) A 72-year-old male patient with a history of elevated thyroid function and a brain tumor is experiencing a psychomotor seizure. When you enter the room, he is standing up and walking in a circle. He is awake but does not respond to your questions. Family members state that he has had three similar episodes over the past six months, which his physician diagnosed as complex partial seizures. Vital signs are pulse, 112; respirations, 16 breaths/min; blood pressure, 166/68 mmHg; and SpO2, 98%. Which action would be most appropriate in the care of this patient? A) Determine if a durable power of attorney exists B) Restrain the patient and apply high-concentration oxygen C) Proceed with a nonemergency transport D) Insert a nasopharyngeal airway to ensure airway patency Answer: C Diff: 2 Page Ref: 590 Objective: 19-7

12 Copyright © 2018 Pearson Education, Inc.

47) Which of these medications is most closely associated with a history of seizures? A) Celexa B) Prozac C) Xopenex D) Depakote Answer: D Diff: 1 Page Ref: 595 Objective: 19-7 48) A 20-year-old patient is unresponsive with snoring respirations. She is in a public restroom and has no family or friends with her. After addressing the airway, breathing, and circulation, you begin the secondary assessment. Which of these signs most likely indicates the patient experienced a seizure? A) Snoring respirations B) Urinary incontinence C) Altered mental status D) Constricted pupils Answer: B Diff: 2 Page Ref: 595 Objective: 19-7 49) The EMT would recognize that a 49-year-old male patient has experienced a primary seizure when his wife states: A) "He has never had a seizure before–this is his first seizure." B) "He had seizures in the past, but they are not sure what causes them." C) "The doctors said that his seizures come from one spot in his brain." D) "His arms and legs were jerking and he would not talk to me." Answer: B Diff: 3 Page Ref: 586 Objective: 19-3 50) A 36-year-old male patient with diabetes and hypertension has experienced a secondary seizure. The patient responds to verbal stimuli with garbled speech, and his airway, breathing, and circulation are intact. Vital signs are pulse, 128; respirations, 20 breaths/min and adequate; blood pressure, 158/96 mmHg; and SpO2, 97% on room air. At this time, it is important that the EMT: A) Administer oral glucose as a precautionary measure B) Position the patient supine on the stretcher and transport C) Check the patient's blood sugar with a glucometer D) Administer supplemental oxygen at 2 lpm to maintain the oxygen saturation at its current level Answer: C Diff: 3 Page Ref: 587 Objective: 19-9

13 Copyright © 2018 Pearson Education, Inc.

51) When treating a patient who has had a seizure, the most important aspect of providing care is to: A) Ensure that the patient does not bite his or her tongue B) Identify the type and duration of seizure, as well as any history of seizures C) Provide supplemental oxygen during the seizure to ensure adequate oxygenation of the brain D) Assess for and manage any life-threatening condition found Answer: D Diff: 2 Page Ref: 585 Objective: 19-7 52) Regarding a generalized tonic-clonic seizure, which of these statements is true regarding the different stages? A) The aura comes before a loss of consciousness B) The patient is postictal prior to convulsion activity C) A loss of consciousness occurs after the clonic (convulsion) phase D) The patient becomes postictal just before the hypertonic phase Answer: A Diff: 3 Page Ref: 588-589 Objective: 19-4 53) You believe that a patient experienced a simple partial (focal motor) seizure. Which of these statements made by the patient would reinforce this suspicion? A) "I cannot remember anything that happened. I must have passed out." B) "I think that I may have lost control of my bladder and bit my tongue." C) "My left arm would not stop shaking. I did not know what was happening." D) "I remember my arms and legs shaking, and then I can't remember anything else." Answer: C Diff: 3 Page Ref: 590-591 Objective: 19-5 54) The EMT would recognize a simple partial seizure when she observes: A) Jerky muscular activity of a single extremity B) Repetitive actions such as lip smacking or finger rolling C) Blank staring that begins and ends abruptly D) Abrupt personality changes, including fits of rage Answer: A Diff: 2 Page Ref: 590-591 Objective: 19-5 55) A complex partial (psychomotor) seizure can easily be mistaken for: A) An asthma exacerbation B) A left-sided stroke C) An absence (petit mal) seizure D) Alcohol intoxication Answer: D Diff: 3 Page Ref: 591 Objective: 19-5 14 Copyright © 2018 Pearson Education, Inc.

56) You have been called for a 71-year-old male patient with seizure activity. When you reach the patient's side, you find him lying motionless on the floor of his bedroom with a family member performing chest compression-only CPR. In this situation, your immediate action would be to: A) Open the airway and begin positive pressure ventilation B) Ask the provider to stop CPR and feel for a carotid pulse C) Take over CPR and attach the automated external defibrillator D) Continue CPR and call for advanced life support assistance Answer: B Diff: 2 Page Ref: 592 Objective: 19-7 57) A young male patient with a seizure history is postictal in his bed. His airway is patent, breathing labored, and radial pulse strong and fast. Vital signs are pulse, 140; respirations, 20 breaths/min; blood pressure, 158/92 mmHg; and SpO2, 96% on room air. At this time, which action would be most appropriate? A) Provide supplemental oxygen B) Insert an oropharyngeal airway C) Start positive pressure ventilation D) Place the patient in a lateral recumbent position Answer: D Diff: 2 Page Ref: 595 Objective: 19-7 58) You are on the scene of a patient who just experienced a seizure. The patient is still in the postictal phase and is not fully oriented. Family members state that he has a history of diverticulitis, gastric ulcers, and diabetes. Your partner asks you if you think he should obtain a blood glucose level. Your response should be: A) "Yes, that is a good idea since he has a history of diabetes." B) "Yes, because if we don't, the hospital may get angry with us." C) "No, since the seizure has stopped, it is not from a blood sugar abnormality." D) "No, because even if the sugar is too high or too low, there is nothing we can do about it." Answer: A Diff: 2 Page Ref: 587 Objective: 19-9

15 Copyright © 2018 Pearson Education, Inc.

Prehospital Emergency Care, 11e (Mistovich et al.) Chapter 20 Acute Diabetic Emergencies 1) You suspect that an unresponsive female patient may be diabetic. To help confirm this suspicion, where would you send your partner to look for vials of insulin? A) Bathroom B) Bedside table C) Refrigerator D) Nightstand Answer: C Diff: 1 Page Ref: 627 Objective: 20-3 2) An elderly patient has an altered mental status. On scene, the patient's son informs you that his father has type 1 diabetes. Based on this information, you would understand that the patient: A) Needs to take insulin B) Has a history of high blood pressure C) Requires daily oral glucose to maintain his blood sugar D) Has a chronically low blood sugar level Answer: A Diff: 2 Page Ref: 610 Objective: 20-2 3) The EMT should suspect that a patient has diabetes when which medication is found on the scene? A) Nitrostat B) Metformin C) Ecotrin D) Lexapro Answer: B Diff: 1 Page Ref: 621, Table 20-3 Objective: 20-11 4) Which of these most accurately describes insulin? A) Pill that must be taken by patients with diabetes B) Hormone that breaks food into simple sugars C) Drug that increases the sugar in the blood D) Hormone that enables glucose to move into the cells Answer: D Diff: 3 Page Ref: 605 Objective: 20-3

1 Copyright © 2018 Pearson Education, Inc.

5) You are reviewing a prehospital care report for a patient with altered mental status that occurred after he took too much insulin. Based on this information, you should assume that at the time of EMS contact, the patient's blood sugar level was probably: A) High B) Low C) Normal D) Concentrated Answer: B Diff: 3 Page Ref: 610 Objective: 20-2 6) Family members have called you for a young female patient with a history of diabetes who took insulin earlier in the day. She is confused and combative. Which of these questions is most important for the EMT to immediately ask the family? A) "How long has she had diabetes?" B) "Do you think that her blood sugar is high?" C) "Did she eat after taking the insulin?" D) "Did she take any drugs or drink alcohol?" Answer: C Diff: 3 Page Ref: 610 Objective: 20-3 7) Your partner reports that he has found some small areas of bruising to the abdomen of a patient who takes insulin. He is puzzled by the finding. Which response would be most appropriate? A) "The patient must have been involved recently in a car crash or suffered some other sort of trauma." B) "There could be several reasons for the bruising, but insulin injections are often administered to the abdomen." C) "That is a concerning finding. Since patients with diabetes are prone to GI bleeds, we should treat her as having one." D) "Why don't we contact the patient's primary care physician and see if there are additional medical issues we are not aware of?" Answer: B Diff: 2 Page Ref: 622 Objective: 20-11 8) What is a glucometer used to evaluate? A) Glucose levels B) Insulin levels C) Glucogen levels D) Insulin and glucose levels Answer: A Diff: 1 Page Ref: 608 Objective: 20-5

2 Copyright © 2018 Pearson Education, Inc.

9) A glucometer reads 50 mg/dL. The EMT would recognize this reading as: A) High B) Normal C) Average D) Low Answer: D Diff: 1 Page Ref: 608 Objective: 20-5 10) With which of these patients would the EMT have the strongest suspicion of a diabetic emergency? A) A confused patient with a blood glucose of 58 mg/dL B) An alert and oriented patient with diabetes and generalized weakness C) A combative patient with a blood glucose level of 122 mg/dL D) A patient with diabetes who complains of chest pain and shortness of breath Answer: A Diff: 2 Page Ref: 608 Objective: 20-5 11) A 17-year-old patient is unresponsive. A medical identification bracelet states that he has diabetes. Which action should you perform first? A) Administer oral glucose B) Determine the type of diabetes C) Assess the airway D) Check for a pulse Answer: C Diff: 2 Page Ref: 622 Objective: 20-11 12) You are instructing EMT students on how to obtain a blood sample for use in a glucometer. You are providing accurate information when you state: A) "It is best to check the blood three times and then use the highest reading." B) "Approximately 15 mL of blood will be needed for an accurate result." C) "A small drop of blood obtained from the patient's fingertip is sufficient." D) "Have the patient raise his or her arm in the air before you obtain the blood sample." Answer: C Diff: 1 Page Ref: 609, Table 20-1 Objective: 20-5

3 Copyright © 2018 Pearson Education, Inc.

13) How would an EMT best describe a confused and combative patient with labored respirations and a blood glucose level of 33 mg/dL? A) Hypoglycemic B) Altered blood glucose C) Hyperglycemic D) Hyper-insulinemic Answer: A Diff: 1 Page Ref: 608 Objective: 20-1 14) A patient with diabetes who took insulin earlier is found unresponsive with a blood glucose level of 29 mg/dL. Which piece of information provided by family gives the best explanation for the patient's altered mental status? A) "He ate pancakes with syrup after taking his insulin." B) "His blood sugar normally goes up after eating." C) "He took only half of his required dose of insulin this morning." D) "He didn't eat breakfast or lunch today because his belly hurt." Answer: D Diff: 3 Page Ref: 610 Objective: 20-7 15) Which individual finding would most lead the EMT to suspect that a patient has hypoglycemia? A) A rapid onset of altered mental status B) A slow pulse rate and low blood pressure C) A blood glucose reading of 256 mg/dL D) Warm and dry skin with a blood glucose of 90 mg/dL Answer: A Diff: 2 Page Ref: 610 Objective: 20-10 16) A patient's glucometer measurement reads 39 mg/dL. The patient is alert, but confused. The patient's son states that he has a friend who becomes unresponsive when his sugar is less than 50 mg/dL; therefore, the reading for his father cannot be correct. Which of these statements would be your best response? A) "You are probably right; I feel comfortable in having him sign a refusal." B) "I agree. Anyone with a blood sugar less than 50 mg/dL should be unconscious." C) "The response to a low blood sugar can vary from patient to patient." D) "I agree. Just to be safe, I am not going to give him any oral glucose." Answer: C Diff: 2 Page Ref: 610 Objective: 20-7

4 Copyright © 2018 Pearson Education, Inc.

17) You determine a patient's blood glucose level to be 289 mg/dL. You would document this result on the prehospital care report as: A) Normal B) Hyperglycemic C) Depressed D) Hypoglycemic Answer: B Diff: 1 Page Ref: 608 Objective: 20-5 18) You are called to an outpatient clinic to transfer a 39-year-old male patient in diabetic ketoacidosis (DKA) to an acute-care hospital. When assessing this patient, which sign or symptom would you expect to find? A) Slow and shallow respirations B) Slow heart rate and high blood pressure C) Blood glucose level of 50 mg/dL D) Fruity or acetone odor on his breath Answer: D Diff: 2 Page Ref: 616 Objective: 20-9 19) You have been called to a residence for a patient with altered mental status. On arrival, a neighbor informs you that he saw the patient washing windows several minutes earlier and then suddenly saw him lying on the ground. He also mentions that the patient has diabetes and has been in and out of the hospital lately because his blood sugar has been dropping suddenly. Your scene size-up reveals the patient to be lying next to a ladder propped up to the second-story window. Water from a bucket is spilled on the side of the house and yard. The patient is supine and unresponsive. What should be your next most immediate action? A) Insert a nasopharyngeal airway B) Take manual spine motion restriction precautions C) Check the glucose level and administer oral glucose if needed D) Open the airway with the jaw-thrust maneuver Answer: B Diff: 2 Page Ref: 625, Figure 20-13 Objective: 20-11 20) You have arrived on the scene of a 64-year-old patient with diabetes. He is supine in bed and exhibits snoring respirations. Family members state that they just tested his blood sugar and it is 25 mg/dL. What should be your immediate action? A) Prepare and administer oral glucose B) Have the family recheck the blood sugar C) Determine what the patient has eaten D) Open the airway with a manual technique Answer: D Diff: 2 Page Ref: 612 Objective: 20-11 5 Copyright © 2018 Pearson Education, Inc.

21) After you administer oral glucose to a patient with diabetes, his level of conscious deteriorates and he becomes unresponsive with snoring respirations. Which action should you take immediately? A) Administer half a second tube of glucose B) Provide high-concentration oxygen via a nonrebreather mask C) Administer a full second dose of oral glucose D) Manually open the airway Answer: D Diff: 2 Page Ref: 612 Objective: 20-11 22) You are transporting a patient with diabetes who has low blood sugar. En route, you receive orders to administer oral glucose. Which piece of emergency equipment is most important to have readily available during this procedure? A) Nasopharyngeal airway B) Suction device C) Oropharyngeal airway D) Automated external defibrillator Answer: B Diff: 2 Page Ref: 613 Objective: 20-8 23) Five minutes after administering oral glucose to a confused patient with diabetes, you do not see any improvement in her mental status. Which action should you take? A) Administer a second dose of oral glucose B) Continue to monitor the patient C) Recheck the expiration date on the oral glucose D) Contact medical direction for further orders Answer: B Diff: 2 Page Ref: 613 Objective: 20-11 24) A patient with diabetes is alert but confused. You have decided to administer oral glucose. Which of these descriptions indicates the proper way to administer the glucose? A) Squeeze the glucose under the patient's tongue B) Place the glucose between the patient's cheek and gum C) Use a tongue depressor to place the glucose on the back of the tongue D) Have the patient slowly swallow the glucose Answer: B Diff: 2 Page Ref: 613 Objective: 20-11

6 Copyright © 2018 Pearson Education, Inc.

25) Which of these patients is a candidate for oral glucose? A) An unresponsive patient with diabetes and a rapid heart rate B) A combative patient with no history of diabetes C) An alert but confused patient with diabetes and cool skin D) A patient who is newly diagnosed with diabetes and who is responsive to painful stimuli Answer: C Diff: 2 Page Ref: 612 Objective: 20-8 26) In which form is oral glucose packaged? A) Gel B) Liquid C) Powder D) Sugar granules Answer: A Diff: 1 Page Ref: 613 Objective: 20-1 27) What would be the priority action to take prior to administering oral glucose to a young female patient? A) Determine the patient's blood glucose level B) Make sure the patient has insulin available if needed C) Ask whether the patient is allergic to glucose D) Determine whether the patient is pregnant Answer: A Diff: 2 Page Ref: 612 Objective: 20-8 28) Prior to the administration of oral glucose, a patient's blood glucose level was 49 mg/dL. After administering the oral glucose, the patient is alert and oriented. Which repeat blood glucose levels would the EMT expect with this clinical presentation? A) "Low" B) 155 mg/dL C) 30 mg/dL D) "High" Answer: B Diff: 2 Page Ref: 608 Objective: 20-8

7 Copyright © 2018 Pearson Education, Inc.

29) When administering oral glucose to a patient with diabetes, the EMT must take precautions to prevent which adverse effect? A) High blood glucose level B) Airway obstruction C) Low blood glucose level D) Increased blood pressure Answer: B Diff: 2 Page Ref: 613, Figure 20-6 Objective: 20-8 30) Medical direction has ordered you to administer one dose of oral glucose to a patient with diabetes who is exhibiting confusion. Which of these best describes what you should do? A) Repeat and question the order to medical direction B) Place half the contents of the tube of glucose in the patient's mouth C) Carefully administer the entire tube of glucose D) Have the patient swallow all of the glucose in the tube Answer: C Diff: 1 Page Ref: 613, Figure 20-6 Objective: 20-11 31) An alert but confused patient with diabetes is complaining of weakness. Your service carries oral glucose, which has been authorized through off-line medical direction. In this case, you should: A) Contact the patient's primary care physician B) Transport the patient for further evaluation in the hospital C) Contact medical direction for authorization to administer the oral glucose D) Prepare and administer the oral glucose Answer: D Diff: 2 Page Ref: 613, Figure 20-6 Objective: 20-8 32) You have been called for an adult male patient who is hypoglycemic and responsive to painful stimuli. The family has oral glucose and wants you to administer it. However, at this time, your medical director has not authorized this agent's use and there are no protocols regarding its use. Given the patient's condition and the family's request, which action should you take? A) Contact ALS backup and remain on scene until they arrive so they can give glucose via the IV line B) Instruct the family to administer the oral glucose C) Initiate transport of the patient to the emergency department D) Carefully administer the oral glucose at the family's request Answer: C Diff: 2 Page Ref: 622 Objective: 20-8

8 Copyright © 2018 Pearson Education, Inc.

33) After you administer oral glucose to a confused and combative patient, she gradually becomes calmer and fully oriented. As a knowledgeable EMT, you would recognize that this patient: A) Had a high blood sugar level B) Now has a decreasing blood sugar level C) Will need a second dose of oral glucose D) Had a low blood sugar level Answer: D Diff: 2 Page Ref: 613 Objective: 20-7 34) A patient with diabetes presents as alert but irritable and confused. His airway is patent with an intact gag reflex and breathing is adequate. You do not have a glucometer available and are not sure if his blood sugar is high or low. In this situation, what would be your next best action? A) Insert a nasal airway B) Administer oral glucose C) Have the family members administer his insulin D) Provide emergency transport Answer: B Diff: 2 Page Ref: 617 Objective: 20-8 35) You are reviewing prehospital care reports and notice that a patient with low blood sugar and no contraindications to oral glucose did not receive the medication. You recognize that this inaction placed the patient at risk for which condition? A) Liver damage B) Infection C) Heart failure D) Brain damage Answer: D Diff: 3 Page Ref: 617 Objective: 20-8 36) Your medical director is providing a review of diabetic emergencies and asks you to describe glucose. Which of these statements would be your most appropriate response? A) "It is a medication that will lower the blood sugar." B) "It is a hormone produced by the pancreas." C) "It is a sugar that is the body's main source of energy." D) "It is a toxin that accumulates in the blood of a patient with diabetes." Answer: C Diff: 3 Page Ref: 604 Objective: 20-1

9 Copyright © 2018 Pearson Education, Inc.

37) You have been called for a patient who is confused and slightly combative. Which of these assessment findings would make you suspicious that the patient has hyperglycemia? A) Decreased appetite according to family members B) Patient's statement that he has not urinated for some time C) Increased respirations with a sweet smell to the patient's breath D) Comment by family that the patient may have taken too much insulin Answer: C Diff: 2 Page Ref: 616 Objective: 20-9 38) Which of these statements made by an EMT indicates a correct understanding of glucose in the human body? A) "Once ingested, glucose is broken down into carbohydrates and used for energy." B) "All cells in the body require glucose for energy and cannot use anything else." C) "Glucose is transformed into fats, which the cells of the body use for energy." D) "The cells of the brain cannot store glucose, and are uniquely sensitive to low glucose levels." Answer: D Diff: 3 Page Ref: 606 Objective: 20-2 39) An unresponsive patient presents with a blood sugar of 30 mg/dL. What is the major threat to this patient's well-being? A) Brain damage B) Dehydration C) Cardiac compromise D) Glucose toxicity Answer: A Diff: 2 Page Ref: 610 Objective: 20-7 40) You are assisting a paramedic who is administering glucagon to a patient. Based on your understanding of how glucagon works in the human body, what is the therapeutic goal of this treatment? A) Increases insulin release from the liver B) Helps red blood cells carry the glucose C) Raises the amount of circulating blood glucose D) Rapidly decreases a high amount of circulating glucose Answer: C Diff: 3 Page Ref: 605-606 Objective: 20-4

10 Copyright © 2018 Pearson Education, Inc.

41) You know an EMT is correctly using a glucometer when she performs which procedural step? A) Avoids cleaning the finger with alcohol prior to pricking it with a lancet B) Wastes the first drop of blood obtained from the patient's finger C) Asks the patient to hold his hand above his head prior to pricking a finger for blood D) Uses a lancet to prick an artery or vein for the blood sample Answer: B Diff: 2 Page Ref: 609, Table 20-1 Objective: 20-5 42) You have been called for a 33-year-old male patient who complains of weakness and dizziness. When getting the history, which of these statements made by the patient would raise your suspicion that the patient has undiagnosed diabetes? A) "I feel the need to eat sugar throughout the day." B) "I get very weak just after eating a meal." C) "I feel very weak in the morning after getting up." D) "I find myself urinating all of the time." Answer: D Diff: 2 Page Ref: 609 Objective: 20-9 43) A patient informs you that she takes Januvia pills for her diabetes. Which of these statements about this patient and her diabetes is true? A) Her pancreas still produces some insulin B) The pills contain insulin for use by the body C) Her body needs glucose that is provided by the pills D) The pills decrease her craving for sugary foods Answer: A Diff: 3 Page Ref: 609-610 Objective: 20-6 44) A patient who is unresponsive has a blood glucose level of 28 mg/dL. Her family states that she took her insulin in the morning 3 hours ago. Which of these statements made by the family would make sense in regard to the patient's current blood sugar level? A) "She is going to the bathroom all of the time." B) "She smoked a cigarette just before becoming unresponsive." C) "She did not eat after taking her insulin." D) "She ate a lot of cake and ice cream last night." Answer: C Diff: 2 Page Ref: 610 Objective: 20-6

11 Copyright © 2018 Pearson Education, Inc.

45) To which of these patients would the EMT elect to administer oral glucose? A) An alert and oriented 32-year-old female with a blood sugar of 52 mg/dL and no history of diabetes B) A confused 44-year-old male with a history of diabetes who is able to swallow C) An unresponsive 24-year-old male with a blood sugar of 24 mg/dL and a history of diabetes D) A confused male patient with a blood sugar reading of 96 mg/dL and a history of diabetes Answer: B Diff: 2 Page Ref: 612 Objective: 20-8 46) What is the primary reason to administer oral glucose to a patient? A) Help sugar get into the brain cells B) Increase the amount of circulating glucose in the bloodstream C) Stimulate the body to release insulin from the pancreas D) Improve the patient's level of consciousness Answer: B Diff: 3 Page Ref: 612 Objective: 20-11 47) The wife of a 43-year-old male patient has called 911 because her husband was difficult to wake up and now is lethargic with garbled speech. She reports a history of diabetes, stroke, renal failure, and high blood pressure. The patient's airway is patent, respirations are tachypneic but adequate, and radial pulse is rapid. Your partner reports the following vital signs: pulse, 136; respirations, 22 breaths/min; blood pressure, 106/62 mmHg; SpO2, 92%; and blood glucose, 642 mg/dL. What would be the most appropriate action in the care of this patient? A) Administer supplemental oxygen B) Administer oral glucose C) Encourage the patient to drink water D) Assist the patient with taking his insulin Answer: A Diff: 2 Page Ref: 622 Objective: 20-11 48) When providing information about diabetes to a group of Emergency Medical Responders, you would tell them that it is a disease: A) In which the body cannot turn carbohydrates into sugar B) In which the patient's blood sugar will suddenly drop C) That always requires insulin to be injected into the body D) In which the pancreas fails to produce the proper amount of insulin Answer: D Diff: 3 Page Ref: 608 Objective: 20-1

12 Copyright © 2018 Pearson Education, Inc.

49) Which of these patients has a normal fasting glucose level? A) A 53-year-old female who is alert and oriented with a glucose level of 117 mg/dL B) A 37-year-old female who is weak and dizzy with a glucose level of 81 mg/dL C) A 67-year-old male with no complaints and a glucose level of 127 mg/dL D) A 12-year-old male with a blood glucose level of 38 mg/dL Answer: B Diff: 2 Page Ref: 608 Objective: 20-5 50) You have been called for a 32-year-old female patient with altered mental status. On scene, you find the patient lethargic and confused. Although information is sketchy, it appears that she has a history of diabetes as well as renal failure for which she receives dialysis. You note no deficits in the primary assessment and the vital signs are pulse, 124; respirations, 28 breaths/min; blood pressure, 106/68 mmHg; and SpO2, 97%. Your partner reports a blood sugar reading of 774 mg/dL. Based on this information, the EMT would understand that definitive care of this patient would involve the administration of which medication at the hospital? A) Insulin B) Glucose C) Intravenous fluids D) Oxygen Answer: A Diff: 3 Page Ref: 605 Objective: 20-9 51) Which assessment finding is the best means by which to delineate hypoglycemia from hyperglycemia? A) Determine the rate at onset B) Examine the signs and symptoms C) Analyze the vital signs D) Evaluate the patient's blood sugar Answer: D Diff: 1 Page Ref: 608 Objective: 20-5 52) The EMT would describe diabetes as a disease state in which: A) Not enough sugar is provided to the body B) There is a problem with the transport of glucose into the cells C) A tumor on the pancreas causes too much insulin to be released D) The body cannot store and maintain sugar reserves Answer: B Diff: 1 Page Ref: 605 Objective: 20-1

13 Copyright © 2018 Pearson Education, Inc.

53) The most common complaint of patients with diabetes for which EMS is summoned is: A) Desire for sugary foods B) Difficulty breathing C) Altered mental status D) Nausea and vomiting Answer: C Diff: 1 Page Ref: 605 Objective: 20-11 54) Long-term complications of diabetes commonly include: A) Kidney disease B) Asthma C) Mental retardation D) Coma Answer: A Diff: 1 Page Ref: 604 Objective: 20-2 55) With normal metabolism, what happens after a healthy patient ingests food? A) Glucose levels in the body drop quickly B) The liver releases stored glucose C) Excess glucose is released in the urine D) Blood levels of insulin rise Answer: D Diff: 1 Page Ref: 606 Objective: 20-3 56) The EMT would recognize dehydration as a concern in a patient with diabetes owing to: A) Decreased desire to drink water B) Excessive urine production and elimination C) Persistent diarrhea from elevated sugar levels D) Vomiting leading to fluid loss Answer: B Diff: 3 Page Ref: 609 Objective: 20-9 57) Within the human body, what effect does insulin have? A) Decreases the level of glucose in the bloodstream B) Carries glucose from the digestive tract into the bloodstream C) Frees stored glucose from the liver so as to elevate the blood glucose level D) Turns carbohydrates into glucose, which can pass into the cells Answer: A Diff: 3 Page Ref: 606 Objective: 20-4

14 Copyright © 2018 Pearson Education, Inc.

58) A glucometer is a tool the EMT can use to measure: A) Insulin levels B) Glycogen levels C) Blood sugar levels D) Carbohydrate levels Answer: C Diff: 1 Page Ref: 608 Objective: 20-1 59) To decrease the possibility of a faulty glucometer reading, you would: A) Use the first drop of blood obtained B) Use venous blood only for testing C) Avoid the use of alcohol as an antiseptic D) Make sure test strips are not expired Answer: D Diff: 2 Page Ref: 608 Objective: 20-5 60) The insulin level in a patient with undiagnosed diabetes is elevated, and the cellular receptors for the insulin are not sensitive (to the insulin). You would anticipate which finding when caring for this patient? A) Peripheral edema from retained fluid B) A low blood sugar reading on the glucometer C) Slowed respirations and heart rate D) Elevated blood sugar levels Answer: D Diff: 3 Page Ref: 605 Objective: 20-9 61) A patient exhibits signs and symptoms of diabetes, but he informs you that he has not been diagnosed with the disease. You decide to check his blood sugar. If the patient truly has diabetes, which of these readings would you expect? A) 465 mg/dL B) 110 mg/dL C) 0 mg/dL D) 70 mg/dL Answer: A Diff: 2 Page Ref: 614 Objective: 20-9

15 Copyright © 2018 Pearson Education, Inc.

62) You are sitting next to a 24-year-old female patient with diabetes who has altered mental status and a blood sugar reading of 31 mg/dL. Family members state that she took her normal amount of insulin this morning and ate her breakfast as usual. Which additional statement would the EMT recognize as contributing to her current condition? A) "She has been taking Motrin every 6 hours for ankle pain." B) "She likes to stay in shape and ran 4 miles after breakfast." C) "She did not take her nighttime insulin last night." D) "She did not eat anything sweet for breakfast this morning." Answer: B Diff: 3 Page Ref: 610 Objective: 20-7 63) A patient with diabetes is exhibiting rapid and deep respirations. As a knowledgeable EMT, you would expect this patient to have: A) hypoglycemia. B) hypoxia. C) fever and tachycardia. D) elevated blood glucose. Answer: D Diff: 3 Page Ref: 616 Objective: 20-9 64) A patient with diabetes is noncompliant with taking insulin. On scene, your assessment indicates this patient has an open airway, rapid and deep respirations, and a rapid radial pulse. His skin is warm and flushed. Vital signs are pulse, 120; respirations, 40 breaths/min; blood pressure, 108/86 mmHg; and SpO2, 98%. The patient's blood sugar reads "high" on your glucometer. Which of these instructions to other caregivers indicates proper care of this patient? A) "We need to apply supplemental oxygen at 15 liters per minute through a nonrebreather mask." B) "Someone needs to start positive pressure ventilation to slow his hyperventilation." C) "Do not worry about his breathing rate, it is actually helping him right now." D) "We need to see if his insulin is here so we can help him administer it to himself." Answer: C Diff: 3 Page Ref: 616 Objective: 20-9 65) Altered mental status resulting from a hypoglycemic episode in a patient with diabetes typically: A) Benefits from Kussmaul respirations B) Is accompanied by a "fruity" breath odor C) Has a sudden onset D) Follows a history of polyuria Answer: C Diff: 3 Page Ref: 621 Objective: 20-10

16 Copyright © 2018 Pearson Education, Inc.

Prehospital Emergency Care, 11e (Mistovich et al.) Chapter 21 Allergic and Anaphylactic Reactions 1) A panicked and anxious patient tells you that he is allergic to peanuts and thinks he may have accidentally eaten some. He states that the last time he ate peanuts he almost died and needed to have a "tube put in his throat." When assessing this patient, which of these signs or symptoms would provide the best evidence that the patient is having an allergic, and possibly an early anaphylactic, reaction? A) Hives on his chest B) Panic and anxiety C) Elevated blood pressure D) Cool and dry skin Answer: A Diff: 1 Page Ref: 634 Objective: 21-7 2) A patient states that he is allergic to poison ivy and was burning it yesterday while clearing brush off his farm when he accidentally inhaled some of the smoke. Today, he awoke and found a red rash to the back of his left hand and called 911. Assessment reveals him to be alert and oriented, with easy respirations and a blood pressure of 128/70 mmHg. Your next action would be to: A) Provide immediate and rapid transport B) Assist the patient with his epinephrine auto-injector C) Obtain a full SAMPLE and OPQRST history D) Call for advanced life support (ALS) assistance Answer: C Diff: 2 Page Ref: 635-636 Objective: 21-8 3) A 39-year-old female patient, who is allergic to shellfish, thinks that she may have accidentally ingested some that was mixed into her food at a restaurant. When assessing her, which of these statements made by the patient would be suggestive that she is in the early stages of an anaphylactic reaction? A) "I feel as though I am getting a fever." B) "My heart suddenly feels as though it is skipping some beats." C) "I feel dizzy and weak." D) "I suddenly feel like a have a lump in my throat." Answer: D Diff: 2 Page Ref: 636 Objective: 21-7

1 Copyright © 2018 Pearson Education, Inc.

4) A patient is covered with hives. He is responsive to painful stimuli but has labored respirations with an inspiratory stridor sound, bilateral wheezing, and a weak and rapid pulse. His vital signs are pulse, 128; respirations, 24 breaths/min; and blood pressure, 80/50 mmHg. Given this presentation, the EMT should suspect that the patient has: A) Sensitization B) Anaphylaxis C) Hypersensitivity D) Localized allergic reaction Answer: B Diff: 1 Page Ref: 636-637 Objective: 21-7 5) During the primary assessment, which of these findings would most lead the EMT to suspect an allergic reaction? A) Hives to the face and neck B) Blood pressure of 100/60 mmHg C) Heart rate of 112 beats/min D) Complaint of dizziness Answer: A Diff: 1 Page Ref: 636 Objective: 21-7 6) A patient who is severely short of breath and covered with hives states that she is allergic to crab. Despite her allergy, she tried some crab dip 10 minutes ago, since it had been "years and years" since she had her last allergic reaction. Which question should the EMT ask first? A) "How many allergic reactions have you had?" B) "Why did you eat the dip if you are allergic to crab?" C) "Which medications are you currently taking?" D) "Do you have an epinephrine auto-injector?" Answer: D Diff: 2 Page Ref: 635 Objective: 21-8 7) When reassessing a patient whom you are treating for an anaphylactic reaction, which of these findings would be of the greatest concern? A) Respiratory rate change from 28 to 10 breaths/min with increasing patient lethargy B) Red, warm, and dry skin, with the patient becoming increasingly agitated C) Pulse oximetry reading of 95% via high-concentration oxygen D) New hives appearing on the chest that were not there upon your arrival Answer: A Diff: 2 Page Ref: 639 Objective: 21-8

2 Copyright © 2018 Pearson Education, Inc.

8) A 23-year-old male patient has called EMS for swollen lips and hives. Upon entering the scene, which of these observations would make the EMT suspicious that the patient is having an allergic reaction? A) A glucometer on a table in the living room B) A new container of penicillin on a table with two pills missing C) Other family members complaining of similar symptoms D) A bottle of expired nitroglycerin on the kitchen counter Answer: B Diff: 2 Page Ref: 633 Objective: 21-6 9) When assessing a patient with a suspected anaphylactic reaction, which of these findings would you expect? A) Weak, rapid radial pulse B) Rapid and strong carotid pulse C) Normal to slightly increased blood pressure D) Slow and bounding radial pulse Answer: A Diff: 2 Page Ref: 634 Objective: 21-8 10) What is the most severe form of an allergic reaction called? A) Hypersensitivity B) Anaphylaxis C) Antigen response D) Rhinorrhea Answer: B Diff: 1 Page Ref: 630 Objective: 21-1 11) If signs and symptoms of anaphylaxis occur rapidly in a patient, the EMT must realize that: A) The reaction will likely be severe B) The reaction will likely be localized C) An increased amount of antigens is present in the patient's body D) Epinephrine will not be effective Answer: A Diff: 1 Page Ref: 630 Objective: 21-8

3 Copyright © 2018 Pearson Education, Inc.

12) On scene, you assisted a patient with his epinephrine auto-injector after he was stung multiple times by bees. You are now transporting the patient to the hospital. As you reassess the patient, which of these findings would best indicate that the epinephrine is benefiting the patient? A) Increasing heart rate B) Complaint of fatigue C) Decreasing wheezing D) Decreasing blood pressure Answer: C Diff: 2 Page Ref: 644, Figure 21-6 Objective: 21-8 13) A young female patient, who is allergic to peanuts, was just exposed to peanut butter in a sandwich at a church picnic. Assessment reveals her to be alert and oriented, with swollen lips and hives on her face. She is breathing adequately at a rate of 18 breaths/min and has a heart rate of 82 beats/min. Your partner advises you that her blood pressure is 118/72 mmHg and room-air SpO2 is 92%. The patient states that she also has asthma, for which she uses an inhaler. What should you do first? A) Assist the patient with her inhaler B) Provide supplemental oxygen C) Start positive pressure ventilation D) Determine the amount of peanut butter consumed Answer: B Diff: 2 Page Ref: 634 Objective: 21-8 14) You have arrived by the side of a lethargic patient who is allergic to latex and was exposed to it when she spilled a latex-containing powder on her right hand. She is able to speak only a few words with each breath, and a quick glance at the pulse oximeter reveals the saturation is in the 80-90% range. Additionally, the patient's tongue is swollen and her respirations are rapid, labored, and noisy. Which action is your immediate priority? A) Insert an oropharyngeal airway B) Wash the latex powder from the patient's hand C) Obtain a heart rate and blood pressure D) Start positive pressure ventilation Answer: D Diff: 2 Page Ref: 634 Objective: 21-8 15) An awake but confused patient experiencing an allergic reaction begins to exhibit stridorous respirations. The EMT knows that the underlying condition responsible for this presentation is: A) Constriction of the small airways B) Spasm of the lower airway C) Swelling to the upper airway D) Loss of the gag reflex Answer: C Diff: 2 Page Ref: 634 Objective: 21-8 4 Copyright © 2018 Pearson Education, Inc.

16) Which of these statements indicates that the EMT understands airway management in a patient suffering from anaphylaxis? A) "When providing positive pressure ventilation, it may be difficult to pass air through the swollen tissues." B) "Epinephrine will help the patient's blood pressure, but not the swollen tissue in the airway." C) "The best way to open the airway in a patient with upper airway edema is to place the patient in an upright position." D) "If an oral or nasal airway is used, the airway will be opened and positive pressure ventilation will not be needed." Answer: A Diff: 3 Page Ref: 634 Objective: 21-8 17) You are responding to a call for an allergic reaction. Your partner is a new EMT and this is his first call. When prepping him on how the scene and the patient may present, which instruction regarding the assessment and treatment of an allergic reaction and/or anaphylaxis would be correct? A) "If the allergic reaction was caused by exposure to chocolate or eggs, we will not be able to administer epinephrine." B) "Epinephrine really should be given to any patient experiencing even a mild allergic reaction to prevent anaphylaxis." C) "Our first priority will not be to determine what caused the allergic reaction, but to treat the patient according to his or her symptoms." D) "If the allergic reaction was caused by food, we will need to administer activated charcoal." Answer: C Diff: 2 Page Ref: 634 Objective: 21-8 18) A patient informs you that whenever she is exposed to a particular soap, she experiences a mild allergic reaction. As a knowledgeable EMT, you should recognize that: A) The soap contains an antigen B) The patient's airway will swell shut if she inhales the soap C) The soap contains antibodies that react in the patient's body D) She must have been exposed to the soap as an infant Answer: A Diff: 3 Page Ref: 630 Objective: 21-1

5 Copyright © 2018 Pearson Education, Inc.

19) The EMT exhibits an understanding of allergic reactions and anaphylaxis when she makes which of these statements? A) "An allergic reaction occurs only when the patient ingests or inhales an allergen; anaphylaxis occurs when the allergen is injected." B) "Allergic reactions are caused by pollen and food; anaphylactic reactions are caused by venoms and medications." C) "Anaphylaxis is a severe allergic reaction that will lead to death without emergency care." D) "Anaphylaxis describes a severe allergic reaction that occurs the first time a person is exposed to an allergen." Answer: C Diff: 3 Page Ref: 630 Objective: 21-1 20) What is the most common cause of an allergic reaction in which the antigen was ingested? A) Bee stings B) Medications C) Food D) Pollen Answer: B Diff: 1 Page Ref: 633 Objective: 21-6 21) You are called to an outpatient surgery center for a patient who developed an allergic reaction while receiving an intravenous (IV) antibiotic. How would you would document the route of exposure? A) Injection B) Contact C) Ingestion D) Topical Answer: A Diff: 1 Page Ref: 633 Objective: 21-6 22) What is the best explanation of why patients experiencing an anaphylactic reaction have difficulty breathing? A) Allergens use the available oxygen molecules B) Antibodies depress the brain's respiratory center C) Histamine constricts the bronchioles D) Antigens increase the body's need for oxygen Answer: C Diff: 2 Page Ref: 631-632 Objective: 21-4

6 Copyright © 2018 Pearson Education, Inc.

23) The EMT shows he understands the use of an epinephrine auto-injector for treating a patient with an anaphylactic reaction when he makes which of these statements? A) "The epinephrine auto-injector is used only when a patient is stung by a bee or bitten by an insect." B) "It is critical that the EMT identify the cause of the allergic reaction prior to administering epinephrine through the auto-injector." C) "The epinephrine auto-injector should never be used if the patient developed the allergic reaction after eating chocolate." D) "Epinephrine administered through an auto-injector can be used for all anaphylactic and anaphylactoid reactions, regardless of the cause." Answer: D Diff: 2 Page Ref: 639 Objective: 21-9 24) After assisting a patient with the use of an epinephrine auto-injector, you note that the patient's blood pressure has improved. As a knowledgeable EMT, you realize that this effect occurred due to epinephrine's ability to: A) Relax the smooth muscle of the blood vessels B) Constrict blood vessels C) Increase the number of circulating red blood cells D) Increase the oxygen-carrying capacity of the red blood cells Answer: B Diff: 2 Page Ref: 640 Objective: 21-9 25) After you assist a patient with his epinephrine auto-injector, which of these statements made by the patient best indicates that the epinephrine is working and the patient is improving? A) "My breathing feels a lot easier." B) "My heart feels as though it is racing." C) "I am getting a little bit of a headache." D) "I am more relaxed and getting sleepy." Answer: A Diff: 1 Page Ref: 640 Objective: 21-8

7 Copyright © 2018 Pearson Education, Inc.

26) Five minutes after you assist a patient with her epinephrine auto-injector, the patient states that she feels much better and would like to refuse additional care and transport to the hospital. Which of these statements by the EMT would be most appropriate given this situation? A) "If you feel better now, there is probably no need to go to the hospital. I will get the refusal form for you to sign." B) "Once we help you take your epinephrine, the law requires that you be transported to a hospital for additional care." C) "Epinephrine can make you very sleepy. You can refuse to be transported, but just make sure that someone is here to keep an eye on you." D) "The epinephrine will start to wear off in 10 or so minutes. Let's see how you are doing then before thinking about refusing transport." Answer: D Diff: 2 Page Ref: 644, Figure 21-6 Objective: 21-9 27) How should an EMT assist a patient in taking his epinephrine auto-injector? A) Press the injector onto the lateral thigh, and wait for it to automatically inject B) Place the injector against the deltoid muscle in the arm, and push the plunger to slowly inject the medication C) Pinch the skin on the anterior thigh, press the injector against the skin, and wait until it activates D) Pinch the skin on the lateral thigh, press the injector against the skin, and rapidly push the plunger to inject the medication Answer: A Diff: 1 Page Ref: 643-644, Figure 21-6 Objective: 21-9 28) For the EMT to achieve the best results when assisting a patient with epinephrine treatment, the medication should be properly injected into: A) An artery B) The skin C) A vein D) A muscle Answer: D Diff: 1 Page Ref: 644, Figure 21-6 Objective: 21-9 29) You have assisted an adult patient with his epinephrine auto-injector. When giving a report to the emergency department nurse, you should inform her that which dose of epinephrine was administered? A) 0.1 mg B) 0.25 mcg C) 0.3 mg D) 0.3 mcg Answer: C Diff: 1 Page Ref: 640 Objective: 21-9 8 Copyright © 2018 Pearson Education, Inc.

30) A football player in full uniform has been stung by a bee. Your assessment reveals him to be confused with stridorous respirations and weak radial pulses. His skin is warm and flushed and covered with hives. Vital signs are pulse, 164; respirations, 28 breaths/min; blood pressure, 74/58 mmHg; and SpO2, 87%. The coach notifies you that the patient is allergic to bees and hands you the patient's epinephrine pen. Your next action would be to: A) Completely expose the patient and administer the epinephrine into his anterior thigh B) Lift the shirt over the patient's abdomen and administer the epinephrine into his abdomen below the umbilicus C) Lift the patient's shoulder pads and shirt and administer the epinephrine into his upper arm muscle D) Administer the epinephrine through a pad-less area of the patient's pants over the side of his thigh Answer: D Diff: 2 Page Ref: 640 Objective: 21-9 31) You are staffing an information booth at the county fair when a mother and her 15-year-old son approach. The mother informs you that her son is severely allergic to bee stings and had an epinephrine auto-injector prescribed to him when he was 5 years old. She hands you the 0.15 mg auto-injector and notes that it expired several years ago. Which of these statements would be most appropriate for you to make? A) "Call your doctor as soon as possible for a new injector because this one is out of date." B) "We have some auto-injectors on the ambulance. I can get one for you to have until you are able to get a new one from your doctor." C) "Make sure to call the doctor since this is out of date. Also ask the doctor if he wants to add a steroid to the injector." D) "Call your doctor to get a new injector and ask him if he wants to change the dose." Answer: D Diff: 2 Page Ref: 643, Figure 21-6 Objective: 21-9 32) In which of these situations would the EMT contact medical direction prior to administering epinephrine to a patient who is experiencing an anaphylactic reaction? A) The patient has an allergy to chocolate or eggs B) The patient's heart rate is 144 beats/min C) The patient is hypotensive and tachycardic D) The patient is not prescribed an auto-injector Answer: D Diff: 1 Page Ref: 643, Figure 21-6 Objective: 21-9

9 Copyright © 2018 Pearson Education, Inc.

33) You have just assisted a patient with his epinephrine auto-injector. The patient states that he has never taken epinephrine before. What would be the appropriate EMT response about how the drug may make him feel? A) "The epinephrine may make you drowsy, so just close your eyes." B) "Be prepared to feel dizzy and nauseated. These effects are common when taking epinephrine." C) "You may feel a little shaky, but that is normal and will pass." D) "If you pass out, that is normal. I will just give you some oxygen if that happens." Answer: C Diff: 2 Page Ref: 644, Figure 21-6 Objective: 21-8 34) When administering epinephrine to an infant or child, the EMT would place the injector at which site? A) Outer buttock B) Hip muscle C) Upper arm D) Lateral thigh Answer: D Diff: 1 Page Ref: 644, Figure 21-6 Objective: 21-9 35) Five minutes after assisting a patient with his epinephrine auto-injector, you determine that the patient is getting progressively worse. The patient has a second auto-injector available; however, written protocols do not address the administration of a second dose. What is the priority action the EMT should take at this time? A) Request advanced life support (ALS) assistance B) Administer the second dose of epinephrine C) Continue to monitor the patient D) Contact medical direction for further orders Answer: D Diff: 2 Page Ref: 644, Figure 21-6 Objective: 21-9 36) Prior to assisting a patient with his epinephrine auto-injector, the EMT must ensure that: A) The patient is improving B) On-line or off-line medical direction for epinephrine administration exists C) The medication is yellow in color D) The patient's heart rate and blood pressure are stable Answer: B Diff: 2 Page Ref: 643, Figure 21-6 Objective: 21-9

10 Copyright © 2018 Pearson Education, Inc.

37) You have been called for a male patient experiencing a severe allergic reaction. The patient is very confused and having great difficulty breathing. He has an epinephrine auto-injector, but your service does not have protocols related to its use. Fortunately, in your EMT class, you learned about and passed a test on the use of an epinephrine auto-injector for allergic reactions. In this case, you should: A) Assist the patient with his epinephrine auto-injector and then contact medical control B) Provide oxygen and respiratory support as needed, followed by immediate transport C) Assist the patient with his epinephrine using implied consent as justification D) Contact the physician prescribing the medication for permission to use the auto-injector Answer: B Diff: 2 Page Ref: 643, Figure 21-6 Objective: 21-9 38) Which of these assessment findings would indicate that the patient is experiencing an anaphylactic reaction after a bee sting and requires immediate intervention? A) Hives to the arm that was stung and a heart rate of 96 beats/min B) Localized swelling around the site of the bee sting C) Swollen lips and oral mucosa, with a blood pressure of 84/50 mmHg D) Edema to the feet, with an SpO2 of 93% on room air and mild dyspnea Answer: C Diff: 2 Page Ref: 638 Objective: 21-7 39) A patient informs you that he found out he is allergic to certain molds and was prescribed an epinephrine auto-injector. He asks you how the epinephrine will help him the next time he is exposed to mold. Your response would be: A) "It will quickly destroy the allergen in the mold." B) "It will provide antibodies to your body and give you temporary immunity." C) "It will reverse the reaction by mimicking the parasympathetic nervous system in your body." D) "It will cause the blood vessels to constrict and your airway structures to relax so that they reopen." Answer: D Diff: 3 Page Ref: 644, Figure 21-6 Objective: 21-9 40) Why would the patient experiencing an allergic reaction tell you that it is easier to breathe following administration of epinephrine? A) Drying of mucus in the airways B) Dilation of the bronchioles C) Increased blood flow to the lungs D) Decreased blood pressure Answer: B Diff: 3 Page Ref: 644, Figure 21-6 Objective: 21-9

11 Copyright © 2018 Pearson Education, Inc.

41) You have been called to a residence for a female patient who awoke with swollen lips and a rash to her arms and abdomen. In addition, she exhibits a mild expiratory wheeze in all lung fields. She called her doctor, who advised her to take some Benadryl and call 911 for emergency care. How will the Benadryl, acting as an antihistamine, benefit this patient? A) It helps the patient's immune system to deactivate the allergen B) It acts as an allergen to stop the allergic reaction through histamine stimulation C) It inhibits the constriction of the patient's small airways by blocking histamine D) It dilates the patient's blood vessels and delivers more blood to the brain Answer: C Diff: 3 Page Ref: 631 Objective: 21-3 42) What is the primary difference between an anaphylactic reaction and an anaphylactoid reaction? A) An anaphylactoid reaction may occur without prior sensitization to the antigen; anaphylaxis requires prior sensitization B) Anaphylactoid reactions are less severe than anaphylactic reactions C) Epinephrine is contraindicated in anaphylactoid reactions, but not in anaphylactic reactions D) Anaphylactoid reactions are commonly caused by environmental substances; anaphylactic reactions are most commonly caused by foods Answer: A Diff: 2 Page Ref: 632-633 Objective: 21-5 43) You have been summoned to an apartment for a 58-year-old female who complains of hives on her chest and abdomen. When asked, she states that she has had several episodes of the hives over the past six months, but allergy testing has not been able to pinpoint a cause. Based on this information, how would you best classify the cause of the patient's reaction? A) Iatrogenic B) Anaphylactoid C) Anaphylactic D) Idiopathic Answer: D Diff: 1 Page Ref: 633 Objective: 21-2 44) You are providing bag-valve-mask ventilations to a patient in anaphylactic shock. There is increasing resistance to ventilation, despite your use of proper ventilation technique. What is the best way to address this situation? A) Stop ventilations and administer an albuterol metered-dose inhaler B) Occlude the pop-off valve on the bag-valve mask and continue ventilations C) Increase both the ventilatory rate and the force of ventilation D) Turn the patient to her side and apply manual pressure over the epigastrium Answer: B Diff: 2 Page Ref: 634 Objective: 21-8 12 Copyright © 2018 Pearson Education, Inc.

45) You have been called to a kindergarten class for a 5-year-old boy who is allergic to peanuts. He ate some peanuts in the form of a birthday treat brought to school by a classmate who is celebrating a birthday. From the door of the classroom, you note the patient to have swollen lips and hives to his face and arms. The patient also looks unresponsive. As you begin the primary assessment, what should be a priority action? A) Contact the patient's parents for permission to treat him B) Inspect the inside of the patient's mouth and assure an open airway C) Determine how many peanuts the patient ingested D) Check breath sounds for wheezing and assess the vital signs Answer: B Diff: 2 Page Ref: 634 Objective: 21-8 46) A patient has been stung by a bee. Which of these findings would be present in anaphylaxis, but not in a local allergic reaction? A) Anxiousness B) Swelling at the site of the sting C) Hives D) Hypotension Answer: D Diff: 2 Page Ref: 638 Objective: 21-4 47) A patient with an allergy to bees and a prescribed EpiPen was working in the garden when she noted swelling to the right side of her face. Assessment reveals swelling that is red and hot to the right cheek area. Her airway is patent, respirations easy and adequate, and radial pulse strong. Her vital signs are pulse, 76; respirations, 16 breaths/min; blood pressure, 156/62 mmHg; and SpO2, 99%. What should be the EMT's action at this time? A) Perform a secondary assessment B) Assist the patient with her EpiPen C) Treat the patient's condition as an anaphylactoid reaction D) Provide immediate and rapid transport Answer: A Diff: 2 Page Ref: 635 Objective: 21-8 48) A patient develops an allergic reaction after taking penicillin. In this case, the penicillin would be recognized by the body as: A) A form of immunoglobulin E B) An antigen C) A mast cell D) An antibody Answer: B Diff: 1 Page Ref: 630 Objective: 21-6

13 Copyright © 2018 Pearson Education, Inc.

49) A patient has just taken her first dose of Zocor for high cholesterol. Within minutes, she begins to develop hives, shortness of breath, and low blood pressure. In this case, the EMT would recognize that the patient is experiencing: A) A sensitized immune response B) Anaphylaxis C) A hypersensitivity reaction D) An anaphylactoid reaction Answer: D Diff: 2 Page Ref: 632-633 Objective: 21-5 50) Regarding the pathophysiology underlying anaphylaxis, which of these statements is true? A) Histamine causes the capillaries to become leaky and bronchioles to constrict B) High blood pressure forces blood from the blood vessels, resulting in airway edema C) Immunoglobulin E molecules make antibodies that cause bronchoconstriction and vasodilation D) Smooth muscle dilation in the lungs impairs the exchange of oxygen Answer: A Diff: 2 Page Ref: 631-632 Objective: 21-3 51) Which of these statements is true regarding both anaphylactic and anaphylactoid reactions? A) Anaphylaxis is caused by medications; anaphylactoid reactions are not B) Administering epinephrine in an anaphylactoid reaction can be fatal C) Both anaphylactoid and anaphylactic reactions are treated in the same way D) Anaphylactoid reactions seldom involve the airway; anaphylaxis does Answer: C Diff: 2 Page Ref: 633 Objective: 21-8 52) A mother has called you for her 9-year-old daughter, who was stung in the right hand by a bee. She states that her older daughter is allergic to bees and she is scared that the patient may also be allergic, although she has never been stung before. Assessment of the patient reveals a red, painful, and swollen area on the back of the girl's right hand. The primary assessment is unremarkable and her vital signs are normal. The mother hands you her older daughter's epinephrine auto-injector. You would: A) Administer the epinephrine into the patient's thigh B) Ask the mother to administer the auto-injector into the patient's thigh C) Wash the patient's hand with soap and water D) Contact medical direction for permission to use the epinephrine auto-injector Answer: C Diff: 2 Page Ref: 631 Objective: 21-8

14 Copyright © 2018 Pearson Education, Inc.

53) Within the body, the release of histamine causes dilation of the: A) Blood vessels B) Small airways C) Upper airway D) Salivary glands Answer: A Diff: 1 Page Ref: 631-632 Objective: 21-3 54) You are providing a continuing education session for your EMS department regarding anaphylaxis, as you have just returned from a national conference at which current issues related to this pathology were discussed. During your session, one of your fellow EMS providers asks you how the body can "suddenly" become allergic to something. What is your best response? A) "The body's white blood cells are uniquely sensitive to certain allergens, and when they are exposed to these allergens for the first time, the body will develop an allergic reaction." B) "It is actually not true that someone 'suddenly' becomes allergic to something. Research shows the person was always allergic to it, but incorrectly assumed he was exposed once before." C) "After a repeat exposure, the newly formed antigens of the body will attack the red blood cells, causing them to die, as well as causing airway closure and bronchoconstriction." D) "After exposure to a new antigen, the body will create antibodies to fight it; these antibodies can be the root cause of the anaphylactic response later." Answer: D Diff: 3 Page Ref: 631 Objective: 21-2

15 Copyright © 2018 Pearson Education, Inc.

Prehospital Emergency Care, 11e (Mistovich et al.) Chapter 22 Toxicologic Emergencies 1) Which of these statements about poisonings is correct? A) A poison is any substance that impairs a person's health by its chemical action B) All toxins are poisonous when in contact with the human body C) The majority of poisonings are intentional and result in homicide or suicide D) The number one cause of poisoning in the United States is medications Answer: A Diff: 2 Page Ref: 651 Objective: 22-1 2) Which of these correctly identifies the routes by which a poison can enter the human body? A) Exposure, ingestion, inhalation, injection B) Topical, inhalation, nasal-oral, absorption C) Ingestion, inhalation, injection, absorption D) Inhalation, injection, nasal-oral, visual Answer: C Diff: 1 Page Ref: 652 Objective: 22-3 3) A 53-year-old female patient has intentionally ingested a large amount of Valium. This medication is primarily absorbed into the body through which structure? A) Stomach B) Esophagus C) Large intestine D) Small intestine Answer: D Diff: 2 Page Ref: 652 Objective: 22-5 4) What is the most common route by which a poison enters the body? A) Inhalation B) Ingestion C) Exposure D) Topical Answer: B Diff: 1 Page Ref: 652 Objective: 22-3

1 Copyright © 2018 Pearson Education, Inc.

5) You have been called for a 13-year-old boy who was caught sniffing glue by his parents. The boy is lethargic and confused, complains of "burning" in his mouth and nose, and has nausea and vomiting. Based on the clinical presentation and mechanism of illness, how has this poison affected the patient's brain? A) Being absorbed through the small intestine into the bloodstream and then going to the brain B) Directly contacting the brain by crossing the mucous membranes of the mouth and nose C) Crossing from the lungs into the bloodstream and then circulating to the brain D) Absorbing fumes through the skin, nose, and eyes and then going on to the brain Answer: C Diff: 3 Page Ref: 652 Objective: 22-8 6) A 44-year-old patient was found unresponsive at home. Based on his mental status and the fact that he was burning a kerosene heater in an enclosed room, you suspect carbon monoxide poisoning. Based on this information, which route of exposure would you recognize as responsible for the poisoning? A) Nasal-oral B) Inhalation C) Ingestion D) Absorption Answer: B Diff: 1 Page Ref: 652 Objective: 22-3 7) A 31-year-old female patient was stung by a hornet while golfing. Your assessment reveals her to be confused with stridorous respirations. Her skin is warm and flushed and covered with hives. A stinger is located on the back of her neck. Based on this information, you would recognize that the route of poisoning for this patient is: A) Absorption B) Anaphylactic C) Topical D) Injection Answer: D Diff: 1 Page Ref: 652 Objective: 22-3 8) A 44-year-old male patient had a pesticide splashed into his eyes. Aside from local irritation and damage to the eyes, he developed systemic complications when the pesticide entered and circulated throughout his body. When completing the prehospital care report, the EMT would note the route of exposure as: A) Absorption B) Visual C) Injection D) Topical Answer: A Diff: 1 Page Ref: 652 Objective: 22-3 2 Copyright © 2018 Pearson Education, Inc.

9) When assessing a patient who has been exposed to a poisonous substance, it is essential to understand: A) Vomiting indicates that the poison has not yet been absorbed into the body B) The signs and symptoms of toxic exposure will vary depending on the specific poison C) The route of entry into the body will determine the signs and symptoms D) Absorption into the body has not occurred if the signs of illness are not apparent within 15 minutes Answer: B Diff: 1 Page Ref: 652 Objective: 22-4 10) Which of these is the EMT's highest priority when presented with a patient who has been exposed to a poisonous substance? A) Identify the poison B) Induce vomiting C) Perform a primary assessment D) Administer activated charcoal Answer: C Diff: 1 Page Ref: 653-654 Objective: 22-4 11) Which of these statements regarding antidotes is correct? A) There are many antidotes for a large number of poisons and toxic substances B) An antidote will neutralize or counteract the effects of a poison or toxic substance C) Antidotes are beneficial because they will remove the poison from the body D) An antidote can only be given in the emergency department by the physician or nurse Answer: B Diff: 2 Page Ref: 653 Objective: 22-1 12) An 11-year-old boy with a history of mild developmental delays has possibly ingested parts of a household plant; his mother states that she found him eating the plant, then immediately called 911. You note that the plant has been overturned and is missing a considerable number of leaves. Assessment reveals the boy to be lethargic and confused; he complains of abdominal pain, accompanied by nausea and vomiting. His airway is patent and his respirations are adequate. Which of these assessment findings would be most suggestive that the poison from the plant leaves has been absorbed into the patient's body? A) Confusion and lethargy B) Nausea and vomiting C) Abdominal pain D) Leaves missing from the plant Answer: A Diff: 2 Page Ref: 654 Objective: 22-5

3 Copyright © 2018 Pearson Education, Inc.

13) The grandfather of an alert and oriented 2-year-old girl found her on the floor playing with pills from his open bottle of Digoxin (a heart medication that slows the heart rate). He cannot remember how many pills were in the bottle, but he thinks that some are missing and suspects that his granddaughter may have eaten them thinking that they were candy. Assessment reveals the patient's airway to be open and breathing adequate. Her pulse is appropriate for a child her age and her skin warm and dry. Her grandfather is not sure of how long ago she may have taken the pills. Given the assessment findings, the EMT would assume: A) Not enough Digoxin was taken to harm the child B) The child apparently did not take any Digoxin C) The child must have spit the pills out before swallowing them D) The Digoxin pills have yet to be absorbed into the body Answer: D Diff: 2 Page Ref: 654 Objective: 22-5 14) A confused and lethargic patient reports that she took a large number of antianxiety pills because she was angry with her husband. Her airway is patent and her breathing adequate. Her pulse is normal and her skin is warm and dry. During the reassessment, the patient reports that she is now dizzy and feels extremely nauseated. The EMT would immediately: A) Perform a jaw-thrust maneuver B) Place the patient in a lateral recumbent position C) Prepare the suction device D) Check the patient's vital signs Answer: B Diff: 2 Page Ref: 654 Objective: 22-21 15) A frantic mother called 911 because she found her 18-month-old daughter playing with a bottle of drain cleaner. When assessing this patient, what would provide the strongest evidence that the child ingested some of the liquid? A) Persistent crying B) Burns to her mouth C) Garbled speech D) Elevated heart rate Answer: B Diff: 1 Page Ref: 654 Objective: 22-11

4 Copyright © 2018 Pearson Education, Inc.

16) You have been called to a residence for a 14-year-old male patient, who states that he intentionally took some medication to kill himself. Assessment indicates that he is alert and oriented, and has an open airway and adequate breathing. Given the situation, the EMT's next action should be to: A) Ask the patient if he feels nauseated B) Determine what was taken C) Check the radial pulse and skin characteristics D) Inquire why the patient took the medication Answer: C Diff: 1 Page Ref: 655 Objective: 22-5 17) You have been called for an unresponsive patient. When you arrive at the scene, hysterical family members direct you to an unresponsive 32-year-old woman who is lying on the bedroom floor. Her husband says that she left a note saying that she wanted to kill herself and took an unspecified poison. Assessment reveals a green-colored liquid in her mouth and respirations that are rapid, shallow, and gurgling. The patient's pulse is weak, and her skin is cool and dry with cyanosis to her fingertips. Your immediate action would be to: A) Start positive pressure ventilation B) Extricate the patient and provide care en route to the hospital C) Manually open the patient's airway and perform suction D) Read the note for clues to what the patient took Answer: C Diff: 2 Page Ref: 678 Objective: 22-5 18) A confused female patient states that she was intoxicated and accidentally drank a glass of household bleach. When transporting this patient, which complication would you continually assess for? A) Vomiting B) Chest pain C) Headache D) Diaphoresis Answer: A Diff: 1 Page Ref: 654 Objective: 22-5

5 Copyright © 2018 Pearson Education, Inc.

19) You have been summoned to a residence for a child who ingested a liquid pesticide. On scene, the worried parents inform you that their 4-year-old son and some friends were playing in the garage and apparently found some pesticide, which their son says he tasted. Assessment indicates that he is alert, oriented, and crying and has some white powder around his mouth. After performing the initial assessment and finding no life-threatening conditions, it is imperative that the EMT make which question or statement: A) "Do you think this was an intentional act?" B) "I need to see the other children." C) "Are all of his shots up-to-date?" D) "These chemicals need to be put out of reach." Answer: B Diff: 2 Page Ref: 654 Objective: 22-4 20) You have assessed a 25-year-old female who states that she took a large number of antidepressants and other medications in an effort to end her life. The primary assessment reveals no life-threatening conditions. Why is it important to get the patient's medical history at this time, rather than later on in the assessment or during transport? A) Patients are generally scared early on and more truthful with information B) Further assessment may not be needed if there is no past medical history C) The police on scene will need the information prior to transport from the scene D) The patient may become confused and be unable to provide information Answer: D Diff: 2 Page Ref: 654 Objective: 22-4 21) A middle-aged male with behavioral problems states that he drank some "chemicals" to kill the bugs in his body. The patient is alert and oriented and shows no life-threatening conditions to the airway, breathing, or circulation. At this point, which of these questions should the EMT ask? A) "Have you thrown up?" B) "When did you drink it?" C) "Do you know exactly what it was you drank?" D) "Are you taking other medications?" Answer: C Diff: 2 Page Ref: 654-655 Objective: 22-5 22) After determining that a patient intentionally overdosed on blood pressure medications, which of these questions would be most appropriate? A) "Why did you take so many pills?" B) "How many pills did you take?" C) "Were you confused about the proper dose?" D) "Are you on other medications?" Answer: B Diff: 2 Page Ref: 654-655 Objective: 22-5 6 Copyright © 2018 Pearson Education, Inc.

23) A 14-year-old female patient has taken an excessive amount of Tylenol with codeine. Family members were asleep when they heard a crash and found her at the bottom of the stairs. When they asked the adolescent if she was okay, she replied that her boyfriend broke up with her and she saw no point in going on with her life, so she took an overdose. When you arrive by the patient's side, you note that she is responsive to verbal stimuli with eye opening and is breathing, has an intact airway, and is breathing somewhat shallow but still adequately. Your first action would be to: A) Open the airway with the head-tilt, chin-lift maneuver B) Determine exactly how much medication she took C) Start positive pressure ventilation with oxygen D) Take manual spine motion restriction precautions Answer: D Diff: 2 Page Ref: 675 Objective: 22-4 24) An alert and oriented 5-year-old child drank some ammonia that was placed in a cup in the garage. He is very scared and complains of abdominal pain, but states that he has not vomited. Assessment shows ammonia in and around his mouth as well as on his breath. He has an open airway and is breathing adequately. His pulse and skin show no abnormalities. Appropriate care for this patient would include: A) Rinsing his face and mouth with water B) Inducing vomiting C) Initiating positive pressure ventilation D) Administering activated charcoal Answer: A Diff: 2 Page Ref: 656 Objective: 22-5 25) Which set of instructions given to another EMT would be most appropriate when managing a patient who ingested numerous household cleaners in an attempt to kill himself? A) "I need you to write down all of the cleaners he may have taken so we can let the hospital know." B) "Let's gather up all of the containers of the chemicals he may have taken and take them with us." C) "We need to pour a sample of each cleaner that he may have taken into containers and take the samples with us." D) "Before we leave, please call the hospital and tell them what he took so that they will know." Answer: B Diff: 2 Page Ref: 656 Objective: 22-5

7 Copyright © 2018 Pearson Education, Inc.

26) Medical direction has ordered you to administer activated charcoal to a patient who ingested a large amount of poison. When looking in your medical kit, which medication would you prepare and administer? A) Nitrostat B) Actidose C) Glyburide D) Ecotrin Answer: B Diff: 1 Page Ref: 656 Objective: 22-7 27) For which of these patients would activated charcoal be indicated given the type of poisoning? A) A 24-year-old female who was sniffing glue and now complains of a headache B) A 17-year-old male who drank bleach 30 minutes ago C) A 27-year-old who intentionally took a large amount of Tylenol 45 minutes ago D) A 53-year-old female who took an overdose of antidepressants earlier in the day Answer: C Diff: 2 Page Ref: 656 Objective: 22-7 28) The family of a 31-year-old female patient reports that she has taken an overdose of her father's diabetes medication. Assessment reveals her to be unresponsive, lying on the floor, with snoring respirations. Her pulse is rapid and her skin is cool to the touch. The patient has no history of diabetes but is allergic to penicillin. It is believed that she took the medication within the past hour. Your initial intervention in caring for this patient would be to: A) Administer activated charcoal B) Induce vomiting C) Perform a jaw-thrust maneuver D) Administer glycogen or intranasal naloxone Answer: C Diff: 2 Page Ref: 655 Objective: 22-5 29) A 42-year-old patient weighs 154 pounds (70 kg). How much activated charcoal would you administer to him? A) 70 grams B) 100 grams C) 154 grams D) 500 grams Answer: A Diff: 2 Page Ref: 657, Figure 22-3 Objective: 22-7

8 Copyright © 2018 Pearson Education, Inc.

30) Medical direction has ordered the administration of activated charcoal to a 3-year-old boy. Which of these doses would be most appropriate? A) 100 mg B) 1 gram/kg C) 50 grams D) 0.5 mg/kg Answer: B Diff: 2 Page Ref: 657, Figure 22-3 Objective: 22-7 31) Medical direction has ordered the administration of activated charcoal to an adult patient who has overdosed on several unspecified medications. Prior to administering the medication, the EMT must: A) Shake the medication thoroughly to mix it B) Determine whether the patient has a prescription for the medication C) Determine whether the patient has a history of diabetes D) Ask the patient if he has had diarrhea over the past two weeks Answer: A Diff: 1 Page Ref: 657, Figure 22-3 Objective: 22-7 32) Which action indicates that the EMT is correctly administering activated charcoal to a patient who has ingested a toxic substance? A) The charcoal is mixed with milk B) Sugar is added to make the charcoal taste better C) The charcoal is squirted into the mouth D) The patient drinks the charcoal through a straw Answer: D Diff: 1 Page Ref: 657, Figure 22-3 Objective: 22-7 33) A patient who has ingested a toxic substance is to receive activated charcoal. The EMT knows that the charcoal will benefit the patient by: A) Coating the small intestine, limiting absorption of the poison B) Neutralizing the poison in the bloodstream C) Adsorbing the poisonous substance in the stomach D) Causing vomiting, thereby removing the poison from the body Answer: C Diff: 3 Page Ref: 657, Figure 22-3 Objective: 22-7

9 Copyright © 2018 Pearson Education, Inc.

34) You are preparing to administer activated charcoal to a patient who has ingested a poison. The activated charcoal authorized by your medical director also contains a cathartic. Which of these statements would you make to the patient regarding this additive? A) "You may experience diarrhea after taking the charcoal." B) "This charcoal will most likely give you a slight headache." C) "The additive in the charcoal may cause you to vomit." D) "If you have a heart condition, I cannot give you this medication." Answer: A Diff: 3 Page Ref: 656 Objective: 22-7 35) You are transporting a patient who overdosed on medications from a community emergency department (ED) to a large medical center for continued care. In the ED, the patient received two doses of activated charcoal. When performing your reassessment of the patient, which of the findings would be most concerning? A) Dark black stool B) Nausea C) Red-colored stool D) Diarrhea Answer: C Diff: 2 Page Ref: 657, Figure 22-3 Objective: 22-7 36) An 18-year-old female patient has eaten a poisonous mushroom. Medical direction has ordered you to administer activated charcoal. Immediately following administration, the patient vomits the medication. She remains alert and oriented with no current life threats to the airway, breathing, or circulation. Your next action would be to: A) Administer a glass of milk B) Place the patient supine C) Reassess and continue transport D) Readminister the activated charcoal Answer: D Diff: 2 Page Ref: 657, Figure 22-3 Objective: 22-7 37) Which of these statements made by an EMT indicates an accurate understanding of inhalation poisonings? A) "The onset of signs and symptoms associated with an inhalation poisoning are often rapid." B) "Inhalation poisons affect the lungs, but they do not circulate throughout the body." C) "It is probably safe to enter the scene of an inhalation emergency if I do not smell anything." D) "Most inhalation poisonings are easily identified by residue or burns around the patient's mouth." Answer: A Diff: 3 Page Ref: 658 Objective: 22-8

10 Copyright © 2018 Pearson Education, Inc.

38) You have been called to a railroad yard for an unknown emergency. When you arrive on scene, you are directed to a tanker car, where two men are lying on the ground next to the railcar. The men are mechanics for the railroad company and were performing some type of repair work on the valves for the tanker car. The only other person around is a security guard, who saw the men suddenly collapse on a surveillance video camera and called 911 immediately. He has an automated external defibrillator (AED) at his side. Your initial action would be to: A) Quickly retrieve the patients and move them to safety B) Take manual spine motion restriction of the head and neck and assess the ABCs for each patient C) Remain at a safe distance and call for fire department assistance D) Determine which chemical the tanker contains and assess the leaks Answer: C Diff: 2 Page Ref: 658-659 Objective: 22-4 39) A worker at a chemical plant was working on a high-pressure tank containing chlorine gas when the valve broke, filling the room with chlorine gas and throwing the patient forcefully against the wall. He was unresponsive and trapped in the room for 10 minutes, until being pulled out by firefighters. When the patient is brought to your location, he appears unresponsive. Your first action in caring for this patient would be to: A) Assess breathing and auscultate breath sounds B) Open the airway using the head-tilt, chin-lift maneuver C) Determine the presence or absence of a pulse D) Take manual spine motion restriction of the head and neck and perform the jaw-thrust maneuver Answer: D Diff: 2 Page Ref: 659 Objective: 22-4 40) Firefighters have pulled a 62-year-old male patient from a bedroom that was on fire. According to the firefighters, the patient was smoking in bed and fell asleep. They found him supine on the floor. Assessment reveals him to be responsive to painful stimuli, with soot in his airway. He is breathing rapidly, but has no evidence of burns to his body. His pulse is 124 beats/min, respirations are 24 breaths/min, blood pressure is 146/90 mmHg, and SpO2 is 95%. After providing the appropriate care, which of these questions would be the most important to ask the firefighters prior to transport? A) "How did the fire start?" B) "How long was the patient trapped in the room?" C) "Which materials were burning in the room?" D) "How fast was the patient breathing when you found him?" Answer: B Diff: 2 Page Ref: 659 Objective: 22-8

11 Copyright © 2018 Pearson Education, Inc.

41) A 44-year-old male patient was using an industrial drain cleaning solvent to clean the drains in his house. The solvent corroded through the pipes on the second floor, spilling into the walls and filling the house with fumes. The patient was in the house for several minutes prior to getting himself out. The primary assessment reveals him to be alert and oriented with a patent airway. He is complaining of dizziness and has rapid respirations. His radial pulse is fast and strong, and his skin is warm and pink. Which action should the EMT take first? A) Insert a nasopharyngeal airway B) Start positive pressure ventilation C) Administer supplemental oxygen D) Provide rapid transport with the patient in semi-Fowler's position Answer: C Diff: 2 Page Ref: 659 Objective: 22-8 42) A patient working at an ice plant is trapped in a room filled with sulfur dioxide. You don a self-contained breathing apparatus and enter the room, finding the patient unresponsive on the floor. The room remains filled with the gas. Your immediate priority in caring for this patient is to: A) Assess the airway, breathing, and circulation B) Remove the patient from the immediate environment C) Start positive pressure ventilation D) Loosen all tight-fitting clothing Answer: B Diff: 2 Page Ref: 658-659 Objective: 22-8 43) A 17-year-old male patient was stung by an unidentified insect. On scene, the sting site to his right forearm showed some redness and mild swelling. During the reassessment, which statement made by the patient would indicate that venom from the sting is being absorbed into the bloodstream? A) "The bite seems to be hurting worse now." B) "I think that my arm is more swollen." C) "The area around the bite feels numb." D) "I feel sick to my stomach and dizzy." Answer: D Diff: 2 Page Ref: 661 Objective: 22-9

12 Copyright © 2018 Pearson Education, Inc.

44) A 32-year-old female patient calls 911 to say that she injected herself with some heroin 15 minutes ago, which she just bought from "some guy" on the street. She has never tried heroin and is now scared about what she has done. Assessment reveals her to be conscious and well oriented. Her airway is open and she is breathing 14 times a minute. Her pulse is moderate in strength and her skin is warm and dry. What would be the appropriate care for this patient? A) Monitor her and initiate transport B) Place a warm pack to the injection site C) Administer intranasal naloxone D) Apply a tourniquet above the site of injection Answer: A Diff: 2 Page Ref: 661 Objective: 22-17 45) At the scene of an industrial accident, you are presented with a male patient who was exposed to a powerful pesticide in powder form. The patient states that he was moving sacks of the pesticide when one broke open, creating a cloud of powder that surrounded him, and which he then inhaled. During the incident, his face and arms became covered with the powder. At the present time, he complains of difficulty breathing and burning of his skin on his face and arms. You note that some of the pesticide remains on these areas of the body. Your first action in caring for this patient would be to: A) Apply supplemental oxygen B) Brush off the remaining pesticide C) Flush the pesticide with copious amounts of water D) Obtain vital signs and a medical history Answer: A Diff: 2 Page Ref: 662 Objective: 22-10 46) While he was changing the battery on a truck, acid from the battery splashed into a 66-yearold man's right eye. The patient now complains of burning pain to his right eye and is having a hard time keeping it open. He is alert and oriented and has no problems with his airway, breathing, or circulation. Your next action would be to: A) Cover both eyes and provide immediate and rapid transport B) Rinse the eye with a mixture of sterile water and sodium bicarbonate C) Place a cold pack to the eye and immediately transport the patient to the hospital D) Irrigate the eye with water for a minimum of 20 minutes Answer: D Diff: 2 Page Ref: 662 Objective: 22-11

13 Copyright © 2018 Pearson Education, Inc.

47) Following a large wedding that occurred the previous night, health authorities announce that some of the food was contaminated with Salmonella. Consequently, your EMS service receives notice that it may receive a significant number of calls involving food poisoning. A new EMT asks you how to treat patients with food poisoning. What is your best response? A) "After assessing the patient, the administration of activated charcoal can be effective if there are no obvious contraindications." B) "The primary role of EMS in this situation is just to transport the patient to the hospital." C) "If the patient has a fever, we can administer a medication to make him vomit so as to get the food out of his system." D) "You should treat the patient based on his signs and symptoms, just like a patient who ingested any other poison." Answer: D Diff: 2 Page Ref: 655 Objective: 22-5 48) You have been called to a residence for a 31-year-old patient who is complaining of confusion, nausea, vomiting, and a headache. His symptoms started this morning and have been getting worse all day. Despite lying in bed and resting all day, he is more fatigued than before. Which of these statements made by the patient would the EMT recognize as most important in relation to the patient's present complaint? A) "I just turned the furnace on last night." B) "I tried some raw seafood last week." C) "My wife is taking penicillin, to which I am allergic." D) "I have not been sleeping well the past few nights." Answer: A Diff: 3 Page Ref: 666-667 Objective: 22-11 49) You have been called for a young female patient who overdosed on several drugs. What is the most essential step for the EMT to perform while caring for this patient? A) Identify and correct the loss of vital function(s) B) Identify the drug(s) and amount(s) taken C) Determine whether the patient has a history of drug abuse D) Identify medical conditions that may be worsened by the drug(s) Answer: A Diff: 1 Page Ref: 678 Objective: 22-5

14 Copyright © 2018 Pearson Education, Inc.

50) You are by the side of a young female college student who is unresponsive. Friends say that she is normally not a heavy drinker, but tonight she drank an excessive amount of alcohol because her boyfriend broke up with her at the party. She responds to painful stimuli and has snoring respirations. Her respiratory rate is 8 breaths/min and her radial pulse is weak. Your immediate action would be to: A) Apply oxygen via a nonrebreather mask B) Administer positive pressure ventilation C) Determine whether drugs are also involved D) Perform a manual airway maneuver Answer: D Diff: 2 Page Ref: 675 Objective: 22-14 51) You are respond to an "unknown/unresponsive" emergency called in by a bystander. On scene, you find the female patient to be conscious but confused and extremely nauseated; she has vomited twice. Examination of the vomitus reveals fragments of different pills. Her past medical history includes diabetes and alcoholism. En route to the hospital, the patient vomits again and suddenly becomes unresponsive. Your first action would be to: A) Get another set of vital signs B) Contact medical command C) Position the patient on her side D) Check the patient's blood sugar Answer: C Diff: 2 Page Ref: 653 Objective: 22-14 52) A 49-year-old patient with alcoholism has fallen down a flight of stairs after consuming twofifths of whiskey. He presents with confused and slurred speech and the smell of alcohol on his breath. Family states that he is "hopeless" and has been drinking more and more every night. They also say that he must be more drunk than usual because he usually does not slur his speech. Assessment reveals his airway to be open, breathing adequate, and circulation intact. He has a hematoma from the fall to the left side of his head and complains of pain to his left wrist. Given these assessment findings, the EMT would be most concerned about the: A) Amount of alcohol consumed B) Possible brain injury C) Possible wrist fracture D) Lack of family support Answer: B Diff: 2 Page Ref: 675 Objective: 22-14

15 Copyright © 2018 Pearson Education, Inc.

53) On arrival at a tailgate party in a parking lot near a college, you observe a crowd around a young man who is seizing. As you get equipment from the ambulance and make your way to the patient, his friends tell you that the patient has been up all night "partying" by drinking alcohol and smoking crack cocaine. At the patient's side, you note that he is no longer seizing, but is unresponsive and has snoring respirations. His respirations are labored at 16 breaths/min and his radial pulse is strong. His skin is cool and diaphoretic. Friends also inform you that he has diabetes; about an hour ago, they heard him say that he thought his blood sugar was becoming low. Which of these interventions would most likely be included in your care? A) Open the airway, provide oxygen, and administer oral glucose B) Open the airway, check the blood glucose level, and administer oral glucose C) Insert an oral airway, provide positive pressure ventilation, and administer oral glucose D) Open the airway, provide oxygen, and give rapid transport to the hospital Answer: D Diff: 2 Page Ref: 678 Objective: 22-4 54) On the scene of a drug-related emergency, the 22-year-old patient tells you that he has been taking amphetamines because he likes the way that they make him feel. Over the past two days, he has been taking more and more, and today he feels as though his heart is racing. He also tells you that he recognizes he has a problem and wants to get help. When you are assessing the patient, which sign or symptom would provide further evidence that the patient has been abusing amphetamines? A) Increased blood pressure B) Auditory hallucinations C) Constricted pupils D) Fatigue and prolonged sleep Answer: A Diff: 2 Page Ref: 676 Objective: 22-14 55) You have been called for a 46-year-old male patient who just smoked crack cocaine. In reviewing his past medical history, which of these conditions would be most of most concern? A) Alcoholism B) Migraine headaches C) Heart disease D) Diabetes Answer: C Diff: 3 Page Ref: 689 Objective: 22-14

16 Copyright © 2018 Pearson Education, Inc.

56) You are responding to a drug-related emergency. While you are en route to the scene, the dispatcher informs you that the patient has overdosed on an unspecified narcotic. Which of these statements made by your partner indicates an understanding of overdoses with narcotic drugs? A) "We will have to be careful because narcotic overdoses can cause hallucinations, and the patient may be violent." B) "When performing the primary assessment, we need to be alert for respiratory depression and have the naloxone ready." C) "If the patient is conscious, we will have to ask about chest pain since narcotics can increase the heart's workload." D) "We need to look at the pupils. If they are constricted, the patient has most likely taken a narcotic and we will have to administer naloxone intravenously." Answer: B Diff: 2 Page Ref: 686 Objective: 22-18 57) You are assessing a young male patient who was found in an alley by sanitation workers. Assessment reveals the patient responds to painful stimuli by opening his eyes, but then closes them once you stop pinching the muscles on his neck. His airway is open and his breathing is shallow. His skin is cool and diaphoretic, and his pupils are constricted and do not react to light. Based on these assessment findings, the EMT would be suspicious of: A) Stroke or hypoglycemia B) Narcotic overdose C) Traumatic assault D) Methamphetamine overdose Answer: B Diff: 2 Page Ref: 685-686 Objective: 22-17 58) A patient who is agitated and hyperactive with dilated pupils and tachypnea has most likely ingested: A) Heroin B) Hallucinogens C) Methadone D) Crack cocaine Answer: D Diff: 2 Page Ref: 689 Objective: 22-14

17 Copyright © 2018 Pearson Education, Inc.

59) You have arrived at a home where the parents found their 13-year-old son in the garage inhaling propane from the family grill. Your assessment reveals the patient to be alert and oriented, with an open airway. His breathing is labored, and his pulse is rapid for his age. When conducting the secondary assessment, which of these findings or history would be of the greatest concern? A) Chemical burn to the right hand B) Abdominal pain C) Bilateral wheezing D) Suicide attempt two years ago Answer: C Diff: 2 Page Ref: 659 Objective: 22-22 60) The EMT is transporting a patient with the diagnosis of Wernicke-Korsakoff syndrome. The EMT recognizes that this patient has a past medical history of: A) Alcoholism B) Drug abuse C) Pancreatic failure D) Liver failure Answer: A Diff: 2 Page Ref: 683 Objective: 22-14 61) This is the third day that a patient is in withdrawal from alcohol. If called to care for this patient, the EMT would recognize the need for: A) Restraints B) Positive pressure ventilation C) Warm blankets D) Seizure precautions Answer: D Diff: 3 Page Ref: 684 Objective: 22-16 62) You are called to an alcoholic crisis center to transport a patient experiencing delirium tremens. When assessing the patient, which signs or symptoms would you associate with this condition? A) Seizure activity, memory impairment, bradycardia, tremors B) Constricted pupils, hypothermia, confusion, hypotension C) Loss of memory, hallucinations, fever, dilated pupils D) High fever, seizure activity, sweating, lethargy Answer: C Diff: 2 Page Ref: 684 Objective: 22-16

18 Copyright © 2018 Pearson Education, Inc.

63) You have been called for a patient with alcoholism who is experiencing a medical emergency. On scene, a home health nurse informs you that the patient is experiencing delirium tremens. As an EMT, you recognize that: A) The patient is extremely intoxicated and in danger of dying B) This is a life-threatening condition requiring rapid transport C) The patient is in the early stages of alcohol withdrawal D) The patient will require the administration of glucose Answer: B Diff: 2 Page Ref: 684 Objective: 22-16 64) Which of these complaints would the EMT expect from a person who has just used crack cocaine for the first time? A) "I have chest pain." B) "My heart feels slow." C) "I see spiders." D) "My nose feels raw." Answer: A Diff: 2 Page Ref: 689 Objective: 22-14 65) Which of these patients would the EMT recognize as a drug abuser? A) A 41-year-old male who tried marijuana for the first time B) A 39-year-old female who takes pain medications even though she is not in pain C) A 46-year-old female who used cocaine for 10 years but has not used it in 5 years D) A 56-year-old male who accidentally took an additional dose of blood pressure medication Answer: B Diff: 2 Page Ref: 684 Objective: 22-17 66) Which of these statements indicates that the EMT understands the condition of withdrawal? A) "Withdrawal is typically not a life-threatening situation and is easily managed by EMS." B) "Withdrawal indicates that a patient is addicted to a drug and is presently overdosed on that drug." C) "Withdrawal from drugs or alcohol can be just as serious a medical emergency as an overdose." D) "Withdrawal is a term that is applied to the cessation of street drugs to which the patient is addicted, not prescription drugs." Answer: C Diff: 3 Page Ref: 684 Objective: 22-16

19 Copyright © 2018 Pearson Education, Inc.

67) You have been called for a "drunk" male patient at a halfway house known to provide living quarters to persons with drug and alcohol problems. Residents tell you that the patient is a heavy drinker and must have had too much last night. This morning he was found "passed out" with alcohol on his breath. Quick assessment of the scene reveals the patient to be prone on a dirty bed in a room littered with liquor containers. Aside from the alcohol, which of these observations would be most significant to the EMT? A) Glucometer on the table B) Smell of urine in the room C) Dried vomit on the floor D) An empty fast-food bag on the bedside table Answer: A Diff: 2 Page Ref: 683 Objective: 22-6 68) You are en route for a possible heroin overdose. As an EMT, you would recognize heroin fits best into which drug category based on its effects? A) Sedatives B) Hallucinogens C) Psychedelics D) Stimulants Answer: A Diff: 1 Page Ref: 686 Objective: 22-14 69) A patient informs you that he smoked marijuana all night and now has great anxiety, hunger, and tremors to both hands. The EMT would recognize marijuana is best described as: A) A hallucinogen B) A cannabis product C) An appetite stimulant D) A sedative Answer: B Diff: 1 Page Ref: 689 Objective: 22-8 70) A drug addict informs you that he has been taking OxyContin and now requires more and more of the drug to take his chronic back pain away. The EMT would recognize this patient as experiencing: A) Overdose B) Dependence C) Tolerance D) Adverse reaction Answer: C Diff: 1 Page Ref: 682 Objective: 22-17

20 Copyright © 2018 Pearson Education, Inc.

71) You have been called to an alcoholism rehabilitation center. On scene, one of the counselors tells you that the patient is a 44-year-old woman who just came to the center this morning. She has not had any alcohol for 24 hours and is now sweating profusely and has an elevated heart rate and blood pressure. Based on this information, the EMT would recognize that the patient is: A) Physically dependent on alcohol B) Experiencing delirium tremens C) Psychologically dependent on alcohol D) Experiencing alcohol tolerance Answer: A Diff: 3 Page Ref: 684 Objective: 22-16 72) The EMT accurately understands alcoholism when he states: A) "The diagnosis of alcoholism is applied to a person who drinks hard liquor, not beer or wine." B) "Surprisingly, in the late stages of their condition, persons with alcoholism are very functional and able to hold down a job and maintain good family relations." C) "Alcohol is classified as a drug on which the addict becomes psychologically dependent, not physically dependent." D) "Aside from the excessive amounts of alcohol, much of the health deterioration that accompanies alcoholism comes from malnutrition." Answer: D Diff: 2 Page Ref: 683 Objective: 22-16 73) You have been called for a 41-year-old female patient who is confused. According to her husband, she is an alcoholic who has been in and out of rehabilitation programs for the past five years. Assessment reveals her airway to be open and her breathing adequate. Her pulse is regular and strong. Which finding would cause the EMT to classify and treat this patient as a high priority? A) Blood pressure of 108/72 mmHg and complaint of nausea B) Smell of alcohol on the breath and complaint of headache C) Complaint of nausea and a temperature of 103.6°F D) Irritable and uncooperative with your effort to treat her Answer: C Diff: 3 Page Ref: 684 Objective: 22-16 74) For which of these patients is a "talk down" technique indicated? A) A 26-year-old who took LSD and is panicked because spiders are after him B) An 18-year-old who is having a "bad trip" after taking his parent's antihypertensive medication to get "high" C) A 59-year-old whose heart is racing after trying cocaine D) A 47-year-old who has chronic alcoholism and is demanding help Answer: A Diff: 2 Page Ref: 678 Objective: 22-15 21 Copyright © 2018 Pearson Education, Inc.

75) When performing a "talk down," which of these statements would appropriately be included? A) "Can you tell me how many rats there are and what their colors are?" B) "Do you see what happens when you take these drugs?" C) "Please ignore the rats and tell me exactly what you took." D) "You are seeing rats because of the drug; this will not last forever." Answer: D Diff: 3 Page Ref: 680 Objective: 22-19 76) You have been called to a party by friends of a 14-year-old female patient with a decreased level of consciousness. On scene, the partygoers state that they were "huffing" when their friend suddenly complained of a headache and then passed out. In reference to the term "huffing," you would recognize this behavior as: A) Injecting drugs intravenously B) Inhaling fumes or vapors C) Smoking methamphetamine D) Sampling multiple prescription drugs Answer: B Diff: 1 Page Ref: 691 Objective: 22-22 77) A 16-year-old male patient has been inhaling paint fumes to "feel good." His grandmother called 911 when she found him doing this in the basement and he said he was having a hard time breathing. The patient is alert and oriented, with a patent airway and adequate breathing. His radial pulse is strong and his skin warm and dry. Your partner reports mild wheezing in both lungs. Vital signs are pulse, 92; respirations, 20 breaths/min; blood pressure, 148/62 mmHg; and SpO2, 96% on room air. The patient has no medical history. Appropriate initial care would include: A) Positive pressure ventilation with supplemental oxygen B) Oxygen by nasal cannula at 8 liters per minute C) Breathing treatment with a metered-dose inhaler prescribed to the grandmother D) High-concentration oxygen through a nonrebreather mask Answer: D Diff: 2 Page Ref: 659 Objective: 22-22

22 Copyright © 2018 Pearson Education, Inc.

78) You suspect food poisoning in a 51-year-old male patient with nausea, vomiting, diarrhea, and fever. He is alert and oriented with no life threats to his airway, breathing, or circulation. His vital signs are pulse, 124; respirations, 20 breaths/min; blood pressure, 198/98 mmHg; and SpO2, 93% on room air. The patient has a history of COPD and hypertension, for which he did not take his medications today. In this situation, what would be the appropriate initial care? A) Oxygen at 2 lpm through a nasal cannula B) Encourage water by mouth for rehydration C) Take the blood pressure pill that the patient missed earlier D) Follow up with his family doctor Answer: A Diff: 2 Page Ref: 656 Objective: 22-5 79) Which of these descriptions is most characteristic of carbon monoxide? A) It generally takes several days for a toxic level of carbon monoxide to accumulate in the body B) Carbon monoxide poisoning symptoms mimic the flu, but it tends to cause a higher fever than the flu C) Carbon monoxide is an odorless gas that can kill a person by causing severe hypoxia D) Carbon monoxide is a tasteless gas that is produced by household appliances such as electric furnaces Answer: C Diff: 2 Page Ref: 666 Objective: 22-11 80) For which of these patients would you be most concerned about accidental exposure to cyanide? A) An auto mechanic working near exhaust fumes B) A coal miner working long hours in a mine C) A worker who paints cars in an auto body shop D) A firefighter battling a smoky structure fire Answer: D Diff: 2 Page Ref: 667 Objective: 22-11 81) A 56-year-old male patient has been exposed to fumes containing cyanide. Why should the EMT call for advanced life support assistance? A) Paramedics may be able to administer a cyanide antidote B) IV fluids are needed to flush the cyanide from the patient's body C) Oxygen will make this patient worse if given by the EMT D) Paramedics can induce vomiting, so as to rid the body of the cyanide Answer: A Diff: 1 Page Ref: 668 Objective: 22-11

23 Copyright © 2018 Pearson Education, Inc.

82) What is the greatest risk related to the ingestion of cyanide? A) Gastrointestinal damage B) High fever C) Widespread hypoxia D) Lung damage Answer: C Diff: 2 Page Ref: 667 Objective: 22-11 83) You have been called to a residence for a cognitively challenged 32-year-old female patient who drank from a bottle of floor cleaner. She is unconscious, exhibits snoring respirations, and is having a difficult time breathing due to swelling from burns in her mouth. Which of these steps should be included in the initial care for this patient? A) Contact poison control B) Insert a nasal airway C) Suction the airway D) Apply high-flow oxygen via nonrebreather Answer: B Diff: 2 Page Ref: 655 Objective: 22-5 84) A 44-year-old male patient who works at a petroleum-refining plant inhaled a significant amount of hydrocarbon fumes and now complains of a headache with associated dizziness and nausea, as well as mild shortness of breath. He is alert and oriented, and there are no life threats to his airway, breathing, or circulation. Which of these steps should be included in his assessment and care? A) Aspirin for the headache B) Intranasal naloxone C) Continuous high-concentration oxygen D) Head-to-toe decontamination Answer: C Diff: 2 Page Ref: 669 Objective: 22-8 85) You are caring for a patient who intentionally drank a compound containing a high percentage of methanol. He is confused and lethargic, but has no obvious deficits to his airway, breathing, or circulation. En route to the hospital, the patient states that he is very nauseated and thinks that he is going to vomit. Your immediate action would be to: A) Check the patient's breath sounds B) Turn the patient on his side C) Assess the patient's abdomen D) Prepare the suction Answer: B Diff: 2 Page Ref: 669-670 Objective: 22-11

24 Copyright © 2018 Pearson Education, Inc.

86) You have been called for a 28-year-old male patient with a history of behavioral problems. He responds to painful stimuli with garbled speech. His airway is patent, but breathing is inadequate. His radial pulse is rapid and his skin cool and moist. He has crackles at the bases of both lungs and the following vital signs: pulse, 108; respirations, 6 breaths/min and shallow; blood pressure, 98/64 mmHg; and SpO2, 92%. A friend informs you that the patient may have drunk a considerable amount of antifreeze, possibly up to 12 hours ago, in a suicide attempt. The first thing you should do is: A) Start positive pressure ventilation B) Call poison control C) Look for confirmation that the antifreeze was ingested D) Provide low-flow oxygen to increase the pulse oximeter reading to 94% Answer: A Diff: 2 Page Ref: 671 Objective: 22-11 87) You have been called to a dilapidated residence for a 26-year-old female patient complaining of generalized weakness. After applying gloves, you enter the home and find the patient on a couch in minimal distress. Your assessment reveals no life threats, but you do note track marks to both hands and arms. When asked about the marks, she states that she is an IV drug user and has hepatitis C. Given this information, you would: A) Apply goggles B) Apply goggles and a gown C) Place a HEPA mask on yourself D) Continue care with gloves Answer: D Diff: 2 Page Ref: 674 Objective: 22-4 88) Family members have called EMS after finding their brother in a highly agitated state. When assessing this patient, which of these findings would raise your suspicion that the patient took a stimulant? A) Pinpoint pupils B) Decreased body temperature C) Blood pressure of 196/104 mmHg D) Slurred speech Answer: C Diff: 2 Page Ref: 677 Objective: 22-19

25 Copyright © 2018 Pearson Education, Inc.

89) A patient has been found unresponsive on her bedroom floor. When assessing her, the EMT would have the highest index of suspicion for a narcotic overdose when he notes: A) Dilated pupils B) Bottle of Valium next to her C) A glass pipe with a sticky residue in it D) Shallow and slow respirations Answer: D Diff: 2 Page Ref: 677 Objective: 22-17 90) A 17-year-old female patient was at a party where she tried a Percocet pill (narcotic pain reliever). After taking the pill, she became scared that she was going to die and is now crying. At the present time, she feels slightly nauseated, but has no other complaints. There are no life threats and her vital signs are pulse, 88; respirations, 14 breaths/min; blood pressure, 108/68 mmHg; and SpO2, 98% on room air. The patient has no medical history but is allergic to peanuts, for which she has a prescribed epinephrine auto-injector. Appropriate care for this patient would include: A) Nonemergent transport B) Talk-down technique C) Asking if she has an epinephrine auto-injector with her D) Supplemental oxygen via a nasal cannula at 2 lpm Answer: A Diff: 2 Page Ref: 688 Objective: 22-17 91) A 19-year-old female patient with diabetes is unresponsive after drinking a large amount of alcohol and trying some "pills" that her friend provided. She has snoring respirations and an intact radial pulse. Emergency Medical Responders are maintaining her airway with a head-tilt, chin-lift maneuver, which has eliminated the snoring respirations. Vital signs are pulse, 116; respirations, 14 breaths/min; minimal chest wall motion with absent breath sounds in the bases; and SpO2, 92%. Advanced life support has been contacted for assistance. Your next action would be to: A) Administer oxygen through positive pressure ventilation B) Identify the pills that the patient took C) Attempt to contact the parents for permission to treat D) Determine the patient's blood glucose level Answer: A Diff: 2 Page Ref: 678 Objective: 22-2

26 Copyright © 2018 Pearson Education, Inc.

92) A female patient in her forties has overdosed and is lying at the bottom of some stairs. Her husband reluctantly informs you that she is addicted to pain medications and was chewing a fentanyl (narcotic pain reliever) patch this evening. She was having difficulty standing up and fell down the stairs. The husband is so frustrated with her that he immediately leaves the scene. The patient has snoring respiration and is incontinent of urine. You first step in caring for this patient would be to: A) Check her pupils for signs of a narcotic overdose B) Open the airway with a head-tilt, chin-lift maneuver C) Apply supplemental oxygen D) Take manual spine motion restriction precautions Answer: D Diff: 2 Page Ref: 675 Objective: 22-2 93) Which of these guidelines would the EMT follow when talking down a patient with a drugrelated emergency? A) Calmly explain to the patient that this is a consequence of abusing drugs B) Be authoritative and tell the patient to follow your directions C) Inform the patient that restraints will be used if he does not listen to you D) Make frequent physical contact with the patient to establish trust Answer: A Diff: 2 Page Ref: 680 Objective: 22-14 94) You are transporting a patient who wants help to rid herself of an addiction to pain medications. She says she has not taken any pain medications since yesterday, and needs help. Which of these findings would you recognize as a sign of withdrawal? A) Her mental status decreases B) Her body temperature falls C) She starts to become agitated D) She begins to cry uncontrollably Answer: C Diff: 2 Page Ref: 684 Objective: 22-16 95) When treating a patient for an overdose, the highest priority of care is placed on: A) Managing the airway and breathing B) Determining what and how much drug was taken C) Establishing the patient's risk for suicide D) Contacting law enforcement Answer: A Diff: 1 Page Ref: 678 Objective: 22-4

27 Copyright © 2018 Pearson Education, Inc.

96) The Poison Control Center has advised you to administer milk to a patient who has overdosed on a medication. Given this information, you would: A) Administer the milk as ordered B) Contact medical command to confirm this treatment C) Transport the patient immediately instead and arrange for ALS intercept D) Contact ALS backup and wait on scene until they arrive Answer: B Diff: 2 Page Ref: 694 Objective: 22-13 97) You have been summoned to an apartment complex for an unknown medical call. At the scene, the police meet you and inform you that the 24-year-old male patient smoked some "bath salts" and is acting erratically. As an EMT, you recognize that bath salts are: A) Drugs that depress the central nervous system B) Drugs that increase circulating norepinephrine C) Drugs made synthetically from bath and shower cleaners D) Drugs that sedate and slow the respiratory drive when taken in excess Answer: B Diff: 2 Page Ref: 688 Objective: 22-19 98) You believe that a male patient has ingested bath salts. Given the nature of this drug, which sign or symptom would help confirm this suspicion? A) Tachycardia B) Hypotension C) Hyperglycemia D) Hypothermia Answer: A Diff: 2 Page Ref: 688 Objective: 22-19 99) A 42-year-old female patient is lying in bed with anxiety after smoking some bath salts 30 minutes earlier. As you enter the bedroom, which finding poses the greatest danger to rescuers, given the nature of this medication? A) Respiratory burns from vapors B) Residual fumes from the drugs C) Patient aggression or violence D) Potential explosion from materials Answer: C Diff: 3 Page Ref: 688 Objective: 22-19

28 Copyright © 2018 Pearson Education, Inc.

100) An alert and oriented 36-year-old male patient is panicked and complaining of chest pain and shortness of breath after snorting some bath salts. He has no past medical history and takes no prescribed medications. The primary assessment is unremarkable with the exception of some expiratory wheezing heard upon auscultation. The vital signs are pulse, 124; respirations, 22 breaths/min; blood pressure, 144/66 mmHg; and SpO2, 92%. To best treat this patient, you would initially: A) Attach the AED B) Administer an albuterol MDI C) Give activated charcoal D) Provide supplemental oxygen Answer: D Diff: 2 Page Ref: 678 Objective: 22-19 101) After a 911 call hang-up, you are dispatched to the caller's residence. When you enter the room with a police escort, you find an unresponsive female patient with a bag over her head. A person in the neighboring apartment tells you that the patient has depression and warned that "there is going to be a death today." Your immediate action would be to: A) Remove the bag and assess for airway occlusion B) Leave the room until it is cleared by the fire department C) Move the patient out to the ambulance before initiating care D) Provide manual spine motion restriction precautions Answer: B Diff: 2 Page Ref: 672 Objective: 22-12 102) Emergency Medical Responders have removed a female patient from her bedroom, where she was found lying on her bed with a suicide bag over her head. The bag is removed and you note the woman to be unresponsive, pulseless, and apneic. Given the use of a suicide bag and the presentation of the patient, which of these statements indicates proper initial care? A) "Do not apply the AED since the arrest was not caused by a cardiac event." B) "I need someone to assess the patient's blood glucose level." C) "Let's open the airway manually first." D) "I need someone to get a pulse oximetry reading immediately!" Answer: C Diff: 2 Page Ref: 672 Objective: 22-12 103) What is the typical dosage for intranasal administration of naloxone? A) 2 mg repeated every 5 minutes only if the patient responds to the initial dose B) 1 mg every 10 minutes C) 1 mg in each nostril D) 2 mg every 5 minutes until the patient regains consciousness Answer: C Diff: 2 Page Ref: 687, Figure 22-21 Objective: 22-18 29 Copyright © 2018 Pearson Education, Inc.

104) Emergency Medical Responders (EMRs) respond to a female patient who was found by her husband in their second-floor bedroom. When the responders arrived, she was lying on her bed, with a tourniquet on her arm. They also found a syringe, with unknown contents, lying on the floor next to the bed. Her husband denies that she abuses any drugs. The EMRs have removed the tourniquet and inserted a nasopharyngeal airway, and they are providing positive pressure ventilations at the rate of 10 breaths/min with a bag-valve-mask device and supplemental oxygen. You note the woman to be unresponsive, with a pulse rate of 52, and apneic. What would your initial proper care include? A) Seek an order to administer naloxone B) Apply the AED C) Extricate the patient from the bedroom D) Immediately move the patient to a firm surface Answer: A Diff: 2 Page Ref: 687, Figure 22-21 Objective: 22-18 105) You are dispatched on a call to a "rave" for a possible ecstasy overdose. Law enforcement is already at the scene and has established scene safety so you can enter. Which kind of signs and symptoms would you expect to find with this type of overdose? A) Signs and symptoms related to CNS depression B) Signs and symptoms related to brain injury or damage C) Signs and symptoms related to hypothermia D) Signs and symptoms related to digestive disorders Answer: B Diff: 2 Page Ref: 689 Objective: 22-20 106) You are a new EMT in a very busy urban EMS system. Before joining this agency, you worked as a volunteer in a rural EMS system, where you did not experience many EMS calls. You are working with a seasoned EMT when you are requested to respond to a residential home for an unresponsive 13-year-old male patient. Upon your arrival, the family escorts you into the garage, where the boy is being cradled by his crying mother. The boy is unresponsive. As you draw nearer to the boy, your partner says to you, "Hey, you smell that moth ball smell? I know what might be going on." Given your partner's statement, how might the odor be related to the patient's condition? A) Chloral hydrate overdose B) Camphor overdose C) Organophosphate overdose D) Acetone overdose Answer: B Diff: 2 Page Ref: 655, Table 22-1 Objective: 22-6

30 Copyright © 2018 Pearson Education, Inc.

107) You are on scene in a remote part of a local park, where an unresponsive female patient was found by a passing security guard. The female patient, who looks to be in her 20s, has needle tracks on her arms; an empty syringe was found nearby. You have initiated spine motion restriction precautions, inserted a nasal airway, and alerted dispatch for ALS backup. The patient is currently breathing adequately and has a blood pressure of 102/78 mmHg, a heart rate of 68 beats/min, and respirations of 12 breaths/min. While awaiting arrival of the ALS unit, which of these steps is most appropriate? A) Administer 1 mg of Narcan either intramuscularly or subcutaneously B) Lay the patient on her side and administer one tube of oral glucose C) Repeat the reassessment and vital signs every 15 minutes D) Administer oxygen so as to maintain a pulse oximeter reading of 94% or greater Answer: D Diff: 2 Page Ref: 661 Objective: 22-9 108) Which of these actions should you and your partner take when you arrive on the scene at a large factory where an unresponsive patient was subjected to a powdered chemical that contacted large portions of his unprotected skin? A) Do not enter the scene until all buildings that make up the factory complex have been evacuated B) Ensure that you are properly protected with gloves and protective gear to prevent the powder from contacting your skin C) Wrap the patient up in a large sheet of plastic to minimize the risk of the powder contaminating other surfaces D) Allow your partner to enter the scene while you remain behind so as to offer "buddy-rescue" should he become overwhelmed by the toxic powder Answer: B Diff: 2 Page Ref: 662 Objective: 22-9 109) When the EMT contacts the Poison Control Center for a female patient who accidentally overdosed on her blood pressure medications when she misread the label, which additional information would the center personnel most likely need to determine care options for the patient? A) The patient's weight B) The patient's birthday C) The most recent bowel/bladder characteristics D) The type of container in which the medication was found Answer: A Diff: 2 Page Ref: 672 Objective: 22-13

31 Copyright © 2018 Pearson Education, Inc.

110) Generally speaking, which technique should the EMT use first when confronted with a patient in a drug or alcohol abuse situation who is experiencing a "bad trip" and expressing some potentially violent tendencies? A) The "take down" technique B) The "assessment down" technique C) The "patient restraint" technique D) The "talk down" technique Answer: D Diff: 2 Page Ref: 678 Objective: 22-15 111) Your EMS unit has arrived on the scene of a male patient who is experiencing auditory and visual illusions after smoking "medical grade" marijuana. While on scene with the conscious patient, your partner leans over to you and whispers, "This is just another pothead trying to get us to think he needs this stuff." Despite your partner's impression, which of these pieces of information would be an indication to you that the patient is actually using the marijuana for prescribed medicinal purposes? A) The patient is currently receiving chemotherapy for cancer B) The patient has a history of high blood pressure C) The patient states he has type 2 diabetes D) The patient lives in a state where marijuana is legal to use Answer: A Diff: 2 Page Ref: 690 Objective: 22-20 112) You are on the scene with a geriatric patient who is semi-responsive and lying in bed. EMS was alerted when a family member went to check on the patient. The patient appears confused when questioned, the skin is cool and dry, the heart rate is 42 beats/min, blood pressure is 96/68 mmHg, and respirations are 12 breaths/min. The family member states the patient just started a new medication yesterday–perhaps that is causing these findings. Which of these medications, if recently prescribed to the patient, could be the cause of the patient's symptoms? A) Calcium channel blocker B) Inhaled bronchodilator C) Oral steroid D) Prescribed dose of acetaminophen Answer: A Diff: 2 Page Ref: 691 Objective: 22-21

32 Copyright © 2018 Pearson Education, Inc.

Prehospital Emergency Care, 11e (Mistovich et al.) Chapter 23 Abdominal, Hematologic, Gynecologic, Genitourinary, and Renal Emergencies 1) When palpating the anterior portion of a patient's abdomen, you note tenderness to the left upper quadrant. As a knowledgeable EMT, you would recognize that which organ is most likely to be involved? A) Kidney B) Gallbladder C) Spleen D) Liver Answer: C Diff: 1 Page Ref: 698 Objective: 23-2 2) If a patient has a retroperitoneal infection, which organ is most likely involved? A) Bladder B) Kidney C) Gallbladder D) Small intestine Answer: B Diff: 2 Page Ref: 720 Objective: 23-2 3) You have been called to transfer a male patient from an outpatient clinic to the hospital. The patient has been diagnosed with peritonitis and requires transport to the hospital for definitive care. As a knowledgeable EMT, you would recognize which of these statements as true? A) The lining of the patient's abdominal cavity is inflamed B) The patient's kidneys and pancreas are infected C) The patient is actively hemorrhaging and requires blood D) The abdominal aorta is inflamed and in danger of rupture Answer: A Diff: 2 Page Ref: 701 Objective: 23-2 4) A patient complains of severe and sharp pain to the right lower abdominal quadrant. Based on location, which organ or structure is most likely involved? A) Liver B) Kidney C) Spleen D) Appendix Answer: D Diff: 1 Page Ref: 702 Objective: 23-4

1 Copyright © 2018 Pearson Education, Inc.

5) A patient informs you that he previously had a section of his small intestine removed due to cancer. You recognize that with the removal of small intestine, the patient is less able to: A) Digest food B) Absorb nutrients C) Secrete insulin D) Break down fats Answer: B Diff: 2 Page Ref: 700, Table 23-2 Objective: 23-2 6) You have been called for a patient with abdominal pain. The patient states that he recently had his gallbladder removed. As an EMT, you realize that: A) His ability to digest fats properly is impaired B) He cannot filter blood normally through the liver C) His ability to absorb nutrients from the intestines is decreased D) He has a more difficult time urinating due to ureter stenosis Answer: A Diff: 2 Page Ref: 700, Table 23-2 Objective: 23-4 7) Which of these definitions best describes the structure and function of the stomach? A) A vascular abdominal organ that filters nutrients from the blood B) A hollow organ that allows the absorption of nutrients into the bloodstream C) A solid organ that regulates the balance of water in the body D) A hollow organ that secretes digestive enzymes and churns food Answer: D Diff: 1 Page Ref: 700, Table 23-2 Objective: 23-2 8) The spleen of a patient has ruptured following a traumatic injury. Because it is a solid organ, the EMT recognizes that the primary threat to life is: A) Blood loss B) Pain C) Infection D) Inflammation Answer: A Diff: 1 Page Ref: 698 Objective: 23-2

2 Copyright © 2018 Pearson Education, Inc.

9) A female patient has developed bacterial peritonitis after sustaining blunt trauma to the abdomen. Damage to which organ was most likely responsible for this condition? A) Spleen B) Small intestine C) Liver D) Ovary Answer: B Diff: 2 Page Ref: 698 Objective: 23-4 10) Which of these conditions would the EMT recognize as an acute abdominal emergency? A) Inflammation of the lower portion of the right lung B) Swelling of the brain and spinal cord C) Infection of the lining surrounding the heart D) Rupture of the bladder Answer: D Diff: 1 Page Ref: 698 Objective: 23-2 11) The EMT knows that the abdomen is located: A) Between the diaphragm and the top of the pelvis B) From the mid-chest to the umbilicus C) Between the nipple line and the coccyx of the spine D) From the bottom of the rib cage to the pelvis Answer: A Diff: 1 Page Ref: 697 Objective: 23-2 12) The pancreas would best be described as which type of organ? A) Vascular B) Hollow C) Solid D) Perforated Answer: C Diff: 1 Page Ref: 699, Figure 23-1 Objective: 23-2 13) The EMT understands the major function of most organs in the abdomen when he states: A) "The major function of the organs in the abdominal cavity is digestion." B) "The basic function of the abdominal organs is the storage of fat for reserve energy." C) "The primary role of the organs in the abdominal cavity is the utilization of oxygen." D) "The fundamental role of the organs in the abdomen is the cleansing and filtering of blood." Answer: A Diff: 2 Page Ref: 698 Objective: 23-2

3 Copyright © 2018 Pearson Education, Inc.

14) The EMT has an accurate understanding of the solid organs contained within the abdomen when she states: A) "Solid organs contain less blood than hollow organs, but will still bleed if injured." B) "Solid organs like the gallbladder are covered by a thick fibrous capsule for protection." C) "Solid organs are structures in the abdomen that break down and absorb nutrients into the blood." D) "Solid organs may receive a lot of blood flow and may bleed excessively if injured." Answer: D Diff: 2 Page Ref: 698 Objective: 23-2 15) The EMT would identify which of these patients as having an acute abdomen? A) A 71-year-old female in respiratory distress and complaining of nausea B) A 37-year-old male with history of drug abuse and liver disease C) A 62-year-old male with no medical history with sudden pain to the lower abdomen D) A 17-year-old female who is pregnant with twins and whose delivery date is next month Answer: C Diff: 2 Page Ref: 697 Objective: 23-3 16) Of these statements, which best demonstrates that the EMT has an accurate understanding of acute abdominal pain? A) "Abdominal pain is more of an emergency when the patient has a medical history related to the abdominal organs." B) "If the abdominal pain is not sharp in nature, it is most likely not a life-threatening condition." C) "Sometimes the cause of abdominal pain will be obvious, but often the cause will not be that apparent." D) "If the patient complains of 'crampy' or colicky pain, the EMT should suspect that a solid organ has ruptured and is bleeding." Answer: C Diff: 2 Page Ref: 697 Objective: 23-3 17) Which of these statements made by the patient would most likely be associated with parietal or somatic pain? A) "I feel as though I have to throw up but can't." B) "It seems to hurt all over my belly and back." C) "It really hurts right here, beside my belly-button." D) "The pain seems to be dull and achy." Answer: C Diff: 2 Page Ref: 700 Objective: 23-3

4 Copyright © 2018 Pearson Education, Inc.

18) A middle-aged female patient has dialed 911 for an abdominal emergency. On scene, she informs you she has a "bad gallbladder" and it has been increasingly bothering her, so she needs transport to the hospital for further care. Your assessment reveals tenderness and mild discomfort to the right upper quadrant of the abdomen. The patient also states that her right shoulder is aching. Given the patient's complaint, the EMT would recognize the shoulder pain as: A) Referred pain B) Shadow pain C) Vascular pain D) Muscular pain Answer: A Diff: 2 Page Ref: 701 Objective: 23-3 19) Your partner is performing the Markle test on a patient who complains of abdominal pain. Which instructions should be provided to the patient to ensure that she performs the test correctly? A) "Please raise each leg one at a time. I will need to know if you feel any pain, especially in your heels when you put pressure back on them." B) "I am going to gently press inward on your belly and then quickly release the pressure." C) "I am going to press on your belly with my hands. Please tell me if you suddenly feel pain in your ankles." D) "I need you to stand on the tips of your toes, then on my command drop onto your heels." Answer: D Diff: 2 Page Ref: 702 Objective: 23-1 20) After performing a Markle test on a 31-year-old female patient with abdominal pain, she locates pain to her left lower quadrant. How would the EMT interpret this result? A) A positive result indicating appendicitis as the cause of the patient's pain B) Additional evidence that the patient has peritonitis C) A negative result indicating that the patient's condition is not life-threatening D) Realization that the gallbladder is the cause of the patient's discomfort Answer: B Diff: 3 Page Ref: 702 Objective: 23-6 21) When EMS enters the room of a 62-year-old female patient with an unknown medical complaint, which of these clues observed during the scene size-up would increase the EMT's suspicion that the patient has an acute abdominal condition? A) The patient is in her nightgown and the air conditioning is turned on "high" B) A portable commode is next to the bed, with a large amount of urine inside C) The patient is lying on her side, with her knees drawn to her chest D) The patient is wearing a nasal cannula attached to oxygen at 2 lpm Answer: C Diff: 2 Page Ref: 708 Objective: 23-4 5 Copyright © 2018 Pearson Education, Inc.

22) Which of these statements indicates that the EMT correctly understands the goal of assessing a geriatric patient with abdominal pain? A) "It is important to determine whether the liver is the cause of pain, since that is the most lifethreatening condition." B) "If the patient has tenderness on palpation of the abdomen, the EMT should assume bleeding is the cause." C) "It is more important to recognize a possible abdominal emergency, as opposed to determining the exact cause, because pain perception may be distorted." D) "To appropriately treat the patient, the EMT must identify the exact cause of the patient's pain." Answer: C Diff: 3 Page Ref: 707 Objective: 23-5 23) You have been dispatched for a patient with abdominal pain. On scene, you find the patient lying supine and complaining of severe pain to his lower abdomen. He is pale and has a look of distress on his face as he answers your questions. Which action should the EMT take after assuring the airway and breathing are adequate? A) Inspect the patient's abdomen B) Obtain the patient's blood pressure C) Palpate the patient's abdomen D) Check the patient's pulse Answer: D Diff: 2 Page Ref: 708 Objective: 23-6 24) Which of these assessment findings is of most concern with a 46-year-old male patient who complains of severe abdominal pain? A) Blood pressure of 90/64 mmHg B) Respiratory rate of 22 breaths/min C) Heart rate of 96 beats/min D) Increased tenderness to the right upper quadrant with palpation Answer: A Diff: 2 Page Ref: 708 Objective: 23-6 25) A 42-year-old female patient is crying and states that she has sharp abdominal pain that she localizes to her right lower quadrant. When assessing this patient's abdomen, the EMT would: A) Start palpating in the left upper quadrant B) Avoid palpating the entire abdomen C) Start palpating in the right lower quadrant D) Avoid palpating the right lower quadrant Answer: A Diff: 2 Page Ref: 709 Objective: 23-6

6 Copyright © 2018 Pearson Education, Inc.

26) A patient with a history of upper gastrointestinal bleeding is complaining of weakness and abdominal pain. He informs you that he vomited 5 minutes before you arrived. Which of these questions is most important that the EMT ask first regarding the vomit? A) "Did you eat anything that made you vomit?" B) "Are you allergic to any foods or medications?" C) "Did you take an antacid over the past 24 hours?" D) "Was the vomit red or did it look like coffee grounds?" Answer: D Diff: 2 Page Ref: 708 Objective: 23-6 27) When assessing a patient with abdominal pain, you note that his abdomen feels rigid. What should you do next as part of the abdominal assessment? A) Ask the patient to take a deep breath and then relax, while you repalpate the rigid area B) Note that the patient has guarding and obtain a blood pressure C) Gently massage the abdominal muscles until they relax D) Increase the pressure of palpation until the rigid muscles relax Answer: A Diff: 2 Page Ref: 709 Objective: 23-6 28) Upon your arrival at the scene of a patient with severe abdominal pain, an Emergency Medical Responder tells you that he is concerned because the patient's blood pressure is 146/88 mmHg and the patient has no history of high blood pressure. He states that it may be a good idea to treat the patient for hypertension as well as the abdominal pain. Your best response would be: A) "Thank you, you are right. We will call for ALS assistance right away." B) "That is concerning. We will have to see if anyone has blood pressure pills the patient can take." C) "Sometimes pain can naturally increase the blood pressure. I will take another reading so we can compare it with the baseline reading." D) "If the patient does not have a history of hypertension, then we will have to transport using lights and sirens." Answer: C Diff: 3 Page Ref: 710 Objective: 23-6 29) When an EMT inspects the abdomen of a patient with pain to his right upper quadrant, it appears large and possibly distended. To determine if the abdomen is indeed distended, the EMT would ask: A) "Do you feel sick to your stomach at this time, as if you are going to vomit?" B) "Does your belly normally seem to be this size?" C) "Have you been doing any abdominal exercises or heavy lifting recently?" D) "Did you eat spicy food last night, or food containing a lot of sodium?" Answer: B Diff: 2 Page Ref: 710 Objective: 23-6 7 Copyright © 2018 Pearson Education, Inc.

30) Which of these assessment findings for a patient complaining of abdominal and back pain would raise the most concern in the EMT? A) Surgical scar in the right upper quadrant B) Pulsating mass above the umbilicus C) History of gallbladder disease D) Increased bowel sounds associated with diarrhea Answer: B Diff: 2 Page Ref: 706 Objective: 23-7 31) Your medical director is providing an in-service program on abdominal pain. He notes that there are several different categories of pain and asks if anyone can describe visceral pain. Which response made by the EMT would be correct? A) "Visceral pain is typically described as 'sharp' and occurs when a hollow organ has become infected." B) "The exact site of visceral pain is easily located by the patient and is typically associated with nausea and vomiting." C) "Visceral pain occurs when the peritoneum lining the abdomen becomes infected and causes a rapid increase in body temperature." D) "A patient with visceral pain typically describes the pain as 'aching' and has a hard time identifying its exact location." Answer: D Diff: 3 Page Ref: 700 Objective: 23-3 32) Which of these assessment findings would increase the EMT's suspicion of a gastrointestinal bleed that is likely occurring slowly? A) Sudden onset of constipation B) Watery and foul-smelling diarrhea C) Black semi-formed stool D) Sharp left lower quadrant pain Answer: C Diff: 2 Page Ref: 703 Objective: 23-6 33) While assessing a patient with acute abdominal pain, you note that his abdomen is very rigid. When asked to relax his stomach muscles, he does. The EMT would recognize this patient as demonstrating: A) Voluntary guarding B) Rebound guarding C) False rigidity D) Involuntary rigidity Answer: A Diff: 2 Page Ref: 709 Objective: 23-1

8 Copyright © 2018 Pearson Education, Inc.

34) You are called to a long-term care facility for a patient with abdominal pain per staff. On scene, you find an 81-year-old female patient whom staff said complained of abdominal pain and then began to vomit several times. The patient has dementia and is occasionally confused and sometimes lethargic, which staff says is her normal status. Assessment reveals her to be confused, breathing 22 times a minute, with a pulse rate of 88 beats per minute. Her blood pressure is 118/68 mmHg and she has an SpO2 of 97% on room air. Staff states that her temperature is 100.8°F and she did not get her medication for dementia today. When treating and transporting this patient, what would most likely represent the EMT's greatest concern during transport? A) Risk for vomiting B) Temperature of 100.8°F C) Respiratory rate of 22 breaths/min D) Not getting her antidementia medications Answer: A Diff: 3 Page Ref: 710 Objective: 23-4 35) Assessment of a 21-year-old female patient reveals her to have severe abdominal pain localized to the right upper quadrant. She is alert and oriented, and her vital signs are stable. After you place her supine on the stretcher, the patient quickly assumes a lateral recumbent position with her knees drawn up to her chest. Which response from the EMT would be appropriate? A) "It would be best if you stay on your back. That position will help decrease the pain." B) "You can stay on your side, but I will need you to keep your legs straight. That will help decrease the pain." C) "I need you to stay on your back and with your head up. If you vomit, that will prevent you from aspirating." D) "You seem more comfortable in that position. If you are, you should stay on your side." Answer: D Diff: 2 Page Ref: 710 Objective: 23-6 36) While you are transporting a 44-year-old female patient with abdominal pain in low Fowler's position, she starts to vomit. Your immediate action would be to: A) Examine the vomit for blood B) Palpate the patient's abdomen C) Place the patient in a left lateral recumbent position D) Place some vomit in a specimen bag for the hospital Answer: C Diff: 2 Page Ref: 710 Objective: 23-6

9 Copyright © 2018 Pearson Education, Inc.

37) An alert but confused patient called 911 with the complaint of abdominal pain. Assessment reveals her to be nauseated with pain that she localizes to the right lower quadrant of the abdomen. She states that she was not able to take her blood pressure medication this morning and wants to take it now with a small glass of juice. You would: A) Allow her to take the medication with juice B) Crush the medication and mix it in a small amount of water C) Tell her that she should not take her medication or drink juice D) Provide her some water instead of juice to take with her medications Answer: C Diff: 2 Page Ref: 710 Objective: 23-6 38) You have completed the initial assessment on a 62-year-old male patient who complains of the sudden onset of epigastric pain. His airway is patent, his breathing is fast but still adequate, and his radial pulse is weak and barely palpable with a delayed capillary refill. His pulse is 124 beats/min and his skin is cool and moist to the touch. Your next action would be to: A) Obtain a blood pressure in both arms B) Perform a secondary assessment C) Place the patient on the stretcher and move him to the ambulance D) Apply supplemental oxygen Answer: D Diff: 2 Page Ref: 710 Objective: 23-6 39) During transport, a 27-year-old female drug addict with acute abdominal pain tells you that she is scared and starts crying. What would your best response be? A) "Don't worry, there is probably nothing wrong. My guess is that you probably ate something bad, but the hospital will let you know for sure." B) "I know that you are in pain. We are 5 minutes from the hospital and I have already called the emergency department and let them know we are coming. They will be prepared for us." C) "This probably has something to do with the drugs you are on. It is very important that you quit." D) "There is nothing more I can do since we are not allowed to give medications for pain. Let's just see what happens at the hospital." Answer: B Diff: 3 Page Ref: 710 Objective: 23-6

10 Copyright © 2018 Pearson Education, Inc.

40) You are providing continuing education for a group of EMTs. The topic is acute abdominal pain. Which of these points would you stress when discussing the EMT's treatment of the patient with acute abdominal pain? A) The EMT can provide more appropriate care for the patient if he or she is able to identify the cause of the abdominal pain B) It is okay to spend extended time on scene while working to identify the exact cause of the abdominal pain C) Patients complaining of nausea and vomiting without pain rarely have life-threatening conditions and can go to the hospital by private vehicle D) Definitive care for patients with abdominal pain is hospital evaluation and possible surgical intervention Answer: D Diff: 2 Page Ref: 701 Objective: 23-6 41) You suspect a patient with right lower quadrant pain may have appendicitis. The patient states that he has no health insurance and wants to refuse care and transport. He asks you what can happen if his condition is appendicitis and he does not go to the hospital. What is your best response? A) "Your appendix could rupture, causing a major infection and shock." B) "The inflammation of the appendix will gradually subside, but in the meantime, the pain will be terrible." C) "If you do not get care, blood may clot in the intestine, causing an obstruction." D) "The pain will most likely subside in a few hours. You may want to try an anti-inflammatory like Motrin." Answer: A Diff: 3 Page Ref: 702 Objective: 23-6 42) Which of these statements made by the patient regarding abdominal pain would lead the EMT to suspect appendicitis? A) "The stabbing pain started in the right side of my belly and spread up toward my stomach." B) "I had back pain for about a week, and now the pain is all throughout my belly." C) "The pain started around my belly button and slowly spread down to the right side of my belly." D) "The pain comes and goes and is more noticeable on the left side of my stomach." Answer: C Diff: 3 Page Ref: 702 Objective: 23-3

11 Copyright © 2018 Pearson Education, Inc.

43) You have been dispatched for a patient with severe abdominal pain. On scene, the 57-yearold male patient informs you that he has a past medical history of chronic pancreatitis, which has "flared up again." Which precipitating factor would you identify as the most likely cause of the exacerbation? A) Recent upper respiratory infection B) Consumption of six beers this afternoon at a picnic C) Dehydration after working outside for several hours D) Use of antibiotic medication for a respiratory infection Answer: B Diff: 2 Page Ref: 702 Objective: 23-6 44) When you are obtaining information for the medical history, which of these findings would be significant for a patient with suspected cholecystitis? A) Start of a new medication for high blood pressure B) History of recent unexplained weight loss C) Abstinence from alcohol over the past six months D) Fried chicken and potato salad for dinner Answer: D Diff: 2 Page Ref: 702-703 Objective: 23-6 45) A patient states that about two hours following dinner, she developed right upper quadrant abdominal pain that radiated to her shoulder. As a knowledgeable EMT, you would recognize this characteristic pain pattern as most suggestive of: A) Urinary tract infection B) Gallbladder inflammation C) Intestinal obstruction D) Gastric ulcer Answer: B Diff: 3 Page Ref: 702-703 Objective: 23-6 46) You have been called for a 31-year-old male patient who experienced sudden left lower quadrant pain while lifting and moving a friend's couch to an upstairs bedroom. The EMT would recognize this history as suggestive of which type of abdominal pathology? A) Cholecystitis B) Appendicitis C) Aortic aneurysm D) Hernia Answer: D Diff: 3 Page Ref: 705 Objective: 23-6

12 Copyright © 2018 Pearson Education, Inc.

47) A 56-year-old male patient informs you that he has gastric ulcers. He has had them for several months, but recently the pain has been steadily increasing. If the cause of the abdominal pain is the gastric ulcers, how would you expect the patient to describe the pain? A) Stabbing pain that radiates into both legs B) Sharp pain near the umbilicus that radiates to the right lower quadrant C) Burning pain located to the left upper quadrant D) Achy pain occurring all throughout the abdomen Answer: C Diff: 2 Page Ref: 704 Objective: 23-3 48) You have been dispatched for a male patient who is "sick." While you are en route, Emergency Medical Responders inform you via radio that the patient has esophageal varices, which appear to have ruptured. You would inform your partner to prepare which materials or piece of equipment? A) Automated external defibrillator B) Suction machine C) Trauma dressings D) Long spinal board Answer: B Diff: 2 Page Ref: 703 Objective: 23-6 49) A patient informs you that he was recently diagnosed with an abdominal aortic aneurysm (AAA). The surgeon did not want to operate on the aneurysm immediately because he felt it was stable and not worth the risks that surgery posed. Tonight, the patient states that he suddenly developed abdominal pain and wants to be evaluated in the hospital to see if the AAA has increased in size. Given this history, which of these assessment findings would cause the EMT to suspect that the AAA is the cause of the abdominal pain? A) "Tearing" pain located to the lower back B) Temperature of 101°F C) Pain that subsides when the patient lies still D) Inability to urinate Answer: A Diff: 3 Page Ref: 705-706 Objective: 23-3 50) While performing the secondary assessment on a patient's abdomen, you note a pulsating mass just above the umbilicus. Your next action would be to: A) Increase the pressure of palpation to determine whether there is pain related to deep organs B) Check the blood pressure in both arms C) Determine the exact size of the pulsating mass D) Palpate the remainder of the abdomen gently Answer: D Diff: 2 Page Ref: 705-706 Objective: 23-7 13 Copyright © 2018 Pearson Education, Inc.

51) When you are obtaining a medical history from a female patient with abdominal pain, she tells you that she has endometriosis. As an EMT, you know that this condition describes: A) Infection of the uterine lining B) Bacterial infection of the bladder C) Infection of the ovaries and/or fallopian tubes D) Endometrial tissue located outside of the uterus Answer: D Diff: 1 Page Ref: 716 Objective: 23-1 52) During a normal pregnancy process, within which structure does the sperm and egg typically meet during fertilization? A) Ovary B) Fallopian tube C) Vagina D) Uterus Answer: B Diff: 1 Page Ref: 714 Objective: 23-8 53) A 38-year-old female patient was violently raped at a party where drugs were being abused. The patient is conscious, upset, and actively bleeding from the vaginal canal. As an EMT, your best care would involve: A) Cleansing the vaginal area to prevent infection B) Carefully placing a sterile dressing into the vagina to control the hemorrhage C) Applying a bulky dressing and gentle pressure to the external genitalia D) Avoiding physical contact with the vaginal area so not to destroy evidence Answer: C Diff: 2 Page Ref: 715 Objective: 23-10 54) A 26-year-old female patient complains of abdominal pain. The pain is generalized to her abdomen and worsens when walking. She also tells you that she is having a foul-smelling, yellow-colored vaginal discharge. The primary assessment reveals no threats to the airway, breathing, or circulation. Her vital signs are pulse, 104; respirations, 16 breaths/min; blood pressure, 108/60 mmHg; and oral temperature of 102.1°F. She finished her last menstrual period three days ago. Which condition do you suspect? A) Ovarian cyst B) Pelvic inflammatory disease C) Ectopic pregnancy D) Mittelschmerz Answer: B Diff: 3 Page Ref: 716-717 Objective: 23-9

14 Copyright © 2018 Pearson Education, Inc.

55) You are transporting a visibly upset female patient who was raped after she let a male stranger into her house. Which of these questions would be appropriate for the EMT to ask? A) "Why would you let someone in your house you did not know?" B) "Do you have any history of a sexually transmitted disease?" C) "Have you ever been sexually assaulted or raped before?" D) "Please tell me where you are hurting." Answer: D Diff: 2 Page Ref: 715 Objective: 23-10 56) Which of these statements is true concerning the functions of the vagina? A) The vagina allows for the passage of a baby during delivery and the outlet of urine from the bladder B) The vagina permits the outflow of menstrual blood and is the site of fertilization for the egg and sperm C) The vagina serves as a passage for the fetus during delivery as well as an outlet for blood during menstruation D) The vagina allows for the excretion of urine from the bladder and accommodates the penis during intercourse Answer: C Diff: 1 Page Ref: 714 Objective: 23-8 57) Pelvic inflammatory disease is best described as: A) An infection of the female reproductive structures B) An ovarian cyst that has ruptured C) A particularly painful menstrual period D) Pain experienced as a female ovulates Answer: A Diff: 2 Page Ref: 716-717 Objective: 23-1 58) The primary assessment of a 27-year-old female patient who has been raped reveals no life threats to the airway, breathing, or circulation. The patient complains of pain in the vaginal area, but a focused assessment of her genitalia indicates no remarkable findings. When caring for this patient, which of these statements would be most therapeutic? A) "Please be assured that you are safe from harm here in the ambulance." B) "I know that this is difficult, but you will need to be tested for HIV and other STDs." C) "I need to look one more time at your genital area to see if there is anything looks different." D) "It may set you at ease to hear that in most cases, the attacker is apprehended by authorities." Answer: A Diff: 2 Page Ref: 715 Objective: 23-10

15 Copyright © 2018 Pearson Education, Inc.

59) A patient who has just been raped states that she will not go with you to the hospital until she showers and cleans herself up. As an EMT, what is your best response? A) "You may shower, but you must then go with us to the emergency department." B) "If you shower, you will destroy evidence needed to help identify the rapist." C) "Instead of showering, I will clean the wounds with sterile water on the way to the hospital." D) "Showering will only delay the care that you can get at the hospital, including medications for pain." Answer: B Diff: 2 Page Ref: 715 Objective: 23-10 60) Which of these statements made by a 27-year-old female patient would make the EMT highly suspicious of mittelschmerz? A) "I had some sharp belly pain, but it seems to be going away now." B) "My menstrual flow seems heavier than normal." C) "I finished my period yesterday and now I am nauseated." D) "It seems that I have had a yellow vaginal discharge for two days." Answer: A Diff: 2 Page Ref: 716 Objective: 23-1 61) A 44-year-old female patient states that she has a history of endometritis. The EMT would recognize that this patient has: A) An infection of the endometrial lining B) An infection of the ovaries C) Endometrial tissue that is growing outside of the uterus D) Absence of a menstrual period Answer: A Diff: 2 Page Ref: 716 Objective: 23-1 62) You are treating a woman who has heavy vaginal bleeding. When estimating the blood loss, which information will be most useful to the EMT? A) Length of time the patient has been bleeding B) Total number of sanitary napkins used C) If associated with abdominal pain the bleeding is considered heavy D) Examination of the present color and rate of bleeding Answer: B Diff: 2 Page Ref: 718 Objective: 23-10

16 Copyright © 2018 Pearson Education, Inc.

63) Which of these descriptions best fits a female patient with a problem related to her urinary system? A) Infection of the fallopian tubes B) Swelling of the ureter C) Damage to the vaginal canal D) Cancerous lesion to the ovary Answer: B Diff: 1 Page Ref: 720 Objective: 23-11 64) In the human body, the urethra: A) Prevents urine from flowing back up and into the kidney B) Passes urine from the bladder to the outside of the body C) Stores urine in the body until it can be eliminated D) Refines the urine produced by the kidney Answer: B Diff: 1 Page Ref: 720 Objective: 23-11 65) A patient has a tumor that has compressed and obstructed her left ureter. Which complication may result from this condition? A) The bladder cannot be completely emptied B) The patient is more prone to urinary tract infections C) The kidneys are impaired in their ability to make urine D) The urine will back up into the left kidney Answer: D Diff: 2 Page Ref: 720 Objective: 23-12 66) The most common cause of urinary tract infections is: A) Poor handwashing B) Overuse of antibiotics C) Frequent urination D) Bacterial invasion Answer: D Diff: 1 Page Ref: 720 Objective: 23-12 67) Which of these complaints would raise the EMT's suspicion that a female patient has a urinary tract infection (UTI)? A) Urinating constantly with associated great thirst B) Pain in the back that spreads into the legs C) Burning sensation every time she urinates D) Vaginal discharge that is greenish in color Answer: C Diff: 2 Page Ref: 721 Objective: 23-12 17 Copyright © 2018 Pearson Education, Inc.

68) A patient with a history of frequent urinary tract infections (UTI) complains of lower abdominal pain and feels as though she has a UTI again. Which assessment findings would cause you to question that the problem is a UTI? A) Temperature of 101.3°F B) Vaginal bleeding C) Flank pain D) Cloudy urine Answer: B Diff: 2 Page Ref: 721 Objective: 23-12 69) A 26-year-old female patient has called 911 because of lower abdominal pain. Your findings are consistent with a urinary tract infection. The patient has a history of urinary tract infections as well as asthma. Her vital signs are pulse, 116; respirations, 14 breaths/min and adequate; blood pressure, 108/56 mmHg; SpO2; 99%; and temperature, 101.6°F. What is the priority of care? A) Nonemergent transport B) Aspirin for fever control C) Albuterol via her metered-dose inhaler D) Advanced life support assistance Answer: A Diff: 2 Page Ref: 725 Objective: 23-6 70) The EMT would recognize that the pain associated with a kidney stone is caused by: A) Inability to empty the bladder of urine B) Passage of the stone through the ureter C) Infection of the bladder and ureter D) Formation of the stone in the kidney Answer: B Diff: 3 Page Ref: 721 Objective: 23-9 71) You are by the side of a patient who thinks he has a kidney stone. After assessing the patient, which of these findings would force you to consider an alternative pathology for the patient's pain? A) Blood-tinged urine B) Frequent urge to urinate C) Tearing pain in the abdomen D) Flank pain that radiates into the groin Answer: C Diff: 2 Page Ref: 706 Objective: 23-6

18 Copyright © 2018 Pearson Education, Inc.

72) A patient with a kidney stone would most likely complain of: A) Severe abdominal pain B) Chest discomfort with dyspnea C) Excessive urination D) Nausea and vomiting Answer: A Diff: 2 Page Ref: 721 Objective: 23-12 73) When transporting a patient whom you suspect has a kidney stone to the hospital, in which position should the patient be placed if no contraindications exist? A) Position of comfort B) Trendelenburg position C) Prone position D) Semi-sitting position Answer: A Diff: 2 Page Ref: 725 Objective: 23-6 74) On scene, a 57-year-old female patient has profuse hemorrhage coming from the dialysis graft located on her left arm. To control the bleeding, you would immediately: A) Place a tourniquet over the graft B) Apply direct pressure to the graft C) Place a tourniquet above the graft D) Avoid contact with the graft and transport the patient Answer: B Diff: 2 Page Ref: 722 Objective: 23-13 75) You are dispatched for a patient with an unknown medical problem at a dialysis center. When you arrive, you are directed to a patient who is sitting in a chair, with a dialysis technician holding a dressing on his arm very tight. You can see blood seeping through the dressing and on the hands of the technician. Given this limited information, what is likely the problem? A) The patient fell and has an open fracture to his arm B) The dialysis graft is bleeding C) The patient has a diabetic ulcer that broke open D) The patient was accidentally cut by a scalpel used by the dialysis technician Answer: B Diff: 1 Page Ref: 722-723 Objective: 23-13

19 Copyright © 2018 Pearson Education, Inc.

76) Assessment of a confused and lethargic patient with diabetes and renal failure reveals her to be lying in bed with notable significant edema to the face, torso, and extremities. Her respirations are extremely labored and associated with a "grunting" sound. Her airway is patent, radial pulse weak, and skin cool and diaphoretic with cyanosis noted to the fingers. Oxygen has been applied by Emergency Medical Responders (EMRs). Your next action would be to: A) Obtain vital signs B) Sit the patient upright C) Place a nasal airway D) Check her blood glucose level Answer: B Diff: 3 Page Ref: 723 Objective: 23-13 77) You have been dispatched to a residence for a patient experiencing an exacerbation of chronic pancreatitis. Which chief complaint do you anticipate the patient having? A) Severe pain B) Chronic diarrhea C) Bloody stool D) Altered mental status Answer: A Diff: 2 Page Ref: 702 Objective: 23-6 78) Which of these conditions involves an infection in the retroperitoneal portion of the abdomen? A) Gastroenteritis B) Hepatitis C) Pancreatitis D) Inflamed spleen Answer: C Diff: 2 Page Ref: 697 Objective: 23-4 79) An EMR tells you that a patient has pain just above the umbilicus, but just below the rib cage. You would document this condition as pain in the: A) Right lower quadrant B) Hypogastric area C) Left upper quadrant D) Epigastric area Answer: D Diff: 2 Page Ref: 699, Figure23-2(b) Objective: 23-6

20 Copyright © 2018 Pearson Education, Inc.

80) A patient with abdominal pain shows you black and loosely formed stool in the toilet. You would advise the hospital that the patient has: A) Melenic stool B) Hematemesis. C) Bile-like stool D) Hematochezia Answer: A Diff: 2 Page Ref: 703 Objective: 23-1 81) At the side of a distressed patient with active and severe esophageal varices, you realize that the priority of care will be: A) Stopping the hemorrhage B) Airway management C) Rapid transportation D) Monitoring the blood pressure Answer: B Diff: 3 Page Ref: 703 Objective: 23-6 82) You have been called to a local nursing home for a restless patient with a four-day history of diarrhea. The primary assessment reveals an open airway, rapid breathing, and a weak and rapid radial pulse. Vital signs are pulse, 124; respirations, 24 breaths/min; blood pressure, 100/86 mmHg; and SpO2, 95%. The patient is confused with a history of Alzheimer's dementia. For this patient, it is most important to evaluate for: A) Intestinal obstruction B) Cause of confusion C) Dehydration D) Blood in the diarrhea Answer: C Diff: 3 Page Ref: 706 Objective: 23-6 83) Which sign or symptom would reinforce the EMT's suspicion that a patient with a history of pancreatitis is having an exacerbation of the disease? A) Upper abdominal pain that radiates to the back B) Nausea, vomiting, and diarrhea C) Right upper quadrant pain with referral to the right shoulder D) Decreased heart rate and elevated blood pressure Answer: A Diff: 3 Page Ref: 702 Objective: 23-4

21 Copyright © 2018 Pearson Education, Inc.

84) You are by the side of an 80-year-old male patient whose wife called EMS because he had tearing-type abdominal pain. The patient states that he was just discharged from the hospital this morning after being treated for gastroenteritis. When questioned further, he states that he remains nauseated, but the medications for the vomiting and diarrhea seem to be working, as he is no longer experiencing either of those symptoms. Since he was just in the hospital, the patient states that he would like to refuse transport and continue his recovery at home. What is your best response? A) "I agree that the medications are working, but be safe and call your family doctor." B) "I don't know; your symptoms don't seem to be related to gastroenteritis." C) "I think you are right about getting better; it is up to you if you want to go back." D) "I am not comfortable with this; it sounds as if your gastroenteritis is not any better." Answer: B Diff: 3 Page Ref: 705-706 Objective: 23-7 85) On scene, a 45-year-old female patient tells you that her current abdominal pain felt similar to a previous small bowel obstruction, so she called for EMS much earlier than the first episode. During your assessment, which signs or symptoms would agree with her assessment? A) Nausea and diarrhea for two days B) Upper abdominal pain with deep inspiration C) Decreased heart rate and blood pressure D) Prominent and high-pitched bowl sounds Answer: D Diff: 3 Page Ref: 705 Objective: 23-6 86) A 23-year-old male patient has extreme pain in his lower right groin with a protruding mass noted in the same area. In discussing this patient with your partner, which statement would be most accurate? A) "I am worried about an abdominal aneurysm; we should palpate gently." B) "I don't think that the mass is related to his pain since it is not pulsating." C) "Are you thinking appendicitis since he does have a low-grade fever?" D) "He really needs to be evaluated; he may have a hernia, obstruction, or something worse." Answer: D Diff: 2 Page Ref: 705 Objective: 23-6 87) What is the major threat to life for a patient with an unstable abdominal aortic aneurysm? A) Hemorrhage B) Infection C) Hypoxia D) Organ compression Answer: A Diff: 1 Page Ref: 705-706 Objective: 23-7

22 Copyright © 2018 Pearson Education, Inc.

88) A severely distressed patient has bright red rectal bleeding and in lethargic. His pulse is 136, respirations are 24 breaths/min, blood pressure is 72/58 mmHg, and SpO2 is 95%. His skin is moist, pale, and clammy, and his mental status is deteriorating. At this time, your priority intervention would be to: A) Apply pressure over the rectum with sterile dressings to control bleeding B) Apply supplemental oxygen C) Apply the AED, but do not turn it on yet D) Transfer the patient to the stretcher for immediate transport Answer: B Diff: 2 Page Ref: 710 Objective: 23-6 89) The most common complaint(s) associated with a gynecologic emergency is (are): A) Vaginal discharge B) Upper abdominal pain and vaginal bleeding C) Lower abdominal pain D) Nausea and vaginal discharge Answer: C Diff: 2 Page Ref: 717-718 Objective: 23-9 90) Which of these statement is true regarding a female who has had both ovaries removed? A) Her eggs must now be fertilized in the fallopian tubes B) She cannot produce any eggs for fertilization C) She is at higher risk for a tubal pregnancy D) She will experience hematuria Answer: B Diff: 3 Page Ref: 714 Objective: 23-8 91) A female patient, who has been sexually assaulted, has called 911 for assistance. She states that she does not know the assailant and showered immediately after the assault. The police have not yet been notified. Which of these statements would be appropriate in the care of this patient? A) "Where are the clothes that you were wearing?" B) "Why did you feel the need to shower?" C) "Do you think that you can identify the person who assaulted you?" D) "It is unfortunate that you showered; it will be harder to get evidence now." Answer: A Diff: 2 Page Ref: 715 Objective: 23-10

23 Copyright © 2018 Pearson Education, Inc.

92) A 50-year-old female states that she has gone through menopause and no longer experiences menstrual periods. Using appropriate medical terms for documentation, the EMT would describe this status as: A) Pre-menses B) Dysmenorrhea C) Pro melena D) Post menarche Answer: D Diff: 2 Page Ref: 716 Objective: 23-1 93) Which of these assessment findings would help convince the EMT that a patient with abdominal pain and vaginal bleeding is experiencing a spontaneous abortion (miscarriage)? A) History of dysmenorrhea B) Upper abdominal cramping C) Last menstrual period six weeks ago D) Pain start acutely this morning Answer: C Diff: 2 Page Ref: 716 Objective: 23-10 94) A patient with lower abdominal pelvic pain, vaginal discharge, and elevated temperature has summoned EMS. In getting a medical history from this patient, which of these findings is the most important piece of information to relay to the receiving emergency department? A) Removal of her gallbladder three months ago B) Surgical removal of a twisted ovary eight years ago C) Internal procedure on her uterus yesterday by the gynecologist D) History of anemia and hypertension Answer: C Diff: 3 Page Ref: 718 Objective: 23-10 95) In discussing patients with chlamydia and gonorrhea with a group of EMT students, you would inform them that: A) These coinfections are treated with antibiotic therapy B) These coinfections often result in the need for surgery C) These coinfections require antiviral agents to be cured D) These coinfections tend to be caused by a parasite Answer: A Diff: 3 Page Ref: 717 Objective: 23-10

24 Copyright © 2018 Pearson Education, Inc.

96) A 21-year-old female patient has called EMS for severe right-sided abdominal pain. Which of these questions should be asked before the others? A) "Have you had a kidney stone before?" B) "When was your last normal period?" C) "Does the pain radiate into your back?" D) "Have you had any vaginal bleeding in the past?" Answer: B Diff: 2 Page Ref: 718 Objective: 23-6 97) The EMT would recognize the profession of urology as applying to: A) All organs in the pelvic cavity B) The kidneys, ureters, bladder, and urethra C) A female's urinary and reproductive organs D) The urinary organs as well as the penis and vagina Answer: B Diff: 2 Page Ref: 719 Objective: 23-1 98) A patient with a history of renal failure, among other things, has an impairment in the ability to: A) Control urination B) Retain adequate fluids C) Regulate electrolytes D) Maintain a normal blood sugar Answer: C Diff: 2 Page Ref: 720 Objective: 23-15 99) When differentiating chronic renal failure from acute renal failure, the EMT realizes: A) Chronic renal failure often requires long-term dialysis B) Acute renal failure is often permanent C) Chronic renal failure is reversible with IV fluid administration D) Acute renal failure is treated with a kidney transplant Answer: A Diff: 1 Page Ref: 721 Objective: 23-13 100) A patient informs you that she has a dialysis fistula on her left arm. You would: A) Clean the fistula with alcohol prior to getting a blood pressure on the arm B) Realize that it is impossible to obtain a blood pressure on the patient C) Immediately wrap the site with roller gauze D) Avoid performing a blood pressure measurement on the patient's left arm Answer: D Diff: 2 Page Ref: 722 Objective: 23-13 25 Copyright © 2018 Pearson Education, Inc.

101) The wife and primary care provider of a disabled male patient tells you that he has a suprapubic catheter in place. Upon assessment of the patient, which of these findings would you expect? A) Surgical tube coming out of the pelvic area B) Tubes coming from the back in the area of the kidneys C) Tubing coming from the urethra of the patient's penis D) Surgical tubing coming from the patient's rectum Answer: A Diff: 1 Page Ref: 723 Objective: 23-14 102) If a patient missed his last dialysis appointment, which complaint would you anticipate? A) Bleeding from the fistula B) Hypotension C) Generalized abdominal pain and fever D) Shortness of breath Answer: D Diff: 2 Page Ref: 722 Objective: 23-13 103) You are preparing to transport a patient with an indwelling urinary catheter. The bag is full and your partner starts to empty it. Which of these statements would be appropriate to make to your partner? A) "Let's leave it full so the hospital can check the urine for infection." B) "Let's make sure that we measure the amount prior to disposing of it." C) "For the catheter to work properly, the bag must be left half-full." D) "We should leave it as is so the hospital can determine the volume." Answer: B Diff: 3 Page Ref: 723 Objective: 23-14 104) When transporting a patient with an indwelling catheter and bag, which of these actions is appropriate? A) Have the caregiver remove it to decrease the risk of infection B) Ensure that it remains on the lap of the patient at all times C) Position the bag below the level of the patient's bladder D) Deflate the balloon to decrease the risk of accidental removal Answer: C Diff: 2 Page Ref: 723 Objective: 23-14

26 Copyright © 2018 Pearson Education, Inc.

105) You have been called to a local dialysis center for a patient with a bleeding fistula on her right upper arm. Despite all care at the center, the site continues to bleed. Your best choice in the initial care of the patient would be: A) To apply a constricting band above the fistula B) To place a pressure dressing over the fistula C) Lowering the patient's arm during transport D) To allow site the bleed to wash bacteria from the fistula Answer: B Diff: 2 Page Ref: 722-723 Objective: 23-13 106) The EMT would recognize a hematologic condition in which of these patients? A) A patient with a large hematoma to the scalp after falling B) A patient with an elevated white blood cell count due to leukemia C) A patient with antibodies in her bloodstream capable of casing anaphylaxis D) A patient with pneumonia who is profoundly hypoxic Answer: B Diff: 2 Page Ref: 710 Objective: 23-7 107) A patient with generalized weakness informs you that she has a history of iron-deficient anemia. You recognize that this patient has a history of: A) An overabundance of RBCs not capable of carrying oxygen B) Problems transferring oxygen from the lungs to the RBCs C) A decreased amount of hemoglobin available to carry oxygen D) Malformed RBCs that have a difficult time carrying oxygen Answer: C Diff: 3 Page Ref: 713 Objective: 23-7 108) The fundamental problem associated with anemia is: A) Thickened blood consistency B) Elevated blood pressure C) Predisposition to infection D) Cellular hypoxia Answer: D Diff: 2 Page Ref: 713 Objective: 23-7

27 Copyright © 2018 Pearson Education, Inc.

109) When assessing a patient with chronic anemia, the EMT would expect which of these findings? A) Pale skin B) Yellow skin C) Joint pain D) Flushed skin Answer: A Diff: 1 Page Ref: 713 Objective: 23-7 110) You have been called for a patient with a sickle cell crisis. In reviewing the pathophysiology of sickle cell anemia, you understand that the underlying problem is: A) Blood that is thick and cool, and has difficulty circulating B) Abnormal hemoglobin formation that means the blood cannot carry oxygen as efficiently C) An abnormally elevated number of white blood cells, which causes microemboli in the capillary beds D) A decreased amount of circulating RBCs that do not release O2 Answer: B Diff: 2 Page Ref: 713 Objective: 23-7 111) A patient with sickle cell crisis will most likely complain of: A) High fever B) Diarrhea C) Severe pain D) Blood-tinged urine Answer: C Diff: 2 Page Ref: 713 Objective: 23-7 112) You are treating a male patient with a sickle cell crisis. Vital signs are pulse, 124; respirations, 22 breaths/min; blood pressure, 156/92 mmHg; breathing, unlabored; and SpO2, 91%. Which instruction would you provide to other EMTs in the care of this patient? A) "Let's place the patient on a nasal cannula with 2 lpm of oxygen." B) "I need someone to administer aspirin so we can thin his blood and get it moving." C) "The best thing we can do is just to transport the patient to the emergency department." D) "Remember that using a blood pressure cuff can cause significant bruising." Answer: A Diff: 2 Page Ref: 713 Objective: 23-6

28 Copyright © 2018 Pearson Education, Inc.

113) A patient with hemophilia fell backward onto his right hip and now complains of pain to that area. The primary assessment reveals no life-threatening conditions. Vital signs are pulse, 112; respirations, 18 mmHg; blood pressure, 102/84 mmHg; and SpO2, 99%. During transport, which of these findings would be of greatest concern? A) Increasing bruising and pain to the right hip becomes evident B) SpO2 reading dips to 95% for a few minutes C) Hematoma formation on the right hip that is palpable D) Heart rate increases to and is sustained at more than 130 beats/min Answer: D Diff: 3 Page Ref: 713 Objective: 23-7 114) Hemophilia is a blood disorder in which the: A) Platelets are unable to transport oxygen B) Blood is unable to form clots properly C) Red blood cells are malformed and do not form clots D) Blood is thin and bleeding is difficult to control Answer: B Diff: 2 Page Ref: 713 Objective: 23-7 115) A 19-year-old patient with hemophilia and behavioral problems has intentionally cut his right wrist. Assessment indicates an open airway, adequate breathing, and profound bleeding from the self-inflicted laceration. Which instructions would you provide to other EMTs taking care of this patient? A) "Let's use normal bleeding-control procedures and see how that works." B) "Let's elevate the patient's arm and place a tourniquet distal to the cut." C) "Since control of bleeding is virtually impossible, we need to transport the patient immediately." D) "We need to quickly apply a tourniquet directly above the cut." Answer: A Diff: 2 Page Ref: 713 Objective: 23-6 116) A 26-year-old patient with hemophilia fell in his garage, cutting his arm on a nail protruding from a door frame. Emergency Medical Responders are on scene holding a bloodsoaked towel on his right forearm that is still bleeding. The patient is alert and oriented with a pale complexion. The EMRs report a pulse of 98, respirations of 18 breaths/min, blood pressure of 142/66 mmHg, and SpO2 of 99% on room air. Your initial priority in caring for this patient is: A) Providing supplemental oxygen B) Immediate transport to the hospital C) Replacing the towel with a sterile dressing D) Stopping the bleeding Answer: D Diff: 2 Page Ref: 713 Objective: 23-7 29 Copyright © 2018 Pearson Education, Inc.

117) Along with immunocompromised patients, which other age group(s) may have a vague presentation of an abdominal pathology due specifically to a depressed inflammatory response? A) Adult patients B) School-age patients C) Geriatric patients D) Teenager patients Answer: C Diff: 3 Page Ref: 707 Objective: 23-5

30 Copyright © 2018 Pearson Education, Inc.

Prehospital Emergency Care, 11e (Mistovich et al.) Chapter 24 Environmental Emergencies 1) What of these patients is most accurately described as suffering an environmental emergency? A) A young female patient who slipped on ice and has pain and deformity to her left arm B) A male patient who fell into a stream and sustained a laceration to his head after hitting a rock C) A female patient who is sick and vomiting after eating potato salad at an outdoor church picnic D) An intoxicated male patient who collapsed in the snow and is complaining that he feels cold Answer: D Diff: 2 Page Ref: 729 Objective: 24-6 2) The EMT shows she understands the regulation of temperature in the human body when she states: A) "If the body becomes too hot or too cold, the brain will send instructions out to the body to either retain or lose heat." B) "If the air outside the body is cold and the body is too hot, the cold air will move into the body to cool it." C) "Since the cellular activity in the body constantly generates heat, the body must always be cooler than the air that surrounds it." D) "For the body's cells to function properly, the human body must always be warmer than the air that surrounds it." Answer: A Diff: 3 Page Ref: 729 Objective: 24-2 3) You are assessing a conscious but confused hunter who became lost in the woods on a cold day. Your assessment shows him to have an open airway, adequate breathing, and a weak radial pulse. His skin is cold to the touch and he is shivering. The EMT should recognize that the: A) Cold has caused the brain to become dysfunctional, as evidenced by the shivering B) Shivering actually represents a small seizure, indicating that the brain is cold C) Shivering is a protective means by which the body is attempting to warm itself D) Shivering indicates that the patient's core body temperature has fallen to less than 90°F Answer: C Diff: 3 Page Ref: 729 Objective: 24-2

1 Copyright © 2018 Pearson Education, Inc.

4) A young male patient who was angry with friends wandered away from a party and spent the night outside uncovered in 40-50°F temperatures. He is confused, has decreased but adequate breathing, and has a weak radial pulse. His skin is cool and capillary refill delayed. You note that he is not shivering. Based on this presentation, what can you safely conclude? A) The patient is exhibiting signs that he is compensating and conserving heat B) The patient's body temperature is most likely dangerously low C) The absence of shivering indicates the body temperature is almost back to normal D) The cause of his confusion is related to alcohol and not the overnight temperatures Answer: B Diff: 3 Page Ref: 730 Objective: 24-6 5) A person is in a hot environment and his body is successfully compensating to off-load excess heat. Which assessment findings best illustrate this process? A) Cool and clammy skin, normal blood pressure, and slow respirations B) Warm and diaphoretic skin, elevated heart rate, and increased respirations C) Diaphoretic skin, decreased heart rate, and decreased blood pressure D) Warm skin, elevated heart rate, decreased respirations, and decreased blood pressure Answer: B Diff: 3 Page Ref: 730 Objective: 24-2 6) What is the most significant mechanism by which the body can lose heat? A) Conduction B) Respirations C) Radiation D) Evaporation Answer: C Diff: 1 Page Ref: 731 Objective: 24-2 7) You are treating a male patient who is hypothermic. What should you do to decrease heat loss caused by radiation? A) Wrap a towel around the patient's head B) Turn off the fan in the back of the ambulance C) Wipe the patient's skin down with warm water D) Apply warm packs to the patient's body Answer: A Diff: 2 Page Ref: 731 Objective: 24-6

2 Copyright © 2018 Pearson Education, Inc.

8) Which of these patients is losing body heat by the mechanism of convection? A) A 41-year-old female who is intoxicated and passed out in the snow B) A 56-year-old male who is confused and perspiring heavily C) A 63-year-old female who took a drug that depressed her brain function D) A 30-year-old male in a cool temperature and exposed to a light breeze Answer: D Diff: 2 Page Ref: 731 Objective: 24-1 9) A patient who is conscious and breathing has been pulled from a stream of cold water. To decrease her loss of heat via the mechanism of conduction, the EMT should immediately: A) Cover her with a blanket B) Remove her wet clothing C) Provide positive pressure ventilation D) Encourage her to stop shivering Answer: B Diff: 2 Page Ref: 731 Objective: 24-6 10) You have been called to a football practice field on a very hot day for a heat-related emergency. On scene, you find a young male patient panting and lying under a tree. His skin is flushed, and teammates are continually wiping the sweat off his face, arms, and chest with towels. Which of these statements to his teammates would be most appropriate? A) "Keep wiping guys. You are helping him retain water that his body needs." B) "Good job guys. By wiping him down you are helping him cool down." C) "Please let him sweat. It is helping his body cool down." D) "To keep him from sweating, let's put his shirt back on." Answer: C Diff: 2 Page Ref: 731-732 Objective: 24-2

3 Copyright © 2018 Pearson Education, Inc.

11) You have been called to a residence for a patient who is sick. On scene, the family tells you that the patient is being treated for "stomach flu" and has had a fever of 102°F for the past two days. She has been taking all of the medications prescribed by her doctor, but this evening became very "sweaty." Assessment shows her to be stable with a pulse of 88, respirations of 18 breaths/min, adequate blood pressure of 128/68 mmHg, and an oral temperature of 100.2°F. Based on these data, the EMT should recognize that: A) The patient's body is attempting to retain heat by covering itself with sweat, which acts as a barrier to heat loss B) The sweat is causing the loss of body heat since the sweat is warmer than the body, causing heat to move from inside to outside the body C) The sweat is causing the patient's body to retain heat since sweat is warmer than the temperature of the body, causing the heat in the body to exit and cool the patient D) The patient's body is cooling itself by transferring heat into the sweat, which is then evaporated into the air Answer: D Diff: 3 Page Ref: 731-732 Objective: 24-2 12) The EMT should recognize which of these patients is experiencing generalized hypothermia? A) A 21-year-old female who was trapped in cold water for 10 minutes before being rescued B) A 19-year-old male who is intoxicated and fell asleep under a ceiling fan for three hours C) A 34-year-old male whose fingers and toes are blue from exposure to the cold D) An 87-year-old alcoholic with hand pain and numbness after sweeping snow without gloves Answer: A Diff: 2 Page Ref: 732-733 Objective: 24-4 13) You have been chosen by the medical director to give a 15-minute presentation on coldrelated emergencies to a local volunteer fire department. In preparing the talk, which teaching point should you emphasize? A) Emergency responders play a critical role in caring for hypothermic patients since the most important phase of care occurs during the first 30 minutes B) Of the two types of cold-related injuries, hypothermia or frostbite (a freezing cold injury), frostbite is more life threatening because it results in the loss of fingers, toes, arms, or legs C) Coma or unresponsiveness in the patient with a cold-related injury is actually protective since it decreases the body's energy requirements D) A cold-related emergency becomes critical when the body temperature falls to less than 79°F since at this point the body loses ability to regulate its temperature Answer: A Diff: 3 Page Ref: 733 Objective: 24-6

4 Copyright © 2018 Pearson Education, Inc.

14) Which of these findings would present earliest in a patient with hypothermia? A) Confusion B) Hypotension C) Shivering D) Bradycardia Answer: C Diff: 1 Page Ref: 734 Objective: 24-3 15) Which of these statements about immersion hypothermia is false? A) When a patient is pulled from cold water, he should be immediately moved to a warmer environment B) Immersion hypothermia typically does not occur until the water temperature is less than 35°F C) Adults typically can withstand exposure to colder water for a longer period of time than children D) It is important to remove the clothes of a patient who has been exposed to cold water as soon as possible Answer: B Diff: 2 Page Ref: 735 Objective: 24-4 16) You have been called for a 78-year-old female patient who slipped while getting out of her bathtub. She is confused and cold to the touch. You learn that the patient slipped 24 hours ago and has been on the bathroom's tile floor ever since. Since it is summer, she also had her air conditioner running on high. The EMT would best categorize this emergency as: A) External urban hypothermia caused by a medical complaint B) Generalized hyperthermia caused by convection C) Urban hypothermia with heat loss caused by conduction D) Localized external hypothermia caused by immobility Answer: C Diff: 3 Page Ref: 736 Objective: 24-4 17) A young boy complains of pain to his fingers after spending several hours outside riding a sled in cold winter temperatures. After ensuring he has no life-threatening conditions, you turn your attention to his hands and note that the fingers are cold to the touch. You suspect that he is suffering from a superficial freezing cold injury. Which sign or symptom would help confirm this suspicion? A) Skin on the fingers that is soft and numb B) History of being exposed to cold temperatures C) Swelling and blisters noted to the fingers D) Firm skin that is cold and numb Answer: A Diff: 2 Page Ref: 736 Objective: 24-1

5 Copyright © 2018 Pearson Education, Inc.

18) On entering an apartment for a call related to a confused elderly female patient, you note that the air conditioner is running on high and the room is extremely cold. Which other clue obtained during the scene size-up would best indicate that this patient may be suffering from urban hypothermia? A) Patient covered with diaphoresis B) Fan blowing directly onto the patient C) Oxygen concentrator next to the patient D) Medication for hypertension next to the patient Answer: B Diff: 2 Page Ref: 736 Objective: 24-4 19) You are called for an elderly woman who has slipped on the ice. On arrival, you find her supine on an icy sidewalk and responsive to painful stimuli with decorticate posturing (flexion). She has blood coming from a laceration on the back of her head and is breathing agonally at 4 times a minute. What should you do immediately? A) Start positive pressure ventilation B) Insert a nasal airway into the right nare C) Perform the manual jaw-thrust maneuver D) Remove the patient to the warm ambulance Answer: C Diff: 2 Page Ref: 748, Figure 24-14 Objective: 24-6 20) An Emergency Medical Responder (EMR) quickly checks the breathing and pulse of a patient with Alzheimer's disease who has been sleeping outside in the cold for several hours. The EMR informs you that the patient grimaces to painful stimuli but is not breathing and does not have a pulse. What is your best response? A) "Let's get the automated external defibrillator ready and apply it to the patient." B) "Someone start CPR, but do not insert an oral airway since this can cause ventricular fibrillation." C) "We need to contact medical direction to see what the doctor wants us to do since the patient is hypothermic." D) "Let's perform the primary assessment again, and this time take longer to assess for breathing and a pulse." Answer: D Diff: 3 Page Ref: 739-740 Objective: 24-6

6 Copyright © 2018 Pearson Education, Inc.

21) When moving a patient with severe generalized hypothermia, which of these instructions is most appropriate to relay to your fellow EMS providers? A) "I want everyone to take extra care in moving the patient very gently to the stretcher. We do not want her to go into cardiac arrest." B) "Be careful when moving the patient to the stretcher; the cold makes the patient very prone to bone fractures." C) "Let's place the patient on her side on the stretcher since her body temperature contraindicates placement of a nasal airway." D) "Let's secure the patient with her arms and legs extended since this position promotes the retention of heat in the body." Answer: A Diff: 2 Page Ref: 742 Objective: 24-6 22) Bystanders have called 911 for a female patient who lives on the street. She has been outside in the cold all night and is muttering incomprehensible words. Her airway is open and her breathing is adequate. Her skin is cold, and a radial pulse is present but weak. What should the EMT do next? A) Perform a secondary assessment B) Obtain a blood pressure C) Assess breath sounds D) Transfer the patient to the ambulance Answer: D Diff: 2 Page Ref: 742-743 Objective: 24-6 23) Which sign or symptom best indicates that the patient with cold skin is experiencing generalized hypothermia? A) Blood pressure of 108/60 mmHg B) Respirations of 24 breaths/min C) Heart rate of 48 beats/min D) Pulse oximetry reading of 95% Answer: C Diff: 2 Page Ref: 741 Objective: 24-6 24) When obtaining a history of a 5-year-old boy with generalized hypothermia, which question should be asked before all others? A) "Are all of his childhood shots up-to-date?" B) "How long was he out in the cold?" C) "When did he eat or drink last?" D) "Has he ever been in the hospital before for this reason?" Answer: B Diff: 2 Page Ref: 740 Objective: 24-6

7 Copyright © 2018 Pearson Education, Inc.

25) Which thermometer should be utilized by an EMT when assessing the temperature of a patient with hypothermia? A) Tympanic thermometer in the patient's ear B) Medical-grade thermometer in the patient's rectum C) Household rectal thermometer in the patient's rectum D) Medical-grade thermometer in the patient's mouth Answer: B Diff: 2 Page Ref: 740 Objective: 24-6 26) The EMT is correctly assessing the skin temperature of a patient who has been exposed to cold temperatures for an extended period of time when he: A) Feels the patient's forehead with the back of his hand B) Places the palm of his hand on the patient's neck C) Feels the skin temperature on the patient's chest D) Places the back of his hand on the patient's abdomen Answer: D Diff: 2 Page Ref: 740 Objective: 24-6 27) When providing needed oxygen to a patient with a core temperature of 95°F and adequate breathing, which strategy is the best option? A) Positive pressure ventilation with warm oxygen B) Humidified oxygen delivered through a nonrebreather mask C) Warmed and humidified oxygen delivered through a nonrebreather mask D) Warm but nonhumidified oxygen delivered through a nonrebreather mask Answer: C Diff: 2 Page Ref: 742 Objective: 24-6 28) A patient with hypothermia is in cardiac arrest. The automated external defibrillator (AED) is applied and delivers one shock. Following the shock, what should the EMT do next? A) Start cardiopulmonary resuscitation B) Reanalyze the patient's heart rhythm C) Provide ventilations but not compressions D) Check for a pulse Answer: A Diff: 2 Page Ref: 742 Objective: 24-6

8 Copyright © 2018 Pearson Education, Inc.

29) You have been called for an alert and oriented male patient. Upon your arrival, his roommate states that the patient spent several hours locked out of his house in the cold temperatures after an evening of drinking since no one was home and the patient lost his house keys. Since the primary and secondary assessments have ruled out immediate life-threatening conditions, you have decided to rewarm the patient. Which of these warming techniques would be most appropriate for this patient? A) Placement in a tub of warm water B) Hot packs to the patient's chest, groin, and armpits C) Massaging of the arms and legs after applying warm blankets D) Blowing hot air with a hair dryer over the patient's arms and legs Answer: B Diff: 2 Page Ref: 743 Objective: 24-6 30) Medical direction has ordered you to begin rewarming an unresponsive patient with a core temperature of 93°F. Which of these measures would most benefit this patient? A) Give him warm fluids to drink to increase internal body temperature B) Gently massage the patient's arms and legs C) Wrap the patient in several warm blankets D) Place one extremity at a time in warm water Answer: C Diff: 2 Page Ref: 743 Objective: 24-6 31) Which of these statements made by an EMT shows that he understands the care of a patient with a freezing cold injury to the foot? A) "While it is recommended to remove wet clothing from the patient, it is best to leave clothing that is frozen to the skin in place." B) "If the patient has blisters on his foot, it is beneficial to break them so the fluid inside will not freeze and cause additional injury." C) "Gentle massage of the foot has been shown to help the patient since this enhances the circulation of warm blood to the frozen area." D) "The EMT should avoid immobilizing the foot since this will further decrease circulation of blood into the frozen area." Answer: A Diff: 2 Page Ref: 744 Objective: 24-6

9 Copyright © 2018 Pearson Education, Inc.

32) You are giving a talk on hunting safety. A hunter asks you when someone should attempt to rewarm a hand or foot that is frostbitten. What is your best response? A) "You never want to rewarm a frozen body part since that can cause additional damage." B) "Attempt to rewarm a frozen body part only if the person still has some feeling in it." C) "It is best to attempt rewarming in any situation because the benefit of rewarming outweighs the risk of not rewarming." D) "Rewarming should take place only when there is no chance that the tissue will refreeze." Answer: D Diff: 2 Page Ref: 744 Objective: 24-6 33) You have been called to a hunting camp for a patient with a severe freezing cold injury to his hand and fingers. The patient is suffering no life-threatening conditions and you elect to rewarm the affected areas. Which of these warming techniques would be most appropriate for this patient? A) Immerse the hand and fingers in water no hotter than 120°F B) Place the hand lower than the heart after rewarming is complete C) Do not stir the water while the hand is submerged to decrease risk for rewarming injury D) Thaw and rewarm the tissue as quickly as possible Answer: D Diff: 2 Page Ref: 744 Objective: 24-6 34) After successfully rewarming a patient's frozen foot and frozen toes, the EMT should: A) Cover the foot and toes with dry, sterile dressings B) Gently massage the foot and toes C) Ambulate the patient to ensure return of adequate motor function D) Place the foot below the level of the heart to enhance circulation Answer: A Diff: 2 Page Ref: 744 Objective: 24-6 35) You are on scene with a patient who has a freezing cold injury. The patient stated that he had no sensation in his hand or fingers prior to the initiation of a rewarming process. You have subsequently begun rewarming his hand and fingers. Which of these statements made by the patient following the rewarming process is of greatest concern? A) "The blisters on my fingers haven't broken yet." B) "My fingers are really starting to hurt now!" C) "There is no pain when I rub my fingers with my other hand." D) "My hand is feeling funny. It seems to be starting to tingle a little bit." Answer: C Diff: 3 Page Ref: 745 Objective: 24-5

10 Copyright © 2018 Pearson Education, Inc.

36) The EMT should recognize heat cramps as the probable cause of a patient's problem when the patient states: A) "I am having a hard time moving my fingers." B) "I cannot remember what day it is." C) "I have pain in my belly and legs." D) "My neck feels as though it is in spasm." Answer: C Diff: 2 Page Ref: 745 Objective: 24-7 37) During an in-service program, your medical director asks if anyone can tell her the cause of heat cramps. What response would be most appropriate? A) "They are thought to occur when the patient does not consume enough water and becomes dehydrated." B) "They are caused when the patient's electrolytes such as salt and calcium become deranged and are too low in the body." C) "They occur when the body can no longer compensate and off-load excess heat." D) "They occur when blood vessels constrict and decrease blood flow to the affected muscle." Answer: B Diff: 3 Page Ref: 745 Objective: 24-7 38) The EMT shows that he understands the danger of heat stroke when he states: A) "In heat stroke, the body loses its ability to rid itself of excess heat, causing the core temperature to rise." B) "Heat stroke is a dire emergency because the body has lost much of its fluid and ability to circulate oxygen." C) "The EMT can identify heat stroke because the patient will have skin that is warm and dry instead of hot and moist." D) "Heat stroke occurs when the patient's blood vessels have over-dilated, causing a mild state of shock." Answer: A Diff: 2 Page Ref: 749 Objective: 24-7 39) Which of these patients with heat emergency would be the highest priority for transport? A) A 24-year-old who is intoxicated and is complaining of pain and spasms to his legs B) A 34-year-old with confusion and cool, diaphoretic skin along with tachycardia C) A 28-year-old responsive to verbal stimuli with tachycardia and cool, moist skin D) A 19-year-old responsive to verbal stimuli with hot dry skin and tachycardia Answer: D Diff: 2 Page Ref: 749 Objective: 24-7

11 Copyright © 2018 Pearson Education, Inc.

40) You are called to a construction site on a hot and humid day for a male patient with a heat emergency. On scene, you find a 49-year-old man who is responsive to verbal stimuli. Coworkers state that the patient had been moving concrete blocks all day in the hot weather and had not rested, eaten, or had anything to drink in the past two hours. During the primary assessment, you note the patient's airway to be open and breathing rapid but adequate. His radial pulse is rapid and bounding, and his skin is hot and dry. Which of these findings makes this patient a priority for immediate and rapid transport? A) Bounding pulse B) Altered mental status C) Hot and dry skin D) Tachypneic respirations Answer: C Diff: 3 Page Ref: 749 Objective: 24-8 41) You have been called to a park for a teenage boy who is "sick." On scene, you find a 16year-old male patient who knows his name but is confused about place and time. Friends state that they have been playing basketball most of the morning and afternoon. The temperature is in the 90s and the humidity is high. The patient's airway is patent and his breathing is fast but adequate. His pulse is weak and rapid. Observation of his skin reveals it to be pale, cool, and moist. Oxygen is being administered via nonrebreather face mask by Emergency First Responders. Which other step would be appropriate care for this patient? A) Place him supine on the stretcher with feet elevated B) Lay the patient down and place him in a side-lying position C) Administer Tylenol tablets by mouth to lower the body temperature D) Pour cold water over the patient and fan him aggressively Answer: A Diff: 2 Page Ref: 752 Objective: 24-8 42) The primary assessment of a teenage male patient who is unresponsive with hot, moist, and flushed skin reveals his airway to be open, breathing shallow, and radial pulse weak. You have initiated positive pressure ventilation with supplemental oxygen. What should you consider doing next? A) Perform a secondary assessment B) Transfer the patient into the ambulance C) Get a medical history D) Perform the primary assessment again Answer: B Diff: 2 Page Ref: 752 Objective: 24-8

12 Copyright © 2018 Pearson Education, Inc.

43) When cooling a patient with possible heat stroke, which of these findings would be of most concern to the EMT? A) Blood pressure increasing to 102/78 mmHg B) The heart rate decreasing from 140 to 120 beats/min C) Observing your partner apply a sheet soaked in tepid water to the patient D) Determining that the patient is now shivering after having cold packs applied to the neck Answer: D Diff: 3 Page Ref: 752 Objective: 24-8 44) You are en route to the hospital with a patient who is unresponsive and has hot, dry skin. His friends state that the patient had been drinking and passed out in the hot sun for several hours before 911 was called. An Emergency Medical Responder is ventilating the patient with a bagvalve mask attached to supplemental oxygen. Which of these actions would be most beneficial to the patient next? A) Be prepared for vomiting by placing the patient in a prone position B) Wrap the patient in a sheet that has been soaked with cold water C) Place cold packs to the patient's groin and armpits D) Gently pour cold water over the patient to rapidly cool him down Answer: C Diff: 2 Page Ref: 752 Objective: 24-8 45) A landscaper who has been working in 105°F weather is found collapsed in a client's yard. He is responsive to painful stimuli and has an open airway. His breathing is shallow, alveolar breath sounds are absent, and radial pulse is weak. The patient's skin is hot and dry and appears to be sunburned. The EMT should immediately: A) Start positive pressure ventilation B) Wrap the patient in a sheet soaked with cool water C) Administer oxygen through a nonrebreather mask D) Attach the AED, but do not turn it on yet Answer: A Diff: 2 Page Ref: 752 Objective: 24-8 46) You are treating a patient with suspected heat cramps. Your medical director has issued standing orders for such treatment given the high number of heat-related emergencies to which your service routinely responds in the summer months. Regarding these standing orders, which of these instructions would seem most appropriate? A) Mix one teaspoon salt in one quart of water and administer half of a glass every 15 minutes B) Make and administer a saltwater solution by combining Gatorade with one teaspoon of salt C) Administer salt tablets every 15 minutes until the cramping subsides D) Have the patient drink one glass of cold tap water every 15 minutes until he has to urinate Answer: A Diff: 2 Page Ref: 753 Objective: 24-8 13 Copyright © 2018 Pearson Education, Inc.

47) A young female patient was weeding her garden when she was bitten on her left hand by an unidentified snake. The primary assessment does not reveal any life threats and vital signs are stable. In caring for this patient and her injury, it is critical that the EMT: A) Apply ice packs to the area of the snake bite B) Administer an epinephrine auto-injector into the site C) Have the patient remove her wedding ring from her left hand D) Attempt to locate the snake for identification Answer: C Diff: 2 Page Ref: 760 Objective: 24-11 48) You are providing a "stand-by" service at the county fair. At midmorning, a 50-year-old male patient approaches your tent and states that he was bitten in the right shoulder by some sort of insect and has tremendous pain. Assessment reveals his shoulder to have a large reddened area with a sting mark in the middle. The skin is warm to the touch. Your care of this patient should include: A) Elevating the right hand B) Applying a cold pack to the site C) Scrubbing the site with soap and sterile water D) Massaging the site gently Answer: B Diff: 2 Page Ref: 760 Objective: 24-11 49) You are transporting a patient who was bitten on her hand by a spider approximately 20 minutes ago. On scene, the primary assessment revealed no life threats and vital signs were stable. While conducting the reassessment, which of these findings would be of most concern to the EMT? A) Swelling to the area of the bite B) Increased pain to the area of the bite C) Complaint of a headache D) Hives to the chest and abdomen Answer: D Diff: 2 Page Ref: 760 Objective: 24-11 50) A patient has been bitten by an unidentified snake. Which of these statements made by the patient would lead the EMT to believe that the snake was poisonous? A) "Its eyes were round." B) "It was about two feet long." C) "Its head was triangular." D) "It had a stripe down its back." Answer: C Diff: 2 Page Ref: 755 Objective: 24-10

14 Copyright © 2018 Pearson Education, Inc.

51) A 44-year-old female patient was hiking in a state park when she was bitten by a snake in her left calf. To get help, she had to hike 4 miles to the first-aid station where you are located. The hike took her 1½ hours. Your assessment reveals a red and swollen area to her leg, and the patient complains of discomfort to that area. Closer inspection reveals two puncture wounds in the center of the affected area. The patient asks you if she is going to die from the snake bite. What would be your most appropriate reply? A) "While you still need to get checked out, if venom was injected, the signs and symptoms of a generalized reaction typically appear very quickly." B) "We need to get you to the hospital immediately. Anytime that a snake bites, it injects venom from its fangs." C) "Based on the way the bite looks, I think that you were bitten by a nonpoisonous snake." D) "Walking all of that distance probably saved you since your body metabolized the poison faster." Answer: A Diff: 3 Page Ref: 755 Objective: 24-10 52) A patient has been bitten by what appears to be a black widow spider. The bite to her left hand is dull and achy, and the patient states that the muscles in her shoulders and back are starting to hurt, as if they are cramping up. After ensuring that no life-threatening conditions are present and administering oxygen, the EMT should: A) Pour a vinegar solution over the bite B) Keep the hand below the level of the heart C) Massage the muscles of the back and shoulders D) Apply hot packs to the bite Answer: B Diff: 2 Page Ref: 757 Objective: 24-11 53) You have been called to a residence for a spider bite. On scene, the homeowner states that he was moving some boxes in his garage when he accidentally grabbed a spider and it bit him. He killed the spider and shows you a brown-colored spider with a "violin shape" on its back. He then states that the bite does not hurt and he sees no need to go to the hospital. As a knowledgeable EMT, you should state: A) "Since the bite of that spider generally causes immediate pain and swelling, you probably can go without additional medical attention." B) "You should be okay by refusing transport, but make sure to wash the bite site daily with soap and water." C) "Let us take you to the hospital. They have an antivenom that will allow this wound to heal and make you resistant to future bites." D) "You really need to go because the bite of that spider does not heal very well and will put you at risk for infection." Answer: D Diff: 2 Page Ref: 757-758 Objective: 24-11

15 Copyright © 2018 Pearson Education, Inc.

54) A 24-year-old female who was hiking in shorts informs you that she has been bitten by an insect on her leg. She states no other complaints. Assessment reveals a tick embedded in her leg. As an EMT, you should provide care to this patient by: A) Wrapping the tick and bite area with roller gauze B) Administering supplemental oxygen C) Removing the tick with tweezers D) Applying a hot pack over the tick Answer: C Diff: 2 Page Ref: 758 Objective: 24-11 55) You have been asked by your medical director to put together an emergency kit specifically for patients who have been stung or bitten by small marine life animals. Which of these items should you place in that kit? A) Cold packs B) Cayenne pepper extract C) Alcohol D) Vinegar Answer: D Diff: 2 Page Ref: 760 Objective: 24-12 56) You have been called by a family for their mother, who is "not acting right." On scene, you find the 69-year-old woman to be confused and seemingly slow in talking. Her airway is open and respirations are 12 breaths/min. Her pulse is 58 beats/min, and her skin is cool to the touch, despite the thermostat keeping the house at normal room temperature. The family states that she has been this way for about three days, but before then seemed to be doing well. Your partner obtains a blood pressure reading of 104/52 mmHg. When getting a medical history, which of these statements made by the family would make sense, given the presentation of the patient? A) "Mom suffers from a low thyroid." B) "Mom takes a daily vitamin every day." C) "Mom had her gallbladder out last year." D) "Mom used to take a blood pressure pill." Answer: A Diff: 3 Page Ref: 736 Objective: 24-3

16 Copyright © 2018 Pearson Education, Inc.

57) A patient has been struck by lightning at a picnic. A doctor on scene states that the patient was in cardiac arrest after the strike, but with 1 minute of CPR, has a heartbeat and weak sonorous respirations. The patient remains unresponsive. Which priority care should the EMT provide? A) Care for the burn injury on the patient's back B) Perform the jaw-thrust maneuver C) Initiate positive pressure ventilation D) Attach the automated external defibrillator Answer: B Diff: 2 Page Ref: 763 Objective: 24-14 58) A young man has been struck by lightning on a golf course and is in cardiac arrest. Other golfers say that he was struck 8-10 minutes ago. What should the EMT do immediately? A) Start cardiopulmonary resuscitation B) Attach the automated external defibrillator C) Ventilate the patient at 10 to 12 breaths/min D) Do a secondary assessment to locate the burn sites Answer: A Diff: 2 Page Ref: 763 Objective: 24-14 59) A 31-year-old groundskeeper has been stung by bees three times in his right arm and twice to his left ankle. After performing the primary and secondary assessments, which reveal no acute disturbances, what should you do next? A) Place a constricting band above the sting sites B) Wash the sting sites with sterile water and soap C) Attempt removal of any embedded stingers D) Administer epinephrine by auto-injection Answer: C Diff: 2 Page Ref: 759-760 Objective: 24-11 60) You are transporting a 44-year-old male patient who was stung multiple times by fire ants. Which of these statements made by the patient should concern you most? A) "I am beginning to feel itchy all over and I'm developing hives." B) "The bites are really beginning to hurt more." C) "I took some Motrin this morning for a headache." D) "My hand seems to be more swollen now." Answer: A Diff: 2 Page Ref: 759 Objective: 24-11

17 Copyright © 2018 Pearson Education, Inc.

61) You have been summoned to a beach for a young male patient who has been stung by jellyfish on both forearms. As the senior EMT on scene, which directions should you provide to the other EMS providers in providing proper care to this patient? A) "Make sure to keep the arms below the heart." B) "Let's put a constricting band on his wrists." C) "We need to rinse the sting sites with peroxide." D) "Let's soak the arms in hot water." Answer: D Diff: 2 Page Ref: 760 Objective: 24-12 62) You have been summoned to a field where a 32-year-old male patient has been struck by lightning. Emergency Medical Responders are with the patient, providing manual spine motion restriction. Once you arrive at the patient's side, what should you do first? A) Apply a cervical collar B) Perform a secondary assessment C) Assess the airway and breathing D) Place the patient on the stretcher Answer: C Diff: 1 Page Ref: 763 Objective: 24-14 63) An intoxicated 24-year-old female patient has been struck by lightning and is conscious but confused. Which of these assessment findings should the EMT investigate further first? A) Burns to both arms B) Irregular heartbeat C) Deformity to the left wrist D) Unequal pupils Answer: B Diff: 3 Page Ref: 763 Objective: 24-14 64) As an EMT working at a mountain resort, you are presented with a 50-year-old female who states that she does not feel well. From your assessment, which of these findings would seemingly indicate that the patient is being adversely affected by the high altitude? A) Vomiting after eating large meals B) Elevated blood glucose with no history of diabetes C) Heart rate of 62 beats/min D) Complaint of dyspnea on exertion Answer: D Diff: 3 Page Ref: 763 Objective: 24-15

18 Copyright © 2018 Pearson Education, Inc.

65) A patient at a mountain ski resort presents to you with notable shortness of breath. She denies any past medical history and takes no medications. Your assessment reveals crackles to the bases of both lungs. After applying oxygen, you realize that the key to helping this patient improve is to: A) Remove her to a lower altitude B) Administer albuterol by a metered-dose inhaler C) Position her in a left lateral recumbent position D) Place her in a supine position with legs elevated Answer: A Diff: 3 Page Ref: 764 Objective: 24-15 66) You are accompanying a team of hikers climbing a high mountain. In the morning, you are summoned to a tent and find one of the climbers confused and complaining of a headache. His airway is patent and respirations adequate. He has no medical history and was in good health until found this morning by his friend. You are suspicious of high-altitude cerebral edema (HACE). What should you do to help the patient? A) Provide spine motion restriction procedures B) Provide supplemental oxygen C) Insert an oropharyngeal airway D) Administer oral glucose Answer: B Diff: 2 Page Ref: 764 Objective: 24-15 67) The primary center for controlling the temperature in the human body is located in the: A) Brainstem B) Skin C) Hypothalamus D) Pons Answer: C Diff: 1 Page Ref: 729 Objective: 24-2 68) An 18-month-old child has a temperature of 105.2°F. Without treatment, what will most likely occur? A) Fluid overload B) Cellular dysfunction or seizures C) Hypertension D) Drying of the skin Answer: B Diff: 2 Page Ref: 752 Objective: 24-7

19 Copyright © 2018 Pearson Education, Inc.

69) In which of these settings will sweating be less effective as a means to cool the body? A) Relative humidity of 98% B) Outdoor temperature of 92°F C) Internal temperature of 102°F D) Indoor humidity of 68% Answer: A Diff: 2 Page Ref: 749 Objective: 24-2 70) Consumption of large amounts of water during prolonged periods of exertion can result in which disorder? A) Over-hydration dementia B) Heat stroke C) Hydration D) Hyponatremia Answer: D Diff: 2 Page Ref: 753 Objective: 24-9 71) You are first to arrive at the side of a pond where a boy playing hockey has fallen through the ice. The child is holding onto the ice at the edge of the break and has a distressed look on his face. Your immediate action would be to: A) Carefully slide on your abdomen out to the patient B) Instruct the patient to remain still until the rescue team arrives C) Tell the patient to keep moving to keep warm D) Place tarps on the ice and carefully walk out to the patient Answer: B Diff: 2 Page Ref: 738-739 Objective: 24-6 72) The EMT would recognize that a hypothermic patient's condition is deteriorating when he observes: A) Slurred speech B) Vigorous shivering C) Red-colored skin D) Painful extremities Answer: A Diff: 2 Page Ref: 746, Figure 24-13a Objective: 24-6

20 Copyright © 2018 Pearson Education, Inc.

73) After ensuring that there are no life-threatening conditions to the airway, breathing, and circulation, the priority in caring for a patient with hypothermia is: A) Providing passive rewarming B) Assessing for complications C) Preventing additional heat loss D) Starting active rewarming measures Answer: C Diff: 2 Page Ref: 741 Objective: 24-6 74) A patient with hypothermia is alert and confused, but complains of left arm pain. His friends found him outside, where the patient spent the night after he got drunk and passed out in the cold. It appears as though he has a localized freezing cold injury to the arm in question. In caring for this patient, which of these treatments would be acceptable? A) Drinking warm water to raise his internal body temperature B) Ambulation to get warm blood flowing to all areas of his body C) Placement in a hot shower to expedite the rewarming process D) Board splint the arm to keep it from moving Answer: D Diff: 3 Page Ref: 744 Objective: 24-5 75) You are caring for a patient with an extensive freezing cold injury to his right hand and arm. Which of these instructions would be appropriate to give to your EMT partner? A) "Why don't we get that ring off his right ring finger." B) "We need to move the right arm to promote blood flow into it." C) "I need you to apply hot packs to the right hand and arm." D) "Will you please carefully cut off the frozen clothing on his arm?" Answer: A Diff: 3 Page Ref: 744 Objective: 24-5 76) Which of these statements made by your EMT partner would require immediate correction when treating a patient with hypothermia and a deep freezing cold injury to his left foot, ankle, and lower leg? A) "Why don't we rewarm his lower left leg as quickly as possible?" B) "It is important that we keep the left leg elevated." C) "We will need to gradually rewarm the patient at a rate of 2-3°F per hour." D) "Since his SpO2 is 98%, we may not need to administer any oxygen." Answer: C Diff: 3 Page Ref: 744 Objective: 24-5

21 Copyright © 2018 Pearson Education, Inc.

77) You are reassessing a patient whom you are treating for heat exhaustion. Which of these findings indicates that the patient is improving? A) The skin is now hot and dry B) The mental status is now alert and oriented C) The patient now exhibits shivering D) The core temperature has increased by only 0.5°F Answer: B Diff: 3 Page Ref: 750-751 Objective: 24-8 78) When treating a confused patient who has been exposed to high temperatures, which of these findings is most concerning? A) SpO2 of 94% B) Bounding radial pulse C) Cramping in arms, legs, and abdomen D) Hot and dry skin Answer: D Diff: 2 Page Ref: 752 Objective: 24-8 79) A patient is confused and combative after playing football in the open sun on an extremely hot day. On the field, you find his airway open, respirations tachypneic but adequate, and radial pulse rapid. The patient's skin is hot and dry. After deciding whether to provide spine motion restriction precautions, your next action would be to: A) Provide the patient with cold water for rehydration B) Apply cool compresses to the patient's arms and legs C) Move the patient to the air-conditioned ambulance D) Ascertain whether the patient has any medical problems Answer: C Diff: 2 Page Ref: 752 Objective: 24-8 80) Regarding snakebites, which of these statements is true? A) More than half of all snakebites involve intoxicated males B) The majority of snakebites involve nonpoisonous snakes C) More than 75% of all poisonous snakebites involve the injection of venom D) The majority of snakebites result in death of the patient Answer: B Diff: 1 Page Ref: 755 Objective: 24-10

22 Copyright © 2018 Pearson Education, Inc.

81) A patient has been bitten by a nonpoisonous grass snake. Your care would include: A) Cleaning and dressing the wound B) Administering epinephrine C) Applying a constricting band D) Emergency transport to the hospital with lights and sirens Answer: A Diff: 1 Page Ref: 755 Objective: 24-11 82) A 43-year-old male patient has removed a tick from his arm prior to your arrival. He is refusing all care at this time. Which of these statements made by the EMT would be most appropriate? A) "Tick bites are often poisonous, not becoming obvious for several days." B) "All tick bites carry the potential for Rocky Mountain fever and Lyme disease." C) "If the tick has been embedded for more than an hour, you require emergency treatment." D) "Tick bites can result in bacterial infections, which can be very serious." Answer: D Diff: 2 Page Ref: 758 Objective: 24-11 83) Medical direction has asked you to remove a tick from behind the left ear of a 5-year-old girl. You would: A) Grab the tick close to its head and pull directly outward B) Grasp the tick with tweezers and gently twist it from the patient's skin C) Scrape it from the patient's skin using a credit card or other similar object D) Refuse to remove the tick given its location and the age of the patient Answer: A Diff: 2 Page Ref: 758 Objective: 24-11 84) The stinger of a bee is evident in the left thigh of a female patient who states that she is allergic to bees and has an EpiPen for such occasions. According to the patient, the sting occurred two hours ago and she just now noticed that the stinger remains in place. She did not take her epinephrine and appears asymptomatic for an allergic reaction. At this time, you would: A) Administer the epinephrine B) Attempt removal of the stinger C) Apply a warm pack to the sting site D) Attempt identification of the type of bee Answer: B Diff: 2 Page Ref: 759 Objective: 24-11

23 Copyright © 2018 Pearson Education, Inc.

85) A 23-year-old female patient has been stung in the right arm by some unidentified marine life. Her arm and hand are swollen and red. The primary assessment is unremarkable and she is complaining of pain to the site of injury. You would: A) Apply cold packs to the affected area B) Place a constricting band to the right wrist C) Hold the right arm above the patient's head D) Remove rings from the patient's right hand Answer: D Diff: 2 Page Ref: 760 Objective: 24-12 86) In addition to brain injury from the high voltage and amperage, the patient who experiences a lightning strike also typically develops: A) Solid-organ rupture B) Severe internal burns C) Trauma from air rushing out of and back in to the strike location D) Surface burns from holding super-heated metal objects Answer: C Diff: 2 Page Ref: 761 Objective: 24-13 87) Which of these conditions, in addition to cool temperatures, is necessary for trench foot to develop? A) Prolonged exposure to moisture B) Direct skin contact with leather or wool C) Physical exertion D) Malnourishment Answer: A Diff: 2 Page Ref: 736-737 Objective: 24-4 88) You are called to treat a homeless person who complains of an inability to walk due to sores and numbness of his feet. The EMR tells you that the patient is well oriented with good pulses and respiration but that his feet are edematous with open sores and broken-down skin. This finding is the typical result of which condition? A) Malnutrition B) Hypothermia C) Frostbite D) Immersion foot Answer: D Diff: 2 Page Ref: 736-737 Objective: 24-5

24 Copyright © 2018 Pearson Education, Inc.

89) You are managing a patient who has been consuming large amounts of water while playing a sport outside for several hours. The patient presents with general malaise, has a severe headache, has vomited once, and complains of photophobia. The patient's core temperature is just slightly elevated, and you see bloating in his hands. During transport, in which position should this patient be placed? A) Fowler's position B) Supine position C) Prone position D) Trendelenburg position Answer: A Diff: 3 Page Ref: 755 Objective: 24-9 90) Following a direct lightning strike, which symptoms should the EMT expect the patient to display? A) Apnea B) An extremely tachycardic pulse C) Ventricular tachycardia D) Hypertension Answer: A Diff: 2 Page Ref: 761-762 Objective: 24-13 91) During the summer months, you work as an EMT at a first aid station that helps people who hike trails in the high mountains of the western United States. One afternoon, you are presented with a 13-year-old male patient who presents as alert and oriented, but complains of chest pain, dyspnea, and a headache. He is tachypneic and tachycardic and has slight inspiratory crackles. Given this presentation, what would be the most likely differential diagnosis? A) Congenital heart disease B) Fall while hiking causing a mild traumatic brain injury C) Altitude-induced cerebral edema D) High-altitude pulmonary edema Answer: D Diff: 2 Page Ref: 764 Objective: 24-16

25 Copyright © 2018 Pearson Education, Inc.

92) During the summer months, you work as an EMT at a first aid station that helps people who hike trails in the high mountains of the western United States. One afternoon, you are presented with a 42-year-old male patient who was just at an altitude of 14,000 feet while hiking. The patient states that while at this altitude he experienced a severe headache, became very uncoordinated and stumbled often, and felt very nauseous. He became scared, so he left the trail to be seen in the first aid station. The patient indicates that he feels better now. Given this description, what would be the most likely differential diagnosis? A) High-altitude pulmonary edema B) High-altitude cerebral edema C) High-altitude coronary thrombosis D) High-altitude neurologic thrombosis Answer: B Diff: 2 Page Ref: 764 Objective: 24-16

26 Copyright © 2018 Pearson Education, Inc.

Prehospital Emergency Care, 11e (Mistovich et al.) Chapter 25 Submersion Incidents: Drowning and Diving Emergencies 1) You have been asked to present information on water safety and drowning to a community civic group. In regard to prevention, which point would you emphasize? A) The vast majority of children who drown do so in public swimming pools B) A significant number of drownings are preventable due to the involvement of alcohol C) Individuals with seizure disorders are safe in the water if they wear personal flotation devices D) Most authorities agree that children younger than 3 years should be taught to swim Answer: B Diff: 2 Page Ref: 770 Objective: 25-2 2) Which statement by the EMT indicates proper application of the term drowning? A) "A drowning occurs when a person dies while in water, regardless of the cause." B) "A drowning occurs anytime a person is submerged in water and dies as a result." C) "A drowning occurs anytime a person is submerged in a liquid and suffers some sort of respiratory impairment." D) "A drowning occurs anytime a person is submerged in a liquid and dies; also, the incident must be deemed to have been preventable." Answer: C Diff: 2 Page Ref: 769 Objective: 25-1 3) Which one of these statements about drowning is true? A) Actual drownings are responsible for only a small number of water-related deaths B) Children younger than 1 year most commonly drown in swimming pools C) Fences around swimming pools are the most effective means of preventing a child from drowning D) Personal flotation devices afford little protection to an individual if he or she panics while in the water Answer: A Diff: 3 Page Ref: 769 Objective: 25-7 4) Friends pulled an unresponsive 15-year-old male from a lake. They were swimming when they lost track of him. Assessment reveals labored breathing and a weak pulse. Which question would give the EMT the most information related to the prognosis of the patient? A) "Does he have a history of breathing problems?" B) "How long do you think he under the surface of the water?" C) "Was he drinking or doing drugs?" D) "Is there any chance he hit his head?" Answer: B Diff: 2 Page Ref: 770 Objective: 25-4

1 Copyright © 2018 Pearson Education, Inc.

5) A 49-year-old man was fishing in a stream of cold water when he became submerged after getting tangled in some hidden roots. The patient was pulled from the stream by friends after approximately 5 minutes of submersion. Your assessment reveals that he is responsive to painful stimuli and breathing 5 times per minute. His pulse is weak and rapid. Which set of instructions would be appropriate to relay to the team of rescuers? A) "Do not wipe the water off him. It will keep his body cool and protect vital organs like his brain and heart." B) "Do not apply the AED if he goes into cardiac arrest. The water was very cold and the AED is contraindicated in hypothermia." C) "I need someone to perform abdominal thrusts to see if he took water into his stomach." D) "Let's move him into the warm ambulance as quickly as possible, and then cut off his clothes and cover him." Answer: D Diff: 2 Page Ref: 779, Figure 25-6 Objective: 25-8 6) Which rescuer would be the most appropriate choice to rescue an injured man clinging to an overturned boat in a shallow lake? A) A seasoned EMT who is a good swimmer, is wearing a personal flotation device, and has a rope around his waist B) A firefighter who is wearing a personal flotation device, does not have any medical training, and is a good swimmer who is trained in water rescue C) An EMT who is a good swimmer and has a personal flotation device in place D) A firefighter who is also an EMT and worked as a lifeguard before becoming a firefighter Answer: B Diff: 2 Page Ref: 772 Objective: 25-7 7) You and several other EMTs are on location where a young female lost control of her car and ended up in a shallow lake. The car is approximately 20 feet from shore and upright, with the wheels touching the bottom of the lake. The woman is on the car's roof; she states that she is not hurt but is scared. What should the EMT do to rescue the patient? A) Swim out to retrieve the patient B) Toss a rope in a throw bag to the patient C) Extend a ladder across the water to the car and the patient D) Encourage the patient to swim to shore Answer: B Diff: 2 Page Ref: 773 Objective: 25-7

2 Copyright © 2018 Pearson Education, Inc.

8) Torrential rains have been falling over the past few days and your service area is experiencing major flooding. While returning to the disaster command post in the ambulance, you are flagged down by a group of people who inform you that a woman is clinging to a tree in the middle of a rapidly moving and swollen stream. She is screaming for help and states that she does not know how much longer she can hang on. What should you do immediately? A) Throw a rope toward the patient B) Call for a water rescue team and boat C) Swim out and retrieve the patient D) Toss a life preserver upstream from the patient, in the vicinity of her location Answer: B Diff: 2 Page Ref: 773 Objective: 25-7 9) The EMT is correctly using a rope in a throw bag when he: A) Tosses the throw bag using an underhand motion toward the patient B) Throws the rope overhand and slightly beyond the patient's location C) Goes downstream and floats the throw bag toward the patient D) Ties the throw bag to a stationary object and throws the rope to the patient Answer: A Diff: 1 Page Ref: 773 Objective: 25-7 10) You have been dispatched to a pool party for a 19-year-old male with shoulder pain. He states that he dove off the diving board and hit the bottom of the pool with his right shoulder and back. Although there is alcohol at the party, he denies drinking. The patient is alert and oriented and has obvious redness and abrasions to his right shoulder, neck, and back. He complains of neck pain and tingling in his right arm, but denies any other discomfort to any other part of the body. Your next action would be to: A) Measure and apply a cervical collar B) Establish manual spine motion restriction C) Provide supplemental oxygen D) Perform the secondary assessment Answer: B Diff: 2 Page Ref: 773 Objective: 26-8 11) The EMT shows she understands the concept of the mammalian diving reflex by stating: A) "It is a protective mechanism that occurs when the spinal column is compressed, such as would occur in a diving accident." B) "It occurs in warm water and serves to increase blood flow to the vital organs." C) "It may provide a benefit to the submerged patient by diverting oxygen-rich blood to the brain and heart." D) "It occurs in cold water and increases the activity of the vital organs, thereby protecting them." Answer: C Diff: 3 Page Ref: 774 Objective: 25-5 3 Copyright © 2018 Pearson Education, Inc.

12) The head of the water rescue team has just informed you that the divers have retrieved a young woman who fell through ice on a pond while walking her dog. The estimated submersion time is 22 minutes. Another EMT asks you if it will be necessary to start resuscitation, given the length of submersion. What is your most appropriate reply? A) "Let's assess her first. If she does not have a pulse, then we will contact medical direction for orders to not resuscitate." B) "Our resuscitation will consist of CPR only. The AED is contraindicated if the patient is hypothermic." C) "Probably not. Given the amount of time she was submerged in the water, there is most likely little that can be done." D) "Absolutely! Given that she has been in cold water, she may still have a chance of survival." Answer: D Diff: 3 Page Ref: 774 Objective: 25-8 13) Which patient has the best chance for an optimal outcome after drowning? A) A 34-year-old female who was submerged for 9 minutes in 39°F water B) A 47-year-old male who was submerged for 8 minutes in a heated swimming pool C) A 22-year-old female who was submerged for 10 minutes in a whirlpool with 90°F water D) A 17-year-old female who was submerged for 10 to 12 minutes in warm salt water Answer: A Diff: 2 Page Ref: 774 Objective: 25-4 14) When working the scene of a water rescue, anytime that the EMT is within 10 feet of the water's edge, it is essential that she don which equipment for personal safety? A) Rope secured to the waist B) Examination gloves C) Personal flotation devices D) SCUBA equipment Answer: C Diff: 1 Page Ref: 772 Objective: 25-7 15) You are by the side of a young male patient who hit his head after diving from a hillside into a shallow lake. The patient is unresponsive and has a large hematoma to his forehead with abrasions all over his face. A paramedic asks you to apply painful stimuli to his arms and legs. You recognize that the paramedic wants you to assess for which type of injury or condition? A) Alcohol intoxication B) True unresponsiveness C) Extremity fractures D) Spinal injury Answer: D Diff: 2 Page Ref: 773 Objective: 25-3

4 Copyright © 2018 Pearson Education, Inc.

16) You have been called to a hotel swimming pool for a confused adult male. Patrons state that the man was drinking most of the afternoon and was in and out of the pool. The man is now very confused and noncompliant with your requests. The patrons moved the patient so that he is lying on the ground, away from the water's edge, when you arrive. His airway is open and his breathing is adequate. His radial pulse is strong, and his skin is wet with water from the pool. While conducting the secondary assessment, you note a large hematoma to the back of his head. What should you do next? A) Perform manual spine motion restriction of his head and neck B) Apply a cold pack to the hematoma and continue the assessment C) Check the patient's pupils and hand grips for equality D) Transfer the patient to the stretcher and provide immediate transport Answer: A Diff: 2 Page Ref: 775 Objective: 25-5 17) A teenage female has been removed from a pool. Your assessment reveals her to be responsive to painful stimuli with decorticate posturing. Her airway has water in it, and she appears to be vomiting water. Breathing is agonal at 8 breaths per minute; you hear a gurgling noise with each attempt at breathing. The patient's pulse is weak and slow. What should be the priority treatment you would render next? A) Place a nasopharyngeal airway B) Administer abdominal thrusts C) Start positive pressure ventilation D) Suction water from the airway Answer: D Diff: 2 Page Ref: 774 Objective: 25-8 18) A child has been pulled from the pool after being submerged for less than 30 seconds. The child is crying and hyperventilating, as well as coughing up water and vomiting. The EMT should: A) Place the child prone to help evacuate water from the lungs B) Administer abdominal thrusts with the child in a supine position C) Place the child on his side and initiate a primary assessment D) Provide supplemental oxygen Answer: C Diff: 2 Page Ref: 777 Objective: 25-8

5 Copyright © 2018 Pearson Education, Inc.

19) A 24-year-old intoxicated female patient was pulled from a pool by hotel personnel. The primary assessment reveals her airway to be open and her breathing adequate. Her radial pulse is strong but fast, and she is persistently coughing. Vital signs are pulse 144 beats/min, respirations 24 breaths/min, blood pressure 166/100 mmHg, and SpO2 92%. What would be considered appropriate care for this patient? A) Abdominal thrusts to remove water from the lungs B) Supplemental oxygen through a nonrebreather mask C) Deep oral suctioning to remove water from the airway D) Precautionary application of the automated external defibrillator Answer: B Diff: 2 Page Ref: 776 Objective: 25-8 20) A patient who was found face-down in a pond is unresponsive, apneic, and pulseless. Friends state that the patient was hot and wanted to take a swim in the pond. Which intervention should be included in your care of this patient? A) Hyperventilation with a bag-valve mask and high-concentration oxygen B) Positive pressure ventilations at 10 ventilations per minute with supplemental oxygen C) Placement of the patient in a prone position to evacuate water from the lungs D) Application of the AED with the delivery of no more than one shock Answer: B Diff: 2 Page Ref: 775 Objective: 25-8 21) While at a public pool obtaining a refusal of care from a patient who was stung by a bee, you hear screaming. Patrons are waving you over to a 13-year-old boy who hit the diving board with his head while diving. He is now in the water and is motionless. The lifeguard is in the water and informs you that the patient's eyes are open and he is breathing with a pulse, but not talking. What should you do next? A) Instruct the lifeguard to carefully float the boy to the side of the pool so you can assess and get him on a backboard B) Have the lifeguard quickly remove the patient from the pool so assessment and emergency medical care can be given C) Position the boy upright against the lifeguard with spine motion restriction precautions observed during removal from the pool D) Pass a cervical collar and long board into the water for providing spine motion restriction precautions to the patient prior to his extrication from the water Answer: D Diff: 2 Page Ref: 775 Objective: 25-6

6 Copyright © 2018 Pearson Education, Inc.

22) You are assessing a young child who was briefly submerged after falling approximately 20 feet from an observation platform into the shallow portion of a lake. Family members jumped into the lake immediately and pulled the girl to safety. She is confused and begins to vomit profuse amounts of water. What should you do immediately? A) Suction using a rigid-tip catheter, while maintaining manual spine motion restriction precautions B) Place the patient in a side-lying position and tell her to "vomit all the water out" C) Check the patient's breath sounds with a stethoscope to determine whether she aspirated any water D) Perform the head-tilt, chin-lift maneuver and use the flexible catheter to suction out the water while maintaining spine motion restriction precautions Answer: A Diff: 2 Page Ref: 775 Objective: 25-8 23) A patient is in cardiac arrest after being submerged in a pond for 15 minutes. You note that his abdomen is greatly distended and it is very difficult to ventilate him. What should you do next? A) Ventilate more forcibly at a rate of 20-24 breaths/min B) Administer chest compressions to evacuate water from the body C) Place the patient on his side and firmly press on the abdomen D) Administer five abdominal thrusts Answer: C Diff: 3 Page Ref: 777 Objective: 25-8 24) A patient has been pulled from a pool in which he was submerged for several minutes. He is pulseless and apneic. An Emergency Medical Responder asks you if you want to press on the man's abdomen to remove the water from his stomach. What is the most appropriate reply? A) "That sounds good. We need to get that water out of him to minimize the risk for an infection." B) "Let's hold off on that until we see how well we can ventilate the patient." C) "That is no longer recommended in the care of the submersion patient." D) "Yes, but let's turn the patient onto his front so the water can drain naturally." Answer: B Diff: 3 Page Ref: 777 Objective: 25-8

7 Copyright © 2018 Pearson Education, Inc.

25) A 22-year-old male was fishing when he was submerged for approximately 30 seconds after he moved into deeper water to untangle his fishing line. He states that he did take some water into his lungs but feels better now and does not see the need to be transported to the hospital. Which response is most appropriate? A) "We will wait here with you for another 20 minutes. If you do not cough, then you should be okay, and I will feel better about letting you refuse transport." B) "Let me listen to your breath sounds again. If they are clear, then you should be okay to refuse additional care and transport." C) "You seem okay now, but it may take a day or two for complications to become apparent. You really should be checked out in the hospital." D) "Your pulse oximetry reading is 97% on room air, which indicates that you are breathing adequately. You should be okay to refuse care, but keep an eye on yourself." Answer: C Diff: 3 Page Ref: 777 Objective: 25-8 26) You are by the side of a male patient who participated in deep-sea diving earlier in the day. The 32-year-old patient informs you that he thinks he pulled a muscle in his shoulder, which has been hurting more as the day goes on. When questioned, he admits to having a headache and blurred vision. You are suspicious of decompression sickness. Which statement made by the patient would be most significant given the situation? A) "I spent some time in a pocket of warm water." B) "I ate seafood the night before." C) "While diving, I saw a shark and came up as fast as I could." D) "I took Tylenol before the dive for a headache." Answer: C Diff: 3 Page Ref: 780 Objective: 25-10 27) You have been called to a local emergency department to transfer a male patient to a specialty facility for further care. The man was involved in a deep-sea dive emergency and has been diagnosed with barotrauma. Which finding should be expected with this condition? A) Pain and deafness to the ears B) Right upper quadrant pain and vomiting C) Itchy, blotchy, or mottled skin D) Weakness or paralysis to the extremities Answer: A Diff: 3 Page Ref: 782 Objective: 25-10

8 Copyright © 2018 Pearson Education, Inc.

28) A confused and lethargic 22-year-old male patient with no medical history came up from a deep-sea dive and is now complaining of shortness of breath and sharp pain, which he localizes to the right side of his chest. He denies nausea, vomiting, or dizziness. You have applied oxygen and are getting ready for transport. How will you transport this patient on the stretcher? A) Lateral recumbent position B) Semi-Fowler's position C) Supine position D) Trendelenburg position Answer: A Diff: 3 Page Ref: 782 Objective: 25-11 29) You have been put in charge of creating a protocol that will direct care provided by EMS. In regard to an ice-water drowning patient who is severely hypothermic and in cardiac arrest, what treatment would you propose? A) Provide CPR only and withhold the AED B) Withhold resuscitation if the patient's temperature is lower than 86°F C) Apply the AED and give one shock if indicated. Contact medical direction if additional shocks are indicated by the AED D) Administer the usual care provided to any other patient in cardiac arrest Answer: C Diff: 2 Page Ref: 772 Objective: 25-8 30) A young male has been pulled from a lake by his friends after being submerged for several minutes. Assessment reveals his airway to be patent, respirations absent, and a weak carotid pulse of 12 beats/min. Which care should you provide at this time? A) Attach the AED, provide one shock if indicated, start CPR, and transport to the hospital B) Administer one shock with the AED, start CPR, and withhold additional shocks until the patient is rewarmed C) Insert an oropharyngeal airway, provide positive pressure ventilation, start CPR, and transport D) Start positive pressure ventilation, place the patient on a long board, and transport to the hospital Answer: D Diff: 2 Page Ref: 775 Objective: 25-8 31) A fisherman fell through the ice and was submerged in a lake for approximately 4 minutes before being pulled out by friends. You find the man to be unresponsive, not breathing, and without a pulse. What should you do immediately? A) Start cardiopulmonary resuscitation B) Apply the automated external defibrillator C) Provide spine motion restriction precautions D) Determine the patient's core temperature Answer: A Diff: 1 Page Ref: 777 Objective: 25-8 9 Copyright © 2018 Pearson Education, Inc.

32) A 23-year-old diver has been brought to the surface by fellow divers after behaving erratically while in the water. Your assessment reveals him to be confused and uncoordinated, with an open airway and adequate breathing. He has a strong radial pulse and SpO2 of 98%. The patient has diabetes, and your partner reports that his blood sugar is 91 mg/dL. The remainder of your assessment is unremarkable. Based on this presentation and history, what would be the most likely cause of his condition? A) Hypoglycemia B) Increased nitrogen levels in the blood C) Hypoxia and hypoglycemia D) Seizure activity Answer: B Diff: 3 Page Ref: 780 Objective: 25-10 33) A diver has summoned 911 after he came to the surface and became short of breath and "itchy" all over. Additionally, he complains of a headache. Assessment reveals a patent airway, adequate breathing, clear breath sounds, and the following vital signs: pulse 92 beats/min, respirations 20 breaths/min, blood pressure 156/62 mmHg, and SpO2 96%. Proper care of this patient should include: A) High-concentration oxygen and epinephrine via EpiPen B) High-concentration oxygen through a nonrebreather face mask C) Epinephrine through an EpiPen, and immediate transport to the hospital D) Oxygen, epinephrine through an EpiPen, and a bronchodilator from an MDI Answer: B Diff: 2 Page Ref: 782 Objective: 25-11 34) A 57-year-old diver has been brought to the surface by fellow divers. He complains of difficulty breathing, dizziness, and severe chest pain. He states that he has a cardiac history and takes nitroglycerin, which is in his belongings nearby. Emergency Medical Responders report the following vital signs: pulse 128 beats/min, respirations 18 breaths/min, blood pressure 88/56 mmHg, and SpO2 93%. After applying supplemental oxygen to the patient, your next action should be to: A) Perform a secondary assessment B) Assist the patient in taking his nitroglycerin C) Administer a bronchodilator through a metered-dose inhaler D) Transport the patient in a head-down position (Trendelenburg) Answer: A Diff: 2 Page Ref: 774-775 Objective: 25-11

10 Copyright © 2018 Pearson Education, Inc.

35) An alert and oriented 29-year-old male diver surfaced and called 911 for severe right ear pain. When asked, he specifically denies nausea, dizziness, or pain other than in his ear. Assessment reveals no threats to the airway, breathing, or circulation. The ear shows no obvious injury or discharge. His vital signs are pulse 96 beats/min, respirations 16 breaths/min, blood pressure 146/76 mmHg, and SpO2 97%. What is the proper care for this patient? A) Transport to a recompression chamber 45 minutes away B) Provide spine motion restriction precautions prior to transport C) Transport to the emergency department 15 minutes away D) Pack gauze in the ear canal to decrease outside pressure changes and reduce pain Answer: C Diff: 2 Page Ref: 782 Objective: 25-10 36) At a pool party, a 48-year-old female was pulled from the water after being submerged for 10 seconds according to bystanders. The patient responds to verbal stimuli by moaning and has an intact airway with adequate respirations. Her heart rate is 116 beats/min and SpO2 is 97%. Which statement made by friends would be considered most important at this time? A) "She is scheduled for knee surgery next week." B) "She is allergic to many medications." C) "She is not a very good swimmer." D) "She takes insulin for her sugar condition." Answer: D Diff: 3 Page Ref: 775 Objective: 25-10 37) As a diver rises too quickly from a deep-sea dive, the nitrogen in his bloods forms everenlarging bubbles. The bubbles then cause damage to internal organs and tissues. The EMT would recognize this outcome as demonstrating: A) Boyle's law B) Dalton's law C) Charles's law D) Henry's law Answer: A Diff: 2 Page Ref: 780 Objective: 25-9

11 Copyright © 2018 Pearson Education, Inc.

Prehospital Emergency Care, 11e (Mistovich et al.) Chapter 26 Psychiatric Emergencies 1) Which patient would the EMT classify as having a psychiatric (behavioral) emergency? A) A 44-year-old female with bipolar disorder who has a fever of 102.5°F B) A 56-year-old male with schizophrenia who has stopped taking his medications C) A 29-year-old male with diabetes who is suddenly confused D) A 36-year-old male with no known history who is hearing voices Answer: D Diff: 2 Page Ref: 787 Objective: 26-1 2) You have been called to a residence for a complaint of chest pain. On scene, you are met by the patient's husband, who informs you that his wife suffers from anxiety attacks and that she received bad news tonight concerning her mother. This news increased her anxiety, and now she just needs to calm down. According to the husband, her doctor said the chest pain the patient gets is just a temporary response to the anxiety. Assessment of the 46-year-old woman reveals her to be extremely anxious and complaining of chest pain. In treating this patient, the EMT should: A) Attach the AED B) Administer nitroglycerin C) Transport the patient to the local cardiac facility D) Check vital signs Answer: D Diff: 2 Page Ref: 788 Objective: 26-7 3) You are by the side of a young male patient with a history of behavioral problems. Friends called 911 after the patient experienced several violent outbursts this evening. Which finding would cause you to suspect a physical cause for this behavior, as opposed to a mental one? A) Dilated pupils that are sluggish to react B) Difficulty making eye contact with the EMT C) Hearing his mother's voice in his head D) Fear that friends are out to steal his money Answer: A Diff: 3 Page Ref: 788 Objective: 26-2

1 Copyright © 2018 Pearson Education, Inc.

4) Bystanders called 911 for a female patient who was screaming obscenities in a store parking lot. On arrival, you find the patient to be sitting in the middle of the parking lot muttering incomprehensible phrases. After conducting the primary assessment and finding no immediate life-threatening conditions, you move her to the ambulance. Additional assessment findings include abrasions to her arm and urinary and fecal incontinence. She also has a bottle of Prozac in her pocket. You reference Prozac in your medication book and find it to be used to treat depression. With this information, what will be a component of how you should treat and/or transport this patient? A) Assist her in taking one dose of Prozac B) Arrange for transport to the halfway home where she lives C) Transport the patient on her left side D) Transport the patient to the inpatient psychiatric hospital where she has previously been treated Answer: C Diff: 2 Page Ref: 802, Figure 26-3A Objective: 26-7 5) According to family members, a 24-year-old female suddenly began to act in a hostile manner. After assessing her, you determine that the patient cannot remember her name or present location and is drooling. Based on these findings, you should suspect a: A) Psychological emergency caused by alcohol consumption B) Behavioral emergency without physical cause C) Psychological condition related to a traumatic experience D) Behavioral emergency resulting from a physical condition Answer: D Diff: 3 Page Ref: 788 Objective: 26-2 6) What is your first priority in managing a patient with a psychiatric (behavioral) emergency? A) Determining whether the patient is a threat to himself B) Evaluating for a preexisting medical condition C) Ensuring the safety of you and your partner D) Assessing and managing the patient's airway Answer: C Diff: 2 Page Ref: 798 Objective: 26-4

2 Copyright © 2018 Pearson Education, Inc.

7) A patient states that she cannot leave the house to go to the hospital "because I am scared I will do something idiotic and lose it and then everyone will look at me and laugh." She states she rarely leaves the house and the last time she did, she began to hyperventilate. This caused her to return home immediately. Based on this information, the EMT should recognize the patient as having: A) Anxiety B) Agoraphobia C) Schizophrenia D) Bipolar disorder Answer: B Diff: 2 Page Ref: 790 Objective: 26-3 8) A woman has called you to help her middle-aged friend, who has stopped eating and talking and has frequent crying spells. Your history reveals that the woman lost her husband a few months ago, and has been withdrawn and despondent ever since. Throughout the assessment, the alert and oriented woman continually states that she wants to be left alone. When you bring the stretcher into the room, the woman states, "I do not want to go to the hospital. Let me sign whatever I need to sign." What is your best response to this situation? A) Contact medical direction and ask for advice B) Recheck mental status and orientation, and then have her sign a refusal-of-care form C) Allow the patient to sign a refusal-of-care form as long as her friend stays with her D) Contact law enforcement to restrain the patient for transport Answer: A Diff: 2 Page Ref: 805 Objective: 26-9 9) You are responding to a patient with a psychiatric problem. While you are en route, dispatch contacts you and states that a mental health provider is present at the scene and you will be transporting the patient to the local psychiatric hospital for treatment of her bipolar disorder. At the scene, which behavior should help confirm that the patient has bipolar disorder? A) Belief that family members are out to get her B) Statements by the patient that she is "just fine" C) Threats of violence toward the EMTs D) Happy, excessively cheerful, and elated behavior Answer: D Diff: 2 Page Ref: 790 Objective: 26-3

3 Copyright © 2018 Pearson Education, Inc.

10) You have been dispatched to transfer a patient with schizophrenia from a hospital emergency department to a behavioral health unit at another hospital. While en route, your partner states that he cannot remember what schizophrenia is. You should tell him that schizophrenia is a psychiatric disorder commonly characterized by: A) Multiple personalities B) The need to be around others C) Delusions and hallucinations D) Feelings of worthlessness Answer: C Diff: 1 Page Ref: 791 Objective: 26-1 11) Given the research related to suicide, which patient should the EMT recognize as the highest risk for a suicidal action? A) A 22-year-old male who has just broken up with his girlfriend of six months B) An 18-year-old male who was just arrested for possession of marijuana C) A 72-year-old widow who was just informed of a diagnosis of Alzheimer's disease D) A 34-year-old male who has been in and out of prison and was just convicted of receiving stolen property Answer: C Diff: 2 Page Ref: 790-791 Objective: 26-2 12) After taking a patient to the hospital for an unsuccessful suicide attempt, your partner states that this is the fifth suicide call he has had this month. Four of them have been women who were unsuccessful; the fifth was a man who shot himself in the head and was dead on the scene. How should you respond to your partner? A) "That is odd since more men than women attempt suicide, and men are generally more unsuccessful than woman." B) "That seems to make sense, given that more women than men attempt suicide, but men are generally more successful." C) "That is unusual given that women are generally more successful and tend to use more lethal means such as guns." D) "Studies have shown that there is no difference in the frequency in which men and women attempt or commit suicide." Answer: B Diff: 3 Page Ref: 794 Objective: 26-2

4 Copyright © 2018 Pearson Education, Inc.

13) You are at the scene where a male patient threatened to take pills to end his life. In talking with the patient, he informs you that he just got a new job that requires more of his time and his wife has been giving him a hard time about it. Tonight, he had enough and just made the comment because he was mad at her. He says that although he is still upset, he does not plan to do anything "drastic." Before taking further action, which fact about the patient should raise the EMT's suspicion that he is indeed at a higher risk for suicide? A) New job with increased stress B) Scar on the patient's neck C) New diagnosis of high blood pressure D) Unsuccessful attempt three years ago Answer: D Diff: 2 Page Ref: 794-795 Objective: 26-5 14) You are preparing a 24-year-old male for transport to the hospital after he stated that he wanted to kill himself. The patient's father pulls you aside and tells you that his son repeatedly has been in the hospital for suicidal statements but is then released and comes home and the whole cycle repeats itself. He is worried that one day his son will actually commit suicide and wonders if there is anything he can do to decrease the chance of this happening. In regard to the most common means by which people commit suicide, you should respond by saying: A) "Lock up all of the household cleaning products, since these are commonly ingested poisons during suicide attempts." B) "The chance of your son committing suicide is low since he has never actually attempted it." C) "I would suggest gathering up all guns and weapons and removing them from the house." D) "It would be best to get rid of all the beer, wine, and alcohol in the house." Answer: C Diff: 3 Page Ref: 794 Objective: 26-4 15) You arrive on scene to find that the police have subdued a violent man with a history of psychiatric problems. The police state they will put the patient on the stretcher. Which action should cause the EMT to take immediate and corrective action? A) The patient is placed and restrained in a prone position on the stretcher B) The patient is placed supine with straps over the chest, waist, and thighs C) Police state that the restraints will not be removed until arrival at the hospital D) Police move very quickly to get the patient onto the stretcher Answer: A Diff: 2 Page Ref: 801-802 Objective: 26-6

5 Copyright © 2018 Pearson Education, Inc.

16) You are at a scene where a patient was calm and compliant until a few seconds ago. Now he is brandishing a knife and threatening to kill you and your partner if you do not immediately leave. The man is intoxicated and police have been notified but have not yet arrived. What is your best course of action? A) Immediately physically restrain the patient B) Encourage the man's wife to calm him down C) Attempt to "talk down" the patient D) Remove yourself, your partner, and family from the house Answer: D Diff: 2 Page Ref: 803, Figure 26-3B Objective: 26-4 17) Which one of these patients is the EMT justified in restraining? A) A patient with severe depression who refuses transport B) A patient who states that he wants to kill himself and desires transport to the hospital C) A patient who threatens to kill himself and his wife and is trying to leave the scene D) A patient with unpredictable violent outbursts but who is calm at the present time Answer: C Diff: 1 Page Ref: 801 Objective: 26-8 18) You have had to restrain a patient who is being involuntarily committed to a psychiatric institution. During the restraint process, the patient vomited. How should you position him on the stretcher? A) On his stomach with head elevated B) Left lateral recumbent position C) Prone with head turned to the side D) Supine in a semi-Fowler's position Answer: B Diff: 1 Page Ref: 803 Objective: 26-8 19) An EMT is transporting a restrained patient who is trying to spit on him. Which action would the EMT be justified in doing? A) Placing a surgical mask over the patient's mouth while being careful not to impede his breathing B) Releasing the restraints and forcing the patient out of the ambulance C) Repositioning the patient in a prone position so he cannot spit D) Placing a roll of gauze loosely into the patient's mouth Answer: A Diff: 2 Page Ref: 805 Objective: 26-7

6 Copyright © 2018 Pearson Education, Inc.

20) Which statement made by an EMT shows a proper understanding of what is required when restraining a patient? A) "To decrease the potential legal problems, it is best to have the patient agree to being restrained and have family members, if present, act as witnesses." B) "One person should be assigned the task of explaining everything to the patient before, during, and after he has been restrained." C) "Always come at the patient with more force than you think is needed so the job of restraining can be done quickly and effectively." D) "Securing the patient's arms and legs to the stretcher with handcuffs is a very effective form of restraint, but should be used only if police are present." Answer: B Diff: 2 Page Ref: 803 Objective: 26-8 21) A group of EMTs is preparing to restrain a large and violent patient. Which instruction from the team leader is most appropriate? A) "Two of us will get him from the front and the other two from the back. Let's do this slowly so no one gets hurt." B) "You grab his head and the rest of us will grab his legs and chest, and then let's get him facedown on the stretcher." C) "You distract him by coming at him from the front and the rest of us will come at him from different directions, each taking a separate limb." D) "Since you are trained in martial arts, try to give him a kick to his stomach; then, when he goes down, the rest of us will grab his arms and legs." Answer: C Diff: 2 Page Ref: 803 Objective: 26-8 22) After restraining a violent patient on the stretcher, which action is it essential that the EMT perform as soon as reasonably possible? A) Check the tightness of the chest strap B) Check the extremities for pulses and motor ability C) Inform the patient of what will occur during transport D) Obtain a set of vital signs, excluding temperature Answer: A Diff: 2 Page Ref: 803 Objective: 26-8

7 Copyright © 2018 Pearson Education, Inc.

23) You are transporting a restrained 23-year-old female patient with a past history of sexual abuse and multiple behavioral problems. She is very upset with being restrained as well as with being transported to the hospital. She informs you that she will take you to court for your actions. Consequently, you are concerned about false accusations. What is your best protection should she follow through with legal action? A) Meet with your service's attorney to discuss the call B) Have your partner sign the prehospital care report C) Remove the restraints during transport if she is cooperative D) Thoroughly document everything that happens on the call Answer: D Diff: 2 Page Ref: 805 Objective: 26-10 24) An alert and oriented 18-year-old female patient has threatened to hurt herself, but is refusing transport to the hospital. She is calm and states that she knows her rights and taking her would constitute false imprisonment. Her family is present and wants her to go to the hospital for help, as she has attempted suicide in the past. Given the situation and the patient's threat, what is your best course of action? A) Physically restrain the patient for transport B) Contact law enforcement for assistance C) Have the patient sign a refusal-of-care form that is witnessed by family D) Transport the patient on her father's consent Answer: B Diff: 2 Page Ref: 805 Objective: 23-9 25) While transporting a patient with schizophrenia, he suddenly screams, "Do you see that snake? It is right next to you! Kill it!" How should you respond to this patient? A) "Where do you see the snake? Can you describe its color? Do you see more than one?" B) "There is not a snake in the ambulance. They could not live in here because there is no food." C) "I do not see a snake; I see a stethoscope. I do not doubt that you believe you see a snake, though." D) "What do you mean a snake? This is a new ambulance. I hope that it did not come with a snake!" Answer: C Diff: 2 Page Ref: 798 Objective: 23-6

8 Copyright © 2018 Pearson Education, Inc.

26) You are assessing and treating a young girl who superficially cut her wrists in a self-stated suicide attempt. Bleeding was not significant and has clotted. While transporting the patient, which statement from the EMT is most appropriate? A) "Can you tell me why you tried something stupid like this?" B) "This really has upset your parents. How does that make you feel?" C) "These cuts are not significant. You didn't really want to do this, did you?" D) "I am going to clean these cuts up and then wrap them." Answer: D Diff: 1 Page Ref: 800 Objective: 26-7 27) Which one of the statements shows that the EMT has a good understanding of dealing with a behavioral emergency patient? A) "It is okay to spend an extended amount of time on scene before transporting patients with behavioral problems." B) "I avoid making eye contact because the patient may perceive that as a threatening gesture and respond with violence." C) "Touch conveys concern for the patient, so I try to touch the patient's shoulder or arm as soon as I arrive at his or her side." D) "If the patient raises his voice to you, you must raise your voice in return. Research has shown that this helps deter violence." Answer: A Diff: 2 Page Ref: 798 Objective: 26-6 28) You are assessing a patient with schizophrenia. Friends called 911 because he was experiencing hallucinations this morning. He is prescribed Thorazine and risperidone for his illness. Which question is most pertinent to the current situation and should be asked first? A) "Do you drink alcohol to help get you through the day?" B) "Have you been taking your medications as prescribed?" C) "Do you think that the hallucinations are real?" D) "Has your psychiatrist considered changing the dose of your medications?" Answer: B Diff: 3 Page Ref: 789 Objective: 26-2 29) You have been called for a young male patient who has been acting withdrawn. As a precaution, dispatch also alerted police, who are now on scene with you. The patient appears depressed and when you ask him if he is thinking about hurting himself, he answers, "Yes." The next question asked by the EMT should be: A) "Do you have a plan?" B) "Do you take medications?" C) "Are there guns in the house?" D) "Do you want to go to the hospital?" Answer: A Diff: 2 Page Ref: 795 Objective: 26-7 9 Copyright © 2018 Pearson Education, Inc.

30) You are preparing to transport a patient with a behavioral emergency to the hospital. As he is being loaded into the ambulance, he smiles and says, "What if I decide to just go crazy and hurt you during the ride to the hospital?" What is your best response? A) "I am in charge here and you will do as I say. Understand?" B) "That is impossible because you are in restraints." C) "If you threaten me again, I will tighten the strap on your chest so it is hard to breathe." D) "If you touch or hurt me, you will be physically restrained immediately. No exceptions." Answer: D Diff: 2 Page Ref: 800 Objective: 26-8 31) You have arrived on the scene where a depressed patient is crying in an upstairs bedroom. When assessing her, which action should be done prior to the others? A) Obtain vital signs B) Determine the patient's medications C) Check a radial pulse D) Assess the patient for suicide risk Answer: C Diff: 2 Page Ref: 800 Objective: 26-7 32) You are responding to a home for injuries resulting from a domestic altercation. In preparation for patient contact, what would you tell your partner? A) "Stand above the patient to achieve a stature of dominance." B) "Let's go in and assess the situation, and then come up with a plan." C) "Always keep the patient in front of you and your eyes on the door." D) "Do not let the patient get between you and the door." Answer: D Diff: 2 Page Ref: 801 Objective: 26-4 33) When at the scene where a patient has attempted suicide, what is the most immediate priority? A) Preserving evidence B) Crew safety C) Assessing life-threatening conditions D) Patient safety and well-being Answer: B Diff: 2 Page Ref: 801 Objective: 26-7

10 Copyright © 2018 Pearson Education, Inc.

34) Your partner is assessing a patient who has a history of antisocial disorder and just got into a heated verbal exchange with his neighbor. He threatened to kill his neighbor, and 911 was called. As you observe the patient, what is the strongest indication that the patient may be ready to have a violent outburst? A) Avoiding eye contact B) Clenching of both fists C) Refusal to answer questions D) Increased sighing Answer: B Diff: 2 Page Ref: 794-795 Objective: 26-4 35) How can the EMT best keep a psychiatric patient calm and avoid provoking a violent outburst? A) Inform the patient of all that is going on and use a calm voice throughout care B) Establish that the EMT is in control of the situation and that how the call goes depends on the patient C) Inform the patient that he is responsible for all of his actions that occur throughout the call D) Involve law enforcement early in the call as a deterrent for violence Answer: A Diff: 3 Page Ref: 797-798 Objective: 26-6 36) After taking a young, depressed female to the hospital, your partner says, "I am tired of running all of these psych patient calls. There are real patients with real emergencies and pain out there. It is just a waste of our time!" Given his thoughts, what is your best response? A) "If we do our jobs right, many behavioral patients can make a solid and quick recovery." B) "It is a sad fact that little can be done to help these patients, but they still need to get help." C) "Patients with mental problems are suffering from emotional pain and injuries, which can be just as real as physical pain." D) "As EMTs, our only goal is getting the patients to the hospital where they can get the help they need." Answer: C Diff: 3 Page Ref: 796 Objective: 26-6

11 Copyright © 2018 Pearson Education, Inc.

37) You have been called to an assisted living facility for a 43-year-old male patient who has a history of anxiety and schizophrenia and who, according to staff, is more confused today. The staff also states that the patient has refused to take his medications for the past three days. Your assessment reveals him to have no primary assessment life threats, but his speech is slurred and he does not make sense when he talks. As you review the paperwork provided to you by staff, you note that the patient has a history of thyroid problems, diabetes, and hypertension. His pulse is 124 beats/min, respirations 16 breaths/min, and blood pressure 154/70 mmHg. At this point, which action is most important? A) Determine the patient's reason for refusing medications B) Restrain and transport the patient C) Check his blood glucose level D) Recheck the blood pressure Answer: C Diff: 2 Page Ref: 802, Figure 26-3A Objective: 26-7 38) You have been called to the post office to assist police with a disorderly male. On scene, you find a middle-aged male patient who states that he needs to be protected because the CIA is out to get him for something he did not do. As a knowledgeable EMT, you should recognize the patient's behavior as: A) Phobia B) Bipolar C) Anxiety D) Paranoia Answer: D Diff: 1 Page Ref: 791 Objective: 26-1 39) A 48-year-old female patient with a history of depression has called for help because she feels suicidal. During transport, the patient refuses to talk to you despite repeated attempts to obtain her past medical history and learn about the events leading up to her call for help. Which statement is most appropriate at this time? A) "It's okay if you do not want to talk, I will be right here if you change your mind." B) "I know that people with depression seem to get better very quickly by taking medications. Have you thought about taking some?" C) "Talking often helps people feel better. If you talk to me, I am sure that it will help." D) "I really must have information related to your medical history; therefore, you must talk to me." Answer: A Diff: 3 Page Ref: 799-800 Objective: 26-6

12 Copyright © 2018 Pearson Education, Inc.

40) You have been called to an extended care facility for a 48-year-old male patient with a history of schizophrenia. On scene, you find the patient to be highly agitated and clutching his left hand while complaining that there are bugs in it. After performing the primary assessment and finding no life threats, you should: A) Determine whether the patient is compliant with his medications B) Physically assess his left hand for any abnormalities C) Ask the patient to describe the bugs D) Reassure the patient that he is safe, and then transport to the hospital Answer: B Diff: 2 Page Ref: 801 Objective: 26-7 41) During transport, a female patient who is restrained begins to cry and states that she is sorry for becoming violent and will not behave in that manner anymore. She requests that the restraints be removed because they are hurting her legs and she does not wish to be restrained any longer. After ensuring that the restraints are not too tight and not harming the extremities, you would: A) Remove the leg restraints, but not the arm restraints B) Remove the restraints, but reapply them if the patient becomes combative C) Leave the restraints in place until patient care is transferred to the hospital D) Free one leg from the restraint, but leave the rest in place Answer: C Diff: 2 Page Ref: 805 Objective: 26-8 42) On scene, a young male experiencing a behavioral emergency talks very rapidly and constantly interrupts you while you attempt to ask him questions. As such you would recognize that the patient is exhibiting: A) Verbal dystonia B) Pressured speech C) Word salad D) Tardive dyskinesia Answer: B Diff: 1 Page Ref: 788 Objective: 26-5 43) A male patient with a history of behavioral problems continually smacks his lips, sticks his tongue out, and turns his head to the left. His arm also jerks upward regularly. Which statement, made by a caregiver, would an EMT recognize as most important and relating to this presentation? A) "He refused to take his morning medications." B) "He can be very delusional at times." C) "He has been on psych medications for many years." D) "Lately, he will only eat potato chips for breakfast." Answer: C Diff: 3 Page Ref: 788 Objective: 26-2 13 Copyright © 2018 Pearson Education, Inc.

44) You have been called for a female patient with a history of bipolar disease who is severely depressed and stated suicidal intentions. Which type of affect would you expect this patient to display? A) Flat B) Dystonic C) Labile D) Manic Answer: A Diff: 2 Page Ref: 788 Objective: 26-7 45) The EMT would recognize a patient as having a phobia when the patient makes which statement? A) "I am unemployed and think that I will lose my home." B) "I found a snake in my house and am having dreams of finding others." C) "I was just diagnosed with cancer and think that it is terminal." D) "I cannot eat meat because they put dangerous bacteria in it that can hurt me." Answer: D Diff: 3 Page Ref: 790 Objective: 26-1 46) You are called by a husband who states that his wife is bipolar and in need of help at the hospital. Which presentation would make the most sense related to this disorder? A) Distortion of speech and thought B) Thoughts of persecution C) Depressed and not wanting to talk D) Paranoia Answer: C Diff: 2 Page Ref: 790 Objective: 26-3 47) A patient experiencing command hallucinations would most likely tell you: A) His arm is stiff and cannot be bent B) He sees worms and parasites crawling on his arm C) His arm smells like rotten meat and is dead D) He was told by Zeus that he must amputate his arm so he will not die Answer: D Diff: 3 Page Ref: 791 Objective: 26-7

14 Copyright © 2018 Pearson Education, Inc.

48) Which type of hallucination is most dangerous? A) Command B) Tactile C) Visual D) Olfactory Answer: A Diff: 1 Page Ref: 791 Objective: 26-4 49) You are at a community crisis center with a male patient who is depressed. What is the most important question you must ask first in the care of this patient? A) "Have you been taking your medications?" B) "Do you feel suicidal or homicidal?" C) "Which behavioral disorders have you been diagnosed with?" D) "Have your medications been changed?" Answer: B Diff: 2 Page Ref: 795 Objective: 26-6 50) You are part of your EMS system's QA/QI program. While reading PCRs, you come across one that pertained to a psychiatric emergency in which the patient was restrained. As you review the PCR, which statement would cause you to suspect that the EMS crew needs remedial education? A) "Each extremity of the patient was restrained with padded leather restraints." B) "The patient was advised repeatedly to adhere to EMS wishes, and was further advised that restraint procedures would occur if he didn't comply." C) "Following extremity restraint, the patient was positioned and secured prone on the cot, with his head turned to the side to allow airway monitoring." D) "The patient failed to comply with repeated instructions of the EMS crew, and became increasingly verbally and physically assaultive. The decision to restrain the patient was agreed upon by the EMS crew, the on-scene PD, and medical direction." Answer: C Diff: 2 Page Ref: 803, 805 Objective: 26-10

15 Copyright © 2018 Pearson Education, Inc.

Prehospital Emergency Care, 11e (Mistovich et al.) Chapter 27 Trauma Overview: The Trauma Patient and the Trauma System 1) A 49-year-old male has been stabbed in the lower right chest. Police tell you that the patient got into an argument with an unidentified man, who then stabbed him before fleeing the scene. After assessing the patient, you are suspicious that the knife punctured the lung and is causing internal bleeding. In this situation, the mechanism of injury would be: A) Stab wound to the right chest B) Assailant with a knife C) Penetrating trauma to the lungs D) Internal hemorrhage Answer: A Diff: 2 Page Ref: 811 Objective: 27-9 2) A 27-year-old male was involved in an altercation and was struck in the ribs with a baseball bat. Assessment reveals intact skin with bruising to the right lateral chest. When palpating this area, pain, instability, and crepitus are all observed. The patient also complains of difficult and painful breathing. The EMT would recognize: A) Thoracic injury secondary to penetrating trauma B) Chest injury caused by blunt trauma C) Pulmonary injury caused by penetrating trauma D) Chest wall injury caused by acceleration forces Answer: B Diff: 2 Page Ref: 830 Objective: 27-2 3) A patient has been critically hurt in a nighttime motor vehicle collision. She was wearing a seat belt and hit a tree at 70 mph. The car she was driving weighed 2 tons and was equipped with air bags that did deploy. Which factor had the greatest impact on the injuries she sustained? A) Weight of the vehicle B) Deployment of air bags C) Speed of the vehicle at impact D) Nighttime conditions Answer: C Diff: 1 Page Ref: 812-813 Objective: 27-3 4) Two cars have collided head-on. One car was traveling at a speed of 55 mph and the other at 35 mph. The total speed of impact would be: A) 20 mph B) 35 mph C) 55 mph D) 90 mph Answer: D Diff: 1 Page Ref: 813-814 Objective: 27-4 1 Copyright © 2018 Pearson Education, Inc.

5) A car, driven by an unrestrained male, strikes a utility pole at 35 mph. Given this scenario, which would be true? A) The patient struck the steering wheel at a combined speed of 70 mph B) The internal organs struck the inside of the body at a speed of 35 mph C) The utility pole absorbed half of the kinetic energy, so that the body collision occurred at a speed of 17.5 mph D) The energy transferred to the body is increased by a factor of 2 Answer: B Diff: 1 Page Ref: 812 Objective: 27-4 6) An unrestrained driver of a car that has struck a tree at 45 mph has suffered a contusion to his heart. The EMT would recognize that this injury occurred during which impact of the collision? A) Vehicle collision B) Body collision C) Organ collision D) Physiological collision Answer: C Diff: 1 Page Ref: 812 Objective: 27-4 7) A patient who was involved in a motor vehicle collision is complaining of neck pain. Which piece of information would most contribute to the EMT's high index of suspicion that the patient may have suffered more significant injuries beyond isolated neck pain? A) Lack of air bags in the car B) Difficulty turning is head to the right C) External damage to the car is significant D) Death of the car's driver Answer: D Diff: 2 Page Ref: 814 Objective: 27-5 8) Emergency Medical Responders have removed an intoxicated and confused patient from a car that struck another vehicle at a high rate of speed. The patient was unrestrained and his car did not have air bags. Assessment reveals deformity bilaterally to the hips and femurs and bruising to the knees. Based on this injury pattern, the EMT would recognize which type of injury mechanism? A) Down and under B) Lateral impact C) Up and over D) High velocity Answer: A Diff: 3 Page Ref: 815 Objective: 27-6

2 Copyright © 2018 Pearson Education, Inc.

9) As you pull up to a motor vehicle collision, you quickly scan the scene for clues as to the type and severity of injuries. Which observation would best indicate that a patient may have a head injury? A) Air bag deployment B) Starburst on the windshield C) Blood on a patient's clothing D) Frontal-type collision Answer: B Diff: 2 Page Ref: 816 Objective: 27-6 10) A "whiplash"-type neck injury is most often observed with which type of collision? A) Frontal impact B) Ejection C) Rear-end impact D) Rollover Answer: C Diff: 1 Page Ref: 817 Objective: 27-6 11) Which piece of information would you provide to a group of young teenage drivers to decrease their chance of suffering a "whiplash"-type injury if involved in a motor vehicle collision? A) Ensure that the head rest is properly positioned B) Position the car's seats in a full upright position C) Make sure the car has side-door air bags D) Properly wear seat belts with a headrest in the down position Answer: A Diff: 2 Page Ref: 817 Objective: 27-6 12) Assessment of a patient involved in a motor vehicle accident reveals him to have crepitus to the left humerus, instability to the left lateral chest wall and flank, and pain on palpation to left hip region. The patient also complains of pain to the right side of the neck. Based on this injury pattern, the EMT would recognize the patient was: A) An unrestrained driver involved in a head-on collision B) The passenger in a car hit from behind C) An unrestrained rear seat occupant in a car struck from the right side D) The driver of a car hit on the driver's side Answer: D Diff: 3 Page Ref: 817-818 Objective: 27-6

3 Copyright © 2018 Pearson Education, Inc.

13) An unrestrained female driving a small car is involved in a rollover-type collision. Why is her risk for serious injury or death significantly increased? A) Rollovers are the result of high speeds B) Smaller and lighter cars tend to roll multiple times C) The risk for ejection is lessened, increasing injuries suffered in the car D) There are more impacts in a rollover causing injury Answer: D Diff: 1 Page Ref: 818-819 Objective: 27-6 14) You are responding to a call for a 4-year-old child hit by a car. When assessing this child, which injury patterns would you recognize as typical based on the child's age and mechanism of injury? A) Deformed femurs, bruises to the chest and abdomen, and head injury B) Left arm and leg deformities, and head injury C) Bilateral ankle deformity, contusions to the back, and face trauma D) Head injury with deformities to both upper arms and neck trauma Answer: A Diff: 3 Page Ref: 819-820 Objective: 27-6 15) A young female wears her lap belt too low, over her upper thighs, because the belt is uncomfortable when worn properly. If she is involved in a head-on "up and over" type of collision, to which injury is she most prone, given the position of her lap belt? A) Internal abdominal injury B) Dislocated hips C) Tibia-fibula fractures D) Bilateral arm fractures Answer: B Diff: 2 Page Ref: 820 Objective: 27-6 16) A patient with severe head and neck pain states that he was properly wearing his seat belt when another car struck him from behind. As a knowledgeable EMT, you would realize that the complaint of: A) Head and neck pain indicates that he was not wearing his seat belt B) Head pain suggests that he was not wearing the seat belt properly C) Head and neck pain could occur even when the seat belt is properly worn D) Head and neck pain indicates that the lap belt was worn without the shoulder harness Answer: C Diff: 2 Page Ref: 817 Objective: 27-6

4 Copyright © 2018 Pearson Education, Inc.

17) When teaching Emergency Medical Responders about air bags, which point would you emphasize? A) Properly placed air bags eliminate the need for seat belts B) The benefit of air bags is lessened in a multiple-vehicle collision impact C) Air bags prevent broken glass from striking the occupant D) Air bags are effective in decreasing injury in a rollover-type collision Answer: B Diff: 1 Page Ref: 820 Objective: 27-6 18) A car has been hit head-on by another vehicle at a moderate rate of speed. Seat belts were in place and the air bags deployed. The patient complains of chest pain. Quick observation reveals a bruise to his sternum. The EMT should immediately: A) Deflate the air bag to get breath sounds B) Check the windshield for starring C) Suspect an injury related to the air bag D) Lift the air bag and check for steering wheel deformity Answer: D Diff: 2 Page Ref: 815 Objective: 27-6 19) As part of your service to the community, you are certified as a car seat specialist and provide monthly classes on the safe transport of infants and children. Over the course of one day, several cars have pulled into your EMS station and asked you to look at their car seats. Which car seat position would indicate that the parent is safely transporting his or her infant (younger than 1 year old)? A) Facing forward in the back seat in a reclined position (60 degrees) B) Facing backward in an upright position in the front seat (45 degrees) C) Facing backward in the back seat in a reclined position (45 degrees) D) Facing forward in the back seat in an upright position (60 degrees) Answer: C Diff: 1 Page Ref: 821 Objective: 27-6 20) You are hosting a discussion on injury prevention when a participant asks you if you believe that helmets for motorcyclists truly make a difference. You would respond by saying: A) "Motorcycle helmets decrease the risk of death when the rider is involved in a collision." B) "Helmets have been shown to prevent motorcycle accidents." C) "Wearing a helmet decreases the chance of being ejected from a motorcycle." D) "A motorcycle helmet decreases the chance of permanent spinal injury." Answer: A Diff: 1 Page Ref: 821 Objective: 27-7

5 Copyright © 2018 Pearson Education, Inc.

21) You are assessing a patient who was involved in a motorcycle accident. He states that he saw an oncoming car and "laid the bike down" to avoid an outright collision. He was wearing a helmet. Which injury pattern would make sense given the action of the motorcyclist? A) Angulation to both femurs with protrusion B) "Road rash" to the entire body surface C) Burn to the inside of the leg from the motorcycle exhaust pipe D) Bilateral broken forearms with upper extremity trauma Answer: C Diff: 3 Page Ref: 821 Objective: 27-7 22) A 23-year-old roofer has fallen 20 feet from the roof of a house into shrubs below. Aside from superficial abrasions and complaint of soreness, you detect no obvious injuries. Consequently, the patient states that as long as his legs are not broken, he does not see the need to go to the hospital. As a knowledgeable EMT, your best response would be: A) "Since this is a work-related injury, you should really be evaluated in the hospital. Worker's compensation may not cover you if you do not." B) "In a fall from this height, you may have hurt some internal organs. You really need to be examined." C) "Since you did not lose consciousness, I feel better about having you refuse. Just keep an eye on your feet and watch for swelling." D) "Although you feel okay now, you will most likely be sore later on. Why don't we go to the hospital and ask about some medications for pain?" Answer: B Diff: 3 Page Ref: 822 Objective: 27-8 23) A patient has been stabbed with a knife at the fifth intercostal space on the right side of the chest in an attempted suicide. Assessment reveals him to be short of breath and coughing. Which question is it most important for the EMT to ask regarding the weapon? A) "How long was the knife blade?" B) "Was the knife clean or dirty?" C) "Did the knife penetrate your chest easily?" D) "Did you push the knife in slowly or quickly?" Answer: A Diff: 2 Page Ref: 823 Objective: 27-9

6 Copyright © 2018 Pearson Education, Inc.

24) A bullet fired from a gun at close range passes through the patient's liver. However, on autopsy, the coroner discovers that the man's pancreas, stomach, and gallbladder were also injured, even though not in direct contact with the bullet. As a knowledgeable EMT, you would recognize that which mechanism is responsible for injuries to these additional organs? A) Cavitation B) Drag force C) Profile impact D) Blunt injury Answer: A Diff: 2 Page Ref: 823 Objective: 27-9 25) Which of these is a high-velocity wound that carries the greatest risk for death when considering the concept of cavitation? A) Shotgun wound to the lung B) Gunshot wound to the spleen C) Stab wound to the liver D) Gunshot wound to the stomach Answer: B Diff: 2 Page Ref: 823 Objective: 27-9 26) You have arrived on the scene of an explosion in a warehouse used to store fertilizers. A paramedic, who has already triaged the nine patients involved, directs you to a young man sitting under a tree. The man states that he is having excruciating ear pain. The initial assessment and rapid trauma assessment reveal no obvious injuries. Given this presentation, the EMT would recognize the injury was likely sustained: A) From the noise of the explosion B) During the primary blast phase C) After the secondary blast phase D) As a result of exposure to superheated air Answer: B Diff: 2 Page Ref: 825 Objective: 27-10 27) A patient, who was constructing a bomb in his basement, has sustained a secondary-phase blast injury. Which presentation would the EMT most likely observe from this blast phase? A) Burns to the head, neck, and chest B) Headache and shortness of breath C) Screwdriver impaled in the thorax D) Complaint of nausea after inhaling fumes Answer: C Diff: 2 Page Ref: 825 Objective: 27-10

7 Copyright © 2018 Pearson Education, Inc.

28) During an in-service program focusing on care of the trauma patient, the medical director asks if anyone can correctly describe the "platinum 10 minutes." What statement by a participant is the best response? A) "If the transport to a trauma center will exceed 10 minutes, EMS should strongly consider the use of an aeromedical helicopter." B) "EMS should spend 10 minutes assessing the patient for life-threatening injuries prior to initiating transport to the emergency department." C) "EMS systems should be designed so that it takes an ambulance no longer than 10 minutes to respond to a trauma call." D) "EMS should initiate transport of the critically injured trauma patient to the hospital within 10 minutes of arriving on scene." Answer: D Diff: 2 Page Ref: 827 Objective: 27-11 29) The EMT understands the importance of evaluating the mechanism of injury when he states: A) "The mechanism of injury is useful in determining the exact injuries the patient has sustained." B) "Evaluating the mechanism of injury is important because it determines whether emergency transport to the hospital is needed." C) "The mechanism of injury is a useful tool in determining whether the patient's outcome will be good or bad." D) "The mechanism of injury can be used to guide the EMT's injury assessment and treatment of the patient." Answer: D Diff: 2 Page Ref: 811 Objective: 27-2 30) After assessing these patients, which would the EMT identify as suffering from multisystem trauma? A) A 24-year-old male who slipped on ice and has tenderness to the right shoulder B) A 61-year-old male who became dizzy and fell down five stairs, and who has a laceration to the head and open fracture of the left lower leg C) A 31-year-old female who has a large laceration to her leg from a hunting knife, with bleeding controlled D) An 89-year-old female who fell in a nursing home and has a hematoma to the back of her head Answer: B Diff: 2 Page Ref: 825 Objective: 27-1

8 Copyright © 2018 Pearson Education, Inc.

31) Which of the statements concerning trauma is most accurate? A) Falls are the leading cause of trauma deaths B) The mechanism of injury is the best predictor of patient outcome C) Multisystem trauma has a higher mortality rate than single-system injuries D) A "spider web" or "star" pattern of cracks on the windshield means the patient impacted the windshield with his head Answer: C Diff: 2 Page Ref: 825 Objective: 27-1 32) Which statement best characterizes the capabilities of a Level III trauma center? A) The hospital can manage all trauma patients and injuries, 24 hours a day, 7 days a week B) The hospital can handle most trauma patients, but will transfer those who require specialty care for specific injuries such as neurologic trauma C) The hospital will provide any emergency surgical services to a trauma patient and then transfer the patient once stable D) The hospital has some surgical capabilities to help trauma patients, but will generally stabilize and them and still arrange transfer Answer: D Diff: 2 Page Ref: 828 Objective: 27-12 33) What is the best description of the goal of a trauma system? A) Getting the right patient to the right facility in the right amount of time B) Providing the highest level of care possible within a 10-minute transport time frame for all patients C) Providing all trauma patients with a complete battery of diagnostic tests and access to specialists D) Transporting as many patients as possible by air medical transportation Answer: A Diff: 1 Page Ref: 827-828 Objective: 27-12 34) A male patient has sustained gunshot wounds to the neck and abdomen. He is unresponsive and has gurgling respirations. Your immediate action would be to: A) Insert a nasal airway and place the patient on a long backboard B) Place a cervical collar and position the patient on a long board C) Start positive pressure ventilation and then place a cervical collar and maintain manual spine motion restriction D) Suction the airway while providing manual spine motion restriction Answer: D Diff: 2 Page Ref: 830 Objective: 27-14

9 Copyright © 2018 Pearson Education, Inc.

35) You are first on the scene of a two-car motor vehicle collision. Two patients are entrapped in one vehicle. In the other vehicle, there is a male driver complaining of chest pain. During this time, your primary concern is: A) Calling for additional help B) Ensuring personal safety C) Triaging the patients D) Determining severity of injuries Answer: B Diff: 2 Page Ref: 828 Objective: 27-14 36) An 8-year-old boy fell forward off of a swing onto outstretched arms. He did not hit his head or lose consciousness and is complaining of left forearm pain. There is deformity to the left wrist, but no other injuries or life threats. His vital signs are pulse 132 beats/min, respirations 20 breaths/min, blood pressure 108/62 mmHg, and SpO2 100%. The proper care of this patient would be to: A) Immediately load and transport the patient, splinting the injury en route B) Apply supplemental oxygen and anatomically splint the forearm when the patient is placed on a long backboard C) Secure the forearm to a padded board splint and apply a cold pack to the injury site before transporting the patient nonemergently D) Call ALS so that the patient can be given intravenous pain medications before splinting the forearm Answer: C Diff: 2 Page Ref: 822 Objective: 27-8 37) You arrive on the scene of a motorcycle accident and find an unresponsive male patient lying in the middle of the road with blood coming from his ears and nose. He was not wearing a helmet and has snoring respirations. Which sequence of events would be most appropriate in the care of this patient? A) Primary assessment, airway and ventilatory management, oxygen administration, spine motion restriction precautions, transport within 10 minutes of arriving on scene B) Primary assessment, oxygen administration, transfer to stretcher, transport within 5 minutes of arriving on scene with spine motion restriction precautions done en route C) Full spine motion restriction procedures, transfer to the ambulance, provide all assessment and care en route to the hospital, with total scene time of 7 minutes D) Primary assessment, spine motion restriction, transfer to the ambulance after 8 minutes of scene time, all other care provided en route Answer: A Diff: 2 Page Ref: 827 Objective: 27-11

10 Copyright © 2018 Pearson Education, Inc.

38) What is the best description of the "platinum 10 minutes" as it applies to care of the trauma patient? A) The goal for total amount of time from the actual injury to arrival of EMS at the scene B) The target maximum scene time for EMS as it applies to the seriously injured trauma patient C) The time it should take the transporting ambulance to get the trauma patient to the closest medical facility D) The target maximum time that the multisystem trauma patient should spend in the emergency department before being transferred to surgery Answer: B Diff: 1 Page Ref: 827 Objective: 27-11 39) Which injury is most serious, warranting immediate and rapid transport to the local trauma center? A) Instability and crepitus to the left lateral chest wall B) Deformity to the right ankle with pain rated as a 10/10 C) Open fracture to the left lower leg D) Large, deep laceration to the arm with bone visible Answer: A Diff: 2 Page Ref: 829, Figure 27-29 Objective: 27-13 40) You are assessing a male patient who was stabbed three times in the chest and abdomen. As you begin your primary assessment, you note the patient to have a decreased level of consciousness and gurgling respirations. Your next action would be to: A) Check his pulse B) Suction the airway C) Obtain an SpO2 reading D) Apply a cervical collar Answer: B Diff: 2 Page Ref: 830 Objective: 27-9 41) A teenage female patient was ejected from a car as it rolled down a hill after she ran off the road. She is responsive, but with signs of early shock and respiratory distress. She has bruising to her anterior abdomen and chest, a laceration to her left temple, and deformity to her left wrist. At which point should the wrist injury be splinted? A) Prior to full spinal motion restriction precautions being provided B) Before transferring the patient to the stretcher C) Just before leaving the scene D) En route to the hospital Answer: D Diff: 2 Page Ref: 830 Objective: 27-5

11 Copyright © 2018 Pearson Education, Inc.

42) You arrive at the scene where a young male patient fell out of a tree he was climbing. Although he fell from a lower branch, he was still reportedly 10-12 feet in the air when he fell. The patient landed on his side on the ground. Which element of the kinematics of trauma would explain the mechanism of any injuries he may have sustained? A) Rapid acceleration B) Terminal velocity C) Rapid deceleration D) Terminal mass Answer: C Diff: 1 Page Ref: 812-813 Objective: 27-3 43) Regarding the field triage of injured patients, which of these patients is NOT a member of a special patient population that would necessitate transport to a trauma center? A) A female patient who is 12 weeks pregnant and who fell on the sidewalk, injuring her hip and wrist B) A 70-year-old male patient with a systolic blood pressure of 104 mmHg after wrecking his car into a tree C) A female patient with a suspected brain injury from a fall who also takes prescribed anticoagulants D) A male patient with a broken femur and burns to 25 percent of his body after jumping from a second-story window of a house that was on fire Answer: A Diff: 2 Page Ref: 829, Figure 27-29 Objective: 27-13

12 Copyright © 2018 Pearson Education, Inc.

Prehospital Emergency Care, 11e (Mistovich et al.) Chapter 28 Bleeding and Soft Tissue Trauma 1) A 24-year-old male has cut his arm with a table saw. Assessment reveals dark red blood flowing steadily from the laceration. As an EMT, you would identify this bleeding as: A) Arterial B) Capillary C) Venous D) Lymphatic Answer: C Diff: 1 Page Ref: 836 Objective: 28-3 2) While en route to a domestic violence incident with possible injuries, the police contact you and report that a male patient has arterial bleeding from a stab wound to the arm. Given this information, which bleeding characteristics do you expect? A) Dark red blood flowing from the injury that is difficult to control B) Bright red blood oozing from the injury site that is easy to control C) Spurting blood that is dark red in color and difficult to control D) Bright red blood that is spurting with each beat of the heart Answer: D Diff: 1 Page Ref: 835-836 Objective: 28-3 3) You arrive at a meat packing plant to find a 30-year-old male patient who was cut in the wrist with a sharp knife. The patient appears pale, and blood is spurting from the laceration. Your immediate action would be to: A) Assess and manage the airway B) Apply direct pressure to the laceration C) Provide supplemental oxygen D) Check the pulse and blood pressure Answer: B Diff: 2 Page Ref: 836 Objective: 28-3 4) As you approach a young male lying on the sidewalk who was called into 911 as an "unresponsive male," you observe an area of dark blood on his pants. He appears lethargic, pale, and diaphoretic. Your first action in caring for this patient would be to: A) Apply oxygen B) Cut the pants C) Assess the airway D) Elevate the leg Answer: C Diff: 2 Page Ref: 840 Objective: 28-4

1 Copyright © 2018 Pearson Education, Inc.

5) A middle-aged male's arm became caught in a grinding machine, and the arm was eventually extricated from the machine by coworkers. As you approach the patient, you note him to be holding a bloody towel over a severely deformed right arm. The patient is alert and anxious, and he has a patent airway. His breathing is adequate and his left radial pulse is fast and strong. You note his skin to be cool and diaphoretic. Given this information, which care would be provided first for this patient? A) Start positive pressure ventilation B) Perform a detailed secondary exam focusing on his arm C) Insert an oropharyngeal airway and administer oxygen D) Administer supplemental oxygen Answer: D Diff: 2 Page Ref: 840 Objective: 28-4 6) Severe external bleeding should be controlled during which phase of the patient assessment? A) Primary assessment B) Scene size-up C) Secondary assessment D) Reassessment Answer: A Diff: 1 Page Ref: 840 Objective: 28-4 7) The EMT is by the side of a young female who was cut while washing dishes. She has a jagged laceration to her left forearm that is bleeding steadily and heavily. When attempting to control the bleeding, the EMT would first: A) Tightly wrap roller gauze around a large sterile dressing placed over the top of the injury B) Apply direct pressure to the elbow to slow the flow of blood into the distal arm C) Place a large sterile dressing over the site, with hand pressure applied over the top of the dressing D) Wrap the injury site tightly with roller gauze and apply a tourniquet above the laceration Answer: C Diff: 2 Page Ref: 840 Objective: 28-3 8) Direct pressure has failed to control an arterial bleed on a patient's lower leg caused by an industrial accident. What would the EMT's next action be? A) Provide oxygen B) Apply a tourniquet C) Provide immediate transport D) Splint the extremity Answer: B Diff: 1 Page Ref: 840 Objective: 28-3

2 Copyright © 2018 Pearson Education, Inc.

9) Which statement shows that the EMT understands the use of a tourniquet in controlling bleeding? A) "A tourniquet should be applied tightly enough that arterial blood flow distal to the tourniquet is completely stopped." B) "If the tourniquet appears to have stopped the bleeding, the EMT should loosen it slightly to allow a small amount of blood to perfuse the tissues." C) "If a blood pressure cuff is used as a tourniquet, the EMT should inflate it approximately 50 mmHg above the patient's systolic blood pressure." D) "Since arteries run close to the body's surface in a joint, the best place to apply a tourniquet is over the joint just above the site of hemorrhage." Answer: A Diff: 3 Page Ref: 837 Objective: 28-4 10) A young male was riding a motorcycle when he lost control and was thrown from the vehicle. The patient is unresponsive and receiving positive pressure ventilation. The secondary assessment reveals bright blood flowing from an unrecognized femoral bleed. Initial management of this bleeding would include: A) Application of a proximal tourniquet B) Direct pressure with a dressing applied C) Elevating the extremity and applying a cold compress D) Placing a tourniquet distal to the injury if direct pressure fails to stop the bleed Answer: B Diff: 2 Page Ref: 837 Objective: 28-3 11) Another crew has called for your assistance in extricating an obese male with a nosebleed from a third-floor bedroom. As you enter the room, which observation indicates proper management of the patient's condition? A) The patient is supine, head turned to the side, blood draining from the nose and mouth, with a cold pack applied to the bridge of the nose B) The patient is sitting upright, head tilted back, with sterile gauze packed in each nostril C) The patient is lying on his side, a loose dressing has been applied below his nose, and a cold pack is on the bridge of his nose D) The patient is sitting upright, leaning forward, with nostrils pinched shut Answer: D Diff: 2 Page Ref: 841 Objective: 28-5

3 Copyright © 2018 Pearson Education, Inc.

12) You have arrived at a scene where a young girl tripped while running, and pushed her arms through a plate glass window while falling. She is responsive to painful stimuli and has blood spurting from a large laceration on her right upper arm. Your partner yells for you to immediately apply direct pressure to the laceration to prevent further blood loss, while he works to control the airway. You have yet to apply gloves, but your uniform shirt has long sleeves, and you normally wear prescription glasses. What would you do next? A) Place a tourniquet above the laceration, being careful not to get blood on your hands B) Apply gloves prior to applying direct pressure over the injury site C) Control bleeding by pushing the skin of the laceration together until you can apply gloves D) Apply direct pressure to the site with your forearm until someone with gloves can take over Answer: B Diff: 2 Page Ref: 840 Objective: 28-4 13) Which statement demonstrates an understanding of shock caused by internal bleeding? A) "Internal bleeding is a less serious life threat than external bleeding because the blood remains in the body." B) "The EMT can control internal bleeding by applying direct pressure to the skin, over the top of the area of suspected internal bleeding." C) "If a patient is bleeding internally, the signs of shock–such as a rapid heart rate and pale, cool, and clammy skin–are not as obvious." D) "You should always suspect internal bleeding in a patient with the signs and symptoms of shock but no external bleeding." Answer: D Diff: 3 Page Ref: 842 Objective: 28-8 14) You believe that a patient who has been shot in the lower abdomen is bleeding internally and is in an early stage of shock. The patient's mental status is decreasing. What would be appropriate prehospital care of this patient? A) Semi-Fowler's position and direct pressure over the injury site B) Supine position and administration of water by mouth C) Oxygen therapy and rapid transport to the hospital D) Oxygen therapy and cold packs to the abdomen Answer: C Diff: 2 Page Ref: 843 Objective: 28-8

4 Copyright © 2018 Pearson Education, Inc.

15) You are having a difficult time controlling bleeding from a small skin avulsion on a patient's ankle. Which statement made by the patient best explains why control has been difficult to achieve? A) "I take Coumadin for my irregular heartbeat." B) "My blood pressure sometimes runs a little high." C) "I take steroids for my lung disease." D) "I drank a lot of alcohol last night." Answer: A Diff: 2 Page Ref: 844 Objective: 28-7 16) In the human body, which condition is present when a person is in shock? A) The cells get glucose and other nutrients, but not oxygen B) The amount of oxygen to the cells is adequate, but CO2 is not being eliminated C) The blood has an adequate amount of oxygen, but not enough nutrients for cell survival D) The cells do not get enough oxygen and waste products accumulate Answer: D Diff: 3 Page Ref: 844 Objective: 28-2 17) Which statement made by your new EMT partner best describes the way to determine the severity of a patient's blood loss? A) "I estimate the blood loss based on the fact that one cup of blood is about 250 mL." B) "If the pulse rate is greater than 100 beats per minute, the patient has lost at least 200 mL of blood." C) "I use the patient's signs and symptoms to judge the severity of the blood loss." D) "I look at the blood pressure, and if it is less than 100 mmHg systolic, the patient has lost 500 mL of blood." Answer: C Diff: 2 Page Ref: 842 Objective: 28-6 18) You have been dispatched to a residence where a woman lacerated her arm after falling while holding a drinking glass. She informs you that when she initially cut herself, blood spurted from the wound; then, within a second or two, it subsided to a mere trickle. She asks why this would happen. What is the correct response? A) "Blood has the ability to clot within a second or two of injury." B) "Blood vessels tend to constrict quickly once they are injured." C) "Red blood cells will quickly repair the injury to the blood vessel." D) "The skin around the injured blood vessels quickly swells and stops bleeding." Answer: B Diff: 3 Page Ref: 835 Objective: 28-4

5 Copyright © 2018 Pearson Education, Inc.

19) When performing the primary assessment, which sign or symptom best suggests that the patient is in shock? A) Skin that is cool and diaphoretic B) Radial pulse of 72 beats per minute C) Bloody shirt from external bleeding D) Pain at the site of injury Answer: A Diff: 2 Page Ref: 842 Objective: 28-2 20) A patient is confused and anxious after falling 20 feet from a ladder onto the grass below. Which clinical finding described is most suggestive of shock? A) Heart rate of 112 beats per minute B) Deformity to the left arm C) Large hematoma to the head D) Constricted pupils Answer: A Diff: 2 Page Ref: 842 Objective: 28-8 21) A young man fell down a flight of stairs and is now restless and confused. His airway is open, and he is breathing adequately at 18 times per minute. He has a radial pulse of 102 beats per minute, a blood pressure of 96/78 mmHg, and his skin is cool and diaphoretic. Your next action would be to: A) Check the blood pressure and assess for injuries causing blood loss B) Administer supplemental oxygen C) Insert a nasal airway and start positive pressure ventilation with oxygen D) Place a cervical collar on the patient and secure him to a long back board Answer: B Diff: 2 Page Ref: 842 Objective: 28-6 22) Which sign or symptom of shock will be observed last? A) Tachycardia B) Increased respirations C) Decreased blood pressure D) Pale and diaphoretic skin Answer: C Diff: 2 Page Ref: 845 Objective: 28-8

6 Copyright © 2018 Pearson Education, Inc.

23) Which statement indicates an understanding of the EMT's prehospital role in caring for the patient in shock secondary to significant soft tissue trauma? A) "The job of the EMT is to assume a trauma patient is in shock and get him or her to the hospital as quickly as possible." B) "Because shock is a life-threatening condition, it is important that the EMT identify the exact cause so proper care can be given." C) "If shock is in its early stage, it is not yet life-threatening and the EMT can take more time in assessing and treating the patient." D) "Since trauma is best treated in the hospital, the EMT should provide care to maintain perfusion to the vital organs and transport." Answer: D Diff: 2 Page Ref: 845 Objective: 28-8 24) Assessment findings for a driver who was ejected from his vehicle in a rollover-type collision include unresponsiveness and bruising to the abdominal and pelvic areas, along with an open femur fracture. The patient has an open airway and is breathing at 32 times per minute with absent alveolar breath sounds. His skin is cool and diaphoretic; the radial pulses are weak. Manual cervical spine motion restriction is being maintained. The EMT's next action would be to: A) Obtain a heart rate and blood pressure B) Start positive pressure ventilation C) Examine the fracture site more closely D) Apply an appropriately sized cervical collar Answer: B Diff: 2 Page Ref: 846 Objective: 28-8 25) A 28-year-old male was cutting limbs from a tree when he lost his footing and fell 20 feet. He is unresponsive and breathing very poorly, at 28 times per minute. His radial pulse is weak and thready, and his skin is cool to the touch. Emergency Medical Responders (EMRs) have placed the patient on a nonrebreather face mask and are holding manual cervical spine motion restriction. A blanket has also been applied and covers the man. Based on these assessment findings, which instruction would you give to the EMRs? A) "Since he is unresponsive, let's open the airway with the head-tilt, chin-lift maneuver." B) "Let's uncover him since increased temperature will dilate his blood vessels and drop his BP." C) "Let's move him to the stretcher for immediate transport to the hospital." D) "Let's take off the oxygen mask and start positive pressure ventilation to assist his breathing." Answer: D Diff: 2 Page Ref: 846 Objective: 28-6

7 Copyright © 2018 Pearson Education, Inc.

26) While cleaning a gun, a 44-year-old man accidentally shot himself in the abdomen. On your arrival at the scene, the patient is responsive to painful stimuli and lying on his side, with his legs drawn to his chest. Blood is evident on his shirt and pants. The vital signs are minimally acceptable and the pulse oximeter reading is 94%. What is the correct sequence of events when caring for this patient? A) Perform a primary assessment and a rapid secondary assessment, move the patient to the stretcher for immediate transport, and start positive pressure ventilation en route to the hospital B) Perform the primary assessment, administer supplemental oxygen, perform the secondary assessment, transfer the patient to the stretcher, and provide rapid transport C) Transfer the patient to the ambulance, provide rapid transport, perform the primary and secondary assessments, and avoid oxygen therapy en route to the hospital D) Transfer the patient to the ambulance, perform the primary assessment, provide oxygen therapy, and provide rapid transport Answer: B Diff: 2 Page Ref: 845-846 Objective: 28-6 27) A 66-year-old female patient has been struck by a car. Your assessment reveals gurgling respirations, rapid breathing, and cool, diaphoretic skin. You also observe bruising to the chest and abdomen. What should you do most immediately? A) Determine the blood pressure B) Administer high-concentration oxygen C) Evaluate for signs of shock D) Suction the airway Answer: D Diff: 2 Page Ref: 861, Figure 28-32B Objective: 28-8 28) You are called for a 2-year-old boy who has a nosebleed. While you are en route to the call, the dispatcher informs you that the patient has hemophilia. As an EMT, you should recognize that: A) The patient requires oxygen B) The patient's blood is infectious C) The bleeding may be significant D) A mask will need to be worn Answer: C Diff: 1 Page Ref: 841 Objective: 28-7

8 Copyright © 2018 Pearson Education, Inc.

29) A male patient has been shot in the chest with a small-caliber gun. Which presentation indicates that he is in an early stage of shock? A) Alert and anxious, pulse 96 beats/min, BP 134/88 mmHg, pale and cool skin B) Slightly confused, pulse 116 beats/min, BP 112/90 mmHg, warm and flushed skin C) Confused and anxious, pulse 144 beats/min, BP 82 mmHg/palpation, cool and mottled skin D) Confused, pulse 44 beats/min, BP 110/68 mmHg, cool and cyanotic skin Answer: A Diff: 3 Page Ref: 845 Objective: 28-8 30) A 24-year-old female was an unrestrained driver of a car that struck a guardrail, causing her to sustain blunt trauma to the chest and abdomen. Vital signs obtained by Emergency Medical Responders are pulse 120 beats/min, respirations 22 breaths/min, blood pressure 100/78 mmHg, and SpO2 96%. As you transport her to the hospital, which finding indicates that the patient is deteriorating? A) Heart rate of 124 beats/min B) Blood pressure 86/50 mmHg C) Blood coming from an abdominal laceration D) Restless and confused mental status Answer: B Diff: 3 Page Ref: 845 Objective: 28-8 31) Which clinical finding is most suggestive of a patient who is decompensating? A) Narrowing pulse pressure B) Heart rate < 100 beats/min C) Pulse oximeter reading < 96% D) Skin that is pale, cold, and dry Answer: A Diff: 1 Page Ref: 845 Objective: 28-8 32) A patient suffered abrasions to the legs after falling on a moving treadmill. In relation to the function of the skin, the EMT should be most concerned about: A) Loss of body heat B) Risk for infection C) Formation of scar tissue D) Swelling and bruising Answer: B Diff: 1 Page Ref: 850 Objective: 28-10

9 Copyright © 2018 Pearson Education, Inc.

33) Which statement regarding contusions is true? A) The epidermis is open and actively bleeding B) Blood vessels in the subcutaneous layer have been broken C) Blood vessels in the dermal layer have ruptured D) Blood vessels in the epidermis have broken but the skin is intact Answer: C Diff: 2 Page Ref: 849 Objective: 28-1 34) What are the three main layers of the skin? A) Dermis, muscle, epidermis B) Epidermis, subcutaneous, muscle C) Dermis, subcutaneous, muscle D) Subcutaneous, epidermis, dermis Answer: D Diff: 1 Page Ref: 847 Objective: 28-10 35) When providing prehospital care to a female patient with a soft-tissue injury, which statement is true? A) Gloves are needed only if the patient has an open soft-tissue injury B) Hands should be washed even if gloves were worn during care C) Sterile gloves are essential to prevent infection to the patient D) Double-gloving is necessary to protect the EMT from blood borne diseases Answer: B Diff: 1 Page Ref: 834 Objective: 28-4 36) When managing a patient with a soft tissue injury, it is essential that the EMT performs which action? A) Thoroughly disinfect dirty lacerations B) Control bleeding with a tourniquet if the bleeding is arterial in nature C) Use the appropriate personal protective equipment D) Cover hematomas with occlusive dressings Answer: C Diff: 2 Page Ref: 834 Objective: 28-4

10 Copyright © 2018 Pearson Education, Inc.

37) Assessment of an 88-year-old female who fell reveals bruising to her right buttock. The skin is intact and the patient complains of tenderness to the area when you palpate. As a knowledgeable EMT, you would recognize this patient has which type of injury? A) Avulsion B) Abrasion C) Open injury D) Contusion Answer: D Diff: 1 Page Ref: 849 Objective: 28-9 38) When assessing a patient, you note a bruise to his chest. On the prehospital care report, this injury is properly documented as: A) Ecchymosis B) Cyanosis C) Erythematic D) Hematoma Answer: A Diff: 1 Page Ref: 849 Objective: 28-1 39) You are called for an 8-year-old boy who has fallen. At the scene, the boy's mother states that her son was running while flying a kite and tripped, striking his face on a rock. Assessment reveals a large reddish-blue lump with intact skin under his left eye. When alerting the hospital emergency department of your arrival, you should inform them that the patient has which type of injury? A) Ecchymosis B) Hematoma C) Evisceration D) Contusion Answer: B Diff: 1 Page Ref: 849 Objective: 28-9 40) A patient sustained a crush injury after being pinned by a car that fell off jack-stands while the patient was underneath the car working on it. The injury occurred to the left upper quadrant of the abdomen. Assessment shows bruising to that area, with instability noted to the lowest rib. The EMT's highest concern regarding this injury should be: A) Fractured rib B) Abdominal contusion C) Injury to the spleen D) Abdominal evisceration Answer: C Diff: 2 Page Ref: 849-850 Objective: 28-2

11 Copyright © 2018 Pearson Education, Inc.

41) You are called for a 78-year-old female who fell earlier in the day. As you enter her kitchen, you observe her sitting in a chair with ecchymotic areas to her left cheek and left arm. There also appears to be blood on her pants. She is on home oxygen for COPD and is breathing in a moderately labored manner. The patient states that she fell a few hours ago, and now the pain and swelling in her arm are unbearable. She has also developed significant lower neck pain. What should you do first? A) Take manual spinal motion restriction precautions B) Place a cold pack to her cheek C) Check her SpO2 level D) Examine her arm for deformity Answer: A Diff: 2 Page Ref: 849 Objective: 28-2 42) A 62-year-old male was the restrained driver of a car that was hit on the driver's side. Emergency Medical Responders have extricated the patient and provided spinal motion restriction precautions, including a cervical collar, long board, and straps. When asked, the patient complains of dizziness as well as left leg and left arm pain. Assessment reveals multiple contusions and deformity to his left forearm and a laceration with minor bleeding to the left side of his face. The patient's airway is open and his breathing is labored. A rapid radial pulse is felt. Skin is cool and dry. Which assessment finding would the EMT address first? A) Bleeding from facial laceration B) Rapid pulse C) Complaint of dizziness D) Labored breathing Answer: D Diff: 3 Page Ref: 850 Objective: 28-2 43) Which observations would cause the EMT to classify a patient's soft tissue injury as an abrasion? A) Intact skin with ecchymosis noted; patient reports minor pain B) Irregular break in the skin; bleeding is moderate but controllable C) Intact skin with large accumulation of dark blood underneath D) Skin is scraped and red; blood is oozing from the injury site Answer: D Diff: 2 Page Ref: 850 Objective: 28-1

12 Copyright © 2018 Pearson Education, Inc.

44) A 49-year-old male was climbing on a truck at a construction site when he fell backward to the ground. He presents with a 2-inch linear wound to the top of his head. Bleeding has been controlled and the skull can be seen through the wound. How should you document this injury on the prehospital care report? A) Avulsion B) Contusion C) Crush D) Laceration Answer: D Diff: 1 Page Ref: 851 Objective: 28-10 45) Prior to your arrival on the scene of a motorcycle crash, an Emergency Medical Responder contacts you via radio and reports that there is one patient who has an avulsion to her left arm. As a knowledgeable EMT, which kind of injury should you anticipate? A) Loose flap of skin torn on the patient's left arm B) Torn skin with an underlying fracture C) Bruised skin with a portion of bone protruding D) Long and deep laceration with moderate bleeding Answer: A Diff: 1 Page Ref: 851 Objective: 28-10 46) A young female was involved in an altercation and stabbed in the abdomen with an ice pick. After being stabbed, she quickly removed the ice pick. She states that there was not much bleeding but does say it "stings" quite a bit. Assessment reveals a small puncture wound to the lower right quadrant with some dried blood around the site. Which best describes the primary focus of the EMT in caring for this patient? A) Keeping the entry site clean and free of infection B) Assessing for internal blood loss or shock C) Assessing for underlying injury to the spleen D) Preventing air from entering the abdominal cavity Answer: B Diff: 2 Page Ref: 852 Objective: 28-11 47) A patient was assaulted and sustained multiple injuries of various types. Which injury does the EMT recognize as presenting the greatest risk for infection? A) Contusion to the chin B) Closed crush injury to the knee C) Abrasion to the right side of the face D) Hematoma to the left foot Answer: C Diff: 2 Page Ref: 850 Objective: 28-10

13 Copyright © 2018 Pearson Education, Inc.

48) A patient has suffered an open crush injury to his elbow. As you approach him, you note the patient is sitting upright and holding a towel to his elbow. Although the towel is soaked with blood, the elbow does not appear to be actively bleeding. Your first action in caring for this patient should be to: A) Apply supplemental oxygen B) Evaluate the patient's airway C) Place a sterile dressing over the injury D) Examine the patient's elbow Answer: B Diff: 2 Page Ref: 854 Objective: 28-4 49) You are transporting a male patient who was stabbed in the left upper quadrant of the abdomen. On scene, the patient was alert and oriented, with a patent airway and adequate respirations. There was minimal hemorrhage from the stab wound, which was covered with a sterile dressing. Oxygen was applied and transport was initiated. During the reassessment, which assessment finding should the EMT consider to be of the greatest concern? A) Statement that the pain at the injury is increasing B) Observation that blood is beginning to ooze from the injury C) Increased redness and edema to the area surrounding the stab wound D) Increased heart rate and unexplained restlessness Answer: D Diff: 3 Page Ref: 845 Objective: 28-6 50) Which instruction is appropriate when helping a new EMT care for a male patient with a laceration to his left arm? A) "When you are finished cleaning the laceration, apply some sterile gauze soaked in sterile saline." B) "It is best to let the wound bleed some. This is the body's natural way of cleaning the wound." C) "Don't worry about removing embedded dirt from the wound, just the dirt around the laceration." D) "Do not cut away his sleeve over the top of the laceration as it will help control the bleeding." Answer: C Diff: 2 Page Ref: 854 Objective: 28-10 51) A patient has suffered a small but jagged laceration to her left hand. When cleaning the wound, it is important that the EMT: A) Use sterile gauze and wipe in a direction away from the site of injury B) Apply an antibiotic ointment prior to cleaning the injury C) Carefully remove any embedded particles from within the laceration D) Place saline-soaked gauze in the laceration Answer: A Diff: 2 Page Ref: 854 Objective: 28-10 14 Copyright © 2018 Pearson Education, Inc.

52) A 40-year-old homeless male is found lying in the street. The physical examination reveals that the patient has a stab wound to the mid-chest. He is unresponsive and his skin is cool to the touch. His respirations are snoring at 40 breaths/min and his pulse is 120 beats/min. Which care should the EMT provide first? A) Apply a sterile dressing to the wound B) Start positive pressure ventilations C) Perform a jaw-thrust maneuver D) Listen to breath sounds Answer: C Diff: 2 Page Ref: 845 Objective: 28-8 53) The EMT's initial concern when treating a patient with a gunshot wound to the chest is: A) Evaluating for shock B) Ensuring an open airway C) Applying sterile bandages D) Looking for an exit wound Answer: B Diff: 2 Page Ref: 845 Objective: 28-11 54) A patient was building a bomb in his garage when it accidentally detonated. Assessment findings from the rapid secondary assessment reveal part of the intestines and a portion of the liver protruding from the right side of the abdomen. Proper care of this injury would include: A) Careful replacement of the organs back into the abdominal cavity B) Direct but gentle pressure applied to the exposed organs to hold them in place C) Application of a dry sterile dressing covered with an occlusive dressing taped on three sides D) Covering the exposed organs with a large sterile dressing soaked with sterile water Answer: D Diff: 2 Page Ref: 855 Objective: 28-11 55) Which of these patients would the EMT recognize as suffering from an evisceration injury? A) A female with a large flap of skin that has been torn loose from her scalp B) A male patient with a knife impaled in his right upper abdomen C) A male patient with a loop of intestine protruding from an open surgical wound D) A female shot in the chest with "bubbling" coming from the wound as the patient breathes Answer: C Diff: 1 Page Ref: 855 Objective: 28-1

15 Copyright © 2018 Pearson Education, Inc.

56) While riding a bike, a young boy fell onto a stick protruding from the ground. The stick impaled him in the neck. The boy immediately pulled the stick out and ran home, where his mother called 911. Assessment reveals a gaping wound to the right neck. All hemorrhage has clotted off and manual cervical spine motion restriction is being maintained. Which instructions should you provide to the other EMTs on scene? A) "Take some sterile gauze and carefully place it inside the open wound to prevent additional bleeding." B) "Place an occlusive dressing over the wound and tape it on all four sides." C) "Apply a sterile dressing over the wound and hold it in place by wrapping roller gauze around his neck." D) "Do not worry about covering the wound; just place a cervical collar on the patient and carefully transfer him to the long board." Answer: B Diff: 2 Page Ref: 857 Objective: 28-11 57) Which patient requires the EMT to remove an impaled object in the field? A) A young female with a broken pencil through her cheek and into the oropharynx; blood from the injury is flowing into her throat B) A male patient with a 6-inch knife in his left upper quadrant; there is active bleeding around the injury site C) A female with a knife om the right side of her chest; she is short of breath and coughing up blood D) A male patient who fell on a screwdriver, which is impaled through his hand; the patient is in pain and requesting that you remove the screwdriver Answer: A Diff: 2 Page Ref: 856 Objective: 28-11 58) A small metal rod was thrown from a metal lathe while it was in operation and is now impaled in a man's inner thigh. Assessment indicates that there is no active bleeding from the site, but the surrounding tissues are ecchymotic. The paramedic asks you to stabilize the rod with bulky dressings. You recognize this action is important because it will: A) Prevent motion of the rod and further internal injury B) Control any internal hemorrhage that may be occurring C) Prevent bacteria from entering into the body D) Apply pressure to the broken blood vessels in the skin Answer: A Diff: 2 Page Ref: 856 Objective: 28-11

16 Copyright © 2018 Pearson Education, Inc.

59) A patient presents with an arrow impaled in the right lower quadrant of his abdomen. He is in excruciating pain and states that he was accidentally shot by a friend while hunting. While there is no external bleeding coming from the injury, the surrounding skin is ecchymotic. Other assessment findings, such as the vital signs, indicate the patient is in early shock. Which action should the EMT perform first? A) Carefully remove the arrow B) Administer supplemental oxygen C) Stabilize the arrow with an occlusive dressing D) Establish manual cervical spine motion restriction Answer: B Diff: 2 Page Ref: 845 Objective: 28-8 60) An alert and oriented patient has had a portion of his hand amputated by a table saw. Assessment reveals the patient to be in great pain, with a moderate amount of dark red blood still coming from the injury site. His airway is open and his breathing is fast, but adequate. The radial pulse is rapid and strong, and his skin is warm and dry. The patient's amputated hand has been placed in a bag of ice by coworkers. What should the EMT do immediately? A) Insert a nasopharyngeal airway B) Remove the amputated part from the ice C) Control the bleeding with direct pressure D) Obtain a full set of vital signs Answer: C Diff: 2 Page Ref: 856-857 Objective: 28-11 61) A patient has had part of his right thumb amputated in an industrial accident. Coworkers have retrieved the thumb and wrapped it in a towel. The EMT demonstrates appropriate handling of the amputated part when he: A) Places the thumb in ice water B) Keeps the thumb warm during transport C) Places the thumb in sterile saline D) Wraps it in a dry sterile dressing Answer: D Diff: 2 Page Ref: 856 Objective: 28-11 62) A female patient has been involved in a motorcycle crash. Your scene size-up reveals her to be unresponsive and lying in the roadway. It also appears that her left leg has been amputated at the knee. What should the EMT do immediately? A) Assess the leg for bleeding B) Perform a jaw-thrust maneuver C) Cover the knee with a sterile dressing D) Locate the amputated leg Answer: B Diff: 2 Page Ref: 840 Objective: 28-11 17 Copyright © 2018 Pearson Education, Inc.

63) You are called for an alert and oriented female who had a cesarean section several days ago. Today, the patient bent over and tore the surgical incision open. There is a moderate amount of bleeding coming from the site, but otherwise the patient is stable. What should the EMT do to control the bleeding? A) Place a saline-soaked bandage over the top of the incision B) Administer supplemental oxygen C) Apply a multitrauma dressing over the incision D) Pack sterile gauze dressings into the incision Answer: C Diff: 2 Page Ref: 857 Objective: 28-12 64) A 54-year-old male patient has been involved in a car crash. Window glass has caused an open wound to his upper neck. Which dressing is best for this injury? A) Sterile gauze B) Self-adhering roller gauze C) Moist sterile gauze D) Occlusive dressing Answer: D Diff: 2 Page Ref: 857 Objective: 28-12 65) Which statement made by the EMT shows an understanding of how an open wound should be bandaged? A) "Bandages are sterile gauze pads used to stop bleeding." B) "I use bandaging material to secure the sterile dressing in place." C) "You must always make sure that the bandage is sterile." D) "After you apply a bandage, you should place a dressing over it." Answer: B Diff: 2 Page Ref: 857 Objective: 28-12 66) You have just applied a pressure dressing to the leg of a woman who suffered a deep laceration to her calf from a piece of broken glass. What should you do next? A) Assess the blood glucose level to determine if hypoglycemia contributed to the accident B) Obtain a follow-up blood pressure C) Apply a tourniquet above the laceration D) Assess for a pedal pulse Answer: D Diff: 2 Page Ref: 858 Objective: 28-12

18 Copyright © 2018 Pearson Education, Inc.

67) Which statement made by the EMT shows an understanding of dressing and bandaging a wound? A) "When applying a bandage over a dressing, I apply it loosely so as not to interfere with circulation to the extremity." B) "If bleeding from a wound cannot be initially stopped with direct pressure, I apply a bandage and dressing so I can take care of other injuries." C) "I always leave the tips of the fingers or toes exposed when bandaging the arm or leg so future assessment of circulation is possible." D) "I always bandage across the joint above the wound since this helps control bleeding by applying pressure to the proximal blood vessels." Answer: C Diff: 2 Page Ref: 859 Objective: 28-12 68) Which statement about dressings and bandages is true? A) Dressings must be airtight to prevent contamination B) Bandages should not be applied until bleeding is controlled C) Dressings are placed over bandages D) Dressings should be placed into open wounds Answer: B Diff: 2 Page Ref: 858 Objective: 28-12 69) A man has been bitten in his arm by his dog. He states that the bite occurred several hours earlier, when he accidentally stepped on the dog's paw. When asked, he tells you that the dog is up-to-date on all her shots, including the rabies vaccine. Assessment reveals two small puncture wounds to the hand with some bruising in the surrounding tissue. The patient wants to refuse treatment and transport. Given these assessment findings, what is the primary reason this patient should be seen in the emergency department? A) Risk for infection B) Possible shock from blood loss C) Need to complete a police report D) Prevent formation of a contusion Answer: A Diff: 3 Page Ref: 853 Objective: 28-10 70) Which type of injury is most likely to result in capillary bleeding only? A) Avulsion B) Abrasion C) Laceration D) Penetration Answer: B Diff: 1 Page Ref: 47 Objective: 28-10

19 Copyright © 2018 Pearson Education, Inc.

71) A construction worker has a metal rod impaled in his right forearm. Assessment of the injury indicates heavy bleeding from around the impaled object. What should you do first? A) Apply direct pressure around the rod B) Place a tourniquet above the injury site C) Attempt once to remove the metal rod D) Apply pressure on the rod to stabilize it Answer: A Diff: 2 Page Ref: 836 Objective: 28-10 72) What is the best description of a topical hemostatic agent? A) A rapid-acting pill that promotes clotting in the body B) A topical medication that reduces the risk of infection C) A dressing that is specifically designed to stop bleeding D) A liquid that stops bleeding by "gluing" the edges of a laceration together Answer: C Diff: 2 Page Ref: 839 Objective: 28-4 73) A pregnant 31-year-old female has been shot in the neck. Currently, she is responsive to verbal stimuli, with an open airway and rapid, but adequate respirations. Her radial pulse is weak and fast. Vital signs are as follows: pulse 152 beats/min, respirations 22 breaths/min, blood pressure 92/76 mmHg, and SpO2 100% on high-concentration oxygen as initiated by the fire department EMRs. Which other intervention by the EMT will most benefit this patient? A) Apply the pneumatic anti-shock garment (PASG) B) Wait on scene for an ALS paramedic with an ETA of 10 minutes C) Place an occlusive dressing on the neck and keep the patient warm during transport D) Apply a bulky dressing to the neck and transport the patient in a semi-Fowler's position Answer: C Diff: 2 Page Ref: 857 Objective: 28-12 74) A 48-year-old male is in shock from injuries sustained in a motor vehicle collision. Which assessment finding(s) is (are) most suggestive that the patient may have internal bleeding? A) Heart rate of 148 beats/min B) Patient responds to painful stimuli with a moan C) Evisceration of abdominal contents D) Pelvic instability with a BP of 78/48 mmHg Answer: D Diff: 2 Page Ref: 842 Objective: 28-6

20 Copyright © 2018 Pearson Education, Inc.

75) You respond to a single-car accident in which an unrestrained 3-year-old child was thrown from the back seat into the windshield, striking his head. As your partner applies spine motion restriction precautions, you note that the child has a patent airway, is breathing well, and is alert and oriented; however, you also notice that blood and fluid are draining from his right ear. How would you treat the bleeding? A) Pack the external ear canal to control bleeding, being careful not to rupture the eardrum B) Quickly cover the ear with your gloved hand C) Apply a hemostatic agent to the ear canal D) Apply a dressing loosely over the ear to absorb the blood and fluid Answer: D Diff: 2 Page Ref: 841 Objective: 28-5

21 Copyright © 2018 Pearson Education, Inc.

Prehospital Emergency Care, 11e (Mistovich et al.) Chapter 29 Burns 1) A fair-skinned patient was exposed to the sun and has a painful superficial burn to her upper back. As a knowledgeable EMT, you would recognize that which layer(s) of the skin has been injured? A) Epidermal B) Dermal C) Epidermal and dermal D) Subcutaneous Answer: A Diff: 1 Page Ref: 876 Objective: 29-4 2) A 20-year-old waitress spilled hot coffee and burned her right forearm. When assessing the burn, you note a full-thickness burn surrounded by partial-thickness burns. The patient complains of pain in the surrounding partial-thickness burn, but not in the center where the full-thickness burn is located. As an EMT, you recognize that the patient does not localize pain to the center of the burn because of: A) Damage of pain receptors in the epidermis B) Injury to pain receptors in the subcutaneous layer C) Destruction of pain receptors located in the dermis D) Injury of the epidermal nerve endings Answer: C Diff: 1 Page Ref: 877 Objective: 29-4 3) A male patient has just been burned with hot water and has blisters on his left arm. The EMT would recognize that: A) This burn can be classified as superficial B) Fluid has collected between the layers of skin C) Medium and large blood vessels have ruptured D) This burn shows signs and symptoms of infection Answer: B Diff: 1 Page Ref: 877 Objective: 29-3 4) A young female, after being rescued from a burning apartment, is found to have partial- and full-thickness burns from her chest down to her feet. As such, the EMT would recognize the potential for the onset of: A) Hyperthermia B) Hypertension C) Hypoglycemia D) Hypothermia Answer: D Diff: 2 Page Ref: 885 Objective: 29-7 1 Copyright © 2018 Pearson Education, Inc.

5) When classifying a burn as either superficial, partial thickness, or full thickness, the EMT would need to determine the: A) Percentage of body burned B) Time of heat exposure C) Agent causing the burn D) Depth of the burn Answer: D Diff: 1 Page Ref: 876-877 Objective: 29-4 6) When triaging multiple burn patients, the EMT would recognize which patient as having the most critical burn? A) A 41-year-old patient with a partial-thickness burn to the upper left arm B) A 22-year-old patient with a partial-thickness burn to the right hand C) A 58-year-old patient with a scald burn to the anterior chest D) A 12-year-old patient with a superficial burn to the neck and chest Answer: B Diff: 2 Page Ref: 878 Table 29-1 Objective: 29-4 7) A 51-year-old female patient tripped in the kitchen and fell against a deep fryer, splashing hot oil onto her right arm. Assessment reveals partial- and deep partial-thickness burns to her arm. You estimate that the burn covers 4 to 5 percent of her body. Which statement, made by the patient, provides the most important information for determining the overall severity of the burn from a medical standpoint? A) "I had a burn like this several years ago." B) "I think that I lost control of my bladder." C) "I have had diabetes for several years." D) "The doctor said that I scar very easily." Answer: C Diff: 2 Page Ref: 877 Objective: 29-7 8) A 21-year-old male sustained a severe burn to his left leg while working on an industrial furnace. Coworkers immediately doused his flaming pants and removed them. Your primary assessment reveals no threats to the airway, breathing, or circulation. While performing the secondary assessment on his leg, which finding would be most critical? A) Partial-thickness burn completely encircling the lower leg B) Pain described as "the worst I have ever felt" C) Blisters on the lower leg that are broken and draining fluid D) Skin on the posterior lower leg that is sloughing off Answer: A Diff: 3 Page Ref: 879 Objective: 29-4

2 Copyright © 2018 Pearson Education, Inc.

9) A patient has a burn that completely encircles his right forearm and hand. After performing the primary assessment, the EMT should: A) Evaluate and compare grip strength in the right and left hands B) Check for motor status, sensory status, and a pulse in the right upper extremity C) Ask the patient to rate the pain on a scale of 1 to 10 D) Carefully break the blisters to decrease the pressure caused by swelling Answer: B Diff: 2 Page Ref: 879 Objective: 29-8 10) A male patient who works in a steel fabricating plant has suffered a flash burn to his right arm and hand. Which assessment finding would assist the EMT in determining that the burn is superficial and not partial thickness? A) Presence of pain B) Poor capillary refill C) Absence of blisters D) Strong radial pulse Answer: C Diff: 2 Page Ref: 876-877 Objective: 29-4 11) You are called by the owner of a restaurant for a female patron who spilled hot tea onto her right thigh. Your assessment reveals the burn area to be soft, red, and painful. The skin is intact with no blister formation noted, although some edema is present. Based on these assessment findings, you would recognize this injury as belonging to which category of burn? A) Superficial B) Partial thickness C) Deep partial thickness D) Full thickness Answer: A Diff: 2 Page Ref: 876-877 Objective: 29-4 12) A female who had hot coffee thrown at her has a superficial burn to the right side of her face. After transferring the patient to the hospital emergency department for continued care, a new EMT asks you if the patient will have a permanent scar on her face. Which is your best reply? A) "She will probably have some degree of scarring, but it will not be very noticeable." B) "It depends; if the skin peels, she will probably be left with some degree of scarring." C) "Since the burn was superficial, a small skin graft can be used to hide any and all scars." D) "Since only the outer layer of skin was burned, there is little chance of scarring." Answer: D Diff: 3 Page Ref: 876-877 Objective: 29-4

3 Copyright © 2018 Pearson Education, Inc.

13) A patient was burned when he accidentally ignited gasoline while working on his car. When performing your assessment of the injury, which finding would indicate that the patient has suffered a partial-thickness burn? A) Pink skin on the left lower leg that is dry and painful to the touch B) Painful skin on the abdomen that is moist and contains blisters C) Intact skin on the wrist that is red and dry and extremely painful when touched D) Leathery skin on the thigh that exhibits little pain when touched Answer: B Diff: 2 Page Ref: 877 Objective: 29-4 14) You have been called for a 44-year-old male who was burned on the right hand and arm after falling against the hot door of an industrial incinerator. When you arrive on the scene, a nurse employed by the company informs you that he sustained a partial-thickness burn. Based on this information, the EMT should expect which characteristics at the burn site? A) Intense pain and blister formation B) Decreased pain but significant edema C) Leathery skin that remains hot to the touch D) Reddened skin with decreased pain sensation Answer: A Diff: 2 Page Ref: 877 Objective: 29-4 15) You are assessing a patient who was burned when she dropped a hot iron on her bare foot. Her foot is red and has a small partial-thickness burn to the anterior portion of the foot. When completing the prehospital care report, which mechanism of injury would you document? A) Scald burn B) Flame burn C) Electrical burn D) Contact burn Answer: D Diff: 1 Page Ref: 881 Objective: 29-6 16) Which assessment finding would best indicate that a patient has suffered a full-thickness burn to his leg? A) Red skin that is painful B) Moist skin with blisters C) Dry skin with little pain D) Charred skin with intense pain Answer: C Diff: 2 Page Ref: 877 Objective: 29-4

4 Copyright © 2018 Pearson Education, Inc.

17) An intoxicated patient has suffered a burn to his left lateral thigh after passing out with his leg touching the side of a kerosene heater. Close examination of the burn reveals tough leathery tissue in the center of the burn, with red skin with blisters surrounding it. Regarding the burn, what would the EMT agree to be true? A) The central portion of the burn is a full-thickness burn and the surrounding portion is a partial-thickness burn B) The pain is originating from the more severely burned center of the injury C) The blister formation indicates that portion of the burn can be classified as full thickness D) The outer skin surrounding the burn can be medically described as eschar Answer: A Diff: 2 Page Ref: 877 Objective: 29-4 18) A patient has suffered a superficial burn to the chest, arms, and face after pouring gasoline on a smoldering campfire and accidently inhaling the plume of smoke and fire that erupted suddenly. He is now coughing. Which statement indicates that the EMT is properly caring for him? A) "We are going to put ice packs to your chest to cool the burn." B) "I am going to apply a special antibiotic lotion to the burned areas to prevent scarring." C) "Throughout transport, I am going to listen to your lungs quite frequently." D) "I need to clean the dirt from the burned area to prevent an infection." Answer: C Diff: 2 Page Ref: 883 Objective: 29-8 19) Which statement made by the EMT indicates an understanding of the prehospital calculation of body surface area (BSA) burned? A) "The BSA can be quickly estimated before evaluating the status of the airway and breathing." B) "Determining the BSA is an estimation and should never delay transport of the burn patient to the hospital." C) "It is important to get an accurate estimation of the BSA burned since this will affect the care provided by the EMT." D) "It is better to underestimate the BSA of a burn patient rather than overestimate the BSA." Answer: B Diff: 2 Page Ref: 883 Objective: 29-7 20) For which patient would the palm method be best to estimate the body surface area affected by a burn? A) A 21-year-old patient with superficial burns to the entire back and neck B) A 27-year-old patient with full-thickness burns to the backs of the legs and arms C) A 3-year-old patient with partial-thickness burns to the anterior chest and abdomen D) A 49-year-old patient with a partial-thickness burn to the inner thigh Answer: D Diff: 2 Page Ref: 880 Objective: 29-4 5 Copyright © 2018 Pearson Education, Inc.

21) Firefighters have just pulled a young woman from a house fire and have laid her on the lawn. The patient is conscious, oriented, and badly burned on the chest, abdomen, and legs. Her clothes are still smoldering. Which action should the EMT take immediately? A) Place an oropharyngeal airway B) Determine the body surface area burned C) Apply oxygen via nonrebreather face mask D) Pour water on the smoldering clothes Answer: D Diff: 2 Page Ref: 883 Objective: 29-7 22) Of the actions described, which indicates proper care of a patient who has experienced a partial-thickness burn to the hand? A) Gently push the fingers together prior to applying a sterile dressing B) Carefully remove rings prior to applying a dry sterile dressing C) Carefully drain blisters to decrease pressure on the underlying tissues D) Clean dirt and debris from the burn prior to applying an antibiotic ointment Answer: B Diff: 2 Page Ref: 886 Objective: 29-9 23) A patient has full-thickness burns to the fronts of both legs and the entire left arm. Using the rule of nines, the EMT should estimate what percentage of the patient's body has been burned? A) 32 percent B) 45 percent C) 27 percent D) 36 percent Answer: C Diff: 1 Page Ref: 880, Figure 29-4 Objective: 29-4 24) When applying the rule of nines to a 10-month-old child, the EMT should recognize which statement to be accurate? A) The head of a child is considered to be 18 percent of the total body B) Both legs combined are approximately 18 percent of the total body C) The front and back of the torso are considered 72 percent of the total body D) The front of the arm is approximately 9 percent of the total body Answer: A Diff: 2 Page Ref: 880, Figure 29-4 Objective: 29-4

6 Copyright © 2018 Pearson Education, Inc.

25) Which statement shows that the EMT understands the palm method of estimating the BSA burned? A) "I do not use the palm method on young children since it is inaccurate in this population." B) "I use the patient's palm to estimate the size of the burn area." C) "If I estimate that four palm areas fit into the burn, then the burn is about 8 percent BSA." D) "I use the palm of my hand to estimate the area of the burn." Answer: B Diff: 2 Page Ref: 880 Objective: 29-4 26) You have been called for an intoxicated male who sustained a full-thickness burn to his leg after passing out against a kerosene heater. After ensuring that his airway, breathing, and circulation are intact, you cool the burn with water and then: A) Carefully remove smoldering fabric that is adhered to the skin B) Cover the burn with a burn sheet soaked in warm sterile water C) Carefully wrap the burn with a dry sterile dressing D) Apply ice packs and secure in place with a sterile dressing Answer: C Diff: 2 Page Ref: 885 Objective: 29-9 27) A patient has experienced partial- and full-thickness thermal burns to the anterior chest, abdomen, and arms in an industrial accident. Which action would be appropriate in the care of this patient? A) Cool the burns with water and cover the burns with a sterile, dry burn sheet B) Cool the burns with water on scene and during rapid transport to the emergency department C) Cool the burns with water and then cover the burns with saline-soaked dressings soaked in sterile water D) Cool the burns on scene and apply ice packs to the burned areas during transport Answer: A Diff: 2 Page Ref: 884-885 Objective: 29-7 28) A patient was trapped in a burning apartment before being rescued by firefighters. As you approach him, you note that his legs and arms are burned. The patient is screaming in pain and coughing, but no longer actively burning. Given this information, which intervention would you perform first? A) Perform a secondary assessment to determine if nonburn injuries are present B) Estimate the percentage of the patient's body that has been burned C) Determine whether the patient has partial-thickness or deep partial-thickness burns D) Decide whether to administer oxygen with a nasal cannula or nonrebreather mask Answer: D Diff: 2 Page Ref: 884 Objective: 29-7

7 Copyright © 2018 Pearson Education, Inc.

29) A 51-year-old male was burned in the face after a steam valve was accidentally opened at his industrial job. His face is red and swollen, and he is screaming in pain. The patient also states that he is having a hard time seeing. What is the EMT's priority concern in caring for this patient? A) Facial disfigurement B) Airway compromise C) Pain management D) Loss of vision Answer: B Diff: 2 Page Ref: 884 Objective: 29-8 30) You are assessing an elderly male patient who was trapped in a burning trailer. Which finding would be most indicative that he has an inhalation injury? A) Coughing of black sputum B) Heart rate of 126 beats/min C) Partial-thickness burn to the left side of the face D) Blood coming from the nose Answer: A Diff: 2 Page Ref: 884 Objective: 29-8 31) A patient has been pulled from a house fire. Assessment reveals him to be lethargic with stridorous respirations, minimal chest wall motion, and a rapid but strong radial pulse. Firefighters are dousing his body with water to cool him and ensure he is extinguished. Which action would the EMT perform immediately once the fire on the patient is adequately extinguished with water? A) Determine if critical areas of the body have been burned B) Administer oxygen through a nonrebreather face mask and obtain a full set of vital signs C) Begin removing the patient's clothes to determine the type of burn D) Provide positive pressure ventilation with a bag-valve mask Answer: D Diff: 2 Page Ref: 884 Objective: 29-8 32) A male patient was injured when a steam valve opened, causing hot steam to contact his face. Which statement made by the patient would be of most concern to the EMT and require immediate assessment or treatment? A) "I think the blisters are breaking." B) "The pain is excruciating!" C) "It is sort of hard to breathe." D) "I think my neck is also burned." Answer: C Diff: 2 Page Ref: 884 Objective: 29-8

8 Copyright © 2018 Pearson Education, Inc.

33) You have been called for a patient who had a large bag of lime powder tear as he was unloading it from a truck. On your arrival, the patient complains of burning to both legs. You note that, aside from the shorts he is wearing, lime dust has covered both legs. Your immediate action would be to: A) Brush as much of the lime off his legs as possible B) Cut his clothes off to better assess the extent of the burns C) Wash his legs off with large amounts of water D) Move the patient to the stretcher for immediate transport Answer: A Diff: 2 Page Ref: 884 Objective: 29-8 34) A student in a science lab has had an unknown chemical splashed into her eye. After assessing the airway, breathing, and circulation, the EMT would immediately: A) Examine the pupils and structures of the eye for damage B) Identify the chemical and determine whether a neutralizing solution can be used C) Flush the eye with copious amounts of tap water D) Cover both eyes with sterile dressing and provide rapid transport to the hospital Answer: C Diff: 2 Page Ref: 886 Objective: 29-8 35) You are transporting a patient who was shocked by an electrical current while repairing a residential electrical service panel. He has partial-thickness burns to the hand and foot where the electricity entered and exited his body, respectively. When reassessing this patient, which finding would be of most concern? A) Increasing pain at burn sites B) Palpation of an irregular heartbeat C) Complaint of a headache D) Swelling of the foot Answer: B Diff: 2 Page Ref: 888 Objective: 29-8 36) A 44-year-old electrician has been shocked. He now responds to verbal stimuli with garbled speech. His airway is open and he is breathing poorly at a rate of 8 times per minute. His pulse is slow and irregular. Which action would the EMT perform next? A) Attach but do not turn on the automated external defibrillator B) Insert an oral airway and apply 15 lpm oxygen with a nonrebreather mask C) Obtain the patient's pulse, blood pressure, and pulse oximetry reading D) Ventilate with a bag-valve mask at 10 breaths/min Answer: D Diff: 2 Page Ref: 888 Objective: 29-8

9 Copyright © 2018 Pearson Education, Inc.

37) A 24-year-old man was shocked while working on his house's electrical system. Assessment reveals a minor burn to the fingers on his left hand. Aside from being slightly shaky, the patient states he feels fine and does not see the need to be treated further or transported to the hospital. Which statement would you make before allowing him to refuse additional care and transport? A) "You really need to be evaluated in the hospital emergency department. The shock puts you at high risk for a stroke within the next 48 hours." B) "You need to get the burn site cleaned to avoid infection. This can be done in the hospital emergency department." C) "Electrical burns typically increase in pain as time goes on. The hospital emergency department can give you pain medication." D) "It would be best to let the emergency physician check you out. Electrical shocks can damage the heart or other internal structures." Answer: D Diff: 2 Page Ref: 888 Objective: 29-8 38) You have arrived on the scene of a large structure fire. Emergency Medical Responders (EMRs) are by the side of a patient who is responsive to verbal stimuli and has significant burns to his body. They report that the patient fell down a flight of stairs while trying to escape and was trapped for several minutes. Their assessment findings include partial- and full-thickness burns to his right arm, right leg, and right side of the face. The EMRs also report that the patient's unburned skin is cool, clammy, and pale, with a delayed capillary refill. His pulse is 144 beats/min, respirations are 22 breaths/min, and blood pressure is 88/50 mmHg. Which instructions would be most appropriate? A) "The patient is in burn shock; let's cool him by pouring cold water on him now and during transport." B) "After reassessing the ABCs, let's do a secondary assessment and look for signs of injuries that are causing shock." C) "The patient is in burn shock; let's soak a sterile burn sheet in water and wrap it around him." D) "Before cooling the patient with water, let's figure out the percentage of his body that is burned, along with the type of burn." Answer: B Diff: 3 Page Ref: 883 Objective: 29-7 39) Why would an EMT be concerned about a partial-thickness burn circumferentially to the chest? A) Impact on the heart B) Large burn area C) Restriction of breathing D) Lung tissue involvement Answer: C Diff: 2 Page Ref: 879 Objective: 29-1

10 Copyright © 2018 Pearson Education, Inc.

40) Which statement made by an EMT shows that he understands the major cause of death in the prehospital setting for a burn patient? A) "Most burn patients who die before getting to the hospital die of airway or bleeding problems, not the burn itself." B) "Most patients who die before getting to the hospital die from burn shock, caused by massive fluid loss from the burns." C) "The most common cause of death for burn patients in the prehospital setting is infection, which can be minimized by applying a sterile burn sheet." D) "Burn patients most often die before reaching the hospital from the increased body temperature caused by the heat of the burns." Answer: A Diff: 3 Page Ref: 883 Objective: 29-7 41) A 27-year-old male fell down a full flight of stairs while escaping his burning apartment. Which assessment finding in this patient should demand your immediate attention? A) Tenderness to the right upper abdominal quadrant B) Full-thickness burn on the hands C) Burns on the face D) Deformity of the left wrist Answer: C Diff: 2 Page Ref: 883 Objective: 29-8 42) An awake but confused patient with a history of diabetes accidentally spilled a pot of hot water on himself while cooking. He has a partial-thickness burn on his right anterior leg. Following an unremarkable primary assessment, which action seems most appropriate? A) Determine the percentage of body surface area burned B) Check the patient's blood glucose level C) Assess for signs of infection at the burn site D) Apply a wet sterile dressing to the burn Answer: B Diff: 2 Page Ref: 883-884 Objective: 29-7 43) Firefighters have pulled a middle-aged male from a burning bedroom. The patient is unresponsive and has obvious deformity of his left ankle. His respirations are rapid, stridorous, and shallow. Your partner inserts an oropharyngeal airway and begins ventilating the patient with a bag-valve mask appropriately. Which action would you perform next? A) Instruct your partner to hyperventilate the patient B) Find out how long the patient was in the burning structure C) Secure the left ankle to a board splint D) Assess the patient's carotid pulse Answer: D Diff: 2 Page Ref: 883 Objective: 29-7 11 Copyright © 2018 Pearson Education, Inc.

44) An industrial worker sustained partial- and full-thickness burns to his right hand after accidentally touching an electric furnace used to incinerate medical waste. Assessment reveals no threats to the airway, breathing, or circulation. Which action would the EMT perform next? A) Pour cool saline water on the hand B) Apply an antibiotic ointment to the burn C) Open any blisters to relieve pressure on the burned tissue beneath D) Gently clean dead skin away from the burn Answer: A Diff: 2 Page Ref: 883 Objective: 29-7 45) An intoxicated 28-year-old female has suffered burns to both legs after passing out next to a space heater, which ignited her pants. Emergency Medical Responders have removed her clothing and cooled the burns. Which action is most appropriate in the continued care of this patient? A) Carefully wrap the burn with a sterile wet burn sheet B) Cover the burns with a clean, white, dry sheet C) Place ice packs to the burn to continue cooling D) Clean the burn with sterile saline and cover it with gauze Answer: B Diff: 2 Page Ref: 885 Objective: 29-7 46) Which layer of the skin contains the larger blood vessels? A) Hypodermis B) Dermis C) Epidermis D) Connective Answer: A Diff: 1 Page Ref: 874 Objective: 29-2 47) Which statement concerning a flash burn is accurate? A) Clothing will typically protect the skin from a flash burn B) A flash burn is the result of welding without proper protection C) A flash burn is caused by UV radiation D) Turnout gear does not provide adequate protection from a flash burn Answer: A Diff: 1 Page Ref: 883 Objective: 29-5

12 Copyright © 2018 Pearson Education, Inc.

48) What is an advantage of the Lund and Browder system of BSA estimation? A) It is integrated in the Broselow pediatric tape system B) It is calibrated so that the EMT can use his own hand to estimate a small-area percentage C) It uses an age-related chart for more accurate determination D) It is more easily memorized than the rule of nines Answer: C Diff: 1 Page Ref: 880 Objective: 29-4 49) What forms the physical barrier against the entrance of undesirable microorganisms, found in the environment, into the body? A) The skin B) The small intestine C) White blood cells D) Eschar tissue Answer: A Diff: 1 Page Ref: 874 Objective: 29-2 50) The release of myoglobin into the blood, caused by an electrical burn, can result in failure of: A) The circulatory system B) The kidneys C) Nerve endings near the burn location D) White blood cell production Answer: B Diff: 2 Page Ref: 875-876 Objective: 29-3 51) A partial thickness burn covering 8 percent of the body would be considered moderate for which patient? A) A 40-year-old patient with accompanying superficial burns over 20 percent of the body B) A 30-year-old patient with burns to the genitalia C) A 25-year-old patient with sickle cell disease D) A 20-year-old patient with an accompanying high-voltage burn Answer: C Diff: 2 Page Ref: 878, Table 29-1 Objective: 29-5 52) A patient is struck by lightning while running from her car to her house. What would be the primary burn cause? A) Electrical B) Contact C) Flash D) Radiation Answer: A Diff: 1 Page Ref: 882-883 Objective: 29-6 13 Copyright © 2018 Pearson Education, Inc.

Prehospital Emergency Care, 11e (Mistovich et al.) Chapter 30 Musculoskeletal Trauma and Nontraumatic Fractures 1) Which type of muscle tissue is found in the forearm? A) Voluntary B) Involuntary C) Integumentary D) Contraction Answer: A Diff: 1 Page Ref: 897 Objective: 30-2 2) Which bodily activity is a function of a voluntary muscle? A) Contraction of the heart B) Movement of food through the intestines C) Chewing food D) Release of hormones into the bloodstream Answer: C Diff: 1 Page Ref: 897 Objective: 30-2 3) Which statement regarding voluntary muscles is true? A) Voluntary muscles are primarily located on the torso B) Voluntary muscles are composed of less tissue mass compared to involuntary muscles C) Voluntary muscles produce movement by extending in most circumstances D) Voluntary muscles generally connect to the bones of the skeletal system Answer: D Diff: 1 Page Ref: 897 Objective: 30-2 4) A patient has a history of tendonitis to the right elbow. Based on this history, the EMT would understand that the: A) Patient suffered a previous fracture to the right elbow B) Tissue connecting the muscle to the bones of the elbow is inflamed C) Muscles surrounding the elbow are inflamed D) Bones making up the elbow joint are infected Answer: B Diff: 1 Page Ref: 897 Objective: 30-6

1 Copyright © 2018 Pearson Education, Inc.

5) Your partner is off from work for an extended period because he is having surgery for a torn ligament. Which structure is in need of that surgery? A) The thick muscle that surrounds a joint B) Tissue that connects one bone to other bones C) Muscle that connects to a bone D) Connective tissue that connects muscle and bone Answer: B Diff: 1 Page Ref: 897 Objective: 30-2 6) Which bone injury would be said to occur in the appendicular skeletal system? A) Rib fracture B) Spinal fracture C) Right-hip fracture D) Skull fracture Answer: C Diff: 1 Page Ref: 899 Objective: 30-2 7) Which statement about different musculoskeletal injuries is true? A) A sprain is an injury to a joint with possible damage to or tearing of ligaments B) Strains are injuries that occur to ligaments and the joints to which they are attached C) A fracture occurs when a tendon connecting muscle to a bone is overstretched and injured D) A dislocation occurs when the bone contained within a joint is broken Answer: A Diff: 2 Page Ref: 902 Objective: 30-3 8) A patient whom you transported several days ago stops by your station to thank you. He reports that he was diagnosed with a strain to the lower right leg. As such, you recognize which structure as being affected by that injury? A) Joint B) Ligament C) Muscle D) Bone Answer: C Diff: 1 Page Ref: 902 Objective: 30-3

2 Copyright © 2018 Pearson Education, Inc.

9) A young female patient was walking on a 7-foot-high retaining wall when she lost her balance, stumbled, and fell from the wall. She impacted a concrete sidewalk surface with her left foot and left hand. Your assessment reveals ecchymosis, deformity, and edema to both the left foot and the left hand. Based on the concept of injury caused by indirect force, where else should the EMT look for injury? A) Left toes B) Left ankle C) Left hip D) Left fingers Answer: C Diff: 3 Page Ref: 903 Objective: 30-4 10) Which assessment finding, in a 32-year-old female involved in a motor vehicle collision, should the EMT recognize as potentially life threatening? A) Deformity of the left femur B) Open fracture to the right forearm C) Dislocation of two fingers on the left hand D) Crepitus palpated to the right humerus Answer: A Diff: 2 Page Ref: 903 Objective: 30-5 11) The rapid secondary assessment of a male patient who was thrown off a motorcycle indicates instability and pain on palpation to the pelvic area. What is the EMT's primary concern related to this injury? A) Risk for infection B) Internal blood loss C) Severe pain D) Pelvic or hip fracture Answer: B Diff: 2 Page Ref: 903 Objective: 30-5

3 Copyright © 2018 Pearson Education, Inc.

12) You are caring for a male patient who was ejected from the passenger side of a vehicle that was traveling at a high rate of speed when it left the roadway and overturned several times. During your primary assessment, you note the patient to have blood in the airway, rapid and shallow breathing, an absent radial pulse, and a weak and rapid carotid pulse. You also visualize that his left leg is severely injured, with gross visible angulations from suspected multiple fractures. Your partner is triaging the other occupants of the vehicle and you can hear the wail of the arriving fire department vehicles. Given this situation, what should you do first? A) Straighten the leg to minimize internal bleeding and then tend to the ABCs B) Hold manual cervical spine motion restriction until the fire department EMTs arrive at your side C) Initiate positive pressure ventilation without oxygen until the fire department EMTs can bring you an oxygen tank D) Suction the airway with a hand-operated suction machine while trying to minimize cervical spine motion Answer: D Diff: 3 Page Ref: 909, Figure 30-15B Objective: 30-7 13) A football player injured his knee during practice and is in pain. His left knee is swollen, ecchymotic, and flexed in an upward position. Your partner reports that the distal skin is warm, and he has located a weak pedal pulse. Which action should you perform immediately? A) Place the patient on a long spine board and apply the traction splint en route to the hospital B) Realign and straighten the injured knee, and then immobilize it by binding it to the other leg C) Splint the knee in the position found prior to providing transport to the hospital D) Apply a traction splint and straighten the knee until the patient's pain is decreased Answer: C Diff: 2 Page Ref: 909, Figure 30-15B Objective: 30-6 14) Firefighters are extricating the driver of a small car that was struck broadside by a delivery truck at a moderate rate of speed. The patient was unrestrained, and the car sustained severe damage to the driver's side. As the male patient is pulled from the car, you note severe deformity to his left thigh area where an open femur fracture has been bleeding severely. Your primary assessment shows the patient to be responsive to painful stimuli, with an open airway and adequate breathing at 20 breaths per minute. His radial pulse is present but rapid at a rate of 124. After applying oxygen and stopping the leg bleed with direct pressure, the EMT should immediately: A) Examine the left leg B) Check for a pedal pulse in the left foot C) Conduct a rapid secondary assessment D) Apply a traction splint to the left leg Answer: C Diff: 2 Page Ref: 904 Objective: 30-7

4 Copyright © 2018 Pearson Education, Inc.

15) Family members report that their 62-year-old mother complained of dizziness and right leg weakness just before falling down a flight of stairs. Your assessment indicates that the patient is responsive, but confused and talking with garbled speech. Her breathing is adequate and her radial pulse is strong. Vital signs are pulse 84 beats/min, respirations 18 breaths/min, blood pressure 188/110 mmHg, and SpO2 96%. There is an open fracture to her right forearm. When instructed to do so, the patient does not move the fingers of her right hand. When caring for this patient, which sequence of actions seems most appropriate? A) Provide spine motion restriction precautions to include securing her on a long spine board, and then place a splint on her right arm prior to beginning a lights and sirens transport B) Provide spine motion restriction precautions to include a cervical collar, long spine board, and straps before extricating her to the ambulance for immediate transport C) Make sure that a radial pulse is present, and splint the right arm prior to moving the patient to the stretcher and then the ambulance, where she can be placed on a long spine board and provided with other spine motion restriction interventions D) Splint the right arm in the position found and transfer the patient to the stretcher for immediate transport to the hospital without taking spinal motion restriction precautions Answer: B Diff: 2 Page Ref: 906-907 Objective: 30-7 16) A patient was struck in the right upper arm with a baseball bat. Which sign or symptom indicates the highest probability that the humerus has been fractured? A) Crepitus felt on palpation B) Pain to the right upper arm C) Hematoma to the upper arm D) Decreased sensation in the right hand Answer: A Diff: 2 Page Ref: 905 Objective: 30-6 17) The secondary assessment of a patient who complains of right leg pain after falling down several stairs reveals a break in the skin where a fractured tibia bone broke through and then recessed back into the leg. The EMT should recognize this as which type of injury? A) Open fracture B) Laceration C) Closed fracture D) Partial fracture Answer: A Diff: 1 Page Ref: 900 Objective: 30-3

5 Copyright © 2018 Pearson Education, Inc.

18) A 9-year-old boy has fallen from a swing. Assessment findings reveal no threats to the airway, breathing, or circulation, but do indicate deformity, pain, and swelling to his right wrist. The right radial pulse is strong and is accompanied by skin that is pink and warm to the touch. Which action indicates that the EMT is properly caring for this patient? A) Straightening the wrist to promote blood flow to the hand B) Maintaining the wrist below the level of the heart to decrease swelling C) Applying cold packs to the wrist to reduce swelling D) Massaging the wrist gently to decrease the pain Answer: C Diff: 2 Page Ref: 907 Objective: 30-6 19) A young man was riding a motorcycle when he was hit head on by a car and thrown from the bike. Your scene size-up reveals him to be lying supine in the roadway with obvious deformity to his right thigh and left ankle, with blood noted on his pants in these areas. The patient assessment reveals he is unresponsive with snoring respirations and is breathing irregularly at 16 times per minute. His radial pulse is moderate in strength, and his skin is warm and dry. Which action should be performed first by the EMT? A) Open the airway using the head-tilt, chin-lift maneuver B) Expose and splint both legs to prevent further injury C) Check for distal pulses in both lower extremities D) Provide manual cervical spine motion restriction Answer: D Diff: 2 Page Ref: 906 Objective: 30-6 20) A 57-year-old male fell 20 feet while setting up holiday lights on his roof. He impacted the ground feet first and has suffered open fractures to both tibias, with both bones protruding through the skin. Your assessment reveals him to be responsive to painful stimuli. His airway is open, his breathing is rapid, and his radial pulse is weak. The skin is cool and diaphoretic. Vital signs are pulse 132 beats/min, respirations 24 breaths/min, and blood pressure 106/90 mmHg. According to family, the patient has no pertinent medical history. Your partner is providing positive pressure ventilation. At this point in the patient's care, it is a priority for you to: A) Cover the patient with a blanket B) Look for other injuries C) Notify the hospital D) Immobilize the fractures Answer: B Diff: 2 Page Ref: 904 Objective: 30-6

6 Copyright © 2018 Pearson Education, Inc.

21) A patient has an injury to the elbow and forearm. While you are conducting the reassessment, which statement made by the patient should concern you most? A) "It hurts when I move my fingers." B) "My hand feels like it is going to sleep." C) "My arm seems to be bruising more." D) "I am still having a hard time moving my arm." Answer: B Diff: 3 Page Ref: 906 Objective: 30-6 22) A female lifeguard fell 10 feet from her chair and now has deformity, swelling, and ecchymosis to her right lower leg. The leg also is rotated medially. Another lifeguard has taken and is maintaining manual cervical spine motion restriction. As your partner conducts the primary assessment and finds no life threats or disturbances in her mental status, you should perform which action first? A) Check the right foot for pulses and motor ability B) Ready the traction splint for application C) Align the leg in a neutral position D) Manually stabilize the lower right leg Answer: D Diff: 2 Page Ref: 906 Objective: 30-7 23) A patient fell and sustained an open fracture to the left humerus. However, assessment reveals the bone to have pulled back into the arm. Bleeding from the site is controlled. How will splinting the left arm benefit this patient? A) Decrease the opportunity for further injury to nerves and blood vessels B) Start the process of healing by aligning and connecting the bone ends C) Eliminate the possibility of infection through the open wound D) Convert the open fracture to a closed fracture and maintain it as such Answer: A Diff: 2 Page Ref: 907 Objective: 30-11 24) What is an emergency care measure for a patient with a possible bone fracture that can be an effective means to reduce pain? A) Administering oxygen B) Applying warm packs to the fracture site C) Splinting the fracture D) Gently massaging the injury site Answer: C Diff: 1 Page Ref: 907 Objective: 30-9

7 Copyright © 2018 Pearson Education, Inc.

25) An alert and oriented 22-year-old male fell off a tractor and now complains of pain to his right leg. As you quickly scan the patient, you note deformity to the tibia-fibula area of the lower extremity. After conducting the primary assessment and manually stabilizing the leg, which action should the EMT take? A) Apply the traction splint B) Transfer the patient to the stretcher C) Check for a pedal pulse D) Splint the leg in the position found Answer: C Diff: 2 Page Ref: 907 Objective: 30-6 26) A vacuum splint has just been applied to the injured arm of a patient who fell backward from a chair while hanging drapes. Which statement or question should the EMT make or ask next after placing the splint? A) "I am going to feel your wrist for a pulse now." B) "Can you rate your pain for me?" C) "I need to feel your arm for tenderness." D) "I am wrapping the splint with roller gauze to help secure it." Answer: A Diff: 2 Page Ref: 907 Objective: 30-8 27) A patient with a closed fracture to the forearm has been properly splinted when which other structures are properly immobilized? A) Wrist and forearm B) Wrist and elbow C) Elbow and forearm D) Shoulder, elbow, and forearm Answer: B Diff: 2 Page Ref: 907 Objective: 30-11 28) While on standby at a semi-professional baseball game, you are summoned onto the field at home plate for a player who complains of severe leg pain after colliding with the catcher of the opposing team. The primary assessment shows no threats to the airway, breathing, or circulation. The secondary assessment reveals a severely deformed knee that is swollen and ecchymotic. The leg is pale and cool, and the patient cannot move his leg when asked to do so. In addition, you cannot palpate a pedal pulse. What should the EMT's priority action be at this time? A) Apply cold packs to the knee and transport immediately B) Apply a traction splint and enough traction until a pulse returns C) Check the patient's radial or carotid pulse D) Attempt to straighten the leg until a pulse returns Answer: D Diff: 2 Page Ref: 907 Objective: 30-6 8 Copyright © 2018 Pearson Education, Inc.

29) A female patient with osteoporosis stepped sideways on her foot and has suffered an open tibial fracture to the distal portion of her right leg. Which intervention indicates that appropriate care is being given for this injury? A) The protruding bone is gently replaced back under the skin B) The EMT cleans dirt from within the wound and off the bone prior to splinting C) A pressure dressing is tightly applied to the open wound prior to splinting D) The bone end and soft tissue wounds are covered with a sterile dressing Answer: D Diff: 2 Page Ref: 907 Objective: 30-11 30) When using an air splint, the EMT must remember that air splints: A) Should never be applied circumferentially B) Do not fully immobilize a possible fracture C) Can decrease circulation in the extremity D) Are contraindicated for joint injuries Answer: C Diff: 2 Page Ref: 909 Objective: 30-9 31) The greatest benefit of applying a traction splint in the field to a deformed femur and thigh is that it can: A) Initiate proper bone healing B) Decrease the pain C) Increase perfusion to the muscle D) Contract the thigh muscles Answer: B Diff: 2 Page Ref: 909 Objective: 30-9 32) A splint applied too loosely can lead to which complication? A) Laceration of previously intact blood vessels B) Conversion of an open fracture to a closed fracture C) Increased risk for infection D) New fractures along the bone Answer: A Diff: 2 Page Ref: 911 Objective: 30-10

9 Copyright © 2018 Pearson Education, Inc.

33) You arrive at a hunting camp for a hunter who stumbled over a rock and caught himself on his outstretched arm. The patient denies hitting his head or neck. There are no threats to the airway, breathing, or circulation. The patient has swelling and ecchymosis to his left wrist. A radial pulse is readily palpated and the skin in the hand is warm to the touch. Which instructions given by the EMT to other rescuers indicates proper care of this patient? A) "After we splint his wrist, we will need to elevate it during transport to decrease the opportunity for swelling." B) "When you splint that arm, make sure that it is somewhat loose so the patient can move it if he becomes uncomfortable." C) "Make sure to wrap that splint as tightly as you can; we need a lot of pressure to prevent additional swelling." D) "Since he still has a pulse in that arm, the swelling must be from a muscle injury. We can forego the splint and just apply ice packs." Answer: A Diff: 2 Page Ref: 907 Objective: 30-6 34) Which question is best asked when attempting to determine the degree of sensation in the hand of a patient who has an elbow injury? A) "Do you have feeling in your hand?" B) "Does your hand feel as though it is asleep?" C) "Can you tell me which finger I am touching?" D) "Can you tell me if you have paresthesia in your hand?" Answer: C Diff: 2 Page Ref: 912 Objective: 30-8 35) You arrive on the scene to help another crew with the extrication from an attic of a female who fell while retrieving holiday decorations. As a result of the fall, the patient has an open fracture to her left lower leg. The patient has been fully immobilized to a long board and has a rigid splint in place to her left leg. What would indicate the extremity has been improperly splinted? A) The splint has immobilized the ankle, knee, and hip regions B) Ice packs wrapped in towels have been applied to the skin between the splints C) The patient is able to flex her left ankle when directed to do so D) The skin of the foot is red and swollen with a weak pedal pulse noted Answer: C Diff: 2 Page Ref: 907 Objective: 30-10

10 Copyright © 2018 Pearson Education, Inc.

36) A patient has swelling and deformity to the wrist. After splinting, in which position should the hand be placed? A) Fingers curled inward B) Fingers extended and spread C) Hand in a fist with thumb inside fist D) Wrist flexed with fingers extended Answer: A Diff: 2 Page Ref: 912 Objective: 30-6 37) Which statement shows that an EMT understands field care of a patient with a possible joint dislocation? A) "If a joint injury is suspected, ice packs–but not splints–are indicated." B) "The care for a patient with a joint injury is identical to that for a patient with a fracture." C) "If a distal pulse is absent, three attempts to straighten the joint can be made." D) "Warm packs–not cold packs–are indicated for a patient with a possible joint injury." Answer: B Diff: 2 Page Ref: 912 Objective: 30-11 38) An alert and oriented auto mechanic had his right thigh pinned between the bumpers of two cars. Assessment findings include deformity and swelling to the right hip area and mid-thigh, along with ecchymosis and swelling to the right knee. The patient complains of excruciating pain to the right leg. Which set of instructions given to other EMTs on the scene would be appropriate? A) "Let's put his leg in the traction splint to help align the femur and reduce pain." B) "Check for a pedal pulse and then carefully move the patient to the stretcher for transport." C) "We need to wrap the right leg in a pressure dressing and then elevate it for transport." D) "Let's provide spine motion restriction precautions now, get the patient in the ambulance, and then provide more care to the leg en route." Answer: D Diff: 3 Page Ref: 906 Objective: 30-7 39) Which assessment finding would contraindicate the use of the traction splint? A) Deformity to the hip B) Decreased pedal pulse C) Numbness to the foot D) Open fracture Answer: A Diff: 1 Page Ref: 913 Objective: 30-11

11 Copyright © 2018 Pearson Education, Inc.

40) Which statement about traction splints is true? A) "Once the traction splint has been positioned under the leg, and the ankle hitch is applied, manual traction can be released." B) "Mechanical traction should be applied until the affected leg is approximately 2 to 4 inches longer than the unaffected leg." C) "Mechanical traction should be applied until the mechanical traction is equal to the manual traction and the patient experiences a reduction in pain." D) "Once the mechanical traction has been applied, the ischial strap must be released to promote circulation into the affected leg." Answer: C Diff: 2 Page Ref: 919; EMT Skill 30-5H Objective: 30-9 41) Which statement about compartment syndrome is true? A) "It can be treated in the prehospital setting by applying warm packs and elevating the extremity." B) "It is commonly associated with bone fractures and can result in hemorrhagic shock if not treated." C) "It describes swelling in the tissues that can result in the loss of an extremity." D) "It is a life-threatening condition that will rapidly lead to death if not recognized and treated." Answer: C Diff: 3 Page Ref: 914 Objective: 30-12 42) You have been called for a male worker complaining of arm pain. At the scene, the 31-yearold patient states that his arm was crushed between two heavy boxes that shifted while being stacked by a forklift. He had minimal pain at that time, but now his right forearm is swollen and painful, especially with movement. The patient also states that the arm and fingers feel as if they are "asleep." You note decreased strength to the arm and a radial pulse that is weak when compared to the left arm. There is no deformity to the arm and the skin is intact, although hard on palpation. Vital signs are pulse 88 beats/min, respiration 18 breaths/min, and blood pressure 134/76 mmHg. Given these assessment findings, the EMT should be suspicious of: A) Compartment syndrome B) Subclinical fracture C) Hemorrhagic shock D) Ischemic stroke Answer: A Diff: 3 Page Ref: 914 Objective: 30-12

12 Copyright © 2018 Pearson Education, Inc.

43) A patient with diagnosed osteoporosis is more likely to sustain which type of bone injury? A) Strain B) Joint sprain C) Nontraumatic dislocation D) Fracture Answer: D Diff: 1 Page Ref: 914 Objective: 30-13 44) An elderly female patient fell and now complains of right hip and right knee pain. When assessing her, which finding would you find most concerning? A) Swelling in the right knee B) Absent pedal pulse C) Crepitus at the right hip D) Ecchymosis in the right hip and knee Answer: B Diff: 2 Page Ref: 904 Objective: 30-8 45) A 67-year-old female complains of severe left lower leg pain. She states that she was walking across the room when she suddenly heard a "crack" and felt excruciating pain in her leg. Her left tibia appears to be deformed and ecchymotic, as well as tender on palpation. Her vital signs are pulse 96 beats/min, respirations 28 breaths/min, blood pressure 168/74 mmHg, and SpO2 98%. Additionally, the patient states that her doctor told her that the steroids she takes can weaken her bones, placing her at greater risk for fractures. Appropriate care for this patient would include: A) High-concentration oxygen by mask B) Full immobilization to the long board C) Direct pressure to the area of injury D) Application of a splint to the leg Answer: D Diff: 2 Page Ref: 914 Objective: 30-13 46) While evaluating a patient who experienced a fall, you note crepitus in the left forearm in the area of the wrist. Crepitus in the wrist is typically caused by: A) Tearing of muscles near the site B) Nitrogen bubbles in the joints C) Subcutaneous air pockets D) Bone ends grinding together Answer: D Diff: 2 Page Ref: 905 Objective: 30-1

13 Copyright © 2018 Pearson Education, Inc.

47) A 13-year-old boy flipped his bicycle and landed on outstretched arms, which caused a fracture to his collar bone. This is an example of: A) A secondary fracture B) A comminuted fracture C) Direct force injury D) An indirect force injury Answer: D Diff: 2 Page Ref: 903 Objective: 30-4

14 Copyright © 2018 Pearson Education, Inc.

Prehospital Emergency Care, 11e (Mistovich et al.) Chapter 31 Head Trauma 1) A woman strikes her forehead against the dashboard of her car as it strikes a tree. Immediately following the impact, her brain shifts back and forth within her skull. Based on the anatomy and physiology of the skull, which statement is true? A) The ridges of the basilar skull can damage the brain as it moves back and forth B) The frontal bone is the thinnest portion of the skull and will most likely be fractured C) The patient's brain will not be injured unless an object from the dashboard penetrates her skull D) The patient's brain will most likely not be damaged because the skull's flexibility is designed to absorb the force of the impact Answer: A Diff: 2 Page Ref: 928 Objective: 31-6 2) You suspect that an unhelmeted male patient who was thrown from a motorcycle may have a basilar skull fracture. As you perform the secondary assessment, which finding would reinforce this suspicion? A) Blood coming from both the ears and the nose B) Dilation and sluggish response of the right pupil to light C) Clear fluid coming from the right ear and the left nostril D) Paralysis of the left arm and leg Answer: C Diff: 2 Page Ref: 928 Objective: 31-5 3) A construction worker was accidentally shot with a nail gun. The nail penetrated the skull approximately ½ inch through the temporal region. Before your arrival, coworkers pulled the nail free. Based on the mechanism of injury, which sign provides the strongest evidence that the meningeal layers have been breached? A) Blood draining from the puncture wound B) Crepitus of the skull at the injury site C) Deformity and depression at the injury site D) Cerebrospinal fluid oozing from the puncture wound Answer: D Diff: 2 Page Ref: 928 Objective: 31-3 4) A middle-aged male died immediately after a self-inflicted gunshot wound to the head. Which portion of the central nervous system was most likely damaged to cause such a rapid death? A) Cerebrum B) Brainstem C) Cerebellum D) Meninges Answer: B Diff: 2 Page Ref: 930 Objective: 31-2 1 Copyright © 2018 Pearson Education, Inc.

5) You are assessing a patient who had a previous brain injury in which a portion of the cerebellum was destroyed. In relation to this specific injury, which finding would the EMT expect? A) Inability to remember information such as past medical history B) Poor muscle coordination when signing his name to the prehospital care report C) Problems with the regulation of heart rate and blood pressure D) Inability to move or feel sensations in one side of his body Answer: B Diff: 3 Page Ref: 930 Objective: 31-2 6) A patient who fell out of a tree has an open skull injury. What would be an appropriate way to describe and document this injury on the PCR? A) Instability to the temporal region of the skull with no break in the overlying scalp B) Crepitus to the back of the skull with a hematoma overlying the unstable area C) Laceration to the top of the scalp with pain to the skull when palpated D) Obvious deformity and instability to the right side of the skull with a laceration to the overlying scalp Answer: D Diff: 2 Page Ref: 930 Objective: 31-4 7) Your assessment of a patient who was involved in a motor vehicle collision reveals a deformity to the left side of the head underneath the hair. On further inspection, you note that the skin overlying the deformity is still intact. Based on these assessment findings, what should be the EMT's greatest concern? A) Possible brain injury B) Fracture of the skull C) Potential for infection of the brain D) Soft tissue trauma to the scalp Answer: A Diff: 3 Page Ref: 930 Objective: 31-5 8) A 41-year-old male patient was struck in the head with a metal pipe during a fight with an angry neighbor. The scene is safe, and as you approach the patient, you note that he is combative and has blood on the left side of his head and face, and on his shirt. His breathing appears to be labored and he is incontinent of urine. Which care should you perform immediately? A) Check his pupils for equality and reactivity B) Apply oxygen at 15 lpm via nonrebreather mask C) Take manual cervical spine motion restriction D) Expose the chest to look for possible bleeding Answer: C Diff: 2 Page Ref: 934-935 Objective: 31-8

2 Copyright © 2018 Pearson Education, Inc.

9) A 61-year-old male fell off a roof. Your primary assessment findings include unresponsiveness, agonal breathing, and a slow and weak radial pulse. The patient's skin is cool and dry. Emergency Medical Responders are maintaining manual spine motion restriction procedures. What should you do immediately? A) Size and apply a cervical collar B) Place the patient in the shock position C) Start positive pressure ventilation D) Apply oxygen via a nonrebreather mask Answer: C Diff: 2 Page Ref: 935 Objective: 31-8 10) A patient who was involved in an altercation was struck on the side of the head with a baseball bat, in addition to suffering several blows to the arms and legs. When assessing this patient, which sign or symptom best indicates the patient has suffered a brain injury? A) Confusion and combativeness B) Complaint of a headache C) Ecchymosis to the side of the face D) Weakness to both arms Answer: A Diff: 2 Page Ref: 935; Figure 31-6 Objective: 31-8 11) A 21-year-old male has a gunshot wound to the head. As you assess him, which finding would you recognize as a nonpurposeful response to pain? A) Garbled and incomprehensible speech when you call his name B) Flexing his arms across his chest when you pinch his shoulder C) Attempting to move his hand away from you as you pinch it D) Exhibiting no response to any stimuli Answer: B Diff: 2 Page Ref: 936 Objective: 31-1 12) Which patient has an isolated brain injury that should be considered the most serious? A) A 31-year-old male who fell from a roof and who is now nonverbal and extends his arms and legs when you pinch the muscles on his shoulder B) A 62-year-old female who fell and hit her head and who is becoming more confused and combative C) A 53-year-old female who crashed her bicycle and who has a large laceration that is bleeding heavily on the back of her head D) A 21-year-old male wearing a deformed motorcycle helmet who does not respond to verbal or painful stimuli after crashing Answer: D Diff: 3 Page Ref: 937; Table 31-1 Objective: 31-9

3 Copyright © 2018 Pearson Education, Inc.

13) You are assessing the pupils of a patient who hit his head after falling from the top of a tractor trailer. Which pupillary finding suggests a closed head injury? A) The right pupil constricts when light is shined into the left pupil B) Both pupils dilate when light is shielded from the eyes C) Both the left and right pupils constrict vigorously in response to light D) The left pupil constricts to light but the right pupil does not Answer: D Diff: 3 Page Ref: 938 Objective: 31-9 14) A 68-year-old female patient complains of a headache and generalized weakness. Her husband informs you that she was with a friend yesterday and was involved in a motor vehicle collision. She did strike her face on the dashboard, but refused treatment on scene. Her past medical history includes a stroke with right arm weakness and high blood pressure, for which she takes medications. When assessing this patient, which finding should concern the EMT most? A) Contusion to her left cheek area B) Weakness to the right-hand grip C) Bruising behind her left ear D) Complaint of pain when she moves her jaw Answer: C Diff: 3 Page Ref: 939 Objective: 31-7 15) The EMT is properly assessing for sensory function in the hands when he tells or asks the patient: A) "Squeeze my fingers with both hands." B) "Can you tell me which finger I am touching?" C) "Do you feel me touching your right hand?" D) "Can you wiggle your fingers?" Answer: B Diff: 2 Page Ref: 939 Objective: 31-9 16) A patient was ejected from a car when it overturned at a high rate of speed. The patient is unresponsive. The primary assessment is complete and you are providing positive pressure ventilation with supplemental oxygen. The patient has a significant deformity to the right side of his head, and both pupils are equal but slow to react to light. The patient also has deformity to the forearms. Vital signs are pulse, 144 beats/min; respirations, 24 breaths/min and inadequate; blood pressure, 90/50 mmHg; and SpO2, 97%. Based on this presentation, the EMT should: A) Look for evidence of bleeding in an area of the body other than the brain B) Reassess the pupils and treat the patient for a severe head injury C) Splint both arms and reassess the vital signs prior to departing the scene D) Check the blood glucose level and prepare for possible seizure activity Answer: A Diff: 3 Page Ref: 940 Objective: 31-8 4 Copyright © 2018 Pearson Education, Inc.

17) When assessing a patient with a head injury, which finding is most indicative of increasing pressure within the skull? A) Blood pressure of 192/106 mmHg B) Heart rate of 132 beats/min C) Respirations of 24 to 28 breaths/min D) Pulse oximeter reading less than 95% on room air Answer: A Diff: 2 Page Ref: 940 Objective: 31-7 18) A woman fell from a second-story window onto the concrete sidewalk below. She is unresponsive and has a large depression to the back and top of her skull. Additional findings include abdominal bruising and an angulated left ankle. Your partner reports that the patient's vital signs are pulse, 68 beats/min; respirations, 14 breaths/min and irregular; blood pressure, 198/112 mmHg; and SpO2, 91% on room air. Based on these assessment findings, what should the EMT suspect is occurring? A) Shock caused by bleeding within the head B) Shock caused by bleeding, most likely in the abdomen C) Severe head injury with increasing pressure within the skull D) Past medical history of untreated hypertension Answer: C Diff: 2 Page Ref: 940 Objective: 31-7 19) A young female was thrown from a horse and is now confused. Assessment findings include an open airway, adequate breathing, and a strong radial pulse. Her vital signs are normal. Which question would be the most important to ask the patient's mother, who was riding with the patient, as soon as possible? A) "Does she have any medical problems?" B) "Are all of her immunizations up-to-date?" C) "Is she allergic to anything?" D) "Did she lose consciousness?" Answer: D Diff: 2 Page Ref: 940 Objective: 31-9

5 Copyright © 2018 Pearson Education, Inc.

20) A 52-year-old male is unresponsive and has bruises and lacerations to the head, chest, and abdomen. Bystanders state that the patient was intoxicated and got into a fight with several patrons of a bar. They state that he was beaten with his opponents' fists, but not other objects or weapons. The patient has snoring respirations, shallow breathing, and a strong radial pulse. His skin is warm and dry. After providing manual spine motion restriction, what should be your next action? A) Insert a nasal or oral airway B) Perform the jaw-thrust maneuver C) Start positive pressure ventilation D) Apply a cervical collar Answer: B Diff: 2 Page Ref: 935 Objective: 31-8 21) A young female patient involved in a motor vehicle collision responds to painful stimuli by extending her arms and legs. Her airway is open, and her breathing is shallow and irregular. A radial pulse that is moderate in strength is palpated. Her pulse is 64 beats/min, respirations are 8 breaths/min and irregular, and blood pressure is 210/110 mmHg, with an SpO2 of 90% on room air. The patient has an obvious deformity to her forehead and a left pupil that is dilated and does not respond to light. Manual spine motion restriction is being maintained by Emergency Medial Responders (). At this time, the EMT's priority action is to: A) Apply a nonrebreather face mask with high-concentration oxygen B) Start positive pressure ventilation at 20 breaths/min C) Perform the secondary assessment to identify additional injuries D) Apply a cervical collar and transfer the patient to the stretcher for immediate transport Answer: B Diff: 2 Page Ref: 935 Objective: 31-8 22) At a construction site, an 18-year-old male was struck in the head by a pallet of concrete blocks as it was being moved by a crane. He has a large laceration to the left side of his head, with noted instability to the skull beneath the wound. Moderate bleeding from the injury site is also noted. The patient's airway is open, his breathing is adequate, and his pulse is strong and regular. Proper care for this patient should include: A) Hyperventilation with high-concentration oxygen B) Direct pressure to control bleeding from the injury C) Rapid transport in an upright position D) A loose dressing over the top of the laceration Answer: D Diff: 1 Page Ref: 942 Objective: 31-8

6 Copyright © 2018 Pearson Education, Inc.

23) A male soccer player was struck in the head with a soccer ball. Players state that he was dazed for several seconds following the impact and then asked the same questions over and over. Presently, he is conscious and oriented to person, but confused about place and time. He also has a reddened area to the side of his head and face. As you proceed with your assessment, his memory continues to improve. Based on these findings, the EMT should suspect which type of injury? A) Concussion B) Open head injury C) Epidural hematoma D) Cerebral contusion Answer: A Diff: 3 Page Ref: 932 Objective: 31-7 24) A young male patient was running through a park shortly before dawn and struck his head on a low-hanging branch of a tree. Emergency Medical Responders have already provided spine motion restriction precautions and are providing supplemental oxygen since the patient was initially unresponsive. During transport, which finding obtained during your reassessment would best indicate that the patient has experienced a concussion? A) Unequal grip strength B) Persistent confusion C) Improving memory D) Elevated blood pressure Answer: C Diff: 2 Page Ref: 932 Objective: 31-7 25) You have been called to the local community hospital for a 48-year-old male who was involved in a motor vehicle collision 2 hours prior. He has been diagnosed with a cerebral contusion and must be transported to a trauma center for specialty care. As a knowledgeable EMT, you recognize that a cerebral contusion is: A) An open skull fracture with increased pressure in the skull B) The formation of a pocket of blood within the brain tissue C) Bruising and swelling of the brain tissue D) Active bleeding between the brain and the skull Answer: C Diff: 3 Page Ref: 933 Objective: 31-7

7 Copyright © 2018 Pearson Education, Inc.

26) You have been called by family members for their mother, who is "not acting right." At the scene, the family informs you that they are concerned because their 68-year-old mother has been complaining of a headache for two days and is now very confused. When asking about a bruise on the right side of her forehead, the family states she fell in church a week ago and hit her head. Since all other aspects of the assessment are unremarkable, you suspect a head injury. Which type of injury would you suspect given the findings and history? A) Subdural hematoma B) Epidural hematoma C) Cerebral concussion D) Cerebral contusion Answer: A Diff: 3 Page Ref: 933 Objective: 31-7 27) A patient complaining of a headache states that he fell and struck the back of his head. Which statement by the patient should be of most concern to the EMT? A) "I lost my balance after stumbling on the rug." B) "I take a blood thinner for my heart." C) "I had a mini-stroke (TIA) five years ago." D) "I had my appendix out two weeks ago." Answer: B Diff: 2 Page Ref: 933 Objective: 31-8 28) A trauma surgeon informs you that the patient whom you transported to the emergency department earlier was diagnosed with an epidural hematoma. The patient was taken into surgery and the hematoma was removed, and now the patient is in critical condition. Based on this information, the EMT should recognize that in this patient: A) Blood from a ruptured vein collected between the dura mater and brain tissue B) Cerebrospinal fluid accumulated in a pocket on top of the brain tissue C) Blood from a ruptured artery formed a hematoma within a section of brain tissue D) A pocket of arterial blood collected between the skull and dura mater Answer: D Diff: 3 Page Ref: 934 Objective: 31-3 29) A young adult female was struck by a vehicle while she was riding her bicycle. She was not wearing a helmet. She is now unresponsive and has a deformity with a deep depression to the temporal area of the head. You do not see any bleeding, and the patient's scalp is intact. What is the greatest threat associated with this injury? A) Fracture of the temporal bone B) Damage to the brain C) Potential for infection D) Loss of cerebrospinal fluid Answer: B Diff: 2 Page Ref: 931 Objective: 31-7 8 Copyright © 2018 Pearson Education, Inc.

30) An unhelmeted rider, who was thrown from his motorcycle when he wrecked, has a large scalp avulsion with obvious skull depression to the top of his head with venous bleeding. These findings suggest which type of injury? A) Intracranial hematoma B) Linear skull fracture C) Open head injury D) Basilar skull fracture Answer: C Diff: 2 Page Ref: 932 Objective: 31-5 31) Which finding, when considered individually, is most consistent with a basilar skull fracture? A) Bruising behind the ear B) Bilateral constricted pupils C) A hematoma over the frontal skull bone D) Altered mental status Answer: A Diff: 2 Page Ref: 930 Objective: 31-5 32) Which finding indicates that a patient who received a blow to the head is suffering from something other than a simple concussion? A) He cannot remember what happened B) He asks the same questions over and over C) His heart rate is 92 beats/min and his blood pressure is 144/86 mmHg D) His pupils are noticeably unequal when assessed Answer: D Diff: 2 Page Ref: 938 Objective: 31-7 33) You have been called to a residence of a patient with diabetes who exhibits an altered mental status. A family member states she could not reach the patient by telephone, so she came over and found the patient awake but confused. The patient can remember his name and address, but cannot remember the day or year. He refuses to go to the hospital, but consents to an assessment. Of these physical exam findings discerned on assessment, which one is most concerning? A) Bruise to the left temple B) Blood glucose level of 77 mg/dL C) SpO2 of 95% on room air D) Heart rate of 82 beats/min Answer: A Diff: 2 Page Ref: 938 Objective: 31-8

9 Copyright © 2018 Pearson Education, Inc.

34) A 57-year-old man tripped and fell on the sidewalk, hitting his head on the concrete. According to witnesses, the patient was unresponsive for several minutes following the fall. Which bit of information would be most critical to relay to the physician in the emergency department? A) Knee replacement two years ago B) Known history of alcoholism C) Position in which the patient was found D) Treatment provided by on-scene witnesses Answer: B Diff: 3 Page Ref: 933 Objective: 31-8 35) An intoxicated teenage male with a history of diabetes fell down five stairs and is now responding to a shoulder pinch with garbled speech. Manual spine motion restriction precautions are being maintained by fire department EMRs. The primary assessment reveals an open airway, adequate breathing, and strong radial pulse. Vital signs are pulse 122 beats/min, respirations 18 breaths/min, blood pressure 108/60 mmHg, and SpO2 97% on room air. The patient's skin is warm and dry. Which action would be performed next? A) Obtain another set of vital signs B) Administer oral glucose C) Start hyperventilation D) Perform a secondary exam Answer: D Diff: 2 Page Ref: 938 Objective: 31-8 36) A young adult male is in police custody after he crashed his car into a utility pole. There is minor front-end damage to the vehicle, and the air bags deployed. The patient was not wearing a seat belt and has an abrasion to his forehead. He is awake and oriented to person, time, and place. His speech is slurred and he has an odor resembling that of beer on his breath. The arresting officer states that he just wants you to check the patient before he transports him to jail. Given this scenario, which action seems most appropriate for the EMT to take? A) Contact medical direction with assessment findings B) Have the patient sign a refusal-of-care form if no threats exist C) Release the patient to the police since he is alert and oriented D) Have the officer sign a refusal-of-care form Answer: A Diff: 2 Page Ref: 942 Objective: 31-8

10 Copyright © 2018 Pearson Education, Inc.

37) An unrestrained middle-aged female was thrown from her car as it rolled in the median of an interstate. She is supine, appears unresponsive, has blood on her face, and has snoring respirations. Your first action upon reaching her side, after taking spine motion restriction precautions, should be to: A) Determine the Glasgow Coma Scale score B) Perform the jaw-thrust maneuver C) Insert an oropharyngeal airway D) Find the source of bleeding and apply direct pressure Answer: B Diff: 2 Page Ref: 941 Objective: 31-8 38) What best describes the purpose of determining a Glasgow Coma Scale (GCS) score in a patient with a head injury? A) The GCS score gives information about the type of injury to the brain B) The GCS score allows the EMT to predict the patient's likelihood of recovery C) The GCS score is important in guiding the EMT's treatment of the patient D) The GCS score helps determine whether a patient's mental status is improving or deteriorating Answer: D Diff: 2 Page Ref: 937-938 Objective: 31-9 39) A young female pedestrian was hit by a car and thrown 15 feet, striking her head against a metal guardrail. You start your secondary assessment and note a deep depression to the right parietal area of her head, with intact skin overlying the area. You should: A) Note the injury and continue assessing the patient B) Apply an ice pack to the injured area C) Gently palpate the depression to determine the possibility of a skull fracture D) Instruct your partner to hyperventilate the patient with a bag-valve mask Answer: A Diff: 2 Page Ref: 938 Objective: 31-8 40) What is considered to be a normal finding when assessing a patient with a possible head injury? A) The left pupil dilates when light is shined into the right pupil B) The right pupil constricts when light is shined into the left pupil C) Both the left and right pupils dilate when light is shined into them D) The right pupil dilates and the left pupil constricts when light is shined into the left eye Answer: B Diff: 2 Page Ref: 938 Objective: 31-9

11 Copyright © 2018 Pearson Education, Inc.

41) A 40-year-old male fell 20 feet from a ledge while hiking. The park ranger is now maintaining manual spine motion restriction for the patient. The patient is combative and confused, but his airway is open and his breathing is adequate. The patient has a radial pulse of 112 beats/min, blood pressure of 96/76 mmHg, and SpO2 of 91% on room air. What should the EMT do first? A) Check the patient's blood glucose level B) Place a cervical collar and secure the patient to a long board C) Begin administering supplemental oxygen D) Perform a jaw-thrust maneuver or a head-tilt, chin-lift maneuver Answer: C Diff: 2 Page Ref: 935 Objective: 31-8 42) A 14-year-old male fell 10 feet from a retaining wall and hit his head on a metal post. He is responsive to verbal stimuli with incomprehensible speech. The secondary assessment indicates blood and fluid coming from inside the patient's left ear. Given these assessment findings, the EMT should: A) Tilt the backboard to the left to allow the ear to drain freely B) Place a folded piece of gauze in the ear canal to stop the drainage C) Suction the blood from the ear so you can determine the source of bleeding D) Place a piece of sterile gauze over the ear to catch the fluid Answer: D Diff: 2 Page Ref: 942 Objective: 31-8 43) A young adult male has a gunshot wound to the left side of his head. He is unresponsive with snoring respirations. He is breathing 6 times per minute with a pulse rate of 52 beats/min and a blood pressure of 192/104 mmHg. His radial pulse is strong and his skin is cool, but not diaphoretic. In addition to spine motion restriction precautions, the first intervention for this patient would be: A) Administering high-concentration oxygen via a nonrebreather mask B) Inserting an oropharyngeal airway C) Performing a jaw-thrust maneuver D) Administering positive pressure ventilation Answer: C Diff: 2 Page Ref: 941 Objective: 31-8 44) Which statement best describes the technique to be used when hyperventilating a patient with a head injury who shows signs of brain herniation? A) Provide 1 ventilation every 3 seconds B) Provide 30 ventilations per minute C) Provide 1 ventilation every 5 seconds D) Provide 16 ventilations per minute Answer: A Diff: 2 Page Ref: 941 Objective: 31-8 12 Copyright © 2018 Pearson Education, Inc.

45) The cerebellum is described as the area of the brain that: A) Is responsible for most conscious and sensory functions B) Consists of two hemispheres, each containing five distinct lobes C) Controls reflexes and assists in maintaining body posture D) Controls the body functions most essential and vital to life Answer: C Diff: 1 Page Ref: 930 Objective: 31-1 46) You are assessing a patient who has sustained blunt trauma to the head. Which sign would be most indicative of the loss of cerebrospinal fluid? A) Red-tinged fluid in the ear canal B) Light bruising around both eyes C) Clear fluid coming from the nose D) Dark bruising on the forehead Answer: C Diff: 2 Page Ref: 928 Objective: 31-8 47) You are transporting a patient with a past medical history of a meningeal tumor. You would recognize this kind of tumor as being located: A) Within the cerebrum B) In tissue surrounding the brain C) Within the bones of the skull D) In tissue composing the brainstem Answer: B Diff: 2 Page Ref: 928 Objective: 31-2 48) Which statement is true regarding linear skull fractures? A) Linear skull fractures are rare and infrequently encountered B) Linear skull fractures are identifiable by a mild depression in the skull C) Linear skull fractures place the patient at risk for blood loss and hypoperfusion D) Linear skull fractures cannot typically be identified with palpation Answer: D Diff: 1 Page Ref: 930 Objective: 31-5

13 Copyright © 2018 Pearson Education, Inc.

49) You are transporting a patient who experienced a right-sided head injury after a large refrigerator fell and pinned his head between the appliance and the floor. On scene, the patient was responsive to verbal stimuli with the following vital signs: pulse, 96 beats/min; respirations, 16 breaths/min; blood pressure, 134/66 mmHg; and SpO2, 95%. During transport, as you reassess the patient, what would be the clearest sign that the head injury is worsening? A) Increasing edema to the right side of the head B) Blood pressure of 108/68 mmHg C) Responding only to painful stimuli D) Heart rate of 112 beats/min Answer: C Diff: 2 Page Ref: 936 Objective: 31-10 50) The parameters of the Glasgow Coma Scale are: A) Eyes, motor, and verbal response B) Vital signs and mental status C) Mental status, vital signs, and verbal response D) Verbal and motor response Answer: A Diff: 1 Page Ref: 937 Objective: 31-9 51) When using the Glasgow Coma Scale (GCS), the EMT understands that: A) The score identifies the type of head injury sustained B) The higher score, the better the patient's neurologic status C) The score must be combined with vital signs D) A lower score numerically indicates less severity, just like the Revised Trauma Score (RTS) Answer: B Diff: 2 Page Ref: 938 Objective: 31-9 52) At an industrial complex, a young male was struck in the head by a large piece of steel thrown from a grinding machine. Assessment reveals a soft, painful depression underlying a tear in his scalp, which is covered with dried blood. Clear fluid is draining from his right ear. The patient is conscious but confused and exhibits the following vital signs: pulse, 84 beats/min; respirations, 16 breaths/min and adequate; blood pressure, 142/76 mmHg; and SpO2, 90%. Emergency Medical Responders are on scene and are maintaining manual spine motion restriction precautions. Which instruction indicates the next step needed in this patient's care? A) "Let's pack his right ear with gauze to stop the loss of fluid." B) "Place manual, direct pressure over the injury to control bleeding." C) "I need someone to check the clear fluid for glucose." D) "Will someone administer supplemental oxygen to the patient? Answer: D Diff: 2 Page Ref: 935 Objective: 31-4 14 Copyright © 2018 Pearson Education, Inc.

53) The normal response of the pupils while examined with a penlight is described as: A) Dilative B) Diplopia C) Conjugate D) Consensual Answer: D Diff: 1 Page Ref: 938 Objective: 31-9 54) A male patient in his thirties fell 10 feet off a loading dock, landing on his head and back. He has deformity and depression to the back of his head and is unresponsive with snoring respirations. Vital signs are pulse, 132 beats/min; respirations, agonal; blood pressure, 136/64 mmHg, and SpO2, 89%. The patient responds to painful stimuli with purposeful motion. Which instruction would you provide other caregivers at the scene? A) "I need someone to start positive pressure ventilation at 12 breaths per minute." B) "The patient is herniating; start hyperoxygenation of the patient immediately." C) "Let's try a nonrebreather mask with high-concentration O2 to increase his oxygen saturation." D) "The airway must be manually opened with a head-tilt, chin-lift maneuver immediately!" Answer: A Diff: 3 Page Ref: 935 Objective: 31-8 55) A patient exhibits retrograde amnesia when she: A) Cannot remember falling and hitting her head B) Responds purposely to painful stimuli C) Is foggy about events occurring after the injury D) Is nonverbal following a head injury Answer: A Diff: 1 Page Ref: 941 Objective: 31-7 56) A patient with Cushing reflex is most likely experiencing which pathophysiology? A) Concussion B) Hypovolemia C) Brain herniation D) Shock Answer: C Diff: 1 Page Ref: 931 Objective: 31-9

15 Copyright © 2018 Pearson Education, Inc.

57) As brain herniation occurs, the body responds by: A) Increasing systolic blood pressure in an attempt to maintain perfusion to the brain B) Increasing the heart rate to more rapidly circulate oxygen-rich blood to the brain C) Decreasing blood pressure to decrease pressure within the skull, thereby improving blood flow D) Discharging cerebrospinal fluid to decrease pressure within the skull and improve perfusion Answer: A Diff: 2 Page Ref: 940 Objective: 31-6

16 Copyright © 2018 Pearson Education, Inc.

Prehospital Emergency Care, 11e (Mistovich et al.) Chapter 32 Spinal Trauma and Spine Motion Restriction 1) Which statement is most accurate regarding the processes needed for a patient to clench the hand into a fist? A) The central nervous system must send a message through the spinal cord and then to the peripheral nerves B) The central nervous system must send a message through the spinal cord and then to the autonomic nervous system C) The peripheral nervous system must send a message to the brain and then back through the peripheral nervous system D) The muscles must signal the brain to send a message through the autonomic nervous system Answer: A Diff: 3 Page Ref: 948 Objective: 32-2 2) Based on the structure of the nervous system, which statement is true? A) A stroke represents an injury to the peripheral nervous system B) A gunshot wound to the spinal cord will directly damage the peripheral nerves C) A deep laceration to the arm can sever peripheral nerves D) Numbness in a fractured leg indicates damage to the autonomic nervous system fibers Answer: C Diff: 2 Page Ref: 948 Objective: 32-2 3) The EMT is caring for a patient who has a problem with the autonomic component of his nervous system. Which sign or symptom would most likely be caused by this condition? A) Inability to swallow B) Slowing of the heart rate C) Inability to move facial muscles D) Trouble remembering his name Answer: B Diff: 2 Page Ref: 948 Objective: 32-2 4) You have been called for a patient with severe back pain. When you are obtaining a medical history, the patient tells you that he has a history of damaged disks in his vertebrae. As a knowledgeable EMT, you should recognize that: A) The ligaments on the side of the spinal column were overstretched B) The vertebrae making up the spinal column were fractured C) The nerves making up the spinal cord have been damaged D) The fluid-filled structures between the vertebrae have been injured Answer: D Diff: 1 Page Ref: 949 Objective: 32-3

1 Copyright © 2018 Pearson Education, Inc.

5) An Emergency Medical Responder reports that a male patient, who was injured while playing football, has bruising to the lumbar region of the body. Based on this statement, the EMT should expect to find bruising in which area? A) Upper back B) Lower neck C) Lower back D) Buttocks Answer: C Diff: 1 Page Ref: 949 Objective: 32-2 6) A 42-year-old man has called 911 because of severe back pain. He informs you that his back pain is related to a recent fracture of his coccyx. Based on this information, where should the EMT expect the patient to be complaining of pain? A) Neck B) Upper back C) Pelvic area D) Lower back Answer: C Diff: 1 Page Ref: 949 Objective: 32-2 7) In which area of the spinal column do the ribs originate? A) Cervical spine B) Thoracic spine C) Lumbar spine D) Sacral spine Answer: B Diff: 2 Page Ref: 949 Objective: 32-2 8) A patient who was injured in a motor vehicle collision has a separation of the rib and its associated spinal vertebrae. Based on the anatomy of the spine, where has this injury occurred? A) Lumbar spine B) Cervical spine C) Middle spine D) Thoracic spine Answer: D Diff: 1 Page Ref: 949 Objective: 32-2

2 Copyright © 2018 Pearson Education, Inc.

9) Which injury is possible based on the anatomy of the spine? A) Fracture to the ninth cervical vertebra B) Dislocation of lumbar vertebra number 6 C) Disk injury between thoracic vertebrae 11 and 12 D) Compression fracture to lumbar vertebra 12 Answer: C Diff: 1 Page Ref: 949; Figure 32-2 Objective: 32-2 10) You arrive on the scene of a motor vehicle collision. Walking toward you is the unrestrained driver of the vehicle that sustained moderate front-end damage. The patient complains of some back pain, but is walking around after the crash and does not appear to be suffering from any neurologic deficits. Although he answers your questions appropriately, he keeps asking you, "What happened?" What is your initial action in caring for this patient? A) Perform the primary assessment B) Obtain the patient's vital signs C) Look for injuries to the patient's head D) Initiate spine motion restriction precautions Answer: D Diff: 2 Page Ref: 959 Objective: 32-9 11) A young intoxicated male patient cannot move his left arm and leg after diving into the shallow end of a pool and hitting the bottom head first. The EMT would recognize which mechanism as most likely responsible for this injury? A) Compression B) Rotation C) Distraction D) Penetration Answer: A Diff: 1 Page Ref: 950 Objective: 32-3 12) A young female driver, who was involved in a motor vehicle collision, complains of cervical pain resulting from a lateral-type mechanism of neck injury. Based on this information, which type of collision most likely took place? A) The patient's car was struck from behind B) The patient's car was struck from the side C) The patient's car struck a utility pole head-on D) The patient's car rear-ended another car Answer: B Diff: 2 Page Ref: 952 Objective: 32-3

3 Copyright © 2018 Pearson Education, Inc.

13) You arrive on the scene of a two-car motor vehicle collision. The patient was the unrestrained driver of a car that struck another car from behind while moving at a speed of 25 mph. During the collision, the patient flew forward and struck the windshield with his head. No air bags were deployed. The patient extricated himself and denies head, neck, or back pain; however, you do note a small cut on his hand, which occurred as he was getting out of the car. When asked about preexisting medical problems, he tells you that he did have herniated disks in his lumbar spine that were surgically repaired several years ago. Based on this information, your strongest reason to initiate spine motion restriction precautions for this patient would be: A) History of back surgery B) Laceration to the hand C) Self-extrication of the patient D) Mechanism of injury Answer: D Diff: 2 Page Ref: 965 Objective: 32-10 14) You have arrived at the scene of a shooting where a middle-aged male sustained a single gunshot wound to the abdomen. As you approach the patient, you note that he is sitting up and attempting to talk with the police officers while holding a blood-soaked towel over the left upper quadrant of his abdomen. Once you get beside the patient, you realize he is speaking in a different language that neither you nor the officers understand. Your initial action in caring for this patient is to: A) Initiate spine motion restriction precautions B) Determine the need for airway management C) Expose the abdomen to assess the gunshot wound D) Obtain a pulse rate, respiratory rate, blood pressure, and SpO2 Answer: A Diff: 2 Page Ref: 965 Objective: 32-10 15) While performing the primary assessment on a patient with an isolated spinal cord injury, you note that he is in severe respiratory distress and struggling to breathe. Where should you suspect the spinal cord injury has occurred? A) Diaphragm B) Cervical spine C) Thoracic spine D) Lumbar spine Answer: B Diff: 3 Page Ref: 956 Objective: 32-7

4 Copyright © 2018 Pearson Education, Inc.

16) As you approach a motorcyclist who was thrown from his bike, you hear him saying that he cannot feel or move his legs. You note obvious deformity to both femurs as well as to his left wrist and forearm. Emergency Medical Responders are holding manual cervical spine motion restriction and have already assessed the PMS in the extremities. Given this information, which assessment should be done next? A) Check for motor function and sensation in the feet B) Expose the legs and look for bleeding C) Check the patient's rate and effort of breathing D) Apply the pulse oximeter Answer: C Diff: 2 Page Ref: 956 Objective: 32-8 17) Which sign or symptom best indicates the patient has experienced a spinal cord injury to the thoracic spine? A) Bruising to the back B) Paralysis of the arms only C) Tingling in the legs D) Cool and diaphoretic skin Answer: C Diff: 2 Page Ref: 952 Objective: 32-7 18) Assessment of a young girl who was hit by a car while riding her bike reveals her to be responsive to painful stimuli with flexion of the extremities; she is also in respiratory distress. There is marked deformity to her thoracic spine and bruising noted to her anterior chest and abdomen. She does not move her legs when a noxious stimulus is applied to the lower extremities. Manual cervical spine motion restriction is being maintained and a cervical collar has been applied by fire department EMRs. Given the critical nature of this patient, which action is most appropriate for her care? A) Rapidly transfer her to the stretcher and secure her to the long board while en route to the hospital B) Transfer her to the long board already placed on the stretcher and secure with straps en route to the hospital C) Quickly but carefully provide full spine motion restriction precautions on scene prior to rapid transport to the hospital D) Wait for family members to arrive and give consent for treatment prior to moving her to the stretcher for immediate transport Answer: C Diff: 2 Page Ref: 959 Objective: 32-11

5 Copyright © 2018 Pearson Education, Inc.

19) An elderly patient fell down a flight of stairs and now complains of neck and back pain as well as weakness to both legs. The primary assessment reveals no life threats to the airway, breathing, or circulation. Manual cervical spine motion restriction is being maintained. What should the EMT do next? A) Apply high-concentration oxygen and move the patient to the stretcher for transport B) Complete the secondary assessment, looking for additional injuries C) Place an oral airway and begin positive pressure ventilation D) Place a cervical collar and secure the patient to the long spine board Answer: B Diff: 2 Page Ref: 956 Objective: 32-8 20) Which response shows that the EMT is correctly assessing motor function in the arms of a patient with a potential spine injury? A) "Can you tell me which finger I am touching?" B) "I need you to flex both arms across your chest." C) "I am going to move your arm; tell me if it hurts." D) "I am going to feel for a pulse in your wrist." Answer: B Diff: 2 Page Ref: 957 Objective: 32-7 21) A 25-year-old male jail inmate was pushed over the railing of a walkway 20 feet above the ground. He is unresponsive and has an open fracture of the left upper arm. How would you determine if the patient has sensation in his legs? A) Since the patient is unresponsive, this assessment would be impossible B) Check for the presence and strength of a pedal pulse C) Pinch his foot and look for movement on the leg D) Lift his leg and look for a facial grimace Answer: C Diff: 2 Page Ref: 957 Objective: 32-7 22) A patient dove into a shallow pool and struck his head on the bottom. Your assessment findings indicate that he has no motor or sensation in his legs, but can move his arms. The EMT should recognize this condition to be: A) Paraplegia B) Quadriplegia C) Hemiplegia D) Tetraplegia Answer: A Diff: 1 Page Ref: 957 Objective: 32-7

6 Copyright © 2018 Pearson Education, Inc.

23) A patient was struck in the back by a heavy piece of wood that was being bent to make a form for a concrete arch. He is responsive, but cannot feel or move his legs. He has bruising to his back and is incontinent of urine. The skin below the injury site is red and warm. As a knowledgeable EMT, you should realize that: A) The paralysis is permanent B) The ability to feel will return C) A cervical collar is unnecessary D) The paralysis may resolve Answer: D Diff: 3 Page Ref: 952 Objective: 32-7 24) A patient is lying under a tree after being involved in a motor vehicle collision. She states that immediately after the crash, she felt okay, but now her legs are numb and tingling. She also has lower back discomfort. Which question is it most important for the EMT to ask next? A) "Do you think that you had a seizure?" B) "Are you allergic to any medications?" C) "How did you get out of the car?" D) "Do you have a history of back problems?" Answer: C Diff: 2 Page Ref: 955 Objective: 32-7 25) When performing the secondary assessment, which sign is most suggestive of a spinal cord injury? A) Pain in the right leg B) Headache and nausea C) Bruising on the back D) Persistent penile erection Answer: D Diff: 2 Page Ref: 959 Objective: 32-7 26) Which statement about the care and treatment of a patient with a spinal injury in the prehospital setting is true? A) Prehospital care for the patient with a spine injury involves spine motion restriction precautions and the identification of life-threatening conditions B) It is important to identify the level of spinal injury so the proper prehospital care can be rendered C) Before transporting the critically injured patient with a spinal injury, the EMT must perform a detailed, head-to-toe neurologic assessment D) If a patient has a possible spine injury but is in shock, it is permissible to forego spine motion restriction precautions since this takes time and keeps the patient from definitive care in the hospital Answer: A Diff: 3 Page Ref: 956-957 Objective: 32-8 7 Copyright © 2018 Pearson Education, Inc.

27) A restrained driver was in a car that struck another car from behind at a low rate of speed. The driver has a bruise to her forehead and complains of tingling in her left leg. She is lying on the side of the road, and another EMT has established manual cervical spine motion restriction precautions. A quick scan of her as you approach indicates that the patient is in little to no distress. What should you do first? A) Check her breathing and radial pulse B) Perform a secondary assessment C) Palpate her cervical spine and apply a cervical collar D) Administer high-concentration oxygen Answer: A Diff: 2 Page Ref: 956 Objective: 32-8 28) A patient who was involved in a diving accident is found with his head positioned so that his left cheek is touching his left shoulder. He states that he has severe pain in his neck and it hurts to move his head. He denies numbness, tingling, or decreased strength in his arms or legs. What is the EMT's best course of action? A) Carefully straighten the head and neck so that a cervical collar can be placed B) Stabilize and maintain the head in the position in which the patient is holding it C) Straighten his neck, and position the patient on his left side on a long spinal board D) Avoid palpation of the cervical spine and apply a short vest-type device Answer: B Diff: 3 Page Ref: 960 Objective: 32-11 29) A male driver was ejected from his vehicle after it rolled several times at a high rate of speed. As you approach the patient, you note that he is unresponsive and struggling to breathe. He also has a laceration to the left side of his face and multiple contusions to his legs. After assigning another EMT to take manual cervical spine motion restriction, what should you do? A) Start positive pressure ventilation B) Insert an oropharyngeal airway C) Apply a properly sized cervical collar D) Open the airway using the jaw-thrust maneuver Answer: D Diff: 2 Page Ref: 960 Objective: 32-8 30) A patient complains of back pain and numbness to both legs after being thrown from a bicycle. When should the EMT first check the motor function, sensory function, and pulses in the legs of this patient? A) Immediately after taking manual spine motion restriction precautions B) Immediately after applying a cervical collar C) During the primary assessment D) As the secondary assessment is performed Answer: D Diff: 2 Page Ref: 956-957 Objective: 32-8 8 Copyright © 2018 Pearson Education, Inc.

31) A patient complains of lower back pain after falling down five stairs. Assessment of the patient's cervical spine reveals no displacement, tenderness, or instability. The patient does state that each leg has feelings of "electrical shocks" shooting through them. How should the EMT care for this patient? A) Release manual spine motion restriction precautions B) Apply a properly sized cervical collar after initiating manual spine motion restriction C) Secure the patient to a backboard without a cervical collar D) Inform the team that spine motion restriction precautions are not needed Answer: B Diff: 2 Page Ref: 959 Objective: 32-10 32) Which instruction would you provide to your team immediately after securing a patient with head, neck, and back pain to the long backboard with appropriate spine motion restriction equipment? A) "Loosen the collar so you can palpate the back of the neck." B) "Maintain manual cervical spine motion restriction until the patient is on the stretcher." C) "Let's check for extremity PMS before we move the patient to the stretcher." D) "Remove the chest strap so the patient can breathe more easily now that his head and legs are secured." Answer: C Diff: 2 Page Ref: 960 Objective: 32-11 33) A teenage boy was found by friends as he attempted to hang himself in a garage using chains suspended from the ceiling. As you approach, you note that the patient is conscious, struggling significantly to breathe, is ashen in color around the mouth and to the hands, and has contusions and swelling that encircle his neck from the chains. After taking manual cervical spine motion restriction and opening the airway, your next action would be to: A) Start positive pressure ventilation B) Size and apply a cervical collar C) Perform a secondary assessment D) Log roll the patient onto a long spine board Answer: A Diff: 2 Page Ref: 960 Objective: 32-8

9 Copyright © 2018 Pearson Education, Inc.

34) While transporting a 38-year-old female who fell from a second-story window, you note that she is becoming confused and her pulse rate is increasing. Additionally, her blood pressure has dropped and her skin is now cool and clammy. Based on these assessment findings, you would recognize: A) Spinal shock B) Hypovolemic shock C) Neurogenic shock D) Head injury Answer: B Diff: 3 Page Ref: 960 Objective: 32-8 35) Which of the instructions from one EMT to another shows the correct application of a cervical collar? A) "Carefully flex his head forward a little so I can pass the collar underneath his neck." B) "Let's log roll the patient to one side so I can apply the cervical collar." C) "Keep his head in neutral position while I apply the cervical collar." D) "I need you to extend the patient's chin backward a little so I can fit the collar under his chin." Answer: C Diff: 2 Page Ref: 969 Objective: 32-11 36) When applying a cervical collar to a young boy who fell out of a tree, the EMT realizes the main purpose of the collar is to: A) Prevent the head and neck from moving B) Maintain spine motion restriction C) Increase the blood flow to the spinal cord in the cervical region D) Remind the patient not to move his head or neck Answer: D Diff: 3 Page Ref: 966 Objective: 32-11 37) You have been called for an 87-year-old male who fell in his kitchen while making breakfast. The patient states that he hit his forehead but managed to catch himself on the way down and did not hit the ground "too hard." Your assessment reveals him to have kyphosis of the thoracic and cervical spine, but no neurologic deficits. His legs, however, "feel weaker." Which action is appropriate given these assessment findings? A) Avoid application of a cervical collar due to the natural deformity of the cervical spine B) Disregard application of a cervical collar since the patient has no neurologic deficits C) Apply a cervical collar and prepare the patient for any discomfort that the collar may cause D) Place a soft cervical collar instead of the rigid cervical collar Answer: C Diff: 2 Page Ref: 965 Objective: 32-11

10 Copyright © 2018 Pearson Education, Inc.

38) When it is necessary to secure a patient to a long spine board during a spine motion restriction process, which of these regions is typically secured last to the spine board? A) Chest region B) Pelvic region C) Abdominal region D) Head and neck region Answer: D Diff: 1 Page Ref: 969 Objective: 32-11 39) A patient is being extricated from a car using a short vest-type device. After the patient has been extricated, the EMT should: A) Remove the vest-type device and secure the patient to a long backboard B) Secure the patient with the vest-type device to a long backboard C) Place the patient in semi-Fowler's position on the stretcher for transport to the hospital D) Secure the patient with the vest device in a supine position on the stretcher with the legs flexed Answer: B Diff: 2 Page Ref: 971 Objective: 32-11 40) You are securing a patient to the long backboard. Of the options listed, which has the elements of securing the patient in the correct order? A) Chest and pelvis, head, legs B) Head, chest and pelvis, legs C) Head, legs, chest, pelvis D) Legs, head, chest and pelvis Answer: A Diff: 1 Page Ref: 969 Objective: 32-11 41) A patient has just been log rolled and positioned onto the long backboard. Which care measure should be performed next, assuming it was not previously completed? A) Apply a properly sized cervical collar B) Secure the patient's head C) Secure the patient's torso with straps D) Release manual cervical spine motion restriction Answer: C Diff: 2 Page Ref: 969 Objective: 32-11

11 Copyright © 2018 Pearson Education, Inc.

42) You are maintaining manual cervical spine motion restriction for a patient who is being log rolled, transferred, and secured to a long backboard. At which point will you release the manual spine motion restriction hold? A) Once the patient has been log rolled onto the long backboard B) After the patient has been properly secured with straps to the long board C) After the patient has been secured to the backboard and transferred to the wheeled stretcher D) After the chest has been secured with a head immobilization device Answer: B Diff: 2 Page Ref: 983 Objective: 32-11 43) The "EMS Management of Patients with Potential Spinal Injury" document published by the American College of Emergency Physicians in 2015 discusses which principle regarding the immobilization of patients? A) The use of a long spine board without a cervical collar is recommended because cervical collars do not achieve a full degree of immobilization B) Current out-of-hospital management of potential spinal injury lacks evidentiary scientific support C) A review of the related literature has demonstrated clinical support for ongoing use of spine motion restriction equipment in all patients with either a minor or major mechanism of injury D) Only a licensed and board-certified emergency department physician is capable of determining whether spine motion equipment is necessary Answer: B Diff: 2 Page Ref: 964 Objective: 32-9 44) Which statement shows that the EMTs are correctly using a vest-type device while removing a patient from an entrapped vehicle location? A) The cervical collar is applied after the torso and legs have been secured to the device B) The head is secured first, followed by the torso and legs C) The head is secured to the device immediately after the cervical collar is placed D) The head is secured to the device after the torso has been secured Answer: D Diff: 2 Page Ref: 971 Objective: 32-13 45) Which of the following findings in a patient with a traumatic mechanism of injury is sufficient to prevent the EMT from "clearing the spine" and necessitate the implementation of spine motion restriction precautions? A) GCS score of 12 B) Blood glucose of 90 mg/dL C) Pulse oximeter of 96% on 2 lpm oxygen D) Pulse pressure of 24 mmHg Answer: A Diff: 2 Page Ref: 965 Objective: 32-6 12 Copyright © 2018 Pearson Education, Inc.

46) A minivan has struck a utility pole. The driver is unresponsive and has life-threatening injuries. Which option would be most appropriate when extricating the patient from the vehicle? A) Apply a vest-type immobilization device B) Perform rapid extrication with a cervical collar applied C) Pull the patient from the car and provide spine motion restriction precautions away from the vehicle D) Remove the patient and place him on a long spinal board in the ambulance during rapid transport Answer: B Diff: 2 Page Ref: 971-972 Objective: 32-13 47) Which device can be used to replace the long spine board when providing full spine motion restriction precautions to an adult? A) Folding stair chair B) Full vacuum mattress C) Half spine board D) Full-body air splints Answer: B Diff: 1 Page Ref: 966 Objective: 32-11 48) You pull up to the scene of a single-car motor vehicle collision. Emergency Medical Responders (EMRs) are maintaining inline manual cervical spine motion restriction of the 56year-old driver, who was unrestrained when she struck a tree at a high rate of speed. As you approach the vehicle, you note that the patient appears unresponsive, with blood coming from her nose and ears. Which instruction to the EMRs is most appropriate? A) "Let me conduct the primary assessment and place a cervical collar on the patient, and then we can start to get her out of this car." B) "Let me conduct the primary assessment, and then we can place a vest-type extrication device on the patient to get her outside the vehicle." C) "After I do the primary and secondary assessment, we will need to get the patient onto the stretcher for immediate transport." D) "Let's suction the airway and get the patient out of the car fast; we can put on a cervical collar once we are outside the vehicle." Answer: A Diff: 2 Page Ref: 955-956 Objective: 32-8

13 Copyright © 2018 Pearson Education, Inc.

49) Which statement about removing a helmet in the prehospital setting is true? A) Helmets should be removed only if they are too tight or if spine motion restriction will be required B) It is acceptable to leave the helmet on a patient if the patient has no airway or breathing problems C) Any patient wearing a helmet should have it removed so the airway and breathing can be properly assessed D) Since helmets should never be removed, the EMT must be creative in providing care around the obstacle of a helmet Answer: B Diff: 3 Page Ref: 972 Objective: 32-14 50) A motorcyclist wearing a full helmet was thrown from his motorcycle after hitting a patch of oil on the road. The bike had been traveling at a high rate of speed. Manual spine motion restriction is being maintained by an off-duty EMT. The primary assessment reveals the patient to be responsive to painful stimuli and breathing rapidly with slight sonorous airway sounds. His radial pulse is weak and fast. As you quickly scan his body, you note deformity to the left femur and lower leg. Your immediate action should be to: A) Apply a cervical collar B) Place a nonrebreather oxygen mask C) Remove his helmet D) Expose his left lower extremity Answer: C Diff: 2 Page Ref: 972 Objective: 32-14 51) A high school football player was hit from behind and now complains of severe back pain and numbness to his right leg. Manual spine motion restriction is being maintained by an assistant coach. The trainer has already removed the face mask from the helmet. Once at the patient's side, what should you do next? A) Remove the shoulder pads as you prepare to place the patient on a backboard B) Carefully remove the helmet C) Apply a cervical collar D) Assess the airway and breathing Answer: D Diff: 2 Page Ref: 972 Objective: 32-8

14 Copyright © 2018 Pearson Education, Inc.

52) A football player was struck in the head during a hard tackle, and is reportedly demonstrating retrograde amnesia to the event. Currently the patient complains of a headache and nausea and reports that he has pain to his upper back and tingling in his left hand. The EMT is providing proper care for the patient when he: A) Removes the helmet to immediately assess the patient's head B) Leaves the helmet in place but removes the face mask first C) Removes the shoulder pads to apply a cervical collar D) Does not place the patient on a long board due to the presence of shoulder pads Answer: B Diff: 2 Page Ref: 973-974 Objective: 32-14 53) A 2-year-old boy fell down a flight of stairs, is crying loudly, and is very difficult to communicate with. Because you are unable to clear the spine, you elect to initiate spine motion restriction precautions. Which instruction would you provide to other EMTs who are initiating this care for the patient? A) "After he is on the board, place a towel behind his head to keep the airway open." B) "It is better if the collar is a little too big; that is more comfortable for him." C) "Let's place a folded towel under his shoulders and back to help maintain head alignment." D) "Use an adult 'no neck' collar since those fit almost everyone." Answer: C Diff: 2 Page Ref: 995 Objective: 32-11 54) A window washer fell 20 feet from scaffolding while washing the windows on an office building. He is alert and oriented, but states that he cannot move or feel his legs. Additionally, because his blood pressure is 72/48 mmHg, you suspect spinal shock. Which other assessment finding reinforces your suspicion of spinal shock? A) Warm and dry skin B) Heart rate of 144 beats/min C) Pulse pressure of 24 mmHg D) SpO2 of 92% Answer: A Diff: 3 Page Ref: 953 Objective: 32-8

15 Copyright © 2018 Pearson Education, Inc.

55) A male patient fell 20 feet from a cliff to a trail below while hiking with his girlfriend. The primary assessment shows him to be confused, with an open airway and shallow breathing. His pulse is 72 beats/min and his blood pressure is 78/50 mmHg. The skin is warm and flushed. The patient has no motor ability or sensation in his legs. What is the most likely cause of this patient's presentation? A) Internal bleeding B) Spinal shock C) Hemorrhagic shock D) Hypoglycemia Answer: B Diff: 3 Page Ref: 953 Objective: 32-1 56) A male patient experienced an injury that completely severed his spinal cord in the thoracic spine. Which presentation would be associated with this injury? A) He will have no sensation in his arms B) He will feel pain below the injury in his legs C) He will have decreased motor ability above the injury D) He will experience paralysis to the legs Answer: D Diff: 3 Page Ref: 952 Objective: 32-7 57) The EMT should recognize a possible spinal column injury with no spinal cord involvement when he discovers which assessment finding? A) Tenderness to the thoracic spine with intact motor and sensory abilities to each extremity B) Intact motor ability to all four extremities with loss of sensation to the legs C) Pain to the cervical spine with loss of sensation to the right arm and leg D) Intact sensation to all four extremities with loss of motor ability to the arms Answer: A Diff: 3 Page Ref: 952 Objective: 32-4 58) The EMT is properly assessing for a spinal cord injury when she: A) Asks the patient if he has pain anywhere along his spinal column B) Palpates the entire spinal column for tenderness C) Asks the patient to spread his fingers apart on both hands D) Checks for distal pulses in all four extremities Answer: C Diff: 2 Page Ref: 957 Objective: 32-4

16 Copyright © 2018 Pearson Education, Inc.

59) A 43-year-old male fell from a roof and cannot move or feel his arms or legs. When assessing the patient, which sign would lead the EMT to suspect the patient is developing spinal shock? A) Cool and moist skin B) Heart rate of 62 beats/min C) Cyanosis to the fingertips D) Seizure activity Answer: B Diff: 3 Page Ref: 953 Objective: 32-4 60) A patient was ejected from a car during a rollover collision at a moderate to high rate of speed. Which presentation in this patient is most characteristic of the incomplete spinal cord injury referred to as a "central cord syndrome"? A) Loss of motor function to the arms with intact motor function to the legs B) Loss of motor function and sensation to one side of the body only C) Loss of sensation to one side of the body, and loss of motor function to the opposite side of the body D) Loss of perfusion to the spinal cord that results in complete loss of motor function and sensation to both the arms and legs Answer: A Diff: 2 Page Ref: 953 Objective: 32-5 61) An elderly patient fell down a flight of basement stairs and is found at the bottom by family members about 20 minutes later. Your assessment reveals that the patient cannot feel painful stimuli to his hips and legs, nor can he move his legs, but he can feel you lightly touching the skin of his legs. Which type of injury may this be? A) Posterior cord syndrome B) Lateral cord syndrome C) Central cord syndrome D) Anterior cord syndrome Answer: D Diff: 2 Page Ref: 954 Objective: 32-5 62) At a scene where a vehicle was involved in a single-car accident, you find the driver walking around at the scene. As you approach, the driver states that he felt fine when he exited the vehicle but is now concerned that his legs feel like they are "going to sleep." What is your first step in treating the driver? A) Place a backboard behind the driver, against his spine B) Assess his airway and circulation C) Assist him in sitting on the cot D) Instruct him to hold his head in a neutral inline position Answer: D Diff: 2 Page Ref: 968 Objective: 32-12 17 Copyright © 2018 Pearson Education, Inc.

63) An injury to a hemisection of the spinal cord that disrupts the spinal tracts on only one side of the cord is: A) Posterior cord syndrome B) Anterior cord syndrome C) Central cord syndrome D) Brown-Séquard syndrome Answer: D Diff: 3 Page Ref: 954 Objective: 32-1 64) Research by the American College of Surgeons Committee on Trauma has shown a higher death rate for patients who were immobilized to a backboard when: A) The injury was the result of penetrating trauma to the head or torso B) The patient was intoxicated at the time of injury C) The injury resulted from a lateral impact automobile accident D) The patient was ambulatory upon EMS arrival Answer: A Diff: 2 Page Ref: 954-955 Objective: 32-6 65) Prior to the revisions in prehospital spinal assessment and care, the primary criterion for determining the need for spine motion restriction was: A) The mechanism of injury B) Evidence of multisystem trauma C) Decided by medical command D) A patient history of kyphosis Answer: A Diff: 2 Page Ref: 964-965 Objective: 32-9 66) Which statement is most accurate concerning spine motion restriction of an ambulatory patient? A) Self-restriction is not permitted if a rigid cervical collar is used B) If the patient is reliable, he can self-restrict while the cervical collar is applied C) If the patient is unconscious, self-restriction cannot be used D) The patient should be reminded to continue self-restriction while being seated on the stretcher Answer: B Diff: 2 Page Ref: 968-969 Objective: 32-12

18 Copyright © 2018 Pearson Education, Inc.

Prehospital Emergency Care, 11e (Mistovich et al.) Chapter 33 Eye, Face, and Neck Trauma 1) When assessing a male patient who was punched in the face by another man, you detect blood in the white portion of his eye. Which optical structure has been injured? A) Sclera B) Cornea C) Lens D) Retina Answer: A Diff: 1 Page Ref: 999 Objective: 33-1 2) A patient complaining of eye pain informs you that he was diagnosed with a corneal abrasion three days ago. He called 911 today because the discomfort has not yet resolved and he desires medical attention. Based on this information, which statement is most accurate? A) The sclera must be infected B) The lens of the eye has been abraded C) The clear structure that covers the pupil has been damaged D) Fluid is leaking from the posterior chamber Answer: C Diff: 1 Page Ref: 999 Objective: 33-1 3) Which statement made by an EMT indicates an understanding of facial anatomy and/or injuries? A) "The face has little blood supply but will readily swell when injured." B) "Airway compromise is not a concern unless the neck is also injured." C) "Injury to the eye(s) takes precedence over any and all other injuries." D) "The only movable bone making up the face is the mandible." Answer: D Diff: 1 Page Ref: 1000 Objective: 33-2 4) A 21-year-old-female has been struck in the left eye by a softball during a game. Which finding is most concerning for an eye injury? A) Edema to the upper eyelid B) Unusual sensitivity to light C) Bruising and swelling to the orbit D) Vision disturbance that resolves by blinking Answer: B Diff: 2 Page Ref: 1005 Objective: 33-4

1 Copyright © 2018 Pearson Education, Inc.

5) A patient has been splashed in the face with battery acid. He now complains of being unable to see anything and of severe burning to his face. You observe that he is holding both eyes tightly shut and has significant burns on his cheeks and eyelids. In this situation, the EMT would assess the eyes: A) Prior to assessing the airway B) After completing the primary assessment C) After determining the patient's level of consciousness D) Following implementation of spine motion restriction precautions Answer: B Diff: 2 Page Ref: 1005 Objective: 33-3 6) A young male golfer has been struck in the right eye with a golf club. The upper and lower lids are swollen shut and ecchymotic. There are also blood clots between the two eyelids. What would be an indication that the EMT is providing proper care to this patient? A) He gently opens the lids to assess for injury to the eye B) He places a numbing medication in the eye C) He irrigates blood from the eye with sterile saline solution D) He wipes any remaining blood from the face but not the eye Answer: D Diff: 2 Page Ref: 1003 Objective: 33-4 7) A patient complains of pain and discomfort to the left eye after being hit in the face with a cloud of dust while working at a manufacturing complex. As you specifically assess the eye, you note some redness to the globe but do not see any obvious foreign object. The most appropriate care you can provide to this patient would be: A) Covering both eyes with a bandage B) Carefully wiping the eye with a soft piece of sterile gauze C) Gently massaging the eye to promote tearing D) Applying gauze and direct pressure over the eye Answer: A Diff: 2 Page Ref: 1003 Objective: 33-4 8) A young girl complains of eye pain after having sand thrown in her face by another girl at the park. When assessing her eyes, you note large particles of sand in the right eye. The sand appears to be on the conjunctiva, which also seems to be scratched and irritated. Your initial management of this injury would be: A) Stop the tearing by applying gentle pressure to the eye B) Attempt to remove the larger pieces of sand using a cotton swab C) Flush the eye with clean water or sterile water from the ambulance D) Gently wipe the inner eye with sterile pieces of gauze Answer: C Diff: 2 Page Ref: 1003 Objective: 33-4 2 Copyright © 2018 Pearson Education, Inc.

9) A man grinding metal took his safety goggles off and was struck in the face with a shower of tiny metal fragments. He now complains of pain and visual disturbance in his right eye. When assessing the eye, you note that the globe is red, with several pieces of metal embedded in the tissue. Proper care of this patient would include: A) Covering the affected eye and applying a cold pack over it B) Attempting removal of the metal with a sterile swab C) Covering both eyes with a bandage D) Flushing the eye with a sterile saline solution Answer: C Diff: 2 Page Ref: 1003 Objective: 33-4 10) A baseball player was struck in the side of the face by a baseball. Assessment findings include swelling and numbness to the left side of the face and left eyelid, as well as crepitus in the same area. Assessment of the eye reveals no obvious injury. The patient denies any problem with his eyesight, but is having difficulty visually tracking your finger upward. What would be appropriate care for this injury? A) Cold packs to the face and eyelid B) Gentle pressure over the face and eye C) Gentle cleansing of the eye with sterile water D) Warm packs to the area of swelling Answer: A Diff: 2 Page Ref: 1004 Objective: 33-6 11) You suspect that a patient has a fracture of the left orbit. Which statement made by the patient would reinforce this suspicion? A) "The light is hurting my eyes." B) "I am seeing two of everything." C) "Everything looks yellow." D) "My eye feels like it is burning." Answer: B Diff: 3 Page Ref: 1004 Objective: 33-6 12) During an altercation, an intoxicated male was cut with a sharp knife. Assessment reveals a laceration, oozing dark red blood, across the right eyelid down to the right cheek. It also appears that the patient's eye was cut with the knife, a suspicion reinforced by the patient's complaint of eye pain and collection of blood in the sclera. The EMT would: A) Clean the incision with sterile water and peroxide B) Apply direct pressure to the eyelid to control bleeding C) Apply cold packs to the eyelid to decrease swelling D) Cover the eye with a sterile dressing soaked in saline Answer: D Diff: 2 Page Ref: 1004 Objective: 33-3 3 Copyright © 2018 Pearson Education, Inc.

13) Which instructions would you provide to the Emergency Medical Responders when treating a patient who was punched in the eye and has blood visible in the anterior chamber? A) "It is important to keep pressure down in the eye, so we will transport the patient in a sitting position." B) "Let's apply a cold pack to the eye and immobilize the patient for transport." C) "Let's put a shield over the eye and bandage it in place to apply some pressure to the eye." D) "We will need to cover both eyes with patches before we transport the patient." Answer: D Diff: 2 Page Ref: 1004-1005 Objective: 33-4 14) You have been called to a high school science lab, where a male student was splashed in the eye with an alkaline solution. The patient complains of burning to the eye as well as pain. After performing the primary assessment and finding no life-threatening conditions, you would: A) Obtain a set of vital signs and determine whether the patient is taking medications or has any past medical problems B) Determine the exact compound that the patient was splashed with and the appropriate chemical antidote C) Hold the patient's eye open and begin flushing with large amounts of tap water, and continue this intervention throughout transport D) Cover both eyes with eye shields and move the patient to the ambulance for immediate transport Answer: C Diff: 2 Page Ref: 1005 Objective: 33-4 15) You have arrived on the scene of an industrial plant where a female worker had a chemical splashed into her eye. She now complains of severe pain in the eye and is flushing it with tap water over a sink. Which question is it most important that the EMT ask first? A) "What is your past medical history?" B) "Are you wearing contact lenses?" C) "Do you take any medications?" D) "Have you ever had an eye injury before?" Answer: B Diff: 2 Page Ref: 1005 Objective: 33-4

4 Copyright © 2018 Pearson Education, Inc.

16) A patient had an unknown chemical splashed into her left eye at a factory. You have been irrigating the eye on scene and are now getting ready to transfer the patient to the stretcher. Knowing that you will need to continue irrigation throughout transport, how will you position this patient on the stretcher? A) Left lateral recumbent B) Right-side-lying position C) Prone with head turned to the patient's left side D) Semi-Fowler's position Answer: A Diff: 1 Page Ref: 1005 Objective: 33-3 17) A young boy was running with a pencil and tripped. The pencil impaled the boy's left eye and remains lodged in place. He is in his mother's arms and both are crying hysterically. Appropriate care for the eye would include: A) Stabilizing the impaled object and providing immediate transport B) Carefully removing the pencil from the eye to prevent the loss of vision, then providing immediate transport C) Applying ice to the affected eye to decrease pressure, swelling, and subsequent damage D) Applying gentle pressure to the pencil and eye to prevent movement, followed by immediate transport Answer: A Diff: 2 Page Ref: 1006 Objective: 33-3 18) An elderly woman experienced a closed head injury when she fell down five steps. She is responsive to painful stimuli and her breathing is irregular and shallow. When evaluating her airway, you note that she has both upper and lower dentures. Her upper dentures are secure, but her lower dentures are loose. Prior to providing positive pressure ventilation, you would: A) Make sure the lower dentures are secure B) Remove both the upper and lower dentures C) Remove the lower dentures D) Leave the dentures as you found them Answer: C Diff: 2 Page Ref: 1009 Objective: 33-3 19) While running in a parking lot, a young boy tripped and fell. He hit his face and mouth on a curb, knocking one of his top front teeth from its socket. Bleeding has been controlled and the tooth found. Which instructions would you give your partner when caring for the tooth? A) "Wrap the tooth in dry gauze and put it in a plastic bag placed in cool water." B) "Scrub the tooth first to get the dirt off of it and then we will put it back in the socket." C) "Make sure to touch it only by its root when scrubbing the dirt off it." D) "Rinse the tooth first, then put it in a cup of saline for transport." Answer: D Diff: 2 Page Ref: 1009 Objective: 33-6 5 Copyright © 2018 Pearson Education, Inc.

20) While working to build a house, a construction worker was hit in the head by a 2-inch × 4inch plank that slid from the roof and fell to the ground below. The patient has a sizable laceration to the left side of his face and is spitting and coughing blood. He also lost several teeth, two of which he is holding. Your initial action when caring for this patient would be to: A) Assess the ability of the patient to maintain his own airway B) Apply supplemental oxygen C) Obtain and preserve the teeth in a cup of saline, or wrapped in saline-soaked gauze D) Apply an occlusive dressing to the laceration and preserve the teeth Answer: B Diff: 2 Page Ref: 1009 Objective: 33-3 21) You have been called for an elderly female who was found by family members on the floor of her bathroom. She is unresponsive and has a bruise to the right side of her face. Her respirations are snoring and shallow. Her skin is cool and dry, and she is incontinent of urine. Her daughter tells you that the patient has a past medical history of stroke, COPD, and seizures. Which intervention would be most appropriate when managing this patient's airway? A) Suction B) Jaw-thrust maneuver C) High-concentration oxygen D) Head-tilt, chin-lift maneuver Answer: B Diff: 2 Page Ref: 1009 Objective: 33-3 22) A 15-year-old female was struck in the mouth with a baseball bat. She has lost her front teeth and is spitting blood. The EMT's priority concern when caring for this patient would be: A) Blood loss B) Jaw fracture C) Seizure activity D) Airway compromise Answer: D Diff: 2 Page Ref: 1009-1010 Objective: 33-6 23) A female patient struck her face on the steering wheel in a head-on motor vehicle collision and has deformity and crepitus to the bridge of her nose. Further assessment reveals bleeding deep within the nasal passages. Prior to your arrival, fire department EMRs have secured the patient to a long spine board and applied a cervical collar and straps. To best address the hemorrhage in the nasopharynx, the EMT would: A) Suction blood from the airway as needed B) Transport the patient on the backboard with her feet elevated C) Apply ice packs to the bridge of the nose to control hemorrhage D) Carefully pack the nasal passages with rolls of sterile gauze Answer: A Diff: 2 Page Ref: 1009-1010 Objective: 33-3 6 Copyright © 2018 Pearson Education, Inc.

24) A frantic mother dialed 911 for her 3-year-old daughter, who stuck a small, dried bean into her ear. Assessment shows the bean to be lodged firmly in the ear canal. When treating this patient, the EMT would: A) Make one attempt to remove the bean with tweezers B) Gently flush the ear canal with sterile water until the bean is dislodged C) Reassure the mother and transport the patient to the emergency department D) Have the patient follow up with her family physician Answer: C Diff: 2 Page Ref: 1010 Objective: 33-6 25) A 36-year-old male patient was involved in an altercation and now has a 2-inch laceration from a knife to the right side of his neck. Bleeding was controlled prior to your arrival by Emergency Medical Responders. Your paramedic partner has asked you to apply an occlusive dressing to the wound. You recognize that the primary benefit of the action is to: A) Prevent the cervical collar from touching the wound, if one is applied B) Control the bleeding from large blood vessels so the patient does not become hypotensive C) Prevent air entry into the circulatory system D) Promote clot formation so as to stop bleeding Answer: C Diff: 2 Page Ref: 1011 Objective: 33-7 26) A young boy riding an ATV did not see a chain stretched between two poles and hit it with his neck while traveling forward. Subsequently, he was thrown backward from the vehicle with significant force. He was not wearing a helmet. Emergency Medical Responders are with the patient and are holding manual spine motion restriction as well as administering oxygen. When you are assessing this patient, which sign would indicate the injury that must be addressed and managed first? A) Difficulty speaking B) Laceration to the forehead C) Open fracture to the left arm D) Partially avulsed ear Answer: A Diff: 2 Page Ref: 1011 Objective: 33-7

7 Copyright © 2018 Pearson Education, Inc.

27) A 22-year-old male attempted to hang himself in a garage using chains suspended from the roof trusses. He was quickly found by family members and taken down to the ground. Currently he is complaining of neck pain as well as mild difficulty breathing. Assessment shows his airway to be patent, breathing adequate, and skin warm and dry. His neck is red, but free of any subcutaneous air or swelling. Your partner has identified an abnormal bony protrusion to a couple of his cervical vertebrae. What should you do first when caring for this patient? A) Place ice packs to his neck B) Insert a nasal airway C) Immediate transport D) Initiate manual spine motion restriction Answer: D Diff: 2 Page Ref: 1011 Objective: 33-3 28) A construction worker has been shot with a nail gun, resulting in a long nail going through his cheek and becoming firmly embedded in the lower gum and jaw. There is considerable blood in his mouth, and the patient is in excruciating pain. Given this scenario, what would be your first priority? A) Removing the nail from the cheek B) Suctioning the airway of blood C) Applying supplemental oxygen to maintain a pulse oximeter reading of 94% or greater D) Placing a cervical collar Answer: B Diff: 2 Page Ref: 1009 Objective: 33-3 29) A 3-year-old child has stuck a crayon in his nose. Assessment reveals the crayon to be deeply embedded in the right nostril, with some irritation and swelling noted. His vital signs are pulse, 124 beats/min; respiration, 20 breaths/min; and SpO2, 99%. What would be most appropriate action to take when caring for this child? A) Nonemergent transport to the hospital B) Supplemental oxygen C) Attempt removal of the crayon with forceps or tweezers D) Place a warm pack to the bridge of the nose Answer: A Diff: 2 Page Ref: 1010 Objective: 33-6 30) A worker in a factory complex has a sliver of metal lodged in the colored portion of his eye. The EMT would recognize the foreign body as lying in the: A) Cornea B) Iris C) Lens D) Sclera Answer: B Diff: 1 Page Ref: 999 Objective: 33-2 8 Copyright © 2018 Pearson Education, Inc.

31) In a patient with conjunctivitis, the EMT would expect to observe a/an: A) Inflamed and irritated cornea B) Infected lens C) Reddened membrane over the sclera D) Nonresponsive pupil Answer: C Diff: 2 Page Ref: 1000 Objective: 33-2 32) The function of the lens of the eye is to: A) Focus light onto the retina B) Allow light into the eye C) Protect the sclera D) Manufacture vitreous humor Answer: A Diff: 1 Page Ref: 1000 Objective: 33-2 33) Blood in the anterior chamber of the eye is called: A) Orbital hemorrhage B) Scleroderma C) Diplopia D) Hyphema Answer: D Diff: 1 Page Ref: 1004-1005 Objective: 33-2 34) What is a general guideline to use when deciding whether to remove a patient's contact lenses? A) Remove only soft contact lenses, not hard lenses B) Remove contact lenses only if the eye has been traumatically injured C) Remove contact lenses only if the eye is traumatically injured and the patient is conscious D) Remove contact lenses if the eye has been injured by a chemical burn Answer: D Diff: 2 Page Ref: 1005 Objective: 33-5

9 Copyright © 2018 Pearson Education, Inc.

35) A construction worker complains of pain to both eyes after an extended period spent welding on a gas well. The light is bothering him, and he rates the pain as a 10/10. He states that this problem occurred once before and was caused by the intense light of his welding tool. The primary assessment reveals no deficits. The patient's vital signs are pulse, 80 beats/min; respirations, 14 breaths/min; blood pressure, 132/68 mmHg; and SpO2, 98%. Although the injury is very painful, his vision is not compromised. Which set of instructions indicates proper care of this patient? A) "Why don't we flush the eyes with the sterile saline on the ambulance?" B) "We will need to flush the eyes with bicarbonate to stop the burning." C) "Applying direct pressure very gently to both eyes may provide pain relief." D) "I do not think that we will need to provide supplemental oxygen." Answer: D Diff: 2 Page Ref: 1003 Objective: 33-4 36) A 21-year-old male has a stab wound to the left side of his neck. Emergency Medical Responders are holding manual spine motion restriction. As you approach the patient, you note blood on his shirt and pale-looking skin color. As team leader, which direction would you give first? A) "Let's look for other injuries on his chest." B) "Can someone start the patient on supplemental oxygen?" C) "I need someone to place a gloved hand over that stab wound." D) "Before we do anything else, we need to place a cervical collar on his neck." Answer: C Diff: 2 Page Ref: 1011 Objective: 33-6 37) A young male who was drinking heavily has been involved in a motor vehicle accident. As an unrestrained driver, his head went through the driver's-side window; in the process, a portion of his ear was severed. Blood is now coming from the remaining portion of his left ear. In overseeing the care of this patient, which action by another EMT would require corrective action? A) A loose dressing placed over the left ear B) Rolled gauze packed into the ear canal to slow bleeding C) The severed ear wrapped in saline-soaked gauze D) Emotional support provided to the patient while on scene Answer: B Diff: 2 Page Ref: 1010 Objective: 33-3

10 Copyright © 2018 Pearson Education, Inc.

38) A female patient with a maxillary fracture has sustained injury to her: A) Orbits B) Lower jaw C) Nose and orbits D) Upper jaw Answer: D Diff: 1 Page Ref: 1000 Objective: 33-1 39) A conscious and alert 32-year-old male was assaulted and struck in the jaw with a baseball bat. He complains of tremendous jaw pain and exhibits an unstable mandible. His airway is patent, breathing adequate, and radial pulse normal. In caring for this patient and his injury, you would: A) Wrap the neck and jaw securely with roller cling to stabilize the jaw B) Insert an OPA to assure airway patency C) Start positive pressure ventilation with supplemental oxygen D) Consider the need to implement spine motion restriction precautions Answer: D Diff: 2 Page Ref: 1001 Objective: 33-3 40) A 23-year-old male struck his face on the steering wheel when his vehicle hit another car from behind. The patient's nose is deformed and actively bleeding. In his care, which intervention would be a priority? A) Make one attempt to realign the nose B) Pack the nares with sterile gauze C) Suction the airway to keep it free of blood D) Insert an NPA to prevent the nasal cavity from swelling shut Answer: C Diff: 2 Page Ref: 1001 Objective: 33-3 41) A 19-year-old female was assaulted by her intoxicated boyfriend. After punching her in the chest, he slashed her face with a knife. Assessment reveals the patient to be stable with a large laceration from beneath her right eye to her right ear. She is very upset and crying and repeatedly asks you if this injury will leave a permanent scar on her face. Your best response would be: A) "You will be fine. You are lucky he didn't cut an artery." B) "I am not worried about your looks; I am worried about preventing infection." C) "When we get to the hospital, you can discuss this more thoroughly with the doctor." D) "You will be fine. The stitches used today do not scar like they used to." Answer: C Diff: 2 Page Ref: 999 Objective: 33-6

11 Copyright © 2018 Pearson Education, Inc.

42) The best way to detect contact lenses for removal is to: A) Shine a penlight into each eye at a slight angle and look for a shadow B) Turn on the overhead lights and look for your reflection over the iris C) Lightly run a gloved finger across the globe of the eye and sweep any lens to the corner D) Apply a sterile contact lens detector over the iris Answer: A Diff: 2 Page Ref: 1006 Objective: 33-5

12 Copyright © 2018 Pearson Education, Inc.

Prehospital Emergency Care, 11e (Mistovich et al.) Chapter 34 Chest Trauma 1) A patient has sustained an injury to his mediastinum. Based on the anatomy of his chest, which structure has been injured? A) Diaphragm B) Esophagus C) Lungs D) Bronchiole Answer: B Diff: 1 Page Ref: 1021 Objective: 34-2 2) Which statement made by an EMT demonstrates an understanding of chest trauma? A) "Open chest injuries are more serious than closed chest injuries, because air and bacteria can enter the chest through the open wound." B) "In general, an open chest injury caused by blunt trauma is more life threatening than a closed one due to increased potential for blood loss." C) "Closed chest injuries are caused by blunt trauma, and can be just as serious as open chest injuries." D) "Closed chest injuries are caused by penetrating trauma and cause more life-threatening injuries than do open chest injuries." Answer: C Diff: 3 Page Ref: 1024 Objective: 34-3 3) A 39-year-old male has a single stab wound to the anterior chest. When notifying the emergency department, you inform them that the patient has a/an: A) Flail chest B) Evisceration C) Laceration D) Open chest injury Answer: D Diff: 1 Page Ref: 1023 Objective: 34-2 4) An Emergency Medical Responder asks you to explain a pneumothorax. You would explain that a pneumothorax occurs when: A) The lung becomes overinflated with air the patient is unable to breathe B) Air accumulates around the lung, causing a portion of it to collapse C) The trachea becomes obstructed, causing the lungs to collapse D) Air enters the lung and alveoli from a traumatic opening in the chest wall Answer: B Diff: 2 Page Ref: 1022-1023 Objective: 34-1

1 Copyright © 2018 Pearson Education, Inc.

5) When assessing a patient, which sign or symptom is most indicative of a pneumothorax? A) SpO2 of less than 90% on supplemental oxygen B) Intense pain with each inspiration C) Decreased breath sounds to the right lung D) Crepitus and instability to the chest wall Answer: C Diff: 2 Page Ref: 1032 Objective: 34-5 6) You are on the scene of a shooting. Your assessment reveals a 23-year-old male who has been shot twice. The first wound is to the left lower quadrant of the abdomen and is actively bleeding. The second wound is to the left lateral chest and bubbles every time the patient exhales. What would your immediate and best action be? A) Obtain a sterile dressing and cover the chest wound B) Cover the chest wound with a gloved hand C) Place direct pressure over the abdominal gunshot wound D) Place the patient on high-concentration oxygen with a nonrebreather face mask Answer: B Diff: 2 Page Ref: 1030 Objective: 34-8 7) A new EMT who just started working with your EMS system asks you to quickly describe a flail segment. Which statement best describes this injury? A) "Multiple rib fractures have bruised the underlying lung." B) "Fractured ribs have collapsed the underlying lung." C) "Multiple fractures to the rib cage are caused by blunt trauma." D) "Two or more adjacent ribs are broken in two or more places." Answer: D Diff: 1 Page Ref: 1024 Objective: 34-1 8) A 23-year-old female has been involved in a serious motor vehicle collision. Which assessment finding best indicates that she has a flail segment? A) Uneven chest wall movement during breathing B) Shortness of breath upon exertion C) Intense pain with deep inspiration D) Decreasing SpO2 reading despite high-flow oxygen Answer: A Diff: 2 Page Ref: 1024 Objective: 34-4

2 Copyright © 2018 Pearson Education, Inc.

9) You are managing a patient with a large flail segment to the right lateral chest. With this condition, you should recognize that the immediate threat to life is: A) Blood loss B) Hypoxia C) Infection D) Rib fractures Answer: B Diff: 2 Page Ref: 1025 Objective: 34-4 10) When assessing a patient who sustained blunt trauma to the chest, which assessment finding is most indicative of an injury to the lung, impairing adequate ventilations? A) Ability to speak only a few words, following by a gasp B) Elevated heart rate and blood pressure C) Respiratory rate of 20 breaths/min D) Extensive bruising to the anterior chest Answer: A Diff: 2 Page Ref: 1031 Objective: 34-8 11) A confused 62-year-old female fell at home, hitting the side of a table with her chest. Assessment reveals instability to the left lateral chest, accompanied by minor bruising in the same area. The patient complains of intense pain every time she breathes. Her airway is patent, breathing is rapid and shallow, and skin warm and non-diaphoretic. Vital signs are pulse, 112 beats/min; respirations, 24 breaths/min; blood pressure, 132/64 mmHg; and SpO2, 90%. Breath sounds are somewhat shallow but still equal to both lungs. Based on those findings, you should treat this patient for which life-threatening condition? A) Shock B) Pain C) Pneumothorax D) Hypoxia Answer: D Diff: 2 Page Ref: 1031 Objective: 34-8 12) A patient has been shot in the chest with a rifle. Assessment reveals a deteriorating level of consciousness and inadequate breathing. Positive pressure ventilation is being administered, and the entrance wound, which is located midclavicular at the second intercostal space on the left side of the chest, has been covered with an occlusive dressing. Your next priority action would be to: A) Check the blood pressure and determine whether a narrow pulse pressure exists B) Place a cervical collar and initiate spine motion restriction precautions C) Look for a bullet exit wound posteriorly D) Get a full set of vital signs and pulse oximeter reading Answer: C Diff: 2 Page Ref: 1023 Objective: 34-8 3 Copyright © 2018 Pearson Education, Inc.

13) An adult female is found unresponsive after being shot in the chest. Which statement made by your partner is of most concern and indicative of patient deterioration? A) "I see an entry wound but cannot find the exit wound anywhere." B) "Her SpO2 has gone from 90% to 93%." C) "I do not think her jugular veins are distended." D) "She is getting more difficult to ventilate." Answer: D Diff: 3 Page Ref: 1033 Objective: 34-8 14) During the secondary assessment, the EMT notes instability and a flail segment to a patient's lower left lateral chest. The EMT would immediately: A) Auscultate breath sounds B) Stabilize the segment with his hand C) Start positive pressure ventilation D) Apply bulky dressings to the area Answer: B Diff: 2 Page Ref: 1030 Objective: 34-8 15) A patient with blunt chest wall trauma has a flail section to her chest. She is conscious and confused, her breathing is rapid and shallow, and breath sounds are diminished bilaterally. After manually stabilizing the flail section of the chest wall, the EMT best treats this condition by: A) Applying ice packs to the flail segment B) Administering supplemental oxygen through a nonrebreather face mask C) Providing positive pressure ventilation with oxygen attached D) Inserting an oropharyngeal airway and ventilating the patient on room air Answer: C Diff: 2 Page Ref: 1031 Objective: 34-8 16) An unrestrained 32-year-old female motorist hit a utility pole head-on at 45 mph. The car sustained extensive damage. The patient has an open airway, is breathing poorly at 32 breaths per minute with absent alveolar breath sounds, and is responsive to painful stimuli. Her radial pulse is rapid and weak, and her skin is cool and cyanotic. Breath sounds are decreased on the right side. Emergency Medical Responders are initiating spine motion restriction precautions. Your next action would be to: A) Start positive pressure ventilation B) Examine the patient for additional injuries C) Place a cervical collar D) Palpate the chest wall for instability Answer: A Diff: 2 Page Ref: 1031 Objective: 34-8

4 Copyright © 2018 Pearson Education, Inc.

17) A 42-year-old male was involved in a fight and was stabbed in the right lateral chest. The knife is still impaled, and the patient now complains of shortness of breath. When he speaks, he gurgles and blood drains from his mouth. His respirations are labored and his radial pulse is weak. Which intervention should you perform first? A) Suction the blood from the airway B) Provide positive pressure ventilation C) Stabilize the knife with bulky dressings D) Remove the knife and seal the injury with an occlusive dressing Answer: A Diff: 2 Page Ref: 1033 Objective: 34-8 18) A construction foreman was stabbed with a screwdriver in the right anterior chest by an angry employee. Prior to your arrival, he removed the screwdriver. He is alert and oriented and complains of pain to the injury site. Your primary assessment reveals an open airway, adequate breathing, and strong radial pulse. Based on these findings, which intervention should be performed next? A) Open the airway using the jaw-thrust maneuver B) Take manual spine motion restriction precautions C) Evaluate the patient's SpO2 level D) Insert a nasopharyngeal airway Answer: C Diff: 2 Page Ref: 1037; Figure 34-22B Objective: 34-8 19) A paramedic has asked that you apply a dressing over a sucking chest wound. Which option is the best choice to cover this injury? A) Dry sterile gauze dressing B) Sterile gauze soaked in sterile water C) Clean washcloth D) Petroleum gauze Answer: D Diff: 1 Page Ref: 1034 Objective: 34-8 20) You have applied a nonporous dressing to a stab wound on a patient's chest. The Emergency Medical Responder assisting you asks you why the dressing was taped on three sides and not all four. Your best response is that taping the dressing on three sides: A) Permits oxygen to still enter the lungs B) Allows trapped air to escape on exhalation C) Causes less pain for the patient D) Decreases the chance of air entering the chest on exhalation Answer: B Diff: 3 Page Ref: 1034 Objective: 34-8

5 Copyright © 2018 Pearson Education, Inc.

21) You have been called to a New Year's Eve party for a male patient who was stabbed in the anterior chest. According to partygoers, the patient went outside with another male and was stabbed. Fifteen minutes later, he was found in the snow, and 911 was called. When assessing this patient, which finding would you consider to be the most serious and warranting immediate attention? A) Complaint of a worsening headache B) Capillary refill of 3 seconds C) Decreased breath sounds to the left lung D) Pain and bruising to the left thigh Answer: C Diff: 2 Page Ref: 1032 Objective: 34-8 22) You are transporting a 19-year-old male with a gunshot wound to the chest. On scene, you covered the wound with an appropriate dressing. Now, as you reassess the patient, you note that the patient is tachypneic and complains of increased difficulty breathing. Breath sounds on the side of the injury are diminished. Your immediate action would be to: A) Tape the dressing on the fourth side B) Lift the dressing from the wound briefly during exhalation C) Provide positive pressure ventilation with high-concentration oxygen D) Cover the wound with another dressing Answer: B Diff: 2 Page Ref: 1034 Objective: 34-5 23) A patient has been involved in a motorcycle crash and has a flail segment to the right lateral chest. His airway is open and he is breathing poorly at 24 breaths per minute with a decreasing SpO2. The EMT shows that he is appropriately caring for this injury when he: A) Administers oxygen through a nonrebreather face mask B) Positions the patient in semi-Fowler's position for transport C) Places an ice pack over the site of injury D) Starts positive pressure ventilation Answer: D Diff: 2 Page Ref: 1033 Objective: 34-8

6 Copyright © 2018 Pearson Education, Inc.

24) Assessment findings for a patient who was thrown from a motorcycle indicate that he has a flail chest wall segment to his right anterior chest. He exhibits labored breathing and an SpO2 of 92%. Breath sounds are clear and equal bilaterally. The segment has been stabilized, and you are prepared to start positive pressure ventilation. Given these assessment findings, which type of injury underlying the flail segment is your primary concern? A) Pulmonary contusion B) Pneumothorax C) Hemothorax D) Rib fractures Answer: A Diff: 3 Page Ref: 1025-1026 Objective: 34-5 25) Assessment findings for a 33-year-old female who was shot in the right side of the chest indicate that she has a pneumothorax. As you care for this patient, your primary concern is monitoring the injury for: A) Arterial bleeding B) Infection at the wound site C) Tension pneumothorax D) Sucking sound at the wound site Answer: C Diff: 3 Page Ref: 1027 Objective: 34-5 26) You suspect that a patient with an open pneumothorax may be developing a tension pneumothorax. Which action is critical given this situation? A) Remove the occlusive dressing B) Lift one side of the dressing during exhalation C) Gently apply pressure to the chest during inhalation D) Ensure that all four sides of the dressing are taped Answer: B Diff: 3 Page Ref: 1027 Objective: 34-8 27) A patient was stabbed in the right anterior chest and is in obvious respiratory distress. As you perform the secondary assessment, which signs and/or symptoms would suggest that she is developing a tension pneumothorax? A) Absent breath sounds on the right, distended neck veins, tracheal deviation to the left B) Absent breath sounds on the left, hypotension, SpO2 of 98%, bradycardia C) Respiratory distress, absent breath sounds on the left, flat neck veins, tachycardia D) Absent breath sounds on the right, tracheal deviation to the right, cyanosis Answer: A Diff: 3 Page Ref: 1027 Objective: 34-5

7 Copyright © 2018 Pearson Education, Inc.

28) The EMT shows that he understands the difference between a pneumothorax and a tension pneumothorax when he makes which statement? A) "A tension pneumothorax causes blood to accumulate around the lung; a pneumothorax involves the accumulation of only air around the lung." B) "A tension pneumothorax causes cardiac output to decrease; a simple pneumothorax does not affect cardiac output." C) "A pneumothorax describes a collapsed lung; a tension pneumothorax involves both a collapsed lung and blood loss." D) "A pneumothorax is caused by a closed chest injury; a tension pneumothorax is caused by an open chest injury." Answer: B Diff: 3 Page Ref: 1026-1027 Objective: 34-7 29) You suspect a trauma patient has a hemothorax to the left lung. Which assessment finding would reinforce this suspicion? A) Neck vein distention and absent breath sounds to the right lung B) Cyanosis and a blood pressure of 210/100 mmHg C) Bradycardia and hypertension D) Respiratory distress and the signs and symptoms of shock Answer: D Diff: 3 Page Ref: 1027 Objective: 34-5 30) On scene, a paramedic directs you to help another EMS crew who is caring for a patient who was hit and trapped under a passenger van that ran into a crowd. As you approach the patient, which scene size-up clue seemingly indicates that the patient is suffering from traumatic asphyxia? A) Wound to the chest making a sucking sound B) Abdominal bruising and distention C) Bluish discoloration to the neck and face D) Bilateral femur deformity Answer: C Diff: 2 Page Ref: 1028 Objective: 34-7

8 Copyright © 2018 Pearson Education, Inc.

31) An unrestrained female driver hit a utility pole with her vehicle at a moderate rate of speed. During the impact, she struck the steering wheel with her chest. The patient's airway is open, and she states that it is painful to breathe. Her pulse is moderate in strength, irregular, and tachycardic. Breath sounds are equal bilaterally, and no jugular venous distention is noted. Assessment of her chest reveals bruising and instability to the sternum. When asked, she denies any past medical history. Based on this mechanism and the assessment findings, the EMT should be suspicious of which condition? A) Pneumothorax B) Traumatic asphyxia C) Tension pneumothorax D) Cardiac contusion Answer: D Diff: 3 Page Ref: 1028 Objective: 34-6 32) Asherman, Halo, and Bolin devices are commercially available devices to use for which injury? A) Flail chest B) Open chest wound C) Commotio cordis D) Pneumothorax Answer: B Diff: 1 Page Ref: 1034 Objective: 34-3 33) An alert and oriented young male fell 5 feet from a stage, impacting a metal railing with the right side of his chest, just under his armpit. The primary assessment is negative for life threats, although he does complain of very painful breathing and has remarkable tenderness and crepitus over the fifth rib laterally in this area. Throughout care, which action is essential to perform? A) Apply ice to area B) Administer aspirin for pain C) Monitor breath sounds D) Splint site with a bulky dressing Answer: C Diff: 2 Page Ref: 1035 Objective: 34-4

9 Copyright © 2018 Pearson Education, Inc.

34) You are transporting a young and healthy female patient who was involved in a motor vehicle collision, during which she struck her chest on the steering column. Although the primary assessment reveals no life threats, the patient does have redness to her sternum, as well as pain and tenderness. Breath sounds are clear and present bilaterally, and vital signs are within normal limits. As you transport and reassess this patient, which sign or symptom would indicate the immediate need for ALS? A) Increased tenderness over sternum B) New complaint of leg pain C) Development of bruise over sternum D) Onset of an irregular heartbeat Answer: D Diff: 2 Page Ref: 1037; Figure 34-22B Objective: 34-9 35) A middle-aged male has been stabbed once in the left anterior chest. His airway is patent, respirations tachypneic, pulse weak and rapid, and skin cool and diaphoretic. Breath sounds are clear and equal bilaterally. The vital signs are pulse, 140 breaths/min; respirations, 24 breaths/min; blood pressure, 100/78 mmHg; and SpO2, 96% on supplemental oxygen. JVD is present. Given this presentation, you would have a high index of suspicion for: A) Pneumothorax B) Hemothorax C) Fail segment D) Pericardial tamponade Answer: D Diff: 3 Page Ref: 1029 Objective: 34-6 36) On scene at a college football game, a wide receiver is unresponsive after colliding with another player and forcefully taking a helmet to the chest. The athletic trainer reports that the patient is pulseless and apneic. Given the mechanism of injury and patient's presentation, which instruction would be appropriate? A) "Let's apply the AED and follow all prompts." B) "Before we do anything, we need to put a cervical collar on the patient." C) "Use the AED but no CPR so we do not worsen the potential chest injury." D) "Someone start positive pressure ventilation while I check for a blood pressure." Answer: A Diff: 2 Page Ref: 1028 Objective: 34-6

10 Copyright © 2018 Pearson Education, Inc.

37) A young boy has an open chest wound to the left lateral thorax after falling off his bike onto a metal spike protruding from the ground. His family is present; they are very upset and crying. As an EMT, you recognize that in this situation, your primary focus is: A) Controlling pain B) Maintaining oxygenation C) Providing emotional support D) Preventing potential wound infection Answer: B Diff: 2 Page Ref: 1031 Objective: 34-8 38) Damage to which thoracic tissues can cause impairment of ventilation from a developing pneumothorax? A) Mediastinum B) Pleural membranes C) Trachea D) Pulmonary arteries Answer: B Diff: 1 Page Ref: 1022 Objective: 34-2 39) The EMT's primary concern with any open injury to the chest is: A) Controlling internal bleeding B) Evaluating for crepitus C) Stopping air entry D) Checking for intercostal muscle damage Answer: C Diff: 1 Page Ref: 1023 Objective: 34-3 40) A young male patient has self-extricated from his vehicle after hitting a telephone pole and rolling his car several times at a high rate of speed. He is spitting blood and has a large area of tenderness and soft tissue injury to the left side of his chest. On scene, you did not observe any paradoxical motion of the chest wall. The patient remains alert and oriented with stable vital signs, but continually complains of painful breathing. During transport, what is it essential that you do? A) Reassess for paradoxical motion of the chest wall B) Apply bulky dressings to the area of chest injury C) Start positive pressure ventilation with supplemental oxygen D) Place a nonporous dressing and ice packs to the injured area Answer: A Diff: 2 Page Ref: 1024-1025 Objective: 34-4

11 Copyright © 2018 Pearson Education, Inc.

41) Which mechanism of injury produces the conditions for traumatic asphyxia? A) Fall of 20 feet from scaffolding onto a concrete sidewalk B) Person strangled but not killed in an attempted rape C) Penetrating chest trauma with a high-powered firearm D) Person pinned between a truck and wall of a building Answer: D Diff: 2 Page Ref: 1027-1028 Objective: 34-7 42) On follow-up, the ED physician informs you that a patient whom you transported earlier was found to have a hemothorax. In this condition: A) The trachea is torn B) Fluid and blood collect around the heart C) The pleural membranes are punctured by a rib D) Blood collects in the chest cavity, collapsing the lung Answer: D Diff: 1 Page Ref: 1027 Objective: 34-1 43) A middle-aged male patient has been stabbed once in the right anterior chest. His airway is patent, respirations tachypneic, pulse weak and rapid, and skin cool and diaphoretic. Breath sounds are clear on the left and severely diminished on the right. JVD and tracheal deviation are present. His SpO2 is 93% on supplemental oxygen via BVM. The vital signs are pulse, 140 beats/min; respirations, 28 breaths/min; and blood pressure, 100/78 mmHg. Your next action should be to: A) Call for an ALS intercept en route to the ED and lift the dressing to see if air escapes during exhalation B) Attempt to relieve the tension while awaiting ALS arrival at the scene C) Await the arrival of ALS to "burp" the dressing between ventilations D) Initiate positive pressure ventilation utilizing FROPVD Answer: A Diff: 3 Page Ref: 1033 Objective: 34-9

12 Copyright © 2018 Pearson Education, Inc.

Prehospital Emergency Care, 11e (Mistovich et al.) Chapter 35 Abdominal and Genitourinary Trauma 1) A patient has been stabbed in the right upper quadrant of the abdomen. Which solid organ should the EMT be most concerned with being injured? A) Liver B) Stomach C) Small intestine D) Kidney Answer: A Diff: 1 Page Ref: 1043 Objective: 35-2 2) The EMT recognizes that the division between the thoracic and abdominal cavities lies approximately: A) At the level of the umbilicus B) At the fifth intercostal space C) Below the twelfth rib D) Above the twelfth thoracic vertebra Answer: B Diff: 1 Page Ref: 1043 Objective: 35-2 3) Which abdominal organ or structure, if seriously injured, would cause the most rapid death? A) Descending aorta B) Spleen C) Stomach D) Large intestine Answer: A Diff: 1 Page Ref: 1043 Objective: 35-2 4) A patient who experienced blunt trauma to the abdomen has a lacerated liver. Given this information, what would represent the most immediate threat to life? A) Liver infection B) Significant pain C) Development of acute peritonitis D) Internal hemorrhage Answer: D Diff: 1 Page Ref: 1043 Objective: 35-4

1 Copyright © 2018 Pearson Education, Inc.

5) You have been called for a male patient who was shot with a rifle. Assessment reveals an entrance wound to the left lower quadrant of the abdomen. A hemorrhaging exit wound to the left lower back is also located. What should you do immediately? A) Cover the abdominal wounds with dry sterile gauze dressings B) Auscultate the patient's breath sounds C) Cover the entrance wound with a gloved hand D) Place a cervical collar on the patient Answer: C Diff: 2 Page Ref: 1046 Objective: 35-5 6) You have arrived on the scene for a patient assaulted with a baseball bat. As you approach, which position would seemingly indicate that he has suffered an injury to the abdomen? A) Supine with legs extended and toes pointed B) Prone with hands on the back C) Standing with restless movement and pacing D) On his side with knees drawn to the chest Answer: D Diff: 2 Page Ref: 1045 Objective: 35-4 7) You have arrived on the scene of a stabbing. As you approach the patient, you note that his shirt is off and a single stab wound to the right lower quadrant of the abdomen is evident. The patient appears unresponsive as you approach. At the patient's side, your first action should be to: A) Cover the wound with a nonporous dressing B) Examine the patient's airway C) Quickly look for additional stab wounds D) Provide oxygen via nonrebreather face mask Answer: B Diff: 2 Page Ref: 1045 Objective: 35-5 8) You have just arrived on the scene of a very serious motor vehicle collision. Emergency Medical Responders (EMRs) have rapidly extricated an unresponsive female from the driver's seat. The EMRs report that she was unrestrained and struck the steering wheel with her chest and abdomen. As you start the primary assessment, you note that the patient has snoring respirations and demonstrates shallow breathing at a rate of 24 breaths per minute. What should you do next? A) Obtain vital signs B) Secure the patient to backboard with a cervical collar placed C) Perform a jaw-thrust maneuver D) Start positive pressure ventilation Answer: C Diff: 2 Page Ref: 1045 Objective: 35-5

2 Copyright © 2018 Pearson Education, Inc.

9) A patient has been assaulted and sustained blunt trauma to the abdomen. As you start the primary assessment, you note that he is vomiting blood and you begin to suction him immediately. After the airway has been suctioned, you should: A) Check the adequacy of the patient's breathing B) Examine the abdomen for specific injuries C) Obtain a full set of vital signs D) Administer oxygen with a nonrebreather face mask Answer: A Diff: 2 Page Ref: 1045 Objective: 35-5 10) A 45-year-old female was at a family reunion and fell off a trampoline. She hit a picnic table with the left side of her lower rib cage. Now, she is very anxious and confused. Her airway is open and her breathing adequate. Her radial pulse is rapid and weak, and her skin is cool and diaphoretic. Vital signs are pulse, 132 beats/min; respirations, 20 breaths/min; blood pressure, 102/78 mmHg; and SpO2, 97% on ambient air. The patient has ecchymosis to the left lower rib area as well as tenderness and instability to the rib cage. She also has a superficial abrasion to her forehead. Given these findings, you would suspect and treat her for which condition? A) Hypovolemic shock B) Hypoxia C) Spinal shock D) Closed head injury Answer: A Diff: 3 Page Ref: 1045 Objective: 35-4 11) You have arrived on the scene of an assault involving a knife. Assessment of the unresponsive 21-year-old male patient reveals him to have sustained multiple lacerations to the arms and abdomen, and a section of his intestine is now protruding through a large laceration in the area around the umbilicus. He has lost a significant amount of blood. His airway is open and he is breathing poorly at a rate of 28 breaths per minute. His radial pulse is weak, and his skin is cool and pale. What should the EMT do first? A) Cover the intestine with saline-soaked gauze B) Secure the patient to a long spine board C) Start positive pressure ventilation D) Check the blood pressure Answer: C Diff: 2 Page Ref: 1045 Objective: 35-5

3 Copyright © 2018 Pearson Education, Inc.

12) A patient has sustained a large abdominal evisceration, such that a large portion of intestines are now exposed. After properly managing this injury, you are preparing the patient for transport. You ascertain that there is no evidence of spinal injury. How should you place the patient on the stretcher for transport? A) Prone position, to maintain pressure on the abdomen B) Semi-Fowler's position, to promote adequate breathing C) Lateral recumbent position with legs extended D) Supine position, with knees and hips flexed Answer: D Diff: 2 Page Ref: 1046 Objective: 35-6 13) Which one of these injuries is most likely to cause peritonitis? A) Contusion to the pancreas B) Stab wound to the small intestine C) Blunt trauma to the spleen D) Bullet injury to the liver Answer: B Diff: 2 Page Ref: 1043 Objective: 35-3 14) A patient has been shot in the abdomen. Assessment reveals that the bullet entered the body in the right upper quadrant and exited the lower right back. Given this finding, the EMT should assume which solid organs may have been injured? A) Liver and gallbladder B) Liver, gallbladder, and right kidney C) Liver, gallbladder, right kidney, and small intestine D) Liver, right kidney, and pancreas Answer: D Diff: 3 Page Ref: 1043 Objective: 35-2 15) A patient has referred pain to her left shoulder. What should the EMT suspect? A) Hemorrhage from the spleen B) Rupture of the small intestine C) Damage to the gallbladder D) Trauma to the right kidney Answer: A Diff: 2 Page Ref: 1043 Objective: 35-5

4 Copyright © 2018 Pearson Education, Inc.

16) A 74-year-old female complains of abdominal pain after falling down five stairs off her front porch and striking her abdomen and left hand on a mailbox at the bottom. The patient also reports pain in her left wrist and right ankle, both of which show obvious deformity. She is alert and oriented, and her abdomen is free of bruising but tender in the left upper and lower quadrants. Her vital signs are pulse, 132 beats/min; respirations, 22 breaths/min; blood pressure, 106/86 mmHg; and initial SpO2, 93% on room air. Oxygen has been applied, and the patient has full spine motion restriction precautions in place. Advanced life support has been requested and is 12 minutes away from your current location. What is your next action? A) Apply ice to the wrist and ankle while awaiting arrival of the ALS unit B) Initiate immediate transport and intercept with the ALS unit C) Monitor the patient and wait for ALS assistance D) Secure the wrist and ankle to board splints, initiate transport, and cancel the ALS unit Answer: B Diff: 2 Page Ref: 1046 Objective: 35-5 17) You have been called for a 42-year-old male who complains of back pain. At the scene, the man tells you that he was lying on the floor when his 5-year-old son unexpectedly jumped feet first onto his back. He gives a medical history of high blood pressure and asthma, for which he takes medications. As you assess the patient, which finding provides the greatest indication that the patient has sustained an abdominal injury? A) Bruise to his back B) Mild nausea C) Discoloration around the umbilicus D) Shortness of breath Answer: C Diff: 3 Page Ref: 1046 Objective: 35-3 18) During a motor vehicle collision, a confused unrestrained male struck the steering wheel with his abdomen. Currently, he is confused and complains of generalized abdominal pain, which he rates as 6/10. Assessment indicates red marks across his lower abdomen caused by the steering wheel. His pulse is 140 beats/min, respirations are 24 breaths/min, blood pressure is 108/88 mmHg, and SpO2 is 86%. The patient looks pale and has moist skin. What is most appropriate when providing oxygen to this patient? A) Provide 2 lpm through a nasal cannula B) Provide 8 lpm through a simple face mask C) Provide 15 lpm through a nonrebreather mask D) Oxygen can be withheld due to the SpO2 reading Answer: C Diff: 1 Page Ref: 1046 Objective: 35-5

5 Copyright © 2018 Pearson Education, Inc.

19) An industrial worker has sustained a laceration to his abdomen. On closer inspection, you note what appears to be fat tissue and a portion of intestine protruding through the wound. Which intervention is most appropriate for treating this injury? A) One attempt at replacement of the eviscerated material in the abdomen using sterile gloves B) Dry sterile dressing applied over the wound C) Direct pressure using gloved palms to the injury D) Occlusive dressing applied over saline-soaked gauze Answer: D Diff: 2 Page Ref: 1046 Objective: 35-6 20) The EMT's care for an open chest wound and for an abdominal evisceration are similar in that: A) Air must be periodically released from both wounds B) Occlusive dressings should be placed over both wounds C) Moist dressings are placed over both wounds D) Both wounds can cause the trapping of air in the body Answer: B Diff: 2 Page Ref: 1046 Objective: 35-6 21) While a 43-year-old man was cleaning a window, the glass broke and a sharp piece sliced his groin. Assessment shows him to have a large laceration to his left scrotum, with a testicle exposed and protruding through the sac. Blood is pouring from the laceration, and the patient rates the pain as an 8/10. What should the EMT do first? A) Apply direct pressure to the injury to stop the hemorrhage B) Provide oxygen at 15 lpm C) Call for advanced life support assistance D) Cover the exposed testicle with moist dressing, and dress it loosely to collect the blood that is escaping Answer: A Diff: 2 Page Ref: 1048 Objective: 35-8 22) A 16-year-old male has been struck in the genitalia by a baseball. Assessment indicates a tremendous amount of edema as well as hematoma formation and bruising to his scrotum. The primary assessment reveals no life-threatening conditions; however, the patient is crying and rates the pain as 10/10. Which action is most appropriate for this patient? A) Rapid transport using lights and sirens B) Application of warm packs to the scrotum C) Apply direct pressure to the site of injury D) Place cool compresses to the testicles Answer: D Diff: 2 Page Ref: 1048 Objective: 35-8

6 Copyright © 2018 Pearson Education, Inc.

23) You have been called for a deeply disturbed psychiatric patient who has cut off the tip of his penis with a steak knife. Assessment reveals the 31-year-old male to be crying, with blood noted to his hands and pants. The primary assessment reveals no acute life threats. At this time, your priority is to: A) Place the amputated penis in sterile water B) Control any active bleeding that may be occurring C) Prevent the patient from urinating D) Determine why the patient cut himself Answer: B Diff: 2 Page Ref: 1048 Objective: 35-8 24) A female patient suffered a straddle injury (genitalia trauma) while trying to balance on top of a chain-link fence, after she fell with a leg on each side of the fence. At this time, you would: A) Apply an ice pack wrapped in a towel to the genital area B) Gently put sterile gauze dressings into the opening of the vaginal canal to stop the bleeding C) Place a moist compress, such as a moist sanitary pad, over the injury to control the bleeding D) Clean the area with sterile saline and apply direct pressure with a gloved hand while monitoring for ongoing bleeding Answer: C Diff: 2 Page Ref: 1048 Objective: 35-8 25) You are transporting a female patient who was sexually assaulted by having a foreign body inserted deep into her vagina. You are attempting to control hemorrhage by the appropriate means. As you reassess the patient, which finding is of most concern? A) Pain increases from 7/10 to 10/10 B) Large blood clots form at the vaginal entrance C) The patient informs you she has a sexually transmitted disease D) Heart rate increases from 116 to 140 beats/min Answer: D Diff: 2 Page Ref: 1047 Objective: 35-7 26) A middle-aged male sustained blunt force trauma to the abdomen, resulting in an isolated tear and perforation of the large intestine. When following up on the patient, what would the EMT suspect occurred, given the organ that was injured? A) The patient developed an infection B) The patient rapidly decompensated into shock C) The patient has profound hypoxia D) The patient developed severe internal bleeding Answer: A Diff: 3 Page Ref: 1043 Objective: 35-2

7 Copyright © 2018 Pearson Education, Inc.

27) You are assessing a young male patient who was assaulted with a baseball bat and struck once on the left upper quadrant of the abdomen. You would recognize that this patient has Kehr's sign when he makes which statement? A) "I am having a hard time breathing!" B) "My belly hurts really bad." C) "My left shoulder is killing me!" D) "It really hurts for me to breathe." Answer: C Diff: 2 Page Ref: 1043 Objective: 35-1 28) The EMT has fundamental understanding of abdominal trauma when he states: A) "Evisceration injuries can be recognized by dark-colored bruising over the top of the injury site." B) "Blunt trauma is especially lethal because of the large number of organs that can be affected." C) "Injuries to solid organs like the bladder tend to be more lethal than injuries to hollow organs." D) "Injuries to hollow organs tend to be worse than injuries to solid organs because of the increased potential for blood loss." Answer: B Diff: 2 Page Ref: 1044 Objective: 35-4 29) At the scene of a motor vehicle accident, a car has extensive front-end damage and a deformed steering wheel. The restrained driver, who was self-extricated, is in the care of EMRs, who have applied a cervical collar and continue to maintain spine motion restriction precautions. The patient's airway is patent and respirations adequate. His radial pulse is rapid and weak, and his skin is cool and diaphoretic. The EMRs report the following vital signs: pulse, 136 beats/min; respirations, 20 breaths/min; blood pressure, 100/68 mmHg; and SpO2, 95% with supplemental O2 being administered. The right forearm is deformed, and the head, chest, and upper back are uninjured according to your assessment. Your next action would be to: A) Start positive pressure ventilation B) Repeat the vital signs C) Change the nonrebreather mask to a nasal cannula D) Check the abdomen for injury Answer: D Diff: 2 Page Ref: 1045 Objective: 35-5

8 Copyright © 2018 Pearson Education, Inc.

30) While returning from a call, you are flagged down by a frantic family member of a 12-yearold male patient. She is standing in the yard, and reports that the child was shot in the abdomen and is now inside the house. She states that she has already called the police, who have not arrived yet, and that the shooter is another family member who has since left the scene. What is your best course of action? A) Go to the porch and have the patient brought to you B) Carefully look for scene hazards and enter once you feel safe C) Move the ambulance and wait for law enforcement to arrive D) Cautiously enter the house and be ready to vacate if any hazards are found Answer: C Diff: 2 Page Ref: 1044 Objective: 35-5 31) If a patient has severe retroperitoneal bleeding from an organ located within this cavity, she has most likely injured her: A) Stomach B) Liver C) Large intestine D) Kidney Answer: D Diff: 2 Page Ref: 1042 Objective: 35-1 32) You are transporting an alert and oriented male who sustained severe abdominal trauma in an assault. The primary survey indicates a patent airway with respirations of 24 breaths per minute. Breath sounds are present bilaterally. The patient has a rapid and weak radial pulse of 144 beats per minute, and his skin is pale and cool. The blood pressure is 92/64 mmHg and SpO2 is 92%. Best management of the respiratory and oxygenation parameters would include: A) Positive pressure ventilation with supplemental oxygen B) High-concentration oxygen through a nonrebreather mask C) Application of a nasal cannula at 4 lpm O2 D) Encouraging the patient to slow his respirations Answer: B Diff: 2 Page Ref: 1046 Objective: 35-5

9 Copyright © 2018 Pearson Education, Inc.

33) On scene, you encounter a restless patient who has been stabbed in the right upper quadrant of the abdomen and now complains of abdominal pain and a dry mouth. Assessment reveals no immediate life threats to the airway or breathing. The radial pulse is rapid and weak, and the skin cool and diaphoretic. Vital signs are pulse, 124 beats/min; respirations, 20 breaths/min; blood pressure, 122/88 mmHg; and SpO2, 90% on room air. The injury is isolated, with no involvement of the head, neck, or back. In managing the scene and the patient, which observation would cause you to immediately intervene? A) An EMR placing his gloved hand over the injury to control the bleeding B) Application of supplemental oxygen C) An EMR placing the patient in a lateral recumbent position with the knees flexed D) Family giving the patient water to drink since he is thirsty Answer: D Diff: 3 Page Ref: 1046 Objective: 35-5 34) You are transporting a male patient with a deep stab wound to the groin, with damage also noted to the genitalia following an assault by his girlfriend. The patient is alert and oriented, albeit restless. Vital signs on scene were pulse, 128 beats/min; respirations, 20 breaths/min; blood pressure, 108/64 mmHg; and SpO2, 92%. Oxygen is being administered via nasal cannula. Upon reassessment, which sign would be most concerning to the EMT? A) Patient's urge to void B) Blood pressure is now 106/88 mmHg C) SpO2 stabilizing at 96% despite O2 therapy D) Heart rate of 132 beats/min Answer: B Diff: 3 Page Ref: 1047 Objective: 35-7

10 Copyright © 2018 Pearson Education, Inc.

Prehospital Emergency Care, 11e (Mistovich et al.) Chapter 36 Multisystem Trauma and Trauma in Special Patient Populations 1) You are by the side of an 88-year-old male who fell. He has a hematoma to his forehead as well as deformity to his left forearm. His daughter informs you that her father has severe dementia and she cares for him at home. As you assess him, he continually screams out, "Mary, help me. I need help!" At this time, which question is most appropriate to ask the daughter? A) "Was Mary his wife?" B) "Is this how he normally is?" C) "Do you know what he means by this?" D) "My mother suffers from dementia; it is a terrible disease, isn't it?" Answer: B Diff: 2 Page Ref: 1061 Objective: 36-10 2) An 83-year-old female fell down the basement stairs. Once at her side, she tells you that her left hip hurts and rates it as 10/10. Since the patient is downstairs, you recognize the need for assistance in extricating her from the residence. As you start the primary assessment, you note a dog at the top of the stairs starts growling at you. At this time, what is your primary concern? A) Airway B) Pain C) Need for additional help D) Dog Answer: D Diff: 1 Page Ref: 1054 Objective: 36-10 3) A car swerved to miss a deer in the road and hit a utility pole at a low rate of speed. Damage to the car is minimal, but the 82-year-old male passenger was not wearing a seat belt and hit his head on the dashboard. He presents as confused and diaphoretic. His daughter, the driver, states that he has a history of dementia, high blood pressure, and diabetes, as well as coronary artery disease. She adds that he took his insulin about 2 hours ago. Which question to the daughter would be most important at this time? A) "How far along is the dementia?" B) "He seems sweaty. Was he feeling okay this morning?" C) "Did he eat anything today?" D) "Does he take medication for the high blood pressure?" Answer: C Diff: 2 Page Ref: 1060 Objective: 36-9

1 Copyright © 2018 Pearson Education, Inc.

4) A 78-year-old male pedestrian was struck by a car at a convenience store. The patient is conscious but confused, and has sustained multisystem trauma. What would represent the best sequence of care for this patient? A) Primary assessment, oxygen, secondary assessment, spine motion restriction precautions, and all other care en route to the hospital B) Primary assessment, spine motion restriction, and all other care en route to the hospital C) Primary assessment, oxygen, secondary assessment, and all other care en route to the hospital D) Primary assessment, oxygen, secondary assessment, cervical collar, and all other care en route to the hospital Answer: A Diff: 2 Page Ref: 1060 Objective: 36-10 5) A 62-year-old female with severe kyphosis fell and hit the back of her head, which is now oozing blood from a small laceration. Family members state that she suddenly complained of a "very bad headache" before collapsing to the kitchen floor. She has snoring respirations and cyanosis in the extremities. Which intervention should you perform first? A) Apply a cervical collar B) Use a jaw-thrust maneuver C) Start positive pressure ventilation D) Cover the laceration Answer: B Diff: 2 Page Ref: 1060 Objective: 36-10 6) You are transporting an elderly female who fell in a nursing home and displays outward rotation of her left leg. She has a lengthy medical history, including dementia. Currently, she is very quiet and receiving oxygen through a nasal cannula at 6 liters per minute. She also is secured to a long spine board. During your reassessment, you note her to be alert but confused, with slightly accelerated respirations and a strong radial pulse. You also note that her SpO2 has dropped from 97% on scene to 91%. You should: A) Perform a head-tilt, chin-lift maneuver B) Request ALS assistance C) Apply supplemental oxygen D) Start positive pressure ventilation Answer: C Diff: 2 Page Ref: 1060 Objective: 36-10

2 Copyright © 2018 Pearson Education, Inc.

7) An unresponsive 10-year-old male was struck by a car while riding his bike. He has blood on his face and presents with snoring respirations. Respirations are approximately 10 breaths per minute, with cyanosis noted to his extremities. Manual spine motion restriction to the neck is being held by Emergency Medical Responders. What should you do next? A) Perform a jaw-thrust maneuver B) Start positive pressure ventilation C) Suction the airway D) Quickly assess for life-threatening injuries Answer: A Diff: 2 Page Ref: 1059 Objective: 36-7 8) An elderly female fell and is now very confused. You are having a problem getting her past medical history. Her daughter states that she is typically alert and very well oriented. What is your next best action? A) Transport the patient without delay B) Contact the patient's doctor for the history C) Transport her to a hospital that has cared for her before D) Ask the patient's daughter for the history Answer: D Diff: 1 Page Ref: 1060 Objective: 36-10 9) You are presenting information on emergency care to a class of pregnant women. A participant asks you what the most frequent cause of injury is to a woman during pregnancy. What is the correct response? A) "Motor vehicle collisions account for about half of all injuries sustained during pregnancy." B) "Falls occur about two times as often as any other cause of maternal injury during trauma." C) "Domestic violence, sadly enough, is the most common cause of injury to a pregnant female." D) "Sexual assault accounts for almost 80 percent of all injuries to females sustained while pregnant." Answer: A Diff: 1 Page Ref: 1056 Objective: 36-5

3 Copyright © 2018 Pearson Education, Inc.

10) A female patient who is 36 weeks pregnant with twins tripped in a parking lot, hitting her head, her wrist, and her abdomen. She is alert and oriented and complains of a headache and abdominal and wrist pain. She denies shortness of breath and has an SpO2 of 96% on ambient air. Assessment reveals a large abrasion and hematoma to her forehead and abdomen as well as deformity to her left wrist. Which intervention would best treat the injuries the fetuses may have suffered? A) Securing the patient to a long spine board B) Covering the abdominal abrasion to prevent infection C) Applying an ice pack to the wrist to decrease pain D) Administering high-concentration oxygen through a nonrebreather mask Answer: D Diff: 2 Page Ref: 1057 Objective: 36-4 11) An unrestrained female patient, who was involved in a minor motor vehicle collision, refuses EMS care or transport. She is seven months pregnant and is alert and oriented. When asked, she denies any complaint. Your best action would be to: A) Have her follow up with her obstetrician B) Contact medical direction for instructions C) Make contact with her obstetrician D) Have the police witness her signing the refusal-of-care form Answer: B Diff: 2 Page Ref: 1056 Objective: 36-4 12) When assessing and treating a traumatically injured female in her third trimester of pregnancy, the EMT must remember which important point? A) Securing her to a long spine board increases her risk for vomiting B) The maternal heart rate naturally decreases as the due date approaches C) Optimal care for the baby will come from caring for the mother D) An increase in blood volume makes the pregnant woman naturally resistant to shock Answer: C Diff: 3 Page Ref: 1057 Objective: 36-5 13) A female who is 32 weeks pregnant was involved in a motor vehicle collision. She is currently secured to the long spine board, with other spinal motion restriction precautions taken. Which patient statement should concern the EMT most? A) "I think that I just had a contraction." B) "I forgot to tell you that I am having twins." C) "My back is really hurting from being on this board." D) "I have not taken prenatal vitamins for a week." Answer: A Diff: 2 Page Ref: 1056 Objective: 36-4

4 Copyright © 2018 Pearson Education, Inc.

14) A female patient who is 35 weeks pregnant tripped and fell down 12 stairs. She now complains of back pain, numbness to her legs, and nausea. She is secured to a long spine board with spinal motion restriction precautions placed and is being non-emergently transported to the hospital for evaluation. En route, the patient exclaims that she is having contractions and thinks that her water may have broken. Which intervention should you perform immediately? A) Provide rapid transport and continue monitoring B) Examine her genitalia for crowning or bleeding C) Tilt the long spine board to the left side D) Release her from the long spine board and prepare for delivery Answer: B Diff: 2 Page Ref: 1056 Objective: 36-4 15) After being assaulted by her boyfriend, an 18-year-old female who is 33 weeks pregnant is found unresponsive in her bedroom. She appears to have been beaten in the head and abdomen, as evidenced by multiple hematomas and lacerations to these areas. Which assessment finding should concern the EMT most and should be addressed first? A) Rapid heart rate B) Bright red vaginal bleeding C) Abdominal contusions D) SpO2 of 93% on room air Answer: D Diff: 2 Page Ref: 1057 Objective: 36-4 16) A leading cause of death in children younger than 14 years could be prevented by: A) Better parental supervision B) Wearing helmets while riding motorcycles C) Safer driving of vehicles D) Prevention of child abuse Answer: C Diff: 1 Page Ref: 1057-1058 Objective: 36-10 17) When treating a pediatric trauma patient, what is important for the EMT to remember? A) The elevated metabolism of a child causes this patient to retain heat, so a blanket should not be applied to a pediatric patient when in shock B) Since the body of a child is smaller than an adult's body, there is less chance of multisystem trauma C) The flexibility of the chest wall in a child can allow for internal injury without obvious external signs D) Because children tend to be healthier than adults, they are able to compensate longer when in shock Answer: C Diff: 2 Page Ref: 1058 Objective: 36-6 5 Copyright © 2018 Pearson Education, Inc.

18) The EMT is correctly using the pediatric assessment triangle on a 3-year-old male trauma patient when he: A) Quickly determines the circulation based on skin color B) Uses it to compare peripheral pulses with central pulses C) Carefully examines the chest for signs of retractions D) Makes a conscious effort to look for bruises to the patient's body Answer: A Diff: 2 Page Ref: 1058-1059 Objective: 36-7 19) The EMT is correctly assessing a 2-year-old male patient who is unresponsive after being assaulted by the mother's live-in boyfriend when he: A) Checks the patient's brachial pulse B) Recognizes a respiratory rate of 30 breaths per minute as abnormal C) Takes the time to get a blood pressure D) Checks capillary refill time to help determine perfusion Answer: D Diff: 2 Page Ref: 1059 Objective: 36-6 20) You suspect that your patient–a lethargic 6-month-old baby–is the victim of child abuse. When performing the primary assessment, which assessment parameter should you use to quickly establish his perfusion? A) Heart rate B) Capillary refill C) Presence of a radial pulse D) Blood pressure Answer: B Diff: 2 Page Ref: 1059 Objective: 36-6 21) You observe a traumatically injured 3-year-old female patient being secured to a long spine board with no padding behind the neck, shoulders, or back during a spine motion restriction intervention. What is the main concern with this oversight? A) Airway compromise B) Extension of the cervical spine C) Bruising to the back D) Pain and discomfort Answer: A Diff: 2 Page Ref: 1059 Objective: 36-6

6 Copyright © 2018 Pearson Education, Inc.

22) The mother of a 4-year-old boy states that he fell off the top bunk bed and hit his head. After falling, he appeared to seize for approximately 30 seconds. Currently, he has nonpurposeful movement to painful stimuli, with a patent airway and slow respirations. His radial pulse is intact, and his skin is warm and dry. You note urinary incontinence. After taking manual spine motion restriction precautions, what should you do next? A) Insert an oropharyngeal airway and provide high-concentration oxygen B) Perform a jaw-thrust maneuver and start positive pressure ventilation C) Apply oxygen, provide full spinal motion restriction precautions, and then initiate rapid transport D) Perform a head-tilt, chin-lift maneuver and administer high-concentration oxygen Answer: B Diff: 2 Page Ref: 1059 Objective: 36-6 23) You are assessing an 86-year-old male who fell down a flight of stairs after getting out of bed to use the bathroom. He complains of severe head, neck, and back pain. Which question is most appropriate to ask this patient? A) "Were you holding onto the railing?" B) "Were you confused when you fell?" C) "Do you know what caused you to fall?" D) "Do you seem to fall a lot?" Answer: C Diff: 2 Page Ref: 1060 Objective: 36-8 24) Starting at which age should the EMT consider transporting any injured patient to a trauma center? A) 70 years of age B) 65 years of age C) 60 years of age D) 55 years of age Answer: D Diff: 2 Page Ref: 1060 Objective: 36-8 25) An 81-year-old woman with severe kyphosis and osteoporosis fell and now complains of neck pain, back pain, and nausea. You can palpate a lateral displacement of a cervical vertebrae. Given the patient's past medical history, what is most appropriate? A) Transfer her to the stretcher and transport in a position of comfort B) Place her in a supine position on the long spine board and pad all voids C) Place her on a backboard without straps or a cervical collar and transport her in supine position D) Position her on her left side and secure her to the long board Answer: B Diff: 2 Page Ref: 1060 Objective: 36-8 7 Copyright © 2018 Pearson Education, Inc.

26) You have been called to a nursing home for a confused 91-year-old male patient who became dizzy and fell, striking his head on the floor. Which statement made by the nurse should be of most concern to you? A) "I just checked his blood sugar and it is 154 mg/dL, but it was 243 at last shift change." B) "He did not have any of his medications today." C) "He has a heart valve problem and takes Coumadin to help prevent his blood from clotting." D) "He is normally confused, and his family just made him a DNR." Answer: C Diff: 3 Page Ref: 1060 Objective: 36-9 27) Assessment of a 79-year-old female with a long history of medical complaints reveals tenderness and deformity to her left hip. Staff at the personal care home state that she slipped in some water and fell onto her buttocks. Currently, she is alert and oriented and describes the pain as 10/10. Regarding care of her leg, what is most appropriate care? A) Apply traction with a traction splint until the pain subsides B) Straighten the leg and splint it in that position C) Determine the presence of a pedal pulse, and then apply the traction splint D) Maintain the leg in the position found and transfer the patient to a long spine board Answer: D Diff: 2 Page Ref: 1060 Objective: 36-8 28) An 89-year-old female with dementia, who is being cared for by her daughter, fell and now exhibits deformity to her left upper arm. As you attempt to splint her arm, she continually screams and pulls away from you. After several attempts, your best course of action should be to: A) Contact advanced life support for assistance B) Coach the daughter on how to help apply the splint C) Transport the patient without a splint and document that splinting was attempted D) Gently restrain the patient and apply the splint Answer: B Diff: 2 Page Ref: 1061 Objective: 36-9 29) You are called to a nursing home for an elderly male patient who is unresponsive. On arrival, you find the patient in a wheelchair with a noticeable hematoma to his forehead and a skin tear to his left wrist. Staff states that the patient had an episode of chest pain just before suddenly passing out and falling to the ground. What should you do immediately? A) Apply spine motion restriction precautions B) Initiate oxygen and look for other injuries the patient may have suffered C) Assume a cardiac event has occurred and apply oxygen D) Perform the primary assessment and apply supplemental oxygen Answer: A Diff: 2 Page Ref: 1060 Objective: 36-8

8 Copyright © 2018 Pearson Education, Inc.

30) Which statement accurately describes multisystem trauma? A) Multisystem trauma is the leading cause of death for persons older than 65 years B) The multisystem trauma patient is less likely to develop shock if otherwise healthy C) Multisystem trauma has a noted higher incidence of morbidity and mortality D) The definitive care for multisystem trauma is cardiopulmonary resuscitation Answer: C Diff: 2 Page Ref: 1054 Objective: 36-1 31) The EMT would recognize which patient as suffering from multisystem trauma? A) A pregnant female with a single gunshot wound to the head B) A 43-year-old male with extremity trauma and shock from a motor vehicle collision C) A 66-year-old female with a deformed arm after having a syncopal episode D) A 23-year-old female with a fractured clavicle sustained in a soccer game Answer: B Diff: 2 Page Ref: 1054 Objective: 36-1 32) Which statement is true regarding care of the multisystem trauma patient? A) Any threat to life should be managed as it is identified B) All care should be provided en route to the hospital C) Spinal motion restriction precautions can be foregone if the patient is near death D) Advanced life support by paramedics is the definitive care for multisystem trauma patients Answer: A Diff: 2 Page Ref: 1055 Objective: 36-2 33) A 57-year-old male has been involved in a motor vehicle collision and is entrapped in his car. He is unresponsive, with agonal gurgling respirations and blood coming from his nose. His respirations show poor effort and are inadequate. The radial pulse is rapid and weak, and the skin cool and diaphoretic. An Emergency Medical Responder is holding manual spinal motion restriction. Which action would you perform first? A) Mechanical extrication from the vehicle B) Application of a cervical collar C) Control a nasal hemorrhage D) Suction the airway with a catheter Answer: D Diff: 2 Page Ref: 1060 Objective: 36-2

9 Copyright © 2018 Pearson Education, Inc.

34) Which actions by the EMS crew have met the platinum ten minutes criteria? A) Assessed, provided spinal motion restriction precautions, and initiated transport within 10 minutes of arriving on scene B) Arrived on scene and by the patient's side within 10 minutes of being dispatched C) Stabilized the patient and called for ALS assistance within 10 minutes of arriving on scene D) Completed the assessment and treated any life threats within 10 minutes of the incident occurring Answer: A Diff: 2 Page Ref: 1055 Objective: 36-3 35) When treating a pregnant female who was involved in a traumatic event during her third trimester, the EMT must remember: A) The amniotic fluid protects the baby from any injury to the mother's abdomen B) An elevated heart rate seen in shock may not be discernable from the elevated heart rate from the patient's increased blood volume C) The diaphragm tends to be lower during pregnancy, making internal chest and lung injury more likely D) Gastric mobility increases during pregnancy, posing an increased risk of vomiting and aspiration Answer: B Diff: 3 Page Ref: 1056 Objective: 36-4 36) You have just arrived by the side of a 23-year-old female who is 37 weeks pregnant and apneic and pulseless after being shot in the abdomen. Regarding care of this woman, which statement would you provide to other rescuers on scene? A) "Sadly, there is nothing that can be done, I will call medical direction and inform the doctor." B) "I am going to call for ALS backup; it is her and her baby's only chance for survival." C) "I need someone to listen for fetal heart tones to see if the baby is still alive." D) "Waiting for ALS backup is not beneficial; we need to provide CPR and get her to the hospital now!" Answer: D Diff: 3 Page Ref: 1057 Objective: 36-5 37) A 31-year-old unrestrained pregnant female was involved in a motor vehicle collision in which she struck the steering wheel. She now complains of chest and abdominal pain. She is conscious, alert, and oriented with no deficits noted to the airway, breathing, or circulation. When asking her questions regarding her status, which questions should you ask first? A) "Have there been any problems with previous pregnancies?" B) "Are you receiving prenatal care from an obstetrician?" C) "Do you feel as though you are having contractions?" D) "Is there a reason why you were not wearing a seat belt?" Answer: C Diff: 3 Page Ref: 1056 Objective: 36-4 10 Copyright © 2018 Pearson Education, Inc.

38) A 37-year-old woman, who is four months pregnant, complains of abdominal and pelvic pain after being assaulted in her home by a live-in boyfriend. During the assault, he struck her in the head, chest, and abdomen with his fists and a chair as she lay on the floor. There are no obvious threats to the airway, breathing, or circulation. Her vital signs are pulse, 124 beats/min; respiration, 22 breaths/min; blood pressure, 114/68 mmHg; and SpO2, 98% on room air. Which intervention is needed for this patient? A) Secure the patient to the long board, with it tilted left for transport B) Provide immediate transport with care such as spinal motion restriction precautions provided en route C) Provide calming reassurance and transport the patient quietly in a position of comfort D) Apply a nonrebreather mask with high-concentration oxygen Answer: D Diff: 3 Page Ref: 1057 Objective: 36-4 39) How would you tell a pregnant woman to restrain herself when traveling in a car? A) Wear all seat belts normally, as you would if not pregnant B) Use the shoulder restraint but not the lap belt to prevent abdominal injury C) Avoid the belt system as long as your car has front and side air bags in place D) The lap belt by itself should be adequate, as long as the car is equipped with air bags Answer: A Diff: 1 Page Ref: 1057 Objective: 36-4 40) You are by the side of an 11-month-old female with a decreased level of consciousness. Caregivers will not provide an accurate history of how the child came to this condition, only that she was difficult to wake up this morning, about 30 minutes ago. When assessing the child, which finding would be most immediately concerning? A) Bruise to the head B) Lack of crying as you assess her C) Bradycardic heart rate D) Healing bruises on the lower legs Answer: C Diff: 3 Page Ref: 1059 Objective: 36-6 41) The EMT is correctly assessing the perfusion status of a 10-month-old when she: A) Obtains a blood pressure B) Feels for a brachial pulse C) Looks for cyanosis in the extremities D) Observes the characteristics of the fontanelles Answer: B Diff: 3 Page Ref: 1059 Objective: 36-6

11 Copyright © 2018 Pearson Education, Inc.

42) The young pediatric patient is more prone to head injury because: A) The skull is still cartilaginous and remains weak in structure B) The chest muscles are weak and cannot support the head C) The head is proportionately heavier and the neck muscles are weaker D) The brain is smaller and floats within the larger skull Answer: C Diff: 3 Page Ref: 1058 Objective: 36-6 43) You are caring for a crying 30-month-old male who fell 16 feet from a second-story window, landing on a patch of soft dirt and leaves. He is alert and oriented and seems to have pain to his left shoulder and arm. Which instruction would you give to the team of EMS providers caring for this patient? A) "When we get him on the long board, place padding under his head." B) "I will need a blood pressure every 5 minutes to see if he is in shock." C) "It is important to keep him cool in the event he is in shock." D) "Let's make sure his SpO2 is greater than 95%." Answer: D Diff: 3 Page Ref: 1059 Objective: 36-6 44) An 84-year-old female fell down six steps, landing on carpet. She now complains of abdominal and pelvic pain. Her airway is open, breathing adequate, and radial pulse intact. Skin is cool and dry, with a noted skin tear to the left elbow with minor bleeding. EMRs report the following vital signs: pulse, 84 beats/min; respirations, 20 breaths/min; blood pressure, 126/68 mmHg; and SpO2, 97% on room air. Her family states that the patient is acting normally. The hips are easily moved and do not appear broken. The patient does not want to go to the hospital and is asking to refuse transport. Your next action would be to: A) See which medications she is taking B) Clean and wrap the skin tear to prevent infection C) Ask family members for their input D) Contact medical direction for authorization to refuse Answer: A Diff: 3 Page Ref: 1059 Objective: 36-8 45) When treating an injured geriatric patient, it is very important to: A) Always administer high-concentration oxygen B) Achieve an oxygen saturation of at least 95% C) Place the patient in a sitting position to promote adequate breathing D) Avoid use of a nonrebreather mask, rather oxygenate the patient via a nasal cannula Answer: B Diff: 2 Page Ref: 1060 Objective: 36-8

12 Copyright © 2018 Pearson Education, Inc.

46) Family has called you for a 78-year-old female with a history of dementia, because she appears more confused than normal. The primary assessment reveals no life-threatening conditions. Her vital signs are pulse, 88 beats/min; respirations, 22 breaths/min; blood pressure, 134/66 mmHg; and SpO2, 95% on room air. Aside from the dementia, the patient has a history of hypertension, COPD, and breast cancer, which resulted in the removal of her right breast 11 years ago. Lately, she has had an upper respiratory infection and is telling family that she has a headache. Your first question to family should be: A) "It sounds like the dementia is getting worse, do you agree?" B) "Do you know if she fell recently and hit her head?" C) "Is there any possibility of diabetes and a low blood sugar level?" D) "Does she take any medications for the high blood pressure?" Answer: B Diff: 3 Page Ref: 1061 Objective: 36-8 47) An 85-year-old male with a history of an irregular heartbeat, diabetes, renal failure, and stroke fell while getting out of bed, striking his head on the nightstand. He is alert and oriented and denies any complaint. Vital signs are all within normal limits. There is a minor abrasion on his forehead, but he denies any associated pain or headache. What should be your next priority? A) Ask the patient if he ate breakfast and check his blood sugar B) Inquire whether he is up-to-date on his tetanus immunization C) Determine whether he takes blood thinners for the irregular heartbeat D) Ask family members if he has developed dementia yet Answer: C Diff: 3 Page Ref: 1060 Objective: 36-8 48) What is the first golden principle for prehospital multisystem trauma care? A) The on-scene time is critical and should not be prolonged beyond 10 minutes B) Determine any additional resources needed C) Ensure safety of the rescue personnel and the patient D) Identify and manage life threats Answer: C Diff: 2 Page Ref: 1054 Objective: 36-3

13 Copyright © 2018 Pearson Education, Inc.

Prehospital Emergency Care, 11e (Mistovich et al.) Chapter 37 Obstetrics and Care of the Newborn 1) Why is the uterus essential to a healthy pregnancy and childbirth? A) It permits the excretion of waste from the baby to the mother B) It protects abdominal organs during fetal growth C) It manufactures specific blood cells needed for fetal immunity D) It powerfully contracts to force the fetus from the mother's body following gestation Answer: D Diff: 2 Page Ref: 1068 Objective: 37-2 2) Which statement regarding the role and function of the cervix during pregnancy is true? A) The cervix constricts near the time of delivery to prevent a premature birth B) The cervix becomes plugged with mucus to prevent contamination of the uterus during gestation C) The EMT can palpate the cervix to measure the contractions that the mother is having during the third trimester D) The "bloody show" comes from the cervix and confirms pregnancy in the first trimester Answer: B Diff: 2 Page Ref: 1069 Objective: 37-1 3) An EMT is presenting a continuing education class on the care of the pregnant female. When discussing the amniotic sac, he is correct to emphasize which point? A) The amniotic sac contains fluid and surrounds and protects the uterus B) The amniotic sac contains a fluid called Wharton's jelly that protects the baby while in the uterus C) The amniotic sac must rupture during the second trimester of pregnancy for the third trimester to progress D) The amniotic sac protects and insulates the baby during gestation Answer: D Diff: 2 Page Ref: 1069 Objective: 37-1 4) You have been called to assist another crew with the birth of a baby. On scene, another EMT informs you that the mother's perineum tore and is bleeding heavily. You would: A) Place a cold pack on the mother's abdomen just below the umbilicus B) Apply a sterile gauze dressing with gentle pressure above the vagina C) Place a sterile dressing between the mother's vagina and rectum D) Insert sterile gauze into the vagina and leave it in place until the bleeding stops Answer: C Diff: 2 Page Ref: 1091 Objective: 37-9

1 Copyright © 2018 Pearson Education, Inc.

5) What are the stages of labor in the order in which they occur? A) Uterine, delivery, and recovery B) Dilation, expulsion, and placental delivery C) Uterine, expulsion, placental, and recovery D) Dilation, delivery, placental, and afterbirth Answer: B Diff: 1 Page Ref: 1081 Objective: 37-8 6) The EMT should recognize a full-term pregnancy has occurred in which patient? A) A 21-year-old woman who is 39 weeks pregnant and feeling dizzy B) A 40-year-old woman beginning her third trimester but whose water just broke C) A 25-year-old woman who is 220 days into pregnancy and complaining of back pain D) A 33-year-old woman who is pregnant with twins and in the eighth month of pregnancy Answer: A Diff: 1 Page Ref: 1069-1070 Objective: 37-9 7) Upon arrival at the home of a woman in labor, a midwife on scene tells you the patient is 7 cm dilated and is complaining of painful contractions. Which stage of labor should the EMT identify? A) First stage B) Expulsion C) Third stage D) Uterine Answer: A Diff: 1 Page Ref: 1081 Objective: 37-8 8) Which statement by the patient best indicates that the patient is in the second stage of labor? A) "The contractions are beginning to hurt now." B) "I think that my water just broke." C) "I feel like I have to move my bowels." D) "It feels good not to have to push anymore." Answer: C Diff: 2 Page Ref: 1083 Objective: 37-8

2 Copyright © 2018 Pearson Education, Inc.

9) Five minutes after delivering a baby, the mother feels the sudden urge to push and a gush of blood comes from the vagina. Given that the mother is not delivering twins, which stage of labor should the EMT recognize? A) Fourth B) Placental C) Second D) Expulsion Answer: B Diff: 2 Page Ref: 1086 Objective: 37-8 10) Prior to delivery, the EMT prepares to create a sterile field around the patient's vaginal opening. This is best accomplished by: A) Placing sheets from the OB kit over both the patient's legs and her abdomen B) Using towels from the patient's residence to wrap each leg and cover her abdomen C) Using a sheet from the patient's bed to cover her abdomen and groin D) Placing a sheet from the OB kit under the patient's hips and another over her abdomen and legs Answer: D Diff: 2 Page Ref: 1088 Objective: 37-9 11) The EMT would use the sterile scissors found in the OB kit to cut: A) Towels to create a sterile field B) The umbilical cord C) The patient's clothing D) The amniotic sac if it is not yet ruptured Answer: B Diff: 1 Page Ref: 1090 Objective: 37-11 12) A 25-year-old female presents with abdominal pain. She appears thin and healthy, but in obvious distress. Of the many questions the EMT may ask, which one should the EMT ask early during the patient interview? A) "How many times have you been pregnant?" B) "Is there any possibility you are pregnant?" C) "Are you having any vaginal discharge?" D) "Are your menstrual periods regular?" Answer: B Diff: 2 Page Ref: 1078 Objective: 37-7

3 Copyright © 2018 Pearson Education, Inc.

13) A female patient is described as primigravida. Consequently, the EMT would recognize that: A) She is pregnant for the first time B) She has only one child C) She has never been pregnant D) She has been pregnant before but miscarried Answer: A Diff: 2 Page Ref: 1078 Objective: 37-2 14) You are assessing a female patient who informs you that she has been pregnant four times before but delivered only once. What would be a correct way to document that information on the prehospital care report? A) Para IV; gravida I B) Gravida III; para I C) Para III; gravida I D) Gravida IV; para I Answer: D Diff: 2 Page Ref: 1078 Objective: 37-2 15) When asked, a young female with abdominal pain replies that she is not sure if she is pregnant. Given this response, which question would provide the best information to determine if the patient may be pregnant? A) "Do you feel pressure in your belly area?" B) "When was your last normal menstrual period?" C) "Have you been vomiting in the morning?" D) "Does it feel like the last time you were pregnant?" Answer: B Diff: 2 Page Ref: 1078 Objective: 37-2 16) When assessing a 27-year-old patient who is eight months pregnant, which statement made by the patient should the EMT be most concerned about? A) "I had some belly cramps two weeks ago." B) "I was a drug addict when I was a teenager." C) "I have not been able to see the doctor this pregnancy at all." D) "My last baby came two weeks late." Answer: C Diff: 3 Page Ref: 1080; Figure 37-8A Objective: 37-7

4 Copyright © 2018 Pearson Education, Inc.

17) You have completed the assessment of a 32-year-old patient who is starting her seventh month of pregnancy. During your assessment, which finding should you be most concerned about? A) Several painless contractions B) Increased desire to eat C) Increased weight over the past month D) Intermittent vaginal bleeding Answer: D Diff: 2 Page Ref: 1073 Objective: 37-7 18) A woman informs you that she is eight months pregnant and fatigues easily. She is apprehensive because when she lies down, she gets dizzy and feels as though she is going to vomit. What is the EMT's best response? A) "We really need to get to the hospital right away to make sure that your baby is okay. This does not sound good." B) "As long as the baby is still moving, you are okay. Just try to get through one more month." C) "Check with your obstetrician, but you may want to rest and sleep on your left or right side." D) "You should really call your doctor. During the eighth month of pregnancy, energy levels usually increase." Answer: C Diff: 3 Page Ref: 1076 Objective: 37-4 19) You have been dispatched to a residence for a female patient who is dizzy and "passing out." An Emergency Medical Responder meets you at the door and reports that the patient is lying in bed and is nine months pregnant. Her pulse rate is 112 beats per minute and her blood pressure is 84/50 mmHg. Respirations are 24 breaths per minute and she has a room air pulse oximetry reading of 97%. What should you do as soon as you reach the patient's side? A) Recheck the blood pressure B) Roll her onto her left side C) Determine any complications of the pregnancy D) Start positive pressure ventilation Answer: B Diff: 2 Page Ref: 1076 Objective: 37-6

5 Copyright © 2018 Pearson Education, Inc.

20) A female patient who is eight and a half months pregnant states that she does not feel well and is weak with clammy skin. She denies chest pain, shortness of breath, and dizziness. She states that she did vomit earlier in the morning and still feels somewhat nauseated. She is alert and oriented and has a patent airway. She is breathing 18 times per minute and has a heart rate of 88 beats per minute. Her blood pressure is 116/70 mmHg and her SpO2 reading is 95% on room air. The most appropriate care for this patient should include: A) Oxygen at 15 lpm via a nonrebreather face mask B) Rapid transport to a hospital with specialized obstetric services C) Quiet and calm transport in the supine position D) Refusal-of-transport form signed with suggested follow-up with her obstetrician Answer: A Diff: 2 Page Ref: 1079 Objective: 37-7 21) You have been called for a 16-year-old female with vaginal bleeding. On scene, you determine the patient to be pregnant and bleeding for 3 hours. Which statement would indicate proper care of the patient? A) "I am going to place this piece of sterile gauze into your vagina to try to stop the bleeding." B) "I know that it is uncomfortable, but try to keep your legs apart so the blood can flow out of your body." C) "I am going to put this sanitary napkin between your legs to collect the blood." D) "I am going to insert my gloved hand into your vagina and apply pressure to try to stop the bleeding from the inside." Answer: C Diff: 2 Page Ref: 1079 Objective: 37-5 22) You arrive at the side of a pregnant patient in her third trimester who has had vaginal bleeding for several hours. Observation reveals several blood-soaked towels on the bed. She is alert and oriented with a patent airway and adequate breathing. Her pulse is 88 beats per minute and her blood pressure is 104/66 mmHg. In caring for this patient you would: A) Provide rapid transport in a supine position B) Clean the outside of the vagina with sterile water and peroxide C) Collect all towels with blood and transport them with the patient D) Prepare her for the possibility that she may have miscarried the baby Answer: C Diff: 3 Page Ref: 1079 Objective: 37-5

6 Copyright © 2018 Pearson Education, Inc.

23) A 31-year-old female in her third trimester of pregnancy lost her balance and fell down a flight of stairs. Treatment included spinal motion precautions and oxygen therapy. When transporting her, the EMT should: A) Transport her to her obstetrician's office B) Tilt the long spine board to the left C) Transport her in a supine position without a cervical collar in place D) Transport her on her right side Answer: B Diff: 2 Page Ref: 1079 Objective: 37-6 24) You are transporting a young female who just had a miscarriage at home. During the transport the woman weeps continually. Which statement is most appropriate given the situation? A) "If you want another child, wait a few months and then get pregnant again." B) "It is best to stay strong; your other child needs your support." C) "I think in the long run you will be okay and things will work out." D) "Is there anything that I can do to help you right now?" Answer: D Diff: 3 Page Ref: 1072 Objective: 37-5 25) Which patient should the EMT be most suspicious of having a spontaneous abortion? A) A 16-year-old woman, 24 weeks pregnant, with painless contractions B) A 21-year-old woman, 6 weeks pregnant, passing vaginal blood clots C) A 32-year-old woman, in the third trimester, passing foul-smelling and bloody vaginal discharge D) A 38-year-old woman, 36 weeks pregnant, having abdominal pain with painless vaginal bleeding Answer: B Diff: 2 Page Ref: 1072 Objective: 37-1 26) You are responding to a low-income apartment complex for a 16-year-old female who is five weeks pregnant with vaginal bleeding. When mentally reviewing possible causes of hemorrhage for this patient, which condition are you most likely to encounter given the information provided thus far? A) Ruptured uterus B) Dysfunctional placenta C) Drug addiction D) Spontaneous abortion Answer: D Diff: 3 Page Ref: 1072 Objective: 37-5

7 Copyright © 2018 Pearson Education, Inc.

27) While en route for a call involving a pregnant patient, an Emergency Medical Responder (EMR) on scene contacts you and reports that the patient is actively seizing. The EMR reports that she is seven months pregnant and has not been feeling well for the past three days (per family). Which instruction is critical for you to relay to the EMR? A) Restrain the patient on her back or left side B) Place a tongue blade in the patient's mouth C) Provide oxygen at 15 liters per minute D) Determine if the patient has a seizure history Answer: C Diff: 2 Page Ref: 1076 Objective: 37-6 28) Which statement made by the EMT indicates an understanding of the relationship between seizures and pregnancy? A) "Prolonged seizures can easily cause the mother and baby to become hypoxic." B) "A short seizure in the pregnant female with a seizure history generally does not require transport." C) "Seizure activity in the pregnant female is typically not life threatening unless the patient has had seizures in the past." D) "If the pregnant patient is seizing, the EMT must quickly make a decision to administer or withhold her antiseizure medications." Answer: A Diff: 2 Page Ref: 1075 Objective: 37-6 29) You have arrived at the side of a 35-year-old female who is 37 weeks pregnant and in labor. Your exam reveals crowning with contractions 60 seconds apart. Your immediate action would be to: A) Place the patient on the cot in the left lateral recumbent position, and then proceed with emergency transport B) Have the patient hold her legs together, and then move her to the ambulance for delivery of the baby C) Obtain vital signs, and then move the patient to the ambulance for a nonemergency transport D) Place the patient on her back, and then open and prepare the OB kit Answer: D Diff: 2 Page Ref: 1087 Objective: 37-11

8 Copyright © 2018 Pearson Education, Inc.

30) You have determined that you will need to perform a field delivery. Which instructions to the patient show that the EMT is properly positioning her? A) "Please lie on your back, draw your knees upward, and spread your legs apart." B) "I am going to place you on your left side with your legs up until the baby's head is visible." C) "Please lie back, and then elevate both legs into the air. Your husband will help you hold them up." D) "I'd like you to lie flat on the bed and bend your right leg while keeping your left leg straight." Answer: A Diff: 2 Page Ref: 1088 Objective: 37-11 31) You are assessing a 29-year-old pregnant female who has signs of imminent delivery. Her past medical history includes HIV infection. Compared to a patient with no infectious disease, how will standard precautions differ for this patient? A) Gloves, a gown, and goggles will be used for this patient; only gloves and a gown are required for a healthy patient in labor B) Gloves are needed for a noninfectious patient in labor; goggles and gloves must be worn for the patient with HIV C) A HEPA filter mask should be used for the patient with HIV, but not for the noninfectious patient D) Gloves, a gown, and eye protection should be worn for both the patient with HIV and the noninfectious patient Answer: D Diff: 2 Page Ref: 1087-1088 Objective: 37-9 32) You are delivering a baby in the patient's house. As the baby's head appears at the opening of the vagina, you notice that the amniotic membrane is still intact. You would: A) Cut the sac with the sterile scissors in the OB kit B) Transfer the patient to the stretcher for immediate transport C) Continue with the delivery of the baby D) Rip open the amniotic sac with your fingers Answer: D Diff: 2 Page Ref: 1088 Objective: 37-11 33) While delivering a baby, you note that the umbilical cord is wrapped around the baby's neck. You would: A) Clamp and cut the cord to prevent strangulation of the newborn B) Stop the delivery and transport C) Attempt to slip the cord over the baby's shoulders or head D) Contact medical direction for directions Answer: C Diff: 2 Page Ref: 1089 Objective: 37-12 9 Copyright © 2018 Pearson Education, Inc.

34) After the EMT observes that secretions are interfering with the baby's breathing, which action indicates that the EMT is properly suctioning the newborn? A) He suctions the nose first, followed by the mouth B) He inserts the bulb syringe deep into the oropharynx to get thick secretions C) He uses the portable suction unit on the "high" setting D) He squeezes the bulb syringe prior to placing it in the baby's nose Answer: D Diff: 2 Page Ref: 1089 Objective: 37-11 35) As soon as the baby is delivered from the vaginal canal, it is critical that the EMT immediately: A) Clamp and cut the umbilical cord B) Initiate drying and warming of the newborn C) Perform the Apgar assessment D) Obtain a pulse oximetry reading from the newborn's right hand Answer: B Diff: 2 Page Ref: 1089-1090 Objective: 37-11 36) You are in the process of performing a field delivery and have just delivered the baby's head. What should you do next? A) Turn the baby clockwise and continue delivery B) Check the neck for the umbilical cord C) Suction the baby's nose, and then the mouth D) Suction the baby's mouth, and then the nose Answer: B Diff: 2 Page Ref: 1089 Objective: 37-11 37) When should the EMT instruct his partner to cut the umbilical cord? A) Immediately after delivery of the placenta B) Following delivery and after the baby has been dried off C) Just prior to passage of the legs through the vaginal canal D) After the 1-minute Apgar score has been obtained Answer: B Diff: 2 Page Ref: 1089-1090 Objective: 37-11 38) Which action shows that the EMT is properly caring for the umbilical cord after delivery? A) The cord is cut 1 inch from the baby's abdomen B) The clamps are removed from the cord after cutting C) The cord is clamped but not cut until arrival at the hospital D) A 6-inch segment of cord is still connected to the baby Answer: D Diff: 2 Page Ref: 1090 Objective: 37-11 10 Copyright © 2018 Pearson Education, Inc.

39) When cutting the umbilical cord, how should the baby be positioned? A) Six inches below the vaginal opening B) Upright and on the mother's chest (if possible) C) Level with the mother's uterus D) Prone and in a head-down position Answer: C Diff: 2 Page Ref: 1090 Objective: 37-11 40) Which statement regarding the placenta, made by your EMT partner while reviewing the OB/GYN protocols, is most accurate? A) "The EMT must deliver the placenta by applying gentle pressure to the abdomen after the baby has been born." B) "The placenta is full of bacteria and should not be transported in the ambulance near the baby." C) "It is essential that the mother not be moved until the placenta has been delivered." D) "The placenta typically delivers itself within 20 minutes of the birth of the baby." Answer: D Diff: 2 Page Ref: 1090 Objective: 37-8 41) After successfully delivering a baby, the EMT notes that the protruding umbilical cord is lengthening and a small gush of blood came out of the vagina. The appropriate action would be to: A) Massage the patient's uterus B) Apply a sanitary pad over the vaginal opening C) Prepare for delivery of the placenta D) Open the vaginal canal using obstetric forceps Answer: C Diff: 2 Page Ref: 1086 Objective: 37-8 42) It has been 30 minutes and the placenta has yet to deliver. The EMT should: A) Continue to monitor the mother and baby B) Apply gentle traction to the protruding umbilical cord C) Place the mother in a knee-chest position D) Move the mother into a side-lying position Answer: A Diff: 2 Page Ref: 1091 Objective: 37-9

11 Copyright © 2018 Pearson Education, Inc.

43) After delivery, you note blood still escaping from the mother's vagina. The sanitary pads that you placed earlier between her legs continue to become soaked. Your next action would be to: A) Transport with lights and sirens B) Insert sterile gauze into the vaginal canal C) Perform a uterine massage D) Place the mother on her left side Answer: C Diff: 2 Page Ref: 1091 Objective: 37-16 44) Which action demonstrates that the EMT is correctly performing uterine massage? A) She uses the lower palms of both hands to firmly press downward on the uterus B) Both hands are positioned at the symphysis pubis and she pushes with an upward motion C) She places her hands on the upper abdomen and pushes the uterus toward the pelvis D) One hand cups the top of the uterus, with one of the hands positioned just above the symphysis pubis Answer: D Diff: 2 Page Ref: 1091 Objective: 37-16 45) The EMT shows that he understands care of the post-delivery mother when he states: A) "Allowing the baby to nurse immediately after delivery can help to control uterine bleeding." B) "We should be concerned if there is any bleeding from the vagina following delivery." C) "If blood is observed coming from the vagina after the baby has been delivered, the mother should be placed in the shock position." D) "Post-delivery hemorrhage can best be controlled by inserting a bulky dressing into the vagina." Answer: A Diff: 3 Page Ref: 1091 Objective: 37-11 46) Which single assessment finding is most consistent with a stable newborn? A) Heart rate of 140 beats per minute B) Respirations of 28 breaths per minute C) Facial grimace when irritated D) Heart rate of 100 beats per minute Answer: A Diff: 2 Page Ref: 1104 Objective: 37-17

12 Copyright © 2018 Pearson Education, Inc.

47) When should the EMT use the Apgar scoring system on a newborn? A) Before and after the baby is dried and warmed B) Ten minutes after birth and then at 20 minutes, if needed C) Just prior to delivery and then five minutes after delivery D) One and five minutes after the baby has been delivered Answer: D Diff: 2 Page Ref: 1104 Objective: 37-17 48) The Apgar scoring system is useful in determining: A) The type of resuscitation needed B) The precise age of gestation C) The newborn's overall condition D) The maturity of the newborn's heart and lungs Answer: C Diff: 2 Page Ref: 1104 Objective: 37-17 49) The components of the Apgar scoring system include: A) Airway, breathing, and circulation B) Heart rate, spontaneous activity, and respirations C) Airway, breathing, and level of consciousness D) Grimace, movement, and skin moisture Answer: B Diff: 1 Page Ref: 1104 Objective: 37-17 50) Assessment of a newborn indicates that his heart rate is 120 beats per minute. How many points should he be awarded according to the Apgar scoring system? A) 0 B) 1 C) 2 D) 3 Answer: C Diff: 2 Page Ref: 1104 Objective: 37-17 51) Assessment of a newborn indicates that she has slow and irregular respirations with a weak cry. You should award her how many points according to the Apgar scoring system? A) 0 B) 1 C) 2 D) 3 Answer: B Diff: 2 Page Ref: 1104 Objective: 13-17 13 Copyright © 2018 Pearson Education, Inc.

52) A newborn's first Apgar score was 6. His second Apgar score is 9. What does this information mean to the EMT? A) Improvement B) Need for extensive resuscitation C) Total score of 15 D) Poor heart and lung function Answer: A Diff: 2 Page Ref: 1104 Objective: 37-17 53) Which assessment finding best indicates that the newborn is significantly distressed? A) Loud crying B) Respiratory rate of 50 breaths per minute C) Apgar score of 8 D) Heart rate of 92 beats per minute Answer: D Diff: 3 Page Ref: 1105 Objective: 37-17 54) When asked by a concerned newborn parent, the EMT should indicate that an acceptable respiratory rate for the newborn is: A) 52 breaths per minute B) 24 breaths per minute C) 70 breaths per minute D) 12 breaths per minute Answer: A Diff: 1 Page Ref: 1105 Objective: 30-17 55) Assessment indicates that a term newborn's respiratory rate is 38 breaths per minute and his heart rate is 80 beats per minute after administration of supplemental blow-by oxygen. The EMT should: A) Administer high-concentration oxygen via a pediatric nonrebreather mask B) Start positive pressure ventilation with supplemental oxygen C) Continue to monitor the baby for another 30 seconds D) Start chest compressions at a rate of 120 compressions per minute Answer: B Diff: 2 Page Ref: 1106 Objective: 37-17

14 Copyright © 2018 Pearson Education, Inc.

56) The EMT shows an understanding of newborn resuscitation when she states: A) "Most newborns require aggressive resuscitation immediately after birth." B) "The EMT must use high-concentration oxygen sparingly because it can be toxic to a baby." C) "Interventions in a newborn should be performed for 30 seconds followed by reassessment." D) "CPR should be provided only when the baby is unresponsive, apneic, and pulseless." Answer: C Diff: 2 Page Ref: 1105-1106 Objective: 37-17 57) After positive pressure ventilation, a newborn's heart rate increases from 80 to 120 beats per minute. The EMT should: A) Contact medical direction for instructions B) Continue positive pressure ventilation with less oxygen use C) Insert an oral airway and resume positive pressure ventilation D) Stop positive pressure ventilation and provide blow-by oxygen Answer: D Diff: 2 Page Ref: 1106 Objective: 37-17 58) Despite positive pressure ventilation, a term newborn's heart rate has decreased from 80 beats per minute to 40 beats per minute. He is breathing spontaneously but shallowly at 40 breaths per minute and has a mottled appearance. Your next action would be to: A) Continue positive pressure ventilation B) Suction the nose and mouth C) Start external chest compressions D) Attach an AED with pediatric pads Answer: C Diff: 2 Page Ref: 1106 Objective: 37-17 59) The EMT is performing CPR on a neonate. What is the correct compression-to-ventilation ratio? A) 3 compressions to every 1 ventilation B) 5 compressions to every 2 ventilations C) 15 compressions to every 5 ventilations D) 30 compressions to every 2 ventilations Answer: A Diff: 1 Page Ref: 1106-1107 Objective: 37-17

15 Copyright © 2018 Pearson Education, Inc.

60) A 31-year-old female is in labor. After ensuring her airway, breathing, and circulation, you assess her perineum and observe the umbilical cord protruding from the vagina. However, the baby is not visible. Your next action would be to: A) Place the patient in a knee-chest position B) Carefully place the cord back inside the vagina C) Place a saline-soaked dressing over the cord D) Gently pull the cord to assist in delivery Answer: A Diff: 3 Page Ref: 1081; Figure 37-8B Objective: 37-13 61) The EMT shows that he understands the seriousness of a prolapsed umbilical cord when he states: A) "The priority when treating a patient with a prolapsed umbilical cord is keeping the cord moist." B) "A prolapsed cord that is pinched can stop the flow of oxygen to the baby and must be addressed immediately." C) "If the cord is observed protruding from the vagina, it should be clamped and cut immediately." D) "A baby can bleed to death inside the uterus if the cord is compressed for more than 10 minutes." Answer: B Diff: 3 Page Ref: 1092 Objective: 37-1 62) During which abnormal birth presentation is it permissible for the EMT to place a gloved hand into the vaginal canal? A) Placenta previa B) Multiple birth delivery C) Prolapsed umbilical cord D) Limb presentation Answer: C Diff: 3 Page Ref: 1093 Objective: 37-13 63) Assessment of a pregnant patient in labor reveals what appears to be the baby's buttocks presenting at the vaginal opening. The EMT would immediately: A) Lay the patient on her left side and administer high-concentration oxygen B) Place both hands on the abdomen and attempt to turn the baby into a head-down position C) Put the patient in semi-Fowler's position on the stretcher and immediately transport D) Place the patient in supine, head-down position, with hips elevated, and administer supplemental oxygen Answer: D Diff: 3 Page Ref: 1094 Objective: 37-12

16 Copyright © 2018 Pearson Education, Inc.

64) You have been dispatched for a young female in labor. On arrival, you find an arm protruding from the vagina. What should you do next? A) Attempt to replace the arm into the vaginal canal B) Place the mother in a knee-chest position C) Insert a gloved hand into the vagina to locate the baby's head D) Have the mother push when she feels a contraction Answer: B Diff: 3 Page Ref: 1096 Objective: 37-12 65) Which statement regarding limb presentation is true during childbirth? A) The EMT can try one time to deliver the baby B) There is little danger unless the umbilical cord is also protruding C) The baby is premature D) A cesarean section birth will likely be required Answer: D Diff: 3 Page Ref: 1096 Objective: 37-13 66) When faced with the possibility of delivering twins, the EMT must remember that: A) There will be only one umbilical cord that needs to be cut B) Twins are typically born after 40 weeks' gestation, making them larger C) Both babies are typically in the vaginal canal at the same time D) The second infant may be born breech Answer: D Diff: 2 Page Ref: 1099 Objective: 37-14 67) You have been called to a crisis shelter for a patient possibly having a baby. On scene, you find an older female patient who is developmentally challenged and cannot communicate normally. Staff states that the patient just came to the shelter and they know nothing about her, other than that she is pregnant. Assessment reveals crowning, and you proceed to deliver what appears to be a healthy baby boy. When assessing the mother after delivery, which finding should increase your suspicion that the woman may be carrying twins? A) The mother continues to bleed vaginally B) The woman's abdomen becomes soft and boggy following delivery C) A large placenta is delivered within seconds of the first baby D) The mother's abdomen remains large after delivery of the first baby Answer: D Diff: 3 Page Ref: 1099 Objective: 37-14

17 Copyright © 2018 Pearson Education, Inc.

68) You have been called to a residence for a 21-year-old female in labor. On arrival, you immediately see that the head of the baby is out of the vagina. While delivering the head, the mother tells you that she is having twins. The EMT should immediately: A) Place the patient on her left side B) Provide blow-by oxygen to the baby C) Call for another EMS crew D) Place the patient on the stretcher for transport Answer: C Diff: 2 Page Ref: 1099 Objective: 37-14 69) While delivering a baby in the field, the EMT notices that the baby is covered with a greenbrown liquid. The EMT should immediately recognize that: A) The baby was distressed and may be hypoxic B) There is an infection in the amniotic membrane and fluid C) The baby is premature and covered with vernix D) The amniotic sac has yet to rupture Answer: A Diff: 3 Page Ref: 1099 Objective: 37-15 70) The EMT would recognize which neonate as premature? A) A baby weighing 8 pounds 6 ounces born at 39 weeks B) A baby weighing 7 pounds 4 ounces born at 36 weeks C) A baby weighing 6 pounds 2 ounces born at 38 weeks D) A baby weighing 8 pounds 6 ounces born at 41 weeks Answer: B Diff: 2 Page Ref: 1099 Objective: 37-17 71) A female patient is 34 weeks pregnant, and you are delivering the baby. As soon as the baby is completely delivered, you should immediately: A) Check the baby's breath sounds B) Administer high-concentration oxygen to the baby C) Clamp and cut the cord D) Dry the baby with a warm towel Answer: D Diff: 2 Page Ref: 1099 Objective: 37-9

18 Copyright © 2018 Pearson Education, Inc.

72) Which finding is of greatest concern when assessing a 33-year-old female who is eight months pregnant? A) Shortness of breath when lying flat B) Recent onset of a fever of 99.3°F C) Daily contractions that are irregular and painful D) Vaginal bleeding not associated with pain Answer: D Diff: 3 Page Ref: 1073 Objective: 37-5 73) The EMT shows that he can accurately differentiate placenta previa from abruptio placentae when he states: A) "Abruptio placentae typically occurs in the first and second trimesters; placenta previa occurs in the third trimester." B) "Bleeding associated with abruptio placentae is typically associated with abdominal pain; bleeding associated with placenta previa is painless." C) "Abruptio placentae rarely results in the death of the baby; the mortality rate of placenta previa is high." D) "The blood lost with placenta previa is dark red; the color of blood associated with abruptio placentae is bright red." Answer: B Diff: 3 Page Ref: 1073 Objective: 37-5 74) You arrive at the residence of a physician who informs you that his pregnant wife requires immediate transport to the hospital because she is bleeding and has a history of abruptio placentae. As a knowledgeable EMT, you should recognize that the greatest threat to the baby is: A) Infection B) Maternal shock C) Hypoxia D) Fluid loss Answer: C Diff: 3 Page Ref: 1073-1074 Objective: 37-5 75) You are transporting a female patient who is seven months pregnant. She has been diagnosed with preeclampsia and is currently hypertensive. Which statement made by the patient indicates she may be transitioning from preeclampsia to eclampsia? A) "I feel like I am going to have a seizure." B) "My legs and feet are more swollen." C) "I have gained 10 pounds just this month." D) "I think that I may have to vomit." Answer: A Diff: 3 Page Ref: 1075-1076 Objective: 37-6

19 Copyright © 2018 Pearson Education, Inc.

76) You have been called to a home for a female in labor. On scene, the family tells you that the 37-year-old patient is 42 weeks pregnant and wanted to have her baby at home naturally. The patient has been in labor for more than 24 hours, but suddenly began complaining of severe and tearing abdominal pain. Which condition should the EMT suspect? A) Placenta previa B) Preeclampsia C) Ectopic pregnancy D) Ruptured uterus Answer: D Diff: 3 Page Ref: 1075 Objective: 37-5 77) A young female complains of sharp pain to the left lower quadrant of her abdomen. She states that her last period was seven weeks ago. With this information, the EMT should be suspicious of: A) Placenta previa B) Appendicitis C) Abruptio placentae D) Ectopic pregnancy Answer: D Diff: 3 Page Ref: 1072 Objective: 37-7 78) The EMT shows she has a proper understanding of Braxton-Hicks contractions when he says: A) "They often are called false contractions and do not require hospital evaluation." B) "They are often irregular and less painful than labor contractions, but they should still be evaluated by the hospital." C) "They are painless, but are capable of delivering the baby very quickly." D) "They can be differentiated from labor contractions by the presence of vaginal discharge." Answer: B Diff: 3 Page Ref: 1083 Objective: 37-9 79) In a normal fertilization, the egg and the sperm meet in which structure? A) Uterus B) Fallopian tube C) Cervix D) Vaginal canal Answer: B Diff: 1 Page Ref: 1068 Objective: 37-2

20 Copyright © 2018 Pearson Education, Inc.

80) During a continuing education session on OB/GYN topics, the EMT would recognize which statement about the umbilical cord as true? A) It connects the placenta and the uterus B) It detoxifies blood before its delivery to the baby C) It contains one vein and two arteries D) It produces hormones for the baby Answer: C Diff: 1 Page Ref: 1069 Objective: 37-2 81) You are caring for an emotionally distraught female who just suffered a miscarriage. During transport, which statement would be the most appropriate one for the EMT to make to the patient? A) "It is better to have this happen at five weeks instead of 25 weeks." B) "Let's just think of the two beautiful children whom you have at home." C) "Maybe at some time in the future, you can get pregnant again and things will turn out okay." D) "I really do not know what to say, but let me know how I can help you." Answer: D Diff: 2 Page Ref: 1072 Objective: 37-7 82) The EMT recognizes the underlying pathophysiology associated with the most common type of ectopic pregnancy is a fertilized egg that is: A) Lodged in a fallopian tube B) Developing without a placenta C) Contaminated with abdominal bacteria D) Attached to the upper region of the uterus rather than to the lower region by the cervix Answer: A Diff: 2 Page Ref: 1072 Objective: 37-7 83) A female patient has acute onset of right-sided abdominal pain, describes it as sharp, and rates it as 9/10. The primary survey reveals no life threats, but her skin is cool and diaphoretic. She states that she has been going to the bathroom more and is worried because her father has diabetes. Her last period was nine weeks ago, and she has a history of irregular periods. Her heart rate is 122 beats/min, respirations are 20 breaths/min, blood pressure is 138/68 mmHg, and SaO2 is 97%. In this scenario, which bit of information is of most concern to the EMT? A) Family history of diabetes B) Increased urination C) Last period nine weeks ago D) Periods irregular in nature Answer: C Diff: 3 Page Ref: 1072-1073 Objective: 37-7

21 Copyright © 2018 Pearson Education, Inc.

84) A 32-year-old female complains of the sudden onset of sharp pain to the left side of her abdomen. She denies any medical history and when asked, admits to being sexually active. Her last menstrual period was two months ago. As an EMT, you should immediately suspect: A) Ectopic pregnancy B) Mittelschmerz C) Pelvic inflammatory disease D) Abruptio placentae Answer: A Diff: 2 Page Ref: 1072-1073 Objective: 37-7 85) When treating a patient whom you believe has an ectopic pregnancy, you know that the primary threat to the patient's life is: A) Infection B) Hemorrhage C) Severe pain D) Pelvic organ damage Answer: B Diff: 2 Page Ref: 1072-1073 Objective: 37-7 86) A 22-year-old female, who is eight months pregnant, calls 911 for vaginal bleeding. Your primary assessment reveals no acute life threats, but the patient does have some evidence of blood in her underwear. The patient denies abdominal pain. Her vital signs are normal. Given these findings, the EMT should be suspicious of: A) Uterine rupture B) Ectopic pregnancy C) Pelvic inflammatory disease D) Placenta previa Answer: D Diff: 3 Page Ref: 1073 Objective: 37-5 87) If the placenta attaches over the cervix, the patient and fetus would be at risk for which condition? A) Placenta previa B) Placental abruption C) Ectopic pregnancy D) Uterine tear Answer: A Diff: 2 Page Ref: 1073 Objective: 37-5

22 Copyright © 2018 Pearson Education, Inc.

88) You have been called for a 35-year-old female who is experiencing vaginal spotting. The patient states that she has noted the discharge of blood from her vagina and is concerned because she is 36 weeks pregnant. She has also experienced some abdominal pain, which she describes as "sharp." Your assessment reveals bleeding as noted and tenderness on palpation just right to the midline of her abdomen. Her pulse is 122 beats/min, respirations are 22 breaths/min, blood pressure is 118/82 mmHg, SpO2 is 96%, and oral temperature is 97.3°F. Based on this presentation, you would: A) Provide low-concentration oxygen and nonemergently transport the patient B) Place the patient in a supine position and immediately transport her C) Administer high-concentration oxygen and immediately transport the patient D) Massage the abdomen, provide high-concentration oxygen, and nonemergently transport the patient Answer: C Diff: 2 Page Ref: 1074 Objective: 37-7 89) You have been called for a 27-year-old female who is 32 weeks pregnant. She states that she is experiencing uterine contractions but feels no pain with them, just a tightening in her abdomen. The contractions began three days ago and seem to be increasing in frequency today. She denies discharge or rupture of her membranes. Given this presentation, you should recognize: A) Probable false labor, and should have the patient follow up with her obstetrician B) Probable false labor, but have the patient transported to the hospital for evaluation C) True labor, so you should assess the vaginal canal for possible crowning D) Possible placental abruption, and immediately must transport the patient with lights and sirens Answer: B Diff: 2 Page Ref: 1083 Objective: 37-7 90) You are by the side of a 19-year-old female who is 36 weeks pregnant. When assessing this patient, which finding best indicates that she is in preterm labor? A) Rupture of the amniotic sac B) Complaint of lower back pain C) Contractions 12 minutes apart D) Feeling nauseated with each contraction Answer: A Diff: 2 Page Ref: 1081 Objective: 37-9 91) Which statement made by a pregnant patient in labor should be of most concern to the EMT? A) "I have terrible pains in my lower back." B) "My water broke two days ago." C) "The contractions seem to last about 30 seconds." D) "I just passed a lot of blood and mucus." Answer: B Diff: 2 Page Ref: 1100 Objective: 37-9 23 Copyright © 2018 Pearson Education, Inc.

92) While examining a pregnant patient who has not received prenatal medical care, you note that you can palpate the uterine fundus at the level of the umbilicus. What is the approximate gestational age of the fetus? A) 38 weeks B) 12 weeks C) 20 weeks D) You cannot estimate fetal age using this method Answer: C Diff: 2 Page Ref: 1086-1087 Objective: 37-10 93) By the time a fetus reaches full term, the tidal volume of the female has increased by: A) 15 percent B) 40 percent C) The tidal volume remains the same but the rate increases by 30 percent D) 25 percent Answer: B Diff: 2 Page Ref: 1070 Objective: 37-3 94) What would the expected preductal SpO2 reading be at 5 minutes after birth? A) 70% to 75% B) Over 94% C) At least 80% D) 85% to 90% Answer: C Diff: 2 Page Ref: 1100 Objective: 37-18 95) Despite the increase in maternal blood volume, the pregnant patient is typically: A) Hypertensive B) Prone to early signs of shock C) Anemic D) Bradycardic Answer: C Diff: 2 Page Ref: 1070 Objective: 37-3 96) When do most women experience a decrease in the nausea and vomiting associated with hyperemesis gravidarum? A) At term B) Between 14 and 20 weeks C) During the first trimester D) About 2 weeks postpartum Answer: B Diff: 2 Page Ref: 1070 Objective: 37-4 24 Copyright © 2018 Pearson Education, Inc.

97) A female patient tells you that she is 38 weeks pregnant. Where would you expect to palpate the top of the uterus during your exam? A) At the level of the umbilicus B) At the level of the iliac crest C) At the level of the xiphoid process D) At the level of the twelfth rib Answer: C Diff: 2 Page Ref: 1087 Objective: 37-10 98) You are completing the delivery of a near term baby boy when you notice that the fluid expelled with the baby is greenish-brown. At this point you should: A) Immediately begin positive pressure ventilation B) Suction the nose and mouth C) Administer high-concentration oxygen D) Quickly suction the nose and mouth only if the newborn's condition is depressed Answer: D Diff: 2 Page Ref: 1099 Objective: 37-15 99) What would the minimum expected preductal SpO2 reading be at one minute after birth? A) 94% B) 85% C) 80% D) 60% Answer: D Diff: 2 Page Ref: 1100 Objective: 37-18 100) According to the neonatal resuscitation pyramid, which level of care will most newborns require? A) Tactile stimulation and drying B) Blow-by oxygen C) Suctioning of the nose and mouth D) Nursing Answer: A Diff: 2 Page Ref: 1105 Objective: 37-19

25 Copyright © 2018 Pearson Education, Inc.

101) How many newborns will require a level of care that falls below the top tier of the neonatal resuscitation pyramid? A) 20 percent B) Approximately half C) One in twenty D) 80 percent Answer: A Diff: 2 Page Ref: 1105 Objective: 37-19

26 Copyright © 2018 Pearson Education, Inc.

Prehospital Emergency Care, 11e (Mistovich et al.) Chapter 38 Pediatrics 1) Which statement made by the EMT is most appropriate regarding dealing with caregivers and children during a medical emergency? A) "I try to include the caregiver in all that I do with his or her child so that the child and caregiver are more comfortable." B) "It is best to separate the caregiver from the child so that a proper assessment and care can be given." C) "I include the caregiver in the care until I get the information that I need, then I remove the child to continue the assessment in the ambulance." D) "I tell the caregiver that everything will be okay so the caregiver remains calm and is better able to help the child." Answer: A Diff: 2 Page Ref: 1117 Objective: 38-2 2) Pediatric patients are patients who range in age from: A) 1 year to 8 years B) birth to 12 years C) birth to 18 years D) 1 year to 15 years Answer: C Diff: 1 Page Ref: 1118 Objective: 38-3 3) You have been called to care for a 21-month-old female who has been bitten by a dog. Given the patient's age, you would appropriately classify the patient as a(n): A) Infant B) School-age child C) Preschooler D) Toddler Answer: D Diff: 1 Page Ref: 1118 Objective: 38-1 4) A coworker is telling you about a recent EMS call in which he provided care to a preschooler. Based on this description, you know that age of the patient would have been: A) 1-2 years B) 3-5 years C) 6-8 years D) more than 9 years Answer: B Diff: 1 Page Ref: 1119 Objective: 38-1

1 Copyright © 2018 Pearson Education, Inc.

5) Which instructions would you provide to a new EMT who is preparing to assess a stable 9month-old boy who has a rash? A) "To keep the baby calm, do not touch him during the assessment." B) "Start at the head and slowly work your way to the feet." C) "Allow the mother hold him as you do the assessment." D) "It is important not to undress a baby during assessment, as this will stress the baby." Answer: C Diff: 1 Page Ref: 1118 Objective: 38-3 6) You are assessing a 2½-year-old female who was involved in a minor car accident. She is currently alert and oriented. While you are assessing her for possible injuries, which assessment procedure would be considered appropriate? A) Assess the child starting at her head and moving to her feet B) If possible, allow the child to hold a favorite toy during the assessment C) Be firm with the child and provide concrete instructions on what you expect D) Touch and hold the child as much as possible Answer: B Diff: 2 Page Ref: 1119 Objective: 38-3 7) You have been called for a 2-year-old girl who has been sick for two days. In preparing to assess the patient, it is important that you recognize that patients in this age group generally: A) Do not like having clothing removed B) Tolerate separation from their caregiver C) Have little fear of strangers D) Like to be touched Answer: A Diff: 2 Page Ref: 1119 Objective: 38-3 8) You are assessing a 5-year-old boy who complains of arm pain after falling down three steps. As you physically assess the patient, he tries to bite your hands. His mother is at his side. Which statement by you is appropriate to make regarding the behavior of biting? A) "If you bite me, I will not help you feel better." B) "Do you bite your mother? Why would you try to bite me?" C) "Mom, you need to stop him from biting me." D) "I know that you hurt, but biting is not okay." Answer: D Diff: 2 Page Ref: 1119 Objective: 38-3

2 Copyright © 2018 Pearson Education, Inc.

9) You are assessing the pupils of a 5-year-old who fell off a bed while playing. Which of the statement would be most appropriate prior to performing this assessment? A) "I am going to look at your pupils with my light." B) "I am going to use this light to look into your eyes." C) "I need to test your visual acuity by shining a light in your eyes." D) "Open your eyes so that I can look at them." Answer: B Diff: 2 Page Ref: 1119 Objective: 38-3 10) Which behavior would the EMT recognize as uncharacteristic of a conscious and stable 2year-old boy who fell and hurt his hand? A) Cries any time you or your female partner touch him B) Does not cry or protest when taken from his mother C) Becomes upset when you lift his shirt to assess his abdomen D) Apologizes several times for falling and hurting his hand Answer: B Diff: 3 Page Ref: 1118 Objective: 38-5 11) After a 29-year-old male with chest pain is removed from the house by stretcher, the man's 7-year-old son throws a temper tantrum. His mother is embarrassed and states that he has not done this since he was 2 years old. Given the situation, your response would be: A) "I would not be concerned. This is a stressful event he doesn't understand, and he may just be having trouble coping with it." B) "You really need to correct this behavior now so it does not happen in the hospital. They will make you take him home." C) "It may be wise to contact your pediatrician and let him know about the temper tantrum. It may be controllable with medications." D) "A 7-year-old is too old for this type of behavior. You may want to talk to his pediatrician." Answer: A Diff: 3 Page Ref: 1118 Objective: 38-2

3 Copyright © 2018 Pearson Education, Inc.

12) A 6-month-old male was outside with his parents and was stung on the tongue after putting a bottle in his mouth that had a bee on the nipple. In comparison to the same injury in an adult, why would the EMT be more concerned with this child? A) A child's airway has more blood vessels than an adult's airway, making bleeding more of a concern B) Children tend to be highly allergic to bee stings while the same allergy in an adult is less common C) The child's tongue is proportionally larger, increasing the chance of airway occlusion from minor swelling D) When injured, a child's mouth produces more saliva, making airway occlusion a major concern Answer: C Diff: 2 Page Ref: 1120 Objective: 38-4 13) An Emergency Medical Responder asks you why you should not overextend the airway on a pediatric patient when performing a head-tilt, chin-lift maneuver. Your reply would be: A) "The pediatric cervical spine is delicate and can be injured if the neck is hyperextended." B) "The trachea of the pediatric patient is very short and can be injured by extension." C) "The cartilage of the trachea is very soft and can 'kink' if the neck is extended too far." D) "The pediatric esophagus is very thick and will occlude the airway if the neck is hyperextended." Answer: C Diff: 3 Page Ref: 1120 Objective: 38-4 14) You have been called for a 2-week-old baby who is sick. Assessment reveals him to have a fever and rhonchi that is audible with a stethoscope over the lungs. Which of the additional assessment findings would be most concerning to you given the age of this patient? A) Respiratory rate of 30 breaths per minute B) Wet diaper in need of changing C) Continual moaning and crying D) Nasal passages occluded by mucus Answer: D Diff: 3 Page Ref: 1121 Objective: 38-4 15) You are securing a 4-year-old-boy on a long spine board during a spine motion restriction process. Which action would be appropriate when performing this intervention? A) Do not apply the chest strap across the thorax B) Place padding between his shoulders/back and the spine board C) Secure the chest and legs to the board after the head D) Place a pillow or padding under the head to maintain normal spinal alignment Answer: B Diff: 2 Page Ref: 1139 Objective: 38-4 4 Copyright © 2018 Pearson Education, Inc.

16) The increased pliability of the child's ribs makes him more prone to: A) Bruising or other injury to the lungs B) Rib fractures and flail segments C) Cardiac arrest from heart damage D) Overinflation of the lungs Answer: A Diff: 2 Page Ref: 1122 Objective: 38-15 17) When assessing a 3-year-old who is in respiratory distress, which assessment finding would be most concerning? A) Retraction of the muscles between the ribs B) Abdominal rise and fall during breathing C) Retractions observed above the clavicles D) Respiratory rate of 28 breaths per minute Answer: C Diff: 3 Page Ref: 1122 Objective: 38-4 18) You have arrived on the scene to help a child who is short of breath. The mother is screaming hysterically that her 3-year-old boy cannot breathe, making assessment of the patient very difficult. Your initial action in the management of this situation would be to: A) Inform the mother that you will call the police if she does not calm down B) Quickly remove the child to the ambulance and assess while transporting C) Summon the police to the residence to subdue the mother D) Have your partner talk to the mother while you assess the child Answer: D Diff: 2 Page Ref: 1117 Objective: 38-2 19) You have been called for a 7-year-old girl who has vomited once and complains of abdominal pain. When you are performing the secondary assessment and obtaining a medical history, which action by you would be considered most appropriate? A) Asking mostly questions that can be answered with a "yes" or "no" B) Standing above the child and smiling while asking questions C) Allowing the child to play with the stethoscope before listening to her lungs D) Using "baby talk" when asking the child questions about her pain Answer: C Diff: 2 Page Ref: 1119 Objective: 38-3

5 Copyright © 2018 Pearson Education, Inc.

20) Which assessment findings related to the mental status of a 1-year-old would the EMT consider normal? A) The patient turns his head to sounds B) The patient is curious about your penlight C) The patient moans when pressure is applied to his finger D) The patient does not keep his eyes open Answer: B Diff: 3 Page Ref: 1119 Objective: 28-3 21) You have been called to a lower-income housing complex for a 1½-year-old child who is short of breath and has a fever. When assessing this patient, which technique would be appropriate? A) Determining the respiratory rate before laying hands on the child B) Auscultating breath sounds over the anterior chest only C) Checking capillary refill by pressing on the skin of the fingers D) Performing the primary assessment after the secondary assessment Answer: A Diff: 2 Page Ref: 1118 Objective: 38-3 22) An 8-year-old boy is unresponsive following the accidental ingestion of his father's high blood pressure medications. His airway is open and his breathing is labored and inadequate. The pulse rate is 48 beats per minute and his skin is cool and diaphoretic. Your partner states that the patient's lungs have crackles in them and the blood pressure is 60/40 mmHg. Which intervention represents the most important care that you will provide to the patient next? A) Determination of the exact medication taken B) Oral airway and high-concentration oxygen C) Administration of activated charcoal D) Positive pressure ventilation Answer: D Diff: 3 Page Ref: 1127 Objective: 38-5 23) When providing care to the pediatric patient who has been injured or ill, the most important aspect of care normally revolves around: A) Airway and respiratory support B) Assessment and support of circulation C) Ability to provide rapid transport D) Knowledge of illnesses that affect children Answer: A Diff: 2 Page Ref: 1133 Objective: 38-5

6 Copyright © 2018 Pearson Education, Inc.

24) Which statement made by another EMT during a continuing education session, regarding treatment of the pediatric patient in the prehospital setting, is correct? A) "When treating a sick pediatric patient in the prehospital setting, the EMT must determine the exact illness so as to provide the most appropriate treatment." B) "If the primary assessment of a sick pediatric patient shows no acute life threats, the patient is stable and will not deteriorate." C) "It is critical that the EMT be able to determine the adequacy of the airway and respiratory status of the patient, as these are commonly the reason for acute deterioration." D) "Since pediatric patients can deteriorate very quickly, they should all be transported with lights and sirens during the ride to the hospital, with ALS intercept arranged for." Answer: C Diff: 2 Page Ref: 1133 Objective: 38-5 25) Assessment of an alert and oriented 9-year-old child with a history of asthma reveals him to be breathing 20 times per minute with adequate chest rise and fall. You also note that he exhibits nasal flaring and has slight retractions of the intercostal muscles. His heart rate is 100 beats per minute and his blood pressure is 102/64 mmHg. On room air, he has a pulse oximeter reading of 98%. The EMT would recognize: A) Compensated respiratory distress B) Late respiratory failure C) Hypoxic respiratory failure D) Decompensated respiratory failure Answer: A Diff: 3 Page Ref: 1133-1134 Objective: 38-6 26) You are assessing a 6-year-old girl with possible pneumonia. She has labored breathing and a fever of 102°F. When you are assessing and classifying her respiratory status, which finding would provide the strongest evidence that she is in respiratory failure? A) Respiratory rate of 36 breaths/min B) Altered mental status C) Fever of 102°F D) Nasal flaring Answer: B Diff: 3 Page Ref: 1135 Objective: 38-8 27) Your partner informs you that the 3-year-old boy you have been called to care for is breathing 26 times per minute. As a knowledgeable EMT, you would recognize this to be: A) A rapid rate B) Adequate breathing C) A normal rate D) Inadequate breathing Answer: C Diff: 2 Page Ref: 1121; Table 38-1 Objective: 38-4 7 Copyright © 2018 Pearson Education, Inc.

28) You have been called for a 4-year-old female who is short of breath. Her mother informs you that she has had a fever and runny nose for two days and today began to have difficulty breathing. Furthermore, the mother is having a difficult time waking her daughter up. You find the girl lying in bed with snoring respirations despite use of the head-tilt, chin-lift airway maneuver. Your assessment reveals her to be responsive to painful stimuli and breathing at 12 times per minute with minimal chest and abdominal rise and fall. Her skin is warm to the touch with cyanosis around the lips. Your partner informs you that her heart rate is 124 beats per minute. What is your immediate action in caring for this child? A) Positive pressure ventilation B) Suctioning the airway C) Application of a nonrebreather face mask and oxygen D) Insertion of a nasal airway Answer: D Diff: 2 Page Ref: 1137-1138 Objective: 38-10 29) What would be the most appropriate instructions to give a new EMT with whom you are working regarding the ventilation of an unresponsive and apneic 3-year-old boy? A) "Let's ventilate him at a rate of 28 breaths per minute." B) "Make sure to give each ventilation over 3 seconds." C) "Deliver one breath every 3 seconds." D) "Do not administer cricoid pressure since it is contraindicated." Answer: C Diff: 2 Page Ref: 1138 Objective: 38-8 30) An infant is short of breath and has rhonchi in both lungs. He is alert with adequate respirations at a rate of 38 breaths per minute. His skin color is pink but cool to the touch. Due to the patient's movement, it is difficult to get a reliable pulse oximeter reading. Additionally, when EMRs place a pediatric mask on his face, he becomes very upset and physically struggles to remove it. In this situation you would: A) Place a nasal airway and start positive pressure ventilation B) Secure the mask to the patient's face using tape C) Allow the mother to hold the infant and provide blow-by oxygen D) Transport the infant with no further attempt at oxygen therapy Answer: C Diff: 2 Page Ref: 1138-1139 Objective: 38-8

8 Copyright © 2018 Pearson Education, Inc.

31) After placing a small rock in her mouth, a 7-month-old girl begins to choke. You are on scene within minutes and find her unresponsive in her mother's arms. You attempt to provide ventilations with the bag-valve mask, but are unsuccessful after two attempts. What would your next action be? A) Attempt ventilation with a pocket mask B) Administer five chest thrusts C) Provide five abdominal thrusts D) Start cardiopulmonary resuscitation Answer: D Diff: 2 Page Ref: 1141 Objective: 38-9 32) You are caring for an 8-month-old boy who is unresponsive after choking on a piece of hard candy. As you look into his mouth, you can see the candy in the back of his oropharynx. What would your next action be? A) Provide five abdominal thrusts B) Attempt to retrieve and remove the candy C) Check the brachial pulse D) Perform a blind finger sweep Answer: B Diff: 2 Page Ref: 1141 Objective: 38-9 33) On scene at a school cafeteria, you are presented with a 11-year-old boy who attempted to eat a small plum whole. He is conscious with stridorous respirations and unable to cough when instructed to do so. His skin color is gray and cyanotic. What would your immediate action be? A) Deliver five abdominal thrusts B) Provide positive pressure ventilation C) Place an oral pharyngeal airway D) Check for radial and carotid pulses Answer: A Diff: 2 Page Ref: 1142 Objective: 38-9 34) You have been summoned to a grocery store for a 6-year-old girl who has collapsed. On scene, you find the girl lying unresponsive on the floor. You quickly open the patient's airway and determine that she is not breathing. When you try to ventilate her with the bag-valve mask, you are unable to do so. Repositioning the head and reattempting ventilation is not successful in passing air into the lungs. What would your next action be? A) Perform a blind finger sweep B) Administer 30 chest compressions C) Ventilate the patient with greater force D) Place an oral airway Answer: B Diff: 2 Page Ref: 1142 Objective: 38-9 9 Copyright © 2018 Pearson Education, Inc.

35) You have been called to a preschool for a seizing child. On arrival, staff directs you to a bed in which a 4-year-old boy is lying. Staff state that the child began making "funny" noises in his sleep, then began to seize for approximately 60 seconds. Your assessment indicates that the boy is responsive to painful stimuli and is breathing, but has gurgling sounds coming from the upper airway. His radial pulse is rapid and strong, and his skin is cool and diaphoretic. What is your immediate action in caring for this child? A) Apply oxygen B) Place a cervical collar C) Suction the airway D) Start positive pressure ventilation Answer: C Diff: 2 Page Ref: 1154 Objective: 38-12 36) You have been called to a local daycare facility for a sick child. At the scene, panicked daycare workers inform you that the 4-year-old child was lethargic all morning and wanted to sleep. After resting for several minutes, she began to "shake all over." This lasted for approximately 30 seconds. Your assessment reveals the patient to be responsive to painful stimuli and breathing adequately. Her airway is open and her radial pulse is strong and bounding. Her skin is hot and moist to the touch, with no signs of cyanosis. You are told that the patient has no medical history. En route to the hospital, the patient's mental status improves. Based on this presentation and information, you assume that the seizure occurred secondary to: A) Hypoxia B) Fever C) Hypoglycemia D) Altered mental status Answer: B Diff: 2 Page Ref: 1150-1154 Objective: 38-12 37) You have been asked to speak to a group of new mothers regarding the emergency treatment of a fever. During your presentation, one of the mothers asks you which temperature would cause a child to seize. Your response would be: A) "When a child's temperature rises above 105°F, seizure activity will occur." B) "It is not the specific temperature that is of concern, but the rate at which the body temperature rises." C) "If the temperature of a child's fever rises slowly over several hours, she is at greater risk for seizures." D) "For a child to seize when she has a fever, she must also have a history of seizures or a cardiac condition." Answer: B Diff: 3 Page Ref: 1157 Objective: 38-12

10 Copyright © 2018 Pearson Education, Inc.

38) You are assessing a 9-month-old boy who has had a fever for the past two days. What would be a likely finding if the fever has caused dehydration? A) Palpation of the anterior fontanelle reveals it to be bulging B) The heart rate is at the high end of the "normal" range C) Respirations are labored and rapid D) The mother informs you that the baby is wetting very few diapers daily Answer: D Diff: 2 Page Ref: 1157 Objective: 38-12 39) The mother of a 2-year-old has called EMS because her son has an axillary temperature of 103.2°F. On scene, your assessment reveals the boy to be confused and lethargic, with a rectal temperature of 104.1°F. When caring for this child, you would: A) Remove the child's clothes and sponge his body with tepid water B) Apply washcloths soaked in cold water until the child begins to shiver C) Administer oxygen and cool the child by sponging his body with alcohol D) Have the mother administer rectal Tylenol and cool the child by sponging his body with cold water Answer: A Diff: 2 Page Ref: 1157 Objective: 38-12 40) On the scene of a medical emergency, you are directed to a 5-month-old baby in his crib. He responds to painful stimuli by whimpering and is breathing at a rate of 50 breaths per minute. His airway is open, and his brachial pulse is fast, weak, and difficult to locate. Assessment of his skin reveals it to be cool and pale in color. According to the mother, her child has had vomiting and diarrhea for four days. She also reports that the father of the baby has diabetes. Based on this information, the EMT would recognize which condition? A) Shock B) Hypoglycemia C) Cardiac arrest D) Heart failure Answer: A Diff: 2 Page Ref: 1148 Objective: 38-11

11 Copyright © 2018 Pearson Education, Inc.

41) A 6-year-old male has a decreased level of consciousness. His mother states that the family does not have health insurance, so they did not take him to the hospital last week when he started vomiting and had copious amounts of diarrhea. She reports that he has not had anything to eat or drink since then. The patient has snoring respirations that are rapid and inadequate. His radial pulse cannot be located, and his carotid pulse is rapid and weak. His capillary refill is 5 seconds and his skin is cool to the touch. What would your first intervention in caring for this child be? A) Take manual spine motion restriction of the head and neck B) Open the airway using the head-tilt, chin-lift maneuver C) Provide positive pressure ventilation D) Attach the automated external defibrillator Answer: B Diff: 2 Page Ref: 1149 Objective: 38-11 42) While in an apartment to provide care for a 52-year-old female with shortness of breath, you note a baby sleeping in a nearby crib. Which observation related to the baby would prompt you to intervene and speak to the family? A) The baby is sleeping on a hard mattress B) The crib is near a closed window C) The baby is sleeping on her abdomen D) The baby is sleeping in the afternoon Answer: C Diff: 2 Page Ref: 1160 Objective: 38-4 43) While attempting resuscitation of a baby with possible SIDS, which question would be appropriate for the EMT to ask the mother? A) "Why did you have the baby in bed with you and not in a crib?" B) "Were there any problems with the pregnancy or his birth?" C) "Do you think that you may have rolled on him and smothered him?" D) "Why do you keep the temperature so cold in this bedroom?" Answer: B Diff: 2 Page Ref: 1160 Objective: 38-14 44) You have arrived at a residence to find a 6-month-old girl in her crib in cardiac arrest. Quick assessment reveals no rigor mortis or lividity. The mother states that she put the baby to bed at 9 p.m. last night and awoke at 3 a.m. to find her in her present state. What would your first action be? A) Ask if the baby has any medical conditions B) Open the airway and provide 2 ventilations C) Inquire whether the mother wants resuscitation started D) Apply oxygen and move the baby to the ambulance Answer: B Diff: 2 Page Ref: 1161 Objective: 38-14 12 Copyright © 2018 Pearson Education, Inc.

45) The mother of a 4-month-old boy awoke to find him in his crib and not breathing, so she summoned EMS. On scene, you are providing resuscitative care while the parents watch your actions in disbelief. Which decision made by the EMT would be most appropriate regarding their presence in the room? A) Allow them to observe the resuscitation if they wish B) Escort them to another part of the house despite them wanting to stay C) Move the patient to the ambulance for continued resuscitation en route without the parents' presence D) Ask the parents to wait in the ambulance for transport Answer: A Diff: 2 Page Ref: 1162 Objective: 38-14 46) A 7-year-old child was riding his bike downhill and struck a parked car. He was thrown from the bike, impacting his head and back on the roadway. Presently, the patient is alert, oriented, and complaining of a severe headache. His airway is open and his breathing is adequate. A radial pulse is rapid and easily palpated, and his skin is cool and moist. After taking manual spine motion restriction of the head and neck, the EMT should: A) Secure the patient to the long spine board B) Consider the need for supplemental oxygen C) Perform a rapid trauma exam and obtain vital signs D) Apply a cervical collar and place the patient on the cot Answer: B Diff: 2 Page Ref: 1163 Objective: 38-15 47) An 8-year-old child was chasing his dog when the boy ran out into the street and was struck by a car. He is responsive to verbal stimuli and has an unstable pelvis and a bruise to the side of his face. His airway is open and his breathing is rapid and shallow. His radial pulse is rapid and weak and his skin is cool, moist, and diaphoretic. The patient also has abrasions to his back after being thrown by the car onto the pavement. Based on these assessment findings, the EMT would treat the patient for which life-threatening condition? A) Hypoxia B) Head injury C) Shock D) Back injury Answer: C Diff: 3 Page Ref: 1148-1149 Objective: 38-15

13 Copyright © 2018 Pearson Education, Inc.

48) You have been called to a home by neighbors because they are concerned for the welfare of the children inside. On scene, you find two children left by themselves. They are playing in a filthy room with no light or heat. They are dirty and have insect bites all over their bodies. One is in a soiled diaper and has hard stool caked to the buttocks. The EMT would best recognize this situation as one of: A) Neglect B) Parental negligence C) Physical abuse D) Felonious care Answer: A Diff: 2 Page Ref: 1168 Objective: 38-18 49) When assessing a 3-year-old child for possible injuries after the child fell down a flight of stairs, which finding would raise the EMT's suspicion that the child may be a victim of physical abuse? A) The child cries when you palpate his arm B) Several bruises are located on his knees and shins C) The child is fearful and appears thinner than other children this age D) Bruises are found on his chest and abdomen Answer: D Diff: 3 Page Ref: 1168 Objective: 38-18 50) After you obtain a refusal at a residence in a neighborhood, a couple approaches the ambulance and inform you that they have concerns that a 6-year-old child who lives in a home two doors away may be physically abused. They state that they heard the child crying and what sounded like physical abuse occurring. In this situation, you would: A) Contact the police and wait in the ambulance until they arrive B) Attempt entry into the home to determine whether the child is injured C) Inform the couple that they must contact a child welfare agency D) Knock on the door and confront the parents about the complaint Answer: A Diff: 2 Page Ref: 1169 Objective: 38-18 51) You are suspicious that a child is being physically abused and neglected. What would be your best course of action in such a situation? A) Confront the alleged abuser B) Document the situation C) Notify other family members D) Place the child in a safe home Answer: B Diff: 2 Page Ref: 1169 Objective: 38-18

14 Copyright © 2018 Pearson Education, Inc.

52) You have arrived at the emergency department with a young child whom you believe is the victim of child abuse. To which person would it be best to report your suspicion? A) Registered nurse B) Security guard C) Emergency physician D) Hospital administrator Answer: C Diff: 2 Page Ref: 1169 Objective: 38-18 53) After the unsuccessful resuscitation of a 3-year-old boy who was shot by his mother's boyfriend during a domestic dispute, you find yourself extremely stressed and anxious over the call. You are having trouble sleeping and the resultant anxiety is affecting your home life. What should be your initial action, given this situation? A) Make a conscious effort to focus on anything positive about the event B) Talk to a close friend or fellow EMS provider about your feelings C) Use the tragedy as an incentive to learn more about pediatric trauma D) Consider finding a profession outside of medicine Answer: B Diff: 2 Page Ref: 1170 Objective: 38-20 54) The mother of a 3-year-old boy has called 911 because her son has a low-grade fever and difficulty breathing. On scene, the mother tells you that she is a licensed practical nurse (LPN), and that she suspects her son is suffering from croup. Which statement made by the mother would reinforce this suspicion? A) "At night, he seems to get a barking-like cough." B) "When he is short of breath, his heart rate increases." C) "There are times when he continually drools." D) "When I turn the air conditioner on, he becomes more short of breath." Answer: A Diff: 3 Page Ref: 1143 Objective: 38-10 55) A concerned parent asks you which vaccine will decrease the risk of her child getting epiglottitis. Which vaccine would you tell her? A) Hepatitis B) Influenza C) Haemophilus influenzae D) Hib vaccine Answer: D Diff: 3 Page Ref: 1143 Objective: 38-10

15 Copyright © 2018 Pearson Education, Inc.

56) You have been called to an apartment for a child who is sick. As you approach the 4-year-old boy, you note that he is sitting upright in his mother's arms with his chin thrust forward. He has a sickly appearance and is drooling. His airway appears open and his breathing is adequate. His radial pulse is strong and his skin is hot to the touch. Which intervention would be most appropriate in the care of this patient first? A) Suctioning secretions from the back of the airway B) Provision of supplemental oxygen via a nonrebreather mask C) Placement of an oropharyngeal airway D) Placing the boy supine for transport Answer: B Diff: 2 Page Ref: 1144 Objective: 38-10 57) A 2-month-old presents with labored breathing, fever, and coughing. Auscultation of the lungs reveals significant wheezing bilaterally. Based on this presentation, an EMT would recognize that the patient most likely has: A) Asthma B) Epiglottitis C) Bronchiolitis D) Croup Answer: C Diff: 3 Page Ref: 1146 Objective: 38-10 58) While performing the reassessment of a 2-year-old male who is severely dyspneic, you note that he is now unresponsive and not breathing. He has a heart rate of 32 beats per minute and his skin is cool and diaphoretic. Which instruction would you provide to another EMT who is helping you transport the patient? A) "We need to get a blood pressure and then I will call medical direction." B) "Please increase the oxygen in the nonrebreather face mask to 15 liters per minute." C) "Let's start chest compressions and positive pressure ventilations." D) "We need to try humidified oxygen to see if he will improve with that." Answer: C Diff: 2 Page Ref: 1150 Objective: 38-11 59) You are treating a 6-month-old patient who was accidentally dropped down a flight of steps, when her mother stumbled at the top of the stairway. The infant will only open her eyes and moan to deep painful stimuli, and tries to withdraw from the pain. Given these findings, you calculate her Pediatric Glasgow Coma Scale score to be: A) 4 B) 6 C) 8 D) 10 Answer: C Diff: 3 Page Ref: 1130; Table 38-5 Objective: 38-7 16 Copyright © 2018 Pearson Education, Inc.

60) What is the best location for assessing breath sounds in the pediatric patient? A) Midaxillary region of the lungs B) Anteriorly at the apex of the lung C) Anteriorly at the base of the lung D) Posteriorly just below the scapula Answer: A Diff: 2 Page Ref: 1127 Objective: 38-4 61) What would be the calculated minimally acceptable systolic blood pressure for a 9-year-old girl who has a significant bleed following an injury involving broken glass? A) 96 mmHg B) 79 mmHg C) 100 mmHg D) 88 mmHg Answer: D Diff: 3 Page Ref: 1121; Table 38-1 Objective: 38-3 62) When preparing to provide positive pressure ventilation to a child, what will be an acceptable tidal volume if the child weighs 22 kg? A) 200 mL B) 154 mL C) 246 mL D) 104 mL Answer: B Diff: 3 Page Ref: 1138 Objective: 38-8 63) An extremely high fever in a 2-month-old infant should be considered which medical emergency until otherwise proven? A) Bacterial pneumonia B) Cerebral abscess C) Viral bronchitis D) Meningitis Answer: D Diff: 3 Page Ref: 1157 Objective: 38-12

17 Copyright © 2018 Pearson Education, Inc.

64) When involved with a pediatric patient, the EMT recognizes that patient assessment begins when: A) You are pulling up on scene outside the house B) You physically touch the patient C) You first lay eyes on the patient D) You arrive by the side of the patient. Answer: C Diff: 1 Page Ref: 1124 Objective: 38-5 65) The EMT is properly assessing the breathing of a 3-year-old when he: A) Counts the respiratory rate for 15 seconds and multiplies by 4 B) Counts the respiratory rate for 30 seconds and multiplies by 2 C) Counts the respiratory rate for 15 seconds and observes chest rise and fall D) Counts the respiratory rate for 10 seconds and multiplies by 6 Answer: B Diff: 1 Page Ref: 1127 Objective: 38-5 66) When assessing a central pulse of a young child, the EMT would palpate the: A) Radial pulse B) Brachial pulse C) Pedal pulse D) Femoral pulse Answer: D Diff: 1 Page Ref: 1128 Objective: 38-4 67) When evaluating capillary refill time in a pediatric patient, the EMT must remember that: A) It is considered more reliable than in an adult B) It should be less than 5 seconds C) It is not a good test of perfusion D) It is not affected by environmental conditions Answer: A Diff: 2 Page Ref: 1128 Objective: 38-4

18 Copyright © 2018 Pearson Education, Inc.

68) You are by the side of a 2-year-old male whose mother states he has a cough that has slowly worsened over the past three days. The patient's airway is patent and respirations labored. Intercostal retractions are accompanied by stridorous respirations. His skin is very warm to the touch. Vital signs are as follows: pulse, 152 beats/min; respirations, 28 breaths/min; SpO2, 97%; and temperature, 100.3°F. Breath sounds are clear and equal. Based on this presentation, you would suspect and treat the patient for: A) Asthma B) Bronchiolitis C) Foreign body airway obstruction D) Croup Answer: D Diff: 3 Page Ref: 1143 Objective: 38-10 69) Croup is recognized as an illness that involves: A) Edema beneath the glottis B) Bronchiolar constriction C) Swelling of the epiglottis D) Infection within the lungs Answer: A Diff: 1 Page Ref: 1143 Objective: 38-10 70) A 2-year-old patient is awake and in respiratory distress with stridorous respirations. His airway is patent and pulses rapid. Vital signs are pulse, 144 beats/min; respirations, 28 breaths/min; SpO2, 93% on room air; and temperature, 101°F. The patient has no medical history, although diabetes runs in the family. Appropriate care for this patient would include: A) High-concentration oxygen by mask B) Checking blood sugar C) Blow-by oxygen that is humidified D) Providing aspirin for fever Answer: C Diff: 2 Page Ref: 1143 Objective: 38-10 71) A 5-year-old male is drooling with stridorous respirations and has a fever of 104°F. Which statement made by the patient would concern the EMT most? A) "I am not hungry at all." B) "It hurts to swallow." C) "I have a headache." D) "My nose will not stop running." Answer: B Diff: 2 Page Ref: 1144 Objective: 38-10

19 Copyright © 2018 Pearson Education, Inc.

72) A 5-year-old boy who is drooling has suspected epiglottitis. His pulse rate is 144 beats/min, respirations are 46 breaths/min, and blood pressure is 110/52 mmHg. The patient is noncompliant with the pulse oximeter, but his skin is normal colored. Which statement made by the EMT indicates proper care of this patient? A) "Let's give him some supplemental oxygen." B) "I need to depress his tongue with the tongue blade to see the swelling." C) "Let's suction his airway to remove the secretions." D) "We need to place him on his side to drain secretions and transport immediately!" Answer: A Diff: 2 Page Ref: 1144 Objective: 38-10 73) A 6-year-old male is in severe respiratory distress with inspiratory stridor. He has pale, moist skin, with cyanosis noted around his mouth. The patient is very lethargic and cannot hold his head up. His mother is panicked and reports that he has a history of asthma. Breath sounds are diminished and his vital signs are pulse, 162 beats/min; respirations, 40 breaths/min; blood pressure, 122/66 mmHg; and SpO2, 81%. What would your immediate action be? A) Apply a nonrebreather mask with 15 lpm oxygen B) Insert an oral airway and ventilate with a bag-valve mask C) Start positive pressure ventilation D) Administer the patient's albuterol MDI and provide immediate transport Answer: C Diff: 2 Page Ref: 1145 Objective: 38-10 74) Which disease or illness would you suspect when confronted with a pediatric patient exhibiting an acute onset of respiratory distress and an audible inspiratory stridor sound, but no cough? A) Croup B) Epiglottitis C) COPD D) Bronchiolitis Answer: B Diff: 3 Page Ref: 1143-1144 Objective: 38-10 75) What would be a primary concern for an 11-year-old female with a prolonged asthmatic episode lasting several days and a low-grade temperature? A) Laryngeal edema B) Croup C) Febrile seizure D) Dehydration Answer: D Diff: 3 Page Ref: 1145 Objective: 38-10

20 Copyright © 2018 Pearson Education, Inc.

76) You are assessing a 12-year-old female in respiratory distress and with a history of asthma. Which statement, made by her mother, would be most concerning? A) "The doctor stopped her prescription for steroids." B) "Two years ago she needed to have a breathing tube inserted into her throat." C) "Her asthma started bothering her last night." D) "There is a history of asthma on her father's side of the family." Answer: B Diff: 3 Page Ref: 1145 Objective: 38-10 77) You are treating a 9-year-old male who is short of breath and has a history of asthma. At the scene, he presents in severe respiratory distress, with intercostal retractions, lethargy, and expiratory wheezing. His pulse is 136 beats/min, respirations are 32 breaths/min, and SpO2 is 95%. You have assisted the patient with his albuterol MDI and are transporting him emergently to the hospital. As you reassess the patient, which finding is most concerning? A) SpO2 of 94% B) Increased wheezing heard bilaterally C) Heart rate of 68 beats/min D) Blood pressure of 100/64 mmHg Answer: C Diff: 3 Page Ref: 1145 Objective: 38-10 78) On scene, a 6-year-old male who is short of breath will not allow EMRs to place a nonrebreather face mask on his face. Assessment indicates moderate respiratory distress with the following vital signs: pulse, 124 beats/min; respirations, 24 breaths/min; blood pressure, 98/56 mmHg; and SpO2, 92%. At this time, you try: A) Using a nasal cannula with 2 lpm oxygen B) Holding the nonrebreather mask firmly to the patient's face C) Start positive pressure ventilation D) Placing a nasal airway and nasal cannula with 6 lpm oxygen Answer: A Diff: 3 Page Ref: 1138 Objective: 38-10 79) A mother asks you what causes her son to get bronchiolitis. You would inform her that this infection is caused by: A) An unknown bacterium B) The same bacterium that causes strep throat C) A pneumonia-type virus D) Respiratory syncytial virus Answer: D Diff: 3 Page Ref: 1146 Objective: 38-10

21 Copyright © 2018 Pearson Education, Inc.

80) A 7-year-old with a fever of 101.3°F and shortness of breath has a patent airway, but tachypneic respirations. His heart rate is 132 beats/min, respirations are 28 breaths/min, and blood pressure is 94/60 mmHg, with an SpO2 of 96% on room air. Crackles and wheezes are noted in the base of the right lung only. Given this presentation, you would suspect and treat the patient for: A) Bronchiolitis B) Pneumonia C) Asthma D) Upper respiratory infection Answer: B Diff: 2 Page Ref: 1147 Objective: 38-10 81) A 1-year-old male is in respiratory distress and wheezing. His pulse is 156 beats/min, respirations are 32 breaths/min, and SpO2 is 95% on 4 lpm of blow-by oxygen provided by onscene EMRs. His mother denies a medical history for the patient, but states that she has asthma and uses an albuterol inhaler. Given this information, what would be appropriate in the care of this child? A) Transport the patient in low Fowler's position B) Contact medical direction for authorization to administer albuterol MDI C) Add humidification to the oxygen D) Increase the oxygen to a high concentration through a mask Answer: C Diff: 2 Page Ref: 1145 Objective: 38-10 82) You are transporting a very sick 3-year-old with suspected septic shock. His airway is patent, respirations tachypneic, and radial pulse weak, with skin that is hot to the touch. Vital signs are pulse, 152 beats/min; respirations, 32 breaths/min; blood pressure, 94/64 mmHg; and SpO2, 94% on a nonrebreather face mask. The patient has a rectal temperature of 103.9°F. Tylenol has been administered. During transport to the hospital, you should be most concerned with which clinical finding? A) Blood pressure of 84/32 mmHg B) SpO2 of 94% despite oxygen C) Heart rate of 156 beats/min D) Temperature of 104.1°F Answer: A Diff: 3 Page Ref: 1121; Table 38-1 Objective: 38-11

22 Copyright © 2018 Pearson Education, Inc.

83) Assessment of a 4-year-old reveals him to be unresponsive with no spontaneous respirations or pulse. Your immediate action would be to: A) Attach the automated external defibrillator B) Transfer to stretcher and immediately transport C) Begin rescue breathing at 20 breaths per minute D) Start chest compressions Answer: D Diff: 2 Page Ref: 1150 Objective: 38-11 84) You suspect a very sick child to be in shock. His skin is dry, radial pulse weak, and blood pressure low. Which statement made by the parents would reinforce this suspicion? A) "He has had a headache since last night." B) "I gave him an aspirin this morning." C) "He has had diarrhea for three days." D) "His stepfather has really bad diabetes." Answer: C Diff: 3 Page Ref: 1148 Objective: 38-11 85) A 14-year-old female with a history of congenital heart disease and evidence of reaching puberty is in cardiac arrest. Which instruction would you give other EMTs on scene? A) "Let's use the adult pads and adult settings for the AED." B) "Just do CPR until ALS arrives since we do not know the specific heart condition." C) "Remember that for pediatric patients, it is 2 shocks followed by 2 minutes of CPR." D) "We will need the special pediatric pads for this patient." Answer: A Diff: 1 Page Ref: 1150 Objective: 38-11 86) A 15-year-old male with a history of seizures presents as oriented. He takes the anticonvulsant Keppra and has no other medical history according to his mother. The mother called EMS because the patient missed taking his medicine and she wants you to "check him out." Which action would be most appropriate for the EMT to take at this time? A) Check the patient's blood sugar B) Obtain a full set of vital signs C) Apply high-concentration oxygen through a nonrebreather mask D) Assist the mother in administering a dose of Keppra Answer: B Diff: 3 Page Ref: 1132 Objective: 38-5

23 Copyright © 2018 Pearson Education, Inc.

87) You note a 5-year-old patient seizing as you approach him. Which observation or piece of information would be most alarming? A) The patient has no history of seizures B) The patient has a history of leukemia C) The patient is incontinent of urine and feces D) The patient has been seizing for 12 minutes Answer: D Diff: 2 Page Ref: 1154 Objective: 38-12 88) As you assess a very sick 3-year-old patient with a fever, rash, and sore neck, you suddenly suspect he might have meningitis. Your immediate action would be to: A) Call for paramedic assistance B) Don additional BSI precautions of a mask and gown C) Administer aspirin for immediate fever reduction D) Place a cervical collar on the patient's neck and immediately transport him Answer: B Diff: 2 Page Ref: 1158 Objective: 38-12 89) You are concerned that a sick 5-year-old patient with lower abdominal pain and a temperature of 100.3°F has appendicitis. He describes the pain as generalized and crampy. Although he complains of nausea, he is not vomiting. His pulse, respirations, blood pressure, and SpO2 are normal, and there are no immediate life-threatening conditions. Based on these assessment findings, his father informs you that he does not wish his son to be transported, and will call the pediatrician in the morning. How would you respond? A) "I'm okay with that; monitor his fever and make sure to call first thing in the morning." B) "I am concerned that your son has appendicitis and by law we must treat and transport him." C) "I have to tell you, if he has appendicitis, it will just get worse and can become a major medical emergency for him." D) "If you insist, there is nothing I can legally do; let me talk to the pediatrician to let him know what is going on." Answer: C Diff: 3 Page Ref: 1159 Objective: 38-2

24 Copyright © 2018 Pearson Education, Inc.

90) A frantic mother has contacted EMS because she believes that her 4-year-old daughter got into her blood pressure medications and ate three or four pills. The patient presents as awake and oriented, with pale skin that is cool and diaphoretic. Her pulse is rapid and respirations adequate. Vital signs are pulse, 116 beats/min; respirations, 22 breaths/min; blood pressure, 82/44 mmHg; and SpO2, 99%. After completing the primary assessment, what would your immediate action be? A) Apply high-concentration oxygen B) Educate the mother on safe storage of medications C) Contact poison control and induce vomiting D) Obtain the name and dosage of the medication, and then call the poison control center Answer: D Diff: 2 Page Ref: 1159 Objective: 38-12 91) A 3-year-old female has been accidentally struck in the chest with a baseball bat during a domestic altercation. Assessment indicates redness to the right lateral chest with no deformity, crepitus, or broken skin. Vital signs are pulse, 124 beats/min; respirations, 22 breaths/min; blood pressure, 108/62 mmHg; and SpO2, 99% on room air. Given these assessment findings, what would your best action be? A) Provide spine motion restriction precautions and treat the patient as if she is in shock B) Release the child into the custody of the police department for temporary foster home placement C) Treat injury as if it is a flail chest and provide immediate transport D) Treat the child for suspected internal chest injury Answer: D Diff: 3 Page Ref: 1163 Objective: 38-15 92) At the scene of an MVC, you must remove an injured child from his car seat and provide full spine motion restriction precautions prior to transport to the hospital. Which statement made by the lead EMT indicates that this procedure is being performed correctly? A) "Let's carefully lift the baby from the car seat, while manually holding her neck, and place her on the long board." B) "As we place the patient on the long board, we will need to insert a towel under her neck to prevent flexion of the head." C) "Before we slide her feet-first from the car seat onto the long board, we will need to apply an infant cervical collar." D) "To get her on the long board, let's tilt the car seat backward, and carefully slide her out headfirst." Answer: D Diff: 3 Page Ref: 1164 Objective: 38-16

25 Copyright © 2018 Pearson Education, Inc.

93) You are transporting a critically ill 4-year-old patient. Regarding the panicked and upset mother, you would: A) Secure the mother to the stretcher and allow her to hold the patient B) Ask the mother to get to the hospital by way of private vehicle C) Secure the mother in the "captain's chair" of the ambulance, behind the patient, during transport to the hospital D) Allow the mother to kneel next to the stretcher so her son can see her Answer: C Diff: 2 Page Ref: 1166 Objective: 38-2 94) Another EMT tells you that he had a call the previous shift in which he thought a 5-year-old boy might be the victim of abuse, since he had bruises on both upper legs and chest. However, the EMT did not communicate this suspicion to anyone or transport the patient. What would your best response be? A) "If you do not take steps to report this suspicion, you can be punished in a court of law." B) "Let's see if we can go back to the house to make a better assessment of the situation." C) "This sounds more like neglect, rather than physical abuse, so there is not much to do at this point." D) "Bruises in these areas are very typical for an active 5-year-old, making abuse unlikely." Answer: A Diff: 3 Page Ref: 1169 Objective: 38-18 95) You are on scene of an MVC in which a 3-year-old female was in a car seat in the backseat. The child appears not to be injured, but the mother would like her taken to the hospital for an evaluation. Since the patient will need a car seat for transport, which criteria would indicate that the car seat she is currently sitting in cannot be used to safely transport the patient? A) The mother sustained minor injuries B) The car was easily driven by the police department from the scene for safe storage C) The front air bags of the car deployed D) The car seat appears to have a crack in the side when impacted by the door handle Answer: D Diff: 2 Page Ref: 1164 Objective: 38-16 96) You are called to a home for a 7-year-old female with a suspected upper respiratory infection. As you assess the patient, you note a bruise to her right upper arm. As you continue to assess and care for this child, you should: A) Apply an ice pack to the arm and transport the patient B) Note the bruise and continue to treat for the respiratory infection C) Ask the mother if the child hurt herself playing D) Treat the respiratory infection and perform a head-to-toe assessment Answer: D Diff: 2 Page Ref: 1169 Objective: 38-18 26 Copyright © 2018 Pearson Education, Inc.

97) A malnourished child would be an example of: A) Physical abuse B) Emotional abuse C) Neglect D) Nutritional abuse Answer: C Diff: 1 Page Ref: 1168 Objective: 38-18 98) The EMT is correctly assessing breath sounds on a 3-year-old male when he: A) Understands that sounds from the left lung can be heard on the right side of the chest B) Listens carefully for at least 1 minute to both sides of the anterior chest C) Listens to both the anterior and posterior aspects of the thorax for 1 minute D) Instructs the child to breathe through his nose as he listens to the posterior thorax Answer: A Diff: 2 Page Ref: 1127-1128 Objective: 38-4 99) You are transporting a child with a small laceration to his leg, sustained after cutting himself of a piece of broken glass. During transport, you observe a pulse oximeter reading of 87% on the frantic, crying, and actively moving child. The child is alert and oriented with a normal pulse and respiratory rate with pink skin that is warm to touch. You should consider: A) Starting positive pressure ventilation with supplemental oxygen B) Attempting to calm the child and obtain another SpO2 reading C) Applying high-concentration oxygen via a nonrebreather mask D) Checking the laceration for uncontrolled bleeding Answer: B Diff: 3 Page Ref: 1131 Objective: 38-5 100) The EMT is correctly assessing a child's apical pulse when he: A) Carefully feels over both carotid arteries for at least 30 seconds B) Uses a stethoscope to listen to and count the femoral pulse rate C) Feels both carotid and brachial pulses for equality and rate D) Listens with a stethoscope to the child's left side of the chest Answer: D Diff: 2 Page Ref: 1132 Objective: 38-5 101) The EMT is correctly assessing a child's BRUE when he: A) Obtains a thorough history B) Immediately starts the child on blow-by oxygen C) Expedites transport to the ED D) Allows the caretaker to hold the child during transport Answer: A Diff: 2 Page Ref: 1160 Objective: 38-13 27 Copyright © 2018 Pearson Education, Inc.

102) Pediatric injury prevention programs are an important aspect of child health care because: A) Young children tend to engage in risky behavior B) New parents routinely place their children at risk C) Almost all child deaths are preventable D) Children inherently lack mature decision-making skills Answer: D Diff: 2 Page Ref: 1168 Objective: 38-17 103) The need for a comprehensive understanding of normal growth and development, which enables the EMS provider to better anticipate the physiologic and emotional needs of the child who is affected by illness or injury, is a principle of: A) Young children's risky behavior B) Pediatric Advanced Life Support (PALS) C) The federal Emergency Medical Services for Children (EMSC) program D) Family-centered care Answer: D Diff: 2 Page Ref: 1170 Objective: 38-19 104) When you form an initial impression for a pediatric patient based on consciousness, breathing, and color, you are following: A) The risk exposure method B) The Pediatric Advanced Life Support (PALS) method C) The pediatric assessment triangle D) Family-centered care Answer: B Diff: 2 Page Ref: 1126 Objective: 38-6 105) You are determining the PGCS score for a 28-month-old female who was stung by a bee while playing in the backyard. You note that the patient is alert. When asked, she tells you that she was stung on the bottom of her foot. At this point in time her GCS score would be: A) 14 B) 15 C) 19 D) 10 Answer: B Diff: 2 Page Ref: 1130 Objective: 38-7

28 Copyright © 2018 Pearson Education, Inc.

106) BRUE evaluations are typically performed on infants who: A) Are at risk for unresolved events B) Had a medical episode that concerned their caregiver C) Had an event that the EMT is unable to resolve D) Have congenital carotid circulatory problems Answer: B Diff: 1 Page Ref: 1159 Objective: 38-13 107) Which statement regarding pediatric injury prevention is most accurate? A) Accident prediction is the critical component of injury prevention B) Accidents are understandable, predictable, and preventable occurrences that result in injury C) Preventing injury results in a reduction in accidents D) Injury prevention and accident prevention are not the same Answer: D Diff: 2 Page Ref: 1167-1168 Objective: 38-17 108) Ensuring that all children and adolescents, no matter where they live, attend school, or travel, have access to and receive appropriate care in a health emergency is the purpose of: A) The pediatric assessment triangle B) The federal Emergency Medical Services for Children (EMSC) program C) Pediatric Advanced Life Support (PALS) D) Family-centered care Answer: B Diff: 2 Page Ref: 1169 Objective: 38-19 109) How can an EMT help prevent the stress and anxiety associated with treating critically injured children? A) Learn and practice the skills and equipment used when treating children B) Engage in discussions with friends about stressful trips C) Utilize available mental health services to help diffuse the stress D) Control your emotions while rendering care Answer: A Diff: 2 Page Ref: 1170 Objective: 38-20

29 Copyright © 2018 Pearson Education, Inc.

Prehospital Emergency Care, 11e (Mistovich et al.) Chapter 39 Geriatrics 1) You have been asked to deliver a talk about EMS and its interaction with the geriatric population. Which point would you emphasize in your presentation? A) Assessment can be difficult since most persons older than 70 have some degree of dementia B) Many elderly patients have a combination of different diseases in various stages C) Geriatrics actually account for a smaller number of EMS calls and transports than might be thought D) Dementia and chronic disease are both inevitable parts of the aging process Answer: B Diff: 2 Page Ref: 1182 Objective: 39-2 2) A new EMT asks you why the elderly are at a higher risk for developing pneumonia, as compared to younger individuals. You inform him that one reason the elderly are more susceptible to respiratory infections because of a(n): A) Impaired swallowing of saliva B) Decreased alveolar gas exchange C) Decreased cough reflex D) Hardening of the major arteries Answer: C Diff: 3 Page Ref: 1184 Objective: 39-2 3) The son of an 88-year-old patient states that he told his mother to double the dose of her antibiotic so she would "get better faster." This represents a problem given that the elderly have a(n): A) Decreased respiratory function B) Increased amount of body water C) Decreased liver and kidney function D) Increased tolerance to medication Answer: C Diff: 3 Page Ref: 1187 Objective: 39-2 4) Which statement demonstrates an accurate understanding of vital signs in relation to the geriatric patient? A) "Due to damage over time to the heart, the resting heart rate is typically less than 60 beats per minute." B) "Fever tends to be more common and more severely elevated in elderly patients." C) "The resting respiratory rate is typically higher in elderly patients." D) "There is no real difference between the vital signs of a geriatric patient and those of a young adult." Answer: C Diff: 3 Page Ref: 1201; Table 39-4 Objective: 39-9 1 Copyright © 2018 Pearson Education, Inc.

5) In reviewing prehospital care reports, you note that more than 75 percent of the patients transported from a local geriatric extended care facility have been diagnosed with hypertension. As a knowledgeable EMT, you would recognize that: A) Hypertension is common among the elderly B) The patients are not taking blood pressure medications C) An infectious disease is spreading through the extended care facility D) All of the patients must have the same physician Answer: A Diff: 2 Page Ref: 1203; Table 39-5 Objective: 39-2 6) You have been called for a conscious and alert 83-year-old male complaining of a fever for three days. He has a history of peripheral vascular disease. When assessing the patient's circulation, you have a difficult time feeling his radial pulse. What would your next action be? A) Start cardiopulmonary resuscitation B) Check the patient's blood pressure C) Locate the patient's carotid pulse D) Apply supplemental oxygen Answer: C Diff: 2 Page Ref: 1204 Objective: 39-2 7) Your partner believes the 90-year-old male you are caring for is dehydrated. Which finding best reinforces his suspicion? A) Increased blood pressure B) Dry and flaky skin C) Poor skin turgor D) Dry mouth and tongue Answer: D Diff: 3 Page Ref: 1200 Objective: 39-9 8) When getting a medical history from a patient, you refer to him as Mr. Wrask. He stops you and asks you to call him Derek. You should: A) Call him Derek during the rest of your interactions B) Continue to address him as Mr. Wrask C) Avoid calling him by his name altogether D) Confirm with family members how they would like him addressed Answer: A Diff: 1 Page Ref: 1198 Objective: 39-9

2 Copyright © 2018 Pearson Education, Inc.

9) Which statement indicates an accurate understanding of administering positive pressure ventilation (PPV) to a geriatric patient? A) "An increase in the force of ventilation is needed to get air into the stiff lungs." B) "If the patient is breathing between 10 and 20 breaths per minute, PPV is not needed." C) "If the geriatric patient has a history of COPD, a pediatric bag-valve mask may be needed." D) "Gentle ventilations that make the chest rise are adequate to provide proper airflow to the lungs." Answer: D Diff: 3 Page Ref: 1205 Objective: 39-9 10) You have been called for an unknown medical emergency at a residence. On scene, you are presented with an 84-year-old patient who is confused and does not obey commands. The best means of determining the patient's normal mental status would be to: A) Examine the patient's pupils B) Question family members C) Look at the patient's medications D) Check the patient's vital signs Answer: B Diff: 2 Page Ref: 1192 Objective: 39-5 11) A patient with advanced dementia who is living with family members is unable to provide an accurate medical history. In this situation, what would the EMT's best course of action be? A) Transport the patient to a hospital where he has been previously treated B) Have the patient's family sign a release-of-medical-information form C) Determine medical conditions based on assessment findings D) Ask family members about the patient's past medical history Answer: D Diff: 2 Page Ref: 1195 Objective: 39-5 12) An elderly female fell down three steps and now complains of back pain. After performing the primary assessment, which question would be most important to ask first? A) "Did you become dizzy and fall or slip and then fall?" B) "Why did you not use the ramp on the other side?" C) "Do you have a history of high blood pressure?" D) "Which medications are you taking?" Answer: A Diff: 2 Page Ref: 1196 Objective: 39-9

3 Copyright © 2018 Pearson Education, Inc.

13) You have been called for a 77-year-old male with chest discomfort. On scene, the patient denies ever having chest discomfort and requests to sign a refusal-of-care form. Before having the form signed, the EMT must consider that many geriatric patients may deny symptoms because of: A) Fear of losing independence and dignity B) Apprehension about having to take medications C) Confusion that chest discomfort often creates in geriatric patients D) Acute onset of dementia and other neurologic diseases Answer: A Diff: 2 Page Ref: 1204 Objective: 39-3 14) You have just arrived by the side of an elderly patient who accidentally overdosed with his daily medications after becoming confused about what to take and when. Given this scenario, what would be a priority in caring for this patient? A) Determine which medications were taken and how much B) Determine the patient's past medical history C) Gather up all medications for transport with the patient D) Evaluate and support the patient's airway and breathing Answer: D Diff: 2 Page Ref: 1200 Objective: 39-9 15) When assessing a 91-year-old female who fell, which assessment finding should concern the EMT most? A) Skin tear that appears infected on her arm B) Heart rate of 90 beats per minute C) Sudden onset of confusion D) History of osteoporosis and leg pain Answer: C Diff: 2 Page Ref: 1202 Objective: 39-6 16) You have just arrived by the side of a nonverbal patient with snoring respirations. The caregiver states that they were eating lunch when the patient suddenly collapsed face first onto the table. She also states that the patient has a history of a stroke, high blood pressure, and colon cancer. Your priority in caring for this patient at this time would be: A) Rapid transfer of the patient to the cot B) Feel for a radial or carotid pulse C) Open and look into the patient's mouth D) Obtain a heart rate and blood pressure Answer: C Diff: 2 Page Ref: 1200 Objective: 39-9

4 Copyright © 2018 Pearson Education, Inc.

17) A patient with kyphosis has fallen and now complains of new-onset neck and back pain with numbness in both arms. Regarding spine motion restriction, you should: A) Avoid spinal motion restriction procedures and transport the patient in a position of comfort B) Transport the patient in a prone position on a spine board with a cervical collar applied C) Place the patient on a spine board without applying a cervical collar D) Place pillows in voids between the patient and the long board during the spine motion restriction process Answer: D Diff: 2 Page Ref: 1205 Objective: 39-2 18) You have been called to an assisted living facility to transport a patient who has been vomiting continuously for 6 hours. She is responsive to verbal stimuli. The patient has both arthritis and severe kyphosis. How should you transport this patient on the stretcher? A) Semi-Fowler's position B) Left lateral recumbent position C) Secured to a long spine board D) Prone with her head turned to the side Answer: B Diff: 2 Page Ref: 1205 Objective: 39-2 19) You have been called for a 79-year-old female complaining of shortness of breath. Due to the patient's increased respiratory rate, cough, and production of discolored sputum, you are suspicious of pneumonia. A family member asks you how she can have pneumonia, when her temperature is only 97.9°F. Your best response is: A) "It is not uncommon for the elderly to have pneumonia without the signs of high fever or chills." B) "You will have to ask the doctor because I cannot give out medical information." C) "There is less need for an increased body temperature since the elderly have a higher number of white blood cells." D) "Higher blood pressure, typically seen in the elderly, tends to inhibit the body from producing a fever." Answer: A Diff: 3 Page Ref: 1188; Table 39-1 Objective: 39-4 20) Family members tell you that 3 hours ago, their 76-year-old mother suddenly became confused and had great difficulty speaking. However, within 15 minutes she returned to normal. Based on this description, the EMT should be suspicious of: A) Stroke B) Dementia C) Alzheimer's disease D) Transient ischemic attack Answer: D Diff: 2 Page Ref: 1193 Objective: 39-1 5 Copyright © 2018 Pearson Education, Inc.

21) While providing a continuing education seminar for your EMS system about geriatric patients, which point about geriatric trauma would you make? A) MVCs are responsible for more than 75 percent of deaths among elderly patients B) Head injuries are easy to detect in the elderly C) Hyperthermia is an early sign of blood loss D) Injuries in elderly patients are most commonly caused by falls Answer: D Diff: 2 Page Ref: 1196 Objective: 39-9 22) Which description best explains the term "silent heart attack"? A) The chest discomfort was so intense the patient could not speak B) The patient had a single episode of chest discomfort followed by cardiac arrest C) The chest discomfort did not radiate into the arms, neck, or back D) The patient had little to no chest discomfort Answer: D Diff: 2 Page Ref: 1187 Objective: 39-1 23) You have been called for a 91-year-old female who fell. Since your assessment reveals bruises to her body in various stages of healing and inconsistencies in information provided by her family, you are suspicious of geriatric abuse. What would your best action be? A) Contact the police and wait on scene until they arrive B) Confront the family members with your suspicion C) Inform your medical director and include your suspicion in the PCR D) Quietly transport the patient and inform the emergency department physician Answer: D Diff: 2 Page Ref: 1198 Objective: 39-8 24) To best communicate with a geriatric patient with diminished eyesight, the EMT should: A) Write questions in large print on paper B) Increase both the pitch and volume of his voice C) Stand in front of the patient and talk normally D) Obtain needed information from family members Answer: C Diff: 2 Page Ref: 1202 Objective: 39-2

6 Copyright © 2018 Pearson Education, Inc.

25) A hearing-impaired patient with a hearing aid is having a difficult time understanding your questions. Your initial action would be to: A) Write the questions on paper B) Slow your speech and increase the pitch of your voice C) Ensure that the patient's hearing aid is properly placed and turned on D) Position yourself in front of the patient and speak loudly Answer: C Diff: 2 Page Ref: 1202 Objective: 39-2 26) You have been called to the side of a confused geriatric female who complains of abdominal pain. Before palpating the patient's abdomen, you would first: A) Explain to family members why you are feeling the patient's abdomen B) Find out if the patient has a Do Not Resuscitate (DNR) order C) Inform the patient that you will be pressing on her abdomen D) Determine whether the patient has a history of respiratory disease Answer: C Diff: 2 Page Ref: 1203 Objective: 39-5 27) An 84-year-old male complains of generalized weakness and an ache in his left shoulder. He also reports mild shortness of breath, but has clear lung sounds and an SpO2 of 93%. When asked, he reports that the discomfort started after he got out of bed 2 hours ago. He denies any known injury to his shoulder or back. He has a history of coronary artery disease, hypertension, TIA, and irritable bowel syndrome. Family members state that the patient has early dementia. There are no obvious life threats, and his vital signs are normal for his age. The appropriate treatment for this patient would include: A) Continue to monitor the patient and transport him B) Provide oxygen and treat the patient as a cardiac patient C) Immobilize the patient's right arm and shoulder D) Apply ice and a sling to the patient's shoulder and transport him Answer: B Diff: 2 Page Ref: 1200 Objective: 39-3

7 Copyright © 2018 Pearson Education, Inc.

28) An 85-year-old male with a history of heart attacks has called 911 for chest discomfort. He states that the pain started 2 hours ago when he was sitting on his porch. When asked, he states that it feels similar to his last heart attack, only this time he is also dizzy. The primary assessment indicates no life threats, and the Emergency Medical Responder on scene reports the following vital signs: pulse, 92 beats/min; respirations, 18 breaths/min; blood pressure, 88/52 mmHg; and SpO2, 92% on room air. The patient has nitroglycerin, but has not taken it yet (you have a standing order to administer nitroglycerin without calling medical direction). After applying oxygen, you should: A) Perform the secondary exam B) Assist the patient with his nitroglycerin C) Provide nonemergent transport D) Assess for neurologic deficits Answer: A Diff: 2 Page Ref: 1202 Objective: 39-3 29) A 92-year-old male states that he "passed out" and fell. He regained consciousness a few moments later, but had shortness of breath and sharp chest discomfort. Which element of the patient's history is of greatest concern? A) He had hip surgery two years ago B) The patient took nitroglycerin and still has pain C) The patient states his left calf is tender and painful D) The patient injured his wrist when he fell Answer: B Diff: 3 Page Ref: 1187-1188 Objective: 39-3 30) Which complication from a pulmonary embolism is of greatest concern to the EMT? A) Pulmonary artery infection B) Hypertension to the pulmonary veins C) Pulmonary edema and diminished breath sounds D) Hypoxia from poor lung perfusion Answer: D Diff: 2 Page Ref: 1190 Objective: 39-4 31) When assessing a patient, which finding should raise the EMT's suspicion that the patient may have a pulmonary embolism? A) Bradycardia with normal SpO2 reading and clear breath sounds B) Sudden onset of shortness of breath with clear breath sounds C) Chest pain that is "dull" and radiates to the arm with abnormal breath sounds D) Chest and neck pain with crackles in the bases of both lungs Answer: B Diff: 3 Page Ref: 1190 Objective: 39-4

8 Copyright © 2018 Pearson Education, Inc.

32) When assessing a geriatric patient for pneumonia, the EMT must remember that: A) A cough with bloody sputum is typically observed with pneumonia B) Hypotension is commonly associated with the pulmonary infection C) A normal body temperature may still be observed in this patient D) Chest pain and chills are almost always present in geriatric patients with pneumonia Answer: C Diff: 2 Page Ref: 1190 Objective: 39-2 33) You are preparing to transport an 86-year-old female with an altered mental status who complains of nausea and vomiting. During transport, the occurrence of aspiration pneumonia would be best prevented by: A) Suctioning the patient's airway as soon as she vomits B) Providing supplemental oxygen C) Giving the patient aspirin prior to transport D) Transporting the patient in a semi-Fowler's position Answer: D Diff: 2 Page Ref: 1205 Objective: 39-2 34) You have been called to a geriatric extended care facility to transport a patient with fever and pneumonia to the hospital for further care and treatment. The primary focus of your care during this call will be: A) Oxygen therapy and safe transport B) Providing PPV while initiating rapid transport C) Actively cooling the patient if he has a fever D) Administering a bronchodilator to open the alveoli up Answer: A Diff: 2 Page Ref: 1190 Objective: 39-4 35) An 81-year-old female called 911 when she suddenly lost vision in her right eye. Based on this description, the EMT should be suspicious of: A) Cataracts B) Glaucoma C) Macular degeneration D) Retinal detachment Answer: D Diff: 1 Page Ref: 1201; Table 39-4 Objective: 39-9

9 Copyright © 2018 Pearson Education, Inc.

36) You have been called to a geriatric extended care facility for a 91-year-old female with pneumonia. The patient is to be transported to a local hospital for further treatment. When asked, the staff confirms and then produces the patient's Do Not Resuscitate (DNR) order. The patient's airway is patent, respirations labored, and skin warm, with an intact radial pulse. The vital signs are pulse, 124 beats/min; respirations, 20 breaths/min; blood pressure, 104/58 mmHg; SpO2, 91%; and temperature, 101.2°F. The patient also complains of chest discomfort and has a cough. Given this situation, the focus of your care should be: A) Rapid transport to the hospital B) Ensuring adequate oxygenation C) Administering aspirin to decrease the fever D) Eliminating the chest discomfort with nitroglycerin Answer: B Diff: 2 Page Ref: 1190 Objective: 39-4 37) You have been called for a 77-year-old female with altered mental status. On scene, you note the patient's inability to speak and vomitus in her airway. Her breathing is labored and inadequate, and radial pulse rapid and weak. She is not moving her right arm or right leg. What should be your first priority? A) Start positive pressure ventilation with supplemental oxygen B) Administer glucose and immediately transport the patient to a stroke center C) Suction the airway and determine adequacy of the breathing D) Establish the time of stroke onset and start high-concentration oxygen Answer: C Diff: 2 Page Ref: 1193 Objective: 39-6 38) An 82-year-old female with nausea and vomiting has called 911. When obtaining her past medical history, she informs you that she suffered a transient ischemic attack (TIA) two months ago. Based on this information, what should the EMT expect to find? A) Altered motor and sensory to one side of the body B) Unequal pupils C) Difficulty in speaking and/or swallowing D) The absence of any neurologic deficits Answer: D Diff: 2 Page Ref: 1193 Objective: 39-9

10 Copyright © 2018 Pearson Education, Inc.

39) An 86-year-old male has experienced a seizure and is now lethargic and confused. His family states that the patient has a history of seizures and was sleeping in bed when the seizure was witnessed by his wife. He remains in bed with an intact airway, adequate breathing, and a rapid radial pulse. You also note him to be incontinent of urine. Appropriate care of this patient would include: A) Considering oxygen and transport with him positioned on his left side B) Securing the patient tightly to the stretcher to prevent injury should he seize again C) Inserting an oral airway and starting positive pressure ventilation with supplemental oxygen D) Considering supplemental oxygen and securing the patient on a long spine board with a cervical collar placed Answer: A Diff: 2 Page Ref: 1193 Objective: 39-6 40) The family of an 87-year-old female has called 911. The patient is confused and exhibiting improper and out-of-character behavior. The family informs you that the patient has diabetes, but takes pills and not insulin. They also state that she has been complaining of dizziness and has been drinking water nonstop as well as urinating. Assessment reveals adequate breathing, skin that is cool, and a radial pulse that is rapid and weak. The glucometer is malfunctioning and not providing a glucose reading. Based on her history, you should be suspicious of and treat the patient for: A) Elevated blood sugar B) Urinary tract infection C) Stroke or transient ischemic attack D) Hypoglycemia Answer: A Diff: 3 Page Ref: 1194 Objective: 39-9 41) You have been summoned for a patient with diabetes who exhibits altered mental status. Which sign would cause you to suspect that the patient has hyperosmolar hyperglycemic syndrome (HHS)? A) Blood glucose level of 52 mg/dL B) Inability to urinate C) Complaint of thirst and dry mouth D) Fruity smell on his breath Answer: C Diff: 3 Page Ref: 1193-1194 Objective: 39-5

11 Copyright © 2018 Pearson Education, Inc.

42) A 77-year-old male got out of bed, became dizzy, and fell onto the floor, hitting the right side of his face. You find the patient sitting on the floor with his wife next to him. His wife states that he did not lose consciousness and that this has happened several times over the past few days, ever since the doctor put him on a new blood pressure medication. Presently, the patient is alert and oriented and complains of dizziness, headache, and neck pain. He has a history of coronary artery disease, emphysema, and hypertension. Which action should the EMT perform first? A) Provide supplemental oxygen to maintain SpO2 of 95% or greater B) Take manual spine motion restriction precautions and begin the primary assessment C) Position the patient supine with his feet elevated and obtain vital signs D) Summon ALS and initiate transport to a stroke center Answer: B Diff: 2 Page Ref: 1196 Objective: 39-9 43) You have been called for an 84-year-old female who fell and hit her head. The patient is confused and has a laceration on her forehead. When getting the history from family members, which statement suggests that the patient is suffering from delirium? A) "She has a history of progressive dementia." B) "Her memory has been fading over the last two years." C) "She could not remember her name yesterday." D) "She wants to eat only junk food these days." Answer: C Diff: 2 Page Ref: 1195 Objective: 39-1 44) Alzheimer's disease is diagnosed by: A) Blood tests B) Autopsy C) Neurologic exams D) Involved memory test Answer: B Diff: 2 Page Ref: 1195 Objective: 39-1 45) You have been called for a geriatric patient with advanced dementia. The staff at the geriatric extended care facility believes that the patient has a urinary tract infection. At bedside, you find the patient to be confused and aggressive. She is on constant oxygen at 3 lpm for COPD. During transport, she repeatedly yells at you and strikes out at you. You would respond by: A) Applying a nonrebreather face mask with 3 lpm of oxygen B) Contacting advanced life support for administration of a sedative C) Tightening the straps so that she is tightly restrained to the stretcher D) Moving away from the patient so she cannot strike you Answer: D Diff: 2 Page Ref: 1195 Objective: 39-5 12 Copyright © 2018 Pearson Education, Inc.

46) An 83-year-old male fell down five stairs to the floor below. He hit his head and now complains of a headache. When getting a history from this patient, which question should be asked before all others? A) "What caused you to fall?" B) "Which medications are you taking?" C) "Do you have a history of medical problems?" D) "What is the name of your primary care physician?" Answer: A Diff: 2 Page Ref: 1196 Objective: 39-9 47) You have been called by the caregiver of a 91-year-old female with dementia. The patient complains of pain and deformity to her upper left arm. As you assess the patient, you note various bruises to the patient's body, and become suspicious of elder abuse. Which action is most appropriate at this time? A) Contact the police to report your suspicion B) Confront the caregiver regarding the bruises to the patient's arm C) Ask the patient if she is being abused D) Treat the patient for a possible broken arm and initiate transport Answer: D Diff: 2 Page Ref: 1198 Objective: 39-8 48) A patient with postural hypotension would most likely make which statement? A) "When lying in bed, I have a hard time breathing." B) "My shoes are tight because my ankles are swollen." C) "I get so dizzy when I get up off of the couch." D) "I go through periods where my heartbeat seems irregular." Answer: C Diff: 3 Page Ref: 1184 Objective: 39-2 49) Pathologic narrowing of the valves within the heart is referred to as: A) Stenosis B) Cardiac hypertrophy C) Dysrhythmia D) Pulmonary edema Answer: A Diff: 1 Page Ref: 1182 Objective: 39-1

13 Copyright © 2018 Pearson Education, Inc.

50) A patient with a history of arteriosclerosis has: A) Weakened heart B) Stiffened arteries C) Hypotension D) Occluded veins Answer: B Diff: 1 Page Ref: 1184 Objective: 39-1 51) Vision disturbances caused by glaucoma occur secondary to: A) Clouding of the lens B) Lack of tear production C) Persistent hypotension D) Damage to the optic nerve Answer: D Diff: 3 Page Ref: 1185 Objective: 39-1 52) A patient with a history of cataracts would most likely complain of: A) Eye swelling and dryness B) Color blindness C) Loss of peripheral vision D) Inability to see clearly Answer: D Diff: 2 Page Ref: 1185 Objective: 39-9 53) A heavy dresser fell onto a 76-year-old male's foot as he attempted to move it across his bedroom floor. His right foot and ankle are bruised, but the patient is refusing treatment or transport since it does not hurt. Which statement made by the patient during your assessment would be most concerning and warrant contact with medical command for further advisement? A) "I have a history of congestive heart failure." B) "I was recently diagnosed with cataracts." C) "I have been told I have peripheral neuropathy." D) "I fell and broke my left ankle last year." Answer: C Diff: 2 Page Ref: 1185 Objective: 39-9

14 Copyright © 2018 Pearson Education, Inc.

54) An 89-year-old female is short of breath and has a fever. Her SpO2 is low, and she has crackles and coarse breath sounds to both lungs. When getting the patient's history from her family caregivers, which statement would make you suspicious that the patient has aspirated something? A) "Her legs and ankles seem more swollen today." B) "She was fine until just after eating lunch." C) "Her doctor prescribed an antibiotic for her sinus infection." D) "She has not been feeling well for three days." Answer: B Diff: 3 Page Ref: 1190 Objective: 39-4 55) Dementia is typically observed as: A) A chronic and irreversible condition B) Altered mental status with a reversible underlying cause C) A chronic condition with a sudden onset D) Altered mental status rooted in a psychiatric disorder Answer: A Diff: 1 Page Ref: 1194-1195 Objective: 39-5 56) A 76-year-old female fell while walking to her bathroom. She is now on the floor and reports severe pain to her right hip. Her airway is patent and her breathing adequate. Family members tell you that she takes medication for high blood pressure, high cholesterol, and depression. When assessing the patient, which assessment finding is most concerning? A) Heart rate of 128 beats per minute B) Hip pain that increases with palpation C) Weak pedal pulse in the right foot D) Respiratory rate of 18 breaths per minute Answer: A Diff: 3 Page Ref: 1196 Objective: 39-7 57) Ageism would occur when the EMT: A) Asks all geriatric males if they remember World War II B) Asks family members for medical information when the patient is confused C) Does not attempt to assess combative elderly patients with dementia because they "always act that way." D) Disregards an advance directive that is properly filled out and presented to the EMS providers Answer: C Diff: 3 Page Ref: 1198 Objective: 39-1

15 Copyright © 2018 Pearson Education, Inc.

58) The EMT would recognize potential geriatric neglect when the patient makes which statement? A) "They are rough with me when I wet myself and need my bedding changed." B) "They do not give me my high blood pressure medications daily as I think they are supposed to." C) "They are spending all my money on illegal drugs." D) "They wake me up every 6 hours at night to see if I am incontinent." Answer: B Diff: 3 Page Ref: 1197 Objective: 39-8 59) When an elderly individual is injured, deterioration of compensatory mechanisms in the patient can result in: A) A rapid onset of shock B) Resistant hypertension C) Agitation D) Chronic depression Answer: A Diff: 1 Page Ref: 1196 Objective: 39-7

16 Copyright © 2018 Pearson Education, Inc.

Prehospital Emergency Care, 11e (Mistovich et al.) Chapter 40 Patients with Special Challenges 1) Which statement about abuse is true? A) It is typically found in those individuals who have lower incomes B) It tends to occur more often to individuals with mental health problems C) It most often occurs to those individuals who are unable to report it D) It is found in all socioeconomic levels Answer: D Diff: 1 Page Ref: 1221 Objective: 40-10 2) A 49-year-old male has summoned EMS because of a problem with his tracheostomy tube. At the patient's side, you note the tracheostomy tube to be protruding abnormally from the stoma in his neck. Your first action in caring for this patient would be to: A) Gently attempt to reinsert the tracheostomy tube B) Quickly establish a baseline SpO2 level C) Establish how the tracheostomy tube became dislodged D) Evaluate the adequacy of the patient's airway and breathing Answer: D Diff: 2 Page Ref: 1228 Objective: 40-2 3) When assessing a 32-year-old male with a history of developmental challenges, you note the protrusion of a clamped tube from his back in the area of his right kidney. The best method of determining what the tube is and why it is in place would be to: A) Contact medical direction B) Ask family members C) Contact the home health nursing agency D) Ask the patient Answer: B Diff: 1 Page Ref: 1215 Objective: 40-4 4) Which description best fits an oxygen concentrator that may be part of a patient's home medical equipment? A) Oxygen device that extracts oxygen from the air B) Nasal cannula connected to a small cylinder of oxygen C) Small portable device that administers high-concentration oxygen D) Small oxygen tank that holds concentrated liquid oxygen Answer: A Diff: 1 Page Ref: 1227 Objective: 40-12

1 Copyright © 2018 Pearson Education, Inc.

5) You have been called to a residence for a 63-year-old female with a history of COPD who is short of breath. The patient is moderately dyspneic with the following vital signs: pulse, 124 beats/min; respirations, 20 breaths/min; blood pressure, 162/56 mmHg; and SpO2, 86%. She is using an oxygen concentrator set at 4 lpm. What is your best course of action? A) Attach a nonrebreather mask to the oxygen concentrator and increase the flow to 12 lpm B) Continue oxygen administration with the concentrator and transport the patient C) Remove the oxygen concentrator and administer high-concentration oxygen to the patient D) Administer medical oxygen from your cylinder at the same rate as the concentrator Answer: C Diff: 2 Page Ref: 1227 Objective: 40-12 6) You are by the side of a 2-month-old baby who was born prematurely and is on an apnea monitor. While the patient is in your care, the apnea alarm emits a loud alarm. Quick assessment of the baby reveals no respiratory activity. What would your immediate action be? A) Place the baby on the cot and transport B) Start positive pressure ventilation C) Initiate high-concentration oxygen and summon ALS D) Allow the apnea monitor to perform a "self-diagnostic" test Answer: B Diff: 2 Page Ref: 1227-1228 Objective: 40-11 7) A family has called you after the apnea alarm for their 1-month-old baby sounded. They state that the settings seem too sensitive and the alarm goes off frequently. They would like the EMS squad to readjust the settings, so that this scenario can be avoided in the future. They do not want the patient transported to the hospital and are prepared to sign a refusal-of-care form. Assessment reveals no life threats to the patient's airway, breathing, or circulation. Given the situation, you would: A) Have a refusal-of-care form witnessed and signed B) Adjust the monitor settings and have a refusal-of-care form signed C) Contact medical direction D) Contact the home health nurse Answer: C Diff: 2 Page Ref: 1227-1228 Objective: 40-12

2 Copyright © 2018 Pearson Education, Inc.

8) A patient has terminal lung cancer and a valid Do Not Resuscitate (DNR) order. His wife has called 911 because her husband's pulse oximeter reading fell below 85% and an alarm sounded. The patient is currently receiving home oxygen through a nasal cannula. Assessment reveals him to be alert and oriented and slightly tachypneic with no abnormal lung sounds. At this time, your next action would be to: A) Start positive pressure ventilation B) Check the SpO2 using your pulse oximeter C) Administer oxygen through a new nasal cannula D) Place the patient on the stretcher for transport to the hospital Answer: B Diff: 2 Page Ref: 1228 Objective: 40-12 9) When assessing a 65-year-old male patient, you note a tube-like device inserted into the lower portion of his trachea. You recognize that the device has most likely been placed to: A) Permit the exchange of O2 and CO2 B) Allow the patient to be fed a liquid diet C) Allow the patient to talk D) Allow easy suctioning of the lower airway Answer: A Diff: 2 Page Ref: 1228 Objective: 40-12 10) When suctioning a 46-year-old male with a tracheostomy tube, which procedural step is appropriate? A) Introduce the suction catheter into the tracheostomy tube by first inserting it into the mouth or nose and then threading it lower B) Suction no more than 5 seconds using a hard-tipped suction C) Introduce 20 mL of sterile water into the tracheostomy tube prior to suctioning D) Insert the soft suction cannula until mild resistance is met and then initiate suction Answer: D Diff: 2 Page Ref: 1229 Objective: 40-12 11) A confused patient with a tracheostomy tube is breathing shallowly at 8 times per minute. The family states they accidentally overdosed him on some of his medications when two different family members were caring for him at two different times today. The patient is nonverbal because of the tracheostomy and has an SpO2 of 81% on room air. What would your immediate action be? A) Attach a bag-valve device to the tracheostomy tube and initiate ventilations B) Place a tracheostomy mask over the tube and administer high-concentration oxygen C) Deflate the cuff on the tracheostomy tube and begin positive pressure ventilation D) Use the bag-valve device with a pediatric mask and ventilate by sealing it over the tube Answer: A Diff: 2 Page Ref: 1228 Objective: 40-3 3 Copyright © 2018 Pearson Education, Inc.

12) The fundamental reason a patient uses BiPAP when sleeping is to: A) Provide high-concentration oxygen B) Keep the small airways open C) Lower the amount of exhaled CO2 D) Decrease the incidence of lower airway infection Answer: B Diff: 3 Page Ref: 1229 Objective: 40-12 13) While assessing a patient on a home mechanical ventilator, the low pressure alarm sounds. What would your immediate action be? A) Start positive pressure ventilation with high-concentration oxygen B) Increase the rate of ventilation on the ventilator C) Check the ventilator tubing for kinking or obstruction D) Check the connections in the ventilator tubing and between the tubing and the tracheostomy for leaks Answer: D Diff: 3 Page Ref: 1230 Objective: 40-12 14) The high pressure alarm is sounding on a home ventilator. As your partner prepares the bagvalve mask, you quickly determine that no problem exists with the machine itself. As positive pressure ventilation is initiated, you note high resistance to the ventilations and have a difficult time adequately squeezing the bag. Your next action would be to: A) Set the ventilator rate faster with a greater tidal volume and initiate transport B) Remove the ventilator, apply oxygen via a nonrebreather mask placed over the tracheostomy tube, continue care, and transport the patient C) Not change the ventilator or settings, but rather summon ALS for on-scene backup or en route intercept D) Replace the ventilator circuit with a new one and try using the home ventilator again Answer: C Diff: 3 Page Ref: 1230 Objective: 40-12 15) At a long-term care facility, a nurse informs you that the patient whom you will be transporting has a peripherally inserted central catheter (PICC) in place. To find this catheter, you should look at the patient's: A) Abdomen B) Anterior chest C) Hand D) Antecubital fossa Answer: D Diff: 2 Page Ref: 1233 Objective: 40-13

4 Copyright © 2018 Pearson Education, Inc.

16) To find an implanted medication port on a patient, the EMT should: A) Examine the patient's arm B) Check the lower abdomen C) Palpate the anterior chest D) Look at the lateral neck Answer: C Diff: 2 Page Ref: 1233 Objective: 40-13 17) A central intravenous catheter is typically placed for which reason? A) Repeated administration of medications B) High risk for cardiac arrest C) Administration of transvenous oxygen D) Easy access for hemodialysis Answer: A Diff: 3 Page Ref: 1233 Objective: 40-13 18) Assessment of a patient who complains of the sudden onset of chest discomfort reveals a central intravenous catheter to his right upper chest. While assessing this piece of medical equipment, which finding related to the catheter should be of greatest concern to the EMT? A) The skin is reddened around the insertion site B) The line is unclamped and without a cap C) There appears to be fluid in the catheter D) The catheter tubing is kinked Answer: B Diff: 3 Page Ref: 1233-1234 Objective: 40-13 19) A patient has accidentally removed his peripherally inserted central venous catheter (PICC). What would appropriate care for this patient include? A) Place an occlusive dressing over the insertion site to prevent air entry B) Apply a tourniquet just above the site to decrease air entry into the body C) Place a splint on the extremity and wrap the site with a pressure dressing D) Apply a bulky dressing to the insertion site to control any bleeding Answer: D Diff: 3 Page Ref: 1233 Objective: 40-13 20) Which statement about peritoneal dialysis is true? A) Peritoneal dialysis is more effective than hemodialysis B) A catheter is inserted into the abdomen during peritoneal dialysis C) Peritoneal dialysis us effective only in patients with acute renal failure D) A constant low-grade fever is normal and common in patients receiving peritoneal dialysis Answer: B Diff: 2 Page Ref: 1235 Objective: 40-16 5 Copyright © 2018 Pearson Education, Inc.

21) You have been called for a male patient who began complaining of chest discomfort while receiving hemodialysis. Assessment shows no threats to the airway, breathing, or circulation. His vital signs and blood sugar are normal. Which question should you ask first? A) "Was the dialysis session completed?" B) "How were you feeling prior to the dialysis session?" C) "How many sessions does he have each week?" D) "When did you eat or drink last?" Answer: A Diff: 1 Page Ref: 1235 Objective: 40-16 22) When assessing a patient undergoing dialysis, including his AV graft, which finding would be of greatest concern to the EMT? A) Feeling of a vibration through the graft B) Palpable pulse in the graft C) Red and hot skin at the graft site D) Strong radial pulse distal to the graft site Answer: C Diff: 2 Page Ref: 1236 Objective: 40-16 23) A patient has a dialysis AV graft in his right forearm. Which statement regarding the EMT's assessment and interaction with this technology is correct? A) A blood pressure cannot be taken in the right arm B) The blood pressure cuff must be placed above the AV graft C) The blood pressure can be palpated only in the right arm D) A blood pressure can be obtained by placing the cuff below the graft Answer: A Diff: 1 Page Ref: 1236 Objective: 40-16 24) A patient who received peritoneal dialysis has called 911 for generalized weakness. Upon arrival of EMS, which finding should be of greatest concern in this patient? A) Heart rate of 112 beats/min B) Temperature of 101°F C) Blood glucose of 146 mg/dL D) Complaint of thirst Answer: B Diff: 3 Page Ref: 1236 Objective: 40-16

6 Copyright © 2018 Pearson Education, Inc.

25) A patient who is completely reliant on his gastric tube for feeding has accidentally pulled it from his body. Which statement is most accurate about this patient? A) He cannot eliminate urine from his body B) He is bleeding internally C) He is unable to receive his oral medications D) He is incapable of digesting fiber Answer: C Diff: 3 Page Ref: 1237 Objective: 40-17 26) When assessing a patient at an extended care facility, you note the presence of an ostomy bag. Based on this finding, you realize that the patient has impaired function of: A) Urinary elimination B) Fluid balance C) Kidney activity D) Fecal elimination Answer: D Diff: 1 Page Ref: 1237 Objective: 40-17 27) The EMT should recognize a Texas catheter when he observes a: A) Catheter inserted into the patient's penis B) Catheter protruding from the patient's pelvic region C) Condom-like device over the patient's penis D) Collection bag attached to the patient's abdomen Answer: C Diff: 2 Page Ref: 1238 Objective: 40-17 28) While assessing a patient with altered mental status, you note a catheter-like device projecting from the center of the patient's pelvic area. This is most likely a(n): A) Ostomy B) Indwelling urinary catheter C) Intraventricular shunt D) Suprapubic catheter Answer: D Diff: 2 Page Ref: 1238 Objective: 40-17

7 Copyright © 2018 Pearson Education, Inc.

29) You are preparing a patient with an indwelling urinary catheter for transport. Pertaining to the catheter, which action must the EMT take? A) Place the catheter bag below the patient's bladder B) Leave the urine in the bag for the hospital to analyze C) Deflate the catheter balloon prior to transport D) Remove the collection bag and clamp the tubing Answer: A Diff: 2 Page Ref: 1239 Objective: 40-17 30) You have been called to a residence for a patient who is sick and not feeling well. The patient has multiple medical problems and is confined to bed, with care provided by family members. The patient has a gastric tube and an indwelling urinary catheter. Which assessment finding should raise your suspicion that the patient has a urinary tract infection? A) Complaint of abdominal pressure B) Cloudy urine in the bag of the urinary catheter C) Complaint of nausea and vomiting D) No urine noted in the urinary collection bag Answer: B Diff: 3 Page Ref: 1239 Objective: 40-17 31) A pediatric patient has an intraventricular shunt, according to the parents. Based on this information, you recognize that there is a problem with which system in the body? A) Genitourinary B) Central nervous C) Digestive D) Cardiovascular Answer: B Diff: 1 Page Ref: 1239 Objective: 40-18 32) An intraventricular shunt benefits a patient by: A) Providing a route to eliminate urine from the bladder B) Draining fluid from around the heart C) Allowing medication to be given to the patient D) Draining cerebrospinal fluid from the brain Answer: D Diff: 2 Page Ref: 1239 Objective: 40-18

8 Copyright © 2018 Pearson Education, Inc.

33) A skilled nursing facility has called you for a patient who has a problem with his intraventricular shunt. The patient is a 21-year-old man who was born with hydrocephalus and had the shunt placed shortly after birth. When assessing the patient, which sign would raise your suspicion that the shunt is occluded? A) Blood pressure of 260/126 mmHg B) Temperature of 101.4°F C) Heart rate of 140 beats/min D) Complaint of nausea Answer: A Diff: 3 Page Ref: 1239 Objective: 40-18 34) A 92-year-old female informs you that she has diabetic retinopathy. You recognize that this condition involves: A) Pressure-related damage to the optic nerve B) Lack of glucose delivery to the eye tissue C) Damage to blood vessels in the eye D) Development of a film over the interior of the eye Answer: C Diff: 3 Page Ref: 1211 Objective: 40-3 35) An 87-year-old female has called 911 for generalized weakness. When assessing her pupils, you note the right eye to be cloudy, making the assessment difficult. Based on this finding, you should be suspicious of: A) Diabetic retinopathy B) Cataracts C) Glaucoma D) Previous stroke Answer: B Diff: 2 Page Ref: 1211 Objective: 40-3 36) You are caring for a conscious 81-year-old male who complains of weakness. The patient has a hearing impairment and is wearing a hearing aid in his right ear. As you attempt to get a history, he shakes his head, claiming that he cannot hear you. What should you do immediately? A) Ensure that the hearing device is turned on B) Raise your voice and speak slowly C) Write the questions on a piece of paper D) Ask family members for a medical history Answer: A Diff: 1 Page Ref: 1212 Objective: 40-3

9 Copyright © 2018 Pearson Education, Inc.

37) A 77-year-old male, who is blind and has a guide dog, called 911 for chest discomfort. As you are preparing to transport the patient, he indicates that he wants to take his dog. How should you respond? A) Kindly tell him the dog must remain behind B) Allow the guide dog to accompany him C) Follow local protocol D) Tell him the dog can come in the ambulance but must remain outside the ED Answer: C Diff: 2 Page Ref: 1212 Objective: 40-3 38) A developmentally challenged male is biting himself and has a bleeding bite to his right hand. As you apply direct pressure to the hand, the patient continually tries to bite you, pulls his arm away, and screams. It seems only his mother can calm him down. At this time, you would: A) Place the patient on a stretcher and provide rapid transport B) Ask the patient's mother to hold direct pressure to the bite C) Advise dispatch that advanced life support is needed D) Move the patient to the stretcher for a nonemergency transport Answer: B Diff: 1 Page Ref: 1214-1215 Objective: 40-4 39) On scene, you are presented with a nonverbal 21-year-old female with multiple disfigurements from birth defects. According to her home health nurse, the patient has a fever, which requires transport to the emergency department. Your partner is familiar with the patient because he transported her last week for the same condition. When obtaining the patient's past medical history, which source should be tried first? A) Nurse B) Mother C) Patient D) Partner Answer: A Diff: 1 Page Ref: 1215 Objective: 40-3 40) You have been called to an extended care facility for a 39-year-old male with shortness of breath. On scene, you discover that the patient attempted suicide several years ago by shooting himself in the head. The patient is now belligerent and swears at you as you assess him. Which question should be asked of the nursing staff first? A) "Has he ever had shortness of breath before?" B) "Does he have a Do Not Resuscitate order?" C) "Has he been given any medications for this condition?" D) "Is this his normal mental status?" Answer: D Diff: 1 Page Ref: 1215-1216 Objective: 40-4 10 Copyright © 2018 Pearson Education, Inc.

41) You are positioning an obese patient who weighs 360 pounds on a stretcher for transport to the hospital. No other life threats exist and the vital signs are stable. Given that the patient is complaining of a headache, which transport position would be most appropriate? A) Place the patient flat, but lying on his right or left side B) Allow the patient to assume a position of comfort C) Place the patient supine, with towel rolls between his shoulders D) Have the patient sit in the back of the ambulance in the captain's chair Answer: B Diff: 1 Page Ref: 1218 Objective: 40-6 42) You have been called to a residence for an unknown medical problem. As you enter the house, which scene clue would most likely suggest the use of home medical oxygen? A) Coughing and wheezing heard from a bedroom B) Wife telling you that her husband cannot breathe C) Metered-dose inhaler of albuterol on the table D) "No smoking sign" posted on the front door Answer: D Diff: 2 Page Ref: 1227 Objective: 40-12 43) You are assessing a patient with COPD who uses an oxygen concentrator. In this case, you recognize: A) The patient cannot leave home with the concentrator B) The patient has a tracheostomy tube with oxygen attachment C) The patient requires less than 6 lpm of oxygen D) The oxygen device humidifies the O2 prior to delivery Answer: C Diff: 1 Page Ref: 1227 Objective: 40-12 44) A patient who requires ongoing home medical oxygen complains of generalized weakness. Assessment indicates a patent airway and adequate respirations. His radial pulse is strong and regular, and his skin is warm and dry. Vital signs are pulse, 92 beats/min; respirations, 18 breaths/min; blood pressure, 156/88 mmHg; and SpO2, 95%. The patient is currently on 2 lpm of oxygen via an oxygen concentrator positioned bedside. What would be appropriate care in the management of this patient? A) Continue oxygen administration at 2 lpm via nasal cannula during transport B) Place the patient on high-concentration oxygen with a nonrebreather mask, with the oxygen provided by your portable tank C) Discontinue oxygen therapy since the SpO2 is greater than 94% and initiate transport D) Administer 4 to 6 lpm oxygen via a simple face mask during transport Answer: A Diff: 2 Page Ref: 1227 Objective: 40-12 11 Copyright © 2018 Pearson Education, Inc.

45) On scene, you are by the side of a 3-week-old male who was born prematurely and has an apnea monitor on. His mother states that she was awakened by the alarm and when she went to check on the baby, he was awake and crying. Assessment reveals him to be resting quietly in his mother's arms. The airway is patent and respirations adequate. Vital signs are pulse, 136 beats/min; respirations, 28 breaths/min; and SpO2, 99% on room air. Since the local emergency department was busy when you were there dropping off a patient about an hour ago, it would be appropriate to: A) Watch the baby for 15 additional minutes and get a refusal of care if he remains stable B) Provide high-concentration oxygen and immediate transport to the hospital C) Recognize a monitor malfunction and help make arrangements for a new one D) Continue assessment of the baby, treat any problems found, and initiate transport Answer: D Diff: 3 Page Ref: 1228 Objective: 40-15 46) A 66-year-old male has summoned your assistance because he is short of breath and his home pulse oximeter is reading 89%, despite 3 lpm of home medical oxygen. Assessment reveals an intact airway and adequate breathing. The patient's radial pulse is normal, and his skin is hot and dry. Vital signs are pulse, 84 beats/min; respirations, 18 breaths/min; and blood pressure, 166/86 mmHg. Breath sounds are clear bilaterally. The patient's pulse oximeter reading is fluctuating between 88% and 90%. In this situation, you would first: A) Check the patient's SpO2 with your pulse oximeter B) Place the patient on 15 lpm of oxygen with a nonrebreather mask C) Request advanced life support assistance D) Increase oxygen flow to 6 lpm through the nasal cannula Answer: A Diff: 3 Page Ref: 1228 Objective: 40-12 47) Which definition correctly identifies a stoma? A) The inner portion of the tracheostomy tube B) The strap that secures a tracheostomy tube C) The opening in the neck for a tracheostomy tube D) The proper name for a tracheostomy tube Answer: C Diff: 1 Page Ref: 1228 Objective: 40-12 48) You are responding to a patient with a tracheostomy tube who is weak and has a fever. In this situation, you can assume: A) The patient can still speak B) The tracheostomy tube is below the larynx C) The patient is ventilator dependent D) The tracheostomy tube is permanent Answer: B Diff: 2 Page Ref: 1228 Objective: 40-12 12 Copyright © 2018 Pearson Education, Inc.

49) At an elderly extended care facility, a ventilator-dependent patient is going to be transported to the emergency department for a suspected urinary tract infection. The ventilator settings are rate, 10; tidal volume, 425; and oxygen concentration, 50%. Given this information, which statement is true? A) A volume of 4250 mL is delivered every minute B) The SpO2 is being maintained at 50% C) A breathing rate of 20 breaths every 3 minutes is being maintained D) The delivered ventilatory rate is incorrect and low Answer: A Diff: 3 Page Ref: 1232 Objective: 40-12 50) Upon your arrival on at a residential address for a patient with a previous traumatic brain injury, you are taken to the patient, who is ventilator dependent. The patient presents as unresponsive, cyanotic, and listless, and has a weak carotid pulse. Ventilator alarms are sounding. What is your immediate action? A) Determine whether the vent is a negative or positive pressure type B) Establish which type of alarm is sounding C) Disconnect the ventilator and start positive pressure ventilation D) Stop the ventilator and obtain breath sounds Answer: C Diff: 2 Page Ref: 1232 Objective: 40-12 51) A ventilator-dependent patient is being ventilated with a bag-valve mask as he is transported to the hospital. Which finding is the best indication that the patient is being adequately ventilated during transport? A) Normal skin color B) Chest rise and fall C) Rate of 12 breaths per minute D) Sustained SpO2 reading of 98% Answer: D Diff: 2 Page Ref: 1228 Objective: 40-12 52) The EMT would recognize the patient has a sensory impairment when he encounters: A) A 49-year-old male who is hearing impaired in one ear B) A 76-year-old female with headaches caused by hypertension C) A 22-year-old female who was born without a left hand D) A 43-year-old male who had a stroke and is paralyzed on his right side Answer: A Diff: 2 Page Ref: 1211 Objective: 40-3

13 Copyright © 2018 Pearson Education, Inc.

53) Which medical problem commonly leads to vision problems? A) Seizure B) Chronic obstructive pulmonary disease C) Diabetes D) Coronary artery disease Answer: C Diff: 1 Page Ref: 1211 Objective: 40-3 54) After assessing a 67-year-old male, your partner informs you that it appears the patient had a stroke and has dysarthria. In relation to the dysarthria, you would anticipate the patient being unable to: A) Swallow B) Pronounce words correctly C) Move one side of his body D) Demonstrate fine motor control of the hands and fingers Answer: B Diff: 2 Page Ref: 1212 Objective: 40-1 55) On scene, you must lead a patient's visually impaired wife from the house to the ambulance. This is best accomplished by: A) Verbally instructing her past obstacles B) Physically carrying her to the ambulance C) Placing her hand on the stretcher for guidance D) Placing her hand on your shoulder as you walk to the ambulance Answer: D Diff: 2 Page Ref: 1212 Objective: 40-3 56) The father of a developmentally challenged 45-year-old male summoned EMS because his son was "not acting right." The patient is awake, but is nonverbal, which is not typical according the father. Your assessment reveals no abnormalities, and vital signs are normal. When calling the chief complaint into the hospital, you would state that the patient: A) "Has an unknown chief complaint at this time." B) "Is awake and nonverbal." C) "Is not acting normally per his father." D) "Appears to be sick and not feeling well." Answer: C Diff: 3 Page Ref: 1215 Objective: 40-4

14 Copyright © 2018 Pearson Education, Inc.

57) A 17-year-old male with cerebral palsy and severe developmental challenges fell down five stairs and is crying. To best determine if the patient is in pain, the EMT should: A) Have the father ask the patient B) Thoroughly assess the patient for injury C) Defer this task to the emergency department D) Call for advanced life support assistance Answer: A Diff: 2 Page Ref: 1215-1216 Objective: 40-4 58) A patient with quadriplegia most likely experienced an injury to his: A) Brain B) Thoracic spine C) Arms and legs D) Cervical spine Answer: D Diff: 2 Page Ref: 1217 Objective: 40-5 59) A patient with a past history of quadriplegia from a four-wheeler accident would most likely be unable to: A) Speak clearly to others B) Hold utensils for eating C) See peripherally while driving D) Turn his head from side to side Answer: B Diff: 3 Page Ref: 1217 Objective: 40-5 60) Which statement would most likely be accurate regarding a 42-year-old female with traumatic paraplegia? A) She cannot move her arms B) She requires a walker for ambulation C) She is ventilator dependent D) She requires a wheelchair Answer: D Diff: 2 Page Ref: 1216 Objective: 40-5

15 Copyright © 2018 Pearson Education, Inc.

61) A patient with hypertension and diabetes weighs 190 pounds. His ideal weight is 160 pounds. This patient would best be described as: A) Bariatric B) Obese C) Hypertensive obese D) Morbidly obese Answer: B Diff: 3 Page Ref: 1217 Objective: 40-6 62) Poor positioning of an obese patient on the stretcher would most likely result in which immediate danger? A) Hypotension B) Skin breakdown C) Respiratory distress D) Back pain Answer: C Diff: 1 Page Ref: 1218 Objective: 40-6 63) The friend of an impoverished 50-year-old male called 911 because his friend had chest discomfort. On scene, the patient informs you that he does have a cardiac history and has not seen his cardiologist in more than a year because he lost his job. The patient does not want treatment or transport since he knows that both are expensive and he will be unable to pay for such care. In this situation, what would your best response be? A) "In this situation, it is your right to refuse care and I respect that." B) "You may be eligible for free or reduced-cost medical care." C) "I will make sure that you do not get a bill for this." D) "You can refuse, but at least follow up with your cardiologist." Answer: B Diff: 2 Page Ref: 1220 Objective: 40-7 64) Hospice care indicates that a patient is: A) Terminally ill and foregoing any additional care until death B) Terminally ill and receiving measures to increase comfort until death C) Terminally ill and but receiving care to reverse the disease process so as to avoid death D) Terminally ill but without a Do Not Resuscitate order Answer: B Diff: 2 Page Ref: 1240 Objective: 40-19

16 Copyright © 2018 Pearson Education, Inc.

65) A terminally ill female with ovarian cancer is receiving palliative care when she: A) Has a morphine pump implanted in her abdomen B) Is to receive surgery to remove her ovaries C) Is undergoing rigorous chemotherapy and radiation therapy D) Completes and signs a Do Not Resuscitate order Answer: A Diff: 3 Page Ref: 1240 Objective: 40-1 66) The family of a patient in hospice care for the final stage of lung cancer has summoned 911 because their father is short of breath. The patient is responsive to verbal stimuli and has an open airway with adequate respiration of 18 to 20 breaths per minute. His skin is cool and diaphoretic with diminished breath sounds bilaterally. Your partner reports a heart rate of 112 beats/min, blood pressure of 96/44 mmHg, and SpO2 of 82% on room air. The patient has a valid DNR order, but family is scared and wants him transported to the ED for evaluation. Your next action would be to: A) Monitor the patient during transport but avoid providing physical care B) Place an oral airway and start positive pressure ventilation C) Provide supplemental oxygen, monitor vital signs, and transport the patient to the ED D) Place the patient in a position of comfort, and reassess the patient en route while providing emotional support Answer: C Diff: 3 Page Ref: 1241 Objective: 40-19 67) Because of the improvements in life span and medical technology: A) Fewer people are now reliant on home medical technology than in the past B) People with certain deficits or medical conditions can remain at home C) The EMT seldom encounters home medical technology problems when providing emergency care D) The EMT seldom encounters terminally ill patients in the home Answer: B Diff: 1 Page Ref: 1210 Objective: 40-2 68) With the current trend of providing medical technologies in the home, the EMT: A) Is often the first resource called if a failure occurs B) Typically does not respond prior to the equipment technician C) Is always required to rely on the patient for operational guidelines D) Must be certified by all manufacturers of home-based equipment Answer: A Diff: 1 Page Ref: 1210 Objective: 40-2

17 Copyright © 2018 Pearson Education, Inc.

69) How important is it to make accommodations such as advocating for the impoverished when providing medical care? A) Very important, because many Americans will live below the poverty level for at least a year B) Not very important, because impoverished or homeless people rarely require any special accommodation for care C) Patient advocacy is not required when dealing with the homeless D) Not very important, because people lower on the economic ladder are less likely to suffer illness Answer: A Diff: 2 Page Ref: 1220 Objective: 40-2 70) You are called to a residence for a 27-year-old woman who fell and hurt her arm. As you begin your assessment, you begin to realize that there may be abuse involved. Suddenly the husband decides that he will transport the woman and you are no longer needed. You are unable to change the husband's mind and he is becoming more agitated. What should you do next? A) Call medical command for guidance B) Ask the patient if she is safe C) Continue to persuade the patient to leave with you D) Retreat to the ambulance and notify law enforcement Answer: D Diff: 2 Page Ref: 1224 Objective: 40-8 71) It is common for victims of human trafficking to: A) Attempt to leave the scene with you B) Display an attitude of independence C) Display an upbeat attitude D) Display feelings that the trafficker is a loving partner Answer: D Diff: 2 Page Ref: 1222 Objective: 40-9 72) At the scene of a fall in a mall parking lot, you begin to suspect that the victim, who received only minor injuries from the fall and refuses transport, is a victim of human trafficking. What should you do next? A) Call the Human Trafficking Hotline B) Call medical command for guidance C) Have the patient sign a refusal-of-care form D) Provide the victim with a shelter's contact information Answer: A Diff: 2 Page Ref: 1222 Objective: 40-9

18 Copyright © 2018 Pearson Education, Inc.

73) In the cycle of domestic violence, in which phase does the violence occur on a regular basis? A) Phase I B) Phase II C) Phase III D) Phase IV Answer: B Diff: 2 Page Ref: 1223 Objective: 40-10 74) Which statement concerning a ventricular assist device is accurate? A) The EMT's SpO2 reading will be accurate in patients with VADs B) An implanted defibrillator is not necessary with a VAD C) You will likely feel no pulse and cannot obtain a blood pressure in patients with VADs D) The most common reason for EMS response related to a VAD is a cardiac event Answer: C Diff: 3 Page Ref: 1234 Objective: 40-14 75) What is the most common reason for an EMS response to a patient with an implanted VAD? A) Myocardial infarction B) Implanted defibrillator discharge C) Infection D) Battery replacement Answer: C Diff: 3 Page Ref: 1234 Objective: 40-14 76) What is a vagus nerve stimulator used for? A) To prevent seizure activity B) To reduce incontinence C) To reduce the incidence of supraventricular tachycardia D) To treat Bell's palsy Answer: A Diff: 2 Page Ref: 1235 Objective: 40-15

19 Copyright © 2018 Pearson Education, Inc.

Prehospital Emergency Care, 11e (Mistovich et al.) Chapter 41 The Combat Veteran 1) You have been dispatched to a Veterans Administration (VA) clinic for a male patient with a behavioral emergency. On scene, the VA physician tells you that the patient is a returning veteran with newly diagnosed post-traumatic stress disorder (PTSD). Based on this information, you would surmise: A) the veteran was involved in active combat while in the service B) the veteran may have been in the military but not necessarily involved in combat C) the veteran most likely had a prior history of mental health issues D) the veteran has sustained physical injury to his brain Answer: B Diff: 3 Page Ref: 1246 Objective: 41-3 2) As an EMT, you realize that post-traumatic stress disorder (PTSD) occurs when: A) The brain is emotionally injured by an external force B) The returning veteran perceives a lack of respect by society or the government C) An individual is exposed to an abnormal or dangerous condition that is difficult to mentally handle D) An individual is overwhelmed by the demands of daily life upon returning from military service Answer: C Diff: 2 Page Ref: 1245 Objective: 41-4 3) When faced with danger, which component of the nervous system is immediately activated for basic survival? A) Sympathetic B) Central C) Voluntary D) Parasympathetic Answer: A Diff: 2 Page Ref: 1246 Objective: 41-2 4) A soldier patrolling the streets in a foreign country is suddenly fired on by a sniper. Which response would you expect to occur within his body? A) The heart rate slows B) The blood pressure drops C) The body temperature drops D) The pupils dilate Answer: D Diff: 2 Page Ref: 1246 Objective: 41-2

1 Copyright © 2018 Pearson Education, Inc.

5) The EMT would recognize an era veteran when a family member states: A) "He has spent his whole life in the military." B) "He served three tours in Vietnam and the Middle East." C) "He was shot in the leg by a sniper in Iraq." D) "He trained for combat but was never deployed." Answer: D Diff: 1 Page Ref: 1246 Objective: 41-3 6) During scene size-up on an unknown medical call, which clue would best indicate that the patient may be a veteran of the military? A) Psychiatric medications on nightstand B) Military-style haircut C) Vintage guns on display in the living room D) Collection of war movie DVDs Answer: B Diff: 1 Page Ref: 1246 Objective: 41-3 7) You are assessing a military veteran who complains of a headache, visual disturbances, and generalized anxiety. You suspect that he may have post-traumatic stress disorder (PTSD). Which question would be most appropriate to obtain additional information regarding the possibility of PTSD? A) "Were you forced to kill anyone?" B) "In which branch of the military did you serve?" C) "Where did you see combat?" D) "Do you go to the VA hospital for any medical care you need?" Answer: C Diff: 3 Page Ref: 1246 Objective: 41-5 8) Which statement is true regarding post-traumatic stress disorder? A) There is no known or universal cure for post-traumatic stress disorder B) All military veterans will eventually become debilitated from PTSD C) The symptoms of PTSD are evident within 6 months of an event D) PTSD can be cured with timely intervention and medication use Answer: A Diff: 2 Page Ref: 1246 Objective: 41-8

2 Copyright © 2018 Pearson Education, Inc.

9) One of the four essential features of post-traumatic stress disorder is: A) Chronic confusion B) Unexplained attraction to war memorabilia C) Feelings of anger D) Migration to crowds for security Answer: C Diff: 2 Page Ref: 1248 Objective: 41-4 10) In a patient with post-traumatic stress disorder (PTSD), the EMT would recognize that a patient is reliving an event when he states: A) "I do not like the smell of diesel fuel because of the airplane crash." B) "I feel so guilty that there was nothing I could do to save them." C) "I notice that my heart rate will sometimes increase for no reason." D) "I cannot sleep well because of nightmares that I am back in the desert." Answer: D Diff: 2 Page Ref: 1248 Objective: 41-5 11) The EMT would recognize which response when a combat veteran patient states, "I still see them burning but cannot do anything about it"? A) Reliving B) Response C) Avoiding D) Anxiety Answer: A Diff: 2 Page Ref: 1248 Objective: 41-5 12) A combat veteran patient with a history of severe PTSD states that he has a headache that feels worse than any other he has ever had. The patient is alert and oriented, with no threats to the airway, breathing, or circulation. His vital signs are pulse, 78 beats/min; respirations, 16 breaths/min; blood pressure, 168/100 mmHg; and SpO2, 95% on room air. What would your next action be? A) Recognize the component of anguish B) Perform a neurologic assessment C) Treat the patient for PTSD and transport D) Administer low-concentration oxygen Answer: B Diff: 2 Page Ref: 1251 Objective: 41-8

3 Copyright © 2018 Pearson Education, Inc.

13) The wife of a military veteran with PTSD states that her husband developed type 2 diabetes 10 years after returning from combat. There is no history of this disease in either side of the family, and the patient is not overweight and does not have any other risk factors for the disease. Consequently, the wife believes the diabetes results from her husband's experience in the military. How would you best respond to her assertion? A) "The diabetes is most likely a result of undiagnosed traumatic brain injury, not the PTSD." B) "It is not uncommon for combat veterans to unexpectedly develop such conditions like this." C) "I think we have to suspect that he sustained an injury to his pancreas at some point." D) "Although it can happen, era veterans–rather than not combat veterans–more typically develop diabetes." Answer: B Diff: 3 Page Ref: 1248 Objective: 41-5 14) The EMT recognizes that alcohol or drug dependence often occurs in individuals with posttraumatic stress disorder (PTSD) because: A) The alcohol or drug is used to blunt the sympathetic nervous system and emotional mental anguish B) The medications that treat PTSD make the individual more prone to alcohol and drug abuse C) The alcohol or drug dependence reflects an individual weakness, since not everyone with PTSD will have substance abuse issues D) The alcohol or drug serves a therapeutic purpose by "slowing down" the brain and allowing mental processing of past experiences Answer: A Diff: 3 Page Ref: 1248 Objective: 41-5 15) When approaching a combat veteran who is highly agitated, the EMT must remember that: A) Combat veterans are more dangerous than civilians because of their training B) Identification of the event that led to the PTSD is essential C) Combat veterans are more likely to assault caregivers and medical providers D) Reassurance with a firm, calm voice may be necessary and should be attempted first Answer: D Diff: 2 Page Ref: 1251 Objective: 41-8 16) When caring for a 34-year-old male with a history of post-traumatic stress disorder (PTSD), which instruction should you provide to your partner? A) "We must remove family members from around the patient." B) "We need to determine whether the patient feels suicidal or homicidal." C) "We need to determine whether the patient has ever killed anyone." D) "We need to determine whether the patient remembers the event that caused the PTSD." Answer: B Diff: 2 Page Ref: 1249 Objective: 41-8

4 Copyright © 2018 Pearson Education, Inc.

17) You arrived on scene at a local restaurant, where you encounter a highly agitated combat veteran who is yelling at patrons. Given the public nature of the scene and the patient's known history of post-traumatic stress disorder (PTSD), which statement indicates appropriate initial management of the situation? A) "Let me see if I can establish a rapport and calm him." B) "We need to contact ALS for chemical restraints." C) "This is more a police matter so we need to contact them." D) "Let's determine if the patient has a weapon before we attempt to physically restrain him." Answer: A Diff: 1 Page Ref: 1251 Objective: 41-8 18) Regarding the risk of suicide for combat veterans, which statement is true? A) Prompt removal of guns will typically prevent a veteran from committing suicide B) There is often no preventing suicide once this course of action is decided C) Most veterans do not tell anyone about their thoughts of suicide D) Combat veterans with PTSD typically do not attempt suicide Answer: B Diff: 3 Page Ref: 1249 Objective: 41-2 19) The signature wound of recent combat missions in the Middle East is typically caused by: A) High-powered guns B) Post-traumatic stress disorder C) Aerial bombings D) Improvised explosive devices Answer: D Diff: 1 Page Ref: 1250 Objective: 41-6 20) A traumatic brain injury (TBI) would most likely be caused by: A) Witnessing fellow soldiers dying B) Accidentally shooting an unarmed civilian C) Nearby explosion of a concussion grenade D) Disappearance of best friend in platoon Answer: C Diff: 2 Page Ref: 1250 Objective: 41-6

5 Copyright © 2018 Pearson Education, Inc.

21) The EMT would suspect that a patient has a traumatic brain injury (TBI), as opposed to posttraumatic stress disorder (PTSD), when he observes: A) Relationship problems B) Confusion and disorientation C) Anxiety and anger D) Left arm amputation Answer: B Diff: 1 Page Ref: 1250 Objective: 41-7 22) A combat veteran's wife called 911 after her husband suddenly became confused and agitated. According to the wife, the patient takes several mood-stabilizing drugs and has a history of traumatic brain injury (TBI) from an improvised explosive device (IED) while on tour in the Middle East. There is no other medical history. On scene, in the patient's bedroom, the patient is staring at you with a face that appears angry. What is your initial priority in the care of this patient? A) Recognize post-traumatic stress disorder and check a blood sugar B) Gather and identify the medications taken by the patient C) Recognize post-traumatic stress disorder and prepare for immediate transport D) Establish a rapport with the patient and ensure personal safety Answer: D Diff: 1 Page Ref: 1251 Objective: 41-8 23) A wife of a returning combat veteran called EMS because her husband is complaining of constant headaches. When you ask about post-traumatic stress disorder (PTSD) or traumatic brain injury (TBI), she states that the patient was injured by an improvised explosive device in Iraq, but cannot remember the event. She explains that the patient has undergone numerous CTs and MRIs of head and has been told by his family doctor that there is no sign of injury. Regarding these medical conditions, you realize that: A) TBI may exist despite the negative findings of the CT and MRI tests B) PTSD is likely and requires immediate identification and treatment for recovery C) TBI and PTSD most likely do not exist since the patient does not have any memory of the event D) TBI and PTSD are not likely since the patient does not have associated symptoms such as dizziness or nausea Answer: A Diff: 2 Page Ref: 1251 Objective: 41-1

6 Copyright © 2018 Pearson Education, Inc.

24) An alteration in brain function or other brain pathology caused by an external force that causes structural brain damage is described as: A) TBI B) PTSD C) A concussion D) An organic brain disease Answer: A Diff: 1 Page Ref: 1250 Objective: 41-1 25) Physical signs and symptoms such as severe neck pain, clumsiness, and trouble swallowing can be indicative of: A) TBI B) PTSD C) Alzheimer's disease D) Paralysis Answer: A Diff: 2 Page Ref: 1250 Objective: 41-7

7 Copyright © 2018 Pearson Education, Inc.

Prehospital Emergency Care, 11e (Mistovich et al.) Chapter 42 Ambulance Operations and Air Medical Response 1) When an EMT is driving an ambulance in emergency mode (lights and sirens), most states do not permit the EMT to: A) Proceed through a red light without stopping B) Drive in the wrong direction down a one-way street C) Turn in any direction at an intersection D) Pass other vehicles in a no-passing zone Answer: A Diff: 1 Page Ref: 1259 Objective: 42-3 2) The EMT shows that she understands the concept of driving with due regard when she states: A) "When responding to an emergency, I drive as fast as possible so that emergency care can be delivered as quickly as possible." B) "When taking a patient to the hospital, I drive as quickly as possible so that I can get back into service for another call." C) "When approaching an intersection, I wait for all traffic to stop before proceeding through." D) "It is acceptable to take a nonemergency patient to the hospital using lights and siren if dispatch informs me that there is another call waiting." Answer: C Diff: 2 Page Ref: 1261 Objective: 42-3 3) An ambulance, while responding to a call using lights and sirens, is involved in a collision at an intersection. Witnesses state that the ambulance slowly went through the red light without first stopping, resulting in a collision with a car. No one is injured, and the driver of the car is insured. Based on the witnesses' accounts of the collision, which outcome is likely to occur? A) The driver of the car is liable for the collision B) The EMT is immune from liability for the collision C) Liability for the collision cannot be assigned since no one was injured D) The EMT driving the ambulance may be liable for the collision Answer: D Diff: 2 Page Ref: 1259 Objective: 42-3 4) You are driving an ambulance with antilock brakes. A deer darts out in front of the ambulance. Your most appropriate action would be to: A) Swerve into the opposite lane B) Apply the brakes for 3 seconds, and then release C) Gently pump the brakes D) Apply constant and firm pressure to the brakes Answer: D Diff: 2 Page Ref: 1260 Objective: 42-4 1 Copyright © 2018 Pearson Education, Inc.

5) You are instructing a group of new EMTs on the basics of driving an emergency vehicle. What would you emphasize as the most frequent situation or location in which collisions involving ambulances occur? A) Parking lots B) Backing up C) Intersections D) Emergency scenes Answer: C Diff: 1 Page Ref: 1261 Objective: 42-4 6) Which statement about the use of an escort, such as a police cruiser, to drive a critical patient to the hospital is true? A) "It is actually safer, because traffic is cleared ahead of the ambulance." B) "It should be used as a last resort, because it increases the risk of an accident." C) "It is a safe way to get a critical patient to the hospital quickly and should be used when needed." D) "It is safer if the ambulance transporting the patient goes first and is followed by the second emergency vehicle." Answer: B Diff: 2 Page Ref: 1261 Objective: 42-5 7) The EMT recognizes that preventive mechanical maintenance performed regularly on the ambulance results in: A) Cleaner ambulances B) Fully stocked ambulances C) More knowledgeable EMTs D) Safer patient transports Answer: D Diff: 1 Page Ref: 1265 Objective: 42-4 8) Since supplies and materials used to treat the sick or injured may be used on every call, it is good practice to replace or restock the ambulance: A) After every call B) Daily and weekly C) At the beginning of a shift D) Weekly and monthly Answer: A Diff: 1 Page Ref: 1266 Objective: 42-7

2 Copyright © 2018 Pearson Education, Inc.

9) In an EMS system, the first assessment of the call and the patient's condition is most often made by the: A) Emergency Medical Responder B) Emergency Medical Dispatcher C) Emergency Physician D) Emergency Medical Technician Answer: B Diff: 1 Page Ref: 1266-1267 Objective: 42-7 10) When an EMS unit is alerted to an emergency call from dispatch, which piece of information is it most essential that the EMT obtain first? A) Location of the call B) Patient's chief complaint C) Number of patients D) Specific conditions or problems Answer: A Diff: 2 Page Ref: 1266 Objective: 42-7 11) While cleaning the ambulance following an emergency call, you find a syringe and a needle that were used by the paramedic to give a patient an injection. What is the proper way to dispose of used syringes and needles? A) Recap the needle, and dispose of it in the garbage B) Remove the needle from the syringe, and dispose of it in the trash C) Bend the needle, and then dispose of it in a special garbage bag at the hospital D) Place the syringe and needle in a special puncture-resistant container Answer: D Diff: 2 Page Ref: 1271 Objective: 42-8 12) A paramedic has asked you to sterilize a laryngoscope blade that was placed in the mouth of a patient during an endotracheal intubation procedure. To sterilize this piece of equipment, you should: A) Wipe the blade down with a solution of 1 part bleach to 10 parts water B) Clean the blade with soap and water, and then soak it in bleach for 1 hour C) Immerse and leave the blade in an EPA-registered and approved chemical sterilant D) Wipe and soak the blade in a solution of 100 parts bleach to 1 part water Answer: C Diff: 2 Page Ref: 1272 Objective: 42-8

3 Copyright © 2018 Pearson Education, Inc.

13) For routine disinfection of a stethoscope after each call, the EMT should: A) Clean it thoroughly with soap and water B) Spray it with a solution of 1 part peroxide and 100 parts bleach C) Soak it for 1 hour in a solution of hot water and bleach D) Wipe it with a solution of 1 part water and 10 parts bleach. Answer: D Diff: 2 Page Ref: 1271 Objective: 42-8 14) You walk into the station at the beginning of your shift and see another EMT laundering sheets and pillowcases used from the previous shift. Which observation should cause you to stop and correct the EMT in relation to this task? A) He uses exam gloves to transfer the laundry into the washer B) He is using an EPA-registered sterilant as the detergent C) He is using a regular laundry detergent in the washing machine D) He handles the dirty laundry as little as possible Answer: B Diff: 3 Page Ref: 1272 Objective: 42-8 15) A 43-year-old male is entrapped in a car that struck a tree at a high rate of speed. He suffered blunt trauma to the chest and has a decreased level of consciousness. In this situation, what is the best reason to transfer the patient by helicopter to the hospital? A) A family member on scene requests it B) The car's driver died C) Prolonged extrication was necessary D) The helicopter base is only 1 mile away Answer: C Diff: 3 Page Ref: 1272 Objective: 42-9 16) Which action is most appropriate when using a helicopter on the scene of an emergency? A) Always approach the helicopter from the front, never from behind B) Point a spotlight at the helicopter to indicate the location of the landing zone when the helicopter is on final approach C) Always approach the helicopter from behind, never from the front D) Keep all EMS and rescuer personnel at least 50 feet from the helicopter as it lands Answer: A Diff: 2 Page Ref: 1275 Objective: 42-9

4 Copyright © 2018 Pearson Education, Inc.

17) Yours is the third ambulance to arrive at the scene of a nighttime motor vehicle collision on a busy road. As you approach the scene, which observation should cause you to alert the incident commander of a problem? A) One ambulance is acting as a physical barrier between the wrecked car and oncoming traffic B) Traffic cones and an EMT are being used to divert and monitor traffic C) One ambulance has been positioned with its lights facing oncoming traffic D) The closest ambulance is parked 200 feet from the wrecked car Answer: C Diff: 2 Page Ref: 1268; Figure 42-4 Objective: 42-6 18) An EMT asks you how he can decrease the chances of being involved in a collision when driving with lights and siren at night. Your advice would be: A) "It is safer to use your high beams with the lights and siren, so that oncoming cars can see you better." B) "If blinded by the lights of an oncoming car, look to the right side of the road and not at the car." C) "Flashing your high beams on and off when approaching another vehicle is a good way to alert him to your presence." D) "Keep all lights on your dashboard to the highest setting so that you can easily see the speedometer to monitor your speed." Answer: B Diff: 2 Page Ref: 1262 Objective: 42-5 19) You are responding to a possible pediatric overdose. It is raining and suddenly the ambulance starts to hydroplane. Your immediate action should be to: A) Firmly apply the brakes B) Turn the steering wheel from side to side to get down through the water C) Increase speed, and then quickly apply the brakes D) Remove pressure from the accelerator pedal Answer: D Diff: 2 Page Ref: 1261 Objective: 42-4 20) Which option best illustrates a basic medical supply that must be carried on an ambulance? A) Written protocols B) Street maps C) Exam gloves D) Self-contained breathing apparatus Answer: C Diff: 1 Page Ref: 1266; Table 42-2 Objective: 42-7

5 Copyright © 2018 Pearson Education, Inc.

21) You have been dispatched for a 2-year-old female who is not breathing. When responding, you realize that your primary goal is to: A) Mentally prepare for what might be required on scene B) Arrive at the scene safely C) Review the protocol for pediatric cardiac arrest D) Get to the scene as quickly as possible Answer: B Diff: 1 Page Ref: 1259 Objective: 42-3 22) While responding to an emergency call, what should you do if you encounter a school bus unloading students with its red lights flashing? A) Slow down and carefully proceed past the school bus B) Continue responding as you were, since you have the right of way C) Stop and wait for all children to cross the road safely D) Stop and determine an alternative route to the emergency Answer: C Diff: 1 Page Ref: 1260 Objective: 42-3 23) As an instructor for safe ambulance operations, which statement should you make to new EMTs about responding with lights and sirens through curves in the road? A) "At night, turn your high beams off prior to entering the curve." B) "It is best to start braking after entering the curve." C) "To be safe, maintain a steady speed through the entire curve." D) "It is easier to control the ambulance when you accelerate just before entering the curve." Answer: A Diff: 2 Page Ref: 1262 Objective: 42-4 24) Which statement is true regarding driving in inclement weather? A) Roads are most hazardous in terms of traction near the end of a rainstorm B) It is best to accelerate when attempting to drive through a large standing puddle C) Roadways typically freeze over sooner than do bridges and overpasses D) Stopping on wet pavement takes approximately twice the distance as stopping on dry pavement Answer: D Diff: 1 Page Ref: 1262 Objective: 42-4

6 Copyright © 2018 Pearson Education, Inc.

25) Which action would reduce the chance of being involved in an emergency response-related collision with the ambulance? A) Unless it is dark out, the headlights should be left off when the red lights and sirens are being used B) During a nonemergency transport, it is important to use the red lights without a siren to make other motorists aware of your presence C) Sirens should be used only when approaching an intersection to warn drivers to stop D) White strobe lights should be used in conjunction with the red flashing lights Answer: D Diff: 1 Page Ref: 1263 Objective: 42-5 26) In terms of high-visibility ANSI vests, which statement is true? A) They help other rescuers identify EMTs as medical personnel B) They provide protection against blood and other body fluids at a trauma scene C) They make EMS personnel more visible to passing motorists D) They provide protection against bent metal and broken glass at an accident scene Answer: C Diff: 1 Page Ref: 1264 Objective: 42-6 27) The use of Class 2 or 3 ANSI high-visibility vests when operating on a roadway is a requirement of: A) Federal law B) The state's EMS departments C) The EMS system's medical director D) The state's department of highways Answer: A Diff: 1 Page Ref: 1264 Objective: 42-6 28) What would cause you to start transporting a critically injured patient to a local hospital (not equipped for major trauma) instead of waiting on a helicopter for rapid transport to a trauma center 50 miles away? A) A sudden drop in patient's pulse oximeter reading while you are providing supplemental oxygen B) The beginning of lightning and poor weather conditions C) The patient's statement that he cannot afford the cost of a trauma center D) A request by the family to go to the local hospital Answer: B Diff: 2 Page Ref: 1273 Objective: 42-9

7 Copyright © 2018 Pearson Education, Inc.

29) You are on the scene of a motor vehicle collision and must look for an area to land a medical helicopter. Which option would be the best choice? A) A flat, sloped square area 40 × 40 feet free of debris B) A small, flat, concrete lot with light posts on all sides C) A level and well-maintained parking lot that remains half-filled with cars D) An open field that is level and preferably 150 feet from the accident Answer: D Diff: 2 Page Ref: 1273 Objective: 42-9 30) A helicopter has been requested at the scene of a motor vehicle collision at night where two people have been critically injured. As safety officer overseeing the incident, which action would cause you to immediately intervene for the safety of all rescuers, bystanders, and patients? A) Soaking down the landing site using water from a fire hose B) Stopping traffic in both lanes of divided highway as the aircraft lands C) Guiding the helicopter to the landing site using a bright spotlight D) Positioning rescuers in front of the aircraft instead of behind it Answer: C Diff: 2 Page Ref: 1273 Objective: 42-9 31) Which organization developed and published "Strategy for a National EMS Culture of Safety"? A) NHTSA B) NAEMS C) NEMSAC D) NTSHA Answer: A Diff: 1 Page Ref: 1256 Objective: 42-2 32) The process of effectively using all resources available to reduce errors, increase safety, and improve performance is called: A) Culture of safety B) Resource conservation C) System status management D) Crew Resource Management (CRM) Answer: D Diff: 1 Page Ref: 1257 Objective: 42-2

8 Copyright © 2018 Pearson Education, Inc.

33) Which organization(s) published the recommended guidelines for securing and tracking vehicle access in the ambulance industry? A) Fire services B) NHTSA C) NEMSAC D) NAEMT Answer: D Diff: 2 Page Ref: 1275 Objective: 42-10 34) According to the recommended guidelines for securing and tracking vehicle access in the ambulance industry, how often should security briefings be held? A) Whenever there is a change in the Homeland Security threat level B) At the beginning of each shift C) No less than once per week D) Daily Answer: A Diff: 1 Page Ref: 1275 Objective: 42-10 35) How can you help prevent the entrance of carbon monoxide from the ambulance exhaust into the unit? A) Extend the exhaust and leave the rear windows closed B) Keep operating generators away from the ambulance C) Do not operate the unit in heavy traffic D) Use exhaust fans Answer: A Diff: 2 Page Ref: 1276 Objective: 42-11 36) What is the risk of operating an ambulance in a fashion that creates a negative cabin pressure? A) Increased oxygen content in the air within the ambulance in the event of a leaking on-board oxygen tank B) Inability to properly exhaust CO2 C) Buildup of CO inside the unit D) Inefficient operation of the exhaust fan Answer: C Diff: 2 Page Ref: 1276 Objective: 42-11

9 Copyright © 2018 Pearson Education, Inc.

Prehospital Emergency Care, 11e (Mistovich et al.) Chapter 43 Gaining Access and Patient Extrication 1) On the scene of a motor vehicle collision, which EMT is engaged in simple extrication of the patient? A) The EMT removing the patient from the car through an open door B) The EMT entering the car through a window to open the patient's airway C) The EMT stabilizing a car on its side to prevent it from rolling over D) The EMT monitoring the scene for safety hazards Answer: A Diff: 1 Page Ref: 1290 Objective: 43-1 2) A roofer on top of a two-story house called 911 after experiencing chest pain and dizziness. You are providing care to the patient on top of the roof, and the fire department has arrived to help remove the patient from his present location. As the patient is brought down the ladder, your highest priority as an EMT is: A) Safety of the patient B) Providing patient care C) Your personal safety D) Safety of the other rescuers Answer: C Diff: 1 Page Ref: 1285 Objective: 43-6 3) You have just arrived on the scene of a motor vehicle collision where a car rolled several times at a high rate of speed. The car is on its wheels, with the restrained driver still in his seat. After ensuring that the scene is safe and gaining access to the patient, what should you do immediately if the patient is found unresponsive? A) Establish a plan to extricate the patient B) Remove pieces of broken glass from around the patient C) Cover the patient and force the door open D) Provide manual spine motion restriction Answer: D Diff: 1 Page Ref: 1289 Objective: 43-6 4) Rescue workers are preparing to remove the windshield and roof from a car to extricate a patient who is trapped. Before the rescue commences, what should the EMT do first, prior to the other actions? A) Cover the patient with a heavy tarp for protection B) Explain to the patient what is about to take place C) Remove himself from the vehicle so that the EMT is not accidentally injured D) Wet the car down with a fire hose to prevent an accidental fire Answer: B Diff: 2 Page Ref: 1291 Objective: 43-6 1 Copyright © 2018 Pearson Education, Inc.

5) Which situation represents simple access to a patient? A) Unlocking a car door by breaking the window using a screwdriver B) Forcing entry into a house by breaking down a locked door C) Climbing stairs to gain access to a patient on an upstairs patio floor D) Extricating a patient from a car after removing one door Answer: C Diff: 2 Page Ref: 1290 Objective: 43-4 6) To gain access to a patient entrapped in a car, the rescuers must use the "jaws of life" to open the door. The EMT should recognize this situation as: A) Complex access B) Rapid extrication C) Simple access D) Hydraulic stabilization Answer: A Diff: 1 Page Ref: 1290 Objective: 43-1 7) You are on the scene of a motor vehicle collision involving two cars. To remove the driver of the first car, rescuers must break the windshield and roll back the roof. While assisting in this operation, which pieces of personal protective equipment will afford you the best protection? A) Sunglasses, steel-toed boots, and fluorescent identification vest B) Latex exam gloves, fluorescent identification vest, and helmet C) Helmet, goggles, turnout gear, leather gloves, and steel-toed boots D) Goggles, latex exam gloves, steel-toed boots, and fluorescent vest Answer: C Diff: 1 Page Ref: 1285 Objective: 43-6 8) You have been dispatched to a motor vehicle collision. On scene, you find that a minivan has completely sheared off a wooden utility pole. The electrical wires are draped over the hood of the minivan and the driver, a woman in her 20s, is screaming that she is not hurt and wants to get out of the vehicle. The wires are not moving, arcing, or making any noise. Which instruction is most appropriate at this time? A) "I need you to jump clear of the car and shuffle away in small steps, keeping your feet together." B) "You need to remain still and calm in your seat; do not touch anything in the car." C) "Carefully get out of the car and shuffle away with small steps, while keeping your feet together." D) "The electrical wires are not charged anymore, but stay where you are until the fire department gets here." Answer: B Diff: 2 Page Ref: 1285 Objective: 43-8

2 Copyright © 2018 Pearson Education, Inc.

9) At the scene of a motor vehicle collision, a driver is entrapped in his car after striking the back of a semi tractor-trailer at a high rate of speed. Which action would be performed first when providing stabilization of the car? A) Set the parking brake B) Disconnect the battery C) Shut off the engine D) Deflate all four tires Answer: C Diff: 2 Page Ref: 1288 Objective: 43-7 10) An elderly patient has fallen in her home and activated the 911 system by pressing a medical alarm worn around her neck. On arrival, you can see the patient on the floor, but she cannot come to the door to open it and let you in. In this situation, your best course of action is to: A) Use a crowbar to pry open a window or door B) Contact a locksmith to pick the locks on the front door C) Open the storm door in the front and kick open the front door D) Break a window in a room away from where the patient is located Answer: D Diff: 2 Page Ref: 1289 Objective: 43-4 11) The rescue captain has asked that you disconnect a car's battery to shut down all power to the vehicle completely at the scene of a one-car MVC. How should you do this safely? A) Remove the negative battery cable first B) Cut the positive battery cable, and then remove the negative cable C) Disconnect the positive battery cable first D) Remove the negative and positive cables simultaneously Answer: A Diff: 2 Page Ref: 1293 Objective: 43-7 12) On the scene of a motor vehicle collision, the driver is sitting inside the car with all windows up. When you attempt to open the doors, you find that they are locked. Given this situation, which statement made to the patient is most appropriate? A) "Very carefully, with as little movement as possible, please roll down the window." B) "Without moving your head or neck, please try to unlock the door." C) "Please remain still. I am calling for the rescue truck to remove the door of your car." D) "Please shield your eyes. I am going to break the window so we can open the door." Answer: B Diff: 2 Page Ref: 1290 Objective: 43-5

3 Copyright © 2018 Pearson Education, Inc.

13) An unresponsive patient is in a car with the doors locked and the windows up. Which action is most appropriate when attempting to gain access to the patient? A) Break the window opposite the patient with a spring-loaded punch B) Use a crowbar to break the lock on the door C) Call for a rescue truck to forcibly remove a door D) Break the window by striking it with a crowbar Answer: A Diff: 2 Page Ref: 1290 Objective: 43-8 14) You are on scene of a serious motor vehicle collision involving multiple cars on a two-lane county road. As the EMT in charge, what is the most appropriate instruction to give other rescuers on scene? A) "Let's make sure to put cones around the cars." B) "We need to route passing traffic no more than 20 feet around the scene." C) "Park the ambulance downhill to make movement of the patient easier." D) "Let's stop all traffic and reroute it to alternative roadways." Answer: D Diff: 1 Page Ref: 1286 Objective: 43-3 15) You are on the scene of a car that hit a tree head-on. The patient has a laceration to his forehead and a deformity to his left forearm. The car is a hybrid that runs on traditional fuel as well as hydrogen gas. The vehicle has moderate front-end damage, but the air bags did not deploy. The patient was wearing a seat belt. Which instruction is most appropriate in this situation to give to the arriving fire department personnel? A) "Place a flare near the oil spill so that no one slips." B) "I need to have the battery cables disconnected." C) "Let's deactivate the air bags by turning the engine off." D) "We will need to puncture the hydrogen tank to prevent an explosion." Answer: B Diff: 2 Page Ref: 1288 Objective: 43-7 16) A car has struck a utility pole at a high rate of speed. The male driver appears to be unresponsive and has snoring respirations. The vehicle has just been stabilized, and you have donned the appropriate personal protective equipment. What should you do next? A) Perform a head-tilt, chin-lift airway maneuver B) Place a tarp over some spilled antifreeze C) Take manual spine motion restriction D) Apply a cervical collar to the patient's neck Answer: C Diff: 2 Page Ref: 1289 Objective: 43-3

4 Copyright © 2018 Pearson Education, Inc.

17) A vehicle has been struck in its front end by another car. The driver is entrapped, and the roof must be rolled back to allow extrication. The car has been appropriately stabilized and you are inside with the patient. The primary assessment is complete and a cervical collar applied. Rescue personnel are ready to begin cutting the vehicle. At this time, the appropriate action is to: A) Place a tarp over the patient B) Remove the windshield C) Apply a short vest-type extrication device to the patient D) Stabilize the vehicle Answer: A Diff: 2 Page Ref: 1290 Objective: 43-8 18) A midsize car has been struck by a small delivery truck and sustained moderate damage to the front passenger area. The car is on its tires, and wood chocks have been placed under the undercarriage of the car. The air bags did not deploy. What is the next step in this rescue? A) Carefully deploy the air bags B) Extricate the patient C) Forcibly open the trunk D) Release air from the tires Answer: D Diff: 2 Page Ref: 1292 Objective: 43-7 19) A vehicle has rolled onto its passenger side after running a red light at an intersection and striking another car. The restrained driver remains in the car, screaming in pain. To manage this scene, what should you do first? A) Determine the patient's injuries B) Remove the roof C) Stabilize the vehicle D) Hold manual cervical spine motion restriction Answer: C Diff: 2 Page Ref: 1292 Objective: 43-7 20) A car has rolled several times and is on its roof. Closer inspection reveals the unrestrained driver lying in the back with the back portion of the roof collapsed. Which instruction should you give first? A) "Let's forcibly remove the back door." B) "We need to take the weight off of the roof posts." C) "Let's open the doors and use them for support." D) "We need to climb in to assess the patient." Answer: B Diff: 2 Page Ref: 1292-1293 Objective: 43-5

5 Copyright © 2018 Pearson Education, Inc.

21) You have been given an order to cut through the "B" posts of a four-door car with a trapped passenger inside. As an EMT with knowledge of vehicle extrication, which posts do you cut? A) The posts that the front doors close against B) The posts that support the front windshield C) The posts that the rear doors close against D) The posts that support the rear windshield Answer: A Diff: 1 Page Ref: 1293 Objective: 43-8 22) What is the earliest point at which you can begin considering if there are obstacles to patient extrication? A) Based on information you receive from the caller B) During the scene size-up C) During your 360-degree assessment D) Upon receiving the call from dispatch Answer: D Diff: 1 Page Ref: 1284 Objective: 43-1 23) What is the most important initial information you receive from dispatch? A) Caller's name B) Call location C) MOI or NOI D) Patient hazards Answer: B Diff: 1 Page Ref: 1284 Objective: 43-1

6 Copyright © 2018 Pearson Education, Inc.

Prehospital Emergency Care, 11e (Mistovich et al.) Chapter 44 Hazardous Materials 1) A small truck has gone over an embankment. The driver has a head injury and is confused, but is able to tell you that he is hauling N-acetyl-para-aminophenol. Which resource would provide you with the best initial information about this chemical? A) Emergency Response Guidebook B) Medical direction C) Truck driver D) TREC-CHEM EMS Guidebook Answer: A Diff: 1 Page Ref: 1308 Objective: 44-5 2) An intoxicated driver drove his car into the side of a slow-moving train, hitting a tanker car. You recognize the need to call CHEMTREC to get information on what is contained within the tanker. Which piece of information is it critical to provide to this agency so it can respond with helpful information? A) Type of railroad car involved B) Name of the railroad carrier C) UN identification number D) Accident location and train identification number Answer: C Diff: 2 Page Ref: 1309 Objective: 44-3 3) On the scene of a hazardous materials emergency, which individual is best suited to provide command and coordination of assets at the incident? A) Fire service rescue technician B) Paramedic C) Hazardous materials technician D) Hazmat specialist Answer: D Diff: 2 Page Ref: 1310 Objective: 44-6 4) You have been trained to the OSHA level of "First Responder Awareness." With this level of training, which activities are you qualified to perform? A) Stop and patch a leak in a broken valve emitting an unknown gas B) Recognize a potential hazardous materials emergency and call for the appropriate resources C) Provide detailed instructions on how to contain a spill of an unknown liquid D) Direct the operations at a hazardous materials emergency until the chemical is identified Answer: B Diff: 3 Page Ref: 1309-1310 Objective: 44-6

1 Copyright © 2018 Pearson Education, Inc.

5) You and your partner are the first emergency personnel on the scene of an overturned tanker truck. The driver appears motionless in the cab of the vehicle. You also note liquid leaking from the rear-valve assembly of the tanker trailer. After properly positioning your ambulance, your immediate priority is to: A) Assess the driver for life-threatening conditions B) Determine which chemical is contained in the tanker C) Evacuate people from cars near the tanker D) Provide manual cervical spine motion restriction for the driver Answer: C Diff: 2 Page Ref: 1310 Objective: 44-7 6) On the National Fire Protection Association hazardous materials classification placard, the blue diamond represents: A) Reactivity B) Health hazard C) Specific hazard D) Fire hazard Answer: B Diff: 1 Page Ref: 1306; Figure 44-4A Objective: 44-3 7) If the red diamond on a National Fire Protection Association placard contains a "4," the EMT should recognize that the material within this container is: A) Liquid B) Radioactive C) A gas D) Very flammable Answer: D Diff: 1 Page Ref: 1306; Figure 44-4A Objective: 44-3 8) You have arrived on the scene of a chemical leak at an industrial park. The placard on the leaking container has a "1" in the blue diamond and a "4" in the red diamond, which means the material is: A) Flammable but represents a low health hazard B) Explosive with a high health hazard associated C) A solid that is very flammable D) A flammable liquid with radioactive properties Answer: A Diff: 2 Page Ref: 1306; Figure 44-4A Objective: 44-3

2 Copyright © 2018 Pearson Education, Inc.

9) You are working the scene of a collision involving a tractor-trailer that is hauling an unknown chemical. After it has been determined that no material has been spilled, the rescue chief asks you to get the truck's shipping papers. Where would these papers most likely be located? A) Shipping vault on the trailer B) Cab of the truck C) Rear compartment on the tanker D) Electrical box between the tractor and the trailer Answer: B Diff: 2 Page Ref: 1305 Objective: 44-5 10) What is the level of training for those who respond to the release of hazardous materials so as to protect people, property, and the environment? A) First Responder Awareness B) First Responder Operations C) Hazardous Materials Technician D) Hazardous Material Specialist Answer: B Diff: 2 Page Ref: 1310 Objective: 44-6 11) Which statement demonstrates that the EMT has a proper understanding of basic operations at the scene of a hazardous materials emergency? A) "Life or health of the EMS providers should never be risked if the threat is only to the environment." B) "It is critical to remove the patients as quickly as possible from the incident site." C) "If you cannot see or smell a chemical, the chance of one being present is very low." D) "Identification of the material is not important, since the approach is the same for all materials." Answer: A Diff: 2 Page Ref: 1310 Objective: 44-8 12) At the scene of a hazardous materials emergency, you have been assigned to the cold zone. In this zone, which type of activities will you be performing? A) Initial decontamination and triage B) Rescue and initial decontamination C) Removal of contaminated rescue gear D) Triage, vital signs, and medical histories Answer: D Diff: 2 Page Ref: 1314 Objective: 44-8

3 Copyright © 2018 Pearson Education, Inc.

13) When working in the "warm" zone, the EMT must remember that: A) Contamination is no longer a threat in this zone B) Water used in this zone does not have to be contained C) Protective gear must be worn in this zone D) Patient care is not performed in this zone Answer: C Diff: 2 Page Ref: 1312 Objective: 44-1 14) A train carrying radioactive materials has derailed. The radiation safety officer informs you that you will be transporting a 31-year-old male who was exposed to the radioactive waves emitted by the material. As such, you should recognize that: A) The patient will require immediate decontamination B) The patient will be vomiting and will have lost his hair C) You will need special clothing to protect yourself from radiation D) The patient represents no contamination threat to you Answer: D Diff: 2 Page Ref: 1315 Objective: 44-10 15) While a factory employee is moving a container of a radioactive liquid, some of the liquid splashes onto his arm. This situation is best be described as: A) Exposure B) Contamination C) Infection D) Contagion Answer: B Diff: 2 Page Ref: 1315 Objective: 44-10 16) The driver of a truck carrying a radioactive substance accidentally came in contact with the material after getting into a crash on the freeway. You are first on the scene, and a quick observation indicates that the driver still has the powder on his arm. Your immediate action would be to: A) Provide basic care, but avoid touching the patient's arm B) Place the patient in the ambulance, but avoid touching him en route to the hospital C) Instruct the patient to remain still and wait for the radiation safety officer D) Rinse the powder off the arm and wrap it with a sterile dressing and roller gauze Answer: C Diff: 2 Page Ref: 1315 Objective: 44-10

4 Copyright © 2018 Pearson Education, Inc.

17) You are at a scene where a radioactive material is emitting alpha rays. As a knowledgeable EMT, you recognize that the proper shielding from these rays can be achieved by: A) Clothing B) Aluminum C) Concrete D) Lead Answer: A Diff: 2 Page Ref: 1315 Objective: 44-10 18) You have arrived at the scene where a tractor-trailer overturned into a small ravine adjacent to the freeway. As you exit the ambulance and are donning your gloves, the driver of the truck approaches and informs you that he was hauling radioactive medical wastes and warns you that there is a car underneath the overturned trailer. Given this situation, what should you do immediately? A) Move the ambulance upwind and uphill of the scene B) Position the ambulance in front of the vehicle to block oncoming traffic C) Attempt to identify the material and type of rays being emitted D) Determine whether any radioactive material has been released Answer: A Diff: 2 Page Ref: 1315 Objective: 44-7 19) A worker has been exposed to beta rays given off by a radioactive material, but is not contaminated. He presents to you when the fire department carries him out from the industrial plant in which there was an explosion and partial building collapse. What should you do first? A) Decontaminate the patient B) Quickly assess for any radiation burns C) Initiate your primary assessment D) Determine the exact material to which he was exposed Answer: C Diff: 2 Page Ref: 1316 Objective: 44-10 20) The EMT would best describe a hazardous material as: A) Any material capable of injuring more than 50 people B) Any material that poses a threat to life C) Any material that is explosive in nature D) Any material that can contaminate the environment Answer: B Diff: 1 Page Ref: 1301 Objective: 44-1

5 Copyright © 2018 Pearson Education, Inc.

21) What does the "M" in the mnemonic TRACEM stand for? A) Machine B) Maximum C) Mechanical D) Medical Answer: C Diff: 1 Page Ref: 1301 Objective: 44-2 22) Which mnemonic can be used to help remember the types of damage that can be caused by hazardous materials? A) FMCSA B) CHEMTREC C) TRACEM D) NFPA Answer: C Diff: 1 Page Ref: 1301 Objective: 44-2 23) What is the LEAST reliable way to determine the presence of hazardous materials? A) A placard on a container B) You senses C) Safety Data Sheets D) WISER Answer: B Diff: 1 Page Ref: 1307 Objective: 44-4 24) When relying on your senses to determine whether a hazardous spill has occurred, you should: A) Assume that the area around any leak is hazardous B) Not approach close enough to smell the material C) Approach the spill from upwind D) Approach the spill only to rescue a known victim Answer: A Diff: 1 Page Ref: 1308 Objective: 44-4 25) A patient has received burns to his arm from a beta particle exposure. This type of damage would be classified as: A) Radiation sickness B) Radiation injury C) Radiation poisoning D) Radiation exposure Answer: B Diff: 1 Page Ref: 1316 Objective: 44-9 6 Copyright © 2018 Pearson Education, Inc.

26) How long can a person who has radiation sickness expect to have symptoms? A) Up to 7 or 8 weeks B) A few days C) Symptoms lasting more than 2 weeks are considered radiation poisoning D) The symptoms do not typically resolve Answer: A Diff: 1 Page Ref: 1316 Objective: 44-9 27) What is one of the most common criminal uses of hazardous materials that the EMT is likely to encounter? A) X-ray radiation B) ANFO C) A methamphetamine lab D) A pressure cooker bomb Answer: C Diff: 1 Page Ref: 1317 Objective: 44-11 28) Where is at EMT at highest risk for exposure to phosphine gas? A) A methamphetamine lab B) A military base C) A hardware store D) A high pressure cylinder Answer: A Diff: 2 Page Ref: 1317-1318 Objective: 44-11 29) Which class of chemicals would a terrorist likely use in an attack? A) Class 4 Flammables B) Class 8 Corrosives C) Class 5 Oxidizers D) Class 2 Gases Answer: D Diff: 2 Page Ref: 1318 Objective: 44-12 30) Which class of hazardous materials includes radioactive sources that may be exploited by terrorists in an attack? A) Class 3 B) Class 1 C) Class 6 D) Class 7 Answer: D Diff: 2 Page Ref: 1318 Objective: 44-12 7 Copyright © 2018 Pearson Education, Inc.

Prehospital Emergency Care, 11e (Mistovich et al.) Chapter 45 Multiple-Casualty Incidents and Incident Management 1) The EMT correctly defines a multiple-casualty incident (MCI) when he states: A) "It is any event that places an excessive demand on rescue personnel and equipment." B) "It occurs when the number of patients requiring care is greater than 10." C) "It occurs anytime the incident requires the medical director to leave the hospital and direct activities on scene." D) "It occurs anytime that five or more people are injured and require transport to different hospitals." Answer: A Diff: 1 Page Ref: 1323 Objective: 45-1 2) The National Incident Management System (NIMS) provides for: A) Specific codes and terms to enhance communications between EMS, fire, and law enforcement personnel B) Increased funding and reimbursement for any EMS agency involved in a disaster response C) A consistent approach to managing a disaster by multiple responding departments and agencies D) The federal government's control of any disaster scene regardless of location Answer: C Diff: 2 Page Ref: 1323 Objective: 45-1 3) You are teaching a group of EMTs about multiple-casualty incidents and the Incident Command System (ICS). Which statement about ICS is true? A) Each EMS agency can use its own codes and terms B) There is a separate commander for the EMS, fire, and law enforcement resources C) Each individual responder is accountable to two supervisors D) Plain English is used for all communications in the United States Answer: D Diff: 2 Page Ref: 1325 Objective: 45-13 4) At a drill involving the rollover of a tourist bus, you have been assigned to the transport area. Which activities will you be performing within this section? A) Transport patients to the hospital or other appropriate facilities B) Coordinate the transfer of patients to ambulances for transport to the hospital C) Triage and assign priorities of EMS care and transport to the hospital D) Triage and provide patient care prior to transport to the hospital Answer: B Diff: 1 Page Ref: 1334 Objective: 45-4

1 Copyright © 2018 Pearson Education, Inc.

5) While responding to an MCI drill involving an explosion, you pass seven ambulances sitting in a parking lot three blocks away from the incident. You should recognize this area as the: A) Transport unit B) Staging unit C) Treatment unit D) Triage area Answer: B Diff: 2 Page Ref: 1334 Objective: 45-4 6) The EMT identifies the primary benefit of triage when she states: A) "Triage is a process by which the number of patients is quickly estimated so the appropriate resources can be summoned." B) "Triage is a system that can be used by the EMT to determine the number of patients who will most likely die." C) "Triage is a system used by the EMT to assign the order of care when an incident involves more than five patients." D) "Triage is used to determine the order in which patients will receive medical care and transport." Answer: D Diff: 2 Page Ref: 1326 Objective: 45-5 7) Approximately 15 vehicles have been involved in a "chain-reaction" collision on a freeway one foggy morning. Your unit is the first ambulance on the scene, and you begin going from vehicle to vehicle to determine the type and severity of each injury experienced by the occupants. This is an example of which kind of triage? A) Emergency B) Primary C) Secondary D) Tertiary Answer: B Diff: 2 Page Ref: 1326-1327 Objective: 45-5 8) At a multiple-casualty incident, a crew brings you a patient with a red tag tied to his wrist. What does this tag indicate? A) Immediate care and transport are necessary B) Delayed care and transport are permissible C) The patient has a minor injury to his wrist D) The patient is deceased Answer: A Diff: 2 Page Ref: 1328 Objective: 45-6

2 Copyright © 2018 Pearson Education, Inc.

9) Which patient triage tag indicates the lowest priority for care and transport on the scene of a multiple-casualty incident? A) Yellow B) Blue C) Black D) Orange Answer: C Diff: 1 Page Ref: 1328 Objective: 45-6 10) You have been assigned to the triage unit at a multiple-casualty incident. A patient with a yellow triage tag arrives. Your immediate action should be to: A) Change the tag to green B) Obtain vital signs C) Arrange for immediate transport D) Triage the patient again Answer: D Diff: 2 Page Ref: 1328 Objective: 45-5 11) Your EMS system uses the Simple Triage and Rapid Transport (START) system for triaging patients at a multiple-casualty incident. Which item(s) should you include in your triage assessment to determine the order in which patients will receive emergency care? A) Chief complaint and vital signs B) Respiratory and perfusion status C) Level of consciousness and past medical history D) Chief complaint, vital signs, and past medical history Answer: B Diff: 2 Page Ref: 1328 Objective: 45-7 12) A bleacher at a college has collapsed, and your crew is the first EMS unit to arrive on the scene. As you enter the gym, several patients are walking around with visible injuries. Which instruction is most appropriate for these ambulatory patients? A) "Go outside and wait by the red fire truck in the parking lot and stay there." B) "Sit down here so that I can do a quick assessment on you." C) "Put this yellow tag on your wrist and go outside to the red fire truck in the parking lot." D) "Leave this gym and wait outside. I will be out in a few minutes after I check others who cannot walk." Answer: A Diff: 2 Page Ref: 1328 Objective: 45-9

3 Copyright © 2018 Pearson Education, Inc.

13) You have responded to a bar for a fight involving approximately 8 to 10 patrons. Weapons were used in the fight, and all the patrons have varying degrees and types of injuries. You are in charge of triage and must use the Simple Triage and Rapid Transport (START) system. You find a young person lying on the floor with blood covering his shirt. Quick assessment reveals him to be breathing at 24 times per minute. What should you do next? A) Start positive pressure ventilation B) Check for a radial pulse C) Determine the mental status D) Place a yellow tag on his wrist Answer: B Diff: 3 Page Ref: 1329 Objective: 45-7 14) A natural gas explosion has destroyed an apartment building. At least five occupants of the building were killed, and another 12 are injured. You are in charge of triage. You are presented with a young woman who is breathing at a rate of 40 breaths per minute after fire department personnel carried her out from the collapse site. Using the Simple Triage and Rapid Transport (START) system for triaging, you should immediately: A) Place a red tag on this patient and move on to the next patient B) Start positive pressure ventilation C) Apply oxygen with a nonrebreather face mask D) Check for a radial pulse and capillary refill Answer: A Diff: 3 Page Ref: 1328 Objective: 45-7 15) At the scene of a multiple-casualty incident, you are triaging patients using the Simple Triage and Rapid Transport (START) system. You are presented with an unresponsive patient who is not breathing. What should you do next? A) Place a red tag on the patient B) Start positive pressure ventilation C) Open the airway D) Place a black tag on the patient Answer: C Diff: 3 Page Ref: 1328 Objective: 45-7 16) As the EMT responsible for triage at a motor vehicle collision, you find a patient who cannot walk due to his injuries; the patient is breathing 18 times per minute and has a radial pulse. What should you do next? A) Check the patient's mental status B) Check the patient's blood pressure C) Move the patient to the treatment area D) Place a yellow tag on the patient Answer: A Diff: 3 Page Ref: 1329 Objective: 45-5 4 Copyright © 2018 Pearson Education, Inc.

17) You are using the Simple Triage and Rapid Transport (START) system. Which patient should have a yellow tag applied prior to being moved to the treatment area? A) A male with a respiratory rate of 8 breaths/min, no radial pulse, and unresponsive mental status B) A female with a respiratory rate of 22 breaths/min, a palpable radial pulse, and who squeezes your fingers when instructed C) A female with agonal respirations, no radial pulse, and unresponsive mental status D) A male with a respiratory rate of 40 breaths/min, a palpable radial pulse, and who squeezes your fingers when instructed Answer: B Diff: 2 Page Ref: 1329 Objective: 45-10 18) A problem with the Simple Triage and Rapid Transport (START) system of triage is that it: A) Is difficult to apply to young children B) Requires at least 10 rescuers C) Is used only for traumatic injuries D) Can be complex and difficult to use Answer: A Diff: 2 Page Ref: 1329 Objective: 45-8 19) You are teaching some Emergency Medical Responders (EMRs) to use the JumpSTART system of triage. Which comment made by an EMR requires you to intervene and provide corrective instruction? A) "I like the JumpSTART system because it uses the same categories of assessment as the START system of triage." B) "If a young child is injured in a multiple-casualty incident, I will use the JumpSTART system to triage him." C) "The JumpSTART system allows me to classify the patient according to his ability to ambulate, his breathing, his perfusion, and his mental status." D) "The JumpSTART system is very nice because it allows me to take 2 minutes to assess each patient." Answer: D Diff: 2 Page Ref: 1330 Objective: 45-8 20) A young child involved in a multiple-casualty incident is found lying on the ground with obvious deformity to the left thigh. Using the JumpSTART system for triage, you note that he is not breathing. What should you do next? A) Tag the patient as black, and then move to the next one B) Provide 30 seconds of positive pressure ventilation C) Open the airway, and then assess for a pulse D) Tag the patient as red, and then move him to the treatment area Answer: C Diff: 3 Page Ref: 1331 Objective: 45-8 5 Copyright © 2018 Pearson Education, Inc.

21) You are in charge of triage at a scene involving 32 patients exposed to carbon monoxide in a church basement. You find a 6-year-old boy who is not breathing. You open his airway and find that breathing does not return. Furthermore, you cannot locate a radial pulse. You should: A) Provide 15 seconds of positive pressure ventilation B) Place a black tag on this patient and move to the next patient C) Provide 15 seconds of CPR and recheck for a radial pulse D) Apply oxygen and check the patient's mental status Answer: B Diff: 3 Page Ref: 1331 Objective: 45-10 22) You are at the scene of a multiple-casualty incident where a church van carrying 15 parishioners overturned. You and your partner are in charge of the treatment unit and are presented with four patients. Which one will you treat first? A) A confused 69-year-old male with a respiratory rate of 42 breaths/min and a rapid and weak radial pulse B) An elderly female patient with a femur fracture and a yellow tag on her wrist C) A 21-year-old male in cardiopulmonary arrest D) A 13-year-old boy who walked to the treatment area complaining of a severe headache Answer: A Diff: 3 Page Ref: 1328 Objective: 45-11 23) In most EMS systems, a multiple-casualty incident is said to occur when the number of patients is: A) 3 or more B) 10 or more C) 20 or more D) 50 or more Answer: A Diff: 1 Page Ref: 1323 Objective: 45-2 24) When a second-story deck collapsed, several patients fell from the deck. They are now lying on the ground below. As the first emergency personnel to arrive on the scene, the initial priority for you and your partner is to determine: A) The number of patients B) The number of ambulances required C) If the scene is safe to enter D) Type and severity of injuries Answer: C Diff: 2 Page Ref: 1325 Objective: 45-4

6 Copyright © 2018 Pearson Education, Inc.

25) Which statement made by an EMT shows a proper understanding of the EMT's role at the scene of a multiple-casualty incident? A) "The EMT will be in charge of providing direct medical care." B) "The role of the EMT will always be to transport patients." C) "The EMT will always be in charge of triage and transport of patients." D) "The role of the EMT will vary depending on the exact situation." Answer: D Diff: 2 Page Ref: 1326 Objective: 45-4 26) At the scene of a multiple-casualty incident, you note that another EMT has become overwhelmed and emotionally incapacitated after unsuccessfully trying to save a child's life. You should: A) Monitor the EMT for improper care B) Remove the EMT from the scene C) Instruct the EMT to go home D) Assess the EMT for a head injury Answer: B Diff: 2 Page Ref: 1335 Objective: 45-14 27) At the scene of a multiple-casualty drill, you have been assigned to the role of transport unit leader. In this role, one of your responsibilities is to: A) Ensure proper care for each patient B) Distribute patients to each medical facility C) Immobilize all patients prior to transport D) Ensure that patients in cardiac arrest leave for the hospital first Answer: B Diff: 1 Page Ref: 1334 Objective: 45-12 28) A large bus with 52 passengers overturned on a busy highway. At the scene, 12 people are dead and 23 others were taken to various hospitals, many with life-threatening injuries. As commander of the incident, which action is most appropriate for you to take after the last patient has left the scene? A) Encourage rescuers to talk among themselves about any feelings or thoughts they may need to share B) Bring all rescuers to a central point and hold a discussion to critique the rescue effort C) Set a date and time for a mandatory stress debriefing D) Promote lighthearted talk about the incident to decrease rescuer tension and stress Answer: A Diff: 1 Page Ref: 1335 Objective: 45-14

7 Copyright © 2018 Pearson Education, Inc.

29) You have been selected to represent your EMS system on a disaster planning committee involving many different agencies. Regarding preparation for a possible disaster, which item is most beneficial to stress in the meeting? A) Specific terminology and codes will need to be established to make communication effective B) All rescuers must be flexible and able to perform any task asked of them at a disaster C) The lay community should become involved in disaster planning by learning basic first aid and CPR D) Separate plans using the NIMS need to be developed for natural and human-made disasters Answer: C Diff: 2 Page Ref: 1336 Objective: 45-3 30) What is the mnemonic for the standard process used during on-scene management of a disaster or MCI? A) START B) SALT C) ICS D) NIMS Answer: C Diff: 2 Page Ref: 1325 Objective: 45-3 31) Which triage system is intended to be adopted as the national standard for MCI triage? A) START B) SALT C) ICS D) JumpSTART Answer: B Diff: 2 Page Ref: 1331 Objective: 45-9 32) What determines the order in which patients are relocated from triage to treatment? A) Priority B) Accessibility C) Space D) NIMS Answer: A Diff: 2 Page Ref: 1333 Objective: 45-11

8 Copyright © 2018 Pearson Education, Inc.

33) Who determines the order in which patients are transported to various hospitals in the area of the MCI? A) The staging unit leader B) The transport unit leader C) Incident commander D) NIMS Answer: B Diff: 2 Page Ref: 1334 Objective: 45-12 34) Dead spots represent a failure in which system at an MCI? A) Decontamination B) Access C) Communications D) Triage Answer: C Diff: 2 Page Ref: 1334 Objective: 45-13 35) When relocating local residents away from a disaster, you should: A) Wait as long as possible to ensure that the relocation is necessary B) Relocate residents to hospitals and clinics as much as possible C) Ensure that a triage system is in place at the rendezvous point D) Keep residents in their social groupings as much as possible Answer: D Diff: 2 Page Ref: 1336 Objective: 45-15 36) Public warnings are best carried out by: A) Television B) Radio C) A PA system D) Any means you have available Answer: D Diff: 2 Page Ref: 1336 Objective: 45-15 37) Almost all disaster victims experience: A) Fear B) Vulnerability C) Guilt D) Anxiety Answer: A Diff: 2 Page Ref: 1337 Objective: 45-16

9 Copyright © 2018 Pearson Education, Inc.

38) What is one of the best ways to help reduce the public's stress associated with an evacuation? A) Assign a small number of triage members to provide psychological first aid B) Group evacuees with strangers to help prevent cross-talk and rumors C) Communicate—assign several rescue workers to gather information and disseminate it to local radio and television stations D) Move as many evacuees as possible to hospital locations to take advantage of resources already in place Answer: C Diff: 2 Page Ref: 1337 Objective: 45-16

10 Copyright © 2018 Pearson Education, Inc.

Prehospital Emergency Care, 11e (Mistovich et al.) Chapter 46 EMS Response to Terrorism Involving Weapons of Mass Destruction 1) You are teaching an EMT class about the role of the EMS response to a terror attack involving weapons of mass destruction (WMD). A student asks you what constitutes a WMD. How would define a WMD? A) Any agent used to cause indiscriminate death and destruction B) Any weapon used to create fear and panic among the general population C) Any agent that is intended to kill a specific segment of the population D) Any biological agent that kills people and animals but does not destroy property Answer: A Diff: 2 Page Ref: 1341 Objective: 46-1 2) Which item best describes use of a weapon of mass destruction? A) Death of 25 people following an explosion in a chemical plant caused by faulty wiring B) Death of 57 people following nighttime tornadoes that struck a suburban area C) Death of 49 people in a crowded mall following intentional dispersal of an unknown bacterial agent D) Accidental release of radioactive gas from a nuclear power plant that impacts thousands of people Answer: C Diff: 2 Page Ref: 1341 Objective: 46-2 3) At the scene of a multiple-casualty incident caused by a weapon of mass destruction, what is your first priority as an EMT? A) Identifying the agent or mechanism of the WMD B) Establishing the number of dead and injured and needed resources C) Establishing a communications system D) Ensuring the safety of yourself and your partner Answer: D Diff: 1 Page Ref: 1343 Objective: 46-4 4) Multiple EMS agencies are involved in a WMD incident involving hundreds of patients. Which statement regarding the different protocols and medical direction used by those different agencies is most accurate? A) EMTs should automatically be able to administer any medication that a paramedic can administer B) Most protocols involving on-line medical direction should convert to standing orders during the event C) On-line medical direction should be used for all treatment by EMS D) All on-line medical direction should be suspended Answer: B Diff: 2 Page Ref: 1343 Objective: 46-3 1 Copyright © 2018 Pearson Education, Inc.

5) How can an EMT best protect herself, as well as get the most appropriate resources, at the scene of a multiple-casualty incident involving a weapon of mass destruction? A) Estimate the number of fatalities and injuries B) Identify the specific agent or mechanism used C) Make an overall determination of the number and types of injuries involved D) Turn over control of the incident to the Homeland Security Department Answer: B Diff: 2 Page Ref: 1344 Objective: 46-5 6) A triage officer at the scene of a suicide bombing assigns you to a patient who has been injured from the secondary effects of the blast. Which type of injuries would you expect in this patient? A) Fractured arm after being thrown to the ground B) Lung injury caused by the blast's pressure wave C) Closed head injury caused by striking a tree D) Facial laceration caused by flying glass and debris Answer: D Diff: 2 Page Ref: 1346 Objective: 46-6 7) In planning for a response to an attack with a weapon of mass destruction involving an incendiary device, the EMT should be prepared to handle which types of injuries? A) Fractures B) Infection C) Burns D) Blast injuries Answer: C Diff: 2 Page Ref: 1347 Objective: 46-7 8) You are participating in a training exercise for an attack with a weapon of mass destruction involving the release of a nerve agent. In the triage area, you are presented with a patient who responds to painful stimuli and has an open airway. He is breathing 8 times per minute and has a weak radial pulse. He is wheezing and has pinpoint pupils. In his pocket, you see a metered-dose inhaler containing albuterol. When caring for this patient, you should first: A) Administer positive pressure ventilation B) Obtain vital signs, including the patient's temperature C) Administer albuterol with the metered-dose inhaler D) Look for an injury causing shock Answer: A Diff: 2 Page Ref: 1348 Objective: 46-9

2 Copyright © 2018 Pearson Education, Inc.

9) A patient has an injury to his arm caused by an unknown vesicant. After assessing the patient's airway, breathing, and circulation, you should: A) Brush the powder from the skin B) Apply a wet sterile dressing to the arm C) Flush the injured area with water D) Transport the patient to the hospital immediately Answer: C Diff: 2 Page Ref: 1349 Objective: 46-9 10) A patient has inhaled cyanide gas. He is alert and complaining of shortness of breath. His airway is patent, his breathing is adequate, and his radial pulse is strong. His skin is warm and dry, and his breath sounds are clear. Given these assessment findings, what should the EMT do first? A) Administer the antidote for cyanide B) Monitor the patient and transport C) Obtain vital signs with the patient in a supine position D) Administer oxygen via a nonrebreather mask Answer: D Diff: 2 Page Ref: 1350 Objective: 46-9 11) An alert and oriented patient has been exposed to the biological toxin ricin. He presents with a fever, cough, mild dyspnea, and generalized weakness. His airway is patent and his breathing is adequate. His skin is warm and dry, and his radial pulse is weak. Which intervention is most appropriate when caring for this patient? A) Administer oxygen to the patient B) Apply a HEPA filter mask to the patient C) Obtain vital signs and transport D) Isolate the patient before administering further care Answer: A Diff: 3 Page Ref: 1352 Objective: 46-10 12) For the EMT, what is the best means of protection against acquiring a disease such as smallpox prior to any sort of exposure? A) Antibiotics B) Immunizations C) Face shields D) Iodine tablets Answer: B Diff: 2 Page Ref: 1353 Objective: 46-15

3 Copyright © 2018 Pearson Education, Inc.

13) At a community meeting about preparations for a biological terror attack, an attendee asks you how anthrax acquired naturally differs from anthrax acquired via terrorism. What is your best response? A) "Anthrax caused by terrorism is easier to treat than a disease acquired naturally." B) "Anthrax caused by terrorism causes signs and symptoms different from anthrax acquired naturally." C) "Anthrax can be acquired only by terrorism; it is not acquired naturally." D) "Anthrax is the same regardless of whether it is intentionally inflicted or acquired naturally." Answer: D Diff: 3 Page Ref: 1351 Objective: 46-10 14) Why would iodine tablets be a benefit to administer to the general public after a nuclear bomb detonation? A) Iodine boosts the immune system B) Iodine protects against all types of cancer C) Iodine decreases the risk for thyroid cancer D) Iodine helps eliminate radiation from the body Answer: C Diff: 2 Page Ref: 1356 Objective: 46-14 15) A chemical agent used by terrorists is described as having a high volatility. You should recognize this descriptor as meaning that the chemical is: A) Highly radioactive B) Easily evaporated C) Very explosive D) In liquid form Answer: B Diff: 1 Page Ref: 1348 Objective: 46-1 16) Which chemical agent will generally kill a person most quickly? A) Liquid vesicant that is splashed on the arm B) Solid agent that contacts the patient's torso C) Aerosolized vesicant agent that is inhaled into the lungs D) Liquid agent that is splashed into the eyes Answer: C Diff: 2 Page Ref: 1349 Objective: 46-8

4 Copyright © 2018 Pearson Education, Inc.

17) Nerve agents typically cause death by triggering which type of organ or system failure? A) Respiratory B) Renal C) Cardiac D) Liver Answer: A Diff: 2 Page Ref: 1348 Objective: 46-9 18) If a patient is exposed to a nerve agent, which assessment finding would the EMT expect? A) Dilated pupils B) Elevated blood pressure C) Pulse pressure greater than 30mmHg D) Increased salivation Answer: D Diff: 3 Page Ref: 1348 Objective: 46-9 19) Your service has acquired the antidote for a nerve agent attack. By which route will this antidote most likely be given? A) Inhalation B) Oral C) Injection D) Topical Answer: C Diff: 1 Page Ref: 1349 Objective: 46-9 20) A localized terrorist attack involves pepper spray being released at a high school dance. While you are en route to the scene, the incident commander reports approximately 50-plus people have been exposed and calls for activation of the county disaster plan. Given the nature of the weapon, what should you anticipate doing on scene? A) Providing positive pressure ventilation B) Decontaminating patients with diluted bleach C) Administering an aerosolized antidote D) Irrigating eyes with saline Answer: D Diff: 2 Page Ref: 1351 Objective: 46-9

5 Copyright © 2018 Pearson Education, Inc.

21) Which statement concerning the biological agents used in terrorist attacks is true? A) The biological agents used by terrorists are more potent than those that occur naturally B) It may take several days to weeks to realize or determine that an attack involving biological agents occurred C) Rain is the ideal condition to disperse a biological agent because water will spread it over a greater area D) The eyes are the most effective portal of entry into the human body for a biological agent Answer: B Diff: 2 Page Ref: 1351 Objective: 46-10 22) A patient who has been exposed to an encephalitis-like agent most likely will complain of: A) Fever and confusion B) Shortness of breath C) Blindness and nausea D) Pain and paralysis Answer: A Diff: 2 Page Ref: 1352 Objective: 46-10 23) Regarding a terrorist attack involving the use of biological toxins, which statement is true? A) The toxins are not nearly as dangerous as the organisms that produce them B) The toxins are living organisms and can easily multiply in the human body C) Biological toxins will typically kill a person within minutes of exposure D) Biological toxins cannot be passed from one person to another Answer: D Diff: 2 Page Ref: 1352 Objective: 46-10 24) Which type of radiation will travel the farthest and penetrate the human body with the greatest ease? A) Alpha B) Beta C) Blast D) Gamma Answer: D Diff: 1 Page Ref: 1354 Objective: 46-11 25) Beta radiation represents the greatest threat to a person's well-being when the: A) Person is within 30 feet of the source B) Person ingests it by eating contaminated food C) Radiation occurs in liquid form D) Source is heated to greater than 100°F Answer: B Diff: 2 Page Ref: 1354 Objective: 46-11 6 Copyright © 2018 Pearson Education, Inc.

26) In a nuclear explosion, the EMT should expect the majority of patients to complain of: A) Nausea B) Headache C) Burns D) Dyspnea Answer: C Diff: 1 Page Ref: 1354 Objective: 46-13 27) The goal of an attack using a "dirty bomb" most likely will be: A) Blast power to destroy buildings and infrastructure B) Thermal forces to induce massive burns C) Dispersal of gamma radiation D) Contamination of an area with radioactivity Answer: D Diff: 2 Page Ref: 1355 Objective: 46-13 28) Which classifications of WMD are combined in the same category when using the mnemonic B-NICE? A) Radiological and nuclear B) Incendiary and explosive C) Biological and incendiary D) Chemical and explosive Answer: A Diff: 1 Page Ref: 1341 Objective: 46-2 29) When planning for a terrorist attack, which concept makes the best use of resources that are locally available? A) Mutual assistance B) Cross-training C) Warehousing D) Rapid response Answer: A Diff: 2 Page Ref: 1343 Objective: 46-3 30) If you are first on the scene of an MCI, it is imperative that you: A) Call for mutual aid after triaging all patients B) Recognize whether it is a terrorist attack early on C) Treat victims as quickly as possible D) Establish the background radiation level Answer: B Diff: 2 Page Ref: 1344 Objective: 46-4 7 Copyright © 2018 Pearson Education, Inc.

31) When you are first on the scene of a terrorist incident involving WMD, which role is of the utmost importance? A) Prehospital care of the injured B) Triage of the injured victims C) Scene size-up D) Identification of the responsible party Answer: C Diff: 1 Page Ref: 1345 Objective: 46-5 32) An example of a quinary effect of an explosion is: A) Burns B) Toxic contaminants C) Fractures D) Electrocution Answer: B Diff: 1 Page Ref: 1346 Objective: 46-6 33) The type of injury most commonly associated with an incendiary device is: A) Fractures B) Burns C) Injury from building collapse D) Radiation exposure Answer: B Diff: 1 Page Ref: 1347 Objective: 46-7 34) Which characteristic do hypergolic substances display when they are combined in a WMD? A) They release a biologic toxin. B) They promote adhesion of flammable substances to the skin. C) They spontaneously ignite when combined. D) They produce chlorine gas when combined. Answer: C Diff: 3 Page Ref: 1347 Objective: 46-8 35) An example of an injury that results from the primary exposure in a nuclear explosion is: A) Blindness B) Flash burn C) Multisystem trauma D) Radiation injury Answer: C Diff: 2 Page Ref: 1354 Objective: 46-12

8 Copyright © 2018 Pearson Education, Inc.

36) The secondary radiation exposure from a nuclear blast is the result of: A) Heat B) Fallout C) Concussion D) Decay Answer: B Diff: 2 Page Ref: 1354 Objective: 46-12 37) Sheltering of patients from radiation can be accomplished with: A) Distance B) 4 inches of earth C) 10 inches of wood D) Iodine Answer: A Diff: 2 Page Ref: 1355 Objective: 46-14 38) Protection of the EMT from chemical agent exposure requires: A) An N-95 mask B) Proper training and equipment C) Short periods of exposure D) Establishing the background contamination level Answer: B Diff: 1 Page Ref: 1347 Objective: 46-15 39) Which statement is true regarding tactical EMS? A) The TEMS curriculum is typically included in state EMT courses B) TEMS programs have been available since 1988 C) Competency in core skills is typically required before responders can participate in a TEMS course D) TEMS is a federally structured educational program provided by the FBI Answer: C Diff: 2 Page Ref: 1356 Objective: 46-16 40) How does the EMT's role in an "active shooter" incident vary from any other unsecured "shots fired" scene? A) The EMT's role is the same in both types of scenes B) EMTs should not respond unless they are TEMS certified C) EMTs should respond only if they are armed D) The EMT's primary concern is patient rescue Answer: A Diff: 2 Page Ref: 1356 Objective: 46-16 9 Copyright © 2018 Pearson Education, Inc.

41) What is the primary concern with cyberterrorism? A) Identity theft B) Shutting down the infrastructure C) Electronic bank robbery D) Stock market manipulation Answer: B Diff: 1 Page Ref: 1357 Objective: 46-17 42) How can cyberterrorism impact medical equipment? A) Equipment can be exploited electronically to cause patient harm B) Hardware orders can be rerouted C) Patients' Medicare accounts are at risk, resulting in equipment repossession D) Cyberterrorism does not have access to medical equipment Answer: A Diff: 1 Page Ref: 1357 Objective: 46-17

10 Copyright © 2018 Pearson Education, Inc.